Physics Guide app

Table of contents :
Notes
Classical Mechanics
1 Newton basic
2 NM advanced
3 Rigid body basic
4 Central Force
20201124_104118920
20201124_104121510
20201124_104124102
20201124_104126706
20201124_104129314
20201124_104131899
20201124_104134494
20201124_104137111
20201124_104139654
20201124_104142278
20201124_104144875
20201124_104147460
20201124_104150055
20201124_104152653
20201124_104155217
20201124_104157823
20201124_104200415
20201124_104203000
20201124_104205586
20201124_104208190
20201124_104210796
20201124_104213356
20201124_104215950
20201124_104218535
20201124_104221150
20201124_104223740
20201124_104226342
20201124_104228928
20201124_104231495
20201124_104234102
20201124_104236692
20201124_104239277
20201124_104241863
20201124_104244446
20201124_104247031
20201124_104249636
20201124_104252207
20201124_104254806
20201124_104257364
20201124_104259958
20201124_104302555
20201124_104305156
20201124_104307727
20201124_104310340
20201124_104312927
20201124_104315507
20201124_104318103
20201124_104320702
20201124_104323307
20201124_104325921
20201124_104328472
20201124_104331078
20201124_104333661
20201124_104336287
20201124_104338874
20201124_104341436
20201124_104344075
20201124_104346640
20201124_104349217
20201124_104351817
20201124_104354390
20201124_104356988
20201124_104359608
5 Rigid body advanced
6 Lagrangian mechanics
7 Hamilton Mechanics
8 small oscillations
Electrodynamics
Introduction to EMT
Electrostatic
Dielectrics
Magnetostatic
Electromagnetic Induction
Maxvels equations
Electromagnetic waves
Relativistic ED and Radiation
Mathamatical methode
calculas
Linear algebra
vectors
Infinite Series
DE and special functions
integral transformation
complex analysis
advanced topics
Quantum mechanics
Old Quantum theory
Mathamatical background
Basic Postulates
One Diamentional Potential
Angular Momentum
Identical Particles
Perturbation Variation
WKB And Scattering
Thermodynamics and statistics
Basic thermodynamics
Thermodynamic Potential and Applications
Basic Consepts of Statical Mechanic
Random Walk and Probability
Ensembles in Statical Mechanics
Quantum Statistical Mechanics
Ideal Fermi system
Ideal Bose System
Questions
CM
Basic Newtonian pt 1
Basic Newtonian pt 2
Angular momentum of a rigid body pt1
Angular momentum of a rigid body pt2
advanced Newtonian
central force
Lagrangian
Hamilton and phase space
canonical transformation
small oscilations
ED
Electrostatic 1
Electronics 2
Capacitor Dielectric
Magnetostatic
Magnetostatic 2
Electromagnetic Induction
Maxwell's Equations
ED Advanced
Electromagnetic wave
mathamatical method
vector
Linear Algebra
DE and Special functions
integral transformation
The Complex Analysis
Probability
Numerical methode Group theory Tensor calculas Greens functions
QM
Old Quantum Theory
basic formulation pt1
basic formulation pt2
1D Potential pt1
1D Potential pt2
3D System
Angular Momentum
Approximation methode 1
Approximation methode 2
Thermodynamics and statistics
Kinetic theory
Second law
Maxvells relation
Equation of state
Entropy Calculation
Phase transition
Microcanonical enseble
Canonical Ensemble pt1
Canonical Ensemble pt2
Quantum statistical Mechanics
Two state and Other systems
Ideal Fermi system
Ideal Bose System
Exam
CM
EMT

Citation preview

11 oo : oo : oo

N e-wtonian Mechanics: Basics Sk J ahiruddin*

*Assistant Professor Sister Nibedita Govt. College, Kolkata Author was the topper of IIT Bombay M.Sc Physics 2009-2011 batch He ranked 007 in IIT J AM 2009 and 008 (JRF) in CSIR NET June 2011 He has been teaching CSIR NET aspirants since 2012

1 @Sk J ahiruddin, 2020

Newtonian Mechanics: Basics

11 oo : oo : 03 1 @Sk J ahiruddin , 2020

Newtonian Mechanics: Basics

Contents 1 Basic Concepts

5

1.1

Mot ion in plane polar coordinat e



10

1.2

velocity and acceleration in polar coordinat e .

12

1.3

Basic kinematic equations . . . . . . . . . . .

20













2 Force and Newton's Laws

3

26

2.1

Constant force, free body diagram, friction . .

27

2.2

Solving differential equation, Dissipative force

51

2.3

Simple Harmonic Motion . . . . . . . . . . . .

65

Conservation of Momentum and Energy

78

3.1

Center of mass































79

3.2

Center of m ass frame





























92

3.3

conservation of moment um .





















97

[email protected]





2

Physicsguide , ir

,



-it.

,r

1

~



11 oo : oo : os

[email protected]

2

P hysicsguide

@Sk Jahiruddin, 2020

3.4

3.5

4

Newtonian Mechanics: Basics

Conservation of Energy in one and two dimension1 16 3.4. 1

Power . . . . . . . . . . . . . .









121

3.4.2

Gravitational potential energy:









121

3.4.3

Potent ial energy of a spring













122















126

3.5. 1

Collision in one dimension .













127

3.5.2

1D collision in COM fr ame .













131

3.5.3

Collision in two dimensions













135

Collisions

. . . . . . . . . . . . .

Variable mass problem 4.1

Rocket P roblem . . . 4.1.1 4.1.2

4. 2























Derivation of equation of motion







150







150

Rocket motion when no external force is present

4. 1.3

149

. . . . . . . . . .

Rocket in gravitational field

Moment um flux . . . . . . . . . . .













152













154













155

11 oo : oo : oa 4.1.3 4.2

Rocket in gravitational field

Momentum flux . . . .

[email protected]









4.4















154













155

Physicsguide

3

@Sk Jahiruddin, 2020

4.3





Newtonian Mechanics: Basics

Chain problems . . . . . . . . . . . . . . . . .

157

4.3.1

Equation of motion of a falling chain .

157

4.3.2

Chain falling on a weight machine . . .

160

Other problems on variable mass . .











162











162

4.4.1

Cart with leaking water tank

4.4.2

Sand on cart . . .

























165

4.4.3

Growing raindrop

























169

11 oo : oo : 1o

[email protected]

4

@Sk Jahiruddin, 2020

1

Physicsguide

Newtonian Mechanics: Basics

Basic Concepts

Before we start our discussion on Basic Newtonian mechanics Let's revise our concepts of dimensional analysis first . You know t hat M stand for mass, L for length, and T for t ime. For example, we'll write a speed as [v] = L / T and t he gravitational constant as [G] = £ 3 / (MT 2 ) You can figure t his out by not ing that Gm 1m 2 / r 2 has the dimensions of force, which in t urn has dimensions M L / T 2 , from F =ma. . Example 1.1: A mass m hangs from a massless string of length f and swings back and forth in t he plane of the

paper. The acceleration due to gravity is g. What can we say about t he frequency of oscillations? The only quantit ies having dimension given in the problem 2 are [m] = M [f] = L , and [g] = L / T . But there is one more quantity, t he maximum angle 00 which is dimensionless (and easy to for get) . Our goal is to find t he frequency, which has units of 1/T. The only combination of our given dimensionful quantities that has units of 1/ T is g/f. But we can't rule

11 oo : oo : 13 are [m] = M [£] = L , and [g] = L / T 2 . But there is one more quantity, t he maximum angle 00 which is dimensionless (and easy t o forget ). Our goal is to find t he frequency, which has units of 1/T. The only combination of our given dimensionful quantities t hat has units of 1/ T is g/£. But we can't rule out any 00 dependence, so the most general possible form of

[email protected]

P hysicsguide

5

@Sk Jahir uddin, 2020

Newtonian Mechanics: Basics

the frequency is w

g

= f (0o)

£

where f is a dimensionless function of the dimensionless variable 0o More general method would be to t ake the dimensionful quantities raised t o arbit rary powers (ma£bgc in this problem) , and t hen writ e out the units of this product in t erms of a, b, and c. If we want to obtain units of 1/T here, then we need

M aLb

L

c

T2

l T

Matching up the powers of the three kinds of units on each side of this equation gives

M : a = 0,

L : b + c = 0,

T : - 2c = - 1

The solution to this syst em of equations is a = 0, b = - 1/ 2, and c = 1/ 2, so we have reproduced the -Jg/i result. Example 1.2: How does the speed of waves in a fluid

11 oo : oo : 16 The solution to this system of equations is a = 0, b = - 1/ 2, and c = 1/ 2, so we have reproduced the ~ result. Example 1.2: How does the speed of waves in a fluid depend on its density, p, and " bulk modulus," B (which

has units of pressure, which is force per area)? Answer with dimensional analysis

[v] = L / T [email protected]

Physicsguide

6

@Sk J ahiruddin, 2020

Newtonian Mechanics: Basics

[p]=M/L

3

and

Tal vdv = adx > v = v5 + 2ax T hese equations are also written as in your + 2 textbooks

(i) v=u +at 2

(ii) s = ut + 1/2at 2 2 (iii) v = u + 2as (iv)

s=

1 (u + v)t 2

00: 00: 54 2

(ii) s = ut + 1/ 2at 2 2 (iii) v = u + 2as 1 (u + v)t 2

s=

(iv)

Example 1.5: From an elevated p oint A , a st one is project ed vertically upward . When the stone reaches a distance h below A, its velocity is double of what it was at a height h above A. Show that the greatest height obtained by the stone above A is 5h/ 3 [email protected]

20

P hysicsguide

@Sk Jahiruddin, 2020

Newtonian Mechanics: Basics

Solution: When t he stone reaches a height h above A

vf = u

2

-

2gh

and when it reaches a dist ance h below A

v~ = u

2

+ 2gh

since the velocity of the stone while crossing A on its return journey is again u vertically down. Also, v2

= 2v1 (by problem) Combining all u

2

=

10 - gh 3

Maximum height u2

H =2g

10 gh 3 2g

Example 1.6: The relation 3t

5h 3

= v'3x + 6 describes the

11 oo :oo : s1 u2

10 gh

5h

2g

3 2g

3

H =-

Example 1.6: The relation 3t = v'3x + 6 describes t he displacem ent of a particle in one direction, where x is in m etres and t in seconds. Find the displacement when t he velocity is zero. Solution: ~=3t-6 Squaring and simplifying x [email protected]

= 3t2 -

12t

+ 12

21

P hysicsguide

Newtonian Mechanics: B asics

@Sk Jahiruddin, 2020

= ~ = 6t - 12 dt v = 0 gives t = 2s

V

Thus we get displacem ent x

=0

projectile motion: Consider a ball thrown t hrough the air, not necessarily vertically. Let x and y be t he horizontal and vertical positions, respectively. The force in the x direction is Fx

=

0, and the

force in t hey direction is Fy = -mg. So F = m a gives

x= O '

an d

••

y

=

-g

(1.11 )

If t he initial position and velocity are (X , Y) and (Vx, Vy) , t hen we can easily integrat e the above equation to obtain (1.12)

11 oo : 01 : oo If t he initial position and velocity are (X , Y) and (Vx, Vy) , t hen we can easily integrat e the above equation to obtain (1.12)

Integrating again we get

and

y(t) = Y

+ Vyt -

1

gt

2

2

(1.13)

These equations for the speeds and positions are all you need to solve a projectile problem.

[email protected]

@Sk Jahiruddin, 2020

22

P hysicsguide

Newtonian Mechanics: Basics

Example 1.7: (a) For a given init ial speed, at what inclination angle should a ball be thrown so that it t ravels

the maximum horizontal distance by the time it returns to the ground? Assume that the ground is horizontal, and t hat the ball is released from ground level. (b) What is t he optimal angle if the ground is sloped upward at an angle /3 (or downward , if /3 is negative)?

Solution: (a) Let the inclination angle be 0, and let the initial speed be V. Then t he horizontal speed is always Vx = V cos 0, and t he init ial vertical speed is Vy = V sin 0 The first thing we need to do is find t he time t in the air. We know that the vertical speed is zero at time t / 2, because the ball

11 oo : 01

: 02

initial speed be V. Then t he horizontal sp eed is always Vx = V cos 0, and t he init ial vertical speed is Vy = V sin 0 The first thing we need to do is find t he time t in the air. We know that the vertical speed is zero at time t / 2, b ecause the ball is moving horizontally at t he highest point. So t he second of equation (1.12) gives Vy= g(t/2). Therefore, t = 2Vy/g. 17 The first of equation (1.13) tells us t hat the horizontal distance t raveled is d = Vxt. Using t = 2Vy / g in this gives The sin 20 factor has a maximum at 0

=

1r /

mum horizontal dist ance traveled is t hen dmax

4. The maxi-

=V

2

/

g

(b) As in part (a), t he first t hing we need to do is find t he t ime t in the air. If the ground is sloped at an angle f3, t hen the equation for the line of the ground is y = (tan /3)x . The path of the ball is given in terms oft by

23

[email protected]

Physicsguide

@Sk Jahiruddin, 2020

Newtonian Mechanics: Basics

x = (V cos 0)t,

and

y

.

1

= (V Sln 0)t - 2gt

2

(1.14)

where 0 is t he angle of the throw, as measured with respect to t he horizontal (not t he ground). We must solve for t he t t hat makes y = (tan f3)x , because t his gives the t ime when t he path of t he ball intersects t he line of the ground. Using equation (1.14) we find t hat y

t=

= (tan /3)x when

2V. -(sin 0 - tan /3 cos 0) g

(1.15)

11 oo : 01 : os equation (1.14) we find that y

t=

= (tan j1)x when

2V. - (sin 0 - tan /1 cos 0)

(1.15)

g

(There is, of course, also the solution t = 0.) Plugging this into the expression for x in equation (1.14) gives

2v x = g

2

2

(sin 0 cos 0 - tan (3 cos 0)

(1.16)

We must now maximize this value for x, which is equivalent to m aximizing t he distance along t he slope. Setting the derivative wit h respect to 0 equal to zero, and using the double-angle formulas, sin 20 = 2 sin 0 cos 0 and cos 20 = cos2 0 - sin2 0, we find t an (3 = - cot 20. This can be rewritt en as t an j1 we have

= -

tan (1r / 2 - 20). Therefore, j1

1 0= 2 jahir@p hysicsguide .in

@Sk Jahir uddin, 2020

7f

/1+ -

2

24

= -(1r / 2

- 20) , so

( 1.17) P hysicsguide

Newtonian Mechanics: Basics

In ot her words, t he throwing angle should bisect t he angle b etween the ground and the vertical.

11 oo : 01

jahir@physicsguide .in

@Sk J ahir uddin, 2020

2

: 01

25

P hysicsguide

Newtonian Mechanics: Basics

Force and Newton's Laws

Det ailed discussion on Newton's Laws can be found in any good elementary mechanics book. All you need to know is the equation (2.1 ) F = ma and its applications.

11 oo : 01

: 1o

good elementary mechanics book. All you need to know is t he equation F = ma (2.1 ) and its applications. T here are many problems associated wit h Newton 's laws. They can be divided into two broad cat egories. First , are t he problems where forces are constant , where you need to draw t he free body diagram. Second, are t he problems where force varies wit h t ime and you need t o solve a differential equation.

jahir@p hysicsguide.in

@Sk Jahir uddin, 2020

2.1

26

P hysicsguide

Newtonian Mechanics: Basics

Constant force, free body diagram, friction

Example 2.1: Three freight cars each of mass M are pulled with force F by an engine as shown in figure.

Friction is negligible. Find t he forces on each car .

11 oo : 01

: 13

tion Example 2.1: Three freight cars each of mass M are pulled with force F by an engine as shown in figure.

Frict ion is negligible. Find t he forces on each car. M

M

M

• a

J_J I LJCJ I LJ!..J I LJ : 3

2

1

Figure 2.1:

Solution: Before drawing the for ce diagram, it is worth thinking about t he system as a whole. The cars are joined and are thus const rained to have the same acceleration. Because t he total mass is 3M, t heir acceleration is F

a = 3M Now we will draw t he free body diagram. The force diagram for t he end car, no 3, is shown, where W is the weight, N is t he upward force exerted by the t rack, and F3 is t he force exerted on car no 3 by car no 2.

[email protected]

27

P hysicsguide

@Sk J ahiruddin, 2020

Newtonian Mechanics: Basics

N

-

M

a M

,- --- -----, I I

I ________ .JI 1

w

a

M

----------

-

a

--------- -

'

I

3

-

I

F3

I

2

I

I_ - - - - - - - - .J

F2

II_ _ 1 I _ _ _ _ _ _ _ _J

F

11 oo : 01

: 1s

N

-

M

a M

----------

1 I I

I

I

' - - - - - ____ .J

a

M

----------

I

3

-

F3

Fj

I

2

-

a

---------F2

I

'--------- -'

. I '--------- -'

I

1

F

w

Figure 2.2: The vertical acceleration is zero, so that N horizontal equation of motion is

=M

W . The

F 3M

F 3

Now let us consider t he middle car, no 2. The ver·t ical forces are as before, and we omit t hem . F~ is the force exerted by the last car , and F 2 is the force exerted by car no 1. The equation of motion is

By Newton's third law, F~ = F 3 , so that F~

[email protected]

28



F / 3. since

P hysicsguide

@Sk Jahiruddin, 2020

Newtonian Mechanics: Basics

a= F / 3M we have /

r,

"\

00 : 01 : 18

Newtonian Mechanics: Basics

@Sk J ahiruddin, 2020

a= F / 3M we have F2=M

F 3M

F

+3

2F 3

The horizontal forces on the first car, no 1, are F, to the right, and , 2F F 2 = F2 = 3 to the left. Each car experiences a net force F / 3 to the right.

[email protected]

@Sk Jahiruddin, 2020

29

Physicsguide

Newtonian Mechanics: Basics

11 oo : 01 [email protected]

: 20

P hysicsguide

29

@Sk Jahiruddin, 2020

Newtonian Mechanics: Basics

Example 2.2: In this two examples we will learn how to find the relations between the accelerations in con-

strained motion. Example-I: Wedge and block system : A block slides on a wedge (a planar surface) which in t urns slides on a horizontal table, as shown in t he figure 2.3 (a). The angle of t he wedge is 0 and its height is h. How are the accelerations of the block and the wedge related? Neglect friction. Example-2: Pulley system: The pulley system shown in figure 2.3 (b), is used to hoist a block. How does t he accele1~ation of the end of the rope relate to the acceleration of the block? I

X

X

h

h

. l

X

y

)(

0

x -X - •

(a)

(c)

(b)

Figure 2.3:

Solution: Example-I: Because the wedge is in contact with the t able, we have the trivial constraint that the vertical acceleration of the wedge is zero. To find the less obvious [email protected]

30

P hysicsguide

11 oo : 01

: 23

with t he table, we have the trivial constraint that the vertical acceleration of the wedge is zero. To find the less obvious [email protected]

P hysicsguide

30

@Sk J ahiruddin, 2020

Newtonian Mechanics: Basics

constraint t hat t he block slides on t he wedge, let X be t he horizontal coordinate of the end of t he wedge and let x and y be t he horizontal and vertical coordinates of the block, as shown in figure 2.3 (c). From the geometry, we see t hat

h -y

= tan0

x- X h - y = (x - X ) tan 0 Differentiating twice with respect to time and rearranging, we obtain t he constraint equation for the accelerations: ••

jj

= (X - x) tan 0

Example-2: Using the coordinates indicated, the length of t he rope is given by l= X

+ 1rR + (X

- h) + 1rR

+ (x - h)

where R is the radius of each pulley. Hence .. X

=

1 .. - -x 2

The block accelerates half as fast as the hand, and in the opposite direction.

11 oo : 01 : 2s opposite direction.

31

[email protected]

Physicsguide

@Sk Jahiruddin, 2020

Newtonian Mechanics: Basics

Example 2.3: Two masses, M 1 and M 2 , are connected by a string that passes over a pulley. The pulley is accel-

erating upward at rate A , as shown in figure below, and t he gravitational force on each mass is W i = Mig. The problem is to find the rate at which t he masses accelerate and the tension Tin t he string. A

T

T

1

2

,.........___, Xp

1 --

I

I

Figure 2.4:

Solution: From t he force diagrams, we have

TT -

= W2 = W1

M1ii1

(2.2)

M2ii2

We have two equations but three unknowns: y 1 , y2 , and T.

11 oo : 01 : 2a •

' T- W1 = M1ii1 T - W2 = M2ii2

(2.2)

\"!ve have two equations but three unknowns: y 1 , y2 , and T. We need a t hird equation, which is the constraint equation that relates the accelerations. The coordinates are shown in 32

[email protected]

P hysicsguide

@Sk Jahiruddin, 2020

Newtonian Mechanics: B asics

the drawing. If YP is measured to the center of the pulley of radius R , then if l is the length of t he string, we have

Different iating twice wit h respect to time, we find

Using A = jjp, we have the constraint condition

(2.3) Equations (2.2) and (2.3) are easily solved:

T = 2(A+g) M1M2

M1 +M2 .. (2A + g)M2 - M1g Yi= M1 + M2 .. (2A + g)M1 - M 2g 2 y = M1 + M2

Note t hat the result is reasonable. If the masses are identical, t hey accelerate equally and t he tension is M (A + g). If eit her mass vanishes, t he other mass falls freely under gravity. If .

.

..

.

.

.

11 oo : 01

: 31

Note t hat t he result is reasonable. If t he masses are identical, they accelerate equally and t he tension is M (A + g) . If eit her mass vanishes, t he ot her mass falls freely under gravit y. If A = 0, t he apparatus is known as "Atwood 's machine," a standby in lecture demonst rations and elementary labs. The purpose of Atwood's machine is to '' decrease gravity" - t he larger mass descends more slowly t han if it were in free fall. [email protected]

P hysicsguide

33

@Sk Jahir uddin, 2020

Newtonian Mechanics: Basics

Example 2.4 : Mass m is whirled at inst antaneous speed v on t he end of a st ring of length R . T he mo-

t ion is in a vert ical plane in the gravit at ional field of t he Earth. The forces on m are the weight W = mg down and the string force T t oward t he center. T he string makes instantaneous angle 0 wit h the horizontal. Find T and t he tangent ial acceleration at any instant A

A

r

0 /

/

-- --- -.....

V

'' '

/

m

/

I

\

I I

I

0

g

I

I

T

I I

\ \

0 \\ \

I

\

' ' ..... _____ .......

w

/ / /

g

/

0

Figure 2.5:

Solution: T he diagram shows the forces and unit vectors ,

A

A

rn,

,.

, r



rr,

T T7



/'l

I

1

,. ,

I



11 oo : 01

: 33

Figure 2.5:

Solution: The diagram shows the forces and unit vectors r and 0 The radial force is - T - W sin 0, so the radial equation A

of motion is

-(T

+ W sin 0) = mar 2 = m r - r0

[email protected]

(2.4)

Physicsguide

34

@Sk Jahiruddin, 2020

Newtonian Mechanics: Basics

The tangent ial force is - W cos 0. Hence

-Wcos0 =

ma0

.

..

(2.5)

= m(r0 + 2r0) 2

0

since r = R = constant, ar = - R equation (2.4) gives

mv

2

2

v T = - - - W sin 0 = m-

R

l

R

-

2

-v / R , and

gR Sln . 0 V2

A string can pull but not push , so that T cannot be negative. This requires that mv 2 / R

> W sin 0. The m aximum value of

W sin 0 occurs when the mass is vertically up; in this case 2 mv / R > W. If t his condition is not satisfied , t he mass does not follow a circular path but starts to fall; f is no longer zero. The tangent ial acceleration is given by equation (2 .5) . since

r=

0 we have

••

a0

= R0

TT T

/"\

11 oo : 01 v ........

: 36

lS con 1 lOn IS no Sa IS e , ., . . . . e mass

oes

not follow a circular path but starts to fall; r is no longer zero. The tangential acceleration is given by equation (2 .5) . since r = 0 we have a 0 = R0 Wcos 0 ••

m

= - gcos0 The whirling block has tangent ial acceleration t hat varies between +g and -g, no matter what t he value of v . On t he downswing t he tangential speed increases, on t he upswing it decreases. If we wanted to swing t he mass with constant velocity, we would need to make a0 = 0 by giving T a tangent ial [email protected]

35

@Sk Jahiruddin, 2020

P hysicsguide

Newtonian Mechanics: Basics

component W cos 0. Now we will consider rotational motion, t ranslational motion, and const raints. Example 2.5: A horizontal frictionless table has a small hole in its center . Block A on the table is con-

nected to block B hanging beneath by a string of negligible mass which passes through the hole. Init ially, B is held stationary and A rotates at constant radius r 0 with st eady angular velocity w 0 . If Bis released at t = 0, what is its acceleration immediately afterward?

I

I

\

/

.,.,---z--

' ...... _

----

'A\

/-+--/---._.,,,,, ,,,

z

00 : 01 : 38

T

Me

z

Figure 2.6:

Solution: The force diagrams for A and B after t he moment of release are shown in the sketches.

[email protected]

P hysicsguide

36

@Sk J ahiruddin, 2020

Newtonian Mechanics: Basics

For t he block on the table we need consider only horizontal forces. The only such force acting on A is the string force T . The forces on B are t he string force T and its weight W B For A it is natural to use polar coordinat es r, 0, while for B t he linear coordinate z is sufficient, as shown in t he force diagrams. As usual, t he unit vector r is radially outward. For convenience, we have taken z t o b e posit ive in the downward direction. The equations of motion are

-T =

MA ••

0

..r-r 0·2 •

= MA(r0 + 2r0) rn

radial , A t angential , A

(2.6)

(2.7) Inn\

11 - T = MA

00:01 :41

..r-r 0·2

••

0

radial , A

(2.6)



= MA(r0 + 2r0 )

t angential , A

(2.7)

WB-T = MBi

vertical, B

(2.8)

Because t he length of the string, l , is constant, we have

(2.9)

r+z=l

Differentiating this twice with respect to time gives the constraint equation r = -z (2 .10) ••

••

The negative sign means that if mass A moves outward, mass B would rise. Combining equations (2.6), (2.8), and (2. 10), we find

[email protected]

Physicsguide

37

@Sk Jahiruddin, 2020

Immediately aft er B is released , r

Newtonian Mechanics: Basics



= r 0 and 0 = w 0 . Hence (2 .11 )

i(O) can be positive, negative, or zero depending on the value of the numerator in equation (2. 11 ). If w 0 is large enough , block B will begin to rise after release. Before release, r = 0, but immediately after , the acceleration has a finite value. It is evident t hat because forces can be applied suddenly, acceler-

11 oo : 01

: 44

i(O) can be positive, negative, or zero depending on the value of the numerator in equation (2. 11). If w 0 is large enough , block B will begin to rise after release. Before release, r = 0, but immediately after, the acceleration has a finite value. It is evident t hat because forces can be applied suddenly, acceleration can change abruptly-acceleration can be discontinuous in time. In contrast, position and velocity are time integrals of acceleration and are therefore continuous in time.

[email protected]

@Sk Jahiruddin, 2020

38

Physicsguide

Newtonian Mechanics: Basics

Example 2.6: A block of mass M rests on a fixed plane inclined at an angle 0. You apply a horizontal force of Mg on the block, as shown in figure. Assume t hat the friction force between t he block and t he plane is large enough to keep the block at rest. What are t he normal and friction forces (call them N and Ff ) that t he plane exerts on the block? If the coefficient of static fri ction is µ, for what range of angles 0 will t he block in fact remain at rest?

11 oo : 01

: 46

enoug o eep and fri ction forces (call them N and Fr ) that the plane exerts on the block? If the coefficient of static friction is µ , for what range of angles 0 will t he block in fact remain at rest?

Mg

Figure 2.7:

Solution: Let 's break the forces up into components parallel and perpendicular to the plane. (The horizontal and vert ical comp onents would also work, but t he calculation would b e a little longer.) The forces are N , Fr, the applied Mg, and the weight Mg , as shown in figure below. Balancing the forces parallel and perpendicular to the plane gives, respec-

[email protected]

P hysicsguide

39

@Sk Jahiruddin, 2020

Newtonian Mechanics: Basics

tively (wit h upward along t he plane t aken to be positive

Fr= Mgsin0 - Mgcos0 N = Mg cos 0 + Mg sin 0

N

11 oo : 01

: 49

N = Mg cos 0 + Mg sin 0

N

Mg

Figure 2.8: The coefficient µ t ells us that Ff I < µN. Using equation above, t his inequality becomes

Mgl sin0 - cos0 :::; µMg(cos0

+ sin0)

(2. 12)

The absolute value here signifies t hat we must consider two cases: Case-1 : If tan 0 > l , then (2.12) becomes

sin0- cos0 < µ(cos0 [email protected]

+ sin0) =? tan0 < ~ + µ -µ

40

@Sk Jahiruddin, 2020

Physicsguide

Newtonian Mechanics: Basics

We divided by 1 - µ, so this inequality is valid only if µ < l. But if µ > l , we see from t he first inequality here t hat any value of 0 (subject to our assumpt ion, tan 0 > l ) works. Case-2: If tan 0 :::; 1, then (2.12) becomes - ~in A -1--

r'()~

A


...~ - ~ I dz

Iz

z

CT

R

l ~~~~ ,_ , _,

I

I I I I

dz

3 ,: Z =R

R

8

Figure 3.6:

Solution: From symmetry it is apparent t hat the center of mass lies on the z axis. Its height above t he equatorial plane is Z= 1 zdM M

11 oo : 04 : 11 Solution: From symmetry it is apparent that the center of mass lies on the z axis. Its height above the equatorial plane is Z = l zdM M The integral is over three dimensions, but the symmetry of the situation lets us treat it as a one-dimensional integral. We mentally subdivide t he hemisphere into a pile of thin disks. Consider a circular disk of radius r and thickness dz . Its vol2 ume is dV = 1rr dz , and its mass is dM = pdV = (M/V) (dV) ,

91

[email protected]

Physicsguide

@Sk Jahiruddin, 2020

Newtonian Mechanics: Basics

where V = ~1r R 3 . Hence

z=

M

l M

V zdV

l

R

1rr 2 z dz

V z=O To evaluate the integral we need to find r in terms of z . 2

Since r = R

2

-

z

2

,

we have R

z

(R

2

-

z

2

)

dz

0 7r

V

1 z2 R 2 - ~ z4 2 4

1 R4 - ~R4 V 2 4 11rR4 3 _ 1 =- R 7r

R 0

11

00:04:14

0

3.2

Center of mass frame

Center of mass frame (COM frame, or CM frame) is a frame where the total momentum of all t he particles is ZERO. Consider a frame S and another frame S' that moves at constant velocity u wit h respect to S as sho,"ln in the figure below. Given a system of particles, the velocity of t he ith particle in S is related to its velocity in S' by

(3.9) 92

[email protected]

Physicsguide

@Sk Jahiruddin, 2020

Newtonian Mechanics: Basics

y' y

S'

L..----• X,

s -----• X Figure 3.7:

Momentum of t he particle in the S and S' frame are t hen

11 oo : 04: 16 Figure 3. 7:

Momentum of the part icle in the S and S' frame are t h en

P =

L m iv i;

(3. 10)

Now we will make S' frame as COM frame (where t he total moment um of a system of part icles is zero), COM frame and CM frame both refer to center of mass frame. If the total momentum is P

I: mivi in fram e S, t hen t h e CM frame is

the frame S' that moves with velocity

P _ I: miv i U =

M =

with respect to S, where M _

(3. 11 )

M

I: m i is t h e total mass.

Then t he momentum in the S' frame is zero. V ·i

[email protected]

p M

= P- P =O

93

@Sk Jahiruddin, 2020

(3. 12)

P hysicsguide

Newtonian Mechanics: Basics

The CM frame is extremely useful.

Physical processes

are generally much more symmetrical in t his fr ame, and t his makes the results more transparent. The Cl\/1 frame is somet imes called the '' zero moment um'' frame. But t he '' center of mass'' name is commonly used because the center of mass of t he part icles doesn 't move in the CM frame, for t he following reason . You h ave already studied the t h e position of the center of m ass which is defined by

_I: miri R c:rvr = M

(3. 13)

11 oo : 04: 19 ing reason . You have already studied the t he position of the center of mass which is defined by _L miri R CM= M

(3.13)

If we t ake derivative of the above equation, we get M a cM

L m,i~= LFi=

F total

(3. 14)

So as far as the acceleration of t he CM goes, we can treat t he system of particles like a point mass at the CM, and then just apply F = ma to this point mass. since t he internal forces cancel in pairs, we need only consider the external forces when calculating F total Along with the CM frame, you will work in lab frame. There is not hing at all special about lab frame. It is simply t he frame (assumed to be inertial) in which t he condit ions of the problem are given. Any inertial frame can be called the '' lab frame." Solving problems often involves switching back and fort h between the lab and CM frames. For example, if [email protected]

@Sk Jahiruddin, 2020

94

Physicsguide

Newtonian Mechanics: Basics

t he final answer is requested in t he lab frame, t hen you may want to transform the given information from the lab frame to the CM frame where things are more obvious, and then transform back to t he lab frame to give the answer Example 3. 7: A mass m with speed v approaches a stat ionary mass M as shown in t he figure. What is t he velocity of center of mass (COM) frame? What are the velocities of the particles in the COM frame?

11 oo : 04: 21 Example 3. 7: A mass m with sp eed v approaches a stat ionary mass M as shown in the figure. What is t he velocity of center of mass (COM) frame? What are the velocities of the particles in the COM frame? m •

M

V



)Ii

Solution: The velocity of the COM frame is the sum over t he moment um of t he lab frame divided by t he total mass.

P

U =-

Af

I: m ·v · --'l

'l

M

mv+M ·O m+M

mv m+M

The velocities of t he two masses in the COM frame is then Vm

= v - u =

Mv m + M'

and

VM

=

0- u = -

mv m +M

We will come back to this example after we learn kinetic energy. [email protected]

Physicsguide

95

@Sk Jahiruddin, 2020

Newtonian Mechanics: Basics

Center of mass frame for two body system: Here we construct the COM frame of two body problem which will be very much useful for our study of tv\ro body problems later. Consider the case of a two-part icle syst em with masses m 1 and m 2 as shown in the part (a) of the figure below. 2

I

m;:,

z

11 oo : 04: 24 very much useful for our study of tv\ro body problems later. Consider t he case of a two-part icle system with masses m 1 and m 2 as shown in the part (a) of the figure below. z

z z'

y X

y

(a)

X

(b)

Figure 3.9:

In t he init ial coordinate system x, y, z, ( part (a) ), the particles are located at r 1 and r 2 and their center of mass is at R == m 1r 1 + m 2 r 2 m 1 +m2

We now set up the center of mass coordinate system, x', y', z', (part (b) ), wit h its origin at t he center of mass. The origins of t he old and new system are displaced by R . T he center of mass coordinates of t he two particles are r ~ == r 1 r ~ == r 2 [email protected]

@Sk J ahiruddin, 2020

96

R R Physicsguide

Newtonian Mechanics: Basics

Center of mass coordinates are t he natural coordinates for an isolated two-body system. Such a system has no external forces, so t he motion of the center of mass is t rivial-it moves uniformly.

11 oo : 04: 21 Cent er of m ass coordinates are t he natural coordinat es for an isolated two-body syst em. Such a syst em has no external forces, so t he m otion of the cent er of mass is trivial-it moves uniformly. By t he definition of center of m ass we have

Distance from m 1 t o m 2 , as a vector , is

Hence

3.3

conservation of momentum

Total external force F acting on a syst em is related to t he total momentum P of t he system by (3. 15) Consider t he implications of t his for an isolated syst em where there is no ext ernal force. In this case

dP dt [email protected]

@Sk Jahir uddin, 2020

97

=

0 P hysicsguide

Newtonian Mechanics: Basics

T he t otal moment um of an isolated syst em is constant . T his is the law of conservation of momentum.

11 oo : 04: 29 @Sk Jahiruddin, 2020

Newtonian Mechanics: Basics

The total momentum of an isolated system is constant. T his is the law of conservation of momentum. The integral form of equation (3.15) is t

Fdt = P (t) - P (O)

(3. 16)

0

We see t hat the change in moment um is the t ime integral of the force. To produce a given change in the momentum in t

F dt have the appropriate

time interval t requires only t hat 0

value; we can use a small force acting for much of the time or a large force acting for only part of the interval. t

F dt is called the impulse.

The integral 0

Now we'll solve some examples involving conservation of momentum and concept of impulse.

[email protected]

@Sk J ahiruddin, 2020

98

P hysicsguide

Newtonian Mechanics: Basics

11 oo : 04: 32 98

[email protected]

@Sk Jahiruddin, 2020

Physicsguide

Newtonian Mechanics: Basics

Example 3.8: A car of mass M is moving with a uniform velocity v on a horizontal road when a small brick

of mass m drops on it from above and stuck wit h it. W hat will be t he velocity of the car brick combo after the event? m m M

M

V

V

Figure 3.10:

Solution: In the horizontal direct ion, t here is no exter-

nal force. since t he hero has fallen vertically, so his initial horizontal momentum = 0 Initial horizontal moment um of the system = M v towards right T he brick sticks to t he roof of the car, so t hey move wit h equal horizontal velocity V. Final horizontal momentum of the

[email protected]

(c)Sk .T::i hir11 clclin . 2020

99

Physicsguide

11 oo : 04: 34

[email protected]

99

Physicsguide

@Sk J ahiruddin, 2020

Newtonian Mechanics: Basics

system is t hen

(M +m)V Momentum conservation gives

M v = (M +m)V Hence we get

V =

Mv M+m

Example 3.9: A man of mass m is standing on a platform of mass M kept on smooth ice. If t he man starts moving on the platform with a speed v relative to the platform, with what velocity relative to the ice does the platform recoil?

Solution: Consider the situation shown in figure below. Suppose the man moves at a speed w towards right and the platform recoils at a speed V towards left. Both the speed are relative to the ice. Hence, the speed of t he man relative to the platform is V + w . Now as given in the question the man's relative velocity to the ice is v, then

V + w =v

w=v-V

11 oo : 04: 37 w=v- V

V +w=v

[email protected]

P hysicsguide

100

@Sk Jahiruddin, 2020

Newtonian Mechanics: Basics

w Platform

V

Ice

Figure 3 .11:

Take the platform and the man together to b e the syst em , there is no external horizontal force on the system . The linear momentum of the syst em remains constant. Init ially, both the man and t he platform were at rest . So the initial momentum 1s zero. •

O= MV -mw Replacing the value of w

MV =m(v- V ) V = mv M+m

11 oo : 04 : 40

[email protected]

@Sk J ahiruddin, 2020

101

Physicsguide

Newtonian Mechanics: Basics

Example 3 .10: T wo ident ical blocks a and b each of mass m slide without friction on a straight track. They are attached by a spring with unstret ched length l and spring constant k; t he mass of the spring is negligible compared to the mass of the blocks. Init ially t he system is at rest . At t = 0, block a is hit sharply, giving it an instantaneous velocity v 0 to t he right. Find the velocity of each block at later times.

Look part (a ) of the figure below. The sit uation is drawn there. since the system slides freely after the collision, t he cent er of mass moves uniformly and therefore defines an inertial frame. Let us t ransform to center of mass coordinates. The center of mass lies at R = mra +mrb m+m 1 = (ra + rb) 2

00: 04: 42

[email protected]

102

@Sk Jahiruddin, 2020

vb(O)

=0

Physicsguide

Newtonian Mechanics: Basics

t--- - ra - - -, ', ra - rb ' Laboratory

=v 0

..

va(O)

coordinates

--~ : R rb - i I

b

r11lo11

a

a

b Q

I I

i- f1, -

I

~ r'a -

rb 1

(a)

l' 'J

I I

!\

Center of mass coordinates

Figure 3.12:

R always lies halfway between a and b. The center of mass

coordinates of a and b are r~

= Ta =

r~

=

1

2

-

(ra - rb)

Tb -

=-

R

1

2

R

(ra - rb)

= - r'a

In t he oart (b) t hese coordinates are drawn. The instanta-

11

oo : 04: 4s

- ra - rb 2

r~ = rb - R 1 = - (ra - rb)

= - r'a

2

In t he part (b) t hese coordinates are drawn. The instantaneous length of t he spring is r a - rb = r~ - r~ . The instantaneous departure of the spring from its equilibrium length l is

r a - rb - l = r~ - r~ - l. The equations of motion in the center

[email protected]

103

P hysicsguide

Newtonian Mechanics: Basics

@Sk Jahiruddin, 2020

of mass system are

mr'a = - k (r'a mr~ = +k (r~ -

r'b - l)

r~ - l)

The form of these equations suggests that we subtract them, obtaining

m (r: - r~) = - 2k (r: - r~ - l) It is n atural to int roduce t h e departure of the spring from its equilibrium length as a variable. Lett ing u

=

r~ - r~ - l , we

have

mu+ 2ku =

0

This is t he equation for simple h armonic motion. The general solution is

u = A sin wt + B cos wt

2k/m. since the spring is unstretched at t = 0, u(O) = 0 which requires B = 0. Then u = A sin wt so that u = Awcoswt. At t = 0 where w =

11

00:04:47

u = A sin wt + B cos wt where w = 2k/m . since the spring is unstretched at t = 0, u(O) = 0 which requires B = 0. Then u = A sin wt so that u = Awcoswt. At t = 0 u(O) and since u

= r~ - r~ -

l

= Aw cos(O)

= r a - rb -

l

= Vo [email protected]

104

@Sk Jahiruddin, 2020

P hysicsguide

Newtonian Mechanics: Basics

so that

A= vo/w Therefore

u = (v 0 /w) sin wt and

u = v 0 cos wt . . andVa ' = -vb, ' we h ave since va' - vb' = u, 1 va = -vb = vo cos wt 2 I

I

The laboratory velocities are •

Va = R + v~ vb = R + v~ •



since R is constant, it is always equal to its init ial value

11 oo : 04 : so •

Va=

R + v~ •

vb = R + v~ •

since R is constant, it is always equal to its init ial value

.

R

=

=

1

(va(O)

2 1 -Vo 2

+ Vb(O))

Putting these results together gives Va=

Vb =

[email protected]

Vo 2

Vo 2

(1 +coswt) (1 - cos wt)

Physicsguide

105

@Sk Jahir uddin, 2020

Newtonian Mechanics: Basics

Example 3.11: A loaded spring gun, init ially at rest on a horizontal frictionless surface, fires a marble at angle

of elevation 0 as shown in part (a) of the picture below. The mass of the gun is M , the mass of the marble ism, and the muzzle velocity of the marble (the speed with which the marble is ejected, relative to t he muzzle) is v 0 What is the final motion of the gun? v 0 sin 0

.,,,--.,,. Vo COS ()

X

(a)

v, {b)

-

Vf

11 oo : 04: s3 cos 0 M

v,

X

(a)

(b)

Figure 3.13:

Take the physical system to be the gun and marble. Gravity and the normal force of the table act on t he system. These external forces are both vertical. Because there are no horizontal external forces, the x component of t he vector equation

F = dP / dt is O = dP.x

dt

[email protected]

106

@Sk Jahiruddin, 2020

Physicsguide

Newtonian Mechanics: Basics

So, Px is conserved:

Px, initial = Px, final Let the initial t ime be prior to firing the gun. Because the system is initially at rest, Px, initial = 0, After t he marble has left the muzzle, t he gun recoils to the left with some speed V1, and its final horizontal moment um is - MV1 Finding t he final velocity of the marble involves another important concept. Physically, the marble's acceleration is due to t he force of the gun, and t he gun 's recoil is due to the reaction force of the marble. Look at the part (b) of the figure.

11 oo : 04: 55 Finding t he final velocity of t he marble involves another important concept. Physically, the marble's acceleration is due to t he force of the gun, and the gun's recoil is due to the reaction force of the marble. Look at t he part (b) of the figure. The gun stops accelerating once the marble leaves the barrel, so at t he instant the marble and the gun part company, the gun has its final speed - V1 At t hat same instant t he speed of the marble relative to t he gun is v 0 Hence, t he final horizontal speed of the marble relative to t he table is v 0 cos 0 - V1 . By conservation of horizontal momentum, we t herefore have

or

V _ mv0 cos0 J- M +m

Interesting, isn 't it? What is t he impact the marble give to t he floor? [email protected]

@Sk J ahiruddin, 2020

107

P hysicsguide

Newtonian Mechanics: Basics

Example 3.12: A block of mass Mis free to slide on a friction-less horizontal floor. The block has a cylindrical

cavity of radius R in the middle of it. The center of mass (CM) of the block lies on the dashed line passing through t he center of t he cavity (see figure). Init ially t he CM of the block is at a horizontal distance X 1 from the origin, Now a point particle of mass m is released from point A into the cavity. There is negligible friction between t he particle and t he cavity surface. When the particle reaches point B , t he CM of t he block is at a

11 oo : 04 : sa t he CM of t he block is at a horizontal distance X 1 from t he origin, Now a point particle of mass m is released from point A into the cavity. There is negligible friction between t he particle and the cavity surface. When the particle reaches point B , t he CM of the block is at a [JAM distance X 2 from the figure. Find (X 2 - X 1) 2009]

----A

- ~ 8

R

X1

Figure 3.14:

Solution: Center of mass of the total system should be [email protected]

Physicsguide

108

@Sk Jahiruddin, 2020

Newtonian Mechanics: Basics

conserved as there is no external force . The COM of t he combined system initially is MX1

+ m(X1 - R) M+m

After t he ball reaches the other side t he COJVI is

11 oo : os : oo M+m After t he ball reaches the other side t he COM is

Equating the initial and final coordinate of COlVI

Hence we get

[email protected]

@Sk J ahiruddin, 2020

109

P hysicsguide

Newtonian Mechanics: Basics

Example 3.13: In t his example we will measure t he speed of a bullet with the help of spring mass system. Consider a block of soft wood on a horizontal frictionless surface. A compression spring with spring constant k and uncompressed length l connects t he block to a wall. The block has mass M , and the spring has negligible

11 oo : os : 03 speed of a bullet wit h the help of spring mass syst em. Consider a block of soft wood on a horizontal frictionless surface. A compression spring with spring const ant k and uncompressed length l connect s t he block to a wall. The block has mass M , and t he spring has negligible mass. At t = 0 a gun fires a bullet of mass m and speed v 0 into the block. The block will absorb the bullet . We don't know v 0 , but we will be able to measure the maximum compression of the wood block, Xmax, after it absorbs the bullet. So we'll find vo in t erms of

k

Xmax •

m M

x~ 0

F igure 3.15:

Our syst em is the bullet and t he block. Let 's say t he block moves back at initial speed ¼ due t o the impulse. By conservation of momentum applied at very short times aft er the [email protected]

110

@Sk Jahiruddin, 2020

P hysicsguide

Newtonian Mechanics: Basics

collision

mvo = (M +m)¼

(3.17)

Conservation of moment um is accurate here, because during the very short t ime of t he collision the horizontal force of t he

11 oo : os : 06 collision

m vo = (M +m) ¼

(3.17)

Conservation of momentum is accurate here, because during the very short time of t he collision the horizontal force of the spring has very lit tle t ime to act. During t he ensuing time, however , t he spring force has plenty of t ime to act-it brings t he system momentarily to rest. Measuring how far the syst em moves can t ell us the speed of the bullet. After the initial impulse, the equation of mot ion of the system is (A1

+ m)x =

- kx

The spring length does not appear in the equation of motion because we have taken a coordinate syst em with x = 0 when the spring is uncompressed. We recognize the equation for simple harmonic motion , which has t he general solution x =

A sin wt + B cos wt

(3.18)

where

k

w=

M+m

Using the init ial conditions x(O) = 0, x(O) = ¼ in equation (3.18) we find A = ¼ /w and B = 0. The position and sp eed of the system are then

¼.

x = - s1n wt w

x= [email protected]

@Sk Jahir uddin, 2020

(3.19)

¼cos wt 111

P hysicsguide

Newtonian Mechanics: Basics

The syst em first comes t o rest at t = t f when wt f = 1r/ 2 which is the maximum compression of t he spring. Using equa.

.

11 oo : os : oa @Sk J ahiruddin, 2020

Newtonian Mechanics: Basics

The system first comes to rest at t = t f when wt f = 1r/ 2 which is t he maximum compression of t he spring. Using equati . (1. l 7J. ( 1.1~) and 1° .t-0)

= -;=====:::::J:::~ J. (.\/ - m) =

Xmax

so that

Vo=

Jk (M + m) m

X max

Example 3.14: A rubber ball of mass 0.2kg falls to t he floor. The ball hits with a speed of 8m/ s and rebounds with approximately the same speed. High speed photographs show t hat t he ball is in contact with t he floor

for '6.t = 10- 3s. What can we say about t he force exerted on t he ball by t he floor? The situation is drawn in figure 3. 16 (a)

jahir@physicsguide .in

@Sk Jahir uddin, 2020

112

P hysicsguide

Newtonian Mechanics: Basics

11 oo : os : 11 [email protected]

Physicsguide

112

@Sk J ahiruddin, 2020

Newtonian Mechanics: Basics

F

(a)

z

(b) - - - - - T - -:,-,,. I

l l

I I

Fav -----i----

1 I I I I I I

Figure 3.16:

The momentum of t he ball just before it hits the floor is P a = - l.6k kg · m/s and its momentum 10- 3s later is P b = + l.6kkg · m/s. Using ft:, Fdt = P b - P a gives F dt = 1.6k - ( - 1.6k) = 3.2k kg· m/s A

ft:,

A

A

A

Although t he exact variation of F with time is not known, it is easy to find t he average force. If the collision time is f:).t = tb - t a, t he average force F av acting during the collision •

18

ta+~t F av l:).t =

[email protected] ,..-...r't,

.,. , •

, ,.

_,..._,...

F dt

ta

Physicsguide

113

,

.,,.

.



,.

.r

1

11 oo : os : 13

[email protected]

Physicsguide

113

@Sk Jahiruddin, 2020

Newtonian Mechanics: B asics

A

= 3 ·2k

F

kg · m/s = 3200k N 10-3s

av

The average force is directed upward, as we expect. The inst antaneous force on the ball is even larger at t he peak, as shown in part (b) of the figure. If t he ball hits a softer surface t he collision t ime is longer , and the peak and average forces are less. So far we have neglected the impact by gravity. So we calculate it now. F = F ball + F grav A

= F ball - Mgk The impulse equation t hen becomes 10- 3

10- 3 A

F ball 0

A

M gk dt = 3.2k kg· m/s

dt 0

The impulse due to the gravitational force is 10- 3

10- 3 A

Mgk dt 0

A

= - Mgk

dt

= -(0.2) (9.8) ( 10-

3

)

k

0 3

= - 1.96 x 10- k kg• m/s This is small in this problem , but in some problems the impact due to gravity is comparable with the impact due to the velocity change. The next problem deals wit h it

11 oo : os : 16 T his is small in this problem , but in some problems the impact due to gravity is comparable with the impact due to t he velocity change. T he next problem deals wit h it [email protected]

114

P hysicsguide

@Sk Jahir uddin, 2020

Newtonian Mechanics: Basics

Example 3.15: A ball weight ing 100 gm, released from a height of 5 m, bounces perfectly elastically off a plate. The collision t ime between t he ball and t he plate is 0.5 s. The average force on the plate is approximately [NET June 2017] (a) 3 N (b ) 2 N (c) 5 N (d ) 4 N Solution: As we see the velocity in which t he ball strike •

lS

2

v = 2gs = 2 x 10 x 5 = 100

V

=

10

We take positive z axis upward so t hat the downward velocity is negative " V = -lOk Init ial moment um is Pi = m v =

0.1 x (- l Ok) = - lk

As it bounces perfectly elastically wit h t he same velocity (opposite direction) t hen t he final moment um is "

"

Pr = m vr = 0.1 x (10k) = lk Change in moment um due to velocity change is

11 oo : os : 1a posite direction) then the final momentum is A

A

Pr= mvr = 0.1 x (10k) = lk Change in momentum due to velocity change is

[email protected]

115

P hysicsguide

@Sk Jahiruddin, 2020

Newtonian Mechanics: Basics

So, t he force due to velocity change is F ball

~p 2 " " = = - k = 4kN ~t 0.5

This is the force on the ball by the plate. We are dealing the force on the plate here. According to t he Newton's third law t he ball exert same force on t he plate but in opposite direction. So t he force on t he plate is A

F plate

=

-4kN

Don't forget gravitational force and the impact due to gravity. Ball exert gravitational force on the plate downwards which •

18

F grav =

"

"

-Mgk = - lkN

So the total force on the plate by the ball is 4 + 1 = 5N

We will come across the impact of gravity when we do the chain problem.

3.4

Conservation of Energy in one and two

dimension

11 oo : os : 21 chain problem.

3.4

Conservation of Energy in one and two

dimension Newton's law in ID is

d2 x m dt2

=

dv mdt =F(x)

F(x)

[email protected]

Physicsguide

116

@Sk Jahiruddin, 2020

(3.20)

Newtonian Mechanics: Basics

We can solve t his by integrating mdv /dt = F(x) with respect to x Xb xb dv m -dx= F(x)dx (3.2 1) Xa di Xa we write the differential dx as dx =

dx dt

Then the integration of LHS is xb

m Xa

dv -dx=m di =m =m

So we get

dt = v dt

11 oo : os : 24 1

')

= -mv . .

b

2

So we get Xb

F (x)dx

(3.22)

Xa

The quantity ~ mv 2 is called t he kinetic energy K , and t he lefthand side can be written K b - K a. If at any point a particle [email protected]

117

Physicsguide

@Sk Jahiruddin, 2020

Newtonian Mechanics: Basics

1 2 with mass m has velocity v , we will call the quantity mv as 2 t he kinetic energy of the particle. Xb

F (x) dx is called the work Wba by the

The integral Xa

force F on the particle as t he particle moves from a to b. Our relation now takes the form

(3.23) This is the Work Energy theorem in one dimension. This means t he work done by t he force to move the particle from a to b is the difference between their kinetic energies at these two points. We then define the potential energy difference of the two positions as Xb

½ - Va=

F( x )dx

(3.24)

Xa

So the difference in potential energy is equals to t he difference in kinetic energy.

11 oo : os : 26 positions as Xb

½ - Va =

F( x) dx

(3.24)

Xa

So the difference in potential energy is equals to the difference in kinetic energy. The potential energy at any point x is defines as the integration of force from a reference point x 0 to the point x with a negative sign . X

V(x)

-

F (x') dx'

(3.25)

Xo

Sum of kinetic and potential energy is the total mechanical [email protected]

118

@Sk J ahiruddin, 2020

P hysicsguide

Newtonian Mechanics: Basics

energy of the particle. 1

mv 2

2

+ V(x) = E

(3.26)

If potential energy is of a particle is positive at any point we understand t hat we have done some work to take the particle at that position and if we release the particle then it goes to t he Zero potent ial energy level. And if the potential energy is negative then we say that the particle is bounded and ''we" need to perform work to take t he particle to the zero energy level. The potential energy is the integration of force, thus force is the derivative of potential.

F(x) = _dV(x) dx

(3.27)

Given V( x), it is easy to take its derivative to obtain F(x) .

11 oo : os : 29 is the derivative of potential.

F(x) = _dV(x) dx

(3. 27)

Given V (x), it is easy to take its derivative to obtain F(x) . But given F(x), it may be difficult (or impossible) to perform the integration in equation (3.27) and write V (x) in closed form. The function V (x) is well defined (assuming that the f'orce is a function of x only, and if needed it can be computed numerically to any desired accuracy Given any force (it can depend on x, v, t , and/or what ever), the work it does on a particle is defined by W _ J Fdx . If the particle starts at x 1 and ends up at x 2 , then no matter how it get s there (it might speed up or slow down, or reverse [email protected]

Physicsguide

119

@Sk Jahiruddin, 2020

Newtonian Mechanics: Basics

direction a few times), we can calculate t he total work done on it by all t he forces in t he setup, and then equate the result with the change in kinetic energy, via X2

X2

W total -

F total

dx =

m

v dv

dx

dx =

1

2

m v2

2

-

1 2

2

mv1

(3.28) Total mechanical energy of a syst em is conserved if there is no dissipative force. We call it the conservative force t hen.For conservat ive forces, t he work done is independent of how t he particle moves. A force that dep ends only on position (in one dimension) has t his property, because t he integral in equation (3.28) depends only on the endpoints. The W = J Fdx int egral is t he (signed) area under the F vs. x graph, and t his ::i.r P::i. i~ inr1PnPnr1Pnt nf hnw t.h P n ::i.r tirl P O'n P~ fr nm r1 tn

'rn

11 oo : os : 31 part icle moves. A force that dep ends only on position (in one dimension) has t his property, because t he int egral in equation (3.28) dep ends only on the endpoint s. The W = J F dx int egral is t he (signed) area under t he F vs. x graph, and t his area is indep endent of how t he particle goes from x 1 t o x 2 For non conservative forces, t he work done dep ends on how t he particle moves. T his happens when forces depend on t or v, because it then matters when or how quickly t he part icle goes from x 1 to x 2 . Consider frictional forces. If you move a brick sliding across a t able with friction from x 1 to x 2 , t hen the work done by friction equals - µm gl~ x . If you slide t he brick again and take it back to x 1 , t he work done is again - µm g ~ x as friction always opposes the motion , t he contribut ions to t he W = J F dx int egral are always add up.

jahir@physicsguide .in

120

@Sk J ahiruddin, 2020

3 .4.1

P hysicsguide

Newtonian Mechanics: Basics

Power

Power is t he rate at which work is done. T he work ~ W by force F on a syst em as it moves t hrough a short distance ~r is F · ~r. If the displacement takes place during time ~ t, t hen t he rate of work is ~ W ~ F. ~r ~t ~t In t he limit ~ t ➔ 0, ~r / ~ t ➔ v , so we have

dW = F . V

dt

(3. 29)

Power can b e eit her posit ive or negative depending on whether t .h P ,xr()r k- i ~

() TI

()r

h,,

t .h P

~,,~tPrn

11 oo : os : 34 t



v , so we

dW = F.

V

(3.29)

dt

Power can be either positive or negative depending on whether the work is on or by the system. 3.4.2

Gravitational potential energy:

We will have detailed discussion on gravitation later in a separate chapter. For now it is sufficient to know that t he gravitational force on earth attracts each object with a force Mg towards earth where g is t he gravitational constant. So the gravitational potential energy is (3.30)

mgz

where z is the upward displacement from the ground which is taken as the reference of zero potential energy. The gravitational potential energy is positive as if we release any part icle [email protected]

121

P hysicsguide

@Sk Jahiruddin, 2020

Newtonian Mechanics: Basics

at upward displacement z t he gravitational force does work and takes back the particle to t he zero potential energy level. If we release a particle ar rest at some height, it comes down and it's potential energy manifest as kinetic energy.

3.4.3

Potential energy of a spring

The forc e which spring exert when elongated or compressed ,

,



1

TT

,

'

,

11 oo : os : 36

3.4.3

Potential energy of a spring

The forc e which spring exert when elongated or compressed by length x is -kx. Here we have taken x to be zero at the equilibrium so that the variable x counts the displacement from the equilibrium position. So the work done is X

W =

}

kxdx = 0

kx

2

2

In two dimension force and displacement may not need to be on the same direction as shown in t he figure below.

F

Figure 3.17:

[email protected]

P hysicsguide

122

@Sk Jahiruddin, 2020

Newtonian Mechanics: Basics

The work done is then

W =

~

F · dr=

Fcos 0dr

(3.31)

Example 3.16: This is one of the most basic problem

where you need to apply t he conservation of energy. A small block of mass m starts from rest and slides along a frictionless loop-t he-loop as shown in the figure. What

11 oo : os : 39 Example 3.16: This is one of the most basic problem where you need to apply the conservation of energy. A small block of mass m starts from rest and slides along

a frictionless loop-the-loop as shown in the figure. What should be t he initial height z, so that m completes t he loop without being fallen down.

m a

mv 2 r

rng+N

z

Figure 3.18:

Solution: Initially the whole energy was kinetic energy

Now if the mass has to complete the loop without falling [email protected]

123

Physicsguide

@Sk Jahiruddin, 2020

Newtonian Mechanics: Basics

down, t hen it must go at the top of t he loop which is t he position a as shown in t he figure. It is easy to see that a is at distance 2R above the ground. So the energy at a is U1 = mg(2R) T T

,

TT

1

')

.

11 oo : os : 42 posit ion a as shown in the figure. It is easy to see that a is at distance 2R above the ground. So the energy at a is U1

== mg(2R)

Equating Ei and E f we get 1 2 mgz == mv + mg(2R) 2

At the position a, the loop the total downward force is N + mg , where N is the normal force exerted by the loop. If mv

2

the block is not to fall down then the total upward force - r must balance the downward force. mv

2

N +mg== R

Substitut ing t he value of v N +mg==

mv 2 R

2mgz

R

- 4mg

If the block just touches loop at point a, t hen N == 0, which would be the lowest limit on N and thus the lowest limit on z.

z 0 + m g == 2mg R - 4mg z == 5R/2

[email protected]

@Sk Jahiruddin, 2020

124

Physicsguide

Newtonian Mechanics: Basics

Example 3.17: This is another basic problem where you need to apply the conservation of energy and momentum together. A small cube of mass m slides down a circular oath of radius R cut into a large block of mass

11 oo : os : 44 Example 3.17: This is another basic problem where you need to apply t he conservation of energy and mo-

mentum together. A small cube of mass m slides down a circular path of radius R cut into a large block of mass M , as shown. M rests on a table, and both blocks move wit hout friction. The blocks are initially at rest, and m starts from the top of the path. Find the velocity v of t he cube as it leaves the block. ----

--- ~

R:

h

I I I

~

Ri v~

--V

1h -R Figure 3.19:

Solution: In the initial position (at the left of the figure) t he cube was at rest. No kinetic energy. So total energy was potential energy only

When the cube left t he block (as shown in t he right of the figure) t he velocities of both the block and cube cont ributes to t he K.E, and the potential energy will be mg( h - R) as the [email protected]

@Sk Jahiruddin, 2020

Physicsguide

125

Newtonian Mechanics: Basics

ball will be at height h - R from t he ground. 1

11 oo:os:47 @Sk Jahiruddin, 2020

Newtonian Mechanics: Basics

ball will be at height h - R from t he ground. 1

K = -mv f 2

2

1

')

+ -MV'"'· 2 '

U1 = mg(h- R )

So the final energy

1 2 E1 = -mv 2

1 2 -MV 2

+

+ mg(h - R)

Equating initial and final energy 1

2

mv

2

+

1

2

MV

2

+ mg(h -

R) = mgh

we get 1

mv

2

+

1

2

MV -mgR=O

2 2 Now we apply conservation of momentum Initial momentum was zero ~ = 0 Final momentum should also be zero P1 = mv-MV

=

0

Hence we get V=

3.5

M M +m

2gR

Collisions

There are two basic types of collisions among particles, elastic ones and inelastic. In elastic collisions the total mechanical [email protected]

@Sk J ahiruddin, 2020

126

P hysicsguide

Newtonian Mechanics: Basics

11 oo : os : 49 [email protected]

Physicsguide

126

@Sk Jahiruddin, 2020

Newtonian Mechanics: Basics

energy (kinetic + potential) is conserved) . In inelastic collisions some kinetic energy is lost so the total mechanical energy is not conserved . The lost energy t r ansform into heat. The moment um is conserved both in elastic and inelastic collisions because of absence of external force.

3.5.1

Collision in one dimension

Example 3.18: Consider the same situation as in ample Example 3.7:. A mass m with speed v proaches a stationary mass M as shown in the ure.what are t he final velocit ies of the particle after collision. (in the lab frame)? m

M

V

(1)

apfigthe

..... m

..

mM



•-•

ex-

Vf

(3)

(2)

Figure 3. 20:

Solution: Let Vf and ½ be t he final velocities of the mass m and M respectively. Then conservation of momentum and energy give, respectively, mv

1

-mv

2

[email protected]

2

+ 0 = mvf + M½

+0 =

1 2 -mvf

2

127

1

2

+ -2 M½

Physicsguide

11 oo : 05 : 52 mv

1 2 - mv

2

+ 0 = mvf + M¼ +0=

[email protected]

1 2 - m vf

2

1

2

+ -2 M½

127

Physicsguide

@Sk Jahir uddin, 2020

Newtonian Mechanics: Basics

We must solve t hese two equations for t he two unknowns Vf and ½. Solving for ½ in the first equation and substituting into t he second gives 2

2

m (v - Vf) m v = m vf + M - -Af2 --2

2

=::::.~►

0 = (m + M)vl - 2mvvf

=::::.~►

0 = ((m + M )vf - (m - M )v) (vf - v)

+ (m -

M )v

2

One solut ion is Vf = v, it satisfies conservation of energy and momentum. But in this solut ion the collision does not happ en. The particles miss each other. The Vf = v (m - M) /( m + M ) root is t he one we want. Substitut ing t his Vf in momentum conservation m v + 0 = mvf + M¼ we get ½.

(m Vf

=

m

M)v

+M

'

and

½ _ f-

2mv m+ M

Now we will consider t he general collision in 1D where both t he particle was moving before t he collision . Let t he collision be as drawn in the figure below

m



Vj

M





•Vi

c> Collisi on

(1)

M

m

•Vf •

•Vr

(2)

Figure 3.21:



11 oo : 05 : 55 Vj

Collision

Vf

(1)

(2)

Figure 3.21:

[email protected]

Physicsguide

128

@Sk Jahiruddin, 2020

Newtonian Mechanics: Basics

The masses be m and M was moving wit h velocit ies Vi and ½ before t he collision, and Vf and ½ are the fin al velocities after t he collision. Conservation of momentum and energy

(3 .32) 1

2

2mvi

1

2

+ 2M½ =

1

2

2mvf

+

1 2 2M½

(3.33)

Rearranging

= M (½ - ½) 2 2 m (vf - vl ) = M (½ - ½ )

m (vi -

Vf)

Dividing t he second equation by the first gives vi+v£ Therefore,

(3.34)

= ½+ ½.

(3.35) Here we get one very important result. In a 1- D elastic collision, the relative velocity of the two particles after the collision is the negative of the relative velocity before the collision. Now do some algebra and prove that 'YYl

_

l\ If

') AIf

11 oo : 05 : 57 collision, the relative velocity of the two particles after the collision is the negative of the relative velocity before the collision.

Now do some algebra and prove that VJ

m- M 2M = - - - V i + - - -¼ m+M m+M 129

[email protected]

(3.36) P hysicsguide

@Sk J ahiruddin, 2020

Newtonian Mechanics: Basics

V1 =

2m m- M M Vi M Vi

m+

m+

(3 .37)

Perfectly inelastic collision: When perfectly inelastic bodies moving along t he same line collide, they stick to each ot her. Let m and M be t he masses, Vi and ¼ be t heir velocit ies before t he collision and V be the common velocity of the bodies after t he collision. By t he conservation of linear momentum, mvi

+ M¼ = mV + MV V = mvi + M¼ m+M

let us calculate how much kinetic energy is lost in this process. The kinetic energy before the collision is 1

2

2mvi

1

+ 2M¼

2

and t hat aft er t he collision is ½(m + M ) V 2 . Using the expression of final velocity, t he loss in kinetic energy due to the

11 oo : 06 : oo 2

pression of final velocity, t he loss in kinetic energy due to t he

[email protected]

Physicsguide

130

@Sk J ahiruddin, 2020

Newtonian Mechanics: Basics

collision is 1 2 1 2 -mv-2 + -MV2 - - (m + M) V 2 2 2 1 2 1

2

1 mM (v; + ½ - 2vi¼) 2 m+M 2

mM (vi - ¼) 2 (m + M)

2

Coefficient of restitution: For a perfectly elastic collision velocity of separation = velocity of approach and for a perfectly inelastic collision velocity of separation = 0 In general, collisions are neither perfectly elastic nor perfectly inelastic. So we can write velocity of separation = e(velocity of approach) where O < e < 1. The constant e depends on t he materials of t he colliding bodies. This constant is known as coefficient of restitution. If e = 1, the collision is perfectly elastic and if

11 oo : 06 : 03 velocity of separation= e(velocity of approach) where O < e < 1. The constant e depends on the materials of t he colliding bodies. This constant is known as coefficient of restitution. If e = 1, the collision is perfectly elastic and if e = 0, the collision is perfectly inelastic. 3.5.2

1D collision in COM frame

In t he COM frame t he total momentum is zero bot h before and after the collision. As t he energy is conserved then t he 131

[email protected]

Physicsguide

@Sk Jahiruddin, 2020

Newtonian Mechanics: Basics

particles just exchanges t he velocit ies in the COM frame. This can be proved in one dimension collisions as follows. We'll discuss the two dimensional case later. Suppose the particle with mass m 1 and m 2 are moving with velocities u~ and u 2 in t he COM before the collision. After t he collision the velocit ies are v~and v;. In the COM frame t he total momentum are always zero. So

from m1u~

+ m2u; =

0 we have

Hence m1v~

+ m2v; = 0 v~/v; = -m2/m1 = U1 I U2 I

I

11 oo : 06 : os Hence m1v~

+ m2v; = 0 v~/v; = -m2/m1 = U 1 I U2 I

I

The kinetic energy before and after t he collision are conserved. So ,

We have already seen V1

v'2 [email protected]

u~ - = k say u'2 132

@Sk Jahiruddin, 2020

Physicsguide

Newtonian Mechanics: Basics

Now in t he energy conservation equat ion substit ute u~ = ku; and v~ = kv; . Then you get >

So we get two set of solutions. One is the trivial one v~ u~; = u~ . Here the particle missed. The real solution is

v;

So what we have proved is, in COM frame lD collision the particle velocities remains same in magnitude but opposite in direction, t he velocities are just reversed. Example 3.19: Now consider the examples Example 3. 7: and Example 3.18: where we have studied a simple lD collision. in the first example we have calculated the velocity of COM frame, t he initial velocities in COM

11 oo :06 : oa direction, t he velocities are just reversed. Example 3.19: Now consider the examples Example 3. 7: and Example 3.18: where we have studied a simple 1D collision. in t he first example we have calculated the velocity of COM frame, t he initial velocities in COM

fr ame and in t he second example we calculated t he final velocities in t he lab frame using conservation of energy and momentum. Now we know that t he velocities simply reversed in COM frame then we cal easily calculate the final velocities in the lab frame more easily.

mv Solution: The velocity of COM frame was - - - In m+M the COM frame the velocities of mass m and M was u~ = Mv mv . and u~ = M respectively. Now in COM frame

m+ M

m+

[email protected]

133

Physicsguide

@Sk Jahiruddin, 2020

Newtonian Mechanics: Basics

t he velocit ies just reversed, so in t he COM frame t he velocities after collision are simply V

,

-

1 -

-

Mv ----· m+ M '

, V2

mv = m+M

To calculate t he velocities in lab frame after the collision can be calculates adding t he velocity of COM frame with v~ and v~. m-M Vlab = v' + mv i i m+M m+M and lab , mv 2mv V2 = V2 + M m+ m+M This agrees wit h t he lab frame velocities calculates in example Example 3.18: .

00 : 06 : 10 and

2mv = + m+ m+M This agrees with the lab frame velocities calculates in example Example 3.18: . lab V2

mv M

, V2

Here is a schematic of general 1D collision both in Lab and in COM frame Lab after collision

Lab before

collision

0

·

v, . v,' + vcm = -u 1+2vcm

Transform to CM frame

0

·

V2- V2'+ vcm -u 2+ 2vcm

'(} Transform back to the Lab frame CM before collision

m,

CM after collision

Vl I

= -u l

-o

V 2I

I

O

= -U2

I

·

Figure 3. 22:

[email protected]

134

@Sk Jahiruddin, 2020

3.5.3

Physicsguide

Newtonian Mechanics: Basics

Collision in two dimensions

In two dimension one extra moment um equation comes as we need to divide the momentum into the coordinate axes and apply conservation of momentum in each axis separately. So we have two momentum conservation equations and one energy conservation equation. In t hree dimension we have t hree momentum equations and one energy equation. We'll illustrate wit h one example.

Example 3.20: A billiard ball wit h sp eed v approaches an identical stationary one. The balls bounce off each other elastically, in such a way that the incoming one

11 oo : 06 : 13 ree momen um equa ions an illustrate wit h one example.

one energy equa 10n.

Example 3.20: A billiard ball wit h sp eed v approaches an ident ical st ationary one. The balls bounce off each

other elastically, in such a way that the incoming one get s deflected by an angle 0 as shown in the figure. What are the final speeds of the balls? What is t he angle which t he stationary ball is deflected?

m



V

at

•m

m



,.,

--- -

VJ

•m

Figure 3. 23:

Solution:

Let

Vf

and ½ be the final speeds of the balls. 135

[email protected]

Physicsguide

@Sk Jahiruddin, 2020

Newtonian Mechanics: Basics

T hen conservation of momentum in x and y direction gives

mv = mvf cos 0 + m¼ cos 0 = m Vf sin 0 - m ¼ sin

(3.38)

Conservation of energy gives 1

2

1

2

-mv = -mvf 2

2

1

2

+ -m½ 2

(3.39)

We must solve these t hree equations for the t hree unknowns Vf, ¼, and . There are various ways to do t his. You may

11 oo : 06 : 16 1

1

2

2

-mv = -mvr 2 2

1

2

+ -m½ 2

(3.39)

We must solve these t hree equations for the three unknowns V£, ½ , and cp. There are various ways to do t his. You may proceed by this way. From t he moment um equations we eliminate cp by taking cp terms in one side of t he equation , square and add.

= m v + mvrcos0 2 2 2 2 2 2 2 2 m ½ cos cp = m v + m vf cos 0 + 2m vv£ cos 0

m½coscp ~~►

0 = mvrsin0 - m½sin cp Add these and get

v

2

-

2vvr cos 0 + v; =

2

½

(3.40)

Now eliminate ½ by combining t his with the energy conservation equation to get.

vr = vcos0 The energy conservation equation then gives [email protected]

136

@Sk Jahiruddin, 2020

Physicsguide

Newtonian Mechanics: Basics

½ = vsin0

(3.41)

Substitute v f and V1 in momentum conservation equation in y direction. Now it looks

m (v cos 0) sin 0 = m (v sin 0) sin cp This gives

(3.42)

11 oo :06 : 1a y direction. Now it looks

m( v cos 0) sin 0 = m( v sin 0) sin

X

Mgx L

~

X+t.X

Mg (x+t.x) L ~

(1)

(2)

(3)

The momentum of the chain at situation (2) is A.nitial

= MV

The mass of the hanging portion is M

11 oo : 07 : 1 a e momen um o ..A.nitial

= MV

The mass of the hanging portion is

M

LX

External force at situation (2) is Mg

F external ( [email protected]

t) = L

X

158

Physicsguide

@Sk Jahiruddin, 2020

Newtonian Mechanics: Basics

The momentum at situation (3) is ..A.nitial

+ !::l.P = M (V + f::l.v)

External force at situation (3) is F external ( t

Mg

+ f::l. t) = L (X + f::l. X)

Now you see that the force is not constant. But you can make f::l. x as small as possible and effectively the force becomes

constant Fexterna1(t) =

:g x in the motion from x to x + !::,.x

So the change in moment um is the integration over force.

f::l. p

=

M f::l. V

t+D.t

=

t

M gx f::l.t F external dt ~ l

Divide by f::l.t and make the limit of f::l.t equation of motion



0, we get the

11 oo : 07 : 20 t+~ t

I:),. p = M I:),. V =

t

M gx /:),.t F external dt ~ l

Divide by /:),.t and m ake the limit of equation of motion dv

2

d x

gx

dt

dt 2

L

/:),. t ➔

0, we get t he

The general solut ion of t he different ial equation is x(t)

= Aelt + B e- lt

[email protected]

Physicsguide

159

@Sk J ahir uddin, 2020

Newtonian Mechanics: Basics

We will apply t he init ial condit ions to t he solut ion. At t ime t = 0 (sit uation (1) )

x(O) = A+ B = Lo At t = 0 t he chain is at rest

Hence

A = B = Lo/2 and t he final solut ion X

4.3.2

Lo

(t) = 2

9-t

eL

- 9-t

+e

L

= L 0 cosh ft L

Chain falling on a weight machine

11 oo : 07 : 23 x (t ) == -

4.3.2

eft

+ e - ft == L 0 cosh

-t L

Chain falling on a weight machine

A chain of mass M and length L is suspended vertically with its lower end touching a scale. The chain is 1~e1eased and falls onto t he weight machine. What is t he reading of the scale when a length x of the chain has fallen? The reading of the weight machine will come from two parts. The first is obviously the weight of t he chain accumulated on the machine. As the chain has fallen t he distance x [email protected]

Physicsguide

160

@Sk Jahiruddin, 2020

Newtonian Mechanics: Basics

on t he weight machine, t he weight due to t his part is

W1 = Mgx/L The second component contributing to the weight is due to the momentum delivered by the chain to the weight machine. As t he chain is falling freely the velocity of t he chain parts at t he instant of hitting t he scale is simply found from v

2

= 2gx

So t he momentum dp delivered in time dt dp = d(m v) = vdm

where the v has t aken to b e constant as t he momentum is delivered in very small time dt .

11 oo : 07 : 26 dp = d(mv) = vdm

where the v has t aken to be constant as t he momentum is delivered in very small time dt. The chain is moving in velocity v, t he length of the chain falls down to the machine in time dt is dx = v dt. The mass per unit lengt h is M / L. So the mass dm which falls on the weight machine in time dt is nothing but dm

M - dx

=

L

=

M - vdt

L

So the momentum dp delivered in time dt dp = d(mv)

=

M M 2 v dm = v L vdt = L v dt 161

[email protected]

Physicsguide

@Sk Jahiruddin, 2020

Newtonian Mechanics: Basics

So the force which will be exerted by t he chain to the weight machine is the rate of change of this momentum delivered dp dt

As v 2

= lim '6.p = M v 2 ~t➔O

L

'6.t

= 2gx, so the force due to momentum delivered becomes dp

M 2 - v

W2= - = dt L

=

X

2Mg-

L

The t otal force on the scale during the fall of t he chain is therefore X X X Mg-+ 2Mg- = 3MgL L L r"i1

1

Jl

I



,



J

,



l

l

rr,1

11 oo :07 : 2a The total force on the scale during the fall of t he chain is therefore X X X Mg-+ 2Mg- = 3MgL

L

L

Check that energy is not conserved in this problem. The chain falling on a weight machine problem can be solved in other ways. Think on the other methods.

4.4

Other problems on variable mass

4.4.1

Cart with leaking water tank

A municipality car of empty mass M 0 contains water tank of m. At t = 0 a constant horizontal force F is applied in the direction of rolling and at the same water tank gets a leakage [email protected]

@Sk Jahiruddin, 2020

162

Physicsguide

Newtonian Mechanics: Basics

and water starts flow out at constant rate dm/ dt . Find the speed of the car when all t he water is gone. Assume the car is at rest at t = 0. Ignore friction. I think this is a very common situation and most of you have seen a gaddi with water leaking from it. Our system consists of t he freight car and water initial mass Mo at t = 0. The bottom is opened at t = 0,. We see that the water starts leaking at steady rate. We call the rate of water leakage is , , which means "/ amount of water falls per second. So that

11 oo : 07 : 31 Our system consists of t he freight car and water initial mass Mo at t = 0. The bottom is opened at t = 0,. We see t hat t he water starts leaking at steady rate. We call t he rate of water leakage is ry, which means ry amount of water falls per second. So t hat

dm/dt = ry Total mass of the car plus water system is

M(t) = M0 + m(t) Total mass M also decreases with t ime as t he water gets out . dM/ dt = - ry. Then

M (t) = Mo - ryt Now we will apply conservation of moment um to solve the problem.

[email protected]

163

@Sk Jahiruddin, 2020

Physicsguide

Newtonian Mechanics: Basics

Say at time t t he cart has mass M (t ) and velocity v(t) which is sit uation (1) in t he figure below. T he moment um is

P (t) = M v

-v

F

F

11 oo : 07 : 33 = Mv

P (t)

F

V

) v+~v

F

~ B v~v (1)

(2)

Figure 4.4:

At time t + fl.t t he cart has mass M - fl.m and velocity v + C:l. V which is situation (2) in the figure below. The small amount of water dm which has just come out from the cart is also moving horizontal! with velocity v + fl.v . Its moment um in vertical direction does not count as out force and motion are in horizontal direction. The momentum is now P(t + fl.t )

= (M - fl.m)(v + fl.v) + fl.m(v + fl.v)

[email protected]

164

@Sk Jahiruddin, 2020

P hysicsguide

Newtonian Mechanics: Basics

The change in moment um fl.P

= P (t + fl.t ) - P (t)

~

M fl.v

(ignoring second order term fl. mfl.v, we always do that in variable mass problems ) So the equation of motion

11 oo : 07 : 36 (ignoring second order term ~ m ~ v, we always do that in variable mass problems ) So the equation of mot ion dv dt

F = dP = Mdv dt dt

F

F

M

M o-,t

Water t ank is emptied at time t 1, so that t1

= m /,

Then we integrate the equation of motion noting t hat at t = 0 : v = 0 and at t = t 1 : v = v f v(t1)

t1

dv = 0

0

v (t1) = -

Pdt' Mo - , t' F Mo - , t1 ln Mo

'

4 .4 .2

F

'

In

Mo M 0 -m

Sand on cart

A car of empty mass M 0 start s moving due to a const ant horizontal force F as shown in the figure below. Sand spills on the car from a stationary hopper. The velocity of sand loading [email protected]

@Sk Jahiruddin, 2020

165

P hysicsguide

Newtonian Mechanics: Basics

is constant and equal to , kg/s. Find the t ime dependence of the velocity and the acceleration of the flatcar in the process of loading. Ignore friction . The situation is drawn in the figure below. Suppose at

11 oo : 07 : 39 the velocity and the acceleration of the flatcar in the process of loading. Ignore f1·iction. The situation is drawn in the figure below. Suppose at t ime t, the velocity of the sand plus cart syst em is v . At t ime mass of sand has already been deposited , so the total t ime mass 1s (situation(l)) M = Mo +,t

,t •

After time ~ t the mass ~ m = dm has been deposited to the cart and velocity has become v + ~ v (situation (2) ).

F

F

-

-

> v

v+ t. v

B

(1)

(2)

Figure 4.5:

There is one fundamental difference in this problem. The [email protected]

@Sk Jahiruddin, 2020

P hysicsguide

166

Newtonian Mechanics: Basics

mass ~ m which will be deposited in t ime ~ t do not have an initial horizontal velocity with respect to the lab frame in which we are observing. This is the fundamental mist ake .

.

11

00:07:41

mass l:lm which will be deposited in t ime l:lt do not have an initial horizontal velocity with respect to the lab frame in which we are observing. This is the fundamental mistake students do here. Although the source of t he sand is moving, but from our viewpoint t he sand is just being deposited to t he cart, the velocity of the sand source has nothing to do with our system and force. The mass may also be raindrops or ice flowing from the sky. In all these cases t here is no initial horizontal velocity of the mass deposited. Initial momentum is

P (t ) = M v Final momentum is

P (t+ l:lt) = (M + l:lm)(v+ l:lv) So the change in moment um is

l:lP

=

P (t

+ l:lt) -

P (t)

dP dt

Mdv dt

=

M l:lv

+ l:lmv

So the force

F

=

=

+ v dm dt

We know that

M(t ) = Mo + r-.; t [email protected]

167

@Sk J ahiruddin, 2020

and

P hysicsguide

Newtonian Mechanics: Basics

dm

dM

11 oo : 07 : 44 [email protected]

167

Physicsguide

@Sk Jahiruddin, 2020

Newtonian Mechanics: Basics

and

dm dM dt = dt = 1

So the equation of motion becomes

dv F = (Mo + ryt) dt

+ vry

dv (F - 1 v) = (Mo+ ryt ) dt v dv t dt 0 ( F - ryv ) 0 (MO + ryt) Perform t he integration, it is not tough _ _!_

,

_ _!_

ln[F - ,v]~

=

_!_

'Y

[ln (m0 + ryt)]~

I In mo + ryt mo F 'Y F mo + ryt = ln F - 1v mo F mo + ryt F - 1v mo

ln F - 1 v

'Y

ln

Simplifying Ft

v = --m0

+ 1t

t he velocity is along the direction of t he force. So writing vectorically Ft v = --m0 + 1 t

[email protected]

@Sk J ahiruddin, 2020

168

Physicsguide

Newtonian Mechanics: Basics

11 oo : 07 : 46 v = ---

m0+,t

[email protected]

168

P hysicsguide

@Sk Jahir uddin, 2020

Newtonian Mechanics: Basics

The acceleration

dv a =dt

4.4.3

F

Growing raindrop

A raindrop of initial mass M 0 starts falling from rest under the influence of gravity. Assume that the drop gains mass from the cloud at a rate proportional to the product of its inst antaneous mass and its instantaneous velocity: dM

dt

= kMV

where k is a constant . We'll calculate t he equation of motion and t hen find t he t erminal velocity. Weill proceed like previous problems. Consider the change in moment um of the drop as it gains mass during the time interval from t to t + ~ t Initial moment um at time t P(t) = MV

Final momentum at t ime t + ~t P (t [email protected]

+ ~t) = (M + ~M)(V + ~V) 169

P hysicsguide

11 oo : 07 : 49 Final momentum at t ime t P (t

+ ~ t) =

+~t

(M

[email protected]

+ ~ M)(V + ~V ) Physicsguide

169

@Sk Jahiruddin, 2020

Newtonian Mechanics: Basics

Change in momentum

The force applied is the gravitational force F

=

M

=

g

dP dt

=

MdV dt

VdM + dt

As it is given that d:: = kMV, the equation of motion becomes MdV kMV 2 = M dt + g or dV dt

= - kV2 g

I'm sure you have solved this differential equation while you studied the disspative force problem in section 2.2. So solve t he problem completely. From t he equation of motion you see t hat the acceleration decreases as the falling drop gains speed. The t erminal is the velocity when the acceleration becomes ZERO and t hus when v = Vterminal, ~i; = 0, hence V'terminal

=

g/ k

Here we end our discussion on Basic Newtonian Mechanics. Next is Advanced Topics in Mechanics

00: 00: 00

N e-wtonian Mechanics: Advanced

® +

Sk J ahiruddin *

*Assistant Professor

" " Sister Nibedita Govt. College, Kolkata " Author was the topper of IIT Bombay M.Sc Physics 2009-2011 bat]" • He ranked 007 in IIT JAM 2009 and 008 (JRF) in CSIR NET June 2~ 0 _ He has been teaching CSIR NET aspirants since 2012

(!>

X

1 @Sk Jahiruddin, 2020

Newtonian Mechanics: Advanced

11 oo : oo : oo

N e-wtonian Mechanics: Advanced Sk J ahiruddin *

*Assistant Professor Sister Nibedita Govt. College, Kolkata Author was t he topper of IIT Bombay M.Sc Physics 2009-2011 batch He ranked 007 in IIT JAM 2009 and 008 (JRF) in CSIR NET June 2011 He has been teaching CSIR NET aspirants since 2012

1 @Sk Jahiruddin, 2020

Newtonian Mechanics: Advanced

11 oo : oo : 03 1 Newtonian Mechanics: Advanced

@Sk Jahiruddin, 2020

Contents 1

2

Conservation of energy in three dimension

3

1.1

4

Conservative force: . . . . . . . . . . . . . . .

Collision in 2D in Center of Mass frame

18

2.1

Formulation of t he problem

2.2

Elastic scattering when target particle at rest 2.2.1























2





















18 23

Kinetic energies in the Lab and COM frame • • • • •

[email protected]





29

Physicsguide

.,

,

11 oo : oo : os

jahir@physicsguide .in

2

P hysicsguide

@Sk Jahir uddin, 2020

1

Newtonian l\/[echanics: Advanced

Conservation of energy in three dimension

T he conservation of energy can be easily extended t o three dimension. Potent ial energy in t hree dimension becomes dependent on t he distance from t he origin (radius vector) and it is t he integration of forces wit h t he radius vector from a reference point ro . r

U(r)

F (r') · dr'

-

(1.1)

ro

Force and displacement may not be in t he same direction as shown in t he figure below. In two dimension case t his becomes F r cos 0. In t hree dimension we need to perform line integral. T he integrat ion of force give t he difference of kinet ic en-

11 oo : oo : os

[email protected]

Physicsguide

3

Newtonian 1/[echanics: Advanced

@Sk Jahiruddin, 2020

ergy as before.

F =mdv dt b

F · dr a

=

dv m--dr dt a b dv m dt · vdt b

(1.2)

a

bm

a

1.1

d

2 dt (v2) dt

Conservative force:

A force F acting on a particle is conservative if and only if it satisfies two conditions: (i) F dep ends only on the particle's posit ion r ( and not on t he velocity v , or t he t ime t , or any other variable; that is, F = F (r) (ii) For any two points 1 and 2 , the work W (l ➔ 2) done by F is the same for all paths b etween 1 and 2 A force F (r) to be conservative a necessary and sufficient condition is to h ave the curl of the force to be zero.

(1.3) Conservative force means the work done is path independent . \"!ve will discuss t hese ooint in details. Let us first write the

11 oo : oo : 1o and su . . . . . cient condition is to h ave the curl of the force to be zero.

(1.3) Conservative force means the work done is path independent. We will discuss these point in details. Let us first write the force and potential energy relation in three dimension. Let us [email protected]

Physicsguide

4

@Sk Jahiruddin, 2020

Newtonian Mechanics: Advanced

consider a particle acted on by a conservat ive force F (r), wit h corresponding potent ial energy U (r), and examine the work done by F (r) in a small displacement from r to r + dr. We can evaluate this work in two ways. On t he one hand, it is, by definition ,

W(r



r + dr) = F (r) · dr

(1.4)

= Fxdx + Fydy + Fzdz

for any small displacement dr with components (dx , dy , dz) . On t he other hand, we have seen that the work W (r ➔ r + dr) is t he same as (minus) the change in PE in the displacement:

W(r

➔ r

+ dr) =

+ dr) - U(r )] = - [U (x + dx, y + dy , z + dz) -dU = -[U(r

U (x, y , z) ]

(1.5) We know from t he definition of derivative dU = U(x + dx, y + dy, z + dz ) - U(x, y, z) au au au ax dx + ay dy + a z dz

(1.6)

So the work done is

W(r



r + dr) = -

au ax dx

+

au ay dy

+

au a z dz

(1. 7)

11 oo : oo : 13 -

-

ax

y

az

So t he work done is

au au au axdx + ay dy+ az dz

W (r -+ r + dr ) = -

(1. 7)

The two expression of work in equat ion (1.4) and (1.7) are equivalent . So we get

au Fx = - ax'

au Fy = - ay'

jahir @physicsguide.in

Fz = -

au az

(1.8) P hysicsguide

5

@Sk Jahir uddin, 2020

Newtonian Mechanics: Advanced

T hat is, F is t he vector whose t hree components are minus t he three partial derivatives of U wit h resp ect t o x , y , and z . A slightly more compact way to writ e this result is t his:

F =

-x au _ -s, au _ zau ax ay az

(1.9)

From vect or calculus we know t hat t he gradient of a scalar function is defined as

nf _ V

-

af af af X ax + y ay + z az A

A

A

(1.10)

So the force is the gradient of t he potential with a negative sign F =-V U (1.11 ) •

This important relation gives us the force F in t erms of derivat ives of U, just as the definit ion (1.1) gave U as an integral of F. When a force F can b e expressed in the form (4.33), we say t hat F is derivable from a potential energy. Thus, we have sho"'rn that any conservative force is derivable from a potential energy.

11 oo : oo : 16 of F. When a force F can be expressed in the form (4.33) , we say t hat F is derivable from a potential energy. Thus, we have sho~rn that any conservative force is derivable from a potential energy. Example 1.1: The potential energy of a certain particle

is U = A xy 2 + B sin C z , where A, B and Care constants. What is the corresponding force?

Solution: To find F we have only to evaluate the three partial derivatives. In doing this, you must remember that [email protected]

P hysicsguide

6

@Sk J ahiruddin, 2020

Newtonian Mechanics: Advanced

auI ax is found by different iating wit h respect to X' treating y and Z as constant, and so on. Thus au j ax = Ay 2 , and so on, and the final result is F = - (xAy

2

+ y2Axy + zBC cosCz )

Now we prove the statements about the conservative force stated before. Suppose a force is conservative which essentially means that the work done to move one point to another point is path independent. Look at the figure below. We do work to move particle from point A to B. Path I is the path AC B , Path II is the path AD F B. The work done is same for both paths.

F · dr

WAcB = ACB

F · dr

= WAD FB = ADFB

Path I C ---------.

s

11 oo : oo : 1s ACB

ADFB

Path I C -----------

s

D

A

Path II Figu1·e 1.1 :

[email protected]

Physicsguide

7

@Sk J ahiruddin, 2020

Newtonian Mechanics: Advanced

So

F · dr ACB

F-dr=O ADFB

Now t he sign of line integral changes when we reverses the direction of integration.

F · dr + ACB

F · dr = 0 BFDA

hence

F · dr = 0 So the work done in traveling the whole path once in a closed cycle is ZERO. From the stokes theorem we know

F · dr = ~() if thP lin P intPP'r::l.l nf

::i.

(V x F ) · n ds

,rPrtnr ::l.r n,,nn

:=i.

rln~Pn r .,,r,rP i~ 7.Prn

11 oo : oo : 21 From the stokes theorem we know

F · dr =

(V x F ) · nds

So if t he line integral of a vector around a closed curve is zero then the curl of t he vector must be zero. V x F = O

Finally we summaries our discussion of conservative force If a force F is conservative in a region then all these following condition satisfies. Any one of the following five conditions implies all the others. • V x F = 0 at every point of the region. [email protected]

@Sk J ahiruddin, 2020

Physicsguide

8

Newtonian Mechanics: Advanced

• § F ·dr = 0 around every simple closed curve in t he region. • F is conservative, that is

JJ F · dr is independent of the

path of integration from A to B. course, lie entirely in t he region.)

(The path must, of

• F · dr is an exact differential of a single valued function.

• F = - V · U , where U is a scalar potential.

11 oo : oo : 23

[email protected]

Physicsguide

9

@Sk Jahiruddin, 2020

Newtonian Mechanics: Advanced

Example 1.2: A particle of mass m moves in the xy plane so t hat its position vector is

r

= acoswti + bsinwtj

where a, b and w are positive constants and a > b. (a) Show t hat t he particle moves in an ellipse. (b) Show t hat t he force acting on t he particle is always directed toward t he origin. (c) Find t he kinetic energy at points A and B of t he figu1~e as shown below. (d) Find t he work done by t he force field in moving the

11 oo : oo : 26 directed toward the origin. (c) Find the kinetic energy at points A and B of t he figure as shown below. (d) Find t he work done by t he force field in moving the particle from A to B. (e) Show that the total work done by the field in moving the particle once around t he ellipse is zero. (f) Show that the force field is conservative. (g) Find the potential energy at points A and B. (h) Find t he total energy of the particle and see t hat it is constant.

[email protected]

10

@Sk Jahiruddin, 2020

P hysicsguide

Newtonian l\/[echanics: Advanced

y 8

m b - i - - - - - - ~-

wt A --L-.,;;.~-----.i-,::...::....-a

X

11 oo : oo : 29

Figure 1.2:

Solution: (a) The position vector is r = xi

+ yj

=

acoswti + bsin wtj

So

x

a cos wt, y = bsin wt

=

Then

+ (y/b) = cos wt+ sin wt= 1 t he ellipse is so given by x 2 / a 2 + y 2 /62 = 1 (b) Assuming t he (x/a)

2

2

[email protected]

2

2

11

@Sk Jahiruddin, 2020

P hysicsguide

Newtonian 1/[echanics: Advanced

particle has constant mass m, the force acting on it is 2

2

dv d r d F = m dt = m dt 2 = m dt 2 [(acoswt)i

=m

2

+ (bsin wt)j ]

2

[- w a cos wti - w b sin wtj] 2

= -mw [a cos wti

+ bsin wtj ] =

2

-mw r

which shows that t h e force is always directed toward the ori•

gin.

11 oo : oo : 31 = =

2 2 m [-w acoswti - w bsinwtj ] 2 2 -mw [a cos wti + b sin wtj ] = -mw r

which shows that t he force is always directed toward t he origin. •

(c) Velocity v =

dr /dt

=

-wasin wti + wbcoswtj

Kinetic energy

~mv

2

2 2

= ~m (w a

sin

2

2 2

2

wt+ w b cos wt)

Kinetic energy at A[ where cos wt = 1, sin wt = O] = Kinetic energy at B [ where cos wt= 0, sin wt= 1] =

1

2 2 mw b

2

1 2 2 mw a 2

(d) Work done B

B

F · dr =

2 2 ( -mw r) · dr = -mw

Physicsguide

2020

Work done

=

r · dr

12

[email protected]

@Sk Jahir uddin,

B

Newtonian Mechanics: Advanced

1

2mw2 (a2 - b2)

=

1

1

2mw2a2 - 2mw2b2

= kinet ic energy at A - kinetic energy at B (e) We can find the work done by directly integrating the force also. See that at A and B , t = 0 and t = 1r /2w respec-

11 oo : oo : 34 Work done = 2mw2 (a2 - b2) = 2mw2a2 - 2 mw2b2 = kinetic energy at A - kinetic energy at B (e) We can find the work done by directly integrating the force also. See that at A and B , t = 0 and t = 1r /2w respect ively. Then t he Work done in moving the particle from A to

B B

F · dr A 1r/2w

[-mw ( a cos wti + b sin wtj )] • [-wa sin wti

+ wb cos wtj ]

2

0 1r/2w

mw

3

(a

2

-

2

b ) sinwtcoswtdt

0

1 2 2 2 2 = -mw ( a - b ) sin wt 2

1r/2w

0

1 2 2 = -mw ( a 2

2

b

-

)

Similarly for a complete circulation around t he ellipse, t goes from O tot= 21r /w. 21r / w

2

3

2

mw ( a - b ) sin wt cos wtdt

Work done 0

l 2 2 = - mw ( a

2

-

2 b ) sin wt

21r / w

2

= 0 0

You can do as you did in part (d ) expect the integral goes from one point to t he same point, say A to A and you get [email protected]

13

Physicsguide

@Sk Jahiruddin, 2020

Newtonian Mechanics: Advanced

zero. A

A

F · dr = A

2 2 ( -mw r) · dr = -mw

A

1

n

(A

A

r · dr A

1

11 oo : oo : 36 zero. A

A

F · dr =

A ( -mw

2

r ) · dr = -mw

2

r · dr A

(f) F

= -mw

2

r = -mw 2 (xi + yj )

We calculate t he curl of force •



I

V x F =

=

• I

+k

k

J

a;ax 8/8y a;az - mw 2 x -mw2 y 0

a a ( ?. ) 8y (0) - 8z -mw~y i)

ax

(- mw 2y) -

i)

8y

. a ( 2) + J 8z -mW X

-

a 8x (0)

(- mw2 x)

=0 As t he curl of t he force is zero, t he force is conservative.

(g) Since the field is conservative t here exists a pot ential V such t h at

F = -mw

2 •

x1 -

2 •

mw YJ =

[email protected]

nv - v =

8V. 8V . 8V k 1 Jax 8y az P hysicsguide

14

@Sk J ahiruddin, 2020

Newtonian Mechanics: Advanced

T hen l'"\ T T

I I'"\

?

l'"\ T T I I'"\

?

l'"\ T T / I'"\

11 oo : oo : 39 @Sk Jahiruddin, 2020

Newtonian 1/[echanics: Advanced

Then

av;ax = mw x, av/ay = mw y, av;az = 0 2

2

from which, omitting the constant, we have V =

1 2 2 -mw x

2

1 2 ? + -mw y~

2

=

1 2 ( ? -mw x~

2

?) + y- =

1 ? ? -mw~r~

2

which is t he required potential. (h) Kinetic energy at any point

Potent ial energy at any point

1 ? 2 V = -mw~r 2

1

.

= mw ( a cos wt + b s1n wt) 2

2

2

2

2

2

Add and get

The energy depends only on the constants a and b so the energy is constant.

[email protected]

@Sk Jahiruddin, 2020

15

Physicsguide

Newtonian Mechanics: Advanced

11 oo : oo : 42 15

[email protected]

@Sk Jahiruddin, 2020

P hysicsguide

Newtonian Mechanics: Advanced

Example 1.3: (a) Show t hat the force field F defined by

is a conservative force field . (b) Find a pot ential corresponding to t he force.

Solution: (a) T he force field F is conservative if and only if curlF = V x F = O •

I

a;ax

V x F =

=



J

k

8/ 8y

a/az



I

=0 (b) As we know the force field F is conservat ive if and only if there exist s a scalar function or potent ial V (x, y, z) such

[email protected]

16

P hysicsguide

11 oo : oo : 44

[email protected]

Physicsguide

16

@Sk Jahiruddin, 2020

that F

= -

Newtonian Mechanics: Advanced

grad V

- VV

=

.

F = _ vv = _ av i _ av j _ av k

ax 2 3 = (y z

-

8y az 2 3 6xz ) i + 2xyz j

2 2

+ (3xy z

-

2

6x z ) k

Hence if F is conservative we must be able to find V such that

av/ ax = 6xz

2

-

8V/8y = -2xyz

2 3

y z 3

2

2 2

av/ az = 6x z - 3xy z

To find the potential we need to integrate each component of the force and omit the common terns. Then 2 2

V = 3x z

-

2 3

xy z

+ 91 (y, z )

where 91 (y , z) is a function of y and z

constant), we have

stant)

11 oo:oo:47

stant) jahir@p hysicsguide .in

17

P hysicsguide

@Sk J ahir uddin, 2020

Newtonian Mechanics: Advanced

The trick is t o add all t hree V' s and t ake one term only once. 3x 2 z 2 and -xy 2 z 3 both t he terms have appeared twice. We take both the t erm only one t ime. So the required potent ial is V

2

2 2

= 3x z

-

2 3

xy z

+c

Collision in 2D in Center of Mass frame

2.1

Formulation of the problem

Consider two part icles of masses m 1 and m2 with velocities v 1 and v 2 respect ively. The center of mass velocity V is V =

+ m 2v 2 m1 + m2

m 1V 1

As shown in t he part (a) of t he figure below, V lies on t he line joining v 1 and v 2 The velocit ies in t he COM system are V 1c

= V1

-

V

m2

- - - ( V1 -

V 9)

11 oo : oo : 49 As shown in the part (a) of t he figure below, V lies on the line joining v 1 and v 2 The velocit ies in t he COM syst em are V1c

= V1 =

-

V

m2 - - - (v1 - v 2) m1 m2

+

[email protected]

Physicsguide

18

@Sk J ahiruddin, 2020

Newtonian Mech anics: Advanced

(a)

(b)

Figure 2.1:

and V2c

= V2 - V =

-m1 - - - (v1 - v 2) m1 m2

+

v 1c and v 2c lie back to back along the relative velocity vector v = v 1 - v 2 as shown in the part (b) of the figure.

The momenta in t he COM system are P lc

= m1 V1c m1m2 = - - - (v1 m1

+ m2

= µv P 2c

=

m2 V 2c

- v 2)

11 oo : oo : s2 P lc

= m1 V1c m1m2 = - - - (v1 - v 2) m 1 +m2 = µv

P 2c

=

= = [email protected]

@Sk J ahiruddin, 2020

m2 V 2c

-m1m2 - - - (v 1 - v 2) m1 +m2 -µv 19

Physicsguide

Newtonian Mechanics: Advanced

Hereµ= m 1m2/ (m1 + m2) is t he reduced mass of the system, t he natural unit of mass in a two-particle syst em. The total momentum in the COM system is zero. The total momentum in t he Lab frame is

and since total momentum is conserved in any collision, V is constant. We can use this result to help visualize the velocity vectors before and after t he collision. Now look at the figure below. Part (a) visualizes the path of two colliding particles before and after the collision in the lab frame. Part ( b ) shows the init ial velocities in the Lab and COM systems. All the vectors lie in the same plane, and v 1c and v 2c must be in opposite direction as t he total momentum in the C system is zero. After t he collision, as shown in part ( c ) , t he velocities in t he COM system are again in opposite direction. Part (c) also shows t he final velocities in the lab system. Note that the plane of part (c) is not necessarily the

11 oo : oo : 55 in the C system is zero. After t he collision, as shown in part ( c ) , t he velocities in the COM syst em are again in opposite direction. Part (c) also shows t he final velocities in the lab system. Note that the plane of part (c) is not necessarily the plane of part (a)

[email protected]

20

Physicsguide

@Sk Jahiruddin, 2020

Newtonian Mechanics: Advanced

~ - __ __ ,.,.. ,,, ,,/4 V2

' , ', v' I V1 c

-

- -----~

~

V2

V

,

V

JI V1 c

Y2

(a)

2c

v'2

(b)

(c)

Figure 2.2:

We will derive the mathematical formulation for the elastic collision and leave t he inelastic collision as the treatment will be complicated. Conservation of energy applied to the COM system gives, for elastic collisions, 1

2 2mi V i c

+

1

2 2 m2v2c

=

1

,2 2mi Vi c

+

1

,2 2 m2V2c

Total momentum is zero in t he C system. We therefore have

11 oo : oo : s1 2 2 m1 Vi c

+

2 2m2v 2c

=

,2 2 m1 V i c

,2 2 m2v2c

+

Total moment um is zero in t he C syst em. We therefore have

Using momentum conservation to eliminate v2c and t he energy equation gives 1 2

2 Vlc

1

= 2

m1

+

v;cfrom

m2 1 m2

or

(2.1) [email protected]

21

P hysicsguide

@Sk Jahiruddin, 2020

Newtonian l\/[echanics: Advanced

Similarly, (2.2)

In an elastic collision, the speed of each particle in the COM syst em is t he same before and after the collision; the velocity vectors simply rotate in the scattering plane. Now consider one of t he particles, m 2 is init ially at rest in the laboratory. So v 2 = 0. This case is important as it happens in many real experiment. In our earlier discussion of velocities before the collision, substitute v 2 = 0. V

m1 = ---V1 m 1 +m2

V1 c = V 1 -

V

m2 = - - - -V 1

m,

+ m')

11 oo : 01 : oo substitute v 2 = 0. V

=

m1

---V1

m 1 Vi c

=

V 2c =

V1 -

- V

+ m2 V

=

m2

- - - -V i

+ m2

m 1 m1

= - - - - -V 1 m1

+ m2

The situation is explained in t he figure below. Here the trajectories after t he collision in the COM and Lab systems are shown. You see that v ~ makes angle 01 , and v ~ makes angle 02 in Lab frame. In the COM frame the velocity vectors rotates with angle 8 which is called the scattering angle. Because t hese angles are in L , t hey are in principle [email protected]

Physicsguide

22

@Sk Jahiruddin, 2020

Newtonian 1/[echanics: Advanced

surable in the lab. The velocity diagrams can be used to relate 01 and 02 to the scattering angle 8 , as we shall do just now Lab V

---►

COM V1

~--·------► Y2c Y 1c

m1 ..._ _ __,.,■ m 2

v 1'

_____ , -----.,,___......,..,~-:= -

0

- - ~-7-: ?~ 0

---- -

Y 1c

Figure 2.3:

Y2c

11 oo : 01

: 02

Figu1·e 2.3:

2.2

Elastic scattering when target particle at rest

Now consider the elastic scattering of a particle of mass m 1 and velocity v 1 from a second part icle of mass m 2 at rest . The scattering angle 8 in t he COM syst em is unrestrict ed, but t he conservation laws impose limitations on t he laboratory angles, as we now show. The center of mass velocity has 23

[email protected]

@Sk Jahiruddin, 2020

P hysicsguide

Newtonian Mechanics: Advanced

magnit ude V =

m1v1 m1 + m2

(2 _3)

The velocity of COM is parallel t o v 1 . The initial velocities in the COM syst em are V 1c

V 2c

=

=

m2

- - - - V1

m1 +m2 m1

(2.4)

- - - - - v1

m 1 +m2

The mass m 1 is scattered t hrough angle 8 in the COM syst em as shown in figure above. This essentially means that the mass m 1 deviates an angle 8 from its original direction in the COM

11 oo : 01 : os V 2c

=

- - - - - V1

m1 +m2

The mass m 1 is scattered t hrough angle 8 in t he COM system as shown in figure above. This essentially means that the mass m 1 deviates an angle 8 from its original direction in t he COM frame. Now look at the diagram below. This is velocity diagram of the collision. Look at the figure 2.3 first . You must see t hat the initial direction of m 1 is same both in Lab and COM frame (although the magnitude of velocities are different) as well as t he velocity of COM frame which is V. 01 is t he angle between t he initial and final direction of mass m 1 in t he Lab frame. 8 is t he angle between the initial and final direction of mass m 1 in t he COM frame. As initial direction of m 1 is same both in Lab and COM frame we draw t he vector direction of V , v1, V1c along same line.

[email protected]

24

Physicsguide

@Sk Jahiruddin, 2020

Newtonian Mechanics: Advanced

;

v'1



V1

01

V

,

; ; V1 c ; ;

;

8

-- --- • V1 c

Figure 2.4:

Now it should be clear from t he velocity addition rule that

11 oo : 01 : os Figure 2.4:

Now it should be clear from t he velocity addition rule that I

.

v 1 Sln

0=

I

.

e

V i c Sln -

and v~ cos 0

= V + v~c

Hence we write

e V1c Sln tan 01 = - - - - V + v~c cos 8 I



since the scattering is elastic, v~c =

V1c •

Hence

sin 8 tan 01 = - - - - V + V1c cos 8 sin8 (V /v1c) + cos 8 V1c

From (2.3) and 2.4 we get V/v1c = m1 / m2. So, [email protected]

25

@Sk Jahiruddin, 2020

P hysicsguide

Newtonian Mechanics: Advanced

sin8 tan0 1 = - - - - - - (m1/ m2) + cos 8

(2.5)

You can easily check few important results

(2.6) and

A

11 oo : 01

: 1o

You can easily check few important results

(2.6) and

(2.7)

(ii)

(iii) If m 1 < m 2 , all scatt ering angles for m 1 in LS are possible. (iv) If m 1 = m 2 , scattering only in t he forward hemisphere (0 :::; 90°) is possible. (v) If m 1 > m 2 , t he maximum scattering angle is possible,

01max, being given by

We'll prove some important relations relating t he scattering angle of target mass m 2 in Lab and COM frame. Look at t he figure below. The scatt ering angles in Lab and COM frame has been drawn together in upper left figure (a), the COM frame alone in upper right (b) and t he vect or diagram 26

[email protected]

@Sk J ahiruddin, 2020

Physicsguide

Newtonian Mechanics: Advanced

of fin al state of target mass both in Lab and COM m 2 below

(C). COM

Lab

v;

00 : 01 : 13 (C). COM

Lab

v; 1

,:,___-==-:::-::--=::,:: :=-v.,_)_-r ~ _ 0

02

0_ 7r-0

COM fl'lmo

(a)

(b) I

/ v'

le

0

V

(c)

Figure 2.5:

We get by balancing the velocity vectors along y axis

. 02 = V2c ' Sln . 8V2' Sln And by balancing the velocity vectors along x axis

[email protected]

27

P hysicsguide

@Sk J ahiruddin, 2020

Newtonian Mechanics: Advanced

Divide first equation by second tan 02 = v I

V 2c

sin8 -

cos 8

11 oo : 01

: 1s

Divide first equation by second tan02

=

sin8 _v____ , - cos 8 V 2c

From equations (2.1 ), (2.2) , (2.3) and (2.4) we easily see t hat the magnitude of velocity of COM frame and t he velocity of m 2 after the collision in COM frame are equal, i.e

(2.8) Hence

sin8 8 tan 02 = = cot 1 - cos 82 We may write this as tan0 2 = tan

7r

8

2

2

(2.9)

(2. 10)

Thus 02 =

1

- (1r - 8 )

(2.11)

2

This is an important result. In t he special case of m 1

=

m2, we already know m1

= m2

[email protected]

8 01 = 2 28

@Sk Jahiruddin, 2020

Combining wit h (2.11) we get

Physicsguide

Newtonian Mechanics: Advanced

11 oo : 01

: 1s

@Sk Jahir uddin, 2020

Newtonian Mechanics: Advanced

Combining with (2.11) we get pi

01



+ 02 = 2 ;

when

m1 = m 2

(2.12)

This result we have proved before.

2.2.1

Kinetic energies in the Lab and COM frame

KE in lab frame

KE in COM frame

Using equation (2.4) we have

Ti c

1

m 1m 2

2

m 2

= - - - - -Vi = - - - - T i

2 m 1 + m2 m 1 + m2 This result shows t hat the init ial kinetic energy in the COM system Tc is always a fraction m 2 / ( m 1 + m 2 ) < 1 of the init ial LAB energy. For the final COM energies, we find 2

T1

[email protected]

@Sk Jahiruddin, 2020

29

P hysicsguide

Newtonian l\/[echanics: Advanced

00 : 01 : 21

29

[email protected]

@Sk Jahiruddin, 2020

P hysicsguide

Newtonian Mechanics: Advanced

and

T he ratio of KE of m 1 before and aft er collision

Using cosine law in figure 2.4

2 V 'l c

=

2 VI'

2

+V -

2VI'V COS 01

Hence

We already know and

V

We also know v~c sin 8 = v~ sin 01 Using t his we get , sin8 V1 . cSln 01

[email protected]

30

P hysicsguide

11 oo : 01

: 23

, sin8 Vlcsin 01

[email protected]

P hysicsguide

30

@Sk J ahiruddin, 2020

Newtonian Mechanics: Advanced

and using (2.5) sine cos 01 sin 8 m1 = - - = cos 8 + tan 01 m2 sin01 So t hat cos 8 + _m_i m2

Doing the substitut ions and some algebra (do yourself) we get

2

m1

cos 8 + -

m2

T his simplifies to

T{ = 1 T1

2m1m2 (l -cos e- ) 2 (m1 + m 2)

(2.13)

This is t he ratio of the KE of t he projectile part icle m 1 before and after collision. Do each step in this formulation carefully. There can be many problems which can be formed based on t his discussion. If you are thinking why did we calculate t he ration of KE of the project ile particle, I would like to mention that in many experiment we have the idea about t he ratio of the KE of t he projectile and we need t o calculat e at which

11 oo : 01

: 26

this discussion. If you are thinking why did we calculate the ration of KE of the projectile particle, I would like to mention that in many experiment we have the idea about the ratio of the KE of t he projectile and we need to calculate at which 31

[email protected]

@Sk Jahiruddin, 2020

Physicsguide

Newtonian Mechanics: Advanced

angle t he particle detector should b e placed, i.e, calculate 01 . We will see in an example shortly. Prove yourself t hat (2. 14)

Example 2.1: What is the maximum angle that 01 can attain for the case V > vie? What is 01 max for m1 and m1

>> m2

= m2?

Solution: The scattering angle 8 depends on the details of t he interaction , but in general it can assume any value. If m 1 < m 2 , it follows from equation (2.5) or t he geometric construction in part (a) of the figure below that 01 is unrestricted. However, the sit uation is quite different if m 1 > m 2 . In this case 01 is never greater than a certain angle 01,max · As figure (b) shows, the maximum value of 01 occurs when v~ is perpendicular to v ~c· Now look at the geometry of part (b) of the figure. You see

(don't forget v~r.

= V1c) If

00 : 01 : 28 c·

Now look at the geometry of part (b) of t he figure. You see

(don 't forget v~c

= V1c)

If

[email protected]

32

P hysicsguide

@Sk Jahiruddin, 2020

Newtonian l\/[echanics: Advanced

and the maximum scattering angle in Lab frame approaches zero. Increasing

e

I

----- .... ....

/

/

1/

\

I

'''

\

\

I

\

\

01,max /

E)

\

D ____ ':

V

I

/

\

I \

I \

(a)

\

''

I

....

I

/

'

-- ----

/

....

/

/

(b)

Figure 2.6:

Think on your fam iliar example, if m 1 /m2 < 1 as in (a), t his is like a flow of sm all m arbles hitting a big ball; t he marbles scatter in all directions.

On t he other h and , if a

moving big ball hits small m arble, then m 1 /m2 big ball do not deflect much. For m1 = m2

>>

1 and t he

11 oo : 01

: 31

marbles scatter in all directions.

On the other hand, if a

moving big ball hits small marble, then m 1 /m2 >> 1 and the big ball do not deflect much. For

m1

=

m2

tan 01 =

sin8 1 + cos 8-

=

tan (8 / 2)

so that

[email protected]

P hysicsguide

33

Newtonian 1/[echanics: Advanced

@Sk Jahiruddin, 2020

Example 2.2: If a particle of mass m 1 collides elastically wit h one of mass m 2 at rest, and if m 1 mass is

scattered at an angle 01 and t he mass m 2 recoils at an angle 02 with respect to t he line of motion of the incident particle, (see figure 2.3) t hen show that sin(202 + 01 ) sin01

m1 m2

Solution: This can be easily done with the help of relation between angles in COM and Lab frame. As we know from equation (2.5) tan01 =

sin8 ------(m1/m2) + cos 8

Again we know from (2 .11) 8 =

7r -

202

or sin 8 = sin(1r and

-

202) = sin 202

11 oo : 01

: 34

or sin 8 = sin(1r

-

202) = sin 202

and Using t hese tan 01 =

sin01 cos 01

sin8 -----(m1/m2) +cos 8 sin 202 (m1/m2) - cos 202

[email protected]

Physicsguide

34

@Sk Jahiruddin, 2020

Newtonian Mechanics: Advanced

Doing cross mult iplication and rearrangement 1

m sin 01 = sin 01 cos 2 + cos 01 sin 202

= sin (01 + 202)

m 2

hence

m1 m2

sin(202 + 01) sin 01

Example 2.3: P art icles of m ass m 1 elastically scat ter from part icles of m ass m 2 at rest . (a) At what LAB angle should a part icle det ector be set to detect part icles t hat lose one-third of t heir m oment um? (b) Over what range m 1/ m 2 is this possible? (c) Calculate t he scattering angle for m 1/m2 = 1

Solution: (a) In t he Lab frame given t hat , 2 m1V1 = ~m1V1

==> ~>

2 V1 = ~V1 ,

11 oo : 01

: 36 =1

(c) Calculate the scat tering angle for m 1 / m 2

Solution: (a) In t he Lab fr ame given t hat ~

, 2 m 1Vi = 3 m1 V1

Ratio of final and initial frame is 2 lm v' T'1 ? 1 1 v~2 2 lm v T1 2 1 1 ~

vr

,

Vi =

~>

2

3 V1

kinetic energy of particle m 1 in Lab 2 3

2

= 1-

2m 1m2 (m1 + m2)

2

(1

- cos e )

This equation can b e solved for cos 0, giving 2 5 (m1 + m2) cos 8 = 1 - - - - - - = 1 -y l 8m1m2 35

[email protected]

@Sk Jahiruddin,

Physicsguide

2020

Newtonian Mechanics: Advanced

2

5 m1 + m2 1 m 1m 2 We need 01 here so use equation (2.5) .

tan 01

=

✓2y - y2

sin8 -----cos 8 + m1/m2

This is the angle where the det ector should be placed. We know the values of m 1 and m 2 before the experiment , so the calculation of 01 can be done from t he knowledge of the masses. (b) Because tan 01 must be a real number , only values for m 1 /m2 where 2 - y > 0 are possible. Therefore, 2

_ 5 (m1 + m2) > 2 0 18m1m 2 -

we write t his as

11 oo : 01

: 39

m 2 where 2 - y > 0 are possible. Therefore, 2

_ 5 (m 1 + m2) > 2 0 18m1m 2

we write t his as 2

-5 Now t aking x

-5> 0

+ 26

= m 1 /m2 we write this equation as -5x

2

+ 26x -

5>0

If we make the inequality as equality, i.e, -5x 2 + 26x - 5 = 0, 1 we get x = , 5 Hence 5

1

m1 - 0

;

Hint and discussion: Put t he force in t he equation of orbit - Equation-1 .2. Get the equation

µA

l --

z2

[email protected]

µk l u----l2 1 - ~ z2

=0 P hysicsguide

42

@Sk Jahiruddin, 2020

Central Force

Substitute uk

l

l

11 oo : 01 : s1 [email protected]

42

Physicsguide

@Sk Jahiruddin, 2020

Central Force

Substitute V =

µk l u----l2 1 - ~ z2

To get 1-

µ>..

l2

V

=0

. Then examine for three cases - 1) >..
.. >

2

l

µ

and 3)

>.. = ~. Convince yourself that only in 1st case you get some kind of orbit and in case two and three the particle spirals in toward the force center.

[email protected]

43

Physicsguide

11 oo : 01 : sg [email protected]

42

Physicsguide

@Sk Jahiruddin, 2020

Central Force

Substitute V =

µk l u----l2 1 - ~ z2

To get 1-

µ>..

l2

V

=0

. Then examine for three cases - 1) >..
.. >

2

l

µ

and 3)

>.. = ~. Convince yourself that only in 1st case you get some kind of orbit and in case two and three the particle spirals in toward the force center.

[email protected]

43

Physicsguide

11 oo : 02 : 02 [email protected]

42

Physicsguide

@Sk Jahiruddin, 2020

Central Force

Substitute V =

µk l u----l2 1 - ~ z2

To get 1-

µ>..

l2

V

=0

. Then examine for three cases - 1) >..
.. >

2

l

µ

and 3)

>.. = ~. Convince yourself that only in 1st case you get some kind of orbit and in case two and three the particle spirals in toward the force center.

[email protected]

43

Physicsguide

11 oo : 02 : os [email protected]

42

Physicsguide

@Sk Jahiruddin, 2020

Central Force

Substitute V =

µk l u----l2 1 - ~ z2

To get 1-

µ>..

l2

V

=0

. Then examine for three cases - 1) >..
.. >

2

l

µ

and 3)

>.. = ~. Convince yourself that only in 1st case you get some kind of orbit and in case two and three the particle spirals in toward the force center.

[email protected]

43

Physicsguide

11 oo : 02 : 01 [email protected]

42

Physicsguide

@Sk Jahiruddin, 2020

Central Force

Substitute V =

µk l u----l2 1 - ~ z2

To get 1-

µ>..

l2

V

=0

. Then examine for three cases - 1) >..
.. >

2

l

µ

and 3)

>.. = ~. Convince yourself that only in 1st case you get some kind of orbit and in case two and three the particle spirals in toward the force center.

[email protected]

43

Physicsguide

11 oo : 02 : 1o [email protected]

42

Physicsguide

@Sk Jahiruddin, 2020

Central Force

Substitute V =

µk l u----l2 1 - ~ z2

To get 1-

µ>..

l2

V

=0

. Then examine for three cases - 1) >..
.. >

2

l

µ

and 3)

>.. = ~. Convince yourself that only in 1st case you get some kind of orbit and in case two and three the particle spirals in toward the force center.

[email protected]

43

Physicsguide

11 oo : 02 : 12 [email protected]

42

Physicsguide

@Sk Jahiruddin, 2020

Central Force

Substitute V =

µk l u----l2 1 - ~ z2

To get 1-

µ>..

l2

V

=0

. Then examine for three cases - 1) >..
.. >

2

l

µ

and 3)

>.. = ~. Convince yourself that only in 1st case you get some kind of orbit and in case two and three the particle spirals in toward the force center.

[email protected]

43

Physicsguide

11 oo : 02 : 1 s [email protected]

42

Physicsguide

@Sk Jahiruddin, 2020

Central Force

Substitute V =

µk l u----l2 1 - ~ z2

To get 1-

µ>..

l2

V

=0

. Then examine for three cases - 1) >..
.. >

2

l

µ

and 3)

>.. = ~. Convince yourself that only in 1st case you get some kind of orbit and in case two and three the particle spirals in toward the force center.

[email protected]

43

Physicsguide

11 oo : 02 : 1 a [email protected]

42

Physicsguide

@Sk Jahiruddin, 2020

Central Force

Substitute V =

µk l u----l2 1 - ~ z2

To get 1-

µ>..

l2

V

=0

. Then examine for three cases - 1) >..
.. >

2

l

µ

and 3)

>.. = ~. Convince yourself that only in 1st case you get some kind of orbit and in case two and three the particle spirals in toward the force center.

[email protected]

43

Physicsguide

11 oo : 02 : 20 [email protected]

42

Physicsguide

@Sk Jahiruddin, 2020

Central Force

Substitute V =

µk l u----l2 1 - ~ z2

To get 1-

µ>..

l2

V

=0

. Then examine for three cases - 1) >..
.. >

2

l

µ

and 3)

>.. = ~. Convince yourself that only in 1st case you get some kind of orbit and in case two and three the particle spirals in toward the force center.

[email protected]

43

Physicsguide

11 oo : 02 : 23 [email protected]

42

Physicsguide

@Sk Jahiruddin, 2020

Central Force

Substitute V =

µk l u----l2 1 - ~ z2

To get 1-

µ>..

l2

V

=0

. Then examine for three cases - 1) >..
.. >

2

l

µ

and 3)

>.. = ~. Convince yourself that only in 1st case you get some kind of orbit and in case two and three the particle spirals in toward the force center.

[email protected]

43

Physicsguide

11 oo : 02 : 2s [email protected]

42

Physicsguide

@Sk Jahiruddin, 2020

Central Force

Substitute V =

µk l u----l2 1 - ~ z2

To get 1-

µ>..

l2

V

=0

. Then examine for three cases - 1) >..
.. >

2

l

µ

and 3)

>.. = ~. Convince yourself that only in 1st case you get some kind of orbit and in case two and three the particle spirals in toward the force center.

[email protected]

43

Physicsguide

11 oo : 02 : 2a [email protected]

42

Physicsguide

@Sk Jahiruddin, 2020

Central Force

Substitute V =

µk l u----l2 1 - ~ z2

To get 1-

µ>..

l2

V

=0

. Then examine for three cases - 1) >..
.. >

2

l

µ

and 3)

>.. = ~. Convince yourself that only in 1st case you get some kind of orbit and in case two and three the particle spirals in toward the force center.

[email protected]

43

Physicsguide

11 oo : 02 : 30 [email protected]

42

Physicsguide

@Sk Jahiruddin, 2020

Central Force

Substitute V =

µk l u----l2 1 - ~ z2

To get 1-

µ>..

l2

V

=0

. Then examine for three cases - 1) >..
.. >

2

l

µ

and 3)

>.. = ~. Convince yourself that only in 1st case you get some kind of orbit and in case two and three the particle spirals in toward the force center.

[email protected]

43

Physicsguide

11 oo : 02 : 33 [email protected]

42

Physicsguide

@Sk Jahiruddin, 2020

Central Force

Substitute V =

µk l u----l2 1 - ~ z2

To get 1-

µ>..

l2

V

=0

. Then examine for three cases - 1) >..
.. >

2

l

µ

and 3)

>.. = ~. Convince yourself that only in 1st case you get some kind of orbit and in case two and three the particle spirals in toward the force center.

[email protected]

43

Physicsguide

11 oo : 02 : 3s [email protected]

42

Physicsguide

@Sk Jahiruddin, 2020

Central Force

Substitute V =

µk l u----l2 1 - ~ z2

To get 1-

µ>..

l2

V

=0

. Then examine for three cases - 1) >..
.. >

2

l

µ

and 3)

>.. = ~. Convince yourself that only in 1st case you get some kind of orbit and in case two and three the particle spirals in toward the force center.

[email protected]

43

Physicsguide

11 oo : 02 : 38 [email protected]

42

Physicsguide

@Sk Jahiruddin, 2020

Central Force

Substitute V =

µk l u----l2 1 - ~ z2

To get 1-

µ>..

l2

V

=0

. Then examine for three cases - 1) >..
.. >

2

l

µ

and 3)

>.. = ~. Convince yourself that only in 1st case you get some kind of orbit and in case two and three the particle spirals in toward the force center.

[email protected]

43

Physicsguide

11

00:02:41

[email protected]

42

Physicsguide

@Sk Jahiruddin, 2020

Central Force

Substitute V =

µk l u----l2 1 - ~ z2

To get 1-

µ>..

l2

V

=0

. Then examine for three cases - 1) >..
.. >

2

l

µ

and 3)

>.. = ~. Convince yourself that only in 1st case you get some kind of orbit and in case two and three the particle spirals in toward the force center.

[email protected]

43

Physicsguide

11 oo : oo : oo

Rigid Body: Advanced Sk J ahiruddin*

*Assistant Professor Sister Nibedita Govt. College, Kolkata Author was the topper of IIT Bombay M.Sc Physics 2009-2011 batch He ranked 007 in IIT J AM 2009 and 008 (JRF) in CSIR NET June 2011 He has been teaching CSIR NET aspirants since 2012

1 @Sk J ahiruddin, 2020

Rigid Body: Advanced

11 oo : oo : 03 1 Rigid Body: Advanced

@Sk Jahiruddin, 2020

Contents 1 Introduction

2

1.1

Velocity and Acceleration in Rotating Frame .

5

1.2

Coriolis and Centrifugal forces . . . . . . . . .

9

Moment of Inertia Tensor 2.1

3

4

MOI Tensor and Principle axis .

















Angular Momentum of Rigid Body

3.2

21

26 •

26

Different expressions of Rotational Kinet ic Energy of Rigid body . . . . . . . . . . . . . . .

30

3.1 Angular Momentum

4

16



























G eneral motion of Rig id body

32

4.1

Rotation . . .





































32

4.2

Euler 's angle .





































36

[email protected]

2

P hysicsguide

1

11 oo : oo : 06 •

jahir@physicsguide .in

































2

@Sk Jahir uddin, 2020





P hysicsguide

Rigid Body: Advanced

4.3

Angular velocity in terms of Euler 's angles . .

37

4.4

Euler Equations . . . . . . . . . . .













39

4. 5

Force free motion of symmetric top













40

4.6

MOI ellipsoid . . . . . . . . . . . .













44

11 oo :oo :oa

[email protected]

3

@Sk Jahiruddin, 2020

1

Physicsguide

Rigid Body: Advanced

Introduction

Rigid bodies, defined to b e a collection of N points const rained so that t he distance b etween the points is fixed. i.e ri -

rj

= Constant

(1.1)

for all i, j = 1, ... , N . A simple example is a dumbbell (two masses connected by a light rod) , or t he pyramid drawn in t he figure. In both cases, t he distances between t he masses is fixed .

Often we will work wit h continuous, rather than discrete, bodies simply by replacing Lti mi ➔ J drp(r ) where p(r ) is

11 oo : oo : 11

Often we will work with continuous, rather than discrete, bodies simply by replacing Lti mi ➔ J drp(r) where p(r) is the density of t he object. A rigid body has six degrees of [email protected]

Physicsguide

4

@Sk Jahiruddin, 2020

Rigid Body: Advanced

freedom 3 Translation

1.1

+ 3 Rotation

Velocity and Acceleration in Rotating Frame

Any external coordinate syst em which is fixed in space is called space fixed axis as shown in OXYZ axis in the figure. It is convenient to define a second set of axes which is fixed with the body, i.e. it moves or rotates with the body. We denote t his set of axes by Cxyz where C is t he origin of this body-fixed'' system of coordinates. Frequently (but not necessarily) C is chosen to be the centre of mass of the body z

y

C

X

O·) : < - - - - - - - Y

X

11 oo : oo : 13

o~

______ v

X

[email protected]

Physicsguide

5

@Sk Jahiruddin, 2020

Rigid Body: Advanced

Let us consider t he situation where Cxyz axes rotate about t he OZ axis where t he points O and C coincide. Our interest will be to find t he relationship between dynamical quantities in the rotating frame of reference with those in the fixed frame. Let us consider what happens to a vector when it is rotated about a fixed axis by an angle 6

~►

p

=-

q

P

-- p -- oq q

Integrating we get

F2 = Plogq Now different iate F 2 with respect to P to get

Q = log q ⇒ q =

eQ

Now the new Hamiltonian is p2 1 H = 2 + 2q2

p 2 + 1 e-2Q = E 2

Constant

11 oo : 01

: 36

Now the new Hamiltonian is

p2 1 H - -2 +2q-2

p 2

+ 1 e-2Q =

E

Constant

2

Now

8H = ( P 2 + l ) e- 2Q = 2E P· = - --

8Q

Integrating

P = 2Et + C We also get p 2+ 1

2

2Et + 2Ct +

2E 35

[email protected]

c2 + 1 E 2

P hysicsguide

@Sk Jahiruddin, 2020

Hamiltonian .Nlechanics

Q = log

2 c + 1 2 2Et + 2Ct + - -

2E

Example 4 .20: If a generating function of typ e 1 is given by

Find t he new canonical variables Q and P

Solution: We see Q p=- =q2 8F1

an d

oq

Writing t he new coordinates as function of t h e old ones gives

Q = pq

2

and

P =

~

q

11 oo : 01

: 39

Writing the new coordinates as function of t he old ones gives Q

= pq

2

P =

and

!q

Example 4.21: Prove t hat the following transformation is canonical and find a suitable generating function for it. Q

p

= ln

q

2

P =-

and

q

2

+l

p_ q

Solution: We know that for type 1 generating function

_8F_1

oq

[email protected]

= P = qeQ

Physicsguide

36

@Sk J ahiruddin, 2020

Hamiltonian Nlechanics

Which gives us

And p = _

8F1

2

= _ q eQ

2

8Q ⇒ g(Q)

=

2

dg

q 2

dQ

+l

eQ

So the generating function is given by 2

q

2

+1

eQ

2

p

q

q

2

+1

eQ

11 oo : 01

: 42

So the generating function is given by 2

q

2

4.4

+1

eQ

Canonical transformation and cyclic co-

ordinates Let us imagine t hat we find coordinates qi t hat are all cyclic. Then

Pi= o, so

are all constant. If H is conserved , then:

[email protected]

@Sk Jahiruddin, 2020

37

Physicsguide

Hamiltonian Niechanics

is also constant in t ime. In such a case the remaining equations of motion:

All coordinates are linear in t ime and the motion becomes very simple. We might imagine searching for a variable transformation to make as many coordinates as p ossible cyclic. The search of such types of coordinat es is guided by t he canonical transformation as we can not t ransform the coordinates and momentums arbitrarily and make the new coordinates cyclic.

11 oo : 01

: 44

mation to make as many coordinates as possible cyclic. The search of such types of coordinates is guided by t he canonical transformation as we can not t ransform the coordinates and momentums arbitrarily and make t he new coordinates cyclic.

5

Poission Bracket

Poisson bracket of two dynamical variables u(p , q, t) and v(p.q.t) is defined by

(5 .1) It is easy to check t hat the following fundamental Poisson bracket relations are satisfied:

There are a few other properties of note. These include: [email protected]

Physicsguide

38

@Sk Jahiruddin, 2020

Hamiltonian lVIechanics

{u, u} = 0

{u ,v} =-{v,u} {au + bv, w}

= a{ u, w} + b{ v, w}

+ {u, w }v

{uv, w} = u{v, w} { U, { V,

W}} + {V, { W, U}} + {W, { U, V}} = 0 df

{f(u) ,v} = du {u ,v } r

,. /

, -....

df

r

"

11 oo : 01

: 47 {uv, w} = u{v, w} + {u, w }v

{ U, { V,

W}}

+ {V, { W, U}} + {W, { U , V}} =

Q

df

{J (u),v } = du {u ,v } df

{u, f (v)} = dv {u, v}

(5.3) Now we need to learn one very important Theorem: A transformation Qj canonical if and only if:

=

Qj(q,p, t) and Pj

=

Pj (q ,p, t ) is

Poission bracket of any two dynamical variable is invarian under a canonical transformation , i.e

[A, B ]q,p = [A , B ]Q,P

[email protected]

39

@Sk Jahiruddin, 2020

(5.5)

Physicsguide

Hamiltonian Nlechanics

Example 5.22: Consider t he following transformation

(a) Is the transformation canonical? (b) Determine the new Hamiltonian. (c) Write down the Hamilton's equations for both H ~rn ilt"ni~nc:: ~nrl c::"l,rP f()r th P , r~ri~ hlPc::

11 oo : 01

: 49

(a) Is the transformation canonical? (b) Determine the new Hamiltonian. (c) Write down the Hamilton's equations for both Hamiltonians and solve for the variables. (d) Check that t he solutions set of both old and new Hamiltonian are same.

Solution: Calculate Poisson Bracket for t he new coordinates with respect to old coordinates

QP

= oQoP _oQoP = I [ ' ]q,p oq op op oq So t he transformation is canonical. (b) q = vf-P and p = 2Qvf-? Substituting the values of p and q in the old Hamiltonian to get t he new Hamiltonian as function of new coordinates.

~ (1 + 4Q

H'(Q, P ) = -

2

)

(c) Hamilton's equations in old coordinates •



q = p;

p = - q

[email protected]

40

@Sk Jahiruddin, 2020

Physicsguide

Hamiltonian Nlechanics

Hamilton's equation in new coordinates

. oH' Q= - = oP

1 ( 2) - - 1 + 4Q 2

p = _oH' = 4PQ oQ

11 oo : 01 : s2 . Q

=

8H' -= 8P

1 ( 2) - - 1 + 4Q 2

p == _8H' == 4PQ 8Q

We solve for old coordinates

q == A sin t + B cos t

q == p == - q;



p == - q == p ;

➔ p

== C sin t + D cos t

At t == O; q == q0 and p == p0 . Hence the solution q == qo cost + Po sin t

and p(t) == - qo sin t + Po cost

For new coordinates the equations are not very easy to solve. However we can just substitute the solut ions in old coordinates to new coordinat es with t he t ransformation given in the problem. Q t = p == - qo sin t + Po cos t () 2q 2 (q0 cos t + Po sin t ) 2

P (t ) = -q == - (qocos t+ Po sin t)

.

2

Example 5.23: Show that t he time indep endent transformation given belovv is canonical and obtain a suitable generating function of type F3 (p, Q) p == p + q2 jahir@p hysicsguide .in

@Sk Jahir uddin, 2020

+ pq2;

Q == tan- 1 q

41

P hysicsguide

Hamiltonian :Niechanics

Solution: 2

Q p = 8Q8P _ 8Q8P == l + q == l [ ' ]q,p 8q 8p 8p 8q l + q2

11 oo : 01

: 55

Solution: Q p

[ '

= 8Q8P _ 8Q8P = l

]q,p

8q 8p

l

8p 8q

2

+q + q2

= l

So the transform ation is canonical. It is given that we need to find the generating function of the type F3 (p, Q). Hence we need to write q and Pin terms of Q and p. q=tanQ;

P = p (l

+q ) +q 2

2

So

p sec Q + tan Q 2

=

2

8F3

q = tan Q = - -

8p

2

P = p sec Q

+ tan

2

8F3

Q = - DQ

From first equation F3 = -p tan Q + f 1 ( Q) and from 2nd equation F3 = -p tanQ - tanQ + Q + f 2 (p) . So t he generating function is F3

=

-p tan Q - tan Q

+Q

otherwise we can write at first

dF = 8 F3d 8 F3dQ 3 8p p + 8Q

= - tan Qdp F3 [email protected]

@Sk Jahiruddin, 2020

=

2

(p sec Q

+ tan

-p tan Q - tan Q 42

2

Q )dQ

+Q Physicsguide

Hamiltonian l\/Iechanics

Example 5.24: F ind the conditions that need to be sat-

11 oo :01 : s1 @Sk Jahiruddin , 2020

Hamiltonian Nlechanics

Example 5.24: Find t he conditions that need to be satisfied by the real constants a , /3, r', 8 so t hat t he t rans-

formation is canonical.

Solution: -y- l 'Y fJ - l _ 8Q8P 8Q8P _ [Q, P ]qp- 8 8 - 8 8 - apryq xO -aq /38q

'

q

p

p

q

= a/38q'Y+fJ - I If the Poisson Bracket is t o be 1 t hen a/38 = - 1 and 1+8- l 0

jahir @physicsguide.in

@Sk Jahiruddin, 2020

43

=

P hysicsguide

Hamiltonian Nlechanics

11 oo : 02 : oo 43

[email protected]

P hysicsguide

@Sk Jahiruddin, 2020

Hamiltonian Nlechanics

Example 5.25: A particle t hat has coordinatedependent mass mq 2 is attached to a spring t hat has spring constant k = mg. The particle's Hamiltonian is

H =

1 2 2mq--? + -2 mgq p2

(a) Derive t he HAmilton's equation of motion. (b) Show that t he coordinat e transfo1~mation z

=

1 - q2 and Pz 2

p

=-

q

is a canonical transformation. Derive t he Hamilton equat ions of motion for z and Pz and find t he general solutions for z(t) and Pz(t ) in terms of int ial values z (O) and Pz(O). (c) Let us define a new coordinate P which is equals to new Hamiltonian. p2

P = H =

z

2m

+mgz

Find another coordinate Q which is canonical to P. (Q and P that make up a pair of canonical coordinates.) Derive t he Hamilton 's equation for Q and P, and solve Q (t) and P (t) in terms of init ial values of Q and P.

[email protected]

44

P hysicsguide

11 oo : 02 : 02

44

[email protected]

Physicsguide

@Sk Jahiruddin, 2020

Hamiltonian :Niechanics

Solution: (a) 8H q= 8p •

.

p

P= -

- -·

mq2'

8H 8q

=

p2

mq 3

-mgq

(b) 8 z 8pz 8 z 8pz l [z, pz] qp= 8 8 - 8 8 =q--0= 1 ' q p p q q

So the t ransformation is canonical. Let us try to find a second type generating function F2(q, Pz) . We have studied t he convent ion of Q and P as the transformed coordinates. Here the new variables are Q = z and P = Pz· >

and

8F2

Z

, 1 2 = Bpz = 2q + f (pz)

iF

As t he partial derivative Pz2 matches with the definition of z (z = ½q2 ), we can take f'(P z) = 0, then. So

Well, now you see that the Hamiltonian becomes in t erms of new variables p2 H= + mgz

:n 2

11 oo : 02 : os Well, now you see that t he Hamiltonian becomes in terms of new variables p2 H = + mgz 2

:n

[email protected]

Physicsguide

45

@Sk Jahiruddin, 2020

Hamilt onian l\/Iechanics

This hamiltonian is exactly same hamiltonian as a particle in a gravitational field. So we get .z = Pz m

d an

. Pz = -mg

So the solution Z

1 2 = - gt 2 Pz

+

Pzo

- t m

+ Zo

= -mgt + Pz

0

(c) Given P = H =

p2 z

+mgz

2m We need to find a Q, such t hat the transformation is canonical. Lets t ry to find a generating function F2 (z, P ).

EJF2 ✓ oz = Pz =2m(P - mgz ) Hence 2~ /2 3 F2( z , P ) = - - - - (P - mgz ) + f (P) 3 mg 2 v'2 = - - - (P - mgz )312 + f (P ) 3g m 1

/2 .

,

1

In

..

.

11 oo : 02 : oa P)

F 2(z,

3 2

= - - - - (P -

mgz) 1

3 mg

+ f (P)

2~ 3 2 = - - - (P - m gz ) 1 + f (P ) 3g m 2 1 2 - (P - mgz) l m

+ J'(P )

46

[email protected]

Physicsguide

@Sk Jahiruddin, 2020

Now use P =

Hamiltonian .Nlechanics

2

J~ + mgz to get 2

Q

= _ Pz + J'( Pz + mgz) 2m

mg

- !-i~ - We could not have

If we choose f' (P) = 0.We get Q = chosen f' (P) = 0. But as we see

{Q, P} = {-

p

p2 z ,

z

mg 2m

+mgz } = l

So P and Qare canonical variables. If we have chosen J' (P) i= 0, then the Q would have been more complex. And as our task is t o choose just another Q which is canonical to P , why should we go for more complex Q when we have already have got simpler Q. So our Q is just Q

= _ Pz mg

Now we have H = P so

P=

-

oH = oQ

0

and

.

oH

Q =-= 1

oP

Hence the solut ion is p -

p ,..

n -

+ -L

n ,.

11 oo : 02 : 1o P= -

f)H

aQ

= 0 and

.Q =oH -= 1 oP

Hence t he solut ion is

P = Po

Q=t

and

+ Qo

Easy, isn't it ?? In t his problem you have seen t hat an otherwise complex problem can be made simple by use of canonical t ransformation. 47

[email protected]

P hysicsguide

@Sk J ahiruddin, 2020

6

Hamiltonian Nlechanics

Equation of motion of a dynami-

cal variable let us consider a dynamical variable (function of Q, p , t ) u u(Q, p , t) Then we have by application of partial derivative . u

=

au . OQi Qi

+

au . api Pi

+

=

au at

Using Hamilton 's equation of mot ion we get .

au a H

au aH

OU

aQi api

api oq,i

at

u= - - - -

-+-

In t erms of Poission Bracket we writ e t he above equation more concisely as

(6. 1) • If u

=

• If u

= Pi, t hen

Qi t hen

11 oo : 02 : 13 • If u

= qi then

• If u

= Pi, then . api aH Pi = {Pi, H} + at = - aqi

These two equations are Hamilton 's equation of motion. • Now if u

=

H then

if = {H H}

'

aH + at

=

aH at

48

[email protected]

Physicsguide

@Sk Jahiruddin, 2020

Hamiltonian :Niechanics

• Next consider what we can say if a quantity u is conserved. Here:

u= 0=

au {u H } + ' at

So we can say if

au at

=

0

and {u, H}

=

0⇒u

is conserved

Another fact, is that if u and v are conserved t hen so is {u , v}. This could potentially provide a way to compute a new constant of motion. To prove it, first consider the sp ecial case where:

8u =av= O at at

~~>

{u, H} = {v, H} = O

then using the J acobi identity we have:

{H, {u,v}}

=

- {u,{v, H }} - {v, {H ,u}}

=

- {u, O} - {v, O}

=

0

11 oo : 02 : 1 s then using t he Jacobi identity we have:

{H, {u,v} } = -{u, {v, H }}-{v,{ H ,u} } = - {u,O}- {v, O}

=

For t he most general case we have

du au dt = {u, H} + at = O

au {u, H} = - at

And dv

dt

= { v , H}

av

+ at

av

{v, H} =-at

O

=

49

[email protected]

Physicsguide

@Sk J ahiruddin, 2020

Hamiltonian lVIechanics

So we get

+ {v, { H , u} }

{{u, v }, H} = {u, {v, H}}

==-

av

u, a

t

+

au v, at

a

at {u , V}

Now the equation 6.1 for tha variable {u, v} is

a

d

dt {u, v} = {{u, v}, H} + at {u, v} Hence we get d

dt {u, v} = 0

7

Infinitesimal Canonical Transfor-

m rition

0

11 oo :02 : 1a dt {u, v} = 0

7 Infinitesimal Canonical Transformation Let us now consider t he generat ing function:

F2( q, P , t) = qi~

+ EG( q, P , t)

where F2 = qi~ is an identity transformation, and El t+ /3

11 oo : 02 : s4 as /3 = - = - t±m aa

dq J2ma - (mwq) 2

⇒ t+/3

mw 2 2a q

1 = ±-arcsin w

integrating we get

q=

±

-

2a

mw

. s1n(w(t + /3) ) 2

so /3 is relat ed to t he phase. Next we consider p and use t his result t o obtain: P=

as aq =

±J2ma - (mwq)

2

=

±-hma cos(w(t + fJ ))

63

[email protected]

P hysicsguide

@Sk Jahiruddin, 2020

Hamiltonian :Niechanics

So we get our familiar solut ion of p and q. To complete t he story, t he constants a and /3 must be connected wit h t he init ial conditions q0 and p0 at t ime t = 0. By squaring p and q and using t = 0 we get

Now dividing p and q at t

= 0 we get

tan /3

= mw qo Po

The choice q0 = 0 and hence /3 = 0 corresponds t o st art ing t he mot ion with t he oscillator at its equilibrium posit ion q = 0 .

9.2

Separation of variable and H-J method

11 oo :02 : s1 The choice q0 = 0 and hence /3 = 0 corresponds to starting the motion with the oscillator at its equilibrium position q = 0 .

9.2

Separation of variable and H-J method

Separation of variables is t he main technique to solve t he H-J equation. In particular, for a time independent H where

if = 8H = 0

(9. 13)

at

we can always separate t ime by taking: (9. 14)

where a 1 has been chosen as the separation constant, then plugging t his into the t ime dependent H-J equation yields 64

[email protected]

@Sk Jahiruddin, 2020

Physicsguide

Hamilt onian l\/Iechanics

(just as in our Harmonic Oscillator example) :

This result is referred to as the time independent HamiltonJacobi equation. since H = 0 , His conserved, and equal to a constant a 1 . If H = E then this constant is energy, a 1 = E. The function W is called Hamilton's characteristic function. Now let's try to solve the central force Kepler problem by H-J method. •

Example 9.28: As an extended example, let us consider the Kepler problem of two masses m 1 and m 2 in a

11 oo : 02 : sg method.

Example 9 .28: As an extended example, let us consider t he Kepler problem of two masses m 1 and m 2 in a central potential (with t he CM coordinate R Lagrangian is:

L =

!mr 2

2

-

V (r)

1

where

m

=

0). The

1

1

m1

m2

- +-

and here m is t he reduced m ass. Any V (r) conserves L = r x p , so the mot ion of r and p is in a plane perpendicular to L. The coordinates in t he plane can be taken as (r, 'ljJ), so:

L= m 2

r2 + r 2 ~ 2

[email protected]

-

V(r )

Physicsguide

65

@Sk Jahiruddin, 2020

Hamiltonian .Nlechanics

'lj; is a cyclic coordinate 2 .

P 'ljJ

= mr 'ljJ

is a constant

Now check that we can write the hamiltonian as H =

+ V (r)

1 2m

As 'lj; is cyclic, then

P 'ljJ

O'.'ljJ

= a1 =

E

is constant. Using:

then the t ime independent H-J equation is:

(9 .15)

11 oo : 03 : 02 As 'ljJ is cyclic, then Pv; _ av; is constant. Using:

then the t ime independent H-J equat ion is: 1 2m

After simplification a,2

2m (a.1 - V(r)) -

t r

Integration gives

Transformation equations are dr

[email protected]

66

@Sk J ahiruddin , 2020

Physicsguide

Hamiltonian Nlechanics

and

9.3

Action-Angle Variables

For many problems, we may not be able to solve analytically for the exact motion or for orbital equations, but we can still characterize the motion. For periodic systems we can

11 oo : 03 : os 9.3

Action-Angle Variables

For many problems, we may not be able to solve analytically for t he exact motion or for orbital equations, but we can still characterize t he motion. For periodic systems we can find t he frequency and t ime period by exploit ing action-angle variables. For periodic motion, we replace P = a 1 by t he action variable J = pdq T he coordinate conjugate to J is t he angle variable

aw w=-8J

After some derivatons and arguments we get v( J) is the frequecy of the periodic motion.

8H (J ) V=--8J

67

[email protected]

Physicsguide

@Sk Jahiruddin, 2020

Hamiltonian :Niechanics

Example 9.29: Consider a general Hamiltonian

H =

p2 2m

+ a xiv

Find the time period of a particle in t his Hamiltonian using action angle variable.

11 oo : 03 : 01 =

H

+a

2m

x iv

Find t he t ime period of a par t icle in t his Hamiltonian using action angle variable.

Solution: As the potent ial is symmetric then the m agnitudes of t he turning points are same

E l/v Xturning

=± a ✓2m (E - axv)dx 0

a

= 2 2m E Now we h ave E

= ax2.

1 - -xvdx

E

o Substituite Q

z = -x

V

-E

so

x=

zE

l/ v

'

Then

J

= 2J2m E

E a

dx =



1/ v

E a

l

1

V

1/ v

l

- zv1 _ 1d z V

zt- 1J1 -

z dz

0

68

[email protected]

P hysicsguide

@Sk J ahiruddin, 2020

Hamiltonian lVIechanics

E l/v l I'(l /v)f (3/2)

a

v

E l/v I'

a

r (t + ~) (t + 1) ½v0f r (~+ ~)

11 oo : 03 : 1o 1 I'(l / v)I'(3/ 2)

=2 2mE

(e + ~) l / v I' (e + 1) ½J7r I'

V

= 2 2mE

E

r (~ + ~) E l / v r (e + 1) = ✓21rmE a r (e + ~) E t+~ = J al f v r (e + ~) a

~ r (t+ 1) P ut t he constants inside anot her constant

and writ e Taking derivative with resp ect to J

=

E t +~ - l 8E 8J

k

T hen the frequency

BE 1 1 V=-=k/E v-2 8J Then the T ime period

T

1

= - =

1

1

E v- 2/k

V

[email protected]

69

Physicsguide

@Sk Jahiruddin, 2020

Hamiltonian Nlechanics

Hence the time period proportional to 1

T ex E v-

1 2

11 oo : 03 : 12 Hence t he time p eriod proportional to 1

T ex E v-

1 2

Example 9.30: Let us consider the Harmonic oscillator problem again

1 2 2 2 2 H = m (p + m w q ) = E = a = constant 2

Now J is " ._,

._,

where a is the constant total energy and w If we substitute q =

,

to

2

= k/m.

/2

a 2 sin 0 Then t he integral reduces mw 27T

J= 2a cos2 0d0 w 0 Limits should b e over one complet e cycle of q.

The integral after integration J= 21ra w

Solving for a

a

[email protected]

H = Jw 21r 70

@Sk Jahiruddin, 2020

The frequency of oscillation is t hen

P hysicsguide

Hamiltonian Nlechanics

11 oo : 03 : 1 s 70

[email protected]

Physicsguide

@Sk J ahiruddin, 2020

Hamiltonian :Niechanics

The frequency of oscillation is then 8H w 1 -=v=-=8J 2n 2n

k m

Example 9.31: A particle of mass m moves in one dimension subject t o t he potent ial

k U(x) = -sin-2-(x-/ a- ) (a) Obtain an integral expression for Hamilton's characteristic function. (b) Under what conditions may action-angle variables be used? (c) Assuming that action-angle variables are perm1ssible, det ermine the frequency of oscillation by the action-angle method. (d) Check your result for the oscillation frequency in the limit of small oscillations. •

Solution: ( a) As the total energy is conserved. H

P2

=-

2m

k

+ - -2 - - = E = sin (x/ a)

constant

The motion is therefore between the t urning points. [email protected]

@Sk Jahiruddin, 2020

71

Physicsguide

Hamiltonian :Niechanics

11 oo : 03 : 11 The motion is therefore between t he t urning points. [email protected]

71

Physicsguide

@Sk J ahiruddin, 2020

Hamiltonian lVIechanics

ing points are t hose where total energy becomes equal with potential energy x_(E) = n1ra + a sinx+(E) = (n

+ l )1ra -

1

k/ E

asin-

1

k/ E

where n is any integer. The Hamilton J acobi equation is then 1 2m

dW

dx

2

k

+ - 2- - =Q sin (x/ a)

Which We may t hen write X

W(x, E ) = v2m,

dx'

E-

2

k

sin (x' / a)

X- (E)

The lower limit may be left as unspecified ; this only changes t he result by a constant. (b) We need t hat the motion is bounded. In our case, x _(E) < x < x+(E ) (c) The action is

E-

dx'

2

k

sin (x' / a)

This integration is tough. However if we substit uite

cos(x/a) = [email protected]

k l 1 - -cos -u

E

72

2

Physicsguide

11 oo : 03 : 20 This integration is tough. However if we substituite

k 1 1 - -cos -u

cos(x/a) =

E

[email protected]

2

72

Physicsguide

@Sk Jahiruddin, 2020

Hamiltonian Nlechanics

Then J becomes 21r

J = aJ2mE

1 - cos u du----

E+k - COSU E -k

0

This integral can be solved by method of contour integration by using techniques of complex analysis. However t he standard integral table may be used 21r

I = 0

du 1 21r b - cosu

1

Jb 2 -

Using t his integral we got, with b = (E J

1

+ k)/(E -

k), 2

J

= ~a( vfE - v'k) , E = ~ +v'k a

Note t hat the minimum energy is frequency is given by

v( J) =

8E J -= -+ 8J ma 2

Emin

= k. The oscillation

2k ma2

2E ma 2

2

(d) With U(x) = k/ sin (x/a) we have

U'(x) = _ 2k . cos(x/a) a

3

sin (x/a) 4

U"(x) = 2k . sin (x/a) a2

+ 3sin

2

2

(x/a) cos (x/a) sin6 (x/ a)

11 oo : 03 : 23 U'(x) = _ 2k . cos(x/a) a

sin 3 (x/a) 4

U"(x) = 2k . sin (x/ a)

+ 3sin

a2

[email protected]

2

2

(x/a) cos (x/a) 6 sin (x/ a)

73

Physicsguide

@Sk Jahiruddin, 2020

Hamiltonian Nlechanics

Setting U' (x*) = 0 we obtain x* = (n + ~) 1ra, where n is an integer . At any of these equilibria, U'' (x*) = 2k/ a 2 . Therefore, the frequency of small oscillations is

Ws.o.

=

U'' (x*)

which agrees t he result of (c)

m

2k ma2

00: 03 : 25

[email protected]

73

Physicsguide

@Sk Jahiruddin, 2020

Hamiltonian l\/Iechanics

Setting U' (x*) = 0 we obtain x* = (n + ½ ) 1ra, where n is an integer . At any of these equilibria, U'' (x*) = 2k/a2 . Therefore, t he frequency of small oscillations is

W s.o.

=

U'' (x*) m

2k ma2

which agrees the result of (c)

[email protected]

74

P hysicsguide

11 oo : 03 : 2a [email protected]

73

Physicsguide

@Sk Jahiruddin, 2020

Hamiltonian l\/Iechanics

Setting U' (x*) = 0 we obtain x* = (n + ½ ) 1ra, where n is an integer . At any of these equilibria, U'' (x*) = 2k/a2 . Therefore, t he frequency of small oscillations is

W s.o.

=

U'' (x*) m

2k ma2

which agrees the result of (c)

[email protected]

74

P hysicsguide

11 oo : 03 : 30 [email protected]

73

Physicsguide

@Sk Jahiruddin, 2020

Hamiltonian l\/Iechanics

Setting U' (x*) = 0 we obtain x* = (n + ½ ) 1ra, where n is an integer . At any of these equilibria, U'' (x*) = 2k/a2 . Therefore, t he frequency of small oscillations is

W s.o.

=

U'' (x*) m

2k ma2

which agrees the result of (c)

[email protected]

74

P hysicsguide

11 oo : oo : oo

Small Oscillation Sk J ahiruddin*

*Assistant Professor Sister Nibedita Govt. College, Kolkata Author was the topper of IIT Bombay M.Sc Physics 2009-2011 batch He ranked 007 in IIT J AM 2009 and 008 (JRF) in CSIR NET June 2011 He has been teaching CSIR NET aspirants since 2012

1 @Sk J ahiruddin, 2020

Small Oscillat ion

11 oo : oo : 03 1 @Sk Jahir uddin , 2020

Small Oscillation

Contents 1 Stability and equilibrium

3

1.1

Bounded and Unbounded motion









4

1.2

Integrat ion of equation of mot ion in lD .







5







2 Small oscillation 2.1

8

Small oscillation in more t han one dimension .

10

3 General formulation of Small Oscillation

13

3.1

Matrix formulation of small oscillation problem

17

3.2

Rigorous solut ion of Linea1· t riatomic molecule

25

3. 3

Making t he non quadratic potent ial quadrat ic

32

jahir @p hysicsguide.in

2

P hysicsguide

n

11

r--,.

0

11

, •

11 oo : oo : os

jahir@physicsguide .in

2

P hysicsguide

@Sk Jahir uddin, 2020

1

Small Oscillat ion

Stability and equilibrium

The result F = - dU/ dx is useful not only for computing the force but also for visualizing the st ability of a system from the potent ial energy plot. Suppose t here is a force on the part icle is F = - dU/ dx , and the system is in equilibrium where there is no force i.e. dU / dx = 0. If t his occurs at a minimum of U it is a st able equilibrium whereas if it is at a maximum of U, the equilibrium is unstable. Say, dU/ dx = 0 occurs at some point x 0 . To t est for stability we must determine whether U has a minimum or a maximum at x 0 . One needs to examine d2 U/ dx 2 at x 0 . Potential Energy, U

Unstable E ulllbrium Neutral equilibrium Position, x

Stable Equilibrium

11 oo :oo :oa

[email protected]

Physicsguide

3

@Sk Jahiruddin, 2020

Small Oscillation

-+ Stable;

-+ Unstable; -+ Neutral

If t he second derivative is posit ive, t he equilibrium is stable; if it is negative, t he system is unstable. If d2 U/ dx 2 = 0, one must look at higher derivatives. If all derivatives vanish so t hat U is const ant in a region about x 0 , t he system is said to be in a condition of neutral.

1.1

Bounded and Unbounded motion

In Mechanics we generally divides the motion in two broad categories. The Bounded system, where energy is negative and t he Unbounded system where the energy is positive. Bounded system may be periodic or non periodic. In this section we will discuss the periodic bounded system t heir oscillation and frequencies.

since kinet ic energy K = E - U can never be negative, the

11 oo : oo : 1o cillation and frequencies.

since kinetic energy K = E - U can never be negative, the motion of the syst em is constrained to regions where U < E.

[email protected]

Physicsguide

4

@Sk Jahiruddin, 2020

Small Oscillation

The points where E = V (K.E = 0) are called t urning points. The following figure is an illustration of bounded motion of one potent ial wit h two different energy and turning points. Potential energy

Total energy E1

E1 1 - - - - - - + - - - - r , - - - - - - - L - - - - - - - + -

Total energy E2 E2

1 -- - - . - -- \.----- -r t --

--'-- - - --,tL-- - . -

Kinetic energy at xA

Potential energy at xA X

1.2

"'

Integration of equation of motion in 1D

1 E=-m 2

dx dt

2

dx

+ V(x)

dt

2

- [E - V(x) ] m

(1.1)

Hence

dx

11 oo : oo : 13 1

E = -m 2

dx dt

2

dx

+ V(x)

2

- [E - V( x) ]

dt

m

(1.1)

Hence dt

=

dx

(1.2)

-----;:=================-

~ [E - V (x)]]

So the t irne p eriod as a function of energy is twice the tirne [email protected]

5

P hysicsguide

@Sk Jahiruddin, 2020

Small Oscillation

t he particle take to go from T(E)

=

x1

to

x2

dx

m

2

2

x 1 (E)

✓[E - V (x)]

(1.3)

Example 1.1: Example: Consider t he potential shown in the figure. Find the p eriod of oscillation

E-- 3U0/2

- - - - U=U0 - - - - - - - U=O +(-----+) ....

b b Time period is twice t he tirne taken to reach from starting point to end point

T= 2

b

O

dx

2b

dx

--+2 1

/2E

b

1

j2(E- Uo)

11 oo : oo : 16 Time period is twice t he t ime taken to reach from start ing point t o end point b

2b

dx

T = 2

2E

0

+ 2

m 26 = 2b 3Uo +

m

m Uo m Uo

1

v'3

[email protected]

2(E-Uo)

b

m

= 2b 1 +

dx

P hysicsguide

6

@Sk J ahiruddin, 2020

Small Oscillation

Example 1. 2 : Determine t he period of oscillation , as a

function of the energy, when a part icle of m ass m moves in fields for which t he potent ial energy is (b) U

2

= - Uo / cosh ax,

- Uo < E < 0,

(a) is already d one in the action angle variable. Here we will get sam e ans by doing similar integration. (E / A)l /n

T = 2

(2rri)

J (E - A xn)

0

= 2

2m E



dx

1/n

E A

1 0

dy

J(l - yn)

By t he substit ut ion yn = u t he integral is reduced to a beta function, which can be expressed in t erms of gamma funct ions:

E -- ·

E

-

A

l /n

f (l / n)

r (~ + 1/n)

(

11 oo :oo :1a By t he substitution yn = u t he integral is reduced to a beta function , which can be expressed in terms of gamma functions:

E

21rm

-- ·

E

1/ n

f (l /n)

-

r (~ + 1/n)

A

Do (b) and (c) similarly. The ans are

(b) T = (1r/a) ✓(2m/ E l), (c) T= (1r/a) ✓[2m/(E+Uo)]

[email protected]

Physicsguide

7

@Sk Jahiruddin, 2020

2

Small Oscillation

Small oscillation

Nearly every bound system oscillates like a harmonic oscillator if it is slightly perturbed from its equilibrium position. >.

0) ~

a>

C

w

r

--------r-----I

\ \

1

/

Parabola

1

I

\ I

I I

/ I

I

//

I I

. -.: .

/..

Look at the figure. Expand U (r) about r 0 , the position of the potential minimum.

dU

U (r) = U (r n) + (r - rn) .

+

l

-=- (r - rn)

2

2

dU . "

+ ···

11 oo : oo : 21 Look at the figure. Expand U(r) about r 0 , t he position of the potential minimum.

dU U(r) = U (ro) + (r - ro) dr

+

2 d U 2 2 (r - ro) dr2

1

ro

+ ... ro

However, since U is a minimum at r0 , (dU/dr ) ro = 0. Furthermore, for sufficiently small displacements. we can neglect the terms beyond the third in the power series. In this case, kx2 U(x) = constant + 2 This is the potential energy of a harn1onic oscillator, where

d2 U k = dr 2

the effective spring constant

;

ro

[email protected]

Physicsguide

8

@Sk J ahiruddin, 2020

Small Oscillation

Frequency of small oscillation is

W =

k

1 d2 U

m

m dr 2

Example 2.3: The time period of a particle of mass m , undergoing small oscillations around x = 0 , in the

potential V =

(a)

1r ,

(C) 21r i\

mL 2

-

mL 2 (b) 21r , 2Vo

Vo mL l /,..

Vo cosh (f) , is

2

[NET Dec 2018]

11 oo : oo : 23 (a)

m L2 7r

(b) 21r

Vo

(c) 21r

mL

m L2 2V0

2

(d) 21r

Vo

Solution: F irst make

2mL

2

Vo

!~ = 0 to find the equlibrium point . dV

Vo I I Vo cosh(L) = 2 ex L + e- x L X

Vo ex/L - e- x /L

➔-

dx



So dV = O

x= O

dx

Now the double derivat ive

Vr _O

2£2 e

x/L

[email protected]

+

e

- x/L

=

1 £ 2V ;

P hysicsguide

9

Small Oscillat ion

@Sk Jahiruddin, 2020

Vo dx2 x=O £ 2 So t he frequency of sm all oscillation is

w=

l d2 V

m dx2

Vo x=O

m£ 2

Hence the time p eriod

T = 21r/w = 21r

2.1

m L2 Vo

Small oscillation in more than one di-

11 oo : oo : 26 T

2.1

21r/w

=

=

21r

m L2

Vo

Small oscillation in more than one di•

mens1on You have already know the theory in small oscillation in 2 and 3 dimension, I'll not repeat t hat . You can proceed in both way. Either calculate the equation of motion and solve directly or just calculate the V and T matrix and need to find t he frequencies of small oscillation by getting

we will proceed wit h interesting examples. As the first and most simple system of vibrating mass points, we consider the free vibration of two mass points, fixed [email protected]

10

P hysicsguide

@Sk Jahiruddin, 2020

Small Oscillation

to two walls by springs of equal spring constant, as is shown in t he figure

I I

-

•, X

.

I

I

,+I

I

- •t x 2

:-

' I I I

-kx l

11 oo : oo : 29 I

. X I , I

-· I

I I I

'

-kx l

The two mass points shall have equal masses. The displacements from the rest positions are denoted by x 1 and x 2 , respectively. We consider only vibrations along t he line connecting the mass points. When displacing the mass 1 from the rest posit ion, there acts the force -kx 1 by t he spring fixed to t he wall, and the force +k (x 2 - x 1 ) by the spring connecting the two mass points. Thus, the mass 1 obeys t he equation of motion

For the second mass we get

[email protected]

@Sk Jahiruddin, 2020

11

Physicsguide

Small Oscillation

\"Ne first determine the possible frequencies of common vibrat ion of t he two particles. The frequencies that are equal for all particles are called eigenfrequencies. The related vibrational states are called eigen- or normal vibrations. These definitions are generalized for a N -particle system. We take the solution i

P

h"t h n ~rt.ir 1P~ ~ h ~ll ,,ihr~tP u rit.h t hP ~~.mP frPn 11 Pnr,, r,,

11 oo : oo : 31 st at es are called eigen- or normal vibrations. These definit ions are generalized for a N -part icle system. We take the solut ion

i.e., both particles shall vibrate with the same frequency w . The specific typ e of the solution, be it a sine or cosine function or a superposition of both, is not essential. We would always get the same condition for t he frequenty, as can be seen from the following calculation. Put t he solut ion in the equation of motion 2

+ 2k) -

=0 2 - A1k + A2 ( -mw + 2k) = 0 A1 ( -mw

A2k

(2.1)

The system of equations has nontrivial solut ions for the amplit udes only if the det erminant of coefficients D vanishes: -mw 2

+ 2k

- k

-k -mw

2

+ 2k

= ( -mw

2

+ 2k)

2 -

k

2

=

0

We t hus obtain an equation for det ermining the frequencies: 4

w

-

k 2 k2 4-w + 3- 2 = 0

m

m

12

[email protected]

P hysicsguide

@Sk Jahiruddin, 2020

Small Oscillation

The posit ive solut ions of t he equation are t he frequencies 3k

k

m

m

These frequencies are called eigenfrequencies of t he system; t he corresponding vibrations are called eigenvibrations or nor-

11 oo : oo : 34 3k

k

m

m

These frequencies are called eigenfrequencies of t he syst em; t he corresponding vibrat ions are called eigenvibrations or normal vibrations. To get an idea about the t ype of t he normal vibrations, we insert the eigenfrequency into t he equat ion (2.1) For the amplitudes, we find

A1 =-A2

for

w1

=

3k m

and

k m

3 General formulation of Small Oscillation Consider a syst em wit h { qi} as the generalized coordinat es. Since the system is conservative, V (q1 , q2 , ... , qN) . Lagrange defined equilibrium as a configuration in which all generalized . h , 1.e. . 8V 0 forces van1s Bqi = .

a2v

- - > 0 Signifies Stable equilibrium 8qi8qj

[email protected]

13

Physicsguide

@Sk Jahiruddin, 2020

and

Small Oscillation

a2 v

- - - < 0 Signifies unstable equilibrium 8qi8qj

Without loss of generality, let us shift t he equilibrium posiJ



J

j

,





I

A\

Tf'

, 1



11 oo : oo : 36 and

a2 v

- - - < 0 Signifies unstable equilibrium 8qi8qj

Without loss of generality, let us shift the equilibrium position to the origin (q 1 = q2 = ... , = qN = 0). If the syst em is disturbed to a configuration {qi}, we can write,

V (q1 ' q2 '

. . .)

= V (0' 0' . . .) +

L 1,

+

Higher order terms

where t he partial derivatives are evaluated at the p osition of equilibrium and all higher order terms'' which involve third order or higher corrections are neglected. If t he potential energy is measured from its minimum value, we choose V (O, 0, ... ) = 0. Along with

av

= O

8qi

0

The leading term in the change in potential energy is then

[email protected]

14

@Sk J ahiruddin, 2020

for stable equilibrium. Let us write

Physicsguide

Small Oscillation

11 oo : oo : 39 @Sk Jahiruddin, 2020

Small Oscillation

for stable equilibrium. Let us write

so that 'l ,J

Now, the kinetic energy in general a quadratic in generalised velocities, and can be written as

The coefficients m i j are in general functions of the coordinates Qk, but may be expanded in Taylor series about the equilibrium position ffiij ( Ql ' Q2' .. . )

=

ffiij ( 0'

0' ... ) +

L k

0

It t urns out t hat t he quantities ;"i.i and t he higher order qk 0 derivatives are negligibly small so that t he coefficients m i j s can be essentially treated as constants having t he same values as they would have in t he equilibrium position . Thus around the equilibrium posit ion, 8

[email protected]

@Sk Jahiruddin, 2020

15

Physicsguide

Small Oscillation

11 oo : oo : 42 15

[email protected]

Physicsguide

@Sk Jahiruddin, 2020

Small Oscillation

Then the Lagrangian has the following structure: £ = T - V =

1

2 I: (T;jl/;C/j -

V;jq;qj)

i,J

Now if you solve the Lagrange's equation of motion

for the given Lagrangian you will get (you need not to do the general case by hand, you will do in the examples)

.

1,

The equation is a homogeneous equation in Ai s and t he condition for existence of t he solution is

This is equation is called t he secular equation. We are only interested in real roots of the above equation. w~s det ermined from t his equation are known as characteristic frequencies or eigenfrequencies.

[email protected]

16

P hysicsguide

11 oo : oo : 44

[email protected]

@Sk Jahiruddin,

3.1

Physicsguide

16

2020

Small Oscillation

Matrix formulation of small oscillation

problem Using symmetry properties of V and T (symmetric matrix) we can write the secular equation as

.

1,

where A = w 2 . Define a column vector

A= • • •

The matrices V and T are given by

V=

½1

V12

• • •

Vi N

T11

T12

• • •

T 1N

½1

V22



• •

V2N

T21

T22



• •

T 2N





• •

• • •

• • •

V N1







V N2



• • •

• •



VNN

andT=



T N 1 T N2

• • •

• • •

• • •

TNN

So t he matrix equation can b e written as a m atrix equation

(3.1) Not e that this equation is not in the form of an eigenvalue equation as VA = AA but VA = ATA. If T is invertible, (in

11 oo:oo:47 (3.1) Note that this equation is not in the form of an eigenvalue equation as VA = AA but VA = ATA. If T is invertible, (in [email protected]

17

P hysicsguide

@Sk J ahiruddin, 2020

Small Oscillation

general it is) one can get an eigenvalue equation w 2 I A.

r - VA 1

=

since we have N homogeneous equations, we have N modes, 2 i.e. N solutions for w . Let us denote the k -th mode frequency by = Ak. Let the vector A corresponding to this mode be written as

wi







We have

Taking conjugate of this equation and changing the index k to i, we get ~

where we have used A to denote t he t ranspose of t he matrix ~ A .. From V A k = AkT A k we get by multiplying with A k

~

We have V A k = AkT A k. Multiply by Ai in LHS. You get ~

~

A ,iV A k = AkAiT A k

11 oo : oo : 49 ~

\'lve have V Ak = AkT Ak. 1\1:ultiply by Ai in LHS. You get ~

~

AiV Ak = AkAiT Ak

[email protected]

Physicsguide

18

@Sk J ahiruddin, 2020

Small Oscillation ~

~

We also have AiV = AiAiT. Multiply by Ak in RHS . You get ~

~

AiV A k = AiAiT A k From the last two equations ~

(Ak - Ai) AiT Ak = 0 Thus, if the eigenvalues are non-degenerate, i.e. if Ai we get the orthogonality condition

i=

Ak,

Note that this is different from the orthogonality condition on eigen vectors for a regular eigenvalue equation. Since (3. 1) does not uniquely determine A , we define normalization condition as

(3.2) The normalization of the eigenvectors are not necessary unless specifically stated in the problem. We can get the idea of normal coordinates without normalization as we see in the examples. Fundamentally speaking vve have a set of coordinate 1J. We are trying to achieve another set of coordinates

11 oo : oo : s2 e e1genvec ors are no necessary unless specifically stated in the problem . We can get t he idea of normal coordinates without normalization as we see in the examples. Fundamentally speaking we have a set of coordinate rJ . We are trying to achieve another set of coordinates ( , which are related by ~>

[email protected]

r~ _-

A-1,n •I

Physicsguide

19

@Sk J ahiruddin, 2020

Small Oscillation

In the form of ( t he equations look like ..

(i

2

+w ( = 0

The set of ( coordinat es are called normal coordinates and these vibrations in the coordinate of ( are called normal mode. Example 3.4: Consider two masses m 1 = 2m and m 2 = m connected by t hree springs, as shown in the following figure. Find t he frequency of oscillation and normal

modes. k = 2k 3

Solution: The Lagrangian of t he system is £

=

1 ·2 1 ·2 1 2 1 2 1 2 m1x1 + m2x 2 - k1 x 1 - k2 (x2 - x 1) - k3x2 2 2 2 2 2

Define T and V matrix

11 oo : oo : 55 Solut ion: T he Lagrangian of t he system is £

=

1 ·2 1 ·2 m1x 1 + m2x 2 2 2

-

1 2 1 2 1 2 k1x1 - k2 (x2 - x1) - k3x 2 2 2 2

Define T and V m atrix

m1 0 0 m2

5k -k -k 3k



'

Non trivial solution exist if

[email protected]

Physicsguide

20

@Sk Jahiruddin, 2020

Small Oscillation

which gives

5k - 2mw 2 -k 3k - mw 2 -k

=0

The secular equation has t he solution w2 = 7 k /2m and 2k / m. The normal modes are now found by substit ut ing t he values of w's in to the secular equation and evaluation of eigenvectors

Writing A =

7k 2m

'

we get ' for w2

=

7 k 2m

5k -k -k 3k

2m 0 O m

For t his normal mode we have A 2 unnormalized

1

-2

= - 2A 1 so t hat we get

11 oo :oo : s1 - 2A 1 so t hat we get

For t his normal mode we have A 2 unnormalized

1

-2 For w2 =

';:,,

a similar calculation gives

1

A=

1

So we get the unnormalised transformation matrix 1

A=

1

-2 1 21

[email protected]

P hysicsguide

@Sk Jahiruddin, 2020

Small Oscillation

We have taken the coordinates as x 1 and x 2 . The normal coordinates ;- -_ A-1-n ~

~>

•I

1 -1

2 If we write w1 =

?k 2m

and w 2 =

1 ~, the general solution is

where a 1, a 2 , b1, b2 are the constants which to b e determined by the initial conditions. The solution can also be written by in t erms of old (x 1, x 2) coordinates 1 1

cosw2t + c2

1 1

sin w2t

11 oo : 01 : oo by the initial conditions. The solution can also be written by in t erms of old (x 1,x 2) coordinates 1 1

1

1

sin w2t

1

1

-2

-2

sinw1t

where c1, c2, c3 , c4 are the constants which to be determined by t he initial conditions x1(0), x2(0), i1(0), i2(0).

[email protected]

@Sk Jahiruddin, 2020

22

Physicsguide

Small Oscillation

Example 3 .5 : Three masses m each , init ially located equidistant from one another on a horizontal circle of radius R. They are connect ed in pairs by three springs of force constant k each and of unstretched length 21r R/ 3. The spring threads the circular tract so t hat the mass is constrained to move on t he circle. Find the normal modes.

11 oo : 01

: 02

Solution: Let 01 , 02 and 03 be t he angular displacements of t he three masses from their equilibrium positions. The Lagrangian is given by £ = T - V, where,

021 + 02 + 02 2 3 and

23

[email protected]

Physicsguide

@Sk Jahiruddin, 2020

Small Oscillation

T and V matrices are 2

V

= kR

2

1 0 0

- 1 - 1

- 1

T = mR

2

- 1 -1 -1 2

2

0 l 0

0 0 1

You can write the secular equation as before and easily show that the normal modes are given by w = 0 and w = (doubly degenerate). For w = 0, the secular equation is -k -k -k 2k -k 2k

R

2

_.,,.

_.,,.

')0

=0

~

11 oo : 01 : os egenerate . For w

= 0, t he secular equation is R

2k -k -k -k 2k -k -k -k 2k

2

=0

which gives A 1 = A2 = A 3 . T he normalized normal coordinate is given by from ATTA = I. We get 1

J3mR 2 For w

2

1 1 1

= 3k/m, we have -k -k -k -k - k - k -k -k -k

A1 A2 A3

=0

which gives A1 + A2 + A3 = 0. The eigenvectors cannot be uniquely fixed. However , if we use the ort hogon ality with [email protected]

Physicsguide

24

@Sk Jahiruddin, 2020

Small Oscillation

t he zero mode, we can t ake t hese two choices (2 , - 1, 1) and (0, 1, - 1) which are ort hogonal to t he first vector and t hemselves ort hogonal. Normalization gives 1

J6mR 2

3.2

2 - 1 - 1

• )

1

J2mR 2

0 1 - 1

Rigorous solution of Linear triatomic

molecule

11 oo :01 : oa ✓6mR2

3.2

✓2mR2

- 1

- 1

Rigorous solution of Linear triatomic

molecule We now consider t he model of linear t riatomic molecule. In t he equilibrium configuration of the molecule, two atoms of mass m are symmetrically located on each side of an atom of mass M as shown in the figure.

All three atoms are on one straight line, t he equilibrium distances apart are denoted by b . For simplicity, we shall first consider only vibrations along t he line of the molecule, and the actual complicated interatomic potential will be approximated by two springs of force constant k joining the t hree 25

[email protected]

Physicsguide

@Sk J ahiruddin, 2020

Small Oscillation

atoms. The positions of three atoms are denoted by x 1 , x 2 , and x 3 In these coordinat es, t he potential energy is

V

k

= 2 (X2 -

X1 -

b)

2

k

+ 2 (X3 -

X2 -

b)

2

We now int roduce coordinates relative to the equilibrium posit ions:

11 oo : 01 V

=-

: 1o

2

b) + - (x3 -

(x2 -

X1 -

Xo2 -

X o1 =

X2 -

b)

2

2 2 We now int roduce coordinates relative to the equilibrium posit ions:

where

b=

Xo3 -

Xo2

The potential energy is now

or V =

k ('r/12 + 2rJ22 + 'r/32 - 2rJ1 'r/2 - 2rJ2rJ3 )

2

Hence the V matrix is

k -k

0 2k - k

V =

-k 0 -k

k

The kinetic energy is

m ( ·2

-2

T = 2 'r/1 + 'r/3)

+

26

[email protected]

@Sk Jahiruddin, 2020

M 2

-2

'r/2

Physicsguide

Small Oscillation

So t he T matrix is

O 0 0 JVJ 0 0 0 m

m

T =

The secular equation is

11 oo : 01

: 13

O 0 0 M 0 0 0 m

m

T = The secular equation is

-k

0 -k

2

2k-w M -k k -w 2 m

= 0

Evaluate the det erminant. You will get the equat ion

The solutions are

k m

'

w3

=

k m

2m 1+ M

Now insert the values of t he frequencies into t he equation of motion

(V -

w

2

T) A= 0

2 (mw - k) a1 +

ka1 Oa1

ka2 2 + (Mw - 2k) a2 + ka2

+Oa3 +ka3 2 + (mw - k) a3

=0 =0 =0 (3.3)

[email protected]

@Sk J ahiruddin,

2020

27

P hysicsguide

Small Oscillation

one by one Insert ion of w = w1 = 0 into (3.3) yields a1 = a2 = a3. The eigenfrequency w 1 = 0 does not correspond to a vibrational motion. but reoresents onlv a uni- form translation of the

11 oo : 01

: 1s

one by one Insert ion of w = w1 = 0 into (3.3) yields a1 = a2 = a3. The eigenfrequency w 1 = 0 does not correspond t o a vibrational motion, but represents only a uni- form translation of the ent ire molecule: • ➔ o ➔ • 1 2

Inserting w = w2 = (k/m) ! into (3.3) yields a 1 = - a3 , a 2 = O; i.e., the cent ral atom is at rest , while t he outer atoms vibrate against each other: +-- • o •



Inserting w = w 3 = {k/m(l + 2m/ M )} 1 into (3.3) yields a 1 = a 3 , a 2 = -(2m/M)a 1 , i.e. , t he two outer atoms vibrate in phase, while the central atom vibrates with opposite phase and with another amplitude: • ➔ +-- o • ➔ 1 2

So without normalization the normal coordinates are (1

= 'r/1 + 'r/2 + 'r/3

(2

=

(3

'r/1 -

= 'r/1

-

(3.4)

'r/3

2m M 'r/2 + 'r/3

With the equations

k

• ••

m

+

2k M

(3 = 0

(3.5) Now we will evaluate t he normalized eigenvectors seriously. • For w = 0, t he equation VA - w 2T A = 0 can be written 28

[email protected]

P hysicsguide

@Sk Jahiruddin, 2020

Small Oscillation

as

(

k

-k

O \

( A1 \

11 oo :01

: 1a

@Sk Jahiruddin, 2020

Small Oscillation

as

k -k 0 - k 2k - k 0 -k k

A1 A2 A3

= 0

which gives A 1 = A2 = A3. Taking each of t hem to b e equal to 1 (in some unit) we get

A=

l

~

1 1 1

where N is to be fixed by normalization. Normalization condition gives

1 N •

1.e. M

0 0 0 M 0 0 0 m

m

1 1 1

+ 2m =

J2m+M • Consider w =

[email protected]

@Sk Jahiruddin, 2020

= 1

N . Thus t he normalized vector is 1

2

1 1 1

1 1 1

! . For t his case, we have 0

-k

0

-k

k

-k

0

-k

0 29

= 0

P hysicsguide

Small Oscillation

11 oo : 01 [email protected]

: 20

Physicsguide

29

@Sk J ahiruddin, 2020

Small Oscillat ion

which gives A2 = 0 and A 1 = A 3 . Using the orthogonality with t he mode w 1 , we take t his mode to be 1

1

0

vR

- 1

Normalization condition 1 N

O 0 0 M 0

0

0

-1

1

m

1 0 -1

0

m

= 1

fixes N = 2m and the displacement vector is 1

1

0

v'2m

- 1

which shows the central mass to be at rest while the end masses displace by the same amount in the opposite direction. 2

• For w =

! + ~, t he equation is -2km M

-k 0

[email protected]

-k

0

kM

-k

m

-k

-2km M

30

A1 A2 A3

= 0

Physicsguide

11 oo : 01

: 23

m

-k

0

jahir@physicsguide .in

- 2 km M

P hysicsguide

30

@Sk J ahiruddin , 2020

Small Oscillation

which gives A 1 = A3 and A 2 = -2;/ A 1. The normalization can be done as in the previous cases. The displacement vector is given by 1

M 2m M + 4m 2

- 2m/M 1

We can now get t he new set of coordinates. As we have started with the set of coordinates 'T/ and found t he normalized eigenvectors t hen the relation between the old set of coordinate and new set of coordinate is rJ

= A(

The A matrix is A=

1

1

1

✓2m+M 1 ✓2m+M 1

J2m+M

1

1 ✓2m

1

1

V2rn

1

JVI

0

2mM

- 1

+4m2

-2m/M 1

✓2mM~4m2 2m M2+2m!Vf

0

1

1

J21n+M

✓2m

M 2mM+4m2

We get t he new coordinate set after some tedious calculation which you need not to do, just see t he results .

A-1

../m

../M

../m

J 2rri+M

J 2m+M

J 2m+M

=

1 2

O

1 2

M

2m

M

J4m+2NI

J 2m+M

J4m+2lllf

11 oo : 01

: 26

which you need not to do, just see the results.

rm

m

rm

J2m+M

J2m+M

J2m+M

A-1=

1

1 2

0

2

M J4m+2M

2m

J 2m+M

M J4rri+2M

31

[email protected]

Physicsguide

@Sk Jahiruddin, 2020

Small Oscillation

Verify that AA- 1 = I Hence (1

1 m+M

= ✓2

(2 =

Vffi'T/1

+ ✓-ivf '//2 + Vffi'f/3

1 - (rJ1 - rJ3) 2

The equation of motions are in normalized coordinates

3.3

Making the non quadratic potential quadrc

Example 3.6: Consider a particle of mass m moving in two dimensions in a potential

V (x,y)

1

2

1

= - kx + Aox 2

2

22

y

1

4

+ 4 A1X,

k , Ao , A1 > 0

(a) At what point (x 0 , y0 ) is the particle in stable equilibrium? (b) Give the Lagrangian appropriate for small oscillations about t his equilibrium position. (c) What are the normal frequencies of vibration in (b)?

11 oo :01 : 2a at point x 0 , y 0 is t e partic e in sta e equilibrium? (b) Give the Lagrangian appropriate for small oscillat ions about t his equilibrium position. (c) What are the normal frequencies of vibration in (b)?

[email protected]

32

P hysicsguide

@Sk Jahiruddin, 2020

Small Oscillat ion

Solution: (a) A point where

and

a2 v

ax2 (dx)2

a2 v

+ 2 8x8y dxdy +

a2 v

ay2 (dy)2 > 0

is a point of stable equilibrium. For t he given potential we find two such points, k/ Ai , 0 and - k/ Ai , 0 (b) V is a minimum at a point of stable equilibrium (xo, Yo) . At a neighboring point (x, y), we have, to second order of the small qualit ies x - xo, y - Yo,

V (x, y) = V (xo, Yo) 1

+2

a2 v 8x2

2

x· Y Ql

(x - xo)

(x - xo) (y - Yo)

Q

a2v

+ 8y2

+2

(y - Yo)

2

xo ,Yo

Evaluate the derivatives for t he equilibrium point ;1, 0 . You will get upto second order approximation of x - x 0 and

11 oo : 01

: 31

fi,

Evaluate the derivatives for the equilibrium point 0 . You will get upto second order approximation of x - x 0 and

y-yo

2k

[email protected]

x-

Physicsguide

33

@Sk Jahiruddin, 2020

Small Oscillation

Shift the coordinate system to the new origin

X

I

=

k X-

A'

f;, 0

.

I

y =y

and take t he new origin as the reference level for potential energy. Then

And the Lagrangian is l ( X. i2 +y. i2) - -1 k L = T - V = -m 2 2

Similarly for t he other point of equilibrium, we set X

II

=x+

k

y

II

=

A' II II l . I I x , y rep ac1ng x , y

(c) The secular equation

y

11 oo : 01

: 33 II

y = y 1 . / I x , y rep ac1ng x , y II

II

(c) The secular equation

2k - mw 2 0

0 k>-.o _ >-.1

=0 mw2

The solutions are

2k m' [email protected]

Physicsguide

34

@Sk Jahiruddin, 2020

Small Oscillation

These are the normal angular frequencies for small oscillations of the system , about both of the points of equilibrium.

Example 3.7: Two mass points of equal mass m are connect ed t o each other and to fixed points by three equal springs of force constant k, as shown in t he diagram. The equilibrium length of each sprint is a . Each mass point has charge +q, and t hey repel each other according to Coulomb law. Set up the secular equation for the eigenfrequencies. ,n

k

m

+q

a

+q

k a

Solution: This problem also has two generalized coordinates x 1 and x 2 , t he coordinates of the two masses start ing from left . x 10 = a and x 20 = 2a are the init ial posit ions of the

11 oo : 01

: 36

Solution: This problem also has two generalized coordinates x 1 and x 2 , t he coordinates of t he two masses start ing from left . x 10 = a and x 20 = 2a are t he init ial posit ions of t he masses. Let rJi and T}2 be t he displacements from the equilibrium posit ions. (rJ1 = x 1 - x 10, T}2 = x 2 - x 2o) So kinetic energy operator will be

[email protected]

@Sk Jahiruddin,

35

Physicsguide

2020

Small Oscillation

The T matrix will be

T

0 0 m

m

As far as potential energy is concerned, besides t he contribut ion from the t hree springs, we will have one extra contribut ion due to t he Coulomb repulsion between t he two charges V

=

1 2 1 2 krJ1 + k (rJ2 - rJ1 ) 2 2

=

1( 2 ') ) 2 2krJ1 - 2kr}1T/2 + 2krJ2

+

1 2 krJ2 + 2 41rEo

+

q2 X1 -

X2

q2 4 I JrEo T/2 - T/1 - a

Because rJ2 - rJi / a / 2

= Q

2co

Problem: A sphere of radius R 1 has charge density p uniform within its volume, except for a small spherical hollow region of radius R 2 located a distance a from the center. [email protected]

25

Physicsguide

@Sk J ahiruddin, 2020

Electrostatics

Find t he field E at the center· of the hollow sphere.

Solution: Consider an arbitrary point P of the hollow region as shown in figure 3. 7 and let

OP = r,

Q'P = r',

00' = a

'

r' = r - a

11 oo : 01

: 1o

. ons1 er an ar 1trary point region as shown in figure 3. 7 and let

OP=r,

00' = a

Q'P=r',

p

, Rt

'

''

'

r' = r - a

\~ ""R2 ~ ., ' '' ;'

\

'('a

/,

Figure 3. 7: Electric field inside hole

If t here were no hollow region inside the sphere, t he elect ric field at the point P would be

[email protected]

26

@Sk J ahiruddin , 2020

P hysicsguide

Electrost atics

If only t he spherical hollow region has charge density p t he electric field at P would be p I E 2 = -r 3so

11 oo : 01

: 13

If only t he spherical hollow region has charge density p t he electric field at P would be

E 2 =p-r' 3Eo

Hence t he superposit ion t heorem gives t he electric field at P as p E = E1 - E2 = - a 3Eo

Thus the field inside the hollow region is uniform. This of course includes t he center of t he hollow. There are many examples in t he standard books like Griffiths. Please do some exercises from Griffiths also.

4

Divergence and Curl of Electric Field

Divergence and curl of Electric field is given by (Remember, here we are dealing with t he electrostatic case, when t he charges of the syst em are not moving. In the dynamic case

j [email protected]

27

P hysicsguide

@Sk J ahiruddin, 2020

Electrostatics

t he cur1 will be different) p

V·E = -

EQ

(4.1)

11 oo : 01

: 1s

@Sk J ahiruddin, 2020

Electrostatics

t he cur 1 will be different) p

V·E = -

(4.1)

V xE = O

(4. 2)

Eo

We write E (x ) in terms of a scalar function V(x ) as

E (x ) = - VV (x )

(4.3)

The very important function V (x) is called the electric potent ial. 1 p (x') d3 x' V(x) = --(4.4) 41rEo Ix - x' Applying it with t he divergence of electric field we get

This is called Poisson's equation. In regions of space where t he charge density vanishes, we're left solving the Laplace equation

(4.6)

5

Electric Potential

It is instructive to construct V(x ) in another way. Let - V (x) be the line integral of E along a curve r from a [email protected]

@Sk J ahiruddin, 2020

reference point x 0 to x

28

Physicsguide

Electrostatics

11 oo :01

: 1a

@Sk J ahiruddin, 2020

Electrostatics

reference point x 0 to x

V(x) = -

r

E · df

(5.1)

This equation says that V (x) is equal to the work that must be done by an external force, acting against the electric force, to move unit charge from x 0 to x along t he path r , because - E is t he force per unit charge that t he external agent must exert. Example: T he electric potential of some configuration is given by the expression e->-.r

V(r) = A -

r

where A and ,\ are constants. Find t he electric field E (r), t he charge density p(r), and the total charge Q .

Solution:

0 E =- v'V= -A-

or

=-A

e- >-.r

r

r ( - A) e->-.r - e->-.r

j [email protected]

@Sk J ahiruddin, 2020

r2

29

A

r A

=

Ae- >-.r (1

+ Ar)

r

r2

Physicsguide

Electrostatics

11 oo : 01 [email protected]

: 21

29

Physicsguide

@Sk J ahiruddin , 2020

Electrost atics

So the potential p=EoV·E

= EoA e-,\r(l + Ar) V ·

r"' r2

But as you know r"' r2

And from t he properties of the delta function

The gradient

a

v7 (e-,\r (l + Ar)) = f Br (e- ,\r( l + Ar)) =

f { - Ae- ,\r(l +Ar) + e- ,\r A} = f (-A 2re- ,\r)

So we get

And hence the charge density p [email protected]

= EoA 30

Physicsguide

00 : 01 : 23

p

= EoA

j [email protected]

30

@Sk J ahiruddin, 2020

Physicsguide

Electrostatics

Hence t he total charge

- Ar _e _ 41rr 2 dr r 00

2

= EoA 41r - .\ 41r

r e-Ar dr 0

6

Surface charge and discontinuity of electric field

Now consider an infinite plane, which we take to be z = 0 , carrying uni£orm charge per unit area, a . We again take our Gaussian surface to be a cylinder, this t ime with its axis perpendicular to the plane as shown in the figure. In t his context, the cylinder is sometimes referred to as a Gaussian '' pillbox'' (on account of Gauss' well known fondness for aspirin).

11 oo : 01

: 26

31

[email protected]

Physicsguide

@Sk J ahiruddin, 2020

Electrostatics

E

-- --

--

z=O

- - .... E

Figure 6.1: Discontinuity of electric field when surface charges are present

On symmetry grounds, we have

E = E (z) z Moreover, the electric field in t he upper plane, z > 0, must point in the opposite direction from t he lower plane, z < 0, so t hat E(z) = -E(-z). The surface integral now vanishes over the curved side of the cylinder and we only get contributions from t he end caps, which we take to have area A . This gives

o-A E · dS = E (z) A - E (-z) A = 2E(z) A = S Eo

11 oo : 01 : 2a of the cylinder and we only get contributions from the end caps, which we take to have area A . This gives

aA

E · dS = E (z) A - E (-z) A = 2E(z) A = -

Eo

S

The electric field above an infinite plane of charge is t here32

j [email protected]

Physicsguide

@Sk J ahiruddin, 2020

Elect rostatics

fore (J"

E (z) = -

2Eo

Note that the electric field is independent of t he distance from the plane! This is because the plane is infinite in extent: t he further you move from it, the more comes into • view. There is another important point to take away from t his analysis. The electric field is not continuous on either side of a surface of constant charge density. We have ' I -

-

I I

__ /!-

✓ L-_1

-

-.
--f

-

+

+ +

.,...---7

-

> ~

+

Figure 8. 1: Conducting sphere in an Electric field

We'll learn how to compute t he electric field in this, and related, situations later.

9

Image Problem

11 oo : 02 : 1o

9

Image Problem

For particularly simple situations, there is a rather cute method t hat we can use to solve problems involving conductors. Although t his technique is somewhat limited, it [email protected]

@Sk J ahiruddin, 2020

47

Physicsguide

Electrostatics

does give us some good int uition for what's going on. It's called t he method of images. Example: A charged particle near a conducting plane: Suppose a point charge q is held a distance d above an infinite grounded conducting plane. What is t he potential in t he region above the plane? It's not just (1/ 41rt 0 ) q/n, for q will induce a certain amount of negative charge on t he nearby surface of t he conductor; the total potential is due in part to q directly, and in part to this induced charge. But how can we possibly calculate t he potential, when we don't know how much charge is induced or how it is distributed?

We're looking for a solution to t he Poisson equation with a delta-function source at z = d = (0, 0, d), together with t he requirement that

R , the contribution from both the surface and volume bound charges need to be calculated

Q tot = (kR) (41r R

2

)

+ (-3k) : 7rR

3

=0 '

so

One important result you need to remember; t he electric field produced by a uniformly polarized sphere of radius R •

IS

Inside: E

=-

1 P, 3Eo

for r < R

Outside: the potential and field is just like a perfect dipole at the origin

V =

1 p ·f 41rEo r 2 '

for

r> R

11 oo : oo : 16 dipole at t he origin 1 V = p ·f 41rEo r 2 '

3

for

r > R

Electric Displacement Vector

We learned that the electric field obeys Gauss' law

[email protected]

6

Physicsguide

@Sk J ahirudd in, 2020

Dielectrics

This is a fundamental law of Nature. It doesn't change just because we're inside a material. But, from our discussion above, we see that there's a natural way to separate the electric charge into two different types. There is the bound charge Pb that arises due to polarisation. And then t here is anything else. This could be some electric impurit ies t hat are stuck in the dielectric, or it could be charge t hat is free to move because our insulator wasn't quite as good an insulator as we originally assumed. The only important thing is that t his other ch arge does not arise due t o polarisation. We call t his extra charge free charge, pf . Within the dielectric, t he total charge density is P = Pb+ PJ ~

and we know Pb

~

= - V .P , hence

(3.1)

11 oo : oo : 19 '

(3.1) ......

-+

and we know Pb= -V.P , hence

Eo V · E

= p=

Pb+ PJ = - V · P

+ PJ

(3.2)

we write t he equation as

(3.3) Now define D [email protected]

EoE

+P

(3.4) Physicsguide

7

@Sk J ahiruddin , 2020

Dielectrics

In integral form

D · da =

Qfenc

(3.5)

where Q ! enc denotes the total free charge enclosed in the volume. That 's quite nice. Gauss' law for the displacement involves only the free charge; any bound charge arising from polarisation has been absorbed into the definition of D

3.1

Linear Dielectrics

In general, the polarisation P can be a complicated function of the electric field E . However , most materials it t urns out

11 oo : oo : 21 3.1

Linear Dielectrics

In general, the polarisation P can be a complicated function of the electric field E . However, most materials it t urns out t hat P is proport ional to E. Such m aterials are called linear dielectrics. They have

(3.6) where Xe is called t he electric susceptibility. It is always posit ive: Xe > 0 Any function that has P (E = 0) = 0 can be Taylor expanded as a linear term + quadratic + cubic and so on. For j [email protected]

@Sk J ahiruddin , 2020

8

Physicsguide

Dielectrics

suitably small electric fields, t he linear term always dominates. To determine when t he quadratic and higher order terms become important, we need to know the relevant scale in the problem. For us, t his is the scale of electric fields inside the atom. In most situations, t he applied electric field leading to the polarisation is a t iny perturbation and the linear t erm dominates. The linearity fails for suitably high electric fields. There are exceptions to linear dielectrics. Perhaps the most striking exception are materials for which P # 0 even in the absence of an electric field. Such materials - which are not particularly common - are called ferroelectric. For

11 oo : oo : 24 There are exceptions to linear dielectrics. Perhaps the most striking exception are materials for which P # 0 even in t he absence of an electric field. Such materials - which are not particularly common - are called ferroelectric. For what it's worth, an example is B aTi0 3 . Well, for linear dielectrics D = EoE

+P

=

EoE + EoXeE

Eo (1 + Xe) E

=

(3.7)

So we define

(3.8)

D =EE where

E

Eo (1 + Xe)

(3.9)

This constant E is called t he permittivity of t he material. (In vacuum, where t here is no matter to polarize, the [email protected]

Physicsguide

9

@Sk J ahiruddin, 2020

Dielectrics

ceptibility is zero, and t he permittivity is Eo . That's why Eo is called the permittivity of free space. The relative permittivity, or dielectric constant, of the material, is defined as Er -

1 + Xe

E

=-

Eo

(3.10)

Example: A metal sphere of radius R 1 carries a charge

Q . It is surrounded, out to radius R 2 by linear dielectric material of permittivity E. Find the potential at the center

11 oo : oo : 26 Eo Example: A metal sphere of radius R 1 carries a charge

Q . It is surrounded, out to radius R 2 by linear dielectric material of permittivity E. Find the potential at the center (relative to infinity) .

Solution: To compute V, we need to know E ; to find E , we might first try to locate t he bound charge; we could get t he bound charge from P , but we can't calculate P unless we already know E (from the relation P = EoXeE ) We can calculate D , as § D · da = Q ! en c

j [email protected]

Physicsguide

10

@Sk J ahiruddin , 2020

Dielectrics

Q R1

11 oo : oo : 29

Figure 3.1: Conductor surrounded by dielectrics

for all points r > R 1 (Inside the metal sphere, of course, E = P = D = D = 0. Once we know D , it is a trivial matter to obtain E , using D =EE

for R 1 < r < R 2

Q 2f , 41rEr

Q 2 r, for r > R2 41rEor

[email protected]

11

Physicsguide

@Sk J ahiruddin, 2020

Dielectrics

The potential is t hen R2

0

V =-

E-dl =00

R2

41r

41rE0 r 2

00

R1

Q

Q

1

0

Q 41rEr 1

dr

2

+ Eo R 2 ER1

dr-

(O)dr R1

1 ER 2

11 oo : oo : 32 00

00

R1 R2

Q 41r

41rE0 r 2 0

Q

dr-

47rEr 2

1

(O)dr R1

1

1

+ EoR 2 ER1

ER 2

The polarization is present inside the dielectric material EoXeQ " P = EoXeE = r; 2 47rEr The bound volume charge

R1 < r < R2

Pb= - V · P

=0

The volume surface charge EoXeQ

at t he outer surface,

41rER~'

-EoXeQ 41rER i '

4

at the inner surface.

Boundary Conditions in dielectric (4.1)

[email protected]

12

P hysicsguide

@Sk J ahiruddin , 2020

Dielectrics

-+

we write t he same equation in terms of E

(4.2) The boundary condit ions in terms of Electric field ...

11 oo : oo : 34 we write t e same equation in terms o

(4.2) The boundary conditions in terms of Electric field ... ~

~

E above -

E below

II

II

E above -

1

(4.3)

= -a Eo

_

E below -

(4.4)

O

Boundary conditions in terms of potentials OV above E above

On

OVbelow E below

-

Vabove

=

On

=

V below

- a1

(4.5)

(4.6)

We use the boundary conditions depending on the problems given.

5

Energy in Dielectrics

Energy stored in a dielectric system is

w=!

D -EdT

2

[email protected]

13

@Sk J ahiruddin, 2020

(5.1) Physicsguide

Dielectrics

Example: A sphere of radius R is filled with material of dielectric constant Er and uniform embedded free charge p f.

What is t he energy of this configuration?

00: 00: 37

Example: A sphere of radius R is filled with material of dielectric constant Er and uniform embedded free charge pf. What is t he energy of this configuration?

Solution: The displacement vector

D(r) = The elect ric field

E (r) =

Then t he electrostatic energy without considering the 2 dielectrics (W = ~ J E dT)

Pi 3Eo

0

21r

2

RG

1

00

-

R

r

4

2

41rr dr

2 R5

9Eo PJ

[email protected]

14

@Sk J ahirudd in, 2020

The total energy (W =

Physicsguide

Dielectrics

1f D · EdT)

11 oo : oo : 39 @Sk J ahiruddin , 2020

Dielectrics

The total energy (W =

if D · EdT ) R

Pi Pi 2 2 r 41rr dr 3 3EoEr 0 00 1 ? PiR3 PiR3 41rr~dr + 3 4 3Eo R r 21r 2 Rs - 1 + 1 9EoPi 5Er

Let's check that W2 is the work done on t he free charge in assembling t he system. We start with the (uncharged, unpolarized) dielectric sphere, and bring in the free charge in infinitesimal inst allments (dq) , filling out t he sphere layer by layer. When we have reached radius r', t he electric field •

IS

E(r) =

Pi r (r < r') 3EoEr p r'3 i r (r' < r < R) 2 3EoEr r Pi r'3 " 3Eo r2 r (r > R)

The work required to bring the next dq in from infinity to

[email protected]

@Sk J ahiruddin, 2020

15

Physicsguide

Dielectrics

11 oo : oo : 42 15

[email protected]

Physicsguide

@Sk J ahiruddin , 2020

Dielectrics

r' is R

dW = - dq

r'

E · di +

E · di R

00

R

= - dq

00

1

1

R

Er

-+-

1 -dr + r2 1

1

r'

R

dq

---

R

dW = -dq

p1r'3 r' 1 -dr 3EoEr R r 2

r'

E · di +

E · di R

00

R

= -dq

00

1

1

R

Er

-+-

r'

1 p 1r'3 -dr+2

r

-

3EoEr

1 1 --r' R

R

1

r

dr 2

dq

This increases t he radius (r') :

so t he total work done, in going from r' = 0 tor' = R , is

W =

41rp

2

f

R

3Eo

21r

1

2

9EoPJ j [email protected]

R

1

1- -

5

r' dr'

Er

0

1

+-

Er

0

Rs -1+ l 5Er

16

Physicsguide

11 oo : oo : 4s

j [email protected]

16

Physicsguide

@Sk J ahiruddin , 2020

Dielectrics

Example: A spherical conductor, of radius a, carries a charge Q. It is surrounded by linear dielectric material of

susceptibility Xe, out to radius b. Find the energy of this configuration.

Rz

Q R1

Figure 5.1: Conductor surrounded by dielectrics: Energy

Solution: The displacement vector is D =

0, _Q......,,. f

41rr2

'

(r < a) (r > a)

So the electric field

E=

(r < a) (a< r < b) (r > b)

11 oo:oo:47 So t he electric field (r < a)

E=

(a< r < b) (r > b)

[email protected]

P hysicsguide

17

@Sk J ahiruddin, 2020

Dielectrics

The work done

w = !2

D. E dT

1 Q2 - - -47r 2 (41r) 2

Q2

1

E b

1

- 1 -81r E r

Q2

b

l 1 -2 2r2dr

a r

r

1

- 1

Eo

r

1

1

+a

1

- -

-

(1 + Xe) a b Q2 1 Xe -+81rEo (1 + Xe) a b 81rEo

5.1

1

00

+-

Eo

b

1

r

2

dr

00

b

1 +-b

Forces on Dielectrics

Force is the derivative of energy F = dW

dx

(5.2)

If capacitance of a system is changing wit h respect to x then t he force (5.3) ..

11 oo : oo : so If capacitance of a system is changing with respect to x then the force (5.3)

Example: Capacitor partially filled with dielectrics

[email protected]

Physicsguide

18

@Sk J ahiruddin , 2020

Dielectrics

Figure below shows parallel plat es of size f x w and separation d , partially filled with a dielectric slab. The dimensions of the slab are x x w x d , with x < f . What is the capacitance? Assume the separation d is small compared to x and f - x, so that end effects are negligible. -Q d

+Q X

e-x

Figure 5.2: A dielectric slab inserted part-way between charged plates. A force is pulling the slab into the space between t he plates

Let the potential difference between the plates be V . The key to this example is t hat the electric field E is the same in the two regions (dielectric and vacumm) between t he plates. By planar symmetry, the field is uniform in both regions and normal to the plates. But E , being normal to the

11 oo : oo : s2 Let the potential difference between the plates be V . The key to this example is t hat the electric field E is the same in the two regions (dielectric and vacumm) between t he plates. By planar symmetry, the field is uniform in both regions and normal to the plates. But E , being normal to the plates, is tangent to t he boundary surface of t he dielectric. Since the tangent ial field must be continuous, t he field is t he same on either side of t he boundary. Thus the electric j [email protected]

19

Physicsguide

@Sk J ahiruddin , 2020

Dielectrics

field is E = 11 V / d in both regions, where 11 is normal to the plates. To determine t he capacitance we must relate V to the charge ±Q on the plates. Apply Gauss's Law to a surface surrounding one plate, e.g. t he lower plate: The flux of D is equal to Q . That is, since D is EE in t he dielectric (of area xw) and EoE in t he vacuum region [ of area (£ - x )w] EExw

+ EoE(R -

x)w

=

Q

Substit uting E = V / d we find that the capacitance is

We might have anticipated this result by noting t hat t he system is equivalent to two capacitors in parallel, with C 1 = EXW / d and C 2 = Eo (£ - x )w / d. Then t he combined capacitance is C1 + C2.

11 oo : oo : 55 We might have anticipat ed t his result by noting t hat t he system is equivalent to two capacitors in parallel, with C 1 = EXW / d and C2 = Eo(R - x )w / d. Then t he combined capacitance is C1 + C2 . Now what is t he force on t he dielectric slab? We can have two cases. The plates are with fixed charge: Plat es are isolated so t hat t heir charges ± Q are constant. The work done by t he electrostatic force if t he slab moves a dist ance dx into [email protected]

20

Physicsguide

@Sk J ahiruddin , 2020

Dielectrics

t he capacitor is Fdx Because the syst em is isolated , conservation of energy says t hat t he work is equal to - dU where U is the tot al energy of t he capacitor. T herefore t he inward force on t he slab is F = - dU / dx To calculate U for this example it is natural to use t he equation U = Q 2 / 2C because Q is constant . The capacitance we calculated. Thus the energy is

U(x) =

Q2d 2w [Ex+ Eo(f - x) ]

So the force on the slab is

F

=

2

Q d (c - to) 2 2w [EX + Eo (R - x)]

For fixed charge the force decreases as x increases. We may also express F in terms of the potent ial difference be-

11 oo :oo : s1 F=

E -

Eo

+ Eo ( .e -

2w [EX

x)]

2

For fixed charge the force decreases as x increases. We may also express F in terms of the potential difference between t he plates, V = Q / C, as F

=V

2

w

(E - Eo) 2d

Plates with fixed potential difference: Now suppose t he plates in Fig. 5.2 are connected to a battery so that the potential difference V is fixed. In this case the work done by the electrostatic force F is not equal to -dU, because j [email protected]

21

@Sk J ahiruddin , 2020

Physicsguide

Dielectrics

additional energy is supplied by the battery. If the slab moves by distance dx , then charge dQ is transferred to the plates from the battery, and so the batt ery supplies energy (d) V. The conservation of energy in this case is dU = (dQ )V - Fdx

Here it is natural to use the equation U = CV 2 / 2, because V is const ant. Also, Q = CV so dQ = (dC )V. Inserting t hese relations into the conservation law gives

Thus the for ce on t he slab is

11 oo : 01 : oo by the electrostatic force F is not equal to -dU, because j [email protected]

21

P hysicsguide

@Sk J ahiruddin , 2020

Dielectrics

additional energy is supplied by the battery. If the slab moves by distance dx, then charge dQ is transferred to the plates f1"om the battery, and so the battery supplies energy (d) V. The conservation of energy in this case is dU = (dQ) V - Fdx

Here it is natural to use the equation U = CV 2 / 2, because V is constant. Also, Q = CV so dQ = (dC )V. Inserting t hese relations into the conservation law gives

Thus the force on t he slab is F =

! dC v2= V 2 dx

2

w

(E - Eo) 2d

Again, a reminder, do all t he exercises which are given inside t he chapters of Griffit hs.

j [email protected]

22

Physicsguide

11 oo : oo : oo

Magnetostatics Sk J ahiruddin Assistant Professor Sister Nibedita Govt. College, Kolkata Author was the topper of IIT Bombay M.Sc Physics 2009-2011 batch He ranked 007 in IIT JAM 2009 and 008 (JRF) in CSIR NET J t1ne 2011 He has been teaching CSIR NET aspirants since 2012

1

@Sk Jahiruddin, 2020

Magnetostatics

11 oo : oo : 03 1

@Sk J ahiruddin , 2020

Magnetostatics

Contents 1

Magnetostatics

4

1.1

Steady current .

1.2

Force on a moving charge





















4





















6









8

Electric current as a source of magnetic field

9

1.3. 1

Biot-Savart Law .























9

1.3.2

Ampere's Law . .























11









13

1.2.1 1. 3













Force on current carrying wire

1.4

Surface Currents and Discontinuit ies

1.5

Magnetic Vector Potential





















17

1.6

Magnetic Dipole . . . . . .





















20

1.6.1 1.7

Torques and Forces on Magnetic Dipoles 28

Electric Current . . . . .























28

1. 7.1























30

j [email protected]

Current Density . 2

Physicsguide

11 oo : oo : 06 1.7. 1

Current Density .



















2

j [email protected]





30

Physicsguide

@Sk J ahiruddin , 2020

l\lfagnetostatics

1.7.2

Conservation of Charge .















30

1. 7.3

Ohm's Law . . . . . . .















31

2 Magnetic Field in a m edium

35

2. 1

Bound Currents ..





























37

2.2

Magnetic intensity





























39

Linear m agnetic material . . .









40

Electromagnet ic Boundary condit ions









42

2.2. 1 2.3

2.3.1

2.3.2

2.3.3

Magnetostatic Boundary condition inside a medium . . . . . . . . . . . . .

42

Boundary condition in both elect ric and magnetic field . . . . . . . . . .

42

Example: Field inside the gap

45









11 oo :oo :oa

[email protected]

3

@Sk J ahiruddin, 2020

1

Physicsguide

1\/Jagnetostatics

Magnetostatics

Charges give rise to electric fields. Current give rise to magnetic fields. In t his section, we will study the magnetic fields induced by steady currents. This means t hat we are again looking for t ime independent solutions to the Maxwell equat ions. We will also restrict to situations in which the charge density vanishes, so p = 0. We can t hen set E = 0 and fo cus our attention only on the magnetic field. We're left with two Maxwell equations to solve:

(1.1) and

(1.2) If you fix the current density J , these equations have a unique solution. Our goal in this section is to find it.

1.1

Steady current

Before we solve (1.1) and (1.2) let 's pause to think about the kind of currents t hat we're considerine: in this section. Be-

11 oo : oo : 11 1.1

Steady current

Before we solve (1.1) and (1.2) let 's pause to think about the kind of currents t hat we're considering in t his section. Because p = 0 , there can't be any net charge. But, of course, [email protected]

@Sk J ahiruddin, 2020

4

P hysicsguide

Magnetostatics

we still want charge to be moving! This means that we necessarily have both positive and negative charges which balance out at all points in space. Nonetheless, t hese charges can move so t here is a current even t hough t here is no net charge t ransport. This may sound artificial, but in fact it's exactly what happens in a typical wire. In that case, there is background of positive charge due to the lattice of ions in the metal. Meanwhile, the electrons are free to move. But they all move together so t hat at each point we still have p = 0. The continuity equation, which captures the conservation of electric charge, is

(1.3) Since t he charge density is unchanging (and, indeed, vanishing) , we have

(1.4) Mathematically, this is just saying t hat if a current flows

11 oo : oo : 13 Since t he charge density is unchanging (and, indeed , vanishing) , we have (1.4) v' · J = O Mathem atically, this is just saying that if a current flows into some region of space, an equal current must flow out to avoid the build up of charge. Note that this is consistent with (1.1) since, for any vector field, v' · (v' x B ) = 0 j [email protected]

5

@Sk J ahiruddin, 2020

1. 2

Physicsguide

l\tlagnetostatics

Force on a moving charge

The most basic form of the magnetic force is the force on a charge q moving with velocity v , exerted by a magnetic field B , which is F=qvxB (1.5) The SI unit of magnetic field is the t esla (T ). One t esla is defined as one newton per ampere-met er. By (1.5) the force on 1 C moving 1 m/ s perpendicular to a field of 1 T is 1 N . A magnetic field of 1 T is a rather strong field. The largest fields t hat can b e produced by conventional electromagnets are about 2 T . For high-field research in laboratories, fields of 10 - 12 T , produced with superconducting mag- nets, are used. The record for steady-state fields is about 30 T . At the surface of the Earth the magnetic field is about 0.5 x 10- 4T . Very large and very small fields are important in astronomy; e. g.10 8 T near pulsars, and 10-10 T in interst ellar space in the galaxy.

11 oo : oo : 16 . At t he surface of t he Eart h the magnet ic field is about 0.5 x 10- 4T . Very large and very small fields are important in ast ronomy; e. g.10 8 T near pulsars, and 10- 10 T in interst ellar space in the galaxy. The direction of t he magnet ic force on q is sideways, i.e., perpendicular to v and to B , as shown in Fig. 8.1 for a posit ive charge. Therefore, the magnet ic force does no work

[email protected]

6

@Sk J ahiruddin, 2020

P hysicsguide

Magnetostatics

on q: dW = F · dx = F · v dt = 0

(1.6)

The magnetic force affects t he direct ion of motion of q, but not its kinetic energy. Because the magnetic force does no work, it is not possible to define a pot ent ial energy funct ion for t he magnet ic force. The magnet ic force is velocity dependent , which is quit e different from t he ot her fundamental forces we encounter in physics. B

V

q

F

11 oo :oo :1a

Figure 1.1: The magnetic force. The force on a moving charge is sideways, perpendicular to both v and B. The figure shows F and the circular trajectory for a positive charge in a uniform field.

Equation (1.5) may be taken as the definit ion of t he magnetic field. From the measurement of t he force on a [email protected]

Physicsguide

7

@Sk J ahiruddin, 2020

Magnetost atics

test charge q, e.g. by observing its deflection for a known velocity, t he defined quantity B could in principle be deduced from (1.5) When both electric and magnetic field is present t hen t he force equation is known as lorentz force equation

F = q(E

1. 2 .1

+V

X

B)

(1.7)

Force on current carrying wire

A current-carrying wire in a magnetic field experiences a force, from the magnetic force on the individual moving charges t hat make the current . Electric current is motion of r.h::-i.r lfP. . ::-i.nrl ( 1 ..~l iR t.hP. forr.P. on

FI.

movin lf r.h ::-i.r lfP. . ~11nnoRP.

11 oo : oo : 21 A current-carrying wire in a magnetic field experiences a force, from the magnetic force on the individual moving charges t hat make the current . Electric current is motion of charge, and (1.5) is t he force on a moving charge. Suppose t he current consists of part icles wit h charge q and linear density n 1 , moving with mean velocity v . T hen the net force dF on a small segment df of t he wire is

(1.8) because n 1df is t he nt1mber of moving cha1~ges in df . T he current I in t he wire is qn 1 v, and v is parallel to df, so the total force on the wire is j [email protected]

Physicsguide

8

@Sk J ahiruddin, 2020

J\/fagnetostatics

F=

1.3

. wrre

I d£

X

B

(1.9)

Electric current as a source of magnetic field

Where does a magnetic field come from? W hat is its source? The most familiar source is a permanent magnet , i.e., a piece of magnet ized iron or ot her ferromagnet ic material. But at a more basic level t he source of B is to be found in electric current. We will study magnet ic materials, and ,.., ,.... ,.... + 1-- ,... + + 1-- ,.... .,,..,,... ,... ,..,..,,.., ,.... + ~ ,... ,... .,,.., ,..1 .,::: ,.... 1 ,..1 ,..,. .C ,... .C,.....,.,,. ,..,. .,,..,,... ,... ,..,..,,.., ,.... + ,... ,..,. .,,..,,... ,.... ,.., .C..,, ,..,. .,,..,,...

11 oo : oo : 24 The most familiar source is a permanent magnet , i.e., a piece of magnetized iron or ot her ferromagnetic material. But at a more basic level t he source of B is to be found in electric current. We will study magnetic materials, and see t hat the magnetic and field of a ferromagnet comes from properties of atomic electrons - t heir spin and orbital motion - which, although not classical currents, do involve dynamics of charged particles. However, now we are concerned with B from macroscopic steady currents.

1.3.1

Biot-Savart Law

The magnetic field of a steady line current is given by the Biot-Savart law: The field dB at a point P, due to an in-

[email protected]

P hysicsguide

9

@Sk J ahiruddin, 2020

Magnetostatics

finitesimal current element I df at a point P ' . Figure below shows the geometry. For this elemental case the Biot-Savart law is A

Biot-Savart law

I dC

p

r

11 oo : oo : 21

I df

Figure 1.2: Elemental form of the Biot-Savart law. Id/!, is t he source of magnetic field; and dB is the resulting field at P.

dB

=

µo Id/!, x

r

(1.10)

41r r 2 Here r is the distance between P' (the source point) and P (the field point), r is the unit vector in the direction from P' to P, and r = rr. Note that t he Biot-Savart law is an inverse-square law, like Coulomb's law, but the direction of t he magnetic field is azimuthal, around the axis of I d/!,. Do few examples from Griffiths. j [email protected]

10

@Sk J ahiruddin, 2020

1.3.2

Physicsguide

l\lfagnetostatics

Ampere's Law

The integral of the current density over t he surface S is the same thing as t he total current I that passes through S. Ampere's law in integral form then reads

B · dr = µ of

(1.11)

C

For t he problems where cylindrical symmetry is there, it is

11 oo : oo : 29 Ampere's law in integral form then reads

B · dr = µof

(1.11)

C

For t he problems where cylindrical symmetry is there, it is easy to apply Ampere's law to calculate Magnetic field. Consider an infinite, straight wire carrying current I . We'll take it to point in t he z direction. The symmetry of the problem is jumping up and down telling us t hat we need to use cylindrical polar coordinates, (r, rp , z ), where

r=

J x 2 + y 2 is t he radial distance away from t he wire. We take the op en surface S to lie in the x - y plane,

centered on the wire. For t he line integral in (1.1 1) to give something t hat doesn't vanish , it's clear t hat the magnetic field has to have some component t hat lies along t he circumference of the disc.

[email protected]

11

Physicsguide

1\/Jagnetostatics

@Sk J ahiruddin, 2020

s C

11 oo : oo : 32 s C z ~

r

Figure 1.3: Magnetic field of long straight vvire

But, by t he symmetry of t he problem, t hat's actually t he only component that B can have: it must be of the form B = B (r )cp . (If this was a bit too quick, we'll derive t his more carefully below). Any magnetic field of t his form automatically satisfies the second Maxwell equation V · B = 0. We need only worry about Ampere's law which tells us 21r

B · dr C

=

B (r)

rd(f)

=

21rrB (r)

=

µof

0

We see that the strength of the magnetic field is

B = µo f cp 21rr [email protected]

@Sk J ahiruddin, 2020

12

(1.12) P hysicsguide

Magnetostatics

The magnetic field circles the wire using the ''right-hand rule" : stick the t humb of your right hand in the direction of the current and your fingers cur1 in the direction of the magnetic field.

11 oo : oo : 34 e magnetic e circ es t e wire using t e rig t- an rule" : stick the t humb of your right hand in the direction of the current and your fingers cur1 in the direction of the magnetic field.

1.4

Surface Currents and Discontinuities

Consider the flat plane lying at z = 0 with a surface current density that we'll call K . Note that K is t he current per unit length, as opposed to J which is the current per unit area. You can think of the surface current as a bunch of wires, all lying parallel t o each ot her. We'll take t he current to lie in t he x-direction: K = K x as shown below. z y

K

Figure 1.4: Surface Current

From our previous result, we know that the B field j [email protected]

@Sk J ahiruddin, 2020

13

Physicsguide

l\tlagnetostatics

should curl around t he current in the right-handed sense. But, with an infinite number of wires, t his can only mean t hat B is oriented along t he y direction. In fact , from the

00: 00: 36 l\/fagnetostatics

should curl around t he current in the right-handed sense. But, with an infinite number of wires, t his can only mean t hat B is oriented along t he y direction. In fact , from the symmetry of t he problem, it must look like z y

I

I I I I

B

Figure 1.5: Magnetic field due t o a Surface Current

wit h B pointing in the -y direction when z > 0 and in t he +y direction when z < 0 we write

B = - B (z) y wit h B (z) = -B(- z). We invoke Ampere's law using the following open surface:

[email protected]

@Sk J ahiruddin, 2020

14

P hysicsguide

Magnetostatics

11 oo : oo : 39 @Sk J ahiruddin, 2020

1\/Jagnetostatics

z y

I I I

I

I

I

I I

)

I

I

I I

I I

Figure 1.6:

wit h length L in t he y direction and ext ending to We have B · dr

±z.

= LB (z ) - LB(-z) = 2LB(z) = µ 0 KL

C

so we find that the magnetic field is constant above an infinite plane of surface current B (z)

= µo K 2

z >0

This is rather similar to the case of the electric field in t he presence of an infinite plane of surface charge. The analogy with electrostatics continues . The magnetic field is not continuous across a plane of surface current . We have

[email protected]

@Sk J ahiruddin , 2020

15

Physicsguide

Magnetostatics

11 oo : oo : 42 15

[email protected]

@Sk J ahiruddin , 2020

Physicsguide

Magnetostatics

In fact, t his is a general result that holds for any surface current K . We can prove this statement by using the same curve that we used in the Figure above and shrinking it until it barely touches t he surface on both sides. If the normal to t he surface is n and B ± denotes the magnetic field on either side of the surface, then (1.13)

Meanwhile, the magnetic field normal to the surface is continuous. We can write t he boundary conditions as (1.14)

and E above ll

-

E below ll _ K - µo

(1.15)

When we looked at electric fields, we saw that the normal component was discontinuous in the presence of surface charge while the tangential component is continuous. For magnetic fields, it's t he other way around: t he tangential component is discontinuous in the presence of surface currents.

j [email protected]

16

Physicsguide

11 oo : oo : 44 currents.

j [email protected]

16

Physicsguide

@Sk J ahiruddin , 2020

1.5

l\tlagnetostatics

Magnetic Vector Potential

At this point it is useful to compare the principles of magnetostatics and electrostatics, given in t he table below. Electrostatics

Magnetostatics

V x B = µ 0J V-B = 0 B cur ls around I

V · E = p/ Eo

V x E=O E di verges from q

Note this difference: The electric field has scalar sources, but the magnetic field has vector sources. Electric charge, i.e., a point source of E from which E diverges, is a common property of elementary particles. Motion of charged particles-current -is the source of B , around which B curls. Magnetic charge, i.e., a point source from which B diverges in t he rest frame of the particle, apparently does not exist in nature, or at least it has not been observed. Such a hypothetical charge is called a magnetic monopole. Dirac showed t hat it is possible to construct a consistent quantum theory wit h both electric charges and magnetic monopoles . However , t he fundamental magnetic charge g

11 oo:oo:47 Such a hypothetical charge is called a magnetic monopole. Dirac showed t hat it is possible to construct a consistent quant um theory with both electric charges and magnetic monopoles. However, t he fundamental magnetic charge g [email protected]

@Sk J ahiruddin, 2020

17

Physicsguide

Magnetostatics

and electric charge e would necessarily be quantized , and satisfy t he relation eg = n/2, where n is an integer. Many experimenters have searched for magnetic monopoles, but so far t he results are negative. Some speCUlative t heories of high-energy physics, such as grand unified field theories, predict t he existence of very massive magnetic monopoles, too massive to be produced at current high-energy accelerators, but which might have been produced in t he big bang. Searching for magnetic monopoles continues to be an interesting experimental challenge If magnetic monopoles do not exist, t hen t he equation V · B = 0 is a universal equation of magnetism. Whether magnetic monopoles exist or not, t he source of the magnetic fields we encounter in physics are not point magnetic charges but rather currents of electric charge, corresponding to the source equation V x B = µoJ. We found in electrostatics t hat it is useful to introduce a scalar potent ial V (x) for t he electrostatic field, such t hat E = - VV. This guarantees that V x E = 0. In an analo-

11 oo : oo : so We found in electrostatics t hat it is useful to introduce a scalar potential V (x ) for t he electrostatic field, such t hat E = - VV. This guarantees that V x E = 0. In an analogous way we may int roduce a vector potent ial A (x) for the

[email protected]

18

@Sk J ahiruddin , 2020

P hysicsguide

Magnetostatics

magnetic field, such t hat

(1.16) This guarantees that V · B = 0 However , (1.16) does not uniquely determine A for a given magnetic field B : If f (x ) is an arbitrary scalar funct ion, t hen A + V f has t he same curl as A (namely B) because V x V f is identically 0. Therefore we may impose a condition on A called a gauge condition, to remove t his ambiguity. The Coulomb gauge condit ion, which we will use in magnetostatics , is

(1.17) Taking (1.16) and (1. 17) together is still not enough to make A (x) unique, because adding a constant does not change eit her V x A or V · A. But imposing an appropriate

11 oo : oo : s2 Taking (1.16) and (1.17) together is still not enough to make A (x ) unique, because adding a constant does not change either v7 x A or v7 •A. But imposing an appropriate boundary condition, such as requiring A -+ 0 at infinity, makes A unique.

Example: Consider the uniform field B j [email protected]

19

Bok. A Physicsguide

@Sk J ahiruddin, 2020

J\/fagnetostatics

vector potent ial function , satisfying (1.16) for the uniform field and the Coulomb gauge condition (1.17) is

A (x ) =

1 B x x 2

=

1

2

" " B0 (-yi + xj )

An example of uniform B is t he field inside a solenoid, so A

for B = Bok we may picture a long, t ightly and uniformly wound solenoid, whose axis is the z axis. Note for t his case t hat A (x ) is parallel to t he surface currents (azimut hal) and t hat A (x ) curls around the B field. The analytic expression of vector potential in vector notation is A (x ) = µo d3 x' J (x') (1.18) 41r v x - x'I

1.6

Magnetic Dipole

Vector potential, while in the general coordinate as shown

11 oo : oo : 55 1.6

Magnetic Dipole

Vector potential, while in t he general coordinate as shown below figure , can be expanded as

[email protected]

20

P hysicsguide

@Sk J ahiruddin , 2020

Magnetostatics

Figure 1.7: Multipole expansion of vector potential

As in t he mult ipole expansion of V, we call the first term (which goes like 1/ r) t he monopole term , the second (which goes like 1/ r 2 ) dipole, t he third quadrupole, and so on .

A (r) = µof

l

41r

r

1 di ' + r2

r' cos adI' ( 1 1 () \

11 oo : oo : sa goes like 1/r 2 ) dipole, t he third quadrupole, and so on.

!

A (r) = µof

r

41r

1

+r 3

1

di ' + -

r' cos adI'

r2

(1.19)

')2 r (

-3 cos2a - -1

2

2

dI / + · •·

The magnetic monopole term is always zero The dipole t erm A dip(r )

=

µo f 41rr 2

r' cos adI'

j [email protected]

=

µof

(r · r') di'

41rr 2

(1.20)

P hysicsguide

21

@Sk J ahiruddin , 2020

l\lfagnetostatics

We define the dipole moment

m

f

da = fa

(1.21)

Then t he vector potent ial for the t he dipole term A

0 m x r · ()µ A dip r 41r r 2

(1.22)

Magnetic field of a magnetic dipole can be written as

(1.23)

The m agnetic field of a (perfect) dipole is easiest to calculate if we put m at t he origin and let it point in the z

11 oo : 01 : oo (1.23)

The magnetic field of a (perfect) dipole is easiest to calculate if we put m at t he origin and let it point in t he z -direction see figure below.

[email protected]

P hysicsguide

22

1\/Jagnetostatics

@Sk J ahiruddin , 2020

z

e

m

I I I I I I ............... I .......... ', I cl> ' ,.......I

y

X

Figure 1.8: Calculation of magnetic field due to a magnetic dipole . . According to Eauat1on Ar1,n (r )

A

µ0 m x r

=-

~

. t he oot ent1al

11 oo : 01

: 03

Figure 1.8: Calculat ion of m agnetic field due to a m agnetic dipole . . µ 0 m x r" . According to Equation A dip ( r) = t he potent ial 2

41r

at point (r, 0, ) is

r

A . ( ) = µ 0 m sin 0). dip r 41f r 2 'fJ

(1.24)

Hence the magnetic field B dip(r )

= V x A = µo~ (2 cos 0f + sin 00)

(1. 25)

41rr

Example:

Find the m agnetic dipole moment of the

book type loop shown in Figure below. All sides have length w, and it carries a current I. 23

[email protected]

P hysicsguide

@Sk J ahiruddin, 2020

z

1 i

Magnetostatics

/2

w

~ Jl>?w

..

I

y

tw

i

w B

+

A

.

I

A

Figure 1.9:

Solution: This wire could be considered t he superposi-

11 oo : 01 : os Figure 1.9:

Solution: This wire could b e considered t he superposit ion of two plane square loops The '' extra'' sides ( AB) cancel when t h e two are put together , since t he currents flow in opposite directions. The net magnetic dipole moment is

v'2Iw 2 ,

its magnitude is

and it points along the 45° line

z =y Example: A circular loop of wire, with radius R , lies in t he xy plane (centered at the origin) and carries a current

I running counterclockwise as viewed from the positive z

(a) What is its m agnetic dipole moment? j [email protected]

P hysicsguide

24

@Sk J ahiruddin, 2020

l\tlagnetostatics

(b ) What is the (approximate) m agnetic field at p oints far from t he origin ? (c) Show t hat, for points on t h e z axis, your answer is con-

sistent with the exact field B = µ~I (R2 : :2 ) 312 when z

(a) m =Ja= I 1rR 2 z

Solution: (b) B ~ I

\

r-..

,,

>> R.

;f; ~f 1

2

(2 cos 0f + sin 00) .

/)

A

/

('

.

r. \

r,

11 oo :01 : oa (a) m =Ja= I 1rR

Solution: (b) B ~

f ~f 1

2

2

z

(2 cos 0f + sin 00)

(c) On the z axis, 0 = O,r = z, f = z( for z > 0). So

B ~ B ~ _µo_I_R_2 z 2z 3 for z < 0, 0 = in place of z 3

1r,

f =

The original ans

R ) to B ~ µ 0 I R 2/ 2 z

-z,

so the field is the sam e, with z

µo f

B = 3.

R2

) ( 2 2 312 2 R +z

reduces (for z

3

>>

So t he ans are same.

Example: (a) A disk of radius R , carrying a uniform surface charge a , is rotating at constant angular velocity w . Find its magnetic dipole moment. (b) A spherical shell of radius R , carrying a uniform surface charge a , is set spinning at angular velocity w. Find t he m agnetic dipole moment of the spinning spherical shell. [email protected]

25

Physicsguide

@Sk J ahiruddin, 2020

Magnetostatics

Also find t he m agnetic vector potential of the spinning shell. 2

Solution: (a) For a ring, m = I 1rr . So t he current due to a ring wit h radius extended to small amount r ➔ r + dr is (see figure (a) below )

11 oo : 01

: 1o 2

Solut ion: (a) For a ring, m = I 1rr . So the current due to a ring with radius extended to small amount r-+ r + dr is (see figure (a) b elow ) z

R in0 Rd0

r r+dr

(b)

(a)

F igure 1.10:

I -+ O"Vdr = O"wrdr So t he magnetic dipole moment R

m

2

1rr 0"wrdr

=

=

4

7rO"wR /4

0

[email protected]

26

Physicsguide

@Sk J ahiruddin , 2020

Magnetost atics

(b) See figure (b) above. T he total charge on t he shaded ring is dq = 0"(21rR sin 0) R d0. The time for one revolut ion is dt

= 21r /w .

So the current in the ring is I =

%; =

O"wR 2 sin 0d0 . The

11 oo : 01

: 13

(b) See figure (b) above. The total ch arge on the shaded

= a(2nR sin0)Rd0 . The

ring is dq is dt

=

21r /w

time for one revolution

.

So the current in the ring is I = ~~ = aw R 2 sin 0d0. The area of t he ring is 1r(R sin 0) 2 , so the magnetic moment of

= (awR 2 sin 0d0) 1r R 2 sin2 0,

t he ring is dm

and the total

dipole moment of the shell is 7r

4 m = awn R

4 sin 0d0 = ( 4/ 3)aw1r R 3

0

Hence ill =

41r CJW R 4"Z

3

Dipole term of the vector potential is . ( ) _ µ 0 m x r _ µ 0 m sin 0 ;;._ A dip r - - - -2 - '-P 2 41r r 41r r In this problem 4

A . _ µo 41r

R 4sin 0 J., _ µoaw R sin 0). 41r 3 aw r2 \f' 3 r2 '-P

dip -

j a [email protected]

@Sk J a hiruddin , 2020

1.6.1

27

Physicsguide

J\/fagnetostatics

Torques and Forces on Magnetic Dipoles

Torn11 P. on

;::i.

mFJJ!nP.tir rlinolP. of rlinol P. mom P.nt. m. w hil P.

11 oo : 01

: 1s

@Sk J ahiruddin , 2020

1.6.1

l\tlagnetostatics

Torques and Forces on Magnetic Dipoles

Torque on a magnetic dipole of dipole moment m while placed in magnetic field B

N = m xB

(1. 26)

Force on a magnetic dipole is zero when placed in a uniform field. But when placed in a non-uniform field t hen there is a net force acts F = V (m · B ) (1.27) Potential energy of a magnetic dipole while placed in a magnetic field is U = - m ·B (1.28)

1. 7

Electric Current

The conceptually simplest example of an electric current is t he current in a thin conducting wire. In an ideal onedimensional wire the current I is defined as t he net charge passing a point P per unit time. In a real wire I is defined as t he charge per unit time passing through a cross section

[email protected]

@Sk J ahiruddin , 2020

of the wire at P.

28

Physicsguide

Magnetostatics

11 oo : 01

: 1a

@Sk J ahiruddin , 2020

Magnetostatics

of the wire at P . (1.29)

The unit of current is the ampere (A), which is the basic electric unit in t he SI system. If t he wire carries a current of 1 A at point P, t hen 1 C of net charge passes P each second. If t he current in a wire is due to charges q moving with mean velocity v, and t he charges have linear density nL ( = number of charge carriers per unit length) , then

(1.30)

In n is the charge carrier density per unit volume and A is t he surface area of t he current carrying wire, q is t he amount of charge each particle carries, t hen t he current is

I = qnAv

(1.31)

What is the current for an orbit ing electron? Well t he charge is e, velocity is v and the radius of atom is taken to be R . Then t he electron crosses a point once in one t ime period. So t he current is

j [email protected]

@Sk J ahiruddin , 2020

29

Physicsguide

l\lfagnetostatics

11 oo : 01

: 21 T

[email protected]

29

@Sk J ahiruddin , 2020

P hysicsguide

Magnetostatics

As T is 21r R / v , t he current I =

ev 21rR

(1.32)

You will need t his expression many t imes

1. 7 .1

Current Density

If the current in a volume of space is due t o charges q with volume number density n ( = number of charge carriers per unit volume) moving wit h mean velocity v , then t he current density is J = qnv (1.33) Note t hat the unit s of qnv are A/ m 2 . Current density is t he flux of electric charge. In general flux is equal to density t imes velocity.

1. 7.2

Conservation of Charge

The net charge of an isolat ed syst em is constant. But charge is not only conserved overall in a syst em , it is also conserved point by point t hroughout t he syst em . This local conservat ion of charge is described mathematically by t he continuity [email protected]

30

P hysicsguide

11 oo : 01

: 23

point by point throughout the system. This local conservat ion of charge is described mathematically by the continuity j [email protected]

Physicsguide

30

@Sk J ahiruddin, 2020

J\/fagnetostatics

equation

V. J = - fJp fJt

(1.34)

Here p(x , t) is the volume charge density (=charge per unit volume), and J (x , t) is the volume current density (= current per unit area). Equation (1 .34) is universally true, for arbitrary time dependence. Integral form of the continuity equation,

d

s

J · dA = - -

dt

(1.35)

states that the rate of charge passing outward through the closed surface S is equal to the rate of decrease of charge in the enclosed volume V. Equation (1.34), or equivalently (1.35) , is a basic equation of electrodynamics, expressing local conservation of charge.

1.7.3

Ohm's Law

How is the current related to t he potential gradient in a conductor? If two terminal points on a conductor are held at a constant potential difference V , e.g., by connecting them to the electrodes of a bat tery, then in equilibrium a steady

11 oo : 01

: 26

How is the current related to the potential gradient in a conductor? If two terminal points on a conductor are held at a constant potential difference V , e.g. , by connecting them to the electrodes of a battery, then in equilibrium a steady 31

[email protected]

@Sk J ahiruddin , 2020

Physicsguide

Magnetostatics

current flows through the conductor. Let I be the total current at either point, i.e., the integrated flux J J · dA t hrough a surface inside the conductor surrounding the point . It is found empirically t hat formany cases t he current and potent ial difference are proportional,

V = IR

(1.36)

The constant of proportionality R is called the resist ance of t he conductor. The SI unit of resistance is the ohm (D), defined by ID = 1V / A . The reciprocal 1/ R is called the 1 conductance, and its unit is n- , or siemens (S) . Ohm's law holds to a very good approximation for many conductors. However, it is not a universal principle, there are examples where it does not hold. The resistance R of a sample of matter is a function of t he geometry (size and shape) of the sample, and of the material composition . For example, the resistance of a uniform cylinder, of length L and cross section A , is proportional to L and inversely proport ional to A ,

R -

nT I Ll

/ 1 ~7 \

11 oo : 01

: 29

cylinder, of lengt h L and cross section A , is proport ional to L and inversely proport ional t o A ,

R = pL / A

(1.37)

The parameter p (not to be confused with charge density 32

j [email protected]

Physicsguide

@Sk J ahiruddin , 2020

l\lfagnetostatics

p(x ) ! is an int rinsic property of t he material called t he resistivity. We may also write a local form of Ohm's law, which is a more basic equation.

J (x ) = aE (x )

(1.38)

Again, a is the conductivity.

Example Two long coaxial metal cylinders (radii a and b) are separat ed by material of conduct ivity a as shown in figure below . If they are maintained at a potent ial difference V , what current flows from one to the other , in a length L? E

a

----------- ---------------

\

I I I

I I

b------ - - - -- -- - - - ----- --- - - J

L

11 oo : 01

: 31

b--------------------------J

L

Figure 1.11:

[email protected]

Physicsguide

33

@Sk J ahiruddin , 2020

1\/Jagnetostatics

Solution: The field between the cylinders is

where A is t he charge per unit length on the inner cylinder. The current is

I=

J- da = a

E · da =

a -AL

Eo

The integral is over any surface enclosing t he inner cylinder. The potential difference between t he cylinders is a

V= b

A E· dl = - ln 27rEO

21raL

I = ln(b/a) V So the resistance of the syst em is

R = ln(b/ a) 21raL

b a

11 oo : 01

: 34

I = ln(b/ a ) V So the resist ance of the syst em is

R = ln(b/ a) 21raL

[email protected]

34

@Sk J ahiruddin , 2020

2

Physicsguide

Magnetostatics

Magnetic Field in a medium

Electric fields are creat ed by charges; magnetic fields are created by currents. We learned in our first course that the simplest way t o characterise any localised current distribut ion is through a magnetic dipole moment m . For example, a current I moving in a planar loop of area A with normal ft has magnetic dipole moment, m

= ! Aft

Magnet ic vector potent ial and magnetic field due to a magentic dipole are

A (r) = µo m x r => B (r) = µo 41r r 3 41r

3(m • f )f - m r3

(2.1)

11 oo : 01

: 37

gentic dipole are

A (r)

=

µo m x r =;- B (r) 4n r 3

=

µo 4n

3(m • f )f - m r3

(2.1)

Current loops, and their associated dipole moments, already exist inside materials. They arise through two mechan1sms: •

• Electrons orbiting the nucleus carry angular momentum and act as magnetic dipole moments. • Electrons carry an intrinsic spin. This is purely a quanj [email protected]

@Sk J ahiruddin, 2020

35

Physicsguide

l\tlagnetostatics

t um mechanical effect. This too contributes to t he magnetic dipole moment. We define t he magnetisation M to be t he average magnetic dipole moment p er unit volume. In most (but not all) materials, if there is no applied magnetic field then the different atomic dipoles all point in random direct ions. This means t hat, after averaging, (m) = 0 when B = 0. However , when a magnetic field is applied , t he dipoles line up. The magnetisation typically takes t he form M ex B. We' re going to use a slightly strange notation for the proport ionality constant . (It's historical but , as we'll see, it turns out to simplify a later equation)

11 oo : 01

: 39

applied, the dipoles line up. The magnetisation typically takes t he form M ex B. We're going t o use a slight ly strange notation for t he proport ionality constant . (It's historical but, as we'll see, it t urns out to simplify a later equation)

M = 1 Xm B µo l + Xm

(2.2)

where Xm is the magnetic susceptibility. The magnetic properties of materials fall into t hree different categories. The first two are dictated by t he sign of Xrri : • Diamagnetism: - 1 < Xm < 0. The magnetisation of diamagnetic materials points in t he opposite direction t o the applied magnetic field. Most metals are [email protected]

@Sk J ahiruddin, 2020

36

P hysicsguide

Magnetostatics

netic, including copper and gold. Most non-metallic materials are also diamagnetic, including, import antly, water with Xm ~ -10- 5 . This means, famously, that frogs are also diamagnetic. Superconductors can be t hought of as " perfect" diamagnets wit h Xm = -1 . • Paramagnetism: Xm > 0. In paramagnets, t he magnetisation points in the same direction as t he field. There are a number of paramagnetic met als, including Tungst en, Cesium and Aluminium. • Ferromagnetism: M i= 0 when B = 0. Materials with t his property are what you usually call '' magnets'' . They're

11

00:01 :41

are a number of paramagnet ic metals, including Tungst en, Cesium and Aluminium. • Ferromagnetism: M # 0 when B = 0. Materials with t his property are what you usually call '' magnets'' . They're t he t hings stuck to your fridge. The direction of B is from t he sout h pole to the nort h. Only a few elements are ferromagnetic. The most familiar is Iron. Nickel and Cobalt are other examples.

2.1

Bound Currents

When a material becomes magnetised (at least in an anisotropic way), t here will necessarily be regions in which there is a current. This is called the bound current. [email protected]

Physicsguide

37

@Sk J ahiruddin , 2020

Magnetost atics

Let 's first give an intuitive picture for where t hese bound currents appear from. Consider a bunch of equal magnet ic dipoles arranged uniformly on a plane like t he left pict ure : The currents in t he interior region cancel out and we're left only wit h a surface current around t he edge. We know t hat t his is surface current a K. We'll follow this notation and call the surface current arising from a constant, internal magnet isation K bound· M

M

11 oo : 01

: 44

and call the surface current arising from a constant, internal magnetisation K bound . M

M

Bound surface current

Bound surface and volume current

Figure 2.1: Origin of bound surface and volume current

Now consider instead a sit uation where the dipoles are arranged on a plane, but have different sizes. We'll put t he big ones to t he left and the small ones to t he right, like In t his case, the currents in the interior no longer cancel. As we can see from the right side of the picture pict ure, t hey go into the page. since M is out of the page, and we've arranged t hings so that M varies from left to right , this suggests that j [email protected]

Physicsguide

38

@Sk J ahiruddin , 2020

J

bound

f"..J

v7

X

J\/fagnetostatics

M.

Bound surface and volume currents are K b ound

=M

ft

(2.3)

M

(2.4)

X

and J bound

= v7

X

Note that t he bound current is a steady current, in the

11 oo : 01

: 47

and J bound

=V

X

M

(2.4)

Note that t he bound current is a steady current, in the sense that it obeys V · J bou11d = 0 .

2.2

Magnetic intensity

Recall t hat Amp'ere's law describes the magnetic field generated by static currents. We've now learned t hat, in a material, there can be two contribut ions to a current: the bound current J bound that we've discussed above, and t he current J free from freely flowing electrons that we were implicit ly talking. Amp'ere's law does not distinguish between t hese two currents; the magnetic field receives cont ributions from both. V

X

B

[email protected]

= µo (J free + J bound ) = µoJ free + µo V x M 39

@Sk J ahiruddin , 2020

(2. 5) P hysicsguide

Magnetostatics

We define the Magnetic Intensity or magnetising field, H as

(2.6) This obeys

V x H=J free and in integral form

(2.7)

11 oo : 01

: 49

H = -B-M µo

(2.6)

V x H = J rree

(2.7)

This obeys and in integral form

H ·di = Irenc

(2. 8)

We see that the field H plays a similar role to the electric displacement D ; the effect of the bound currents have been absorbed into H , so that only the free currents cont ribute. Note, however, that we can't quite forget about B entirely, since it obeys V · B = 0 . In contrast, we don 't necessarily have '' V · H = O'' . Rather annoyingly, in a number of books H is called t he magnetic field and B is called t he magnetic induction. But this is stupid terminology so we won't use it.

2.2.1

Linear magnetic material

For most of t he magnetic material the magnetization vector and magnetic intensity are proportional. They are called j [email protected]

@Sk J ahiruddin , 2020

40

P hysicsguide

l\lfagnetostatics

linear magnetic material

(2.9) The constant of proport ionality Xm is called the magnetic

11 oo : 01 : s2 linear magnetic material

(2.9) The constant of proportionality Xm is called the magnetic susceptibility; it is a dimensionless quantity t hat varies from one substance to another - positive for paramagnets and negative for diamagnets. From the definition of H

or B =µH

(2.10)

where (2.11)

µ is called the permeability of the mat erial.

Do some example from Griffiths.

[email protected]

@Sk J ahiruddin , 2020

2.3

41

Physicsguide

1\/Jagnetostatics

Electromagnetic Boundary conditions

11 oo : 01 : s4 @Sk J ahiruddin, 2020

J\/fagnetostatics

2.3

Electromagnetic Boundary conditions

2.3.1

Magnetostatic Boundary condition inside a medium

Boundary conditions inside the magnetic medium. (2.12)

and II H above

-

- K

II

H below -

A

f X n

(2.13)

t his can also be written as in terms of B

11 B above

2.3.2

-

11 B below -- µo (K

X

n")

(2. 14)

Boundary condition in both electric and magnetic field

In the presence of surface charge, the electric field normal to t he surface is discontinuous, while the electric field tangent to the surface is continuous. For magnetic fields, it's t he other way around: in the presence of a surface current, the magnetic field normal to t he surface is continuous while t he magnetic field tangent to the surface is discontinuous. [email protected]

@Sk J ahiruddin, 2020

42

Physicsguide

Magnetostatics

11 oo :01 : s1

@Sk J ahiruddin , 2020

Magnetostatics

What happens with dielectrics? Now we have two opt ions of the electric field, E and D , and two options for the magnetic field, B and H . They can 't both b e continuous because they're related by D = EE and B = µH and we'll be interested in sit uation where E( and possibly µ) are different on either side. Nonetheless, we can use the same kind of computations t hat we saw previously to derive the boundary conditions. Roughly, we get one boundary condit ion from each of the Maxwell equations.

The tangential component of the electric field Is continuous.

The normal component of the electric field is discontinuous

Figure 2.2: Boundary conditions of electric field

For example, consider t he Gaussian pillbox shown in the left-hand figure above. Integrating t he Maxwell equation V · D = p free t ells us that the normal component of D is discontinuous in the presence of surface charge, (2.15) j [email protected]

(c)Sk J a,hir11ddin. 2020

43

Physicsguide

l\tlae:netostatics

11 oo : 02 : oo 43

j [email protected]

P hysicsguide

@Sk J ahiruddin , 2020

l\lfagnetostatics

where n is the normal component pointing from 1 into 2. Here CJ refers only to the free surface charge. It does not include any bound charges. Similarly, integrating V · B = 0 over t he same Gaussian pillbox tells us that the normal component of the magnetic field is cont inuous. (2.16)

To det ermine the tangential components, we integrate t he appropriate field around the loop shown in the righthand figure above. By Stoke's theorem, this is going to be equal to t he integral of the cur1 of t he field over the bounding surface. This tells us what the appropriate field is: it 's whatever appears in the Maxwell equations with a curl. So if we integrate E around the loop, we get the result

nx

(E 2 -

E 1)

=

o

(2.17)

Meanwhile, integrating H around t he loop tells us t he discontinuity condit ion for t he magnetic field (2.18)

where K is the surface current.

[email protected]

44

Physicsguide

11 oo : 02 : 02 where K is the surface current.

44

jahir@physicsguide. in

Physicsguide

@Sk J ahiruddin, 2020

2.3.3

1\/Jagnetostatics

Example: Field inside the gap

Here is a very interesting problem. AC-magnet is shown in Figure . All dimensions are in cm. The relative permeability of t he soft Fe yoke is 3000 . If a current I = 1 amp is to produce a field of about 100 gauss in t he gap, how many t urns of wire are required? Fe

T9 1

20 1

2

1 7~

1--7

T 9 1

Figure 2.3: Magnetic field in the gap

Solution: You must know t he boundary condit ion of t he magnetic field. Consider t he surfaces which act like the cross section of t he magnet (red colored in t he image) . The

11 oo : 02 : os Solution: You must know t he boundary condit ion of t he magnetic field . Consider t he surfaces which act like t he cross section of t he magnet (red colored in t he image) . The

[email protected]

P hysicsguide

45

@Sk J ahiruddin, 2020

Magnetostatics

normal component of B is continuous

And there are no parallel component of Magnetic field (remember t hat in a solenoid t he field is along the axis only) . So, t he magnetic intensity in the gap is H = J}_, while that µo inside t he magnet is H = µoµ B , where µr is t he relative perr meability of the iron. Ampere's circuital law

H · dl = N I applied to the closed loop passing through inside the magnet and covering the whole magnet

hence 1 d + -(4l - d) µr now apply the given data - l = 20 cm = 0.2 m , d = 2 cm = 0.02 m, and B = 100 gauss = 100 x 10- 4 Tesla.

N

= 100

X

10-

4

Ll.'71" >< 1 n-1 >< 1

0.0

2+

0.2

X

4 - 0.02

~nnn

11 oo : 02 : 01 j [email protected]

Physicsguide

45

@Sk J ahiruddin , 2020

Magnetostatics

normal component of B is continuous

And there are no parallel component of Magnetic field (remember that in a solenoid t he field is along the axis only) . So, the magnetic intensity in the gap is H = J}_ , while that µo inside t he magnet is H = µoµr B , where µr is t he relative permeability of the iron. Ampere's circuital law

H · dl = N I applied to the closed loop passing through inside the magnet and covering the whole magnet

hence 1 d + -(4l - d) µr now apply the given data - l = 20 cm = 0.2 m , d = 2 cm = 0.02 m, and B = 100 gauss = 100 x 10- 4 Tesla.

N

= 100 41r

X

4

1010- 7 X 1 X

0.0

2

+

0.2

4 - 0.02 3000

X

= 161 t urns j [email protected]

46

Physicsguide

11 oo : oo : oo

Electromagnetic Induction Sk J ahiruddin Assistant Professor Sister Nibedita Govt. College, Kolkata Author was t he topper of IIT Bombay M.Sc Physics 2009-2011 batch He ranked 007 in IIT JAM 2009 and 008 (JRF) in CSIR NET J t1ne 2011 He has been teaching CSIR NET aspirants since 2012

Contents 1 Changing Electric and Magnetic Field

3

1

@Sk J ahiruddin, 2020

Electromagnetic Induction

11 oo : oo : 03 1

@Sk J ahiruddin , 2020

Electromagnetic Induction

Motional emf .

































3

1.2 Faraday 's La~r .

































3

1.3 Induct ance . . .

































13

1.3.1

Self Inductance . .





















13

1.3.2

Mutual Inductance





















15





















18

1.1

1.4

Magnetostatic Energy

j [email protected]

2





Physicsguide

11 oo : oo : 06

j ahir@physicsguide. in

2

@Sk J ahiruddin , 2020

1

1.1

P hysicsguide

Electromag11etic Induction

Changing Electric and Magnetic Field Motional emf

When charge moves there acts the Lorenz force on t hem F

= qE + qv x B

(1.1)

The charges move according t he force experienced by t hem. The motion of t he charges generates a current and t hus an emf on t he medium in which t he charges are moving. This is called motional emf. Lorentz force law is very useful to determine t he direct ion of t he induced current . To calculate t he magnitude we need to know t he Faraday's law of induction.

1.2

Faraday's Law

When the magnetic flux ( = J B · dS) somehow changes in a closed conducting system (be it a closed loop of wire or

11 oo :oo :oa When the magnetic flux ( q> = J B · dS) somehow changes in a closed conducting syst em (be it a closed loop of wire or [email protected]

Physicsguide

3

@Sk J ahiruddin , 2020

Electromagnetic Induction

somet hing a bend and closed rod) t here creat es an emf

(1.2) The significance of t he negative sign is, t he induced emf t ries t o oppose t he change of the magnetic flux . The basic principle t he Faraday's law states is: A changing magnetic field induces an electric field.

E · dl = - dq> = -

dt

aB. da

at

(1.3)

This is Faraday's law, in int egral form. We can convert it to differential form by applying Stokes; theorem: \7

X

E= -

aB

at

(1.4)

This equation t ells us t hat if you change a magnetic field , you'll creat e an electric field. In t urn, this electric field can be used to accelerate charges which , in this context , is usually thought of as creating a current in wire. The process of creat ing a current t hrough changing magnet ic fields is called induct ion. Faradav's law tells us t hat if vou chan e:e the mae:netic

11 oo : oo : 11 usually thought of as creating a current in wire. The process of creating a current t hrough changing magnetic fields is called induction. Faraday's law tells us that if you change the magnetic flux t hrough S then a current will flow. There are a number [email protected]

@Sk J ahiruddin, 2020

4

Physicsguide

Electromagnetic Induction

of ways to change t he magnetic field. You could simply move a bar magnet in the presence of circuit, passing it through the surface S; or you could replace t he bar magnet with some other current density, restricted to a second wire C', and move t hat; or you could keep the second wire C' fixed and vary the current in it, perhaps turning it on and off. All of these will induce a current in C. However, there is then a secondary effect . When a current flows in C, it will create its own magnetic field. This induced magnetic field will always be in the direction that opposes the change. This is called Lenz's law. If you like, " Lenz's law'' is really just the minus sign in Faraday's law. Example A metal bar of mass m slides frictionlessly on two parallel conducting rails a distance l apart (see figure below) . A resistor R is connected across t he rails, and a uniform magnetic field B , pointing into the page, fills the entire region.

11 oo : oo : 14 uniform magnetic field B , pointing int o the page, fills the entire region.

j [email protected]

P hysicsguide

5

@Sk J ahiruddin, 2020

Electromagnetic Induction

V

m

Figure 1.1: Current creat ed by moving bar in a magnetic field

(a) If the bar moves to the right at speed v, what is the current in the resistor? In what direction does it flow? (b) What is the magnet ic force on the bar? In what direction? (c) If the bar starts out wit h speed v 0 at time t = 0, and is left to slide, what is its speed at a later t ime t ? (d) The initial kinetic energy of t he bar was, of course,

11 oo : oo : 16 (c) If the bar starts out with speed v 0 at time t = 0, and is left to slide, what is its speed at a later time t ?

~

(d) The initial kinetic energy of the bar was, of course, Check that the energy delivered to t he resistor is

mv5.

exactly

~mv5.

Solution: (a) E = -~if!t = - B l9lf = B lv·E = IR dt ' ⇒ I = BAv. The minus sign tells you t he direction of flow [email protected]

Physicsguide

6

@Sk J ahiruddin, 2020

Electromagnetic Induction

of the current. (v x B) is upward, in the bar, so t he current is downward through the resistor . (b) F

= IlB =

2

to the left.

B~ v,

v . Hence V

= Voe -

B2l2

mR

t

(d) The energy goes into heat in the resistor. The power 2

delivered to resistor is 1 R , so dW - I 2R - B 2z2v2 R - B 2z2

dt -

we have taken a

R2

-

2 2

B l • rnR'

So

dW

dt

-

2 - 20.t

R Vo e

= amv02 e- 2at

The total energy delivered to the resistor is oo

e- 2atdt =

e-2at oo amv02 - -

- 2a

n

= a.mv 2 -

1

°2a

1

= -mv 2 2

°

11 oo : oo : 19 we have taken a = -

2 2

B l



mR '

So

dW

dt

= amv02e- 2at

The total energy delivered to the resistor is W

oo

2 =am v0

e- 20.tdt

1

e - 20.t oo

=

0

amv02 - - 2a

= 0

amv 02 2a

1

=

-mv 2 2

°

Example: Faraday's disk generator: A metal disk of radius a rotates with angular velocity w about a vertical axis, t hrough a uniform field B , pointing up. A circuit is made by connecting one end of a resistor to the axle and the other end to a sliding contact , which touches the outer edge of the disk (See figure) . Find the current in the resistor. [email protected]

P hysicsguide

7

@Sk J a hirudd in , 2020

Electromagn etic In duction

B

B (Sliding contact)

I R

Figure 1.2: Current created by rotating metal disk

Solution: The speed of a point on the disk at a distance s from t he axis is v = ws , so t he force per unit charge is f mag = v x B = wsB§. The emf is therefore ra

ra

, ,D ,.,,2

11 oo : oo : 22 Solution: The speed of a point on the disk at a distance s from the axis is v = ws , so t he force per unit charge is f mag = v x B = wsB§. The emf is t herefore a

a

fmagds = wB 0

0

So t he current is

wBa 2 sds = - 2 2

I = ~ = wB a R 2R The flux law or Faraday-Letz rule can also be written as in terms of electric field E · dl

j [email protected]

= - d dt

(1.5)

Physicsguide

8

@Sk J ahiruddin, 2020

Electromagnetic Induction

Example: A uniform magnetic field B (t), pointing straight up , fills t he shaded circular region, made by conducting material, as shown in t he figure below. If B is changing with t ime, what is the induced electric field?

B(t)

I

\

-- -- ..... ' ..... _-- S

\

I

11 oo : oo : 24 I

\

'

.....

_--

S

\

"-"" I ......

Amperian loop Figure 1.3: Induced Current by changing magnetic field

Solution: E points in the circumferential direction, just like t he magnetic field inside a long straight wire carrying a uniform current density. Draw an Amperian loop of radius s, and apply Faraday's law: d1> d 2 2 dB E · dl = E(21rs) = - dt = - dt (1rs B (t)) = -1rs dt [email protected]

@Sk J ahiruddin, 2020

9

Physicsguide

Electromagnetic Induction

Hence

E = -~dB 1J 2 dt If B is increasing, E runs clockwise, as viewed from above. Example: A line charge A is glued onto t he rim of a

wheel of radius b,. T he spokes are made of some nonconducting material. The wheel is then suspended horizontally, as shown in figure below so t hat it is free to rotate. In the central region, which is made by conducting material, out to radius a, there is a uniform magnetic field B 0 , pointing up. Now someone turns the field off. What happens?

11 oo : oo : 26 ducting material. The wheel is then suspended horizontally, as shown in figure below so that it is free to rotate. In the central region, which is made by conducting material, out to radius a, there is a uniform magnetic field B o, pointing up. Now someone turns the field off. What happens?

j [email protected]

10

P hysicsguide

@Sk J ahiruddin , 2020

Electromag11etic Induction

E l

11 oo : oo : 29

E Rotation direction

di

Figure 1.4: Charged disk rotates because of changing B

solution: The changing magnetic field will induce an electric field, curling around the axis of the wheel. This

electric field exerts a force on the charges at the rim, and the wheel st arts t o t urn. According t o Lenz's law, it will rotat e in such a direction t hat its field tends to restore t he upward flux. The mot ion, then, is counterclockwise, as viewed from above. Faraday's law, applied to the loop at radius b , says

[email protected]

@Sk J ahiruddin , 2020

dif> E-dl = E (21rb) = - - = dt

Physicsguide

11

Electromagnetic Induction

-

2 dB 1ra dt '

2

or

E = _a dB 4J 2b dt

The torque on a segment of length dl is (r x F) , or b)..E dl. The total torque on t he wheel is therefore

11 oo : oo : 32 2b dt

T he torque on a segment of length di is (r x F ), or b>..E dl. The total torque on t he wheel is t herefore

N

a 2 dB 2b dt

= b>..

dl = -b>..1ra 2 dB dt

T he angular moment um imparted to t he wheel is 0

N dt

=

->..1ra2 b

dB

=

>..1ra 2 bB0

Bo

It doesn 't matter how quickly or slowly you turn off the field; t he resulting angular velocity of t he wheel is the same. Now the quest ion is - where is the angular momentum coming from? wait for the next section.

[email protected]

@Sk J ahiruddin , 2020

1.3 1.3.1

12

Physicsguide

Electromagnetic Induction

Inductance Self Inductance

Suooose that a constant current I flows along some curve

11 oo : oo : 34 ance 1.3.1

Self Inductance

Suppose that a constant current I flows along some curve C. T his gives rise to a magnetic field and hence a flux = J8 B · dS t hrough t he surface S b ounded by C . Now increase t he current I . This will increase t he flux . But we've just learned t hat t he increase in flux will, in t urn, induce an emf around t he curve C. T he minus sign of Lenz's law ensures t hat this acts to resist t he change of current. T he work needed to build up a current is what 's needed to overcome t his emf. If a current I flowing around a curve C gives rise to a flux = J8 B · dS t hen t he inductance L of t he circuit is defined to be

(1.6) The inductance is a property only of our choice of curve C . Example: Self inductance of a Solenoid: A solenoid consists of a cylinder of length l and cross-sectional area A . We take l >> -JA so t hat any end-effects can be neglected. A wire wrapped around t he cylinder carries current I and j [email protected]

13

P hysicsguide

@Sk J ahiruddin, 2020

Electromagnetic Induction

winds N t imes per unit length. The magnetic field t hrough t he cent re of t he solenoid t o be T 1\ T

I

-t

,-, \

00: 00: 36 Electromagnetic Induction

winds N t imes per unit length. The magnetic field t hrough t he cent re of the solenoid to be

B = µoIN

(1. 7)

Figure 1.5:

This means that a flux through a single turn is 1> 0 = µ 01 NA. The solenoid consists of Nl t urns of wire, so the total flux is with V = Al the volume inside t he solenoid. The inductance of the solenoid is there£ore

[email protected]

14

@Sk J ahiruddin , 2020

1.3.2

Mutual Inductance

Physicsguide

Electromagnetic Induction

11 oo : oo : 39 @Sk J ahiruddin, 2020

1.3.2

Electromagnetic Induction

Mutual Inductance

Say there are two loops placed in reasonably closed distance and current 11 passes through loop 1. The current will create magnetic field. T hat magnetic field will create magnetic flux around the second loop (loop 2) . It can be proved t hat the flux flown in the second loop due to current in 1st loop is proportional to I 1 .

~Loop2

~---t-r-:::::

~Loopl

Figure 1.6: Mutual inductance

[email protected]

@Sk J ahiruddin , 2020

15

Physicsguide

Electromagnetic Induction

11 oo : oo : 42 [email protected]

15

@Sk J ahiruddin , 2020

Physicsguide

Electromagnetic Induction

(1.8) where M 21 is the constant of proportionality; it is known as the mutual inductance of t he two loops.

M 21 is a purely geometrical quantity, having to do wit h t he sizes, shapes, and relative positions of the two loops. If we reverse the situation: current 12 passes t hrough loop 2. The current will create magnetic field. That magnetic field will create magnetic flux around loop 1. It can be proved that the flux flown in the loop 1 due to current in loop 2 is proportional to 12 with the same proportionality constant . i.e

(1.9) and

(1.10) Whatever the shapes and positions of the loops, the flux t hrough loop 2 when we run a current I around loop 1 is ident ical to t he flux through loop 1 when we send t he same current I around loop 2. We drop the subscripts and call t hem both M .

j ahir@physicsguide. in

16

Physicsguide

11 oo : oo : 44 t hem both M .

j [email protected]

Physicsguide

16

@Sk J ahiruddin , 2020

Electromagnetic Induction

Example: A short solenoid (length l and radius a, with n 1 turns per unit length) lies on t he axis of a very long

solenoid (radius b, n 2 t urns per unit length ) as shown in Figure below. Current I flows in the short solenoid. What is the flux t hrough t he long solenoid? What is t he mutual inductance of the system?

Solution: Since the inner solenoid is short , it has a very complicated field; moreover , it puts a different flux t hrough each turn of the outer solenoid. It would be a very tough task to compute the total flux this way. However, if we use t he equality of the mutual inductances, the problem becomes very easy. Just look at the reverse sit uation: run the current / t hrough the outer solenoid, and calculate the flux through t he inner one. The field inside t he long solenoid is constant:

B = µ 0n 2 I so t he flux through a single loop of the short solenoid is 2

B1ra = µ 0n 2 I 1ra

2

There are n 1 l t urns in all, so the total flux t hrough the inner solenoid is

11 oo:oo:47 There are n 1 l t urns in all, so the total flux through the inner solenoid is

[email protected]

Physicsguide

17

@Sk J ahiruddin , 2020

Electromagnetic Induction

This is also the flux a current J in the short solenoid would put through the long one, which is what we set out to find. Incidentally, the mutual inductance, in this case is

1.4

Magnetostatic Energy

The definition of inductance is useful to derive the energy stored in the magnetic field. Let's take our circuit C with current I. We'll t ry to increase the current. The induced emf is E == _ dct> == - L dI dt dt As we mentioned above, the induced emf can be thought of as the work done in moving a unit charge around the circuit. But we have current I flowing which means that, in time bt, a charge I bt moves around the circuit and the amount of work done is

dI bW == Eibt == - LI dt bt dW == - LidI == _ LdI J.L

J.L

C")

2

J.L

11 oo : oo : 49 of work done is dI et

where k is a constant, and e = 1/ ,FoJio

Solution:

Well' t h is can be taken as a reverse type

of problem where we will calculate t he charge and current distribut ions from t he given potent ials. In generally we cal,

'

. ,

.

.

11 oo : 01 : os Solution: Well' this can be taken as a reverse type of problem where we will calculate the charge and current distributions from the given potentials. In generally we calculate t he opposite. First we'll det ermine t he electric and m agnet ic fields, using B = v' x A and E = - v'V -

E=-

8A

at

~!

µok ,, = - 2 (et - xl)z

and

B = "v

X

A = -µo -k 8 (et4e 8 x

X

) 2y,, =±-etµok ( 2e

X

)y,,

(plus, for x > O; minus, for x < 0). These are for x < et; when lxl > et E = B = 0. see figure below. Calculating every derivative in sight, We find j [email protected]

Physicsguide

24

@Sk J ahiruddin , 2020

JVIaxwell's equations

-et

et X

~ket 2

II

1,-~

11 oo : 01 : oa X

~kct 2

~kt 2

-et X

Figure 3 .1: charge and current from given potent ials

and

aE

at

µokc,_ = - 2 z;

_aB _ _ ± -µo_ky"

at

2

As you can easily check, Maxwell's equations are all satisfied , with p and J both zero. Notice, however , that B has [email protected]

25

P hysicsguide

@Sk J a hiruddin, 2020

Maxwell's equations

a discont inuity at x = 0 , and this signals the presence of a surface current K in the yz plane; boundary condition 1...B I 1...B I = K 1 x n gives µ1 1 µ2 2

kt y = K

X

and hence

K

= ktz

X

11 oo : 01 lµ1 B 1II - lµ2 B 2II

: 1o

= K J x n gives kty = K

X

X

and hence

K

3.2

= ktz

Gauge Transformation

From the relations of fields and potentials you can easily verify t hat if you change the electric and magnetic vector potentials according to the following relation t hen t he elect ric and magnetic field remains unchanged.

A'= A + v'A V' = V- 8A

(3.8)

at

A is a scalar function of position and time A = A(x .y.z, t) Gauge transformations are broadly classified into Coulomb's Gauge and Lorentz 's gauge. The Gauge depends on t he choice of A Coulomb's Gauge:

(3.9) [email protected]

@Sk J ahiruddin , 2020

26

P hysicsguide

Maxwell's equations

In which case we apply Coulomb's Gauge? Of course in static configuration. Where the charges, currents and fields are not changing with respect to time. What is t he condit ion of A? Well the gauge choice tells us

11 oo : 01

: 13

In which case we apply Coulomb's Gauge? Of course in static configuration. Where the ch arges, currents and fields are not changing with respect to time. What is t he condition of .X? Well the gauge choice tells us t hat v7 · A = 0 for both b efore and after t h e transformation of the potentials. Take divergence both side of the relation A' = A

+ v7 .X.

You get

v7 · A' = v7 · A

+ v7 · v7 .X

hence as both v7 · A = 0 and v7 · A' = 0. We get 2

v7 .X = 0 That is the choice of .X of Coulomb's gauge

Lorenz gauge:

av v7 ·A = -µoEo- -t 8

(3.10)

Where will we apply Lorentz gauge? Definitely in dynamic situation where t h e charge currents and fields changes with time. j [email protected]

@Sk J ahiruddin , 2020

27

Physicsguide

Maxwell's equations

And what is t he choice of .X? . Well t he relation v7 • A =

-µ 0 c0 ~~ is valid both before and after t he t ransformation . •

1.e

11 oo : 01

: 1s

@Sk J ahiruddin , 2020

Maxwell's equations

And what is t he choice of A? . Well t he relation V · A = -µ 0 c0 ~1; is valid both before and after t he t ransformation . •

1.e

V . A = - µoEo

av

av' V ·A'= -µoEo -

at ;

at

Now you again take divergence both side of t he relation A'= A + V A. You get

V · A' = V · A

+ V · VA

Use Lorentz gauge conditions to get

t hen apply gauge transformation basic criteria V'

=V-

-a>-. at

So we get

[email protected]

@Sk J ahiruddin, 2020

or

28

P hysicsguide

lVIaxwell's equations

11 oo : 01

: 19

@Sk J ahiruddin , 2020

Maxwell's equations

or

av -µoEo at

2

a A av + µoEo at2 = -µ oEo at

2

+V A

hence the choice of A in Lorentz gauge is

Now we explore more with some examples Does the following set of potent ials represents a physical electromagnetic field? Example:

A = (2x -wt)i+(y- 2z)]+ (z -

cp = 3xyz-4t;

2eiwt ) k

Solution: The most obvious way is to calculate t he Elect ric and magnetic field from the given potentials and check

whether they satisfy the Maxwell's equations. But the shortcut in t hese type of problems is to check the gauge condit ion. As t he potentials are t ime dependent then you check whether t he the Lorentz gauge V · A = -µ 0Eo ~~ condition is valid here. Which of the following t ransformations

Example:

(V, 1 ) >

>

(V' ,

A') of the electrostatic potential V and t he

j [email protected]

@Sk J ahiruddin , 2020

29

Physicsguide

Maxwell's equations

11 oo : 01

: 21

[email protected]

29

P hysicsguide

@Sk J ahiruddin , 2020

vector potential June 2015]

Maxwell's equations

A is a gauge transformation?

/

➔,



/

➔,



[NET

A

(a) (V = V +ax,A = A +atk) (b) (V

=

V

+ ax, A

=

➔,

/

A

A - atk)



A

(c) (V =V +ax, A = A+ati) (d) (V'

=

V

+ ax , 1 =

A- ati)

Solution: As t he transformation condit ion of t he potent ial is the same so you definitely get the A from there. As V' = V - 9ft, then A= -axt. Hence the vector potential

A' = A + VA; Example: The scaler and vector potent ials cp(;i:, t) and (;i:, t) are determined up to a gauge transformation

A

qJ , , qJ'

= qJ - ~ and

A , , A' = A + V (

where ( is an

arbitrary continuous and differentiable function of If we further impose the Lorenz gauge condit ion

;i: and t.

~.A+! it = 0 t hen a E;ossible choice for t he gauge function ~(;i:, t) is (where w, k are nonzero constants wit h w = c

Ji )

(a) cos wt cosh (c)cosh wt cos

[NET Dec 2014]

1.;i: 1. ;i:

[email protected]

1. ;i: 1. -t

(b) sinh wt cos (d )cosh wt cosh 30

Physicsguide

11 oo : 01

: 24

C



(a)cos wt cosh k .? (c)cosh wt cos

1.?

j [email protected]



(b )sinh wt cos k.? (d )coshwt cosh

1.? Physicsguide

30

@Sk J ahiruddin, 2020

Maxwell's equations

Solution: Check t he Lorentz gauge condition over >. (the t here )

2 a2t V t = µoEo at2

for all the opt ions. The ans is (d) . Don't for get to use

1 2 C =-µoEo

4

w

2

k2

Momentum in EM Field

4.1

Poynting's theorem

We know that ELECTROMAGNETIC ENERGY DENSITY, i.e, the energy stored in electromagnetic fields, per unit volume, 1s 1 U=2 •

Now, Suppose we have some charge and current configurat ion which , at t ime t, produces fields E and B . In t he next instant , dt , the charges move around a bit. Now the question is -- How much work, dW, is done by the electromagnetic forces acting on these charges, in

11 oo : 01

: 26

instant, dt , the charges move around a bit. Now the question is -- How much work, dW, is done by the electromagnetic forces acting on these charges, in [email protected]

@Sk J ahiruddin, 2020

31

Physicsguide

lVIaxwell's equations

t he interval dt? After some calculation we can find that the rate at which work is done on all the charges in a volume Vis

dvV dt

1 d dt v 2

s

(E x B ) · da (4.1)

where S is the surface bounding V. This is Poynt ing's theorem; it is the '' work- energy t heorem'' of electrodynamics. The first integral on the right is the total energy stored in the fields , J ud3r The second term evidently represents t he rate at which energy is t ransported out of V, across its boundary surface, by the electromagnetic fields. Poynting's theorem says, then, that the work done on the charges by the electromagnetic force is equal to the decrease in energy remaining in t he fields, less the energy that flowed out through the surface. The energy per unit t ime, per unit area, transported by t he fields is called the Poynting vector: 1 S - (Ex B ) (4.2) µo

11 oo : 01

: 29

The energy pe1~unit time, per unit area, transported by the fields is called the Poynting vector: 1 S _ - (E x B ) (4.2) µo S · da is the energy per unit time crossing the infinitesimal surface da - the energy flux. Generally, S is called the 32

j [email protected]

Physicsguide

@Sk J ahiruddin , 2020

Maxwell's equations

energy flux density). we can write Poynt ing theorem more compactly d

dW

S · da

dt

(4.3)

s

If there is no charge and current - empty space - no work is done on the charges -

au dT = at

~~ = 0 S-da= -

(V. S)dT

so we get

au =-V. S at

(4.4)

This is the '' cont inuity equation'' for energy - u (energy density) plays t he role of p ( charge density), and S takes the part of J (current density). It expresses local conservation of electromagnetic energy. Example: A long coaxial cable carries current I. The current flows down the surface of t he inner cylinder , radius

11 oo : 01

: 31

of elect romagnetic energy. Example : A long coaxial cable carries current I . The

current flows down t he surface of t he inner cylinder, radius a, and back along t he outer cylinder , radius b as shown in figure below.

[email protected]

Physicsguide

33

@Sk J ahiruddin , 2020

Maxwell's equations



b a

I

I -

Figure 4.1:

(a) Find t he magnet ic energy stored in a sect ion of length l.

(b) Now, assume that the two conductors are held at potential difference V, and carry current I . Calculate the power t ransported down the cables, Solution: (a) According to Ampere's law, the field between t he cylinders is

11 oo : 01

: 34

Solution: (a) According to Ampere's law, the field between the cylinders is B = µof

cp

21rs

outside the field is ZERO. Energy per unit volume is I

µ 0f

2µo

21rs

2

µ 0f

2

81r2s2

The energy in a cylindrical shell of length l , radius s, and [email protected]

Physicsguide

34

@Sk J ahiruddin , 2020

:Nlaxwell's equations

t hickness ds, then , is µof 2

ds s

81r2s2

Integrating from a to b, we have: W = µof 2z ln

b

a

41r

This energy calculation is also a very simple way to calculate t he self-inductance of t he cable. The energy can also be 2 written as ½Lf . Comparing the two expressions, you see

L = µol ln 21r

b a

(b) Take t he charge density of the inner cylinder to be A. Then t he electric field is (Apply Gauss law) A l

11 oo : 01

: 37

(b) Take t he charge density of the inner cylinder to be A. Then t he electric field is (Apply Gauss law)

E=

A



27rEQ S

We have already calculated the magnetic field

B = µo l!;p 21r s

So the Poynting's vector is

S

l (E x B ) =

=

µo

z

Al l 41r2Eo s2

35

j [email protected]

Physicsguide

JVIaxwell's equations

@Sk J ahiruddin , 2020

Poynting's vector represents the energy transported p er unit t ime p er unit area. So the Power (energy flow per unit time) is t he surface integral over the Poynting's vector b

S · da = a

Al S21rsds = 27rEo

b

1 Al -ds = ln(b/ a)

27rEo

a S

Now t he potential is b E.

V=

A

di=

27rEO

a

b

1 -ds

a S

=-

A

ln(b/a)

27rEO

Hence

P = IV ·

'

4.2

as it should be

Force on char!!es - Stress Tensor

11 oo : 01

: 39

Hence

P = IV ·

'

4.2

as it should be

Force on charges - Stress Tensor

Tot al electromagnet ic force on t he charges in volume V is

(E + v x B )pd r = 3

F= V

This can be calculated as (you don't need t he derivation. If you st ill want , t hen see Griffit hs.)

F=

d T -da - Eoµ os dt ++

[email protected]

3

Sd r

(4. 5)

V

P hysicsguide

36

@Sk J ahiruddin, 2020

Maxwell's equations

++

where T is t he Maxwell's st ress t ensor defined component wise as •

1

(4.6)

T he indices i and j refer to t he coordinates x, y, and z, so t he stress t ensor has a total of nine component s

(Txx, Tyy, Txz, Tyx, and so on) The Kronecker delta,

Oij,

is 1 if t he indices are t he same

11 oo : 01

: 42 x,

The Kronecker delta,

Oij,

is 1 if the indices are the same

and zero otherwise

Thus components can be calculated as

Txx = -l Eo ( E x2 - Ey2 - E z2) + - l ( B x2 - B y2 - B z2) 2 2µo 1 Txy = Eo (E xEy) + - (B xB y) µo Dot products of T with a vector a can be defined as (a · T )J· -.

~ a-~-· L....t 1., 1.,J '

.

i=x,y,z

i=x,y,z

[email protected]

P hysicsguide

37

@Sk J ahiruddin , 2020

Maxwell's equations

In the static case t he second term drops out, and the electromagnetic force on the charge configuration can be expressed entirely in te1~ms of t he stress t ensor at the boundary: F =

H

H

T -da

( static )

(4.7)

s

Physically, T is the force per unit area (or stress) acting on the surface. More precisely, ~ j is the force (per unit area) in the i t h direction acting on an element of surface oriented in the 1 th direction - '' diae:onal' elements

11 oo : 01

: 44

f-+

Physically, T is t he force per unit area (or stress) acting on the surface. More precisely, ~ j is the force (per unit area) in t he i t h direction acting on an element of surface oriented in t he j t h direction - '' diagonal' elements (Txx, Tyy, T22 ) represent pressures, and '' off-diagonal'' elements (Txy , Txz , etc.) are shears. To be more illustrative, suppose you are asked to calculate t he force in t he z direction. If you know t he stress tensor then the force can be directly calculated as f-+

(T ·da )z = Tzxdax + Tzyday + Tzzdaz To calculate t he force using stress tensor is a very lengthy task, so we avoid t hat calculation altogether.

j [email protected]

Physicsguide

38

@Sk J ahiruddin , 2020

4.3

Maxwell's equations

Momentum density

Momentum stored in the field can be represented by

(4.8) V

Momentum stored per unit volume is called the moment um density (A O \

11 oo : 01

: 47 (4.8) V

Momentum stored per unit volume is called t he moment um density (4.9) g = µocoS = Eo(E x B ) Momentum per unit t ime flowing in through the surface is represented by the integral of Maxwell's stress tensor . f-+

T -da s

In empty space 8g

f-+

- = v'· T

at

(4.10)

This is the '' continuity equation'' for electromagnetic momentum, with g (momentum density in the role of p ( charge "' density) and - T playing the part of J; it expr·esses t he local conservation of field momentum. Angular moment um stored in the field per unit volume •

IS

£ = r x g = Eo [r x (E x B)] [email protected]

@Sk J ahiruddin, 2020

39

(4.11) Physicsguide

lVIaxwell's equations

Do some examples and exercises on momentum and angular momentum from Griffiths.

00 : 01 : 49 l j [email protected]

= r x g = Eo [r x 39

@Sk J ahiruddin , 2020

(E x B)]

(4.11) P hysicsguide

:Nlaxwell's equations

Do some examples and exercises on moment um and angular moment um from Griffit hs.

j [email protected]

40

P hysicsguide

11 oo : oo : oo

Electromagnetic Wave Sk J ahiruddin Assistant Professor Sister Nibedita Govt. College, Kolkata Author was t he topper of IIT Bombay M.Sc Physics 2009-2011 batch He ranked 007 in IIT J AM 2009 and 008 (J RF) in CSIR NET J t1ne 2011 He has been teaching CSIR NET aspirants since 2012

1

@Sk J ahiruddin , 2020

EM wave

11 oo : oo : 03 1

EM wave

@Sk J ahiruddin, 2020

Contents 1

M axwe ll's equations in vacuum - EM wave 1.1

Solution of EM wave . . . . . . .













6

1.2

General solut ion of wave equation













11

1.3

Energy and Momentum in E lectromagnetic Waves

2











































Maxwell's equation in linear medium 2. 1

2.2

3

4

21

Reflection and Transmission at Normal Incidence . . . . . . . . . . . . . . . . . . . . . .

23

Reflection and Transmission at Oblique Incidence . . . . . . . . . . . . . . . . . . . . . .

24

M axwe ll's equations in C onductors 3. 1

16

Attenuation and skin depth 3.1 .1

j [email protected]

. . . . . .

Skin depth in a poor conductor:

2

25 •





27





28

Physicsguide

11 oo : oo : os 3.1.1

Skin depth in a poor conductor:

j [email protected]



2

EM wave



29

Reflection and transmission of EM from a conducting surface . . . . . • • • •

30

3.1.2

Skin depth in a good conductor:



4

5

28

P hysicsguide

@Sk J ahiruddin , 2020

3. 2













Wave guide

33

4.1

TE Waves in a Rectangular Wave Guide . .

35

4.2

TM Waves in a Rectangular Wave Guide . .

38

4.3

Resonance ca,,ity . . . . . . . . .













40

4.4

Dielectric inserted into waveguide













41

Retarded Potential

42

11 oo :oo :oa

[email protected]

Physicsguide

3

EM wave

@Sk J ahiruddin , 2020

1

Maxwell's equations in vacuum EM wave

Maxwell's equations in vacuum are

(ii) v' · B

=

0,

(iv) v' x B

=

(1.1)

µoEo ~~

They constitute a set of coupled , first-order , partial differential equations for E and B. They can be decoupled by applying the curl to (iii) and (iv):

v7

X

(v7

X

2

E ) = v7 (v7 · E ) - v7 E = v7

aB

at

X

a a2 E = - at (v7 X B) = - µ oEo at2 Similarly for magnetic field V

X

(V

X

2

B ) = V (V · B ) - v7 B = V X =

a

µoEo at (V

X

E)

=

aE µo Eo at

a2 B

-µoEo at2

Or, since v7 · E = 0 and v7 · B = 0 " ' ) T""'O

r,') -r,.

11 oo : oo : 11 µoEo at

X

aB

a

2

= µoEo at (V

X

E) = -µoEo at2

Or, since V · E = 0 and V · B = 0

a2E

2

(1.2)

V E = µoEo 8t2 ' [email protected]

Physicsguide

4

EM wave

@Sk J ahiruddin , 2020

In vacuum, t hen, each Cartesian component of E and B satisfies the three dimensional wave equation,

v2f

a2 f 2

= I

(1 .3)

v 8t2

So Maxwell's equations imply that empty space supports the propagation of electromagnetic waves, traveling at a speed v=

1

~

=

8

3.00 x 10 m /s

(1.4)

But this is something that we've seen before. It's the speed of light c ! This, of course, is because these electromagnetic waves are light. The emergence of light comes from looking for solut ions of Maxwell's equations in which t he electric and magnetic fields change with time, even in the absence of any external charges or currents. The essence of the physics lies in the two Maxwell equations on the right: if the electric field shakes, it causes the magnetic field to shake which, in t urn, causes the electric field to shake, and so on. The simple calculation that we have just seen represents one of t he most important moments in physics. Not onlv

11 oo : oo : 14 field shakes, it causes the magnetic field to shake which, in t urn, causes the electric field to shake, and so on. The simple calculation that we have just seen represents one of t he most important moments in physics. Not only are electric and magnetic phenomena unified in the Maxwell equations, but now optics - one of the oldest fields in science - is seen to be captured by t hese equations as well. j [email protected]

5

@Sk J a hiruddin , 2020

1.1

Physicsguide

ElVI wave

Solution of EM wave

We've derived two wave equations, one for E and one for B . We can solve these independently, but it's important to keep in our mind that the solutions must also obey t he original Maxwell equations. This will then give rise to a relationship between E and B. Let's see how this works. We'll start by looking for a special class of solutions in which waves propagate in the x -direction and do not depend on y and z. These are called plane-waves because, by construction, t he fields E and B will be constant in t he (y , z) plane for fixed x and t The Maxwell equation V · E = 0 tells us that we must have Ex constant in this case. Any constant electric field can always be added as a solution to the Maxwell equations so, without loss of generality, we'll choose this constant to vanish. We look for solut ions of t he form

11 oo : oo : 16 have Ex constant in this case. Any constant electric field can always be added as a solution to the Maxwell equations so, without loss of generality, we'll choose this constant to vanish. We look for solutions of the form

E = (O, E (x, t), O)

[email protected]

Physicsguide

6

E11 wave

@Sk Jahiruddin, 2020

where E satisfies the wave equation

2-2 a2 E2 - v2E = o c 8t

The most general solut ion to the wave equation takes t he form

E (x, t ) = f (x - et)

+ g(x + et)

Here f (x - et) describes a wave profile which moves to the right with speed e. (Because, as t increases, x also has to increase to keep f const ant). Meanwhile, g(x + ct ) describes a wave profile moving to the left wit h the speed c. The most important class of solutions of this kind are t hose which oscillate wit h a single frequency w . Such waves are called monochromatic. For now, we'll focus on t he rightmoving waves and t ake t he profile to be the sine function.

11 oo : oo : 19 The most important class of solutions of t his kind are those which oscillate wit h a single frequency w . Such waves are called monochromatic. For now, we '11 focus on t he right moving waves and take t he profile t o be the sine function. (We'll look at t he opt ion to take cosine waves or ot her shifts of phase in a moment when we discuss polarisation). We have

E = E osin

w

X

- -t

(1.5)

C

which in generally writ t en as

E = Eo sin(kx - wt ) [email protected]

@Sk J ahiruddin, 2020

7

(1.6) P hysicsguide

EM wave

where k is t he wavenumber . The wave equation requires t hat it is relat ed to the frequency by

(1.7) Equations of t his kind, expressing frequency in terms of wavenumber, are called dispersion relations. Because waves are so important in physics, there's a whole bunch of associated quantit ies which we can define. They are: • The quantity w is more properly called the angular frequency and is t aken t o be posit ive. The actual frequency f = w / 21r measures how often a wave peak passes you by. But because we will only talk about w, we will be lazy and just refer to t his as frequency.

11 oo : oo : 21 quency and is taken to be positive. The actual frequency f = w / 21r measures how often a wave peak passes you by. But because we will only talk about w, we will be lazy and just refer to t his as frequency. • The period of oscillation is T

= 21r / w

• The wavelength of t he wave is ,\ = 21r / k. This is the property of waves t hat you first learn about in school. The wavelength of visible light is between ,\ rv 3.9 x 10-7 m and 7 x 10-7 m . At one end of the spectrum, gamma rays have wavelength ,\ rv 10- 12 m and X -rays around ,\ rv 10- 10 to 10- 8 m. At t he other end, radio waves have ,\ rv 1cm to 10 km. Of course, the elecj [email protected]

8

@Sk J ahiruddin, 2020

Physicsguide

EM wave

t romagnetic spectrum doesn't stop at t hese two ends. Solutions exist for all A. Although we grow up thinking about wavelength, moving forward the wavenumber k will turn out to be a more useful descript ion of the wave

• E 0 is the amplitude of the wave. So far we have only solved for the electric field. To determine the magnetic field, we use V · B = 0 to tell us that B x is constant and we again set B x = 0. We know that t he other components B y and B z must obey t he wave equat ion ~\ 0:~~ - V 2 B = 0. But their behaviour is dictated by

11 oo : oo : 24 So far we have only solved for the electric field. To determine the magnetic field, we use V · B = 0 to tell us that B x is constant and we again set B x = 0. We know that t he other components B y and B z must obey t he wave equat ion c\ ~t~ - V 2 B = 0. But their behaviour is dictated by what t he electric field is doing through the Maxwell equat ion V x E = -8B / at. This tells us t hat

B = (0,0, B ) with

8B - -t 8

=

8E - --x 8

we find B

=

Eo

=-

-kEo cos(kx - wt) sin(kx - wt )

C

We see that the electric E and magnetic B fields oscillate in phase, but in orthogonal directions. And both oscillate [email protected]

P hysicsguide

9

EM wave

@Sk J ahiruddin, 2020

in directions which are orthogonal to t he direction in which t he wave travels. We can visualize t he wave propagation like t his

k

11 oo : oo : 26 E

k

Figure 1.1: Electromagnetic wave propagation

We can often write t he solut ions of E and Bin complex notation

E ==

Eo-S, ei(kx- wt)

'

B ==

E a z ei(kx- wt)

(1.8)

C

This is strange because t he physical electric and magnetic fields should certainly be real objects. You should t hink of them as simply the real parts of t he expressions above. But the linearity of t he Maxwell equations means both real and imaginary parts of E and B solve the Maxwell j [email protected]

@Sk J ahiruddin , 2020

10

Physicsguide

EM wave

equations. And, more importantly, if we start adding complex E and B solutions, then the resulting real and imaginary pieces will also solve the Maxwell equations . The advantage of this notation is simply t hat it's typically easier to manipulate complex numbers t han lots of cos and sin formulae. However, you should b e aware t hat this notation comes with some danger: whenever you compute something which . ..

.

.

11 oo : oo : 29 ier to manipulate complex numbers t han lots of cos and sin formulae. However, you should be aware t hat this notation comes with some danger: whenever you compute something which isn 't linear in E and B - for example, the energy stored in t he fields , which is a quadratic quantity - you can 't use the complex notation above; you need to take t he real part first .

1.2

General solution of wave equation

Above we have presented a particular solution to t he wave equation. Let's now look at the most general solut ion with a fixed frequency w. This means that we look for solut ions within t he ansatz,

E

= E oei(k ·x-wt)

and

B

(1.9)

= B oei(k ·x-wt)

where, for now, both Ea and Bo could be complex-valued vectors. (Again, we only get t he physical electric and [email protected]

11

Physicsguide

EM wave

@Sk J ahiruddin , 2020

netic fields by taking t he real part of these equations). The vector k is called the wavevector. Its magnitude, k = k, is the wavenumber and the direction of k points in t he direction of propagation of the wave. The expressions in ( 1.9) already satisfy the wave equations of fields, if w and k obey t he dispersion relation w 2 = c2 k 2 '

,..

.

,

,

..

,.

. ,

11 oo : oo : 31 is the wavenumber and the direction of k points in the direction of propagation of the wave. The expressions in (1.9) already satisfy the wave equations of fields, if w and k obey t he dispersion relation w 2 = c2 k 2 We get further constraints on E 0 , B o and k from the original Maxwell equations. These are

V ·E

=

0 ⇒ ik · E 0 = 0

V ·B

=

0 ⇒ ik · B 0 = 0

'v x E =

~ ~~ ⇒ ik x E 0 =

iwB 0

Linear Polarisation: Suppose that we take E 0 and B 0 to be real. The first two equations above say that both E 0 and B o are orthogonal to the direction of propagation. The last of t he equations above sa)'S that E 0 and B 0 are also orthogonal to each other. You can check that t he t hird Maxwell equation doesn't lead to any further constraints. Using t he dispersion relation w = ck, the last constraint above can be written as

k x (Eo/c) = Bo [email protected]

12

(1.10) Physicsguide

EM wave

@Sk J ahiruddin, 2020 A

This means that t he t hree vectors k , E 0 / c and B o form a right-handed orthogonal triad. Waves of t his form are said to be linearly polarised. The electric and magnetic fields oscillate in fixed directions, bot h of which are transverse to t he direction of propagation.

11 oo : oo : 34 1s means t at t e t ree vectors O orm a right-handed ort hogonal t riad. Waves of t his form are said to be linearly polarised . T he elect ric and magnetic fields oscillate in fixed directions, bot h of which are transverse to t he direction of propagat ion. Evidently, E and B are in phase and mut ually perpendicular; t heir (real) amplit udes are related by k I Bo= - Eo = - Eo W

C

Circular and Elliptic Polarisation: Suppose t hat we now take E 0 and B 0 to be complex . The actual elect ric and magnetic fields are just t he real parts of (1.9) but now t he polarisation does not point in a fixed direct ion. To see t his, write E a = a - i f3 The real part of t he electric field is t hen

E = a cos(k · x -wt ) + f3 sin(k • x - wt ) with Maxwell equations ensuring t hat a · k = ,8 · k = 0. If we look at t he direction of E at some fixed point in space, say t he origin x = 0 , we see that it doesn 't point in a fixed direcj [email protected]

13

P hysicsguide

ElVI wave

@Sk J a hiruddin , 2020

t ion. Instead, it rotates over t ime wit hin t he plane spanned by a and ,B (which is t he plane perpendicular to k). A



1



1

I

1

1

I

1

I

11 oo : oo : 37 t ion. Instead, it rotates over time wit hin t he plane spanned by a and /3 (which is the plane perpendicular to k). A special case arises when the phase of E 0 is ei1r:/ 4 , so that a = 1/31 , with the further restriction t hat a · /3 = 0. Then t he direct ion of E traces out a circle over time in t he plane perpendicular to k. This is called circular polarisation. The polarisation is said to be right-handed if (3 = k x a and left-handed if (3 = - k x a A

A

In general, t he direction of E at some point in space will t race out an ellipse in t he plane perpendicular to t he direction of propagation k. Unsurprisingly, such light is said t o have elliptic polarisation. If t he wave propagate in t he z direction then you take E points in the x direction, and B points in t he y direction

E (z t ) = E '

ei(kz- wt) x 0

'

taking t he real part

E (z, t) = Ea cos(kz-wt+b) x ,

B (z, t) =

! Ea cos(kz-wt+b) y C

We geralize to monochromatic plane waves traveling in an arbitrary direction. Wave vector , k , pointing in the [email protected]

@Sk J ahiruddin , 2020

14

P hysicsguide

E11 wave

t ion of propagat ion, whose magnitude is t he wave number

11 oo : oo : 40 @Sk Jahiruddin , 2020

EM wave

t ion of propagation, whose magnit ude is t he wave number k. The scalar product k · r is t he appropriate generalization of k z so

E (r , t ) =

B (r , t) =

E oei(k •r - wt) n

1

'(k

-Eoe i

·r-wt

)

(k A

X

n)

C

where

=

1

-k A

X

E

(1.11)

C

n is t he polarization vector .

Because Eis transverse,

A

n· k = 0 The elect ric and magnetic fields (the real part of the complex fields) in a monochromat ic plane wave wit h propagat ion vector k and polarization ft are E (r , t ) = E 0 cos(k · r - wt + c5)n 1 B (r , t) = - E0 cos(k · r - wt+ c5)(k x n) A

(1.12)

C

[email protected]

@Sk J ahiruddin , 2020

15

P hysicsguide

E M wave

11 oo : oo : 42 15

[email protected]

Physicsguide

EM wave

@Sk J a hiruddin , 2020

1.3

Energy and Momentum in Electromagnetic Waves

Energy per unit volume in electromagnetic field is 1 u =2

(1.13)

for monochromatic plane wave

B

2

=

1

-c2 E

2

=

µoEoE

2

(1.14)

electric and magnetic contributions are equal. t hen the energy density (wave propagating in z direct ion) (1.15) As the wave t ravels, it carries this energy along with it. The energy flux den- sity (energy per unit area, per unit t ime) t ransported by the fields is given by the Poynting vector 1 S =-(Ex B) µo For monochromatic plane waves propagating in t he z direct ion, 2 S = cE 0 cos (kz - wt+ 8)z = cuz

E5

j [email protected]

16

Physicsguide

11 oo : oo : 4s t ion,

j [email protected]

16

Physicsguide

@Sk J a hiruddin , 2020

ElVI wave

moment um density stored in t h e field is 1

g =- S c2

so for plane monochromatic wave 1 E 2 cos 2(k z - wt g = -Eo O

+ u~) z = A

C

1 -uz A

C

7

In the case of light , the wavelength is so short ( rv 5 x 10- m) , 15

and the period so brief ( rv 10- s) , t h at any macroscopic measurement will encompass m any cycles. That 's why we are interested only in time average value. The average of cosine-squared over a complet e cycle is 1 2'

so (1.16)

(1.17)

(1.18) The average power p er unit area t ransported by an elect romagnetic wave is called t h e inten sity :

I

(S)

1

2

= cEo E 0 2

(1.19)

11 oo:oo:47 The average power per unit area t ransported by an elect romagnet ic wave is called t he intensity :

I _ (S) = [email protected]

1

2

2

cEoE 0

(1.19) P hysicsguide

17

@Sk J ahiruddin, 2020

E11 wave

When light falls (at normal incidence) on a per£ect absorber , it delivers its momentum t o t he surface. In a t ime

~t, t he momentum transfer is ~p = (g )Ac~ t, so the radiation pressure (average force per unit area) is

(1.20) If the radiation is totally reflected back along its original path, the force per unit area is

2I

P =c

Example: A radio station on t he surface of the Earth t ransmits a sinusoidal wave with an average total power of 50 kW. Assuming the signal is radiated uniformly in all directions above t he ground, what are t he amplitudes, E 0 and B 0 of the respective electric and magnetic fields detected by a satellite at a distance of 100 km from t he transmitt ing antenna at the station?

Solution:

As the station is on t he surface of the earth

t hen all the radiation must have gone above, not inside the

11 oo : oo : so by a satellite at a distance of 100 km from t he transmitting antenna at t he station?

Solution:

As the station is on t he surface of the earth t hen all t he radiat ion must have gone above, not inside the ground. So we t ake a hemisphere centered on t he radio [email protected]

18

P hysicsguide

EM wave

@Sk J ahiruddin , 2020

station wit h a radius r = 100km. (we would t ake full sphere when the radiation will be in the space, like sun) . The surface area of the hemisphere is 2 2 5 A =21rr = 21r (1 x 10 m)

= 6.28

10

x 10 m

2

Since all the radiated power passes through t his surface, the average power per unit area (i.e., t he intensity) is I = p = 5.00 x 104W = 7.96 x 10- 7W / m 2 A 6.28 x 1010 m 2

t he intensity is t he average value of t he P oynt ing vector .

E0 =

2µ 0 cl

== ✓2 (41r

= 2.45

X

X

10- 7 ) (3

X

108 m/s) (7.96

2

10- V / m

Rn = En / ('. = 8.17

X

10- 11 T

X

10- 7W / m 2 )

11 oo : oo : s2 E0

=

2µ 0 cI

= ✓2 (41r

= 2.45

X

X

10- 7 ) (3

X

108m/s) (7.96

X

10- 7W / m 2 )

10- 2V /m 11

Bo = Eo/c = 8.17 x 10- T

Example: You have kept your hand at a distance of 2 m from a 60 W light bulb . find j [email protected]

Physicsguide

19

EM wave

@Sk J ahiruddin , 2020

(a) t he intensity (b) t he radiation pressure (c) the force acting on your hand (d) t he p eak electric and magnetic fields

Solution : directions.

Assume t he bulb radiates uniformly in all

(a) If t he average power is (P ) = 60W, the intensity at

R = 2m is

2

2

I = (P )/ Area = (P )/41rR = 60W/ 41r(2m) = l .19W/ m (b) The radiation pressure at R = 2m is 2

8

2

9

P rad = I/c = 1.19W / m / 3 x 10 m /s = 3.98 x 10- P a (c) The force on your hand is

2

11 oo : oo : 55 (b) The radiation pressure at R = 2m is Prad

I/c

=

=

2

8

1.19W / m / 3 x 10 m/s

2

=

9

3.98 x 10- Pa

(c) The force on your hand is F = Prad Ahand = (3.98

X

9

10- Pa) (0.10m

X

0.18m)

11

= 7.2 x 10- N (d) We knovv From earlier we have P rad [email protected]

@Sk

and Physicsguide

20

Jahiruddin, 2020

Ea

Bo = -

EM wave

and so Ea = c✓2Pradµo So

C

E 0 = (3 x 10 m/s) ✓2 (3.98 x 10- 9 Pa) (41r x 10- 7 ) 8

= 29.7 V /m B0

2

=

29. 7V / m 3 x 108 m/s

9 89 10- sT · x

=

Maxwell's equation in linear mediuIJ

Inside matter , but in regions where t here is no free charge or free current, Maxwell's equations become (i) (ii)

V-D V ·B

= =

0

(iii)

'

0

)

(iv)

v x E = _ aB

at

V x H=

aD

at

(2.1)

11 oo : oo : s1 (i)

v7·D == O

(iii)

(ii)

v7·B==O

(iv)

'

'

aB at v7 X H == ao at

(2.1)

aB at aE X B == µE at

(2.2)

v7

X

E == -

In terms of E and B

(ii)

v7 x E == -

(iii)

(i) v7 · E == 0, v7·B == O j

(iv)

j [email protected]

V

21

Physicsguide

EM wave

@Sk J ahiruddin , 2020

velocity of propagation of E NI wave inside a medium is 1

C

yif]j,

n

(2.3)

V - --- -

(2.4)

n ==

n is the index of refraction of the substance. For most materials, µ is very close to µ 0 , so

(2.5) where

Er

is the dielectric constant. Since

Er

is almost

always greater than 1, light travels more slowly through matter-a fact that is well known from optics.

11 oo : 01 : oo where

Er

is the dielectric constant. Since

Er

is almost

always greater than 1, light travels more slowly t hrough matter-a fact that is well known from optics. energy density would be 1 U=2

(2.6)

Poynting vector 1 s =- (E X B) µ

(2.7)

Intensity

(2.8)

[email protected]

Physicsguide

22

EM wave

@Sk J ahiruddin , 2020

Boundary conditions

(i) E1Er = E2Et, (ii) B/- = Bf ,

2.1

(2.9)

Reflection and Transmission at Normal Incidence

You have done the derivation before. I just state the results If f3 _ µ1v1 µ 2v2 '

,

=

µ1n2

µ2n1

11 oo : 01

: 03

You have done the derivation before. I just st at e the results If /3

_

µ1 v 1

=

µ 2v2

µ 1n2 µ2n1

t hen the reflected and transmitted component of wave

E

2

EoR =

l

+ (3

Ot

For maximum medium t he p ermeabilities µ are close to their values in vacuum, µ ~ µo then f3

=

v1/v2,

and we h ave (2.10)

Eo1, In t erms of indices of refraction

(2.11)

Eo1, 23

[email protected]

Physicsguide

EM wave

@Sk J ahiruddin , 2020

R eflection and transmission coefficients

IR

R

11

=

EoR Eo1

2

2

n 1 - n2 n1

(2.12)

+n2

and

Jy T == 11 2.2

--

E2V2 E1V1

Eor Eo1

2

4 n 1 n2 (n1

+ n2)

2

(2.13)

Reflection and Transmission at Oblique Incidence

11 oo : 01

2 .2

: 06

R e fle ction and Transmission at Oblique Incide nce cos0r --; cos01 a- /3

a EoR =

(2.14) (2.15)

a + /3

These are known as Fresnel 's equations The reflection and t ransmission coefficients for waves polarized parallel to the plane of incidence are

R

T

Ir 11

--

E2V2 E1V1

IR 11

=

EoT Eo1

EoR Eo1 2

2

cos0r cos01

j a [email protected]

a - /3 a+ /3 =

24

(2.16) 2

a+ /3

2

(2.17) P hysicsguide

ElVI wave

@Sk J a hiruddin , 2020

3

a/3

2

Maxwell's equations in Conductors

Inside conductor charges are free fo move. And free current density follows Ohm's law JJ

= o-E

C- - .... .... . 1\ff _ __......,,....,._11 ,_ --.. ... , ..... ~ : -,...._ ~ - - - ,,..,. . . . --

(3.1)

11 oo : 01 : oa Inside conductor charges are free fo move. And free current density follows Ohm's law

J J = o-E

(3.1)

So our Maxwell's equations becomes .

1

(1) V · E = - p1 , E (ii) V · B = 0,

...)

(

111

'7

v X

.

(1v ) V X

E

8B = - Ot

(3.2)

BE

B = µo-E + µ E

ot

We apply now, t he continuity equation for free charge,

(3.3) Using Ohm's and Gauss Law t his becomes

(3.4) T he solution becomes

(3.5) [email protected]

@Sk J ahiruddin, 2020

25

P hysicsguide

E11 wave

Thus any initial free charge p(O) dissipates in a characterist ic t ime T Ej a- . This reflects t he familiar fact t hat if you put some free charge on a conductor, it will flow out to t he edges. The time constant T affords a measure of how "good ' a con ductor is: For a "perfect'' conductor, o- = oo and, = O; for a '' good' conductor, , is much less than t he other relevant t imes in the problem.

11 oo : 01

: 11

some free charge on a conductor , it will flow out t o the edges. The time constant T affords a measure of how "good' a con ductor is: For a " p erfect '' conductor, a= oo and a "good' conductor ,

T

T

= O; for

is much less than the other relevant

t imes in the problem. After all free charges disappear Maxwell's equation b ecomes

(i) V · E = 0, (iii) V x E = - ~~ (ii) V · B = 0, (iv) V x B =µ Et~ + µaE

(3.6)

Applying the curl t o (iii) and (iv) , we get modified wave equations for E and B :

a2E

2

V E = µE at2

+ µa

aE

at '

2

a2 B

V B = µE at2

+ µa

aB

at (3. 7)

These equations have plane wave solutions

E(z, t ) =

-

E oei(kz-wt) '

B (z, t ) =

-

B oei(kz-wt )

(3.8)

But with one difference. Now the wave vector has b ecome complex

(3.9) 26

[email protected]

Physicsguide

EM wave

@Sk J ahiruddin , 2020

You can break the wave vector in real and complex part ~

k and get

= k + i~

(3.10)

11 oo : 01

: 13

You can break the wave vector in real and complex part ~

k

= k + i~

(3.10)

and get 1/2

2



1+

2

(3.11)

+ 1

EW

and

1/ 2 (J'

- 1

1+

3.1

2

(3.12)

EW

Attenuation and skin depth ~

The imaginary part of k results in an attenuation of t he wave (decreasing amplit ude wit h increasing z) :

E (z, t) =

B (z, t)

Eo e - r;,zei(kz- wt)'

= B oe- r;,z ei(kz - wt)

(3.13) The distance it takes to reduce t he amplit ude by a factor of

1/e (about a t hird) is called t he skin depth: 1 -

d

(3.14)

~

it is a measure of how far t he wave penetrates into the con~ ductor. Meanwhile, t he real part of k determines the wavej [email protected]

@Sk J ahiruddin, 2020

27

P hysicsguide

EM wave

length , the propagation speed, and t he index of refraction, in t he usual way:

11 oo : 01

: 16

@Sk J ahiruddin , 2020

ElVI wave

lengt h , the propagation speed , and t he index of refraction, in t he usual way: .\ ==

ck n== -

2n k '

3.1.1

(3.15)

w

Skin depth in a poor conductor:

For poor conductor a

w

K,

~w

Eµ 2

(3.16)

WE

w

K,

~

w



(3.20)

a

l +

2

1/ 2

2

- 1

EW



a

2

EW

(3.21)

µaw

1/2 ,...._, ,...._,

2

So for a good conductor

2

(3.22)

µaw For good conductor as a

>> WE so,

you can see (3.23)

or

21r

A= k

A = 21rd or d= 21r "" '

21r ('..J

-

(3.24)

Example : (a) Find t he skin depth of pure water. You are given that for pure water conductivity a= dielectric constant k = 80 .1 and µ ~ µ 0 . j [email protected]

@Sk J ahiruddin, 2020

29

l/ (2.5 x

5

10

),

P hysicsguide

EM wave

11 oo : 01

: 21

[email protected]

P hysicsguide

29

EM wave

@Sk J ahiruddin, 2020

(b) Find the skin depth (in nanomet ers) for a typical metal whose conductivity a ~ 107 , the frequency w ~ 1015 with E ~ Eo and µ ~ µo.

Solution: (a) Conductivity shows that the water is a poor conductor. Hence we will take approximation d = 1. rv K,

2 •

(7

d = (2) (2.5 x 10

5

(80.1) (8.85 X l0-12) 41T X 10- 7

)

= 1.19 X 104 m

(b) We have proved that for good conductor d ~

2 µaw



Put t he values to get

d

=

8

l X

107

=

1. 3 x 1o-8

=

13nm

So the fields do not penetrate far into a metal

3.2

Reflection and transmission of EM from a conducting surface

Suppose a EM wave is traveling from a medium to a conductor. What will be t he fraction of t he wave reflected and [email protected]

30

Physicsguide

11 oo : 01

: 23

Suppose a EM wave is traveling from a medium to a conductor. What will be t he fi·action of the wave reflected and j [email protected]

P hysicsguide

30

EM wave

@Sk J ahiruddin, 2020

t ransmitted in the conducting surface. Well you can prove ~ t hat if k 2 is the complex wave vector of the conductor, µ 1 , µ 2 are t he magnetic suscept ibilities and v 1 is the velocity of the wave in the first medium then define µ1 v1 ~

~

/3 = - k 2

(3.25)

µ2w

The amplitude of t he reflected and transmitted wave is ~

EoR =

(3.26)

~

1 + /3

~

For a perfect conductor (a = oo), k 2 = oo, so f3 infinite, and so ~

~

Eo1z = - Ea1,

~

EoT = 0

(3.27)

In this case the wave is totally reflected, wit h a 180° phase shift. That's why excellent conductors make good mirrors. You seen t hat mirror is designed by a thin silver coating onto t he back of a pane of glass. The glass has nothing to do with the reflection; it's just there to hold t he silver in t he 1·ight place and transmit light without absorption. Calculate the reflection coefficient for light at an air-to-silver interface Example:

11 oo : 01

: 26

Example: Calculate the reflection coefficient for light at an air-to-silver interface

[email protected]

31

P hysicsguide

EM wave

@Sk J ahiruddin , 2020

at opt ical frequencies (w = 4 x 10

15

/

s)

Solution: 2

R=

~ 2

~

1 - ;3

1 - ;3

~

~

1 + ;3

1 + ;3

where ~

JJ

rv

=

1 - ;3* ~

1 + ;3*

_ µ1V1k~ _ µ 1V1 (k + . ) - -- 2 - -2 'lK,2 µ 2w µ2 w

since silver is a good conductor (a K,2

~

>> EW)

k 2 rv = W

so

Let

=c

(6

X

107 ) ( 47f X 10- 7) = 29 (2) (4 X 1015 )

Then n

( l - 1' - i,1 \ ( l - 1' + i ,1 \

( 1 _ 1') 2 + 1'2

{\

{\')

11 oo :01 : 2a (6

=c

X

10 7 ) ( 41r X 10- 7 ) = 29 (2) (4 X 1015 )

Then

R= 32

j [email protected]

@Sk J ahiruddin , 2020

4

Physicsguide

EM wave

Wave guide

G uided waves are waves which are not free to move everywhere in the space. Dimension of t he medium t hrough which t he wave passe is restricted. The boundary of t he medium must be conductor. An d as the boundary is a p erfect conductor then E

=0

and B = 0 inside the m aterial

itself, and hen ce t he boun dary conditions at t he inner wall are.

(i) (ii)

=O B J_ = 0

E ll

(4. 1)

Let us take the medium is rest ricted by x and y and the wave is propagating along x direction.

11 oo : 01

[email protected]

: 31

Physicsguide

33

EM wave

@Sk J ahiruddin , 2020

X

Conductors

a

------------------

--+---►

z

y

Figure 4. 1: Guided wave

Boundary conditions and the Maxwell's equations gives

(i) and

a2 8x2

a2

+ 8y2 + (w/c)2 -

k2 Ez = 0

(4.2)

11 oo : 01 a2

a2

(i)

Ox2

and

a2

(ii)

: 34

+ [)y2 + (w/c)2 -

a2

Ox2

+ [)y2 + (w/c)2 -

k2 Ez = 0

(4.2)

k2 B z = 0

(4.3)

If Ez = 0, we call t hese T E (t ransverse electric'') waves; if B z = 0, t hey are called T M (''transverse magnetic") waves; if both Ez = 0 and Bz = 0 , we call them TEM waves. You can prove t hat TEM waves can 't occur in hollow rectangular wave guide. [email protected]

34

@Sk J ahiruddin, 2020

EM wave

TE, mode



Fz = 0 Bz = exist

T M mode



Ez = exist B z = 0

TEM mode

4.1

P hysicsguide



Ez = Bz = 0

TE Waves in a Rectangular Wave Guide

We need solution of (4.3) . t ake

Bz(x, y) = X (x) Y (y) and do t he separation of variable. You get t he solut ion

Bz = Bo cos(m1rx/a) cos(n1ry/b) wit h t he disp ersion relation

(4.4)

11 oo : 01

: 36

Bz = Bo cos(m1rx/a) cos(n1ry/b)

(4.4)

wit h the dispersion relation

k = J(w/c)

2

-

1r

2

[(m/ a)

2

+ (n/b)

2

]

(4.5)

Define

(4.6) Now you see if W


7.5 GHz.

4.2

TM Waves in a Rectangular Wave Guide

The solution is

Ez = F 0 sin

j [email protected]

m1rx

a



Sln

n1ry

b

(4.12)

Physicsguide

38

EM wave

@Sk J ahiruddin, 2020

if either of t he m and n is zero the solution itself is zero. So the lowest frequency mode is for TM wave in rectangular wave guide is T E11.

Example: A waveguide has a square cross section of side 2a. For Tm modes of wavevector k, the transverse electromagnetic modes are obtained in terms of a function ~ (x, y) which obeys the equation r-

,...,.,.,

,....J)

,

11 oo : 01

: 47

Example : A waveguide has a square cross section of side 2a . For T m modes of wavevector k, t he transverse elect romagnet ic modes are obtained in terms of a funct ion 'lf;(x, y) which obeys t he equat ion 32 8 2 X

+

32 8 2

y

w2 2 -

+ (C

2

k ) 'lf; (x,y) = 0

with t he boundary condit ion 'lf; (±a, y) = 'lf; (x, ±a) = 0. T he frequency w of the lowest mode is given by [NET June 2016] 2 2 2 (a)w2 = c2(k2 + 4;2) (b)w = c (k + ~) 2 2 2 2 2 2 (c)w = c (k + (d)w = c (k +

;:2)

;:2)

Solution: As t he mode is T M , the lowest frequency mode is T M 11 . So t he frequency w of the lowest mode is W 11 = C7r

[email protected]

1 2a

2

+

39

@Sk J ahiruddin , 2020

4.3

1 2a

2 =

v'2 C7r 2a

Physicsguide

EM wave

Resonance cavity

Resonant cavity produced by closing off t he two ends of a rect angular wave guide, at z = Oand at z = d, making a perfect ly conducting empty box. z

11 oo : 01

: 49

Resonant cavity produced by closing off the two ends of a rectangular wave guide, at z = Oand at z = d, making a perfectly conducting empty box. z



d

o " r - - - - - - -- t - -

y

X +----------+

b

Figure 4.2: Resonance Cavity

We can prove t hat t he cut off frequencies for both TE and TM modes in a resonance cavity are given by Wzmn

= c1r ✓(l/d) 2 + (m/a)2 + (n / b) 2

Vtmn

=

2 ~ ✓(l/d)

+ (m/a) + (n/ b) 2

40

j [email protected]

(4.13) 2

(4.14) Physicsguide

@Sk J ahiruddin, 2020

EM wave

Example : What should be t he 3rd dimension of a cavity of cross section 1 cm x 1 cm which operates at T E 103 mode at 24 GHz??

Solution: l

=

1, m

=

0, n

=

3. use

11 oo : 01 : s2 Example: What should be t he 3rd dimension of a cavity of cross section 1 cm x 1 cm which operates at T E 103 mode at 24 GHz??

l = l , m = 0, n = 3. use

Solution:

a= 0.01 , b = 0.01 , c = ? use t he formula of 24

2

X

X 10 9

to get 2

2

1 0.01

X 10 8

3

4.4

V zmn

3

+ o+

2

C

Dielectric inserted into waveguide

When t here is a dielectric / magnetic medium inside of a 1 dielectric inst ead of vacuum, just replace c by - -

ftµ

Vmn

=

1

m

2ftµ

a

For most of t he material µ Vmn

[email protected]

@Sk J ahiruddin , 2020

Example:

frequency T E 0

=

~

n

2

2

(4.15)

+ b

µ 0 , so t hen

1

m a

2 ✓ErEOµO C

m

2~

a 41

2

+

2

n b

+ n

2

(4.16)

2

b Physicsguide

EM wave

For a air filled wave guide. t he cut off = 1.8756 GHz. What will be the cut off

00 : 01 : 54 EM w ave

@Sk J ahiruddin, 2020

Example:

frequency T E 0

For a air filled wave guide. t he cut off = 1.8756 GHz. What will be the cut off

frequency if one dielectric of relative permeability E = 9Eo is inserted in t he dielectric? Solution: you have already derived the formula for cut off frequency for t he dielectric inserted wave guide. The ans •

IS

V dielectric = V vacuum / mn

5

mn

;z- = 1· 8756/3 GHz

y cr

Retarded Potential

Suppose you are measuring E and B at a position at a part icular t ime t. If t he system is static then it easy. What if t he source charge and current are changing with time? Electromagnetic signals travels with t he speed of light. So t he charge distribution of little backward time actually contributes in the field calculation. The time we need to calculate the charge and current distribution is called t he retarded time (5.1) [email protected]

@Sk J ahiruddin, 2020

42

Physicsguide

EM wave

11 oo : 01 : s1

@Sk J a hiruddin, 2020

EM wave

The formula to calculate the potentials t hen

V (r ,t) = -

1

P (r' ' tr) d3 ,

r,

47rEQ

1/,,

A (r , t )

=

µo 1r 4

(5.2) Here p ( r', tr) is t he charge density that prevailed at point r' at the retarded time tr. Because t he integrands are evaluated at the retarded time, t hese are called retarded potential. We will explore through examples. Example:

An infinite straight wire carries the current

l (t) =

0,

for t< 0

l o,

for t > 0

That is, a constant cu1~rent 10 is t urned on abrupt ly at t Find the result ing electric and magnetic fields.

= 0.

I dz

Figure 5.1: Retarded potent ial calculation for a current carrying wire which have started at t = 0 j [email protected]

(c)Sk J a,hir11ddin. 2020

43

Physicsguide

ElVI wa.ve

11 oo : 02 : oo 43

j [email protected]

P hysicsguide

@Sk J ahiruddin , 2020

E M wave

Solution: The wire is electrically neutral, so t he elect ric scalar potential is zero. Let the wire lie along the z axis. The ret arded vector potent ial at point P is

For t < s / c, the '' news'' has not yet reached P , and the potential is zero. For t > s / c, only the segment z < J (ct )2

-

s2

cont ributes. Outside t his range tr is negative, so I (tr) = O; t hus

The electric field is

E (s t ) = ' [email protected]

- fJ A = -

fJt 44

µol oc z 21T" (et )2 - s2

J

P hysicsguide

11 oo : 02 : 03 E (s t ) =

'

-

8A

at

= -

µo l oc z 21r ✓ (et)2 - s 2

44

[email protected]

Physicsguide

EM wave

@Sk J ahiruddin , 2020

and t he magnetic field

A

Ast



oo we recover the static case: E = 0, B = (µ 0 I 0 / 21r s) 0. Find t he electric and magnet ic fields generated. (b) Do t he same problem for delta funct ion current

I (t) = qo8(t) Solution: (a) We proceed exactly as the previous example. We know for t < s/c, A = O; for t > s/c. Now when calculating A we replace the time dependent current, we must replace t he t ime in the expression of current by t he //,, retarded t ime t - - Then you replace //,, = ✓s 2 + z 2 C

A (s, t) =

✓(ct)2 -s2 k(t - ✓s2

2 o

+ z2 / c) ---;::::=:====--dZ ✓s 2 + z 2

11 oo : 02 : 04 we mus rep ace e 1me 1n v~~e expression o curren /1,, ret arded time t - - Then you replace /1,, = ✓ s 2 + z 2 C

A (s, t ) =

✓(ct)2 -s2 k(t - ✓s2

2

+ z2 / c) ---;::::::::======--dZ ✓s 2 + z 2

o

[email protected]

P hysicsguide

45

EM wave

@Sk J ahiruddin , 2020

✓(ct) 2 -s 2

dz

✓(ct) 2 - s 2

1

dz

---;::::========== - ✓s2 + z2 C 0

0

t ln

et+ J(ct) 2 - s 2 s

1

- -J(ct) 2 - s2 C

The electric field E (s t)

'

t

= _

8A

at

s et+ J(et) 2

=

kz ln et+ ✓(et) 2~ s

_µ 0

1 -

s2

s

1

2

-

s

2c2 t

2

e+ -----;:::::==== 2 2 2 J(ct)

s

-

+ 2e2 t - - ----;:::::==== 2e J(et) 2 - s 2 1

et

et

+ -----;::=== -----;::=== 2 2 2 2 ✓(et)

s

-

s

✓(et)

( or zero, fort< s/e)

-

s

11 oo : 02 : oa

46

j a [email protected]

Physicsguide

ElVI wave

@Sk J a hiruddin , 2020

The Magnetic field B (s, t) = -

8AZA 08

..a /3 on the right-hand side. But this means t hat E1µvpa must be proportional to Eµvpa We only need to determine the constant of proport ionality. To do this, we can look at

11 oo : 01 But t his means that

1

E µvp 0. To t he right of S, however, the curve is concave downwards 2 2 so t hat df / dx is decreasing with c and hence d f / dx < 0 In summary, at a stationary point df / dx

==

0 and

(i) for a minimum, d2 f /dx 2 > 0 (ii) for a maximum, d2 f / dx 2 < 0 (iii) for a stationary point of inflection, d2 f / dx 2 d2 f / dx 2 changes sign t hrough t he point.

==

0 and

Example : Find the positions and natures of t he stationary points of t he function

f (x)

==

2x

3

-

3x

2

-

36x

+2

Solution: The first criterion for a stationary point is t hat df / dx == 0, and hence we set

df == 6x 2 dx from which we obtain

-

6x - 36 == 0

(x - 3)(x + 2) j [email protected]

17

==

0

physicsguide CSIR NET, GATE

@Sk J ahiruddin, 2020

Hence t he stationary points are at x

Basic Calculus

3 and x

- 2.

@ Sk Jahiruddin) 2020

Basic Calculus

Hence the stationary points are at x = 3 and x = - 2. To determine the nature of t he stationary point we must 2 2 evaluate d f / dx :

d2f dx 2

= 12x - 6

Now, we examine each stationary point in turn. For 2 2 x = 3, d f / dx = 30. since t his is posit ive, we conclude t hat x = 3 is a minimum. Similarly, for x = - 2, d2 f /dx 2 = -30 and so x = - 2 is a maximum.

1.5

Curvature of a function

Look at the figure 1.3 below. Let 0 be the angle made with the x -axis by the tangent at a point P on the curve f = f (x) , with tan 0 = df /dx evaluated at P. Now consider also t he tangent at a neighbouring point Q on the curve, and suppose that it makes an angle 0 + ~0 with the x -axis, as illustrated in t he figure.

j [email protected]

@ Sk J ahiruddin, 2020

18

physicsguide CSIR NET ) GATE

Basic Calculus

j [email protected]

18

physicsguide CSIR NET, GATE

@Sk J ahiruddin , 2020

Basic Calculus

f(x) C

p

0

,, , -- , , ,, ,, ,- , ,, , ,, , ,, , 0 + 11.0 -- 0 ,, , -,

,

X

Figure 1.3: T wo neighbouring t angents t o t he curve f (x) whose slopes differ by !:l.0. The angular separation of the corresponding radii of t he circle of curvature is also !:l.0

It follows t hat t he corresponding normals at P and Q, which are perpendicular to the respective tangents, also intersect at an angle !:l.0. Furt hermore, t heir point of intersect ion, C in t he figure, will be t he posit ion of the cent re of a circle that approximates the arc PQ , at least to t he extent of having t he same tangents at t he extremit ies of the arc. This circle is called t he circle of curvature. For a finite arc PQ , t he lengths of GP and CQ will not, j [email protected]

19

physicsguide CSIR NET, GATE

Basic Calculus

@Sk J ahiruddin, 2020

in general, be equal, as t hey would be if f = f (x) were in fact the equation of a circle. But, as Q is allowed to t end to P, i.e. as ~0 ➔ 0, they do become equal, their common value being p, the radius of the circle, known as the radius of curvature. It follows immediately t hat the curve and the circle of curvature have a common tangent at P and lie on the same side of it. The reciprocal of the radius of curvature, p- 1 defines the curvature of the function f (x) at the point p The radius of curvature can be defined more mathematically as follows. The length ~ s of arc PQ is approximately equal to p~0 and, in the limit ~0 ➔ 0, t his relationship defines pas ds d0

(1.12)

It should be noted that, ass increases, 0 may increase or decrease according to whether t he curve is locally concave upwards (i.e. shaped as if it were near a minimum in f (x) ) or concave downwards. This is reflected in the sign of p, which therefore also indicates the position of t he curve (and of the circle of curvature) [email protected]

@Sk J ahiruddin, 2020

20

physicsguide CSIR NET, GATE

Basic Calculus

@ Sk Jahiruddin, 2020

Basic Calculus

relative to the common tangent , above or below. Thus a negative value of p indicat es t hat t he curve is locally concave downwards and that t he t angent lies above the curve. We next obtain an expression for p, not in terms of s and 0 but in t erms of x and f (x) . The expression , t hough somewhat cumbersome, follows from the defining equation (1.12) , the defining property of 0 t hat tan 0 = df / dx f' and t he fact that the rate of change of arc length with x is given by

ds dx

df dx

1+

2 1/ 2

(1.13)

From the chain rule it follows that

ds d0

P=

=

ds dx dx d0

Differentiating both sides of tan 0 = df / dx with resp ect to x gives 2 d0 d f 2 sec 0- = - 2 _ J'' dx dx •

2

2

from which , using sec 0 = 1 +tan 0 = 1 + (! ')

j [email protected]

@ Sk J ahiruddin, 2020

21

2

,

we can

physicsguide CSIR NET, GATE

Basic Calculus

@Sk Jahiruddin, 2020

Basic Calculus

obtain dx/d0 as dx d0

1 + tan 0

1 + (!')2

!''

!''

2

Subst it uting t hese result s into the expression of p, we get t he final expression for p 1 + (!')2 p=

3/2

(1.14)

! ''

Example: Show t hat t he radius of curvature at t he

p oint (x, y ) on t he ellipse

x2

y2

a2

+ b2 =

4 2

4

1

2 312 4 4 / (a b ) x )

has magnit ude ( a y + b and the opposit e sign to y . Check t he sp ecial case b = a , for which the ellipse becomes a circle.

Solution: Differentiating t he equat ion of t he ellipse with respect to x gives

2x a2 [email protected]

@Sk Jahiruddin , 2020

+

2y dy _ O b2 dx -

22

physicsguide CSIR NET, GATE

Basic Calculus

@ Sk J ahiruddin , 2020

Basic Calculus

and so

dy dx

Taking derivative again b2 a2

b4 a2y3

y - xy' y2

y2

x2

b2

+ a2

where we have used the fact that (x, y) lies on t he ellipse. We note that d2 y / dx 2 , and hence p, has the opposite sign to y 3 and hence to y . Substituting in the final expression of radius of curvature (1.14), we get

PI =

[1 + b4x2 / (a4y2)] 3/ 2 - b4/ (a2y3)

( a4y2

+ b4x2 ) 3/ 2 a4b4 2 312 x )

For the special case b = a, IP reduces to a- (y + and, since x 2 + y 2 = a 2 , this in turn gives PI = a, as expected. 2

2

2

Calculus of Two variables

This section will be updated soon. For t he time being please have some examples to learn t he basics. [email protected]

@Sk J ahiruddin, 2020

23

physicsguide CSIR NET, GATE

Basic Calculus

Basic Calculus

@Sk J ahiruddin 1 2020

Example: Find the maximum and minimum points of t he function x3

-

y3

-

2xy

+2

Solution: We know for maximum or minimum points of functions of two variables. If f x

=

i! =

0 and

fy =

~[

= 0 Then at point (a, b)

(a, b) is a minimum point if at (a, b), f xx > 0, f yy > 0, and f xx f yy > J;y (a, b) is a maximum point if at (a, b), fxx < 0, f yy < 0, and f xx f yy > J;y (a, b) is neither a maximum nor a minimum point if f xx fy y < J;y Now apply t his to find the answer which is (0, 0) is a saddle point, (-2/ 3, 2/3) is a maxima

[email protected]

©Sk J ahiruddin 1 2020

24

physicsguide CSIR NET, GATE

Basic Calculus

@Sk J ahiruddin, 2020

Basic Calculus

Example: Find t he volume of the largest rectangular

parallelepiped wit h edges parallel to t he axes, inscribed x2 y2 z2 in the ellipsoid 2 + 2 + 2 = 1 with the help of a b c Lagrange's multiplier.

Solution: We take a point (x, y , z) in the corner in t he first octant where the box touches t he ellipsoid. Then (x , y, z) sat isfies t he ellipsoid equation and the volume of t he box in the first octant is xyz. There are 8 oct ants, so t he volume will be 8xyz. Our problem now becomes is to maximize f (x, y, z) = 8xyz, where the relation between (x, y, z) is the ellipsoid equation

cp ( X' y' z)

=

x2

y2

+ a

b2

2

z2

+

2 =

C

1

We know in the method of Lagrange's unknown multipliers if f is t he function to be maximized and t he function cp describes a relation between t he variables we write,

F (x, y, z ) =

[email protected]

@Sk J ahiruddin, 2020

f + >..cp = 8xyz + ).. 25

x2

y2

z2

a2 + b2 + c2

physicsg11ide CSIR NET, GATE

Basic Calculus

Then we set the t hree part ial derivatives of F equal to

Then we set the three partial derivatives of F equal to

0: oF 2x = 8yz + .\ · - 2 = 0 ox a oF 2y oy = 8xz + .\ · b2 = 0 oF oz

2z

= 8xy + .\ · - 2 = 0 c

Mult iply t he first equation by x, the second by y , and t he third by z, and add to get

3 · 8xyz

+ 2.\

y2

z2

+ b2 +

c2

x2

a2

=0

Using the equation of t he ellipsoid, we can simplify t his to

24xyz + 2,\ = 0

~~>

,\

= - l2xyz

Substituting ,\ into t he oF/ o x equation, we find that

j [email protected]

@ Sk J ahiruddin, 2020

26

physicsguide CSIR NET, GATE

Basic Calculus

8y z - 12xy z · x2

2x

=0

a2 1 = -a2 3

The other two equations could be solved in t he same way. We can also easily guess the solution from symmetry 2 2 2 2 t hat the solutions will be y = b and z = c . Hence the maximum Volume is

1

8xyz =

1

8abc

3vl3

Example: Make t he variable t ransformation r

=x+

vt, s = x - vt in t he wave equation 2

2

0y

10 y

ox 2

v 2 ot2

write the transformed form and solve the equation. Solution: The first step is t he most important step

oF oFor oFos oF oF o a -ox = + = + = or + OS or ox OS ox or OS j [email protected]

@Sk Jahiruddin, 2020

and

27

F

physicsguide CSIR NET I GATE

Basic Calculus

@Sk Jahiruddin, 2020

Basic Calculus

and

8F 8F8r 8F8s 8F 8F - = --+-- = v--v- = v at ar at as at ar as

8 ar

a

F

as

Now we can easily derive

a aF a a ax ax ar + as 32p 32p 32p 2 8r 2 + 8r8s + 8s 2

8F ar

8F + -a-s

And

aF

8F a {) = v at 2 at at ar as 2 2 2 8 F 8 F 8 F = v2 - - - 2 - - + - 8r2 aras 8s 2 2

a

8F V ar -

8F VOS

Just put t hese relation in the original differential equation to get 1 8 2F 8 2F -2- -2 = 4- - = 0 v 8t

8r8s

28

j [email protected]

physicsguide CSIR NET, GATE

@Sk Jahiruddin, 2020

l\ If" _ _ 1.L ! _

Basic Calculus

1 -

T __ .L -

-

-- -

1

@Sk J ahiruddin, 2020

3

Basic Calculus

Multiple Integral

We discuss t he multiple integral calculations here Example : Evaluate t he double integral ,.. ,..

(2x - 3y )dxdy ., ., A

A is t he t riangle wit h vertices (0, 0), (2, 1), (2, 0) Solut ion: (2, 1)

,0)

Here, the area is included by the curves y =

[email protected]

@ Sk J ahiruddin, 2020

29

1, x =

2.

physicsguide CSIR NET, GAT E

Basic Calculus

@Sk Jahiruddin, 2020

Basic Calculus

So, (2x - 3y )dxdy = 2

2

dy-3

xdx 0

A

2

X

2

0 2

2

X

X

2

ydy

dx 0 x2

0

xdx- - 3 dx 2 8 0 2 3 2 x dx - 0 8 0 3 2 3 2 x 3x 8 5 - --- - 1--3 o 83 0 3 3

=2

Example: " "

2

12y cos xdxdy

V VA

Where A is t he area included by the curves y the x axis and the line x = 1r / 2.

j [email protected]

@Sk J ahiruddin, 2020

n

1

, •

30

= sin x,

physicsguide CSIR NET, GATE

Basic Calculus

.

7r

s1n x

2

2

12y cos xdxdy = 12

y2dy

cosxdx

A

0

0 . 3 sin x

zr. 2

= 12

cosxdx~ 0

0

! .

3

s1n xcosxdx

=4

=

0 7r 4

.

Sln -

2

sin4 x zr. 4x ~ 4

=l

Example: Find the volume above the triangle with vertices (0, 0), (2, 0) , and (2, 1), and below the paraboloid z = 24 - x 2 - y 2

Solution: Volume of the region is given by,

V=

dxdydz X

2

2

dy

dx 0

0

0

[·. · talking idea from t he first problem] X

2

2

(24 - x 0

2

-

2 y )dxdy

0

[email protected]

31

physicsguide CSIR NET, GATE

@ Sk J a hiruddin, 2020

-X

2 0

dy0

2

= 12 n

2

2

dx

=24

Basic Calculus

-X

2

2 x dx

dy -

0

2

1 x dx - ').

n

3

0 2

3

x dx n

2

dx 0

1

x dx - ?. 4

-X

2 0

y2dy

Basic Calculus

@ Sk J a hiruddin , 2020

2

2

= 12

xdx -

0

x2

2

2

°

X -

2

dy -

0

2

X

2

dy-

0

=6

X

dx

= 24

= 12

X

-

0

1 2 1

-

2

2

3

0 X -

2

-

°

4 4 - 2 - 1 = 131 6 6

0

0

x dx -

x4

dx

-

1

24

1 24 X

0 4

x

4

2

°

Example: A dielectric lamina with charge density p = ky covers t he area between t he parabola y = 16 - x 2

and the x axis. Find t he total charge.

Solution:

Area

-4

+4

We need to det ermine the area ment ioned in t he figure, j a [email protected]

32

physicsguide CSIR NET , GATE

Basic Calculus

@ Sk J a-hiruddin , 2020

.

r

.

r

A

t

-1

l'""'i

')

So t he total charge will be given by, 16-x2

4

kydxdy = k

pdxdy =

Q=

dx -4

2 16-x2

4

=k -4

k 2 k 2

y . dx 2

ydy 0

k

0

4

[256 + x

4

-

2

32x ) dx

-4

256x

=

+

x5

5

x3

- 32

4

3

-4

k 2 X 45 256 X 8 + - 32 2 5 2048 _ 4096 k 2048 2 + 5 3 k 16384 8192k -X-2 15 15

2 X

X

43

3

Example: Evaluate the triple integral

ydxdzdy y= - 2 ..- z= l

[email protected]

x =y+z

33

physicsguide CSIR NET, GATE

@Sk Jahiruddin, 2020

Basic Calculus

Solution: 2

3

2y+z

ydxdzdy y= - 2

r3

z=l

x=y+z r2

@Sk J ahiruddin, 2020

Basic Calculus

Solut ion: 3

2

2y+z

ydxdzdy y=-2 z=l 3

x=y+z 2

ydy

dz [2y z=l

y=-2 3

2

3

dz y

y=-2 3 3

3

- 2

+ z - (y + z)]

=

z =l

33 + 23

35

3

3

Example : Find the volume between t he planes z = 2x + 3y + 6 and z = 2x + 7y + 8 and over the square in the (x, y) plane wit h vertices (0, 0), (1, 0), (0, 1) , (1 ,

1). Solution:

j [email protected]

@Sk J ahiruddin, 2020

34

physicsguide CSIR NET, GATE

Basic Calculus

[email protected]

physicsguide CSIR NET, GATE

34

@ Sk Jahiruddin , 2020

Basic Calculus

(0, 1)

(1, 1)

(0,0)

(1,0)

The volume will be given by, 1

dxdydz ==

V ==

dx 0

1

dy

dz 2x+3y+6

0

1

dy[(2x + 7y + 8) - (2x + 3y + 6)]

dx 0

0 1

1

dy[4y + 2]

dx 0

0 1

1

dx

==4 0

==2

2x +7y+8

1

1

ydy

+2

0

1

dy

dx 0

0

+ 2 == 4

j ahir@physicsguide. in

35

physicsguide CSIR NET, GATE

@Sk J ahiruddin 1 2020

Basic Calculus

Example: Find the volume in the first octant bounded by the coordinate planes and the plane x + 2y

+z = 4

Solution: 1z X

+ 2y + Z

=4

y=2 Y-->

j [email protected]

@Sk J ahiruddin, 2020

36

physicsguide CSIR NET, GATE

Basic Calculus

36

j ahir@physi csgui de.in

physicsguide CSIR NET, GATE

@ Sk Jahiruddin, 2020

Basic Calculus

The volume will be given by, 2

V=

dxdydz

=

dy

dx 0

0 2

4-x-2y

4- 2y

0

4- 2y

dy 0

dx{4-x-2y } 0

2

= 4

4-2y

dy 0

dx-

dy

0

0

2

=4

2

0

xdx- 2

M- s -

0 2

0

2y

0

{4 - 2y }ydy

{4 - 2y} dy- 2

2

dx

2

0

[4{4 - 2y} - -{ 16 + 4y 2 [16 -

ydy

2

1

2

4-2y

2

0

1

{4 - 2y }dy - 2

0

4- 2y

2

2

-

2

16y} - 2{4y - 2y }]dy

+ M- sy + 4y

2

J

0 2

2

[8 - 8y + 2y ]dy = 8y -4y + ; 2

2

. 0

=

}6 -¼ + 16 3

j [email protected]

3

2 0

=

16 3

37

physicsguide CSIR NET, GATE

Linear Algebra Sk J ahiruddin Assistant Professor Sister Nibedita Govt. College, Kolkata Aut hor was the topper of IIT Bombay M.Sc Physics 2009-2011 batch He ranked 007 in IIT JAM 2009 and 008 (JRF) in CSIR NET Jt1ne 2011 He has been teaching CSIR NET aspirants since 2012

1

@Sk J ahiruddin , 2020

Linear Algebra

Linear Algebra

@Sk J ahiruddin , 2020

Contents 1

Linear Vector Space

7

1.1

Definit ion . . . .



7

1.2

Linear dep endence of functions: Wronskian .

10

1.3

Basis . . . . .



































13

1.4

Inner Product



































17























21























21

1.4.1









Hilbert Space . .





















1.5

Some useful inequalit ies

1.6

Gram-Schmidt orthogonalisation

. .









25

1.7

Linear function and Linear Operators









27

1. 7.1

Linear function of vect or















27

1.7.2

Linear Operator . . . . .















28

1. 7.3

Properties of linear operators









30

[email protected]

@Sk J ahiruddin, 2020

2

physicsguide CSIR NET, GATE

Linear Algebra

Linear Algebra

@Sk J ahiruddin, 2020

4

3. 2 Lower and upper triangular matrices .







64

3.3

Symmetric and ant isymmetric matrices







65

3.4

Orthogonal matrices







66

3.5

Hermitian and anti-Hermitian matrices







68

3.6

Unitary matrices































69

3.7

Normal matrices































71

. . . . . . . . . .

Eigenvalues and Eigenvectors

4. 1 eigenvalues .

74





































74





































75

4.2

eigenvectors

4.3

Eigenvalue and eigenvectors of normal matrices 77

4.4

Eigenvectors and eigenvalues of Hermitian and anti Hermit ian matrices •

82

Eigenvectors and eigenvalues of a unitary matrix . . . . . . . . . . . . . . . . . . . . . . .

85



4.5

[email protected]

@Sk J ahiruddin, 2020

4



















physicsguide CSIR NET, GATE

Linear Algebra

@Sk J at1iruddin , 2020

4.6

4. 7

Linear Algebra

Simultaneous eigenvectors and commutation of matrices . . . . . . . . . . . . . . . . . . .

86

Cayley Hamilton and General properties of eigenvalues . . . . . . . . . . . . . . . 4.7.1

Some properties of eigenvalues

4.7.2

Some examples to evaluate eigenvalues 91









89









89

5 Similarity transformations and diagonalization 99 5.1

Change of basis . . . . . .





















99

5.2

Similarity transformation .





















101















103

5.3

5.2 .1

Unitary transformation .

5.2.2

Physical Significance of Similarity Trans£ormation . . . . . . . . . . . . . . . 105

Diagonalisation of matrices .

6 Function of matrices

j [email protected]

@Sk J ahirt1ddin, 2020



















106

111

5

physicsguide CSIR NET , GATE

Linear Algebra

Linear Algebra

@Sk J ahiruddin , 2020

6. 1

6.2

7

8

9

Function of matrices by similarity transformation . . . . . . . . 6.1.1

Trace formula



























112



























117

Function of matrix by using power series expansion and Cayley Hamilton t heorem . . . 118

130

Quadratic form 7.1

Basis invariance of Quadratic form











131

7.2

More examples on Quadratic form .











136

141

Exercises 8. 1

Ans Keys

8. 2

Solutions .













































































Ans Key of More Exercises . .

[email protected]

@Sk J ahiruddin, 2020

6

150 180

More Exercises with Ans Keys 9. 1

148

















182

physicsguide CSIR NET, GATE

Linear Algebra

(ii) t he set is closed under mult iplication by a scalar (any complex number) to form a new vector Aa,. So if a is an element of t he set, and A is a scalar t hen Aa belongs to the set. The operation being bot h distribut ive and associative

A· (a + b)= A·a +A ·b

(1.3)

(A+µ) · a = A ·a + µ · a

(1.4)

A· (µ. a)=(A ·µ )· a

(1.5)

[email protected]

physicsguide CSIR NET, GATE

7

@Sk J ahiruddin, 2020

Linear Algebra

where A and µ are arbit rary scala1~s; You need to know specifically t hat the set not needed to be closed under multiplication. Means if a and b are two elements of the set then a· b need not necessarily belong to t he set. (iii) t here exists a null vector O such t hat a + 0 = a for all a; (iv) mult iplication by unity leaves any vector unchanged, i.e. 1 · a = a ; (v) all vectors have a corresponding negative vector - a such that a + (-a) = 0 . We note t hat if we restrict all scalars to be real t hen we obtain a real vector space (an example of which is our ,..

.,.

_,

,

.

,

\

.

,

,

Linear Algebra

@Sk J ahiruddin, 2020

where A and µ are arbitrary scalars; You need to know specifically that the set not needed to be closed under multiplication. Means if a and b are two elements of the set then a· b need not necessarily belong to t he set. (iii) t here exists a null vector O such that a + 0

= a for

all a; (iv) multiplication by unity leaves any vector unchanged , i.e. 1 · a = a ; (v) all vectors have a corresponding negative vector -a such that a + (-a) = 0 . We note t hat if we restrict all scalars to be real t hen we obtain a real vector space (an example of which is our familiar t hree-dimensional space); otherwise, in general, we obtain a complex vector space. We note that it is common t o use t he terms 'vector space' and 'space', instead of t he more form al 'linear vector space'. The span of a set of vectors a , b , ... , s is defined as the set of all vectors that may be written as a linear sum of the

[email protected]

@Sk J ahiruddin, 2020

8

physicsguide CSIR NET, GATE

Linear Algebra

Linear Algebra

@Sk J ahiruddi11 , 2020

original set, i.e. all vectors x

= aa + ,Bb + · · · + o-s

(1.6)

t hat result from t he infinite number of possible values of t he (in general complex) scalars a , ,8, . . . , a-. If x in (x = aa + ,Bb+ · · ·+a-s) is equal to O for some choice of a, ,8, ... , o(not all zero), i.e. if

aa + ,Bb + · · · + o-s =

0

(1. 7)

t hen the set of vectors a , b , ... , s , is said to be linearly dependent. In such a set at least one vector is redundant, since it can b e expressed as a linear sum of t he others. If, however, (aa + ,Bb + · · · + a s = 0) is not satisfied by any set of coefficients (other than the t rivial case in which all the coefficients are zero) t hen t he vectors are linearly independent, and no vector in the set can be expressed as a linear sum of t he others. If, in a given vector space, t here exist set s of N linearly indep endent vectors, ( but no set of N + l linearly independent vectors exist ) then t he vector space is said to be N -dimensional. [email protected]

@Sk J ahiruddin, 2020

9

physicsguide CSIR NET, GATE

Linear Algebra

Example: Find whether these following sets form vec-

@Sk J ahiruddin , 2020

Linear Algebra

Example: Find whether these following sets form vec-

tor space or not with t he given dimensions in the bracket? (a) Non singular N x N matrices (dimension = N 2 ) (b) Polynomial functions of x ( dimension = infinite) (c) Absolutely convergent series (dimension = infinite) Ans: (a) No, (b) Yes, (c) Yes Now non singular N x N matrices are not closed under addit ion. We can add A + (-A ) = 0 where O then a singular matrix. Thus the set is not closed and it is not a vector space. Check that for (b) Polynomial functions of x and (c) Absolutely convergent series all the properties of vector space remains valid.

1.2

Linear dependence of functions: Wronskian

Before we proceed we need to know about Linear dependence of functions and the Wronskian. The functions [email protected]

@Sk J ahiruddin, 2020

f 1 ( x), f 2 ( x), · · · , f n ( x) 10

are linearly depen-

physicsguide CSIR NET, GATE

Linear Algebra

Linear Algebra

@Sk J ahiruddin , 2020

dent if some linear combination of them is ident ically zero, t hat is, if there are constants k1 , k2 · · · , kn, not all zero, such t hat For example, sin 2 x and ( 1 - cos 2 x) are linearly dependent since sin2 x - (1 - cos 2 x) = 0 •

But sin x and cos x are linearly independent since there are no numbers k1 and k2 not both zero, such t hat

k1 sin x

+ k2 cos x

is zero for all x We shall be particularly interested in knowing t hat a given set of functions is linearly independent. For this purpose t he following t heorem is useful If f1(x), f2( x), · · · , fn(x) have derivatives of order n- 1, and if the determinant

[email protected]

@Sk J ahiruddin, 2020

11

physicsguide CSIR NET, GATE

Linear Algebra

@Sk J ahiruddin , 2020

Linear Algebra

So, t he functions are independent of each other. (b) Here Wronkskian of the functions is

+ l )ex 0 ex (x + 2)ex l ex (x

= (x + 2)ex{xex - ex} - ex{x(x + l )ex - xex} = (x + 2)(x - l) e2x - x 2 e2x = (x - 2)e2x =/= 0 So, t he functions are independent of each other.

1.3

Basis

If V is an N -dimensional vector space then any set of N linearly independent vectors e 1 , e 2 , . .. , eN forms a basis for V. If x is an arbitrary vector lying in V then the set of N + l vectors x , e 1 , e 2 , . . . , eN , must be linearly dependent and t herefore such t hat

where t he coefficients a, /3, .. . , x are not all equal t o 0, and in particular x =I= 0 . Rearranging previous equation we may [email protected]

@Sk J ahiruddin, 2020

13

physicsguide CSIR NET, GAT E

Linear Algebra

Linear Algebra

@Sk J ahiruddin , 2020

write x as a linear sum of the vectors ei as follows: N

x

= x1e1 + x2e 2 + · · · + xNe N = L xiei

(1.10)

i =l

for some set of coefficients x i that are simply related to t he original coefficients, e.g. x 1 = -a/x, x 2 = -/3 /x, etc. Since any x lying in the span of V can be expressed in terms of t he base vectors ei, the latter are said to form a complete set. The coefficients Xi are the components of x wit h respect to t he ei -basis. These components are unique, since if both N

x

N

= L xiei

and

x

= L yiei

i=l

t hen

i =l

N

L (xi - Yi) ei = 0

(1.11)

i =l

which, since the ei are linearly independent , has only t he solution Xi = Yi for all i = 1, 2, ... , N From t he above discussion we see t hat any set of N linearly independent vectors can form a basis for an N -dimensional space. If we choose a different set e~, i 1, ... , N then we can write x as j [email protected]

@Sk J ahiruddin , 2020

14

physicsguide CSIR NET, GATE

Linear Algebra

@SIle of an inner product. In effect t he notion of an inneI 111 u I (a b) is a generalisation of the dot product to more a h •t 1·ac · vector spaces. Alternative notations for (alb) are . . ~ simply a •b .

(' I

l. I

'

Linear Algebra

@Sk J ahiruddin, 2020

2 x 3 m atrix as a linear combination of these six m atrices) . Since there are 6 basis vectors, t he dimension of t his space is 6.

1.4

Inner Product

The inner product of two vectors, denoted in general by

(a b), which is a scalar function of a and b. The scalar or dot product, a · b _ a b cos 0, of vectors in real t hreedimensional space (where 0 is the angle between t he vect ors), was introduced in t he last chapter and is an example of an inner product. In effect the notion of an inner product (a b) is a generalisation of the dot product to more abstract vector spaces. Alternative notations for (alb) are (a, b), or simply a ·b. The inner product has the following propert ies:

(i) (a b) = (b a)* (ii) (a Ah + µc) = A(a b) + µ(a c) We note that in general, for a complex vector space, (i)

j [email protected]

@Sl 0 for all A, so we may choose A = re- ia and require that , for all r

This means that t he quadratic equation in r formed by setting the RHS equal to zero must have no real roots. This, in t urn, implies that

which, on taking t he square root (all factors are necessarily posit ive) of both sides, gives Schwarz's inequality. Triangle inequality:

la+ bi


Linear Algebra

e1

e1 = A

e1 A

X2 -

(e1)t · X2 e1 A

X3 -

(e2) t · X3 e2 -

;►

A

e2

e2

=

e2

(e1) t · X3 e1 A

;>

A

e3

=

e3

le3

• •



Example: For the given sets of vectors, use the GramSchmidt method to find an orthonormal set. A = (0, 2, 0, 0), B = (3, -4, 0, 0), C = (1, 2, 3, 4)

Solution: e1

j [email protected]

= A / A = (0, 1, 0, 0)

26

physicsguide CSIR NET, GATE

@Sk J ahiruddin , 2020

Linear Algebra

e3= e3/ 1. 7

e 3 = (0, 0, 3, 4)/5

Linear function and Linear Operators

1. 7 .1

Linear function of ve ctor

A function of a vector, say

f (r ),

is called linear if

where a is a scalar.

Example: If F (r) = A x r for any given vector A, t hen is F (r) a linear vector function? Solution:

j [email protected]

~ n

1

T

1



1 1 •

nn n A

We know t hat F (r) is linear function if

27

physicsguide CSIR NET, GAT E

T



@Sk J ahiruddin , 2020

Linear Algebra

Hence

F(r1 + r2) = A x (r1 + r2) = A x r1 + A x r2 = F(r1) + F (r2) Again

F(ar) = A x (ar) = a(A x r) = aF(r) As both t he condit ions are satisfied then F (r) is a linear vector function.

1. 7.2

Linear Operator

A linear operator operat ing on linear combination of two vectors gives

A (Aa + µb) = AAa + µAb

(1.18)

where A, µ are scalars. Another equivalent d efinition is: 0 is a linear operator if

O(A + B )

=

O(A) + O(B )

[email protected]

28

and

O(kA)

=

kO(A)

physicsguide CSIR NET, GATE

Linear Algebra

@Sk J ahiruddin, 2020

We say that A 'operates' on x to give the vector y. We note that the action of A is independent of any basis or coordinate system and may be thought of as 'transforming' one geometrical entity (i.e. a vector) into another. Example: Is square root a linear operator? We are asking, is ✓A + B the same as ✓A + JB? The answer is no; taking the square root is not a linear operation. Example: Is taking the complex conjugate a linear operation? We want to know whether A + B = A + B and kA = kA. The first equation is t rue; t he second equation is t rue if we restrict k to real numbers.

2xD + 7 a linear operator? Where 2 d d 2 D and D stands for - and respectively. 2 2

Example: Is x D

2

-

dx

dx

Solution: We see that O is a linear operator if

O (A

+ B) =

O (A)

0 = x 2 D 2 - 2x D A(x) and B = B (x) j [email protected]

@Sk J ahiruddin , 2020

+ O (B)

and

O( kA)

= kO(A)

+ 7 so we take two function of x, A 29

=

physicsguide CSIR NET, GATE

Linear Algebra

Linear Algebra

@Sl< J a hiruddin , 2020

O(A

+B)=

2

(x D

2

-

2x D

+ 7)(A + B )

dA - 2 dA - x dx2 x dx -

2

+

7A

2 d2 B

dB + x dx2 - 2x dx

+7

B

= O(A) + O(B ) Similarly 2

O(kA ) = (x D

2

-

2xD + 7) (kA) = k(x D 2

2

-

2x D

+ 7)A

= kO(A ) As both t he condit ions are satisfied t hen O is a linear operator.

1. 7 .3

Properties of linear operators

If x is a vector and A and B are two linear operators t hen it follows that (A + B)x = Ax + Bx

(AA)x = A(Ax) (AB)x = A(Bx) where in t he last equality we see t hat t he action of two linear operators in succession is associative. The product of two j [email protected]

@Sk J a hiruddin , 2020

30

physicsguide CSIR NET , GATE

Linear Algebra

@Sk J ahiruddi11, 2020

Linear Algebra

linear operators is not in general commutative, however, so t hat in general ABx i= BAx. In an obvious way we define t he null (or zero) and identity operators by

Ox = 0

and

Ix

= x

for any vector x in our vector space. T wo operators A and B are equal if Ax = Bx for all vectors x. Finally, if t here exists an operator A- 1 such t hat

t hen A- 1 is the inverse of A . Some linear operators do not possess an inverse and are called singular, whilst those operators that do have an inverse are t ermed non-singular.

j [email protected]

@Sk J ahiruddin, 2020

31

physicsguide CSIR NET, GATE

Linear Algebra

@Sk J ahiruddin, 2020

2

Linear Algebra

Matrix: D efinition and Basic Properties

A M x N matrix is written as

A=

A11

A12

• • •

A 1N

A 21

A 22

• • •

A 2N

• • •

• • •

AMI

AM2







• • •

• • •

A 1111 N

Significance of matrix is deep. A matrix may be a vector, tensor , operator etc. We slowly dig into details. Whe never a m atrix is w ritten , it is w ritten with respect to some basis

Vectors are represented by column matrix

x=

• • •

We note t hat in a different basis e~ the vector x would be represented by a different column matrix containing t he j ahir@physi csgui de.in

@Sk J ahiruddin , 2020

32

physicsguide CSIR NET, GATE

Linear Algebra

Linear Algebra

@Sk J ahiruddin , 2020

components x~ in the new basis, i.e.

x' =

2.1

• • •

Basic Matrix algebra (A + B)ij = Aij + Bij (AA)ij = AAij

(AB)ij =

L AikB kj k

Example: The matrices A, B and C are given by

A=

2 - 1 3 1

'

Find the matrix D

[email protected]

(c)Sk J ahiruddin , 2020

B=

1 0 0 -2

- 2 1

'

-1 1

= A + 2B - C

33

physicsguide CSIR NET, GATE

Linear Algebra

Linear Algebra

@Sk J ahiruddin, 2020

Solution:

D=

2 -1 3 1

+2

1 0 0 -2

-2 1 -1 1

2 +2x l -(-2) -1+ 2 x 0 - 1 3+2x0-(-1) 1 + 2 x(- 2)-1

2.2

6 -2 4 -4

Multiplication:

The product AB of an M x N m atrix A with an N x R matrix B is itself an M x R matrix P, where N

Pij =

L AikB kj

for i = 1, 2, ... , M ,

j

= l , 2,

... , R

k =l

For example, P

= AB may be written in matrix form

I

[email protected]

@Sk J ahiruddin, 2020

A11

A12

A13

A21

A 22

A23

34

I

B 11

B 12

B 21

B 22

B 31

B 32

physicsguide CSIR NET, GATE

Linear Algebra

Linear Algebra

@Sk J ahiruddin , 2020

W here

= P 21 = P12 = P 22 = P 11

+ A12 B 21 + A13 B 31 A21B11 + A22B21 + A23B31 A1 1B12 + A 12 B 22 + A 13 B 32 A11 B 11

A21B12

+ A22B22 + A23B32

Mult iplication of more t han two matrices follows naturally and is associative. A(BC) _ (AB)C Mult iplication of matrices is not , in general, commutat ive. AB i= BA in general. T here are sp ecial cases when AB = BA

Example Evaluate P = AB and Q = BA where

A=

3 2 - 1 0 3 2 1 - 3 4

2 -2 3

'

B=

1

3

0 2 1 1

Solution: T he element ~ j of the matrix P = AB is found by taking t he 'scalar product of t he i th row of A with t he j th column of B. For example, P11 = 3 x 2 + 2 x 1 + (- 1) X 3 = 5, P 12 = 3 X (-2) + 2 X 1 + (- 1) X 2 = -6, etc. [email protected]

@Sk J ahiruddin, 2020

So we get

35

physicsguide CSIR NET, GATE

Linear Algebra

Linear Algebra

@Sk J ahiruddin , 2020

So we get

3

P = AB =

2

0 3 1 - 3

- 1

2 - 2 3 1 1 0 3 2 1

2 4

5 - 6 8 9 7 2 11 3 7

Similarly we get

Q = BA =

2 - 2 3 l 1 0 3 2 1

3

2

- 1

0

3

2

9 - 11 6 3 5 1

4

10

1 -3

9

5

These results illustrate t hat, in general, two matrices do not commute.

The property t hat matrix multiplication is distribut ive over addit ion, i.e. t hat

(A+ B )C = AC + B C;

[email protected]

@Sk J ahiruddin, 2020

C(A + B) = CA + CB

36

physicsguide CSIR NET, GATE

Linear Algebra

[email protected]

37

physicsguide CSIR NET 1 GATE

@Sk J ahiruddin , 2020

Linear Algebra

t he transpose of its complex conjugate, or equivalently, the complex conjugate of its t ranspose, i.e. Example: Find the complex conjugate of t he matrix 1 2 3i

A=

1+i 1 0

Solution: By taking the complex conjugate of each element we obtain immediately A* =

1 2 - 3i 1- i 1 0

We can prove that

Example: Find the Hermitian conjugate of the matrix 1 2 3i

A=

1+i 1 0

Solution: Taking t he complex conjugate of A and t hen forming the transpose we find

[email protected]

1

1- i

2 -3i

1

38

0 physicsguide CSIR NET, GATE

Linear Algebra

@Sk J ahiruddin, 2020

t he transpose of its complex conjugate, or equivalently, the complex conjugate of its transpose, i.e. Example: Find the complex conjugate of t he matrix 1 2 3i 1+i 1 0

Solution: By taking the complex conjugate of each element we obtain immediately A* =

1 2 -3i 1- i 1 0

We can prove that

Example: Find the Hermit ian conjugate of the matrix 1 2 3i A= 1+i 1 0

Solution: Taking t he complex conjugate of A and t hen forming the t ranspose we find

j [email protected]

1

1- i

2

1

-3i

0

38

physicsguide CSIR NET, GATE

Linear Algebra

@Sk J ahiruddin , 2020

Check th at you obtain the same result, of course, if you first t ake the transpose of A and then t ake the complex conjugate.

2 .5

Trace

Trance of a matrix is defined by N

Tr A =

A11

+ A 22 + · · · + A NN

=

L Aii i=l

Trace is linear Tr(A ± B)

= Tr A ± TrB

Trace of the product of two matrices is indep endent of t he order of their multiplication

N N N N N N TrAB = L(AB)ii = L L Aij B ji = L L Bjt Aij = L(BA) i =l

i= l j = l

i =l j =l

j=l

Trance is same in cyclic permutation. Tr ABC = Tr BCA = Tr CAB j [email protected]

@Sk J ahiruddin, 2020

39

physicsguide CSIR NET, GATE

Linear Algebra

@Sk J ahiruddin, 2020

2. 6

Linear Algebra

Determinant

You know how to evaluate t he determinant. For example

det A = A

A

A11

A 12

A13

A21

A 22

A 23

A31

A 32

A 33

= - A21 (A12 A 33 - A13A32)

+ A22 (A11A 33 -

A 13A31)

- A23 (A11 A 32 - A12A31)

= A1 1 (A22A33 - A23A32)

+ A13 (A21A32 -

+ A12 (A23A31 -

A21A33)

A22A31)

Properties of determinants

(i) Determinant of t he transpose: The transpose matrix AT has t he same determinant as A itself, i.e.

j [email protected]

@Sk J ahiruddin , 2020

40

physicsguide CSIR NET , GATE

Linear Algebra

@Sk J ahiruddin , 2020

Linear Algebra

A

(ii) Determinant of the complex and Hermitian conjugate:

(iii) Interchanging two rows or two columns: If two rows (columns) of A are interchanged, its determinant changes sign but is unaltered in magnitude. (iv) Common factor: If all t he elements of a single row (column) of A have a common factor, A , then this factor may be removed; the value of the determinant is given by t he product of the remaining determinant and A Clearly this implies that if all the elements of any row (column) are zero t hen A = 0.

It also follows t hat if every element of t he N x N matrix A is multiplied by a constant factor A t hen

AAI = AN A

j ahir@physi csguide.in

@Sk J ahiruddin , 2020

41

physicsguide CSIR NET, GATE

Linear Algebra

@Sk J ahiruddin , 2020

Linear Algebra

(v) Identical rows or columns: If any two rows (columns) of A are identical or are multiples of one another, t hen it can be shown that A = 0 . (vi) Adding a constant multiple of one 1~ow (column) to anot her: The determinant of a matrix is unchanged in value by adding to t he elements of one row (column) any fixed multiple of t he elements of another row (column). (vii) Determinant of a product: If A and B are square matrices of the same order t hen

AB l = IA B

BA

and in the extended version AB . .. G = A B I . . . G = IA IG . . . B I = IA .. . GB

It shows that the determinant is invariant under permutation of the matrices in a multiple product.

[email protected]

(c)Sk J ahiruddin. 2020

42

physicsguide CSIR NET , GATE

Lin ear Algebra

Linear Algebra

@Sk J ahiruddin , 2020

Example: Evaluate the determinant

2 -2

0

1

3

1 0 1 3 -3 4 -2 -2 1 - 2 - 1

A

Solution: Taking a factor 2 out of t he third column and t hen adding t he second column to t he third gives

1

A = 2

0

0 1

1 -1

3

1

0

1

3

1

0

1

0

1

= 2

3 -3 2 -2 -2 1 - 1 - 1

3 -3 - 1 -2 -2 1 0 - 1

Subtracting the second column from the fourt h gives

A =2

1

0

1

3

0

1

0

0 1

3

-3 - 1

-2 1

0

-2

We see t hat the second row has only one non-zero elej [email protected]

43

physicsguide CSIR NET, GATE

You can find t he inverse of a matrix by (i) Finding cofac44

j a [email protected]

@Sk J a hiruddin, 2020

physicsguide CSIR NET , GATE

Linear Algebra

tors (ii) Row and column operations. We physics students generally find the inverse by constructing first t he matrix C containing the cofactorsof t he elements of A. Then take the t ranspose of C and divide er the determinant of A.

For a 2 x 2 matrix, the inverse has a particularly simple form. If the matrix is

A= t hen it s determinant - A- is given by A = A11 A22 A12 A21 , and t he matrix of cofactors is

Thus t he inverse is

-

Linear Algebra

@Sk J ahiruddin , 2020

tors (ii) Row and column operations. We physics students generally find t he inverse by constructing first the matrix C containing t he cofactorsof t he elements of A. Then take the t ranspose of C and divide er the determinant of A.

A-1) (

= ( C)i = cki

Al

ik

Al

For a 2 x 2 matrix, the inverse has a part icularly simple form. If the matrix is

A= t hen its determinant -A- is given by A = A11 A22 A 12 A21 , and the matrix of cofactors is

Thus the inverse is

j [email protected]

@Sk J ahiruddin , 2020

45

physicsguide CSIR NET, GATE

Linear Algebra

Linear Algebra

@Sk J ahiruddin, 2020

A-1

=

1

cT

A

A11A22 -

A12A21

A 22

-A12

-A21

A11

Example: Find the inverse of the matrix 2

A=

4

3

l - 2 - 2 -3 3 2

Solution: We first determine A : A

= 2[-2(2) - (-2)3] + 4[(-2)(-3) - (1)(2)] + 3[(1)(3) - (- 2)(-3)] = 11

This is non-zero and so an inverse matrix can be constructed . To do this we need t he matrix of t he cofactors, C, and hence cT. We find

2 C

=

4 -3 1 13 - 18

- 2 7

[email protected]

@Sk J ahiruddin, 2020

and

-8

46

cT

=

2 4

1

- 2 7

13 -3 - 18 -8

physicsguide CSIR NET, GATE

Linear Algebra

47

j [email protected]

physicsguide CSIR NET, GATE

@Sk J ahiruddin, 2020

Linear Algebra

Let t he columns of an M x N matrix b e interpret ed as t he components in a given basis of N (M-component ) vectors v 1, v 2, ... , v N, as follows:

t

t

t

A=

Then the rank of A , denoted by rank A or by R( A), is defined as t he number of linearly independent vectors in t he set v 1, v 2, ... , v N, and equals the dimension of t he vector space spanned by t hose vectors. Alternatively, we may consider the rows of A to contain the components in a given basis of the M (N -component vectors w 1 , w 2 , . . . , w M as follows:

A=

• • •

The rank of A is also equal to the number of linearly indep endent vectors in the set w 1 , w2 , ... , w M . Rank of a matrix in generally determined by row reduction. It can also be det ermined by finding how much j [email protected]

48

physicsguide CSIR NET, GATE

Linear Algebra

@Sk J ahiruddin , 2020

Let t he columns of an M x N m atrix b e interpret ed as t he com ponents in a given basis of N (M-component) vectors v 1, v 2, . .. , VN, as follows:

t

t

t

Then t h e rank of A , d enoted by rank A or by R(A), is defined as t h e number of linearly indep en dent vectors in t he set v 1, v 2, ... , v N, and equals t he dimension of t he vect or sp ace spanned by t hose vect ors . Alt ernatively, we m ay consider t he rows of A t o contain t he compon ent s in a given basis of t h e M( N -component vectors w 1 , w 2 , ...

, WM

as fol-

lows:

• • •

The rank of A is also equal to the numb er of linearly in dep endent vectors in t he set w 1 , w 2 , ..

. , wM .

R ank of a m atrix in generally det ermined by row redu ct ion. It can also b e det ermined by finding h ow much [email protected]

@Sk J ahiruddin , 2020

48

physicsguide CSIR NET, GATE

Linear Algebra

@Sk J ahir11dd in, 2020

Linear Algebra

submatrices wit h non zero determinant can be found. YOll need to know t hat a su bmatrix of matrix A is any matrix t hat can be formed from t he elements of A by ignoring one, or more t han one, row or column. Now what is row reduction? Row reduction are done by using t hese st eps. 1) Interchange two rows 2) Mult iply (or divide) a row by a (nonzero) constant 3) Add a multiple of one row to another; t his includes subtracting, t hat is, using a negative multiple. Example : Make row reduction of the matrix

2 0 - 1 2 6 5 3 7 2 - 1 0 4 Solution: Step 1: Subtract 3 times t he first row from the second row and subtract t he first row from the third 2 0 - 1 2 row. T his gives: 0 5 6 1 0 - 1 1 2 [email protected]

@Sk J ahiruddin, 2020

49

physicsguide CSIR NET, GATE

Linear Algebra

@Sk J ahiruddin, 2020

Linear Algebra

Solution: 1 0 1 0 -1 -2 -1 0

The determinant of the matrix is =

2

2

5

3

2

4

8

6

0 1 0 - 1 -2 -1 0 2 2 2 3 1

2

4

2

0

1

6

0 0

- 1 -2 0 0 2 2

1

2 4 0

2

2

3 0

2

4

6 0

1

R3 - R1 R'4 -_ R42 =

=

-2

51

0

0 0

- 1 - 2 0 0 1 1

[email protected]

0 0

- 1 -2 0 0



R;

0 3 0 6

2 2

3 0 3 0

physicsguide CSIR NET, GATE

@Sk .Jahiruddin , 2020

Linear Algebra

R~ = R4 - R3

=

1 0 0 0 - 1 - 2 0 0

- 2

1

2

0

0

3 0 0 0

Now,

1 0 0 - 2 - 1 - 2 0 = - 2[- 2

1

2

X

3] = 12

3

So this 3 x 3 sub-matrix of t he main matrix have Non-Zero determinant. So, Rank of t he matrix is 3.

Example: Find t he rank of the matrix

1 2

2 - 1 O - 1

1 -2

4 1

0 -3

Solution: Row reduced echelon form will b e 1 2

2 0

1 -2

- 1 - 1

4 1

0

-3

1 0 - 1/ 2 1/ 2 0 1 -1/ 4 7/ 4 0 0 0 0

As the matrix is now have two non zero rows. It will have rank TWO .

j [email protected]

(c)Sk .Jahiruddin , 2020

52

physicsguide CSIR NET, GATE

Linear Algebra

j [email protected]

52

physicsguide CSIR NET, GATE

@Sk J ahiruddi11 , 2020

Linear Algebra

Example: Find the rank of

1 0 2 1

A=

0 2 4 2

0 2 2 1 Solution: After some elementary row operations we get

A=

1 0 2 1

1 0

2

1

0 2 4 2

0 2

4

2

0 2 2 1

0 0 -2 -1

We can't make any more reduction to make any of the more rows and columns to be zero. So the matrix have rank THREE.

Example: Find the rank of the matrix by checking the non singular submatrices.

1 1 0 -2 2 0 2

2

4 1 3

1

Solution: The largest possible square submatrices of A must be of dimension 3 x 3. Clearly, A possesses four such j [email protected]

53

physicsguide CSIR NET, GATE

j [email protected]

physicsguide CSIR NET, GATE

54

@Sk J ahiruddin, 2020

2.9

Linear Algebra

Consistency of solutions of linear equations

Suppose you are given t his set of equations

=2 6x + 5y + 3z = 7 2x -y = 4 2x

z

We write the linear equations in the form

Mr = k where

M=

2

0 -1

6

5

3

2 -1

0

X

r

=

y z

'

k=

2 7 4

Now we form t he augmented matrix which we call A . Note t hat t he fi1~st t hree columns of A are just t he columns of M , and t he fourth column is the column of constants on the right hand sides of the equations, i.e the k matrix. So

A= j ahir@physi csgui de.in

2 0 - 1 2 6 5 3 7 2 - 1 0 4 55

physicsguide CSIR NET, GATE

Linear Algebra

@Sk J ahiruddin, 2020

Now let 's consider the general problem of solving m equations in n unknowns. Then M has m rows (corresponding to m equations) and n columns (corresponding to n unknowns) and A has one more column (t he constants). The following summary . outlines t he possible cases. 1) If (rank M) < (rank A), the equations are inconsist ent and t here is no solution. 2) If (rank M ) = (rank A) t here is one solut ion.

=

n (number of unknowns),

3) If (rank M) = ( rank A ) = R < n, t hen R unknowns can be found in terms of the remaining n - R unknowns. .

.

Example: Write and row reduce t he augmented ma-

trix for the given set of equations to find out whether the given set of equations has exactly one solut ion, no solutions, or an infinite set of solutions.

[email protected]

@Sk J ahiruddin, 2020

56

physicsguide CSIR NET, GATE

Linear Algebra

@Sk J ahiruddin, 2020

Linear Algebra

(a)

X

=4 -y + 2z = 3

X

-y + 2z = 3

- x +y- z

Solution:

The characteristics inhomogeneous matrix is - 1

1

-1

4

1 1

- 1 -1

2 2

3 3

=

[R;

=

R3-R2]

- 1

1

- 1

4

1

-1

2

0

0

0

3 0

So, we get equations

-x + y - z = 4 X

0. x

. .. (i)

-y + 2z = 3

... (ii)

+ 0.y + 0. z = 0

··· (iii)

From equation (iii), we can say t he equation will have infinite number of solutions. (i)

+

(ii) ⇒

[email protected]

(c)Sk J ahiruddin, 2020

z = 7 and x = y - 11 .

57

physicsguide CSIR NET, GATE

Linear Algebra

Linear Algebra

@Sk J ahiruddin, 2020

(b) X

-y + 2z = 5

+ 3y - z = 4 X -y + 2z = 3

2x

Solution:

The characteristics inhomogeneous matrix is 1 - 1

2

2 3 - 1 1 - 1 2

5

4 3

1 - 1 =

[R;

=

R3 - R1] 2 0

3 0

2

5

- 1 0

4

-2

From the last line we are getting that, O.x

+ O.y + O.z =

- 2

This is an inconsistent equation. So there will be No Solution.

2.9.1

Homogeneous Equations

When the constants on the right hand sides are all zero ( k matrix is zero) then t he set of equation are called homogej [email protected]

@Sk J ahiruddin, 2020

58

physicsguide CSIR NET, GATE

Linear Algebra

Linear Algebra

@Sk J ahiruddin , 2020

x+y = O neous equations. Examples are , and x - y= O X -

2y + 3z = 0

+ 4y 2x + 2y X

6z

=

0

3z = 0

Homogeneous equations are never inconsistent ; t hey always have the solut ion ''all unknowns = O'' (often called the ''trivial solution'' ). If the number of independent equations (that is, t he rank of t he matrix) is t he same as t he number of unknowns, this is the only solution. If the rank of t he matrix is less t han the number of unknowns, there are infinitely many solut ions. Important t heorem to know: A system of n homogeneous equations in n unknowns has solutions other than the trivial solution if and only if t he det erminant of the co efficients is zero. Example: For what values of A does t he following set of equations have nontrivial solut ions for x and y? ( 1 - A)x

2x [email protected]

@Sk J ahiruddin , 2020

+ 2y = 0

+ (4 - -A)y = 0 59

physicsguide CSIR NET, GATE

Linear Algebra

[email protected]

59

physicsguide CSIR NET, GATE

@SI..

a 12 a22 -

• • •

• • •

a n1

a n2

>..

• • •

a1n

• • •

a2n

• • •

• • •

• • •

a nn -

=0 A

is called the characteristic equation of A. On expanding the determinant, the characteristic equation t akes t he form

where k' 's are expressible in terms of the elements a i j . The roots of this equation are called the eigenvalues or latent roots or characteristic roots of t he matrix A .

j ahir@physicsguide. in

@Sk J ahiruddin , 2020

74

physicsguide CSIR NET, GATE

Linear Algebra

j [email protected]

74

physicsguide CSIR NET ) GATE

@Sk J ahiruddin ) 2020

4.2

Linear Algebra

eigenvectors

Let A is an N x N matrix. Then t he column matrix x which satisfies A x = ,,\x is called the eigenvector of the matrix A. Let us illustrate with an example. Example: Find t he eigenvalues and eigenvectors of t he 5 4 matrix

1 2

Solution: The characteristic equation is [A - .X] 5-,,\ 4 1 2 - ,,\ or

=0

or

=0

(.X-6)(.X -1 ) = 0

• • •

.X = 6 1 '

Thus the eigenvalues are 6 and 1 If x, y be the components of an eigenvector corresponding to t he eigen value ,,\ , then

j [email protected]

75

physicsguide CSIR NET ) GATE

[email protected]

physicsguide CSIR NET, GATE

75

@Sk J ahiruddin , 2020

Linear Algebra

[A - ,\J]X

for

=

5- ;\ 4 1 2- ,\

- 1

A= 6, we have

X

=0

y

4

X

1 -4

y

=0

This gives only one indep endent equation -x + 4y

=

0.

Now we choice. We can choose any of the value of either x or y . It is a general convent ion t o take he sm aller value to b e 1. We choose y

=

1 here . So we get the eigenvector

4

1

for

A = 1, ,ve h ave

4 4

X

1 1

y

=0

This gives only one indep endent equation x we choose x

= 1, then

+y =

0. If

we get the eigenvector

1 - 1 [email protected]

76

physicsguide CSIR NET, GATE

- 1 76

j [email protected]

physicsguide CSIR NET, GATE

@Sk J ahiruddin, 2020

Linear Algebra

. We must keep into mind that the eigenvectors obtained are not normalized. To normalize we need to divide by t heir modulus. The normalized eigenvectors are

1

v'17

1

4 , 1 •

~

4 1

We will discuss a lot about the physical significance of t he eigenvalue and eigenvectors as we proceed furt her. Let us first start wit h the properties of eigenvalues and eigenvectors of the special matrices.

4.3

Eigenvalue and eigenvectors of normal matrices

We have already told the t he normal matrices are the mat rices which follows At A = AA t . We also showed that both Hermitian and unitary matrices (or symmetric and orthogonal matrices in t he real case) are examples of normal matrices. We now discuss t he properties of the eigenvectors and eigenvalues of a normal matrix. 77

If x is an eigenvector of a normal matrix A with corresponding eigenvalue A then A x = Ax, or equivalently,

(A - A]_)x = 0 Taking B = A - A]_ . Then the previous eq becomes B x= 0 and, taking the Hermitian conjugate, we then have

However, the product BtB is given by

B tB = (A - A]_)t(A - A]_)= (At - A*) (A - A]_)

= At A - A* A - AA t + AA* Now since A is normal, AAt = At A and so

and hence B is also normal. We then find

[email protected]

@Sk J ahiruddin , 2020

hence

78

physicsguide CSIR NET, GATE

Linear Algebra

78

[email protected]

physicsguide CSIR NET, GATE

@ Sk J ahiruddin, 2020

Linear Algebra

hence

Therefore, for a normal matrix A, t he eigenvalues of At are the complex conj ugates of t he eigenvalues of A. Let us . . now consider two eigenvectors xi and x 1 of a normal matrix A corresponding to two different eigenvalues >..i and Aj . We t hen have

. Ax1

.

== Aj X1

Mult iplying the second equation on the left by (xi) t we obtain

(xi) t AxJ == Aj (xi ) t xj On t he LHS use (AB • • • G)t == e t ••• Bt At and the property just proved for a normal matrix to writ e At xi == >..;xi

From the previous two equations we get

j [email protected]

79

physicsguide CSIR NET, GATE

Example: Prove t h at a normal m atrix A can be written 80

[email protected]

physicsguide CSIR NET, GAT E

@Sk J ahiruddin , 2020

Li11ear Algeb ra

in t erms of its eigenvalues Ai and orthonormal eigenvectors . xi as N

A= L ..\ixi (xi)t i =l

Solution: You can show t h at bot h sides of the expression give t he same result when acting on an arbitrary vector

y. Since A is normal, we m ay expand y in t erms of the eigen. vectors x i , as shown in previous equation. Thus, we h ave

Ay

=

N

N

i= l

i =l

AL a,ixi = L ai..\ix i

Alternatively, t he action of LHS on arbit rary vector y is

i= l

since ai

= (x i) t

i= l

y.

We see t h at the two expressions

for t h e action of each side of the given equation on y are ident ical; which confirms that t h e equation is correct .

jahir@physicsguide. in

82

physicsguide CSIR NET, GATE

@Sk J ahiruddin , 2020

Linear Algebra

But (xi ) t xi is t he modulus squared of t he non-zero vec. 2 tor x and is t hus non-zero. Hence Al must equal Ai and t hus be real. The same argument can be used to show t hat t he eigenvalues of a real symmet ric matrix are t hemselves real. Since an Hermitian matrix is also a normal matrix, its e ige nvectors are orthogonal ( or can b e made so using the Gram- Schmidt orthogonalisation procedure ). Alternatively we can prove the orthogonality of the eigenvectors directly. Proof: Consider two unequal eigenvalues t heir corresponding eigenvectors satisfying

Ai

and

Aj

and

Axi = Aixi Axj

=

A j Xj

Taking t he Hermit ian conjugate of t he first equation we find (xi) t At= Ai (xi) t . Mult iplying t his on t he right by x J we obtain

j [email protected]

83

physicsguide CSIR NET, GAT E

@ Sk J ahiruddin , 2020

Linear Algebra

But ( xi ) t xi is t he modulus squared of t he non-zero vec.

2

t or x and is t hus non-zero. Hence Al must equal Ai and t hus be real. The same argument can be used to show t hat t he eigenvalues of a real symmetric mat rix are t hemselves real.

Since an H ermitian matrix is also a normal matrix, its e ige nvectors are orthogonal ( or can b e made so using the Gram- Schmidt orthogonalisation procedure ). Alternatively we can prove the orthogonality of the eigenvectors directly. Proof: Consider two unequal eigenvalues Ai and AJ and t heir corresponding eigenvectors satisfying

= Aixi Axj = A j X j Axi

Taking t he Hermit ian conjugat e of t he first equation we find (xi) t At = Al (xi) t . Mult iplying t his on t he right by x J we obtain

[email protected]

@Sk J ahiruddin , 2020

83

physicsguide CSIR NET, GATE

Linear Algebra

Linear Algebra

@Sk J ahiruddin, 2020

Therefore mat rices that are anti-Hermit ian are also normal and so have mut ually orthogonal eigenvectors. T he propert ies of t he eigenvalues are also simply deduced , since if Ax = AX t hen

Hence A* zero).

= - A and so A must be pure imaginary ( or

In a similar manner to t hat used for Hermit ian matrices, t hese propert ies may be proved directly.

4.5

Eigenvectors and eige nvalues of a unitary matrix

A unitary matrix satisfies At = A- 1 and is also a normal matrix, wit h mutually ort hogonal eigenvect ors. To invest igate t he eigenvalues of a unitary mat rix, we note that if Ax = AX then

[email protected]

@Sk J ahiruddin , 2020

85

physicsguide CSIR NET, GATE

Linear Algebra

@Sk J ahiruddin , 2020

Linear Algebra

and we deduce that AA* = A 2 = 1.

Thus, the eigenvalues of a unitary matrix have unit modulus.

4.6

Simultaneous eigenvectors and commutation of matrices

Two different normal matrices can have a common set of eigenvectors if, and only if, they commute Proof: Let A and B be two N x N normal matrices and . xi be t he i th eigenvector of A corresponding to eigenvalue Ai, i.e. .

.

Ax2 = Aixi

for

i = 1, 2, .. . , N

Init ially we assume that the eigenvalues are all different (i) F irst suppose that A and B commute. Now consider .

ABxi

.

=

BAxi

.

=

B Aixi

=

AiB x

.

2

where we have used the commutativity for the first equality j [email protected]

@Sk J ahiruddin, 2020

86

physicsguide CSIR NET, GATE

Linear Algebra

@Sk J ahiruddin, 2020

Linear Algebra

combination of the eigenvectors, N

x= L

cixi

i= l

Now consider the following N

ABx = AB L

N

cixi = A L

N

ciµixi =

L ci>..iµixi

i =l

i= l

i= l

N

N

N

and BAx = BAL Cix'i = B L i= l

CiAiXi = L

i =l

ciµi>..ixi

i =l

It follows t hat ABx and BAx are t he same for any arbit rary x and hence t hat (AB - BA)x

=0

for all x . That is, A and B commute. It should be not ed t hat if an eigenvalue of A, say, is degenerate then not all of its possible sets of eigenvect ors will also const it ut e a set of eigenvectors of B. However, provided t hat by taking linear combinations one set of joint eigenvect ors can be found , the proof is still valid and t he result still holds. j [email protected]

@Sk J ahiruddin , 2020

88

physicsguide CSIR NET, GATE

Linear Algebra

Linear Algebra

@Sk J ahiruddin , 2020

combination of the eigenvectors, N

x

= L cix i i =l

Now consider the following N

ABx = AB L

N

N

cixi = AL ciµixi =

L ci>..iµixi

i= l

i =l

i =l

N

N

N

and BAx =

BAL i= l

Ci Xi

=

BL

CiAiXi =

i= l

L CiµiAiXi i =l

It follows t hat ABx and BAx are t he same for any arbit rary x and hence that

(AB - BA)x = 0 for all x. That is, A and B commute. It should be noted t hat if an eigenvalue of A , say, is degenerate then not all of its possible sets of eigenvectors will also constit ute a set of eigenvectors of B. However , provided t hat by taking linear combinations one set of joint eigenvect ors can b e found, the proof is still valid and the result still holds. jahir@physicsguide. in

@Sk J ahiruddin , 2020

88

physicsguide CSIR NET, GAT E

Linear Algebra

88

j [email protected]

physicsguide CSIR NET, GATE

@Sk J ahiruddin, 2020

4. 7

Linear Algebra

Cayley Hamilton and General properties of eigenvalues

We know that a scalar,\ is an eigenvalue of the matrix A if and only if ,\ is a solution to t he characteristic equation det(A - -\n) = 0

This is called characteristic equation and will give a polynomial of n degrees for n x n matrix. Now Cayley Hamilton t heorem says every matrix follows its characterist ic polynomial. For illustration I say that if characteristic equation of matrix A is

t hen t he matrix follows aA + bA + cA + dn = 0 2

3

4 . 7 .1

Some properties of eigenvalues

(i) eigenvalues and eigenvectors of a shift ed matrix A - an : A x = -\x j [email protected]

.,..-...,..... ...

- . . . ..

,_,..,...,



(A - an)x = (-\ - a)x 89

....

physicsguide CSIR NET, GATE

.

,

.

@Sk J ahiruddi11, 2020

Linear Algebra

(ii) eigenvalues and eigenvectors of A- 1

:

Ax = AX (iii) The eigenvalues of A and AT are the same (as their characteristic polynomials are the same), but there is no simple relationship between t heir eigenvectors. (iv) eigenvalues and eigenvectors of powers of A :

In general, the eigenvalues of Ak are Ak, and t he eigenvectors are t he same as t hose of A . (v) The eigenvalues of a triangular matrix are clearly its diagonal entries. The eigenvalues of a block triangular matrix (vi) If A is a real matrix with a complex eigenvalue A = µ+iv and corresponding complex eigenvector v = x+ iy, t hen the complex conjugate A* = µ- iv is also an eigenvalue with complex conjugate eigenvector v = x - iy (vii) The sum of the eigenvalues of a matrix equals its t race j [email protected]

90

pbysicsguide CSIR NET, GATE

Linear Algebra

@Sk J ahiruddin , 2020

(viii) The product of t he eigenvalues is t he determinant of the matrix.

4 . 7 .2

Some examples to evaluate eigenvalues

Example: Find the eigenvalues and normalised eigenvectors of the real symmetric matrix

3 l 1 -3 3 -3 - 3 1

A=

1

Solution: The characteristic equation is 1- A

l

1

1- A

3 -3

3

-3

-3 - ,\

=0

Expanding out t his determinant gives

(1 - A) [( l - A)(-3 - A) - (-3)(-3)]

+ 1[(-3)(3) - 1(-3 - A)] + 3[1(-3) - (1 - A)(3)] = 0 Simplify to get

[email protected]

@Sk J ahiruddin, 2020

91

physicsguide CSIR NET, GATE

Linear Algebra

Xl

=

J2' J2'

j [email protected]

0

v'2

1 1 0

physicsguide CSIR NET, GAT E

92

@Sk J ahirt1ddin , 2020

Linear Algebra

Repeating the last paragraph, but with the factor 2 on t he RHS replaced successively by ,\ 2 = 3 and ,\ 3 = - 6, gives two further normalised eigenvectors x

2

1

= - (1

x

v'3

3

1

= - (1

v'6

- 1

In t he above example, t he t hree values of ,\ are all different and A is a real symmetr·ic matrix. Thus we expect, and it is easily checked, t hat t he t hree eigenvectors are mut ually ort hogonal, i.e.

Example: (Degenerate eigenvalue) Find t he eigenvalues of the matrix and then construct an orthonormal set of eigenvectors for the matrix

A=

[email protected]

1 0 3 0 -2 0 3 0 1

93

physicsguide CSIR NET, GAT E

Repeating the last paragraph, but with the factor 2 on t he RHS replaced successively by ,\ 2 = 3 and ,\ 3 = -6, gives two furt her normalised eigenvectors x

2

1

= - (1

x

v'3

3

1

= - (1

v'6

- 1

In t he above example, t he t hree values of,\ are all different and A is a real symmetr·ic matrix. Thus we expect, and it is easily checked, t hat the t hree eigenvectors are mutually orthogonal, i.e.

Example: (Degenerate eigenvalue) Find the eigenvalues of the matrix and t hen construct an orthonormal set of eigenvectors for the matrix

A=

[email protected]

1 0 3 0 -2 0 3 0 1

93

physicsguide CSIR NET, GATE

@Sk J ahiruddin, 2020

Lii1ear Algebra

Characteristic equation

0

3

@Sk J ahiruddin, 2020

Linear Algebra

Characteristic equation

0

0 - 2 - ,\

3 0

3

0

1- ,\

1 - ,\

0=

2

= -(1 - -\) (2 + ,\) + 3(3)(2 + ,\) 2 = (4 - -\)(,\ + 2) Thus ,\ 1 = 4, ,\ 2 = - 2 = -\ 3 . The eigenvector for ,\ 1 = 4 1

is found from x =

T

X1

x2

X3

1 0 3 0 -2 0 3 0 1 This matrix equation gives three linear equations which say t hat x = z and y = 0. Hence we choose an engenvector of the form (x, 0, x) and get by normalization

X

j [email protected]

@Sk J ahiruddin, 2020

1

1

=-

y12

94

1 0 1

physicsguide CSIR NET, GATE

Linear Algebra

94

[email protected]

physicsguide CSIR NET, GATE

@Sk J ahiruddin, 2020

Linear Algebra

1 0 3 0 -2 0 3 0 1 This matrix equation gives three linear equations which says x- = -z and y is arbitrary. So we need to construct two vectors with t he general form (x, y, -x) which will be 1, 0, - 1). orthogonal to the first eigenvector

( 12

Now we see that a general column vector that is orthogonal to x 1 is T x = a b -a This is well fitted with our general form of next two eigenvectors which is (x, y, -x) . So we choose t he first one to be x2

=

T

0 1 0

and t he second one X

j [email protected]

3

1 =~

T

- 1 0 1

95

physicsguide CSIR NET, GATE

j a [email protected]

physicsguide CSIR NET , GATE

95

@Sk J ahirt1ddin, 2020

Linear Algebra

We have taken these two eigenvectors in the form that they are mutually orthogonal and both are orthogonal to t he first eigenvector also as asked in the question.

We don't always get n eigenvectors for a n x n matrix. For example try to find out the eigenvectors of the matrix.

1 2 1 A=

l - 1 1

2

0

1

Solution: The characteristic equation of the matrix A

=

1 2 1 1 - 1 1

2

0



IS

1 3

2

2

0 = det (A - Al) = -A + A + 5A + 3 = -(A + 1) (A - 3) There is a double eigenvalue A1 = - 1 and a simple eigenvalue A2 = 3. In case of A1 = 2 we get

A-Ail= A+I =

2 2 1

2 2 1

X

1 0 1 2 0 2

1 0 1

y

2 0 2

z

So we have the only choice

(-2y, y, 2y)T

=0

@Sk J ahiruddin , 2020

Linear Algebra

eigenvectors are respectively

l T )2 (1, 1, 0) l T ~(1 , -1, 1) and

l T v'6(1 , - 1, -2)

(b) Eigenvalues are 8, - 1, - 1. Eigenvector of corresponding eigenvalue 8 is ½(2, 1, 2)r. Any two vectors orthogonal to (2, 1, 2)T and which are themselves orthogonal would be t he eigenvectors of eigenvalue - 1, for example

1

l T )2(1, 0, -1) and

l T v'18(1, - 4, 1) 8

(c) Eigenvalues are 4, -2 , -2 Eigenvector of corresponding eigenvalue 4 is }z(l , 0, l )T Any two vectors orthogonal to }z(l , 0, l )T and which are themselves orthogonal would be t he eigenvectors of eigenvalue -2, for example }2(-1, 0, l)T and (0, 1, o)T [email protected]

@Sk J ahiruddin, 2020

98

physicsguide CSIR NET, GAT E

Linear Algebra

5

Similarity transformations and diagonalization

5.1

Change of basis

Suppose we take a basis e i : i = 1, 2, ... , N , into our N -dimensional vector space then we may write a vector like

We represent x in this basis by the column matrix as T

We now consider how these components change as a result of a prescribed change of basis. Let us introduce a new basis e ~ : = 1, 2, . . . , N which is related to t he old basis by N

eJ= L S-ijei i=l

Sij

eJ

t he coefficient is the i th component of with respect to t he old (unprimed) basis. For an arbitrary vector x it follows t hat N

x

=

N

J\T

N

L xiei = L xJeJ= L xJ L sijei i =l

[email protected]

@Sk J ahiruddin, 2020

j =l

j =l

99

i =l

physicsguide CSIR NET, GATE

Linear Algebra

From this we derive t he relationship between the compo-

From t his we derive t he relationship between t he components of x in the two coordinate systems as N

xi

=

L sijXJ j =l

which we write in t he matrix form as

x = Sx' where S is t he transformat ion matrix associated wit h the change of basis. Furthermore, since t he vectors e3 are linearly independent , the matrix S is non-singular and so possesses an inverse s- 1 . Multiplying x = Sx' on the left by s- 1 we find X

' = s-1X

which relates the components of x in t he new basis to t hose in the old basis Comparing x' = s- 1x and ej = L :1 s ij e i we note that the components of x transform inversely to the way in which t he basis vectors e i themselves t ransform. This has to be so as the vector x itself must remain unchanged.

j [email protected]

100

physicsguide CSIR NET, GATE

@Sk J ahiruddin, 2020

5.2

Similarity transformation

Linear Algebra

Linear Algebra

@Sk J ahiruddin , 2020

5.2

Similarity transformation

We may also find the transformation law for the components of a linear operator under t he same change of basis. Now, t he operator equation y = Ax (which is basis independent) can be written as a mat rix equation in each of the two bases as y

= AX,

y'

= A' X'

But, using x = Sx' we may rewrite t he first equation as

y' =

Sy'= ASx'

s- ASx' 1

Comparing this with y' = A'x' we find that t he components of the linear operator A transform as (5.1) Equation 5. 1 is an example of a similarity transformat ion - a transformation t hat can be particularly useful in converting matrices into convenient forms for computation. Given a square matrix A , we may interpret it as representing a linear operator A in a given basis e i . From equat ion 5 .1 however , we may also consider the matrix A' = j [email protected]

@Sk J ahiruddin , 2020

101

physicsguide CSIR NET, GATE

Linear Algebra

e same as Hence The eigenvalues are same v ........

j [email protected]

physicsguide CSIR NET, GATE

102

@Sk J ahiruddin, 2020

Linear Algebra

s- 1As -

A' - ,\ ].

,\]. = s - 1 (A - Ali )S

s- 1 s1A -

,\]. = A - ,\].

(iv) The value of t he trace is unchanged: as we know t hat the sum of t he eigenvalues is the t race

TrA' =

1

LA:i = LLL (s- )ij Ajkski i

i

j

k 1

= LLL ski (s- )i_j Ajk i

=

j

k

LL 6kj Ajk = LAjj = j

k

TrA

j

(v) You must know t hat A and A' have different sets of eigenvectors in general.

5.2.1

Unitary transformation

An important class of similarity transformations is that for which S is a unitary matrix; in this case A' = s- 1AS = st AS. Unitary transformation matrices are particularly important , for the following reason. If the original basis e i

j [email protected]

102

physicsguide CSIR NET, GAT E

@Sk J ahiruddin, 2020

Linear Algebra

s- 1AS- ..\li

A' - ..\ li

s- 1 s1A -

=

s-1 (A -

..\li)S

..\]_ = A - ..\]_

(iv) The value of t he trace is unchanged: as we know t hat t he sum of t he eigenvalues is t he t race

TrA'

=

LA~i = LLL (s- )ij Ajkski 1

i

=

k

LLL ski (s- ) ijAjk 1

i

=

j

i

j

k

L L 6kjAjk = L Ajj = Tr A j

j

k

(v) You must know that A and A' have different sets of eigenvectors in general.

5.2.1

Unitary transformation

An important class of similarity t ransformations is that for which S is a unitary matrix; in this case A' = s- 1 AS =

st AS. Unitary transformation matrices are particularly important, for the following reason . If t he original basis ei j [email protected]

103

physicsguide CSIR NET, GATE

@Sk J ahiruddin , 2020

Linear Algebra

is orthonormal and t he transformation matrix S is unitary t hen t he new basis is also ort honormal. This can b e proved as follows.

(e~e3) = L skiekl L srjer r

k

= L ski L Srj (ek er) r

k

=

L szi L SrjOkr = L sziskj = (sts)ij = Oij r

k

k

Furt hermore, in addition to the properties of general similarity t ransformations, for unitary transformations the following hold. (i) If A is Hermit ian or ant i-Hermitian then A' is also Hermit ian or anti-Hermitian respectively, i.e. if At = ±A t hen

(ii) If A is unitary (so t hat At

= A- 1 ) then A' is unitary,



since

(A')t A' = (stAs) t (stAs ) = = jahir@physicsguide. in

@Sk J ahiruddi11, 2020

stAtAs = s- 1 ].s = 104

stAts stAs ]_

physicsguide CSIR NET, GATE

Linear Algebra

tained in the matrix M. We say that this matrix is an operator which maps the plane into itself. In three dimension this argument can be easily extended, t he mapping then becomes from one point in three dimensional space to another point in t he three dimensional space. j [email protected]

105

physicsguide CSIR NET, GATE

@Sk J ahiruddin , 2020

Linear Algebra

This transformation from one set of points to another set of points which are caused by the matrix M can be a rotation, reflection or any other kind of transformation. The special case of a linear transformation which preserves the length of a vector is called Orthogonal Transformations. We can easily p1·ove t hat if the matrix M is orthogonal t hen it preserves the length of a vector. det M = l corresponds geometrically to a rotation, and det M = - 1 means that a reflection is involved.

All matrices need not to be orthogonal. But every matrix cause some deformation on t he set of points upon which it acts. Now A' = s- 1 AS is the matrix which describes in the (x', y') system the same deformation that A describes in the (x, y) system.

5 .3

Diagonalisation of matrices

@Sk J at1iruddin , 2020

Linear Algebra

This transformation from one set of points to another set of points which are caused by the matrix M can be a rotation, reflection or any other kind of transformation. The special case of a linear t ransformation which preserves the length of a vector is called Orthogonal Transformations. We can easily prove t hat if the matrix M is orthogonal then it preserves the length of a vector.

det M = l corresponds geometrically to a rotation, and det M = - 1 means that a reflection is involved. All matrices need not to be orthogonal. But every mat rix cause some deformation on t he set of point s upon which it acts. Now A' = s- 1 AS is the matrix which describes in the (x' , y') system t he same deformation t hat A describes in t he (x, y) system.

5.3

Diagonalisation of matrices

Suppose t hat a linear operator A is represented in some basis e i : i = 1, 2, . . . , N by t he matrix A. Consider a new

[email protected]

@Sk J ahiruddin, 2020

106

physicsguide CSIR NET, GATE

Linear Algebra

Linear Algebra

@Sk J ahiruddin , 2020

basis

.

xJ

given by N

x j

L

=

s ij e i

i =l

where the x J are chosen to be the eigenvectors of t he linear operator A , i.e. .

AxJ

=

. Aj XJ

In the new basis, A is represented by the matrix A' = s-1AS, which has a part icularly simple form, as we shall see shortly. The element S ij of S is t he i th component, ( in t he old (unprimed) basis,) of t he j t h eigenvector xi of A , i.e. the columns of S are the eigenvectors of t he matrix A :

t t S=

[email protected]

xl

x2

t ...

XN

107

physicsguide CSIR NET, GATE

Linear Algebra

@Sk J ahirudd in, 2020

.

.

.

Linear Algebra

@Sk J ahiruddin, 2020

t hat is, Sij = (xj )

i .

Therefore A' is given by

L L (s- ) ik A kz Szj = L L (s-l ) ik A kl (xj ) = L (s-1 )ik Aj (xj ) = L Aj ( s- ik s kj = Aj bij

1

1

(s- As)ij =

k

l

l

k

l

k

k

1

)

k

So the matrix A' is diagonal with the eigenvalues of A as t he diagonal elements,

A' =

A1 0 0 A2 •

• • •

0 • • •



• •

0

• • •



0



0

AN

So, given a matrix A, if we construct the matrix S that has t he eigenvectors of A as its columns t hen the matrix A' = s- 1 AS is diagonal and has t he eigenvalues of A as its diagonal elements. We require S to be non-singular ( S # 0), t he N eigenvectors of A must be linearly independent and form a basis for the N -dimensional vector space. [email protected]

@Sk J ahirudd in, 2020

108

physicsguide CSIR NET, GATE

Linear Algebra

@Sk Jahiruddin, 2020

Linear Algebra

It may be shown that any matrix with distinct eigenvalues can be diagonalised by this procedure. If, a general N x N square matrix has degenerate eigenvalues then it may, or may not, have N linearly independent eigenvectors. If it does not then it cannot b e diagonalised.

For normal matrices (which include Hermitian, anti-Hermitian and unitary matrices) the N eigenvectors are indeed linearly independent. Moreover, when normalised, these eigenvectors form an orthonormal set (or can be made to do so). Therefore the matrix S with these normalised • eigenvectors as columns, 1.e. whose elements are S i.1 = (xJ) i , has t he property

(sts) ij =

L

(st)ik (S)kj

=

L

k

=

t j ' i ) X ( X

k

ski skj

=

L

(xi): (xj)k

k

~ = Uij

Hence S is unitary (s- 1 = can be diagonalised by

st)

and the original matrix A

Therefore, any normal matrix A can be diagonalised by j [email protected]

(c)Sk J ahiruddin , 2020

109

physicsguide CSIR NET, GATE

Linear Algebra

@Sk J ahiruddin, 2020

Linear Algebra

a similarity transformation using a unitary transformation matrix S . Example Diagonalise t he matrix

1 0

A=

3

0 -2 0 3 0 1

Solution: The matrix A is symmetric and so may be diagonalised by a t ransformation of the form A' = stAS where S has the normalised eigenvectors of A as its columns. We have already found t hese eigenvectors in a previous example.

X

1

1 = -

J2

1

0 1

X

2

= -

0

1

J2

x3 = -

1

1

J2

0

- 1 0

1

So t he S matrix which is obtained by writ ing t he eigenvectors in the columns

s=

1

v'2

1

0

- 1

0

v'2

0

1

0

1

Alt hough the eigenvalues of A are degenerate, its t hree eigenvectors are linea1·ly independent and so A can still be j [email protected]

@Sk J ahiruddin, 2020

110

physicsguide CSIR NET, GATE

Linear Algebra

Linear Algebra

@Sk J ahiruddin, 2020

diagonalised. Thus, calculating 1

0

1

0

v'2

0

- 1

0

1

4

0

stAS we obtain

1 0 3 0 -2 0 3 0 1

1

0

- 1

0 1

v'2

0 1

0

0

0 -2 0 0 0 -2 You can see that the matrix has the eigenvalues of A in its diagonal elements.

6

Function of matrices

Function of matrices can be obtained by Two methods. 1) By using similarity transformation properties and 2) Expanding t he matrix in series and using Cayley Hamilton t heorem. We will discuss both.

[email protected]

@Sk J ahiruddin, 2020

111

physicsguide CSIR NET, GATE

Linear Algebra

111

[email protected]

physicsguide CSIR NET, GATE

@Sk J ahirudd in, 2020

6.1

Linear Algebra

Function of matrices by similarity transformation

If a m atrix A is diagonalized by t h e similarity t ransforma1AS , so that A'= diag (A1, A2, .. . , AN), then t ion A'=

s-

we know

N

Tr A' = Tr A =

N

L Ai

A' I = IA I =

i =l

IT Ai i =l

If a m atrix can be diagonalized by similarity transfor1AS, t h en we h ave m ation A' =

s-

Now we illustrate t h e use of equat ion (6.1) to find funct ion of matrices. but b efore we begin we need to know one important result

Important: If a matrix is diagon al, say A

=

X

0

0 y

then j ahir@physicsguide. in

112

physicsguide CSIR NET, GATE

j [email protected]

112

physicsguide CSIR NET, GATE

Linear Algebra

@Sk J al1iruddin, 2020

f (A)

=

0

0 f (y)

0

X

Proof: Let A =

J(x)

0 y xn 0 0 yn

We can prove An=

by induction on n

In case n = 1 it is true by t he definition of A Suppose that Ak =

X

. Then we have

0

X

0 y

k+l

0

Thus our hypothesis is true by induction method. Then for any function series

f (A)

which can be expanded in



[email protected]

113

physicsguide CSIR NET, GATE

j [email protected]

113

physicsguide CSIR NET I GATE

@Sk J ahiruddin , 2020

Linear Algebra

The expansion in series is valid for matrix also

Hence X

2

x 0 0 y2

0

0 y

x3 0 0 y3

+ ...

or

=

f( A) 2 3 ao + a1x + a2x + a3x + ...

0 2 ao + a1y + a2y + a3y 3 + ...

0

Which can b e easily seen as

J(A) =

J(x) 0

0 J(y)

Example: Consider the matrix

A=

[email protected]

5 -2 -2 2

114

physicsguide CSIR NET, GATE

j [email protected]

114

physicsguide CSIR NET, GATE

@Sk Jahiruddin, 2020

Linear Algebra

ou can easily verify t hat t he eigenvalues are 1 and 6 and the corresponding eigenvect ors are 1

-2

v'5

v'5

2

1

v'5

v'5

So upon diagonalizing we get

s- As = A' 1

S = l

where

)

and

v'5

A' =

1 - 2 2 1

1 0 0 6

Then for any function consisting of sums of powers of A , we have

J(A) = J (sn s- ) = Sf(A')s= S diag (Ai, A2, ... , An) s-1 1

1

where t he Ai are t he eigenvalues of A , and diag is a diagonal matrix, whose diagonal entries are given as t he arguments. T he second step above follows from t he observat ion t hat

(SA' s- )n = $A' s- s A' s- s A' s1

1

1

1

...

SA' s- : = SA'ns-

1

V

n terms j [email protected]

115

physicsguide CSIR NET, GAT E

j [email protected]

physicsguide CSIR NET, GATE

115

@Sk J ahiruddin , 2020

Linear Algebra

A pp lying t his to any function of matrix A we get f(A) = S

1 5

1 -2 2 1 1

1 -2

5

2

-

f (1)

0

0

f(6)

s - 1

f (1)

0

0

f(6)

1

2

-2 1

f (1)

2f (1) -2f(6) f(6)

1

f(l) + 4f (6) 2f( l ) - 2f (6) 2f (1) - 2f (6) 4f (1) + f(6)

1 5

We apply t his result in two cases

A4

=~ 5

1

4

+4 · 6

2 . 14

4

4

2·1

2 . 64

-

l e =5 A

4.

14

2·6

4

1037 -518 - 518 260

+ 64

e + 4e 2e - 2e 1 1 6 6 2e - 2e 4e + e 1

6

1

6

e -

5

Now we will prove trace formula j [email protected]

116

physicsguide CSIR NET, GATE

116

j ahir@physicsguide. in

physicsguide CSIR NET, GATE

@Sk J ahiruddin , 2020 6.1.1

Linear Algebra

Trace formula

From equation (6. 1) we get exp A' =

s-

1

(expA)S.

By choosing t he similarity transformation which diagonalizes A , we get A' = diag ( A1, A2, . .. , AN) , and so

exp A = exp A' = exp [diag (A1 , A2 , .. . , AN)]I N

=

ldiag (exp A1, exp A2 , .. . , exp AN) =

II exp Ai i=l

Rewriting the final product of exponentials of the eigenvalues as the exponential of the sum of t he eigenvalues, we get

N

exp Al =

II exp Ai = exp

= exp(Tr A)

i =l

Hence we have proved the trace formula exp A = exp(Tr A ) I j [email protected]

117

physicsguide CSIR NET, GATE

[email protected]

117

physicsguide CSIR NET, GATE

@Sk J ahiruddin , 2020

6.2

Linear Algebra

Function of matrix by using power series expansion and Cayley Hamilton the ore m

Let us st art with an example. You will get everything.

0 1 find, sin kA, cos kA, For the matrix A = 1 0 ei kA , where k is a scalar number .

Solution: Given that, A = 2

equation of the matrix is A

0 1 1 0



Characteristic

=1

0 1 1 0

0 1 1 0

1 0 0 1

1 0 0 1

0 1

0 1 =A 1 0

and so on ... So, A 2 ... = A

=

A4

1 0 =

ekA,

A6

= ... =

=I

J ; A3 = A5 = A7 =

Now expand the functions in power series.

j [email protected]

118

physicsguide CSIR NET, GATE

[email protected]

118

physicsguide CSIR NET, GATE

@Sk J ahiruddin , 2020

Linear Algebra

1. 3 5 kA) (kA ) ( sin kA =kA - - - - + - - + · · · 3! 5! k 3A k 5A =kA - + - + ... 3! 5!

=k

3

k 0 l 3! 1 0

O l 1 0

ks + 5!

0 1 1 0

0 k3

k5

k -JT+ w+ ·· · 0 sink sink 0 2. 2 4 (kA) kA) ( cos kA = I - --+--+ · .. 2! 4! k2I k4J = I - - + - + ··· 2! 4! k4 1 0 k2 1 0 2! 0 1 + 4! 0 1

1 0 0 1 0

k2

k4

l -21 + 41+ · ··

cos k 0 0 cos k jahir@physicsguide. in

119

physicsguide CSIR NET, GATE

[email protected]

119

physicsguide CSIR NET, GATE

@Sk J ahiruddin , 2020

Linear Algebra

3. ekA

- I

-

kA

(kA)2 (kA)3 (kA)4 (kA) 5 ... + + 2! + 3! + 4! + 5! + 2 (kA) (kA) 4 ! +--+--+ ··· 2! 4! (k A)3 (k A)5 + kA + - - + - - + ... 3! 5! k3 A k5A + kA + - 3! + - 5! + · · ·

1 0 0 1

+

k

k

2

+ 2! O l 1 0

+

1 0 0 1

k3 3!

4

+

1 0 0 1

k 4!

ks 5!

0 1

1 0

+

0 1 1 0

+ coshk 0 cosh k 0

j [email protected]

+

sinh k sinh k 0 0

120

coshk sinhk sinhk coshk

physicsguide CSIR NET, GATE

Linear Algebra

@Sk J ahirudd in, 2020

4.

i kA

4

3

5

(i kA) (ik A ) (ikA ) (i kA) - + + 2! + 3! + 4! + 5! + (ikA )2 ( i kA) 4 ! +--+--+ ·· · 2! 4! 3 5 .kA (ikA) (ik A ) + 'l + - - + - - + •· · 3! 5! ik 3 A ik 5 A + i kA - - - + - - + · · · 3! 5!

eikA _

I

2

k2 1 0

1 0 0 1

+

ik

2!

3!

1 0

+

0 1 3 ik

O 1

k4

...

1 0 0 1

4!

5

ik + 5!

0 1 1 0

0 1 1 0

0 k2

0

+

k4

l - 21+41+ ·· · . k k3 k5 i -3T+51+ · ··

0

cos k 0 0 cosk j ahir@physicsgui de. in

+ 121

i sin k i sin k 0 0

physicsguide CSIR NET, GAT E

[email protected]

physicsguide CSIR NET, GATE

121

@Sk J ahiruddin, 2020

Linear Algebra

cos k i sin k i sin k cos k You could have solved t he problem by similarity transformat ion. That could have been easier also. As we see t he

1 0 diagonalized matrix A' in this case would b e A' 0 - 1 and t he S matrix S

=

F2

1

1

1 - 1

Hence

f( A)

/ (1) 0 0 / (-1)

=~

1

1 2

1

1 2

/(1) + /( -1) / (1) - / (-1) /(1) - /(- 1) /(1) + /(- 1)

[email protected]

1

1 -1

1

1 - 1

1

1

1 -1

/(1) /(1) /(- 1) - /(- 1)

122

physicsguide CSIR NET, GATE

[email protected]

122

physicsguide CSIR NET, GATE

@Sk J ahiruddin , 2020

Linear Algebra

Now when J(A )

.

1

=-

s1nkA

2

= sin kA we get sin k - sin k sin k + sin k sin k + sin k sin k - sin k 0 sink sink 0

We see that we have got the same result quite easily. Now do the rest three sums cos kA , ekA, eikA . Verify your result from previous example. Personally I prefer the similarity transformation method while evaluating any function of a matrix

Example: You are given two Matrices A and B. Consider t he matrices eA, eB . expand some of the t erms in the infinite series of eA, eB and eA+B. Multiply the two series of eA , and eB . Now show that

Solution: We know, eA

eB j [email protected]

A2

A3

2!

3!

B2

B3

2!

3!

= I + A + - + - + ··· = I + B + - + - + ··· 123

physicsguide CSIR NET, GATE

[email protected]

124

physicsguide CSIR NET, GATE

@Sk J ahiruddin, 2020

Linear Algebra

t hen

Solution:

\A/e can prove by induction

Now suppose that J(A) is a function of A defined by a Taylor series in non negative powers of A , where the coefficients of the Taylor series are assumeed to commute with both A and B. It is easy to see that:

[J(A), B] = J'(A) [A, B] where J' denotes the formal derivative of an operator argument A . ex A

f

applied to

= L n -¾(xA)n is such a function. Therefore, n.

Now define a operator

j [email protected]

125

physicsguide CSIR NET, GATE

[email protected]

124

physicsguide CSIR NET, GATE

@Sk J ahiruddin, 2020

Linear Algebra

t hen

Solution:

\A/e can prove by induction

Now suppose that J(A) is a function of A defined by a Taylor series in non negative powers of A , where the coefficients of the Taylor series are assumeed to commute with both A and B. It is easy to see that:

[J(A), B] = J'(A) [A, B] where J' denotes the formal derivative of an operator argument A . ex A

f

applied to

= L n -¾(xA)n is such a function. Therefore, n.

Now define a operator

j [email protected]

125

physicsguide CSIR NET, GATE

j [email protected]

125

physicsguide CSIR NET, GATE

@Sk J ahiruddin , 2020

Linear Algebra

By construction, G (x) is invertible, with

c-1 (X) = e- xBe- xA We differentiate w.r.t. x

dG (x) dx

dexA --exB

dexB + exA__ dx dx = AexAexB + exAexBB

= (A + B )exAexB + [exA, B ] exB = (A+ B + x[A , B] )G(x) Integrating by separation of variable

logG(x)

+ k = xA + xB +

1 2 -x [A , B ] 2

Now define

So the equation can be written

e- F(x) (G'(x) - F'(x)G(x)) = 0 [email protected]

126

physicsguide CSIR NET, GATE

Linear Algebra

@Sk J ahiruddin , 2020

We get from the definit ion of F (x)

[F' (x), F (x)] = A + B + x[A, B ], xA + xB + ~x [A , BJ 2

1

= x [A + B , [A, B]] + x [[A, B], A + B]

2

2

2

=0 Since [A, [A, B]] = [B , [A, B]] = 0 For any operator satisfying [F' (x), F (x)] t o show t hat:

= 0, it 's easy

d e- F(x) = - F' (x )e-F(x) dx Our differential equation becomes

ddx [ e-F(x) G (x)]

G (x)

=

=0

eF(x) c o

where G 0 is an op erat or which does not depend on x . Checking t he special case x = 0, we find G = 1 and F = 0. j [email protected]

@Sk J ahiruddin, 2020

127

physicsguide CSIR NET, GATE

Linear Algebra

Linear Algebra

@Sk J ahiruddin , 2020

Therefore Go = 1. Eliminating F and G in favor of A and B , we find:

Setting X

=1

since [A , B ] commutes with A and B , this is equivalent to:

More generally it can be proved that when a, j3

,.. 1 a ,.. 1 a ,.. ) · m. 1ett1ng '±' = a 1 2 3 in v '±' = h1 8u1 e1 + h2 8u 2 e2 + h3 8u3 e3 and substit uting into the above equation, we find

j [email protected]

90

physicsguide CSIR NET 1 GATE

@Sk J ahiruddin, 2020

Vect ors

Repeating the analysis for V · (a2e2) and V · (a 3e3 ) adding t he results we get the final result.

2.8.3

Laplacian

In the expression of divergence use 1 8 ,-.

a = V = h

1 OU1

e1

1 8

+

1 a h2h3 a h1h2h3 8u1 h1 8u1 a h1h2 a I'.:\

7

-I'.:\-

1 8 ,-.

+ h 2 OU2 e2 + h 3 OU3 e3

You easily get the expression

v 2 =

A

,

and

11 oo : 04: 12 You easily get the expression

v 2 =

1 a h2h3 a h1h2h3 OU1 h1 OU1 fJ h1h2 fJ +OU3 h3 OU3 (2.73)

2.8.4

Curl

The curl of a vector field a = a 1e1+a2 e2 +a3e3 in orthogonal curvilinear coordinat es is given by

(2.74)

91

j [email protected]

physicsguide CSIR NET 1 GATE

@Sk J ahiruddin, 2020

Vectors

This can be proved as follows Let us consider t he subexpression V x (a 1e1) . since e1 = h 1V u 1 we have y7 X

(a1e1)

= y7 X (a1h1 V u1) = V (a1h1) x V u1 + a1h1 V x V u1

But V x V u 1 = 0, so we obtain "

'\7 X (a1e1)

= '\7 (a1h1) X ::

· ( nm. 1 8 e" 1 + -h 1 8 e2 " + -h 1 8 e3 "' ) a 1h1 1n v '±' = -h 1 8 U1 2 8 U2 3 8 U3 and substit uting into the above eauation, we find · Lett1ng

m.

'±'

=

11 oo : 04: 1 s A

y'

X (a1e1) =

y'

(a1h1) X ::

. '±' m. · ( nm. l 8 "' 1 8 "' 1 8 "' ) Lett1ng = a 1 h1 1n v '±' = -h e 1 + -h e 2 + -h e3 8 8 8 1 U1 2 U2 3 U3 and substituting into the above equation, we find

Similarly find V x (a2e 2) . you get t erms in e 3 and e1 , . Find y7 X ( a3e3) and you get terms in e1 and e 2. Just add and you get the result.

j [email protected]

92

physicsguide CSIR NET 1 GATE

@Sk J ahiruddin, 2020

2. 9

Vectors

Exersizes

2.1. For the twist ed space curve y 3 given parametrically by

+ 27axz -

8l a2y = 0,

show that t he following hold: (a) ds/du = 3,J2a (I + u 2 ), where s is t he distance along t he curve measured from the origin

11 oo : 04: 17 x = au

'

show that t he following hold: (a) ds / du= 3yl2a ( 1 + u 2 ) , where s is t he distance along t he curve measured from the origin (b) the length of the curve from t he origin to the Cartesian point (2a, 3a, 4a) is 4vl2a (c) the radius of curvature at the point with parameter u is 2 2 3a(l +u ) (d) the torsion T and curvature K, at a general point are equal (e) any of t he Frenet-Serret formulae that you have not already used directly are satisfied. 2.2. The shape of the curving slip road joining two motorways, that cross at right angles and are at vertical heights z = 0 and z = h, can be approximated by the space curve

vl2h r = --ln cos 7r

Z 1r

h 2



1

+

vl2h l 1r

physicsguide CSIR NET 1 GATE

93

j [email protected]

. Z1r • k n sin 2h J + z

@Sk J ahiruddin , 2020

Vectors

Show that the radius of curvature p of the slip road is (2h/rr) csc(zrr / h) at height z and t hat t he torsion T = - 1/ p. To shorten the algebra, set z = 2h0/ 1r and use 0 as the parameter. 2.3. Parameterising the hyperboloid x2

y2

z2

-+---= 1 a2 b2 c2

11 oo : 04: 20 rameter. 2.3. P aramet erising t he hyperboloid

x2

y2

z2

+ = l a2 b2 c2 by x = a cos 0 sec N Un

converges

However , if I: Vn diverges and Un > Vn for all n great er than some fixed number then I: Un diverges.

j [email protected]

16

physicsguide CSIR NET, GATE

11 oo : oo : 44 some fixed number t hen

j [email protected]

I: U n

diverges.

16

physicsguide CSIR NET , GATE

@Sk J ahiruddin , 2020

1.3.3

Infinite Series

D' Alembert's ratio test

The ratio t est det ermines whether a series converges by comparing the relative magnitude of successive t erms. If we consider a series

I: U n p

and set

= lim

(1.17)

n➔ oo

t hen if p < 1 t he series is convergent; if p > 1 the series is divergent; if p = 1 then the behaviour of t he series is undetermined by this t est.

1.3.4

Ratio comparison test

Consider t he two series L U n and I: Vn and assume t hat we know the latter to be convergent. It can b e proved that if

for all n greater than some fixed value N then convergent . Similarly, if

I: U n

is also

11 oo:oo:47 for all n great er than some fixed value N then convergent. Similarly, if

[email protected]

physicsguide CSIR NET, GATE

17

Infinite Series

@Sk J ahiruddin, 2020

for all sufficiently lar·ge n , and diverges.

1.3.5

I: Un is also

I: Vn diverges t hen I: U n

also

Quotient test

Consider the two series limit p

I: Un and I: Vn, = lim

n ➔ oo

and define p as the

Vn

Then , it can be proved that: (i) if p =I= 0 but is finite then I: Un and L Vn eit her both converge or both diverge; (ii) if p = 0 and I: vn converges t hen I: U n converges (iii) if p = oo and I: v11, diverges then I: Un diverges.

1.3.6

Integral Test

We will replace t he n in the general n th term of t he series by x and write it as a function f (x) . Then it can be shown t hat , if the limit of the integral

11 oo : oo : 49 We will replace t he n in the general n th term of t he series by x and write it as a function f (x) . Then it can be shown t hat , if the limit of the integral N

f (x) dx

lim

N ➔ oo

[email protected]

physicsguide CSIR NET, GATE

18

@Sk J ahiruddin , 2020

Infinite Series

exists, t he series I: Un is convergent. Otherwise the series diverges. Not e that the integral defined here has no lower limit.

Example: Determine whet her t he following series converges: 00

1

L (n - 3/ 2) n =l

2

4

4

4

4

= + + 9 + 25 + · · ·

Solution: Replacing n by x we get 1

f (x) = (x - 3/ 2)2 Applying the integral t est , we consider N

lim

N➔oo

1

(x - 3/2)2dx =

J~oo

- 1 N-3 / 2

=0

since t he limit exist s the series converges. Note, however , t hat if we had included a lower limit , equal to unity, in t he integral t hen we would have run into problems, since t he integrand diverges at x = 3/ 2

11 oo : oo : s2 since the limit exists the series converges. Note, however , that if we had included a lower limit, equal to unity, in the integral t hen we would have run into problems, since t he integrand diverges at x = 3/ 2

j [email protected]

19

physicsguide CSIR NET 1 GATE

Infinite Series

@Sk J ahiruddin , 2020

1.3. 7

Cauchy's root test

Cauchy's root test may be useful in testing for convergence, especially if the n th terms of t he series contains an n th power . If we define t he limit p

= lim (Un)l / n n➔oo

t hen it may be proved that t he series I: Un converges if p < 1. If p > 1 t hen t he series diverges. Its behaviour is undetermined if p = 1.

1.4

Alternating Series

An alt ernating series can be written as 00

L (-1 n=l

) n+lun

= U1

-

U2

+ U3 -

U4

+ U5 -

···

11 oo : oo : s4 An alt ernating series can be written as 00

L (-l) n+lun n=l

=

U1 -

U2

+ U3 -

U4

+ U5 -

···

with all U n > 0. Such a series can be shown to converge provided

(i) U n

➔ 0

as n ➔ oo and

(ii) Un < Un- l for all n > N for some finit e N . If t hese conditions are not met t hen the series oscillates. [email protected]

20

physicsguide CSIR NET, GATE

@Sk J ahiruddin , 2020

2

Infinite Series

Examples on Series Convergence Find whether of the following series converges or diverges?

CX)

Solution: Let us take,

L

Un

where, Un

=

n! l OOn.

n=l

Now, we will check t he ratio test. p

= lim

n➔ oo

(n + 1)! 100n n+ l - - x - - = lim = lim -1oon+l = n! 100 n ➔ oo

n➔ oo

CX)

1

L 2n-n2

n=5

00

11 oo :oo : s1 n➔ oo

Un

n,➔ oo

oon+1

1

00

L 2n

n ➔ oo

1 -n2

n=5

Solution: We will use a Special Comparison Test here, it says, 00

If

L bn is a convergent series of positive terms and an~ 0 n=l 00

and

abnntends to a (finite) limit, then L anconverges. n=l

j [email protected]

21

physicsguide CSIR NET I GATE

@Sk J ahiruddin , 2020

Infinite Series

1 Now, here we take, bn = n which is a convergent series. 2 . 1 1 l1m -;n➔ oo 2n 2n - n 2

Now, oo ~

So, L..t

. l1m

n2

= n➔ oc 1 -2'-n

= 1

I

n.

loon converges.

n =l

00

n(n + 1)

Solution: Use t hese propert ies 00

(a) If

Lb

1i

n=l

is a convergent series of positive terms and

11 oo : 01 : oo Solution: Use t hese prop ert ies 00

(a) If

L bn is a convergent series of positive terms and n =l 00

an > 0 and an/bn tends to a (finit e) limit , then L

an con-

n =l

verges. 00

L dn is a divergent series of positive terms and

(b) If

n =l

an > 0 and an,/ dn tends to a limit greater t han O (or tends 00

to + oo), t hen L

an diverges.

n =l

[email protected]

22

physicsguide CSIR NET) GATE

@Sk J ahiruddin ) 2020

Infinite Series

Here t he n t h term of t he series series go to

n 00

Now L 1 as n



oo·

'

1 n+7

1

1

n

n

- is a divergent series wit h dn = - and an/ dn ➔

n= l ➔ oo .

Hence t he series diverges. oo (-2) n

11 oo : 01 00

Now

: 02

1

1

L -n is a divergent series with dn = -n and an/ dn -+ n=l

1 as n-+ oo . Hence t he series diverges.

Solution: This is an alternating series and an alternating series converges if the absolute value of t he terms decrease steadily to zero, i.e. , if

lan+l ~ an and lim an = 0 n --t

Now, here limn-too an = limn-too

(-2)n

=

n2

oo [Check using

L'Hopital Rule]. So, t he series will diverge. [email protected]

23

physicsguide CSIR NET, GATE

@Sk J ahiruddin, 2020

Infinite Series

Find t he interval of convergence of the series

.

Solution: We can writ e 4

3

l -x/2+x /4-x / 8+ ... +(-x)n/2n+ ....

I n,

.I

00

I rr

.

=

I

CX)

00

n =O

n=O

L anXn = L Un

11 oo : 01 : os 4

3

l-x/2+x / 4-x /8+ ... +(-x)n /2n+ ... . =

L anXn = L Un n =O

n= O

CX)

=

p

. 1Im

Un+l Un

X

=L

2

n=O

Now, if the series converges in any region, then p < l. So, here,

3

X

2

0 is unity. Hence, 32

j [email protected]

physicsguide CSIR NET I GATE

Diff eq and Sp funs

@Sk J ahiruddin , 2020

y(t) =

t et' /a

e - t/a

- -dt' o a

= e - t/a et/a - 1 = 1 - e - t/a

(b) With f (t) = b(t), t he integration will be trivial:

y(t ) =

(c) For

e - t/a

t 6 (t') et' /a

- - - -dt' a

f (t) = 13- 1e - tff3 H (t), y(t) =

1

= e - t/a X -

a

= e-t/a

= --

a

with f3 < a , we have

t et' I a e - t' I f3

e - t/a

e - t/a

----dt' o a/3 e (a-1 - ,e-1)t'

t

a/3 (a -1 - 13-1) 0

e - t/(3

e - t/a

f3 -a

f3 - a

11 oo : 01

: 31

e- t/ fJ

e- t/ a

(3 -a

(3 - a

e-t/a _ e- t/fJ

a-

/3

As f3 ➔ 0, f (t) becomes very strongly peaked near t = 0, but with the area under the peak remaining constant at unity. In the limit, t he input f (t) becomes a 6 -fuction, the [email protected]

physicsguide CSIR NET ) GATE

33

Diff eq and Sp funs

@Sk J ahiruddin ) 2020

same as that in case (b) . It can also be seen that in the same limit t he solution y(t) for case (c) tends to that for case (b), as is to be expected. Solution: 2.4.. We see that t he equation is homoge-

neous in x and y and so we set y and obtain

=

vx, wit h

g; =

v + xg~,

av 2 + 3v v+x-=---

8x v- l 8v - 2 - 3v - v 2 + V x-=------v- l 8x dx (l - v)dv X v 2 + 2v + 2 2 (v+ 1)2 + 1

2

v + 2v + 2 v- l

v

+l

(v+ 1) 2 + 1 1 1 2 => lnAx = 2tan- (v + 1) - ln [1 + (v + 1) ] 2 2 2 1 ln { B x [1 + (v + 1) ]} = 4tan- (v + 1)

11 oo : 01

: 33

2 (v+ 1) 2 + 1

⇒ lnAx 2

ln { B x [1 + (v + 1)

2

]}

= =

V

+1

(v+ 1) 2 + 1 1 1 2tan- (v + 1) ln [1 + (v + 1) 2 ] 2 4tan - 1 (v + 1)

On setting v = y / x t his b ecom es

B [x 2 + (y + x) 2] = exp 4tan- 1

y+ x X

Solution: 2.5 .. As x and y only appear in the [email protected]

physicsguide CSIR NET, GATE

34

Diff eq and Sp funs

@Sk J ahiruddin , 2020

n ation x + y we set v = x + y wit h dv / dx = l + dy / dx . The equation and its solution t hen b ecome

dv = 1 _ v dx 3v - 4 3v - 4 dx = 2v - 4 dv = ⇒

ln ( x

x+ k

+y -

2)

= =

3 2 2 + 2v - 4

3 v + ln(v - 2) 2 1 k - ( x + 3y) 2

=

dv

3 (x + y) + ln(x + y - 2) 2

Solution: 2.6 .. (a) Writing p = dy/dx, the equation b ecomes y

=

xp2

dy 2 p = - = p dx

+p

+ 2x pp' + p'

11 oo : 01

: 36

Solution: 2.6 .. (a) Writing p = dy/dx, the equation becomes y = xp2 + p dy 2 p = - = p + 2xpp' + p' dx 2 dp = p +(2xp+l)dx (b) In different ial form , this equation becomes

df

=

p( l - p)dx - (2xp + l )dp = 0

(c) We now apply t he standard test for the existence of an IF for f (x, p)dx + g(x, p)dp: 35

j [email protected]

physicsguide CSIR NET, GATE

Diff eq and Sp funs

@Sk J ahiruddin, 2020

l

8g

f

ax

8f 8p

1

1

p(l -p)[-2p- (1-2p)] = -p(l -p)

As t his is a function of p alone, an IF exists and is given by

exp

- 1 p-(l---p-) dp

= exp =

1

1

p

1 -p

-----

dp

exp [- ln p + In (1 - p)] =

With this IF, the equation becon1es

1

- p p

11 oo : 01

: 38 p

=

- p

exp [- lnp + ln(l - p)] = _l _-_ P p

Wit h t his IF, t he equation becomes

(1 - p)2dx - (2xp + 1)(1 - p) dp = 0 p

dp d[( l-p) x ]--+dp = O 2

p

( 1 - p) x - ln p + p 2

=c

This gives x = (c + lnp - p)( l - p)- 2 and, together with y = p + p 2x, gives a full paramet erisation, x = x(p) , y = y(p), of the solut ion. [email protected]

36

physicsguide CSIR NET , GATE

@Sk J a hiruddin, 2020

Diff eq and Sp funs

Solution: 2. 7 .. Writ ing dy / dx have

dv -dx = 2YP,

du_ dx - 2x,

=

p and dv / du

dv yp q--- du - x'

x

X 2 xy 2q -(u+v- 1)-q +xy= O y y

We now multiply by 2

uq

-

Y X

and substitute again:

(u + v - l )q + v = 0

v (l - q) - uq + q + u q2 = 0 q

X

p= -q

Making the substitution gives 2

=

y

q, we

11

00:01 :41

We now multiply by

uq

2

-

Y

X

and substitute again:

(u + v - l )q + v = 0 2

v (l - q) - u q + q + u q

v

=0

= uq +

q

q- 1

, Clairaut 's form

As t he equation now has Clairaut 's form it has two solutions. (i) The first is

C

v =cu+

or

c- 1 In all cases a > 13 > 0

y

2

cx

-

2

C

= --

1

c-

2

2 2

2

• For c > 1, t his is a hyper bola of t he form y - a x = 13 . • For 1 > c > 0, it is a hyp erbola of the form x 2 - a 2 y 2 = 132 • For c < 0, t he conic is an ellipse of the form y 2 + a 2x 2 = [email protected]

37

physicsguide CSIR NET, GATE

Diff eq and Sp funs

@Sk J ahiruddin , 2020

(ii) The second (singular) solution is given by

d dq

q

q- l - 1 (q - l )2

or or

+u= O

+u =

0

1

q

= l ± VU

Subst ituting t his into thje equation converted in Clariot form, expressed in t erms of x and y t hen gives

/32

.

11 oo : 01 OT

q

: 44 1

= l ±-

vu

Substituting t his into thje equation converted in Clariot form , expressed in t erms of x and y then gives y2

=

x2



1

-

X

2

=x ±x±x+l =(x±l) y

2

= ±(x ± l )

These lines are the four sides of the square t hat has corners at (0, ± 1) and (±1, 0) Solution: 2.8 ..

(a) With dy / dx appearing in the first term and y in the second (and nowhere else), t his is a linear first-order ODE and therefore has an IF given by j [email protected]

38

physicsguide CSIR NET 1 GATE

@Sk J ahiruddin, 2020

µ(x) = exp

Diff eq and Sp funs

X

a2

+ x2

= (a2 + x2) 1; 2 When multiplied t hrough by t his, t he equation becomes d dx

(a2

+ x2) 1/2 y =

x ( a2

+ x2) 1/2

11 oo : 01

: 46

When multiplied t hrough by this, t he equation becomes d (a2 + x 2) 1/2 y dx ⇒

= x (a2 + x2) 1/2

~ ~ (a2 + x2) 3/2 + A

(a2 + x2 ) 1/ 2 y =

a +x A y - - -+---3 (a2+x2)1; 2 2



2

(b) Again, an IF is needed ; this t ime given by

µ(x) = exp

- tan x dx

=

exp(ln cos x) = cos x

The equation now becomes

:x (y cos x) = cos x => y cos x = sin x + A The given boundary condition is that 3 / v'2 = 1/ v'2 + A , establishing A as v'2 . The final answer is therefore y = tan x + y'2secx [email protected]

39

@Sk J ahiruddin, 2020

physicsguide CSIR NET, GATE

Diff eq and Sp funs

Solution: 2.9 .. At first sight this non-linear equation may appear to be homogeneous, but t he t erm A/ x rules this

out. since it is non-linear, we set dy / dx = p and rearrange t he equation to make y, which then appears only once, the subject: 2 y A p - - p+ -=0 X

X

11 oo : 01

: 49

out. since it is non-linear, we set dy / dx = p and rearrange t he equation to make y, which then appears only once, t he subject: 2 y A p - - p+ -=0 X

X

A

xp-y + -

p

y

=0 A

= xp+ -

p

This is now recognised as Clairaut's equation with F (p) = A / p. Its general solut ion is therefore given by y = ex + for arbitrary c

4

It also has a singular solut ion (containing no arbitrary constants) given by

d dp

A P

+X

=

0, :::;> p =

A X

Hence

y=x

j [email protected]

@Sk J ahiruddin , 2020

A x

A + -------;:::= = 2ffx A /x

40

physicsguide CSIR NET I GATE

Diff eq and Sp funs

The final result was obtained by substit ut ing for p in y = xp + A /p Solution: 2.10 .. The equation is not homogeneous and the two variables x and y appear in different linear corn-

11

oo : 01 : s1

The final result was obtained by substituting for p in y = xp + A /p

Solution: 2.10 .. The equation is not homogeneous and the two variables x and y appear in different linear com-

binations on the two sides of the equation. We t herefore seek shifts in their origins that will make the expression for t he derivative homogeneous, i.e. remove the constant terms from both its numerator and denominator. To do this we set x = X + a and y = Y + /3 We then require

3a + 3/3 + 3 = 0 and

5a + /3 - 7 = 0

These have the straightforward solut ion a= 2 and /3 = -3; with these values the original equation reduces to This is now homogeneous and to solve it we set Y = vX and obtain

[email protected]

41

physicsguide CSIR NET ) GATE

Diff eq and Sp funs

@Sk J ahiruddin ) 2020

dY _ dX 7

-v+

X dv dX

11 oo : 01 : s4 @Sk J ahiruddin, 2020

Diff eq and Sp funs

dY _ X dv dX - V + dX X dv _ dY _ _ 3X + 3Y _ dX - dX v - 5X + Y v 3 + 3v - 5v - v 2 5+v

We now separate the variables and use method (iii) for a partial fraction expansion, obtaining dX

5+v A B = = + 2 x 3 - 2v - v 3 + v 1 - v 1 3

=+ 2(3+v) 2(1- v)

ln X =

1

2

ln (3 + v) -

3

2

ln (1 - v)

+k

Re-substituting for v, X and Y, gives

+3

1/ 2

x-2 = A 3+-Yx- 2

l -y+3 x-2

-3/2

1 2

= A-(3_x_+_y_-_3)_1_ (_x_-_2_) (x - y - 5)3/2 Finally, t his result can be re-written as [email protected]

@Sk J ahiruddin, 2020

42

physicsguide CSIR NET, GATE

Diff eq and Sp funs

11 oo :01 : s1

@Sk J ahiruddin, 2020

Diff eq and Sp funs

3

(x - y - 5) = B (3x

+ y - 3)

Solution: 2.11 .. After being divided through by x, this equation is in t he form of a Bernoulli equation with n = 2, i.e. it is of the form

dy dx

+ P (x)y =

Here, P (x) = x- 1 and Q(x) y 1 - 2 = y - 1 and obtain

dy dx

d dx

I v

Q(x)yn

=

x - 512 . So we set v

I dv v 2 dx

The equation becomes I dv I ----+-= v2 dx vx dv v --dx x d V dx x

j [email protected]

(c)Sk J a,hir11ddin. 2020

1

-2 2 v x5/

I for which t he IF is 1/ x x 512 ' I x7/ 2

43

physicsguide CSIR NET, GATE

Diff ea and Sn fi1ns

11 oo : 01 : sg 43

j [email protected]

physicsguide CSIR NET I GATE

Diff eq and Sp funs

@Sk J ahiruddin , 2020

2 1 3 - = + , using y ( 1) X 5 x 512 5 1 2 1 3x - = ---+y 5 x 312 5 5x3/2 y = 2 + 3x 5/ 2 V

= 1

Solution: 2.12 .. Since x appears only in the combinat ion xdy / dx it will probably make t he solution simpler to take y as t he independent variable and x as the dependent one. With t his in mind, we re-arrange t he equation as

dx tan y dy

+x =

. 2 sin y

In stanadard form

dx dy

+ x cot y = 2 cosy

The IF is clearly exp(ln sin y) can be written

= sin y, and

the equation

d (x sin y) = sin 2y dy . 1 x sin y = - cos 2y + k 2 [email protected]

44

physicsguide CSIR NET, GATE

11 oo : 02 : 02 -(x sin y) = sin 2y dy

1 cos 2y + k 2

x sin y = 44

[email protected]

physicsguide CSIR NET ) GATE

@Sk J ahiruddin ) 2020

If y(O)

= 1r / 2

X=

Diff eq and Sp funs

t hen k

= -

~ and the solution is

- 2 cos 2 y . 2 Sln y

- 1 - cos2y ----2siny

= - cos y cot y

Solution: 2.13 .. As the equation contains only derivat ives, we write dy/dx = p and d2 y/dx 2 = dp/dx; t his will

reduce the equation to one of first order: dp dx

+ P2 + P = 0

Separating the variables we get dp p(p + 1)

= -dx

We now integrate and express the integrand in partial fract ions: 1

1

- - - - dp= p p+l ln(p) - ln(p + 1)

=A-

dx x

= B e-x

p

p+ l

e- x p = C -e Now p

= dy/dx and so

x

11 oo : 02 : 04 --=

p+ l

B e-x

e- x p = C -e - x

= dy/dx and so

Now p

[email protected]

45

physicsguide CSIR NET, GAT E

Diff eq and Sp funs

@Sk J a hiruddin , 2020

-x

dy dx

e

C - e-x y = ln ( C - e-x )

+D

= ln ( C - e-x ) - ln(C - 1), since we require y(O) = 0, =

C -e- x ln - - C- 1

This is as far as y can be det ermined since only one boundary condition is given for a second-order equation. As C is varied the solution generates a family of curves satisfying the original equation. A variety of other forms of solut ion are possible and equally valid, t he actual form obtained depending on where in t he calculation the boundary condit ion is incorporated. They include

11 oo : 02 : 01

j [email protected]

46

physicsguide CSIR NET, GATE

@Sk J ahiruddin , 2020

3

Diff eq and Sp funs

0 DE of second and higher order

If f (x) = 0 t hen the equation is called homogeneous; otherwise it is inhomogeneous. In order to solve any equation of t his form we must first find the general solution of the complementary equation, i.e. t he equation formed by setting f (x) = 0.

dny an (x) dxn

dn-ly + an- 1 ( x) d xn- 1

dy + · · · + a1 ( x) dX

+ ao (x)y = 0

If t he n solutions of the equation are Y1 (x), Y2 (x) , .. . , Yn (x), t hen t he general solut ion is given by the linear superposition

11 oo : 02 : 1o If t hen solutions of the equation are Y1 ( x), Y2 (x), ... , Yn (x), t hen t he general solution is given by the linear superposition

If the equation has f (x) =/- 0 (i.e. it is inhomogeneous) then Yc(x) is only one part of the solution. The general solution is t hen given by y(x) = Yc(x) + Yp(x) [email protected]

@Sk J ahiruddin, 2020

47

physicsguide CSIR NET, GATE

Diff eq and Sp funs

where yp(x) is t he particular integral, which can be any function that satisfies the equation (3. 1), provided it is linearly independent of Yc(x) .

3.1

Linear equations with constant coefficients

If the am in equation ( 3 .1 ) are constants rather than funct ions of x t hen we have

This is called linear equation with constant coefficient and are very common in Physics.

Finding CF: We do find the CF by putting y = Ae>-x. The auxiliary equation then

11 oo : 02 : 12 Inear equa I n are ve1~y common in Physics.

Wl v......

Finding CF: We do find the CF by putting y The auxiliary equation t hen

= A e.\x.

This equation have n roots. We t hen have t hree main possibilities.

[email protected]

48

physicsguide CSIR NET, GATE

Diff eq and Sp funs

@Sk J ahiruddin , 2020

(i) All roots real and distinct. The CF is then .

(ii) Some roots complex. If one of the roots of t he auxiliary equation is complex , say a + i/3, then its complex conjugate a - i/3 is also a root. The solut ion for that part icular two roots

This can also be written as in two other very useful form y

= eax (c1 sin f3x + C2 cos f3x) ;

y

= ceax sin(,Bx + 1')

(iii) Some roots repeated If, for example, ..\ 1 occurs k t imes (k > l ) as a root of t he auxiliary equation, t hen we

11 oo : 02 : 1 s y

= ceax sin(,Bx + --y)

(iii) Some roots repeated If, for example, A1 occurs k t imes (k > l ) as a root of the auxiliary equation, then we need to find k - l solutions that are linearly indep endent of those already found and also of each other. Complementary function is then

Example:

Solve

I y'' - 6y' + 9y = 0 I

49

j [email protected]

physicsguide CSIR NET 1 GATE

@Sk J ahiruddin, 2020

Diff eq and Sp funs

Solution: We write the equation as

(D

2

-

6D

+ 9) y =

0

or

(D - 3)(D - 3)y

=

0

Since t he roots of the auxiliary equation are equal, we write t he result y

= (Ax+ B) e3x

Finding PI: There are many methods to find t he PI. (i) Substutution of trial functions. (ii) D operator method. (iii) Variation of parameter .

11 oo : 02 : 11 (i) Substutut ion of trial functions. (ii) D operator method. (iii) Variation of parameter. (iv) Inverse laplace and fourier transformation. (v) Green 's function method. Among these t he first two methods are useful for some particular functions in the RHS, not for all. The last three methods are more powerful and can be applied for many different types of functions in t he RHS of the different ial equation. [email protected]

50

physicsguide CSIR NET, GATE

@Sk J ahiruddin, 2020

Diff eq and Sp funs

Firstly, we discuss the trial solution method. We are attempting to solve equations like t his

(1) If f (x) = aerx then try

IYp(x) = berx I (2) If f (x) = a 1 sin rx ) t hen t ry

+ a 2 cos rx (a 1 or a2

may be zero

11 oo : 02 : 20 IYp(x) = berx I (2) If f (x)

= a 1 sin rx + a 2 cos rx (a 1 or a 2 may be zero

) t hen t ry

IYp(x) = b1 sin rx + b2 cosrx I (3) If f (x) = ao + a1x + · · · + aNxN (some a,,.,i may be zero) t hen try

(4) If f (x) is t he sum or product of any of t he above t hen try Yp ( x) as the sum or product of the corresponding individual trial functions. Now we go into more details with examples j [email protected]

51

physicsguide CSIR NET I GATE

@Sk J a hiruddin , 2020

Diff eq and Sp funs

ICase - 11We are trying to solve eq like (D - a)(D - b)y

=

F (x)

=

kecx

Take PI

cecx C xecx C x2ecx

if c is not equal t o eit her a or b if c equals a or b, a i- b

(3.3)

if c =a= b

ICase - 21To find a particular solution of (D -

a) (D -

11 oo : 02 : 22 C x ecx C x 2 ecx

if c equals a or b, a =I= b if c = a = b

(3.3)

ICase - 21To find a particular solution of ( D b)y =

a) (D -

k sin ax k cos a x

First solve ( D - a) (D - b)y

= k eio:x, then t ake real or

imaginary part .

- 3 I A particular solution Yp of (D - a)(D - b)y = ecx Pn(x) where Pn(x) is a polynomial of degree n is I Case

ecxQn(x) xecxQn(x) x 2ecxQn(x)

if c is not equal to eit her a or b if c equals a or b, a =I= b if c = a = b

(3.4) [email protected]

52

physicsguide CSIR NET ) GATE

Diff eq and Sp funs

@Sk J ahiruddin ) 2020

where Qn(x) is a polynomial of the same degree as Pn(x) with undet ermined coefficients to be found to sat isfy t he given different ial equation.

Example: Solve

ly'' + y' - 2y = l 8x ex I

11

oo : 02 : 2s

Example: Solve

ly''

+ y' -

2y

= l 8xex I

Solution: We write the eq as (D - l )(D + 2)y = l 8xex We have a= l , b = - 2, c = 1; also Pn(x ) = l 8x = P1 (x) is a polynomial of degree 1. We also see that c = a -=I= b. So follow equation (3.4) to see that t he form of t he solution is

We substitute this into the cliff eq to find A and B

+ B) 2 y; = ex (Ax +Bx+ 4Ax + 2B + 2A) 2yp = ex(6Ax + 3B + 2A) = l 8xex 2

y~ = ex (Ax +Bx+ 2Ax

y; + y~ -

So we must have

6A

=

18, 3B + 2A

[email protected]

=

0,

or 53

A = 3 ' B = -2 ' physicsguide CSIR NET, GATE

Diff eq and Sp funs

@Sk J ahiruddin, 2020

Hence the solut ion

Example Solve I y''

+ y' -

2y = x

2

-

xI

Solution: We take v,., = Ax + B x + C. substituite the 2

11 oo : 02 : 2a

Example Solve I y''

+ y' -

2y

=x

2

-

xI

Solution: We take Yp = Ax + B x + C, substituite the t rial function into the cliff equation and get the particular solution Yp = _ x2 + 1) 2 2

!(

Example Solve d2

dx~

+ 4y = x

2

sin 2x

Solution: CF is easy m 2 + 4 = 0

Ye ( X )

= C1e2ix

+ C2e -2ix

= d1 cos 2X

· 2X + d2 sin

we first guess at a suitable trial function for t his case should be

( ax

2

2

+ bx + c) sin 2x + (dx + ex + f) cos 2x

j [email protected]

@Sk J ahiruddin, 2020

54

physicsguide CSIR NET, GATE

Diff eq and Sp funs

However, we see that t his trial function contains t erms in sin 2x and cos 2x , both of which already appear in t he complementary function .

11 oo : 02 : 30 However, we see t h at t his trial function contains t erms in sin 2x and cos 2x , b oth of which already appear in the complementary function. We must t hen multiply the trial function by the smallest integer p ower of x which ensures t h at n one of the resu lting terms appears in CF. So t he new t rial function is

3

2

2

3

(ax + bx + ex) sin 2x + (dx + ex + f x) cos 2x Put t he t rial function into t h e diff equation and get

Yp(x) =

x3 - - cos 2x 12

x2 + - sin2x 16

x + - cos 2x 32

The general solut ion is t hen

2

3

x x x sin 2x + cos 2x = d 1 cos 2x + d2 sin 2x cos 2x + 16 32 12

Example:

Solve

y'' + y' - 2y = (D - l )(D + 2)y =(ex)+ (4 sin2x) [email protected]

@Sk J ahiruddin, 2020

55

+ (x

2

-

x)

physicsguide CSIR NET, GATE

Diff eq and Sp funs

We see the solut ions separately. Then add.

11 oo : 02 : 33 @Sk J ahiruddin , 2020

Diff eq and Sp funs

We see the solutions separately. Then add.

(D - l )(D

+ 2)y = ex

(D - l )(D has PI Yp2

=-

1

5

has PI

Yp1

l X = 3x e

+ 2)y = 4sin 2x

cos 2x -

3 .

5

sin 2x

1 2 (D - l )(D + 2)y = x - x has PI yp3 = - (x + 1) 2 Adding these three solutions, we see that 2

Yp

= Ypl + Yp2 + Yp3

=

l x eX - l cos 2x - 3. 1( 2 ) sin 2x x +l 2 5 3 5

Example: Solve

(D

2

+ 2D + l 7)y = 60e- 4 x sin 5x

Solution: The Complementary function is easy m 2 + 2m + 17 = 0. Hence C.F = e- x(A sin 4x + B cos 4x) To get the part icular integral solve ( D 2 + 2D + l 7)y = 60e(- 4x+ i 5x) first and t hen t ake only the imaginary part of [email protected]

56

Diff eq and Sp funs

@Sk J ahiruddin , 2020

1' T

physicsguide CSIR NET, GATE

1

11 oo : 02 : 3s [email protected]

physicsguide CSIR NET , GATE

56

Diff eq and Sp funs

@Sk J a hiruddin , 2020

t he solution. Now as know

J( D)y =

eax

Hence the P.I is the imaginary part of e-4x+i5x

(x(-4 + 5i))2 + 2(x(-4 + 5i)) + 17 Do some algebra to arrive at t he answer .

3.2 3.2.1

Variable coefficients and other forms Euler and Legendre form

T his type of equation called Legendre form. ( a, (3 and t he an are constants ) T his type of equation can be solved by making t he substit ut ion a x + (3 = et . We then have dy dx j [email protected]

a

dt dy dx dt

dy ax + (3 dt 57

physicsguide CSIR NET , GATE

1

f"1

r

11 oo : 02 : 38 dx

ax + f3 dt

dx dt

[email protected]

57

physicsguide CSIR NET, GATE

Diff eq and Sp funs

@Sk J ahiruddin , 2020

and

d dy dxdx

a2

(a x

+ /3)2

d2y dt 2

dy dt

A special case of Legendre's linear equation, for which a = 1 and /3 = 0, is Euler 's equation ,

ndny anx dxn

dy + · · · + a 1 x dX

+ aoy = f (x)

(3. 6)

it may be solved in a similar manner to the above by substituting x = et . And if f (x) = 0 you can substit ute y = x>- which will give you same solution . We will give you an example now

Example Solve

2d2y x dx2

dy + x dx - 4y = 0

Solution: First we make t he subst itution x = et , which, after cancelling et, gives an equation with constant coefficients,

d dt

d -- 1 y dt

[email protected]

+

dy dt - 4y = 0 58

d2y - 4 = 0 dt 2 y physicsguide CSIR NET, GATE

11 d dt

d -- 1 y dt

+

00:02:41

dy dt - 4y = 0

[email protected]

58



d2y - 4 = 0 dt 2 y

physicsguide CSIR NET , GATE

@Sk J a hirudd in, 2020

Diff eq and Sp funs

The general solution is t hen ,

Since gives

f (x) =

0 here, you can substitute y

= x>- which



This has the solutions ,\ eral solution

3.2.2

= ± 2, so we get t he same gen-

Variable coefficient of order 2

Consider the equation

d2 y dx 2

dy + a1(x) dx

+ ao(x)y = f (x)

We focus special case when the coefficients a0 ( x) and

a1 (x) are upto order 2, means they are algebric functions and can contain terms uoto x 2 not of hiQ:her orders.

11 oo : 02 : 43 dx-9

+ a1 x dX + ao x y = f x

We focus special case when the coefficients a0 (x) and a 1 ( x) are upto order 2, means they are algebric functions and can contain terms upto x 2 not of higher orders. [email protected]

physicsguide CSIR NET, GATE

59

Diff eq and Sp funs

@Sk J ahiruddin , 2020

TO solve the equation,

substit uite

y

= u(x)v(x);

1 -2

u(x) = exp

Solve for unkonwn v, t hen put the value of v in the value of y. Example: Solve

Solution: Divide t he equation by 4x 2 . We get 2

d y dx-?

l dy

l

2

+ -X dX + 4X 2 (x -

We get a 1 (x) = l /x and a0 (x) f (x) = 0. Now substit uite y

= vu = vexp

1) y = 0

= (x

2

-

Av

vx

1) / 4x

2

and

11 oo : 02 : 46 We get a1 (x) = 1/x and a0 (x) f (x) = 0. Now substituite

(x

2

-

1) / 4x

2

and

Av

y = vu = vex p

yx

We get

j [email protected]

physicsguide CSIR NET 1 GATE

60

@Sk J a hiruddin , 2020

Diff eq and Sp funs

T his equation is easily solvable.

=

V

1

C1

sin 2X

1

+ C2 cos 2X

So t he solut ion is y = -

V

yx

3.2.3

=

+

· 1 1 C1 Sln X C2 COS x 2 - - - - 2---

yx

Dependent variable y missing.

P ut

y1

= P,

II

I

y =p

Then the equation reduces to first order. Example:

Solve

d2 y dx2

dy + 2 dx = 4x

Solution: This is transformed by t he substitution p =

00: 02: 48 d2 y dx2

Solve

dy + 2 dx = 4x

Solution: This is transformed by the substitution p = dy / dx to t he first-order equation

dp dx

+ 2p = 4x 61

[email protected]

physicsguide CSIR NET , GATE

@Sk J a hiruddin, 2020

Diff eq and Sp funs

This is now linear equation. Solution is

p

=

dy dx

=

ae- 2x

+ 2x -

l

where a is a constant . Thus by direct integration t he solution to the original equation

3.2.4

Independent variable x missing.

Put

y

I

= P,

dp dp dy dp y = dx = dy dx = P dy II

Example:

Solve

dy dx

2

=0

11 oo : 02 : s1 Example:

dy Solve

dx

2

=0

Solution: Put dy/dx = p and d2 y/dx 2 = p(dp/dy) and hence get t he first order equation

dp 1 + YP dy

+ P2 = 0

62

j [email protected]

physicsguide CSIR NET I GATE

Diff eq and Sp funs

@Sk J ahiruddin , 2020

This is separable. The technique to solve has been discussed before. Solution is

Using p = dy /dx we get

dy p =-=± dx

cy -y2

Simplifying we get

3.2.5

Variation of parameters

y2

11 oo : 02 : s3

3.2.5

Variation of parameters

Variation of paramet er is a very powerful technique in finding particular integrals for linear OD Es with variable (and constant) coefficients. We need to know all of the complementary functions to use variation of parameters to find the PI. General technique. We need to find the solution of, dny an(x) d

+ xn

[email protected]

dy · · · + a1 ( x) d X

63

@Sk J ahiruddin ) 2020

+ ao (x) y = f (x) physicsguide CSIR NET ) GATE

Diff eq and Sp funs

The CF is

We transfer the constants to unknown functions and thus assume PI of this particular form

We choose k~ (x)y1(x) + k; (x) y2 (x) k~ (x) y~ (x) + k; (x) y; (x)

And so on ....

+ · · · + k~(x)yn(x) = 0 + · · · + k~(x) y~ (x) = 0

11 oo : 02 : s6 k~ (x)y1(x) k~ (x) y~ (x)

+ k;(x)y2(x) + · · · + k~(x)yn(x) = 0 + k; (x) y; (x) + · · · + k~(x) y~(x) = 0

And so on ....

2 k~(x)yin- )(x)

2 + k;(x)y~n- )(x)

2 + · · · + k~(x) yti- )(x) = 0 an X

So, remember that t he unknown coefficients ~rill be always as derivatives. The order of derivatives of t he CF's will be increased by every equation. Notice t he last equat ion carefully. RHS has been changed ...! We will give example of 2nd order different ial equation. Say you have

d2y dx2

dy + P (x) dx

+ Q(x)y = J(x) 64

[email protected]

physicsguide CSIR NET , GATE

@Sk J a hiruddin , 2020

Diff eq and Sp funs

The CF of t he equation is

We then take t he PI as

Then we get two equations

+ k;(x )y2 (x) = 0 k~ (x )y~ (x) + k;(x )y; (x) = J (x)

k~ (x )Y1 (x)

('

1

11

7

\/

1

7/ /

\

('

I

1

I

11 oo : 02 : sg + k;(x )y2(x) = 0 k~ (x )y~ (x) + k;(x )y;(x) = f (x) k~ (x )Y1 (x)

Now you can find t he k(x )' and k~(x) from t hese two equations and t hereaft er by integrating find k1 and k2. Example: solve

Use the variation of parameters method to

d2y dx 2

+ y = cosec x

subj ect to the boundary conditions y(O) = y(w/2) = 0

Solution: The CF is Yc(x) =

c1

sin x

+ c2 cos x

We assume t he PI

j [email protected]

@Sk J ahiruddin,

65

physicsguide CSIR NET, GATE

2020

Diff eq and Sp funs

Now look at t he general procedure to solve

ki (x) sin x + k~ (x) cos x = 0 ki (x) cos x - k~(x) sin x = cosecx Solving these equations for k~ (x) and k~ (x) gives

k~ (x) = cos X CSCX = cot X k~ (x) = - sin x csc x = - 1 TT



.

'

,

1

I

\

,

11 oo : 03 : 02 k~(x) k~ (x)

cotx

= COSXCSCX = = -

sin x csc x

=

- 1

Hence, ignoring the constants of integration, k1 ( x) and k2 ( x) are given by k 1 ( x) = ln (sin x)

k2(x ) = -x The general solution is t hen y (X)

= [C1 + ln (sin X) ]sin X+ (C2 - X) cos X

Apply the boundary conditions y(O) find c1 = c2 = 0 and so

y (x)

= ln (sin x) sin x

[email protected]

@Sk J ahiruddin, 2020

66

=

y(n/2)

=

0 we

- x cos x physicsguide CSIR NET, GATE

Diff eq and Sp funs

Example: Given that one solution of t he following

differential equation is x

Find t he other solution.

Solution: Solution:

We need to substitute y

11 oo : 03 : 04 Find the other solution.

Solution: Solution: We need to substitute y = u(x )v(x) as another solution. So, we get y' = xv'+ v, y'' = xv'' + 2v' . So the differential equation becomes

x 3 ( xv'' + 2v') + x (x v' + v) - xv = 0 or 2 4 3 x v'' + (2x + x ) v' = 0 Separating the variables and integrating, we get

dv'

2

1

X

2 X

-+

v'

dx,

1

In v' = - 2lnx + -

X

+ lnK

Solving for v' , integrating again, and writing y gives

uv



67

[email protected]

Diff eq and Sp funs

@Sk J ahiruddin , 2020

v'

K l/x X

2e

physicsguide CSIR NET, GATE

'

v = - Kel fx,

y = - K xel/x

Thus the general solution of t he given equation is y A x+ B xe 1 /x

=

11 oo : 03 : 06 v =

X

e 2

,

v

= - Ke ,

= - K xe

y

Thus the general solution of t he given equation is y = Ax+ B xe 1/x We can find the second solut ion in another way using Wronskian which will learn now.

3.2.6

Wronskian: Linear independence of functions and Second solution

If f 1 (x) , f 2 (x), · · · , fn(x) have derivatives of order n- 1, and if t he determinant

W=

!1(x) f{ (x) !{' (x)

! 2(x) f~(x) !~' (x)

• •

• • •



• • •

• • •

• • • • •

fi n- l)(x) JJn- l)(x)

fn(x) f~(x) Ji(x)

(3.7)

• •





• • •

t o

JJn- l) (x)

Then the functions are linearly independent . The deterj [email protected]

68

physicsguide CSIR NET 1 GATE

@Sk J ahiruddin , 2020

Diff eq and Sp funs

minant W is called the Wronskian of the functions. It should be noted, that W = 0 does not guarantee t hat t he functions are linearly dependent. Only if the Wronskian •

'

,



,

1

I

1

,,

r

11 oo : 03 : 09 minant W is called t he Wronskian of the functions. It should be noted, that W = 0 does not guarantee t hat t he functions are linearly dependent. Only if the Wronskian is zero over the ent ire range of the variable, t hen the funct ions are linearly dependent over t his range Example: Show that t he functions 1, x, sin x are linearly independent. Solution: We write and evaluate t he Wronskian, 1

W =

X



Sln X

0 1

cosx

0 0



=-



Sin X

- Sln X

since - sin x is not ident ically equal to zero, t he functions are linearly independent . Example: Check linear dependency of t he t hree funct ions cp 1 = ex , 0 vanish, and we obtain the indicial equation 1

11 oo : 04 : 40

L

(n + a) (n + a - 1)

n=O

+ 2(n + a ) + 4x

11 a nX '

=0

If we set x = 0 t hen all terms in the sum with n > 0 vanish, and we obtain the indicial equation 1 a (a-1) + a = O 2 j [email protected]

101

physicsguide CSIR NET 1 GATE

Diff eq and Sp funs

@Sk J ahiruddin , 2020

This has roots a = 1/ 2 and a = 0. since these roots do not differ by an integer, we expect to find two independent solutions to the equation. T he coefficients of xn vanish separately

If we choose the larger root, a = 1/ 2, of the indicial equation then previous equation becomes - an -1

an = - - - -

2n(2n + 1)

Setting ao = 1, we find an = (- 1)n/ (2n + 1) !, and so the solution becomes

11 oo : 04: 42

oo

(- l )n

vx- (VX) 3!

=

3

n

+ (VX) 5!

5 -

·· · =

To obtain the second solut ion we set a [email protected]

102

sin VX

= 0 (the smaller

physicsguide CSIR NET, GATE

@Sk J ahiruddin, 2020

Diff eq and Sp funs

root of t he indicial equation) which gives an-l an= - - - - 2n(2n - 1)

Setting a0 = 1 now gives an = (- l )1i/(2n) !, and so the second (independent ) solution becomes

00 (

y 2 (x) = ~ -

l )n

L..J (2n) !

(Jx)2

(y14)4

2!

4!

xn = l - - - + - -

n=O

- · · · = cosVX

We check that y1 (x) and y2(x) are linearly indep endent by computing the Wronskian:

vV = Y1Y; - Y2Y~

= sin VX 1

= -2VX

1

- 2VXX sin VX ('

Slll

2

cos VX

1

VX cos VX 2 X

c. 2 C.) 1 y X + cos y X = -2VX-:/ 0

11 1

= sin VX 1

= - 2,/x

oo : 04: 4s

- 2VXX sin VX ( .

2

r::.

Sln y X

+ COS

2

1

cos VX

VX cos VX 2 X

C.) 1 y X = - ,/x-/ 0 2

since W # 0, the solutions y1(x) and y2(x) are linearly independent. Hence, t he general solution is

y(x) =

c1

sin VX + c2 cos VX

j [email protected]

103

physicsguide CSIR NET I GATE

Diff eq and Sp funs

@Sk J ahiruddin , 2020

Example: Find series solut ion about x = 0 11

x(x - l )y

+ 3xy + y = 1

0

Solution: Dividing through by x (x- 1) to put the equat ion into standard form, we obtain II

3

1

I

y + (x- l )y + x(x- l )y=O We see p(x) = 3/(x - I ) and q(x) = 1/[x(x - 1)] . As x = 0 is a singular point but xp(x) = 3x/(x - 1) and x 2 q(x) x / (x - 1) are finite there, it is a regular singular point We put CX)

y

= x(J

L an x n n= O

Take derivative and get

3

00

\. ' r~ , _\~ ~n+u-1

11 oo : 04 : 48

Take derivative and get 3 00 2 L (n + a )(n + a - l )anxn+a- + x _ l L (n + a)anxn+a-l n=O n=O 00

l

00

n+a

Divide by xa- 2 , and get oo

L n=O

3x X (n + a )(n + a - l ) + --(n +a) + - x- l x- l

[email protected]

104

physicsguide CSIR NET ) GATE

Diff eq and Sp funs

@Sk J ahiruddin ) 2020

multiply by x - l to get 00

L [(x - l )(n + a )(n + a - l ) + 3x(n +a) + x]anxn = 0 n=O

If we set x = 0 t hen all t erms in t he sum with t he exponent of x greater t han zero vanish, and we obtain t he indicial equation

a(a - 1) = 0 which has the roots a = l and a = 0. since the roots differ by an integer, it may not be possible to find two linearly independent solut ions. We are guaranteed, however, to find

11 oo : 04 : so which has the roots CJ = l and CJ = 0. since t he roots differ by an integer, it may not be possible to find two linearly independent solut ions. We are guaranteed , however, to find one such solution corresponding to the larger root, CJ = l. The coefficient s of xn vanish separately we get t he recurrence relation

+ CJ) ( n - 2 + CJ ) an-1 - (n + CJ) ( n + CJ + 3(n - l + CJ )an- 1 + an-1 = 0

(n - l

l ) an

simplify to get

[email protected]

105

physicsguide CSIR NET, GATE

Diff eq and Sp funs

@Sk J ahiruddin , 2020

On substit ut ing

CJ

= l into this expression, we obtain n +l

n and on setting ao = l we find an = n + l ; so one solut ion is given by

00

Y1 (x) = x

L (n + l ) xn =

x ( 1 + 2x

+ 3x + •••) 2

n=O X

(1 -

X)2

,

11 oo : 04: s3 Y1 (x) = x

L (n + l ) xn =

x (1 + 2x

+ 3x + •••) 2

n=O X

(1 -

X) 2

If we attempt to find a second solut ion (corresponding to the smaller root of t he indicial equation) by setting a = 0 we find

n

n -1 But we require ao -=/- 0, so a1 is formally infinite and the method fails. We can find a second solution by Wronskian.

Example: Find a second solution of t he above equation by Wornskian method

j [email protected]

106

physicsguide CSIR NET, GATE

Diff eq and Sp funs

@Sk J ahiruddin , 2020

Solution: As we have seen t he method of obtaining a second solution in subsection 3.2.6 One solution we have already derived X

Y1 ( X)

So now using

= (1 - X) 2

11 oo : 04: 55 So now using

X

X

Y2 ( X)

(1

- 2 X2

= (1 - X) 2 X

X

X)2

(1 -

X

(1 -

X)2

exp -

X2) 4 - -- exp [-3 ln(x2 2 X2

3 ---dx1 X1 - 1 - l )]dx 2

- 1

X2

ln x

1

+-

X

[email protected]

107

physicsguide CSIR NET, GATE

@Sk J ahiruddin, 2020

Diff eq and Sp funs

4.2

Special equations and functions

4.2.1

Legendre equation and Polynomials

This is t he Legendre equation (1 - x

dx2

--2-dx2

X)2

(1 -

X2

(1 -

x X2

X

)4

X2

2

)

y'' - 2xy'

+ /!, (/!, + 1)y = 0

x = 0 is an ordinary point. So substitute

11 oo : 04 : sa This is t he Legendre equation

(1 - x

2

)

y'' - 2xy' +f(f + l )y

=0

x = 0 is an ordinary point. So substitute

You get CX)

L

2 [n(n - l )anxn- - n(n - l )anxn - 2nanxn + f(f + l )anxn]

n =O

=0 From here 00

L {(n + 2) (n + l )an+2 -

[n(n + 1) - f (f + l )]an } xn = 0

n=O

We now get the recurrence relation

[n(n + 1) - f(f + 1)) an+2 = (n + l )(n + 2) an [email protected]

108

physicsguide CSIR NET, GATE

Diff eq and Sp funs

@Sk J ahiruddin , 2020

for n = 0, 1, 2, ... . If we choose a0 we obtain the solution

=

1 and a 1

0 then

=

-

...

11 oo : os : oo

-

.. .

whereas on choosing a0 = 0 and a 1 = 1 we find a second solution

y2(x) = x-(R- 1)(£+ 2)

x3

x5

+(R- 3)(R- l )(R+ 2) (R+4) ! -· · · 31 5

In many physical applications the paramet er Rin Legendre's equation is an integer, i.e. R = 0, 1, 2, .... In t his case, the recurrence relation gives

_ [R(R + 1) - R(R + 1)] a i+2 -

(R+ l )(l + 2)

_ ai -

0

t he series terminates and we obtain a polynomial solut ion of order R . These solutions, after normalization, are called the Legendre polynomials of order R they are written Pi(x) and are valid for all finite x . It is conventional to normalise Pi(x ) in such a way t hat Pi(l ) = 1 and as a consequence j [email protected]

109

@Sk J ahiruddin , 2020

physicsguide CSIR NET 1 GATE

Diff eq and Sp funs

Pi( - 1) = (- 1)i . The first few Legendre polynomials are easily constructed and are given by

Po(x) = 1, P1 (x) = x 2 P2(x) = ~ (3x - 1) , P3(x) = ~ (5x 3 - 3x ) 4 2 P4(x) = ~ (35x - 30x + 3) , Ps(x) = ~ (63x 5 - 70x 3 + 15x)

11 oo : os : 03 •

easily constructed and are given by

Po(x) = 1, P1(x) = x 2 3 P3(x) = ½(5x - 3x) P2(x) = ~ (3x - 1), 4 2 3 5 P4(x) = ~ (35x - 30x + 3) , Ps(x) = ~ (63x - 70x + l 5x) First few Legendre polynomials are shown here graphically. 2

y

Po

- 1

1

-0.5

X

- 1

-2

Second solut ion of the Legendre equation is Qe(x ) is, an [email protected]

110

physicsguide CSIR NET, GAT E

@Sk J ahiruddin, 2020

Diff eq and Sp funs

infinite series that converges only for

lxl < 1

l +x l- x

11 oo : os : 06 IxI
2

11 oo : os : 29 find

[(a

[(a+ 1)

2

2

an

+ n)

-

v

2

]

v

-

2

] a1

=0

+ an- 2 = 0

for n > 2

Substituting a = ±v you get

(1 ±2v)a 1 = 0 n(n±2v)an+an-2= 0

forn > 2

We consider now the form of the general solut ion to Bessel's equation for two cases: (i) v is not an integer [email protected]

119

physicsguide CSIR NET, GATE

Diff eq and Sp funs

@Sk J ahiruddin , 2020

(ii) v is an integer (including ZERO)

( i) v is not an integer We need to set 1

an = - n(n ± 2v) an-2

=0 It is a general convention (have physical insight which you will learn while doinf advanced electrostatics problems) t o set 1 a0 -- ') ±vl' / 1 -!- /JI\

11 oo : os : 32 It is a general convention (have physical insight which you will learn while doinf advanced electrostatics problems) to set 1 ao = 2±vr (1 ±v) Then we write the solut ion as 1

l v(x) = r (v + 1) X

2

V

l X 1--v +l 2

2

1

1

X

+ (v +l)(v+ 2) 2!

2

oo (- l )n x =L n!f(v+n+ l ) 2

4 -

...

v+2n

n=O

(4.1) We replace v by -v which give us J_v(x)

j [email protected]

120

@Sk J ahiruddin , 2020

physicsguide CSIR NET 1 GATE

Diff eq and Sp funs

The functions l v (x) and J_v (x) are called Bessel funct ions of the first kind, of order v . You must know that Bessel functions of half-integer order are expressible in closed form in terms of trigonometric functions. Example: Calculate

J±1;2(x) .

Solution: Use t he general formula of Bessel function for non integer v as derived above. You get

11 oo : os : 34 Example: Calculate J±1; 2 (x) .

Solution: Use t he general formula of Bessel function for non integer v as derived above. You get

Using the fact that r(x + 1) find that , for v = l / 2

=

121

[email protected]

xr (x) and

r (½)

=

fa,

we

physicsguide CSIR NET , GATE

@Sk J a hiruddin , 2020

Diff eq and Sp funs

(½x)5/2

(½x)9/2

1,r (~) + 2,r (;) - · · · (½x)5/2 (½x)9/2 1! ( ~) (t) fa + 2! ( ~) ( ~) (1) fa - ... x2 x4

1 - - + - - ··· 3! 5!

fT

11 oo : os : 36 -x 2

-x 2

-x 2

(~) fi - 1! (~) (~) fi + -2,-(~-)-(~-)-(t_)_fi_1r - ... (t x)1/ 2 x2 x4 1 - - + - - ··· (~) fi 3! 5! 2

1x ) 1/ 2 Sln . X

(2

(~) \j'ff

.

-s1nx

X

7rX

For v = -1/ 2 we get

2 - cosx 7rX

(ii) v is an integer Firstly, let us consider the case v = 0, so that t he two solutions to the indicial equation are equal, and we clearly obtain only one solution in the form of a Frobenius series. j [email protected]

122

physicsguide CSIR NET I GATE

Diff eq and Sp funs

@Sk J a hiruddin , 2020

Use equation 4. 1 to get (- l )nx2n

oo

x2

x4

x6

= 1 - 22 + 2242 - 224252 + ... In e-eneral. however. if v is a oositive intee-er then the

11 oo : os : 39 n =O

22nn!I'(l x2

= 1-

22

+ n)

x4

+

x6

2 24 2 -

2 24 25 2

+ ...

In general, however , if v is a positive integer then the solutions of the indicial equat ion differ by an integer. For t he larger root , o-1 = v, we may find a solution l v (x), for v = l , 2, 3, ... , in the form of the Frobenius series. First three Bessel functions are plotted below J(x) 1

0.5

- 0.5 123

[email protected]

physicsguide CSIR NET ) GATE

@Sk J ahiruddin ) 2020

Diff eq and Sp funs

For the smaller root, o-2 = -v, the recurrence relation (n(n ± 2v )an+ an-2 = 0 for n > 2) b ecomes

n(n - m)an + an-2 = 0 IYYl

-

')

I) !

.

, c,

Ti l'"\TT T

t"'ITI

CIT TCI TI

for n > 2

.+.

'Y"\ l'"\C, 1

1T TCI

.

, n + orro r

.

1

Cl

IYYl

-

11 oo : os : 42 For t e sma er root,

(n(n

± 2v)an + an- 2 = 0

= -v, t e recurrence re ation for n > 2) becomes

2

m = 2v is now an even posit ive integer , i.e. m = 2, 4, 6, .... Starting with ao -=/=- 0 we can calculate a2, a4, a5, ... , but when n = m t he coefficient an is infinite, and the method fails to produce a second solution . Replace v by -v in the definition of Jv(x) given in equat ion 4. 1, you will be able to show t hat, for integer v,

So, Jv(x) and J_v (x) are linearly dependent. In this case, we cannot write the general solution to Bessel's equation in t he form y(x)

= c1 Jv(x) + c2 J-v(x)

So we define the function

Yv(x) = Jv(x) cos_v1r - J_v(x) Sln V1r

(4.2)

This is called a Bessel function of the second kind of order v. It is also called Weber or Neumann function. As [email protected]

@Sk J a hiruddin , 2020

124

physicsguide CSIR NET , GATE

Diff eq and Sp funs

Bessel's equation is linear , Yv(x) is clearly a solut ion , since it is just t he weighted sum of Bessel functions of the first kind.

11 oo : os : 44 Bessel's equation is linear , Yv(x) is clearly a solut ion, since it is just t he weight ed sum of Bessel functions of the first kind. It can be shown that t he Wronski an of Jv(x) and Yv(x) is non-zero for all values of v . Hence Jv(x) and Yv(x) always constitute a pair of independent solutions.

Example: If n is an integer , show that

Solution: From equation 4.2 Yn+ l / 2(X )

=

Jn+1;2(x) cos (n + ½) 7f

If n is an integer , cos (n

-

. ( + 1) Sln n ?. 1f

J-n-1;2(x )

-

+ ½) 1r =

0 and sin ( n

+ ½) 1r =

(-l)n

The Neumann function (equation 4.2 ) becomes an indeterminate form 0/ 0 when v is an integer , as for integer v we have cosv1r = (-l )v and J_v(x) = (-l) vJv(x) . This j [email protected]

@Sk J ahiruddin, 2020

125

physicsguide CSIR NET, GATE

Diff eq and Sp funs

indeterminate form can be evaluated using l'Hopital's rule

11 oo:os:47

@Sk J ahiruddin , 2020

Diff eq and Sp funs

indeterminate form can be evaluated using l'Hopital's rule . Therefore, for integer v, we set

Jµ(x) cos µ1r - J_µ(x)

Yv(x) = lim - - -.- - - µ ---tv

Sl n

µ 7r

which gives a linearly indep endent second solution for t his case. Thus, we may write the general solut ion of Bessel's equation, valid for all v as

The functions Y0 ( x), Y1 ( x) and Y2 ( x) are plotted in figure below

[email protected]

@Sk J ahiruddin, 2020

126

physicsguide CSIR NET, GATE

Diff eq and Sp funs

11 oo : 05 : 50 [email protected]

126

Diff eq and Sp funs

@Sk J ahiruddin ) 2020

1

physicsguide CSIR NET ) GATE

Y(x)

Yo

0.5

ox

1

-0.5

-1

Finally, in some applications, it is convenient to work with complex linear combinations of Bessel functions of t he first and second kinds given by

t hese are called, respectively, Hankel functions of t he first and second kind of order v .

[email protected]

127

physicsguide CSIR NET ) GATE

11 oo : 05 : 53 •

[email protected]

127

physicsguide CSIR NET , GATE

Diff eq and Sp funs

@Sk J a hiruddin , 2020

Orthogonality: b

x l v(ax)Jv(f3x)dx = 0

for a-:/

/3

a

Recurrence relations: Try to prove these relations. You may not need to remember all. But prove yourself at

least once.

(i) d~ [xv Jv(x )] = Xv l v- l (x) d (ii) dx [x- vl v(x)] = -x- vl v+1(x)

(iii) x J~(x)

+ v l v(x) = x l v-1 (x)

(iv) xJ~(x) - v l v(x) = - xlv+1(x)

(v) l v-1(x) - l v+1(x) = 2J~(x) (vi) l v-1 (x)

+ l v+1 (x) =

2v

- Jv(x) X

Relation (i) and (ii) can also be written as integral form

Jxv l v-1(x )dx = xvl v(x) Jx-v l v+l (x )dx = -x-v l v(x) How to prove these? Let me give you some hints. Relation (i) and (ii) comes directly from the definition

11 oo : 05 : 55 How to prove these? Let me give you some hints. Relation (i) and (ii) comes directly from the definition

128

j [email protected]

@Sk J a hiruddin ,

physicsguide CSIR NET, GATE

2020

Diff eq and Sp funs

of Bessel function. The prove of relation (i) is t he following.

00

=

L

n= O

(- l )nx2v+2n-1 2v+2n-ln!f(v+n)

00

=

l )nx(v- l )+2n

( -

x v l v- 1(x)

Expand out the derivative on t he LHS of (i) and divide by xv-I , you obtain relation (iii) Similarly,expand the derivative on t he LHS of (ii) and divide by x v+I , you get relation (iv) Add relation (iii) and (iv) to get (v) Subtract (iv) from (iii) to get (vi) 1 2

Example: You are told t hat = (2/ 1rx) 1 sin x 1 2 and J_ 1; 2 (x) = (2/ 1rx ) / cos x,. Now express J3; 2 (x) and J1; 2 (x)

J_ 3; 2 ( x) in trigonometric functions.

11 oo : 05 : 5a Subtract (iv) from (iii) to get (vi) Example: You are told t hat J1; 2 (x) = (2/ 1rx ) 112 sin x and J_ 1; 2 (x) = (2/ 1rx ) 1/ 2 cos x,. Now express J3; 2 (x) and

J_ 3; 2 ( x) in t rigonometric functions. Solution: From relation (iv) 129

[email protected]

physicsguide CSIR NET, GATE

@Sk J ahiruddin, 2020

Diff eq and Sp funs

1;12(x)

J3;2 (x) = ~ J1;2(x) 2

2

1

2x 2

2

1/ 2 •

Sln X -

1fX 1/2

1 .

- Sln X

1fX

1

1/ 2 COSX

1fX

+-

2x

2 1fX

1/2 •

Sln X

COS X

-

X

From relation (iii)

J_3;2(x) = - ~ l-1; 2(x) + J~1;2(x) 2

1 2x 2 1fX

2 1fX 1/2

2

1/ 2

cosx -

1 - - COS X

-

1fX

1/ 2

1 • Sln X - 2x

2 1fX

1/2

cosx

. Sln X

X

We see that, by repeat ed use of t hese recurrence relat ions, all Bessel functions Jv(x) of half integer order may be expressed in terms of trigonometric functions. So as the

11 oo : 06 : oo We see that , by repeat ed use of these recurrence relat ions, all Bessel functions l v (x) of half integer order may be expressed in terms of trigonometric functions. So as the Yv(x) . Generating function: The Bessel functions l v (x), where v = n is an integer, can be described by a generating [email protected]

physicsguide CSIR NET, GATE

130

@Sk J ahiruddin , 2020

Diff eq and Sp funs

funct ion in a way similar to that discussed for Legendre polynomials

G(x, h) = exp

00

h- .!.

X

2

(4.3)

h

n=-oo

Integral representations: Bessen function can be written as integral for n an integer. 7f

1

Jn(x) = -

7r

cos( n0 - x sin 0)d0 0

Special case n = 0 J0 (x) =

4 .2.5

1

21r

exp(ixsin0)d0

n

2

O

21r

1 =

exp (ix cos 0)d0

n

2

Spherical B essel functions

0

11 oo : 06 : 03 21r 4.2.5

21r

O

0

Spherical Bessel functions

When solving Helmholtz' equation (V + k ) u = 0 in spherical polar coordinates, t he radial part R(r) of t he solut ion satisfy the equation 2

131

j [email protected]

2

physicsguide CSIR NET 1 GATE

@Sk J ahiruddin, 2020

Diff eq and Sp funs

where f is an integer. You see that This equation looks very much like Bessel's equation and can be reduced to it by writ ing R(r) = r - 1!2 S(r), (Do t his !) in which case S(r) then satisfies 1 f+2

Doing change of variable x we obtain

=

2

S= O

kr and letting y ( x)

1 f+2

=

S (kr),

2

y= O

where t he primes now denote d/ dx . This is Bessel's equat ion of order f + and has as its solutions y(x) = J p_+ 1; 2 (x) and Yt+ 1; 2 ( x) . The general solution of spherical Bessel equa-

1

11 oo : 06 : os

where t he primes now denote d/ dx . This is Bessel's equa-

t

t ion of order f + and has as its solut ions y(x) = Jt+ 1; 2 (x) and Yt+ 1; 2 (x). The general solution of spherical Bessel equat ion (4.4) can then be written

1 2

1 2 /

The functions x- ! Jt+i ; 2 (x) and x- ½+ 1; 2 (x), after normalization, are called spherical Bessel functions of t he

[email protected]

132

@Sk J ahiruddin, 2020

physicsguide CSIR NET, GATE

Diff eq and Sp funs

first and second kind, respectively, and are defined as follows:

For f

=0 •

. ( )

Jo

X

=

Sln X X

Recurrence relations:

t ion is given by

COS X

n0( x) = - - x t he f th spherical Bessel func-

11 oo :06 : oa t he I!, th spherical Bessel func-

Recurrence relations:

t ion is given by I

l d

xdx

fo(x)

where fg(x) denotes either jg(x) or ng(x) First two spherical Bessel functions are

= sin x x2 n (x) = _ cosx

. (x) J1 1

X

_

.'

s1n x

x2

j a [email protected]

X

= (l.. _ l ) sin X _ 3 cos x x3 x x2 3 n2(x) = - (; 3 - ; ) cos x - ~~ x

(x) . J2

cosx '

133

physicsguide CSIR NET I GATE

@Sk J a hiruddin , 2020

4.2.6

Diff eq and Sp funs

Hermite equation and function

Hermite equation is y'' - 2xy'

+ 2v y = 0

This has an essential singularity at x = oo. The paramet er v is a given real number , although it nearly always takes an integer value in physical applications. The Hermite equation appears in the descript ion of the wavefunction of t he harmonic oscillator. Any solution of Hermite equat ion is called a Hermite function.

11 oo : 06 : 1 o The Hermite equation appears in the description of the wavefunction of t he harmonic oscillator. Any solution of Hermite equat ion is called a Hermite function . Do t he series solut ion about x == 0 yourself. First few Hermite polynomials are

Ho(x) == 1, H 1 (x) == 2x, 2 H2(x) == 4x

2

-

H3(x) == 8x - 12x H4(x) == 16x 4 - 48x 2 + 12 2, H5(x) == 32x 5 - 160x 3 + 120x

The Rodrigues' formula for t he Hermite polynomials is given by Rodrigues' formula:

e [email protected]

@Sk J ahiruddin ) 2020

Orthogonality: 00

- 00

Generating function:

Recurrence relations:

134

- x2

physicsguide CSIR NET ) GAT E

Diff eq and Sp funs

11 oo : 06 : 13

Recurrence relations:

Hn+1(x) = 2x Hn(x) - 2n Hn-1 (x) H~(x) = 2n Hn-1 (x)

135

[email protected]

physicsguide CSIR NET , GATE

Diff eq and Sp funs

@Sk J a hiruddin , 2020

4.3

Exercises

4.1. Find a series solution of Hermite equation about x

y'' - 2xy' + 2vy = 0 choose a0 = (- l )nl 2n !/(n/2)! , for odd n take 1

2

(n- l ) !

=0

11 oo : 06 : 16 choose a0 = (- 1)nl 2n !/ (n/2) !, for odd n take

4.2. Solve Laguerre's equation about x

=0

xy'' + (1 - x )y' + vy = 0 choose a0

=1

4 .3. Find power solutions in z of t he differential equation

about z = 0

+ 9z 5y =

zy'' - 2y'

0

Ident ify closed forms for the two series, calculate their Wronskian , and verify that they are linearly independent . Compare the Wronskian wit h t hat calculated from t he different ial equation. 4.4. Investigate solut ions of Legendre's equation at one of

its singular points as follows.

j [email protected]

@Sk J ahiruddin , 2020

136

physicsguide CSIR NET, GATE

Diff eq and Sp funs

(a) Verify t hat z = I is a regular singular point of Legendre's equation and that t he indicial equation for a series solution in powers of ( z - I) has a double root a = 0 . (b) Obtain the corresponding recurrence relation and show t hat a polynomial solut ion is obtained if R is a positive integer. (c) Determine t he radius of convergence R of t he a = 0

11 oo : 06 : 1 a solution in powers of ( z - l ) h as a double root a

=0

.

(b ) Obtain t he corresp onding recurrence relation and show t h at a polyn omial solut ion is obtained if R, is a p osit ive integer. (c) Determine the radius of convergence R of the a

=

0

series and relat e it to the posit ions of the singularities of Legendre's equation. 4.5. ( a ) Find series solutions of the equat ion y'' - 2zy' - 2y

0. Identify one of the series as y 1 ( z)

=

=

2

exp z and verify this

by direct substitution. (b ) By setting y2 (z) = u( z )y1 (z ) and solving the resulting equ ation for u(z), find an explicit form for y2 (z) and deduce t h at X

I

(X)

~

n . ?. +I - - - - (2x)~n

n= O

2(2n

e- v dv = e- x ~ 2

2

o

+ 1) !

4.6. Find the radius of convergence of a series solution ab out

t he origin for t he equation ( z

+ az + b) y'' + 2y =

2

0 in the

following cases:

a = 5 b = 6·

'

'

(b) a = 5, b = 7 2

Show t h at if a and b are real and 4b > a then the radius of convergence is always given by 6112 . [email protected]

137

@Sk J ahiruddin, 2020

physicsguide CSIR NET, GATE

Diff eq and Sp funs

+ z-

3

y = 0, show t he origin b ecomes a regular singular point if the indep endent variable is ch anged from z t o x = l/ z . Hen ce find a series solution of the form y 1 (z) = anz-n .B y setting Y2( z ) = u(z )y1 (z ) an d expanding t he result ing expression for du/ dz in p owers 4. 7. For t he equation y''

~r

11 oo : 06 : 21 4. 7. For t he equation y'' + z- 3 y = 0, show t he origin be-

comes a regular singular point if the indep endent variable is changed from z to x of the form Y1( z ) =

=

l / z . Hence find a series solution

I:r anz-n.By setting Y2( z) = u(z )y1(z)

and expanding t he result ing expression for du/ dz in powers of z-

1

,

show that y 2 (z) is a second solut ion with asymptotic

form y2 ( z)

=

1 c z + In z 2

+0

ln z z

where c is an arbitrary constant 4.8. Use t he explicit expressions of Spherical Harmonics as

given inside the notes to verify for

.e = 0, 1, 2 t hat

and so is independent of the values of 0 and for any

cp. This is true

.e , but a generatly proof is more involved. This result

helps to reconcile intuit ion with t he apparently arbitrary choice of polar axis in a general quantum mechanical system. 4.9. Express the function 2

2

2

f(0 , cp) = sin 0 [sin (0 / 2) cos cp + i cos (0 / 2) sin ] +sin (0 / 2) as a sum of spherical harmonics. jahir@physicsguide. in

@Sk J ahiruddin , 2020

138

physicsguide CSIR NET, GATE

Diff eq and Sp funs

4.10. Use the generating function for t he Legendre polynomials Pn (x) to show t hat

11 oo : 06 : 23 @Sk J a hiruddin ,

2020

Diff eq and Sp funs

4.10. Use the generating function for t he Legendre polynomials Pn (x) to show t hat

(2n) ! o P2n+1(x)dx = (- 1) 22n+ln!(n + 1)! n

l

and except n = 0 1

P2n(x)dx = 0 0

4.11. Do step by step to prove the result

2

1

Pn(z) Pn(z)dz = - -1 2n + 1 (a) Square both sides of the generating-function definition of the Legendre polynomials, (X)

(1 - 2zh + h

2

)-

112

=

L Pn(z) hn n=O

(b) Express the RHS as a sum of powers of h, obtaining expressions for the coefficients (c) Integrate the RHS from -1 to 1 and use the orthogonality property of t he Legendre polynomials. (d) Similarly integrate t he LHS and expand t he result in powers of h. (e) Compare coefficients.

j [email protected]

@Sk J a hiruddin ,

2020

139

physicsguide CSIR NET , GATE

Diff eq and Sp funs

4.12. By substituting y(x) as x 112 _f (x) reduce Stokes' equa-

11 oo : 06 : 26

@Sk J a hiruddin , 2020

Diff eq and Sp funs

2 4.12. By substitut ing y(x) as x 1 f (x) reduce Stokes' equation, d2y dx2 + Axy = 0 1

to Bessel's equation. Hence show t hat a solut ion t hat is 1 2 finite at x = 0 is a mult iple of x / J1; 3 j ~ 4.13. The hypergeometric equation is

x( l - x)y'' + [c - (a+ b + l )x]y' - aby = 0 Solution of this equation is

ab x

+ F (a, b, c; x) = 1 + - c 11.

=

f (c) r (a)f (b)

2 a(a + l )b(b + 1) x

f n=O

( l) + · · · 21. cc+ f (a + n)f (b + n) xn r (c + n)

n!

F (a, b, c; x) is known as t he hypergeometric function or hypergeometric series. Now, Identify the series for the following hypergeometric functions, writing them in terms of better-known functions. (a) F (a , b, b; z) (b) F (l , 1, 2;-x) (c) F (½, 1, ~; -x 2 ) 2 (d) F ( ~, ~, ~; x ) [email protected]

@Sk J a hiruddin , 2020

140

physicsguide CSIR NET , GATE

Diff eq and Sp funs

11 oo :06 : 2a [email protected]

140

physicsguide CSIR NET, GATE

@Sk J ahiruddin, 2020

Diff eq and Sp funs

4.14. If y(x, a) is a solut ion of the equation

1 2 -x + a y 4

=0

Determine which of the following are also solutions: (i) y( a, -x), (ii) y( -a, x), (iii) y( a, ix) and (iv) y( -a, ix)

j ahir@physicsguide. in

141

physicsguide CSIR NET, GATE

11 oo : 06 : 31

[email protected]

141

physicsguide CSIR NET , GATE

Diff eq and Sp funs

@Sk J a hiruddin , 2020

4 .3 .1

Ans keys

2 2m ([n/ 2] denotes t he inte4 .1. '\:""[ri/ ](l )m n! (2x)nD m=O m!(n-2m)! ger part of n/2)

4.2. I:: =o(- l )m (m!)2(~- m)!Xm

4 .3.

Y1 =

a 0 sin z 3 and y 2 = b0 cos z 3

4.4. t he series converges in a circle of radius 2 cent red

on z

=

l.

4 .5. ( b ) ex2 A

Je- v dv 2

4.6. (a) R = 2

4 7

• • YI

( )Z

-

ao

(b)R = v'7 '\:""oo (-l)n D n=O (n+ l )(n!) 2 zn

4.8. This is a proof. 4.9. ~

Yao -

.!. Y,0 3 1

4 .10. This is a proof. 4 .11 . This is a proof. 4 .12. This is a proof.

.1. y - 1 15 2

11 oo : 06 : 34 4.10. This is a proof. 4.11. This is a proof. 4.12. This is a proof. j [email protected]

142

physicsguide CSIR NET 1 GATE

@Sk J ahiruddin , 2020

4.13. (a) (1 - z)-a, (b) ; ln(l x- 1 sin- 1 x

Diff eq and Sp funs

+ x),

4.14. (i) Yes (i) No (iii) No (iv) Yes

(c) x- 1 tan- 1 x, (d)

11 oo : 06 : 36

[email protected]

143

physicsguide CSIR NET, GATE

@Sk J ahiruddin, 2020

4.3.2

Diff eq and Sp funs

Solutions

Solution: 4.1. Since x

0 is an ordinary point of t he

equation, put 00

substituting you get 00

L [(m + l )(m + 2)am+2 + 2(v -

m)am] xm = 0

m=O

Demanding that t he coefficient of each power of x vanishes, we obtain the recurrence relation

a 2 m+

2(v - m) - - - - - - - -a

-

(m + l )(m + 2)

m

In nearly all physical applications, the parameter v takes integer values. Therefore, if v = n, where n is a non-negative integer, we see that a n+ 2 = a n+4 = · · · = 0, and so one solution of Hermite's equation is a polynomial of order n. Fo1~ even n, it is convent ional to choose a0 = (- l )nf 2n!/(n/2) !,

11 oo : 06 : 39 teger values. Therefore, if v = n, where n is integer, vve see that an+2 = an+4 = · · · = 0, lution of Hermite's equation is a polynomial even n, it is conventional to choose a0 = (whereas for odd n one takes

j [email protected]

144

a non-negative and so one soof order n. For 2 1)n/ n !/ (n / 2) !,

physicsguide CSIR NET I GATE

@Sk J ahiruddin, 2020

Diff eq and Sp funs

These choices allow a general solution to be written as

Hn(x) = (2x)n - n(n - 1)(2x)n- l

+ n(n -

l )(n - 2)(n - 3) ( x) 71__ 4 _ 2 2! [n/2] I = ~ (-l)m n. (2x)n-2m ~ m !(n- 2m)!

...

m= O

where Hn(x) is called the nth Hermite polynomial and the notation [n/2] denotes the integer part of n/2. We should note t hat Hn(-x) = (- l )nHn(x) .

Solution: 4.2. Since t he point x = 0 is a regular singularity, put 00

y(x) =

L

amXrri+a

m=O

Substit ute t his into the equation and divide t hrough by xa- l, we get 00

11 oo : 06 : 42 y(x) = .__ amx m=O

Substitute this into the equation and divide through by xa- l , we get 00

L ((m + a )(m + a - l ) + (1 - x)(m + a )+ vx]amxm = 0 m=O

Setting x = 0, so that only the m = 0 term remains, we obt ain the indicial equation a 2 = 0, which trivially has a = 0 as its repeated root. Thus, Laguerre's equation has only [email protected]

@Sk J ahiruddin ) 2020

145

physicsguide CSIR NET ) GATE

Diff eq and Sp funs

one solution and it, in fact , reduces to a simple power series. Substituting a = 0 into above equation and demanding that t he coefficient of xrri+l vanishes, we obtain the recurrence relation m-v am+l = (m + l )2 am In nearly all physical applications, the paramet er v takes integer values. Therefore, if v = n , where n is a non-negative integer, we see t hat an+l = an+2 = · · · = 0 , and so our solution to Laguerre's equation is a polynomial of order n. It is conventional to choose a0 = l , so that t he solution is polynomial



1

1 \n __

,l

11 oo : 06 : 44 po ynom1a

Ln(x)

n

Check that first few Laguerre's polynomials are

Lo(x) = 1

L 1 ( x)

= - x +1

[email protected]

L 2 ( x)

146

= (x

2

-

4x

+ 2) / 2!

physicsguide CSIR NET , GATE

Diff eq and Sp funs

@Sk J a hiruddin , 2020

Solution: 4.3. Putt ing the equation in its standard form shows t hat z = 0 is a singular point of the equation

but , as -2z/ z and 9z 7 / z are finite as z ➔ 0, it is a regular singular p oint . We t herefore substit ute a Frobenius type solut ion , 00

y(z ) = zo-

L anzn wit h ao -/= 0 n=O

We get 00

00

n=O

n=O 00

+9L n=O

anzn+o-+5 = 0

11

00:06:47

n=O

n=O 00

+9L

5 a nzn+a-+

=0

n=O

Equating the coefficient of za-- l to zero gives the indicial equation as a(a - l )a0

-

2aao

=0

a = 03

'

These differ by an integer and may or may not yield two independent solut ions. The larger root , a = 3, will give a solution; the smaller one, a = 0, may not.

(a) a = 3 j [email protected]

147

physicsguide CSIR NET, GATE

Diff eq and Sp funs

@Sk J ahiruddin , 2020

2

Equating the general coefficient of zm+ to zero (with a = 3) gives

(m + 3)(m + 2)am - 2(m + 3)am + 9am - 6 = 0 So the recurrence relation is am=⇒

9am - 6

m(m + 3) 9

a5 = - - - - -a 5 P

6p(6p + 3)

6 = -

p-

The solution is t hen 00

111

(.r,)

=

Q,n z

3)

a6p- 6

2p(2p + 1)

11 oo : 06 : 49 9 a5 = - - - - -a 5 P 6p(6p + 3) p -



-1 Pao

a6p- 6

6 = -----

2p(2p + 1)

(2p + 1)!

The solution is t hen

=

(b)

o-

oo

(- l )n 3 2 a0 ~ - - - - z ( n+l )

~ (2n + 1)!

=

a0 sin z 3

=0

Equating t he general coefficient of a-= 0) gives

[email protected]

zm-l

to zero (with

physicsguide CSIR NET, GATE

148

@Sk J ahiruddin, 2020

Diff eq and Sp funs

Hence t he recurrence relation is 9am - 6

am= - - - - -

m(m- 3) 9

a 6P = -

a 6p-6

6p(6p - 3) a 6p- 6

= - 2p(2p - 1)

So the solution

oo (- l )n 6n

3

11 oo : 06 : s2 e so ution oo (- l )n 6n

3

We see that a = 0 does, in fact, produce a (different) series solution. This is because the recurrence relation relates an to an+6 and does not involve an+3 the relevance here of considering the subscripted index m + 3 is t hat 3 is the difference between t he two indicial values . We now calculate the Wronskian of the two solutions, Y1

= ao sin z 3 and Y2 = bo cos z 3

W (y1, Y2)

:

= Y1Y~ - Y2Y~ 2 3 3 = ao sin z ( -3boz sin z ) 2 = -3aoboz i= 0

jahir@physicsguide. in

149

-

bo cos z

3

( 3aoz

2

cos z

3

)

physicsguide CSIR NET, GATE

@Sk J ahiruddin , 2020

Diff eq and Sp funs

The fact t hat the Wronskian is non-zero shows t hat the two solutions are linearly independent. We can also calculate the Wronskian from the original equation in its standard form, 2 ' y '' - -y

z

as

+ 9z 4 y = 0

11 oo : 06 : s4 equation in its standard form,

y

11 -

2 1 4 - y + 9z y = 0 z

as

W = C exp

z

-

- 2

-

u

du

2

C exp (2ln z) = Cz

=

This is in agreement with the Wronskian calculated from t he solutions, as it must be.

Solution: 4 .4 . (a) In standard form, Legendre's equat ion is ii 2z i R( I!+ l ) - 0 y l - z2y + l - z2 y This has a singularity at z

-2z(z - 1) l - z2



=

l , but, since

d R(f! + l )(z - 1) 1~ l -z2

2



0

asz ➔ l

bot h limits are finite, the point is a regular singular point. We next change the origin to the point z j [email protected]

150

physicsguide CSIR NET 1 GATE

Diff eq and Sp funs

@Sk J ahiruddin, 2020

u

=

z - l and y(z) 111 _

= l by writing

=

j(u) . The t ransformed equation is

2(u + 1)

- u(u + 2)

1 1

+

/!(/! + l ) y = 0 - u(u + 2)

or

-u(u + 2)f

11 -

2(u + l )f

1

+ R(R + l )f =

0

11 oo : 06 : s1

or

-u(u + 2)f'' - 2(u + l )f'

+ f(f + l )f =

0

The point u = 0 is a regular singular p oint of this equation and so we set f(u) = u(T I:~=oanun and obtain 00

-u(u + 2) L (a

+ n)(a + n

- l )anu s0 )

I 1(s)

I so

C

c/s

0

ctn

en!/ sn+l

0

2

2 a ) 2 a )

eat tneat

a/ (s + 2 s/ (s + 1/(s - a) n! /(s - a)n+l

- : ........ 1-.. _ ..,_

~

sin at cos at

I ( ~2

~.2\

0 0

a a

I I

.

11 oo : oo : 26 a/ (s + a ) 2 2 cos at s/ (s + a ) eat 1/(s - a) tneat n! /(s - a)n+l 2 2 sinh at a/ (s - a ) coshat s/ (s 2 - a 2 ) sin at

1.3 1.3.1

2

2

0 0 a a la la

Properties of Laplace 'Iransformation Linear property

Laplace Trans£ormation is linear

j ahir@physicsguide. in

10

@Sk J a hiruddin , 2020

1.3.2

physicsguide CSIR NET I GATE

Integral Transformations

Shifting property

We must know t he shifting property of Laplace transformat ion CX)

0 CX)

J(t)e- (s- a)tdt . n

(1.1)

11 oo : oo : 29 00

£ [eat f (t )]

=

f (t )eate- stdt 0 00

f (t )e- (s- a)t dt

(1.1)

0

= f (s - a) 1.3.3

Laplace transformation of derivatives

Laplace t ransform of first derivative

£

oo

df dt

d+

_'.I

0

dt

e- st dt

(1.2)

= [f (t)e -st]; + s 0

= - f (O) + sf(s),

for s > 0

Laplace transform of second derivative

[email protected]

11

physicsguide CSIR NET) GATE

@Sk J ahiruddin ) 2020

Integral Transformations

2

= s f (s) - sf (O) General nth Derivative

dt (0),

for s > 0

(1.3)

11 oo : oo : 31

General nth Derivative

nf n-If() n-2df( ) dn-Ij () =s -s O -s - 0 - ··· - - - 0 dt dtn-I for s > 0

(1.4)

1.3.4

Laplace transform of integral t 0

1.3.5

f( u)du = ~£ [! ] s

(1.5)

Laplace transformation of Heavyside function

if t < 1 if t >

0

H (t - c) =

[email protected]

12

C

C

physicsguide CSIR NET, GATE

@Sk J ahiruddin , 2020

Integral Transformations

Then £ { H (t - c) f (t)} =

e - cs £{ f

(t + c) };

_c-I {e-cs f (s) } = H (t - c) f (t - c)

(1.6)

11 oo : oo : 34 + c) };

£ { H (t - c)f (t)} = e- cs £{ f (t

(1.6)

£ - 1 { e- cs f( s) } = H (t - c)f (t - c)

Example: Write t he following function in terms of Heavyside function and find its Laplace t ransformation

f(t) =

2

if O < t < 1

lt2

if 1 < t < ~ 7r if t > ½1r

2

cost

Solution: In terms of heavyside function 1

f (t) = 2(1 - H (t - 1)) + t

2

2

+ (cos t) H

H (t - 1) - H

1 t - -7r 2

1 t - 1r 2

Indeed, 2(1 - H (t - 1)) gives

f (t)

for O < t < 1, and so

on We must write each term in f (t) in t he form j(t-a) H (ta) . T hus, 2(1 - H (t - 1)) remains as it is and gives t he t ransform 2 ( 1 - e- s) / s. Then

j [email protected]

@Sk J ahiruddin , 2020

~

1 2 - t H (t - 1) 2

13

physicsguide CSIR NET, GATE

Integral Transformations

00: 00: 37 Integral Transformations

1 2 - t H (t - 1)

2

2

1

=2

1

2

-(t - 1) + (t - 1) + 2 2 1

H (t - 1) = 1 2H -t 2

1

1

e + + s 3 s 2 2s

- s

1 t - - Jr 2

1 1 t - -Jr 2 2 1

7r

7r2

s3

2s

8s

_ +_2 +_

2

1 t - -Jr 2

7r

+ -2

1 t - -1r 2

H

e - 1rs/2

and

=

1 (cos t) H t - - Jr 2 1 2 - sin t - - Jr 2

1

H t-

1

- 1r

2

s

2

e - 1rs/2

+1

so t oget her

Y (J) = 2 - 2 e- s + s

s

1

s3 [email protected]

@Sk J ahiruddin , 2020

we know

7r

1

s3 7r2

+ -2s-2 + -8s 14

1

1

+ -s2 + -2s

e

e- 1rs/2 _

l s2

-s

+1

e - 1rs/2

physicsguide CSIR NET , GAT E

Integral Transformations

11 oo : oo : 39 @Sk J ahiruddin , 2020

Integral Transformations

we know

2 {J (t - a)H(t - a)} = e- as F (s) write f (t - a) = g(t ), hence again write f for g So you get

f (t)

=

g(t + a) and then

2{J(t) H (t - a)}= e- as 2{J(t + a )} Thus

1 2 -t H (t - 1) 2

as before. Similarly for 2 { ~ t 2 H (t - ~ 1r) } . Finally, by 2 {J (t) H (t - a) } = e-as2{J(t + a)} we get

2

costH

[email protected]

@Sk J ahiruddin, 2020

t-

1

1r

2

?

,...2

=

e-1TS /

=

e - 1rs/ 22

=

-e- 1rs/2

15

COS

1 t + -Jr 2

{ - sin t} 1

s2 + 1

physicsguide CSIR NET, GATE

Integral Transformations

11 oo : oo : 42 15

[email protected]

physicsguide CSIR NET, GATE

@Sk J ahiruddin , 2020

1.3.6

Int egral Transformations

Some more properties

£ [J (at)] =

1- s

-f a

a

for n

= 1, 2, 3, .. .

(1.8)

00

j(t) t 1.3. 7

(1. 7)

j (u)du

(1.9)

s

Examples:

Example: Find the Laplace transform of J(t) = t sin bt Solution: Using equation (1.8)

-

f (s)

= (-

d

d

b

1) ds 2 [sin bt] = - ds

32

+ b2

2bs

for s > 0 Example: (1r/S) l/2 j [email protected]

1 2

Prove L [t 1 ]

= ~(

16

112 3 1r / S )

and L [t - 112 ]

=

physicsguide CSIR NET, GATE

11 oo : oo : 44 Example: (1r I S )l/2

1 2

Prove L [t 1 ]

j [email protected]

~

=

112 3 ( 1r / S )

16

and L [t- 112 ]

=

physicsguide CSIR NET, GATE

@Sk J ahiruddin , 2020

Integral Transformations

Solution: We know t he standard integral 00

exp (-x

2

1

dx

)

0

= \in 2

2

Put x = ts in that integral. Hence 2xdx dx = sdt/ (2\l'st), we get OO

sdt and

Is In e- st _V _ t -l f2dt = _v_ 0

11

2

0

2 00



2

t - l/2 e- stdt

t - 1/2

7r

=

s

0

Integrating the LHS of t his result by parts gives

e-st2t1; 2

00

00

0

7r

st (-s) e- 2t 112 dt =

s

0

The first term vanishes at both limits, and the second is a multiple of the required Laplace transform of t 112 . Hence,

2

00

tl/2 0

1

7r

2s

s

e- sttl/ 2dt = -

Example: Use the propert ies of Laplace transforms to prove the following without evaluating any Laplace integrals

11 oo:oo:47 e 0

dt=2s

s

Example: Use t he properties of Laplace transforms to prove the following without evaluating any Laplace integrals [email protected]

17

physicsguide CSIR NET , GATE

@Sk J a hiruddin, 2020

Integral Transformations

explicitly:

.z [t5/2]

= 1/fos- 7/2 (b) .Z [(sinh at) / t] = ~ In [(s + a)/ (s - a)) , s > a 1 2 2 2 2 2 (c) .Z[ sinh at cos bt] = a (s - a + b ) [ ( s - a) + b J-

(a)

[(s + a) 2 + b2] - 1 Solution: (a) We use the general result for Laplace transforms that for n

=

l, 2, 3, . ..

If we take n = 2, then f (t) becomes t 112 , for which we found t he Laplace t ransform in t he previous example that 1 2

L [t 1 ]

=

3 112 ½(1f / S ) .

_z t5/2 = _z y0r 2

3 2

So Now t2tlf2

= (- 1)2 d2

ds 2

__5 s -1/ 2 = _15_y0r_1f s-7/2 2

8

(b) Here we apply a second general result for Laplace transforms which states that

11 oo : oo : 49 (b) Here we apply a second general result for Laplace t ransforms which states that 00

J(t) t

f(u )du s

[email protected]

18

physicsguide CSIR NET , GATE

Integral Transformations

@Sk J a hiruddin , 2020

provided limt➔o[f (t )/t] exist s, which it does in t his case. 00

sinh (at)

t

s

00

1 2

=

a 2 2du, u - a 1

u-a s+ a s- a '

s

1 - ln 2

u> al 1

du

u+a s > la

(c) T he t ranslation prop erty of Laplace t ransfo rms can be used here t o deal with the sinh(at) factor , as it can be expressed in terms of exp onent ial funct ions:

.

2[s1nh (at) cos(bt)]

=2

1

eat

2

cos(bt) - 2

s-a l s+a - 2 ( s - a) 2 + b2 2 ( s + a)2 + b2 l

1

(s2 -

a 2 ) 2a + 2ab2

2 [(s - a)2 + b2 ] [(s + a)2 2 2 2 a (s - a + b )

+ b2 ]

[(s - a) 2 + b2] [(s + a) 2 + b2]

11 oo : oo : s2 1 2 [(s - a) 2 + b2 ] [(s + a) 2 + b2 ] 2 2 2 a (s - a + b )

[(s - a) 2 + b2 ] [(s + a) 2 + b2 ] The result is valid for s > a

j [email protected]

19

physicsguide CSIR NET 1 GATE

@Sk J ahiruddin, 2020

1.4 1.4.1

Integral Transformations

Convolution Convolution of two functions

Convolut ion of t he functions f * g is defined by

f

and g which is written as

X

f(x) * g(x) =

f(t)g(x - t)dt

(1.10)

0

Example Find convolution of 1/ vt and t 2 Solution: Here 1

f (t) = -Ji so

1

f(x) = .,fi H PnrP

and

and

g(t) = t

2

g(t - x) = (t - x)

2

11 oo : oo : s4 f (t ) = ,Ji so

J (x)

=

1

Jx

g(t)

and

9(t - x)

and

=

=

t

2

(t - x)

2

Hence

jahir@physicsguide. in

20

physicsguide CSIR NET , GATE

@Sk J a hiruddin , 2020

1

,./i * t

2

=

Integral Transformations

f (t) * g(t)

= t

x-

1 2 /

[t

2

+x

2

-

2tx

-

2tx 112 + x 312 dx

]

dx

x=O t

t 2 x - 1l 2 x=O

=

2 2 t t22x 1; 2 - 2t-x3/2 + - x 5/ 2 3 5 x=O

= 2t2 . tl/2 -

it . t3/2 + 2 t5/2 = 3

5

16 t5/2

15

You can prove the following properties for convolution

f*9 = 9 * f

f * (91 + 92) = f (j * 9) * V = j

* 91 + f * 92

* (9 * V)

(1.11)

11 oo : oo : s1 f*9 ==9*f

f * (91 + 92) == f * 91 + f * 9 2 (j * 9) * V == f * (9 * V)

(1.11)

f*O==O*f==O 1.4.2

Convolution theorem for Laplace transformation

If .sf{f(x) } == F (s) and .sf{9 (x) } == G(s), then

.sf{f (x) * 9(x) } == .sf{f(x)}.sf{g (x) } == F (s)G(s) (1.12) j [email protected]

21

@Sk J ahiruddin , 2020

physicsguide CSIR NET I GATE

Integral Transformations

If t he functions and 9( s) t hen

f

and 9 have Laplace transforms

f (s)

t

f (u)9(t - u)du == f (s)g(s)

(1.13)

0

where the integral in t he brackets on t he LHS is t he convolution of f and 9 denoted by f * 9. The convolution defined above is commutative, i.e. f * g == g * f, and is associative and distributive. We also see t hat t 1

.sf- [J(s)g(s)] ==

f (u)g(t - u)du == f * g 0

From the definition of Laplace Transformation

11 oo : 01 : oo t 1

2 - [J(s)g(s)]

f (u)g(t - u)du = f * g

= 0

From the definition of Laplace Transformation 00

J(s)g(s) =

00

e-suJ(u)du 0

e- stg(v) dv

0 00

00

dve-s(u+'u)J (u )g(V)

du 0

0

Now letting u + v = t changes t he limits on the integrals, with t he result that 00

J(s)g(s) =

00

dtg( t - u )e- st

duf (u) 0

u

[email protected]

22

physicsguide CSIR NET ) GATE

@Sk J ahiruddin ) 2020

Integral Transformations

Now, look at the picture below. The shaded area of integration may be considered as t he sum of vertical strips. However, we may inst ead integrate over t his area by summing over horizontal strips as shown in figure . Then the integral can be written as I

I

I = II

, ll

I

I

/ / / / /

/ / /

/ /

/ / /

~ II

ll

11 oo : 01

: 02

ll

(h)

t

J(s)g(s) =

ll

00

dtg(t - u)e- st

duf (u) 0

0

t

00

dte- st 0

f (u)g(t - u)du 0

t

J(u)g(t - u)du 0

[email protected]

@Sk J ahiruddin, 2020

1.5

23

physicsguide CSIR NET, GATE

Integral Transformations

Inverse Laplace transformation

Example : Find inverse Laplace transformation of s+3

f (s) =

s(s + 1) -

3

Breaking into partial fraction f(s) = - 2 s - -. Hence J(t) = 3 - 2e- t s+ l

Solution:

Example: Find

11 oo : 01 : os .

2 - -. Hen ce s+ l

rea 1ng into part1a

ract1on

s

f (t) = 3 - 2e- t

Example: Find

3s - 137

L-l s

2

+ 2s + 401

Solution:

f

3(s + 1) - 140 = £-l (s + 1) 2 + 400 s+ l = 3_2-l

20

_7_2-l

(s + 1)2 + 20 2

Hen ce using first shifting t heorem we get

f (t) = e- t(3 cos 20t -

j [email protected]

24

7 sin 20t)

physicsguide CSIR NET, GATE

Integral Transformations

@Sk J ahiruddin , 2020

Example: Find the inverse t ransformation of

se-4s

(3s

+ 2)(s -

2)

Solution : We rewrite t he function as e - 4s (3s

s

+ 2)(s - 2) = e-4s F (s)

11 oo :01 : oa Solution : We rewrite t he function as s

- 4s _ _ _ _ _

e

(3s

+ 2) (s - 2) = e

- 4s p () 8

Now write F (s) into partial fraction

A

B

F (s) = 3s+ 2 + s- 2 s(A + 3B) + (2B - 2A) (3s + 2)(s - 2)

A(s - 2) + B (3s + 2) (3s + 2)(s - 2)

= 1/ 4

Hence we find A= B Now

F (s) =

-1 4

3(s+~ )

+

-1 4

s- 2

Inverse Laplace t ransformation of F (s) is

J(t) =

1

2t

- e - ""f

12

25

[email protected]

1

+ -e

2t

4

physicsguide CSIR NET , GATE

@Sk J ahiruddin , 2020

Integral Transformations

The main problem was

Y (s) = e- 4s F (s) So now using t he property of LT of hevisid e function we get

](s) = H (t - 4)f(t - 4) F,y::. rnnlP~ Finrl t h P in, rPr ~P

T,.: :i nl .:::irP tr.:::in~fn r m.:::i t.inn nf

11 oo : 01

: 11

So now using the property of LT of heviside function we get

f (s) = H(t -

4)f(t - 4)

Example: Find t he inverse Laplace transformation of 1 d ----an

s (s 2 + w 2 )

1

s 2 (s 2 + w 2 )

Solution: WE have from t he basic definitions 1 sin wt 2-1 s2 +w2

w

So using t he property, equation ( 1. 9)

2 -1

1

t

s (s 2 + w 2 )

o

sin wt --dT

=

W

1 W

(1 cos wt) 2

Using this result an applying the property again we get 1 t

1 w2

(1 - cos wT)dT = 0

Sln WT

w2

w3

@Sk J ahiruddin , 2020

T

Sln WT

w2

w3

t 0



t [email protected]



26

physicsguide CSIR NET, GATE

Integral Transformations

We could have got t he result by breaking t he funct ion in partial fractions also.

1.6

Solving Differential equation by Laplace

11 oo : 01

: 13

We could have got t he result by breaking t he function in part ial fractions also.

1.6

Solving Differential equation by Laplace transformation

Example: Solve using t he concepts of Laplace t ransformation y1 - 5y

=

y(O) = 0

e5x;

Solution: Taking t he Laplace transform of both sides of this differential equation we find t hat

T hen we obtain

[sY(s) - O] - 5Y(s) =

1

1 _ 8

Y( s) = (s - 5)2

5

Finally, taking t he inverse transform of Y ( s), we obtain

y(x)

1

2 - {Y(s)} = 2 -

=

j [email protected]

1

1

(s - 5) 2

27

=

xe5x

physicsguide CSIR NET 1 GATE

@Sk J ahiruddin, 2020

Integral Transformations

Example: Solve using the concepts of Laplace transformation y

II _

y - t, _

y(O)

=

1,

1

11 (0)

=

1

11 oo : 01

: 1s

@Sk J ahiruddin, 2020

Integral Transformations

Example : Solve using the concepts of Laplace transformation

y'' - y = t,

y(O) = l ,

y' (0) = 1

Solution: Taking Laplace transformation both sides we get, [ with Y = 2(y)] 2

s Y - sy (0) - y' (0) - Y ~~>

= 1/ s

2

2

(s - l)Y =s+ l + l /s

,

2

Y =s+ l + 1 s2 - l s 2 ( s 2 - 1) Simplification of the first fraction and an expansion of t he last fraction gives Y =

1

s- l

+

s2

1

1

- 1

s2

we obtain

y(t) = 2 - 1(Y )

=2-1

1

+2-1

s-l = et + sinh t - t_

1

s2

l

-

-2-1

1 s2

Example : Solve the set of equations

y' - 2y + z = 0 z' -y - 2z = 0 [email protected]

@Sk J ahiruddin, 2020

28

physicsguide CSIR NET, GATE

Integral Transformations

By using the technique of Laplace transformation. Given

11 oo :01

:1

a

@Sk J ahiruddin , 2020

Integral Transformations

By using the t echnique of Laplace transformation. Given t he init ial conditions y 0 = 1, z 0 = 0

Solution:

Taking Laplace transformation of both the equations and calling L(z) = Z and L (y) = Y pY - Yo - 2Y + Z

=0 pZ - zo - Y - 2Z = 0 Substituting the initial condit ions

(p - 2)Y + Z = 1 Y - (p - 2)Z = 0 We get by simply solving t hese two equations by algebric method

p-2

Y = ---· (p - 2) 2 + 1 ' Hence y

j [email protected]

@Sk J ahiruddin, 2020

2

2

z = e t sin t

= e t cost;

29

physicsguide CSIR NET, GATE

Integral Transformations

11 oo : 01 [email protected]

: 20

29

physicsguide CSIR NET, GATE

@Sk J ahiruddin , 2020

1. 7

Integral Transformations

Exercises

1.1. Find

(a)£ { e- 2x sin 5x}

(b )£ {x cos V7x}

1.2.

sin 3x

(b)£ x1;2

(a)£ { e- xx cos 2x}

X X

(d)£

sinh 2tdt 0

1.3. Find X

0

l 4 -e- t sin 3tdt t

1.4. Find

s

(a) £-1 s

(c) £-1

2

+6

s+l s2

-

9

5s

(b) £-1 (d)

.z-1

s2

-

2s + 9

1.5. Find

(a)

.z-1

(c)

.z-1

s

s 2 - 3s

[email protected]

(b)

.z-1

s+4 s 2 + 4s + 8

+4 30

physicsguide CSIR NET, GATE

11 oo : 01

: 23

s+ 2 (c) _z- 1 s2 - 3s + 4 j [email protected]

physicsguide CSIR NET 1 GATE

30

@Sk J ahiruddin, 2020

Integral Transformations

1.6. Use partial function to decompose

1

(a) (s + l ) (s 2 + 1) s+3 (c) (s - 2) (s + 1) 1. 7. Find

(a).z-1 (c) _z- 1

s+3 (s-2) (s + l ) 1

(b) _z- 1

1

1.8. Find J(x) * g(x ) when (a) J(x) = e3x and g(x) = e2x

(b) J(x)

=

x and g(x)

=

x2

1 6 1 1 1.9. Find (a) .z- , (b) .zs 2 - 5s + 6 s2 - 1 convolut ions

by

1.10.

Find .Z{g(x) } if

0 x< 4 (x - 4)2 X > 4 '

(a) g(x) =

(b) g(x) =

0

x



X

2

X

4

11 oo : 01 : 2s (x - 4) 2 (b) g(x)

0 2 x

=

X X

31

[email protected]

X

>4

4 physicsguide CSIR NET, GATE

@Sk J ahiruddin, 2020

Integral Transformations

1.11. Solve t h e initial value problem by using t he con cept of Laplace transformation 9 =, 0 y+y+y II

1

y(O) = 0 .16,

I

y (0)

=0

1.12. Solve the following differential equation by using t he con cept of Laplace transformation

y

I

+ y = s1n x; •

y(O ) = 1

1.13. Solve by using t he concept of Laplace transformation 8 sin t

+ 9y =

yll

if O < t < if t >

0

1r

1r

1

The initial conditions are y(O) = 0, y (0) = 4

1.14. Solve by using the con cept of Laplace transformation W

y

+ y = Sln X

I

1

.

-

z = ex

1

z +w+y= l w(O) = 0, y(O) with t he boundary condition

=

1,

z (O)

=

1

11 oo :01 : 2a z'+w+y= l w(O) = 0, y(O)

=

1,

z (O)

=

1

=

1,

z(O)

=

1

with the boundary condition

w(O ) = 0,

y(O)

[email protected]

32

physicsguide CSIR NET I GATE

Integral Transformations

@Sk J ahiruddin , 2020

1.15.

w'' - y + 2z = 3e-x -2w' + 2y' + z = 0 2w' - 2y + z' + 2z'' = 0

with t he boundary condition

w (0)

=

1,

w' (0)

=

1,

y ( 0) = 2,

z (0)

=

2,

z' (0)

= -

2

11 oo : 01

[email protected]

33

: 31

physicsguide CSIR NET ) GATE

Integral Transformations

@Sk J ahiruddin ) 2020

1. 7.1

Ans keys

1.2. (a)

(d)

2

(s+1) -4 [(s+1) 2 +4]2 '

(b)

105 r,;; -9/2 16 V 7r S '

(c) =

Z!: 2 -

arctan §..3,

s(s; - 4)

1.3.

2(s - 4) 2

1

v18

s 3 (s - 4) 2 arctan 3 + (s - 4) (s 2 + 9)

1

1r

-

1.4. (a) cos v'6x, (b) ~xsin x, (c) cosh 3x+~sinh 3x, (d) ex sin y'8x

1.5. (a) e2xcos 3x+je 2xsin3x, (b) e- 2xcos 2x+e- 2xsin2x, (c) e(3/ 2)x cos {; x + v'7e(3/ 2)x sin {; x

11 oo : 01

: 34

2 2 2 2 1.5. ( a) e x cos 3x+je x sin 3x, (b) e- x cos 2x+e- x sin 2x, (c) e( 3/ 2)x cos {; x + ,/7e(3/ 2)x sin x

f

1.7. (a) ~e 2x - ~e-x, (b) ½e-x - ½cosx + ~ sin x, (c) 2x cos 2x + 1 e- 2x sin 2x - .i. cos x + 1sin x + -1.e65 65 65 130 1.8. (a) e3x - e2x' (b) 112X4 1.9. (a) e3x - e2x [email protected]

,

(b) 3ex - 3e- x 34

physicsguide CSIR NET, GATE

Integral Transformations

@Sk J ahiruddin , 2020

1.11. e- 0 -5t(0.16 cos 2.96t + 0.027 sin 2.96t) 1.12. ~e-x - ½cosx + ½sin x 1.13. sin t -1 sin 3t + [sin (t - 1r) - ; sin(3(t - 1r))] u(t-

1r) +; sin3t 1.14. 1 - ex, ex + sin x, cos x 1.15. w(x) = ex

y(x) ==ex + e- x z(x)

=

2e- x

11 oo : 01

j [email protected]

: 36

35

physicsguide CSIR NET, GATE

Integral Transformations

@Sk J ahiruddin , 2020

1. 7.2

Solutions

Solution: 1.1. (a) Setf (x) =sin5x

F (s)

=

.2'{f(x) }

=

.2'{sin5x} =

5 s 2 + 25

t hen apply shifting property (1.1)

(b) Set f (x) = cos v!7x s s2 + 7

t hen apply equation (1.8) you get

11 oo : 01 (b) Set f (x)

: 39

= cos v!7x

F(s)=2{cosv'7x}=

s

3

../7 s2 + ( 7)2

s

2

+7

t hen apply equation (1.8) you get

s

d -2' { x cos v'7x} = - ds

s

2

+7

Solution: 1.2. (a) Let f(x) = xcos2x. You can easily see t hat s2 - 4 F(s) - - - - (s2+ 4)2 Then use shift ing property

[email protected]

physicsguide CSIR NET, GATE

36

@Sk J ahiruddin, 2020

Integral Transformations

(b) Define f (x) =

VX. Then

7 2

x1 =x

3

VX = x

3

f (x)

You can easily see that

1 2 3 F( s) = .2'{f(x) } = -2'{ VX} = ~s- / 2 t hen apply equation (1.8) you get

-2' {x3VX} = (c) Taking

f (x) = sin 3x,

F (s) •

d3 (- 1)3-3 ds

, .

3

s

2

+9 /-a

r.\

l

-~s- 3/2

= 105 ~s- 9/2

2

16

then or

F (t ) -

3

t

2

+9

11 oo : 01 (c) Taking J(x)

F (s) -

: 42

= sin 3x, then 3 s2

F (t ) -

or

+9

3

t2 + 9

Then using equation (1.9) we get 00

sin3x X

s

3

t2

+9

dt = lim

R-too

lim arctan

R-too

= lim

R-too

t

R

3

8

arctan

s = - - arctan2 3

R

3

-

3

R

s

t2 + 9 dt

s arctan 3

7r

(d) Taking J (t ) = sinh 2t, t he Laplace transformation is

[email protected]

physicsguide CSIR NET, GAT E

37

@Sk J ahiruddin , 2020

Integral Transformations

Then using Laplace t ransformation property of integral, equat ion ( 1.5), we get X

sinh 2tdt 0

1 s

2 s2

-

2 4

s (s 2

-

4)

Solution: 1.3. We have already proved

.2 sin3x X

Use shifting theorem for a

7r

=- 2

s arctan -

3

= - 4 in equation (1.1), on t his

11 oo : 01

: 44

Solution: 1.3. We have already proved

sin3x

2

1r

=- 2

X

Use shifting theorem for a result to get 1

-

e- 4 x

s arctan 3

= - 4 in equation (1.1) , on t his 1r

=- -

sin3x

2

X

s +4 arct an - 3

Now we also know X

j(t)dt

=

0

1 - F (s) s

So X

1

1 s+4 - - - arct an - 2s s 3 1r

4

-e- t sin 3tdt

t

0

Now we know

j [email protected]

38

physicsguide CSIR NET 1 GAT E

@ Sk J ahiruddin, 2020

Integral Transformations

Use the above formula in our Laplace transformation for n = 1, we get X

,2 X 0

1

1r

2

2s

s

2

l 4 -e - t sin 3tdt t s+4

arctan

3

3

+ s [9 + (s + 4)2]

Finally using the shifting property again for a= 4 -,

11 oo : 01 n

1

2s2

s2

: 46

s+4

arctan

3

3

+ s [9 + (s + 4)2·]

Finally using the shifting property again for a

n 1 s - - - - - - - arctan 2 (s - 4) 2 ( s - 4) 2 3

=4

3 + - - - -2 - (s - 4) (s + 9)

Solution: 1.4. (a) s s2

+6

Hence s

g -1 s2

+6

= cos .J6x

(b) 2 {sinax} =

~>

. d 2 { x sin ax } = - ds

[email protected]

a

2

2

s +a

2sa

a

82

+ a2

39

physicsguide CSIR NET, GATE

Integral Transformations

@Sk J ahiruddin, 2020

Hence

g-1

5s

=2-1

~(2s) (s2

2s

+ 1)2

5 . = xs1n x 2

11 oo : 01

: 49

2s (s2 + 1)2

(c)

2 - 1 s+ l s2

-

9

1

s

=2- 1 +21 2 s

9

-

= cosh 3x + 2

-

9

3 ) _ ( 3 82 3 2 3 ) _ ( 82 32 1

- 1

1 = cosh 3x + 3 2

s2

-1

1

= cosh 3x + sinh 3x

3

(d) We see

2 2 2 2 s - 2s + 9 = (s - 2s + 1) + (9 - 1) = (s - 1) + ( v'8) Hence 1

1

1

Jg

s 2 - 2s + 9

(s - 1)2 + ( vf8)2

Jg

(s- 1)2+(vf8)2

40

j [email protected]

@Sk J ahiruddin, 2020

physicsguide CSIR NET I GATE

Integral Transformations

So using first shift ing t heorem

2-1

s2

-

1 2s + 9

= 1 2-1

J8

Jg

11 oo : 01 : s2 So using first shifting theorem

2-1 s2 -

1 2s + 9

v'8

= 1 2-1

v'8

1 . = ,./8 ex sin v'Sx

Solution: 1.5. (a)

2 -l

= 2 _l

S

(s- 2)2 + 9

= 2 -1

(s-2)+2 (s-2) 2 +9

+2 -1

s- 2

(s- 2) 2 + 9 cos 3x + 2-

1

= e x cos 3x + 2 -

1

=

2 ex 2

(s- 2) 2 + 9

3

=

2 e x cos 3x + - 2 - 1 3

=

2 ex

cos 3x + ~ 3

(s- 2)2 +9 2 (s - 2) 2 + 9 2 3

2

2 ex

2

3 (s - 2) 2 + 32

sin 3x

(b) Completing the square in t he denominator, we have

[email protected]

41

physicsguide CSIR NET ) GATE

@Sk J ahiruddin ) 2020

Hence

s+ 4 s2 + 4s + 8

Integral Transformations

s+4

(s + 2)2 + (2) 2

11 oo : 01 : s4 @Sk J ahiruddin, 2020

Hence

Integral Transformations

s+4

s+4

s2 + 4s + 8 Now decompose the function as

s+4 s+2 2 - - - - = - - -2- - -2 + - - -2- - -2 s2 + 4s + 8 (s + 2) + (2) (s + 2) + (2) hence we get

s+4 £ -1 s 2 + 4s + 8

s+2 +£- 1

(s + 2) 2 + (2) 2

= e- 2x cos 2x + e- 2x sin 2x (c) Completing the square in t he denominator, we get 2

s -3s+4 =

2

s - 3s

9

9

4

4

3 s- 2

+ - + 4- -

vl7

2

+

2

So we have

s+2 s 2 - 3s

s+ 2

+4

We now rewrite the numerator as

3 7 s+2=s--+-= 2 2 [email protected]

@Sk J ahiruddin, 2020

3 s-2 42

vl7 + v'7 2 physicsguide CSIR NET, GATE

Integral Transformations

2

11 oo :01 : s1

@Sk J a hiruddin , 2020

Integral Transformations

so t hat 3 s-_ _ _ _2_ _

s+ 2 s 2 - 3s + 4

v17

2

(s-~ ) +

2

v'7

+ V7

2

v'7

2

(s-~) +

27

2

27

Then finally

.z-1

s+ 2 s 2 - 3s + 4

s--32 v'7

v'7

+ v7.z-1

2

2

v'7

2

=

2

ft + V7e(3/ 2)x Sln . -ftX

e( 3 / 2 )x COS - X

2

2

Solution: 1.6. (a) To t he linear s + l , we associate the fraction A/ (s + l ); whereas to the quadratic factor s 2 + 1, we associate t h e fraction ( B s + C) / (s 2 + 1) . We then set l A Bs + C - - - -2- - = - - + -2 - ( s + l ) ( s + 1) - s + l s +1 Clearing fractions, we obtain

or j a [email protected]

(c)Sk J a,h ir11ddin. 2020

43

physicsguide CSIR NET, GATE

Intee:ral Transforma,t ions

2

11 oo : 02 : oo or 43

j ahir@physicsguide. in

physicsguide CSIR NET I GATE

@Sk J a hiruddin , 2020

s

2

Integral Transformations

(0) + s(O) + 1 2

= s (A + B ) + s( B + C) +(A+ C) Equating coefficients of like powers of s, we conclude t hat A + B = 0, B + C = 0, and A + C = l . The solution of this set of equations is A = ~, B = and C = ½· 1 ·t t· t h 1 · t A B s+C Su b s t 1 u 1ng ese va ues 1n o (s+l)(s2+i ) -= s+l + 8 2+ 1 we obtain the part ial-fractions decomposition -

-½,

1

(s

+ l) (s 2 + 1)

1 2

- s

+l

+

_ls + l 2 s2

+1

2

(b) To t he quadratic factors s 2 + 1 and s 2 + 4s + 8, we associate the fractions (A s + B )/ ( s 2 + 1) and (Cs + 2 D )/ (s + 4s + 8) . We set A s+ B l ) ( s 2 + 4s + 8) - s 2 + l l

(s2 +

+

Cs+ D s 2 + 4s + 8

and clear fr actions to obtain

or

[email protected]

44

physicsguide CSIR NET , GATE

11 oo : 02 : 02 or 2

3

s (0)+s (0)+s(O)+ 1

2

3

s (A+C)+s ( 4A+ B + D )+s(8A+4B+C )

[email protected]

44

physicsguide CSIR NET ) GATE

@Sk J ahiruddin ) 2020

Integral Transformations

Equating coefficients of like powers of s, we obtain A + C =

0, 4A + B + D = 0, 8A + 4B + C = 0, and 8B + D = 1 The solution of this set of equation is

A=-4

B= 7

C= 4

D= 9

65

65

65

65

Therefore 4

7

4

9

-65S + 65 + 65 8 + 65 (s2 + 1) ( s2 + 4s + 8) s2 + 1 s 2 + 4s + 8 1

(c) To the linear factors s - 2 and s + 1, we associate respectively the fractions A/(s - 2) and B /(s + 1) . We set

s +3 A B = + (s-2)(s+l)- s-2 s+l and, upon clearing fractions, obtain

s+3

A (s + 1) + B(s - 2) = s(A + B ) + A - 2B

Hence

A - 2B = 3·

'

A+B= l

We immediately obtain A= 5/3 and B

s+3 _ 5/3 (s- 2)(s+ l )-s- 2

= -2/3. Thus 2/3 s+ l

11 oo : 02 : os A - 2B = 3·,

A+B= l

We immediately obtain A= 5/3 and B

s+3 _ 5/ 3 (s- 2)(s+ l ) - s-2 [email protected]

= -2/3. Thus 2/3 s+ l

physicsguide CSIR NET, GATE

45

@Sk J ahiruddin, 2020

Integral Transformations

Solution: 1.7. (a) Using results of problem 1.6. (c) we

get

s+3 (s - 2)(s + 1)

1

1

s-2

s+ l

2 -x 5 2x = -e - -e 3 3

(b) and noting t hat

--s 2 + 2 2 s +1 1

1

l

s

- -

2

s

2

1

+2

+1

1 s2

+1

And t hen using results of problem 1.6. (a) we get 1

2 -1 (s

+ 1) (s 2 + 1)

= ~2-1 2 1

= -e 2

1

s - x

1

+1

- - COS X 2

- ~2-1 2

s s

2

+1

1 +-2-1 2

1 s

1 .

+ - SID X 2

(c) From t he results of problem 1.6. (b) we get

2 -1

1 (s 2 + 1) ( s 2 + 4s + 8)

2

+1

11 oo : 02 : 01 (c) From t he results of problem 1.6. (b) we get 1

,2- 1

(s 2

+ l) (s 2 + 4s + 8) 4

4

7

= ,2- 1 -65s + 65 + ,2-1 s

2

65 8

+l

s 46

j [email protected]

2

9

+ 65

+ 4s + 8

physicsguide CSIR NET, GATE

Integral Transformations

@Sk J ahiruddin , 2020

The first term can b e evaluated easily if we note t hat 4 65

7

s

2 -+ s +1

1

s

65

2

+1

To evaluate the second inverse transforms , we must first complet e the square in the denominator, s 2 2 2) + (2) , and t hen note that 4

s2

9

+ 65

4

s+2

+ 4s + 8

65

(s + 2)2 + (2) 2

65

8

2

+ 4s + 8 = (s + 2

Therefore 1

,2-1

(s2

+ 1) (s 2 + 4s + 8)

4 s 7 = - - . 2 -1 - + -.2-1 2 65 s +1 65

1

s

2

+l

s+2 1 2 - - - - - + -.2-1 65 (s + 2) 2 + (2) 2 130 (s + 2) 2 + (2) 2 4 7 4 2 1 2 =cos x + sin x + e- x cos 2x + e- x sin 2x 4 + _ _z-1

65

65

..

,....

65

I

\

TT

130

.

\

11 oo : 02 : 1o + - 2-1 - - + 2-1 2 2 65 (s + 2) + (2) 130 4 7 4 2 e- x cos 2x sin x + cos x + 65 65 65

=-

Solution: 1.8. (a) Here f (t )

47

[email protected]

(s + 2) 2 + (2) 2

1 + e- 2x sin 2x 130

e3t, g(x - t)

=

=

e 2 (x- t),

physicsguide CSIR NET, GATE

@Sk J ahiruddin, 2020

Integral Transformations

and X

f (x) * g(x)

X

e3te2(x- t) dt

= 0

0 X

=

e3te2xe- 2tdt

=

e2x

et dt

=

e2x

[et]!=; = e2x (ex -

1)

0

= e3x _ e2x (b) Here f (t ) = t and g(x - t ) = (x - t) 2 = x 2 T hus,

-

2xt

X

f (x ) * g (x) =

t (x

2

-

2xt

+t

2

)

dt

0 X

=x

2

X

tdt - 2x 0

=X

2

x2

2

2 t dt +

X

0

x3 x4 -2x-+3

4

Solution: 1.9. (a) Note t hat

1

1

1

1

t 3dt

+ t2 .

11 oo : 02 : 13 -

2

3

4

12

Solution: 1.9. (a) Note t h at

1

1 (s -3)(s- 2)

s 2 -5s+6

1

1

s -3 s- 2

Defining F(s) = 1/(s - 3) and G(s) = 1/(s - 2), we h ave from inverse Laplace transformation f(x ) = e3x and g(x ) = e2 x and using the result of problem 1.8. (a)

2 -1

s2 -

1

=

5s + 6

f (x) * g(x) 48

[email protected]

e3x * e2x = e3x - e2x

=

physicsguide CSIR NET, GATE

Integral Transformations

@Sk J ahiruddin , 2020

(b) Note that

2 -1

6 (s - l )(s + 1)

6 s2 -

1

1

1

(s - 1) (s + 1) Defining F(s) = 1/ (s - 1) and G(s ) = 1/ (s + 1), we h ave from inverse Laplace transformation f (x) = ex and g( x) = e- x Now

2 -l

~ 82

1

= 62'- 1{F(s)G(s)} = 6ex * e- x X

=

6

X

et e- (x- t)dt

e 2t dt

6e-x 0

0

= 6e-x

=

e2x - 1 2

= 3ex - 3e- x

Solution: 1.10. We know t hat, If F(s ) = 2{f(x)}, +'h e n

11 oo : 02 : 1 s = 6e-x

e2x - 1

= 3ex - 3e-x

2

Solution: 1.10. We know t hat, If F(s ) = 2{f(x) },

t hen

2{u(x - c)f(x - c)} =

e - cs F(s )

Conversely,

0 x c (a) If we define f( x) = x 2 , t hen g(x) can be given compactly as g(x ) = u(x -4)f(x -4) = u(x - 4)(x - 4) 2 . Then, 49

j [email protected]

physicsguide CSIR NET 1 GATE

Integral Transformations

@Sk J a hiruddin, 2020

noting that 2 { f (x) } = F( s) t heorem we conclude that

=

2/ s3 and using the above

2{g(x)} = 2 { u(x - 4) (x - 4)

2

2 = e- s s3 4

}

(b) We first determine a function f (x) such t hat f (x 4) = x 2 . Once t his has been done, g(x ) can be written as g(x) = u(x -4)f (x - 4) and then t he t heorem stated at t he beginning of t he solut ion can be applied. Now, f (x -4) = x 2 only if

f( x ) = f(x

+4-

4)

=

(x

+ 4)

2

=

x

2

+ 8x + 16



since, 2

1fi

11 oo :02 : 1a f(x)

f(x

=

+4 -

4)

=

(x

+ 4)

2

x

=

2

+ 8x + 16



since,

2 {f (x)}

=

2 {x

2

}

+ 82{x} + 162{1} =

2

16

8

+ + s3 s~ s ?

it follows that

2{g(x)}

=

2{u(x - 4)f(x - 4)}

= e-

4

2

s

8

16

-+-+s3 s2 s

Solution: 1.11. Taking Laplace transformation both side 2

s Y - 0.16s + sY - 0.16 + 9Y = 0 ~~>

(

s

2

+ s + 9) Y = 0.16 (s + l )

[email protected]

50

physicsguide CSIR NET, GATE

Integral Transformations

@Sk J ahiruddin , 2020

The solution is t hen

y _ O.l6 (s + 1) _ 0.16 (s + ½) + 0.08 - s2 + s + 9 (s + ~) 2 + 345 Using first shift ing theorem we get 1

y(t ) = 2 - (Y)

= e- t/

2

0.16 cos

35 t 4

0.08

.

+ ~ v'35 Sill

35 t 4

= e- o. 5t(O.l6 cos 2.96t + 0.027 sin 2.96t) Solution: 1.12. Taking the Laplace t ransform of both sides of the differential equation, we obtain

11 oo : 02 : 20 = e- o. 5t(O.l6 cos 2.96t + 0.027 sin 2.96t) Solution: 1.12. Taking the Laplace transform of both sides of the differential equation, we obtain

2 {y'}+2{y} = 2 {sin x}

or

[sY (s)- l ]+Y( s) =

1

s2 + 1

Solving for Y ( s), we find 1

1

Y (s) = - - + (s + 1) (s 2 + 1) s + l Taking the inverse Laplace transform , and using the result

j [email protected]

51

physicsguide CSIR NET I GATE

Integral Transformations

@Sk J ahiruddin , 2020

of Problem 1. 7. (b) we obtain

y(x) = 2- 1{Y(s)} 1

1

= 2 -1 - - - - + 2 -1 ( s + 1) (s 2 + 1) s+ 1 - x - -1 COS X -e

2

3 = -e 2

2

x

1 - - COS X 2

l

+ -1 Sln . X + e-x

+

2 1 . - Sin X 2

Solution: 1.13. The RHS (P.I part) of t he differential

11 oo : 02 : 23 -

2

2

3 -x 1 = -e - - COS X 2 2

In X

1 .

+ - Sln X 2

Solution: 1.13. The RHS (P .I part) of t he different ial equation can be written as 8[sint - u(t - 1r) sint] where

u( t - 1r) = H (t - 1r) The Heaviside function The subsidiary equation is

s Y - 0 · s - 4 + 9Y = 8£ [sin t - u(t - 1r) sin t] 2

= 8£ [sin t + u(t - 1r) sin(t - 1r)] 1

= 8 (1 + e- 7rS) - S2

52

[email protected]

+1

physicsguide CSIR NET) GATE

@Sk J ahiruddin ) 2020

Integral Transformations

simplified (s

2

+ 9) Y = 8 ( 1 + e-1rs)

1

s

2

+1

+4

The solution of t his subsidiary equation is 8 (1

+ e- 7rS)

4

Y =---+2 2 2 ( s + 9) (s + 1) s + 9 Apply partial fraction reduction

11

oo : 02 : 2s

The solution of t his subsidiary equation is 8 (1 + e-7rS)

4

Y = 2- - - + ( s + 9) ( s 2 + 1) s 2 + 9 Apply part ial fraction reduction

8 (s

2

1

+ 9) (s + 1) 2

s

2

1

+1

1 s2

+9

1

+1 -

since the inverse transform of 4/ tain y

s

2

4

+ 9 + s2 + 9 ( s 2 + 9) is i sin 3t, s2

we ob-

= £-l (Y)

=

+

. 1 . 3 Sln t Sln t

3

sin (t - 1r) -

1

3

sin (3 (t - 1r))

U (t

- 1I") +

4 .

3

Sln 3t

Hence, if O < t < 1r, then

y(t ) = sin t + sin 3t [email protected]

53

physicsguide CSIR NET, GATE

@Sk J ahiruddin , 2020

Integral Transformations

and if t > 1r, then

y(t) =

4

3

sin 3t

Solution: 1.14. DenoteY{w(x) }, Y {y(x) }, and Y {z(x) } by W ( s), Y (s), and Z ( s), resp ect ively. T hen , taking Laplace t ransforms of all t hree different ial equations, we have

11 oo : 02 : 2a =-

3

Sln

Solution: 1.14. DenoteY{w(x) }, Y {y(x) }, andY{ z(x) } by W (s), Y (s), and Z (s), respectively. Then, taking Laplace t ransforms of all t hree different ial equations, we have

[sW(s) - O] [sY (s) - 1] - Z(s)

+ Y (s) = -

1

+ sW(s) + Y(s) 82 1

=

l

[sZ(s) - 1] + W (s)

=

+ 82 1

s sY (s) - Z(s) = s- l

or

s-l

1

1

+ Y(s ) = -W(s) + Y( s) + sZ(s) s

s+ l s The solut ion to this last system of simultaneous linear equations is 2

- 1

W( s) - - - s(s - 1)

8)

y(

s +s = -(s---1-)-(s2_ +_ 1)

Z(s) - -

s2

8 -

+1

Using the method of partial fractions and t hen taking in-

j [email protected]

54

physicsguide CSIR NET, GATE

Integral Transformations

@Sk J ahiruddin , 2020

verse transforms, we obtain

w(x)

=

y -1{W (s) } = y -1

y(x) = y -1 {Y(s)} = y -1 [

_1 - _1_ s s- l l ..

+

l)

1_

11 oo : 02 : 31 verse t ransforms, we obtain

w(x)

z(x)

=

=

1

2 - {W(s) } =

1

2 - {Z(s)}

=

1 1 2 - - - - - = 1 - ex s s -1 1 1 --+ = ex +sin x s - 1 s2 + 1 1

2-

s

1

s

2

= cosx

+1

Solution: 1.15. Taking Laplace t ransforms of all t hree

differential equations, we find that 2

[s W (s) - s - 1] - Y (s)

3

+ 2Z(s) = - +-

1

8

-2[sW (s) - 1) + 2[sY(s) - 2) + Z(s) = 0 2[sW(s) - 1) - 2Y (s) + [sZ(s) - 2)

+ 2 [s Z(s) - 2s + 2] = 0 2

s+

+ 2sY(s) + Z(s ) = 2 2 2sW (s) - 2Y(s) + (2s + s) Z(s) = 4s - 2sW(s)

The solution to t his system is

28 1 W(s) Y(s) s- 1 (s - l )(s + 1)

2 Z(s) = -s-+-1

Hence,

w(x)

=

ex y(x)

[email protected]

=

2 -

1

1

1

s- 1

s+ l

--+-55

@Sk J ahirudd in, 2020

physicsguide CSIR NET, GATE

Integral Transformations

z(x) = 2e-x

00: 02: 33 Integral Transformations

@Sk J ahiruddin , 2020

z(x) = 2e-x

[email protected]

@Sk J ahiruddin , 2020

56

physicsguide CSIR NET, GATE

Integral Transformations

11 oo : 02 : 36 [email protected]

56

physicsguide CSIR NET, GATE

@Sk J ahiruddin , 2020

Integral Transformations

Dirac Delta function

2 2.1

Definition

The 8 function is different from most functions encountered in the physical sciences but we will see that a rigorous mathematical definition exist s. The 8 function can b e visualised as a very sharp narrow pulse (in space, time, density, etc.) which produces an integrated effect having a definite magnitude. Basic definition of Delta function

f (t) 8(t - a) dt = f (a)

(2.1)

provided t he range of integration includes the point t = a; otherwise t he integral equals zero. This leads immediately t o two further useful results: b

8(t)dt = 1

for all a,b > 0

(2.2)

- a

and

8(t - a)dt = 1 j [email protected]

57

(2.3)

physicsguide CSIR NET, GATE

T

,

,

,..,.,

r

,•

11 oo : 02 : 38 [email protected]

57

physicsguide CSIR NET, GATE

@Sk J ahiruddin , 2020

Integral Transformations

provided t he range of integration includes t = a We need to know b

cp (t)fJ (t - to) dt =

(to) , a < to < b otherwise

0,

a

(2.4)

And

u(x - a) =

1,

X

0,

X

>a 0

(2.7)

0

2.2.3

Fourier Transform of a 5 Function

g(a)

1

00

5(x - a) e- iaxdx

= --

vt2i

.

- oo

=

1

.

--e- iaa

vt2i

(2.8)

Then the inverse definit ion

5(x - a) =

2.2.4

1

00

vf2i 21r

(2.9) -00

Delta function in three dimensional Carte•

s1an 00

00

00

if>(x, y, z)5 (x - xo) 5 (y - Yo) 5 (z - zo) dxdydz -ex::

-oo

-oo

= if> (xo, Yo, zo ) (2.10)

11 oo : 02 : 44 = (xo, Yo , zo ) (2.10)

[email protected]

59

@Sk J ahiruddin , 2020

2.2.5

physicsguide CSIR NET, GATE

Integral Transformations

Delta function in three dimensional spherical polar

b(r - ro) = b(r -ro)b(0-_ 0o) b(cf> - o) r 2 Sln 0

(2.lla)

f (r, 0, cp) 6 (r - ro) dT = f (ro, 0o , o)

(2.llb)

6 (r - ro) = 6 (r - ro) 6 (cos 0 - cos 0o) 6 (cp - cf>o ) / r

2

(2.llc)

2.2.6

Delta function as Divergence and Laplacian

(2.12) In some Books 6(r) is written as

2.2. 7

.t H (t) - e- >.(t+z)H (t _

00

+ z)dt

>.z e zo

where z0 = 0 for z > 0 and z0 = z for z < 0; so

>.z 2>.t e e

a(z ) = ,\2

00

-2,\

The Fourier transform off (t) is given by ~

1

f(w ) = ~ [email protected]

00

o

1 - >.t - iwtdt -e e ,\ 99

1 =------

~ , \(,\

+ iw)

physicsguide CSIR NET, GATE

00: 04: 35 1 f(w) = y'2n

00

~

1 - >.t - iwt dt -e e

1 =------

y'2n,\(,\ + iw)

A

o

j [email protected]

99

physicsguide CSIR NET, GATE

Integral Transformations

@Sk J ahiruddin, 2020

T he special case of the Wiener-Kinchin t heorem in which both functions are t he same shows that t he inverse Fourier 2 t ransform of t he energy spectrum, v'2n j (w) , is equal to the auto-correlation function, i.e.

from which t he stated result follows immediat ely.

4

More Exercises

4.1. Find t he Fourier t ransformation of t he function

f(x ) =

0

forx

e-x/r sin( k 0 x)

forx

0

4.2. Find Inverse Laplace transformation of

6-s 2 s + 4s + 20 Solve these differential equation by Laplace Transformation 4.3.

d2 y

dy

dx

dx

-

+ 4 2

+ 4y = x 2e- 2t

wit h t he boundary condit ion y(O)

=

y' (0) = 0 1





1

r"'1 rt TT"'\.

,,_, T ,-,, rr,

,......

I,

~ T""\

11 oo : 04: 38 4 .3.

d2 y - 2 dx

dy +4 dx

+ 4y = x 2e- 2t

with the b oundary condition y(O) = y'(O) = 0 [email protected]

100

physicsguide CSIR NET, GATE

Integral Transformations

@Sk J ahiruddin , 2020

4 .4.

dY dy dx 2 + 4 dx + 5y = 8(x - x 0 )

with t he boundary condition y(O) = y'(O) = 0 4 .5. Evaluate

1T

coshx8''(x - l )dx 0

4 .6. Expand the function in Fourier Series.

0,

J(x) =

sin x,

2

J(x) =

j [email protected]

101

physicsguide CSIR NET 1 GATE

@Sk J ahiruddin, 2020

Integral Transformations

Find t he cosine t ransform of f(x) and use it to write f(x) as an integral. Use your result to evaluate 2

2

00

cos a sin a/2 da 2 o a 4.9. What would be the apparent frequency of a sound wave

represented by 00

_

cos60nnt

7

4.10. Find inverse Laplace Transformation of the functions

(a)

l +p (p + 2)2

2p - 1 p 2 - 2p + 10

(b)

(c)

p e- p1r

(p2

+ 1)

4.11. Find t he Laplace Transformation of

J(t) =

sin(x - vt) , t > x/v 0. t < xiv

11 oo : 04: 43 (p2

+ 1)

4.11. Find t he Laplace Transformation of

J(t) =

sin(x - vt), t > x/v 0, t < x/v

4.12. Solve t he following sets of equations by the Laplace

t ransform method [email protected]

102

physicsguide CSIR NET, GATE

@Sk J ahiruddin, 2020

Integral Transformations

z' + 2y = 0 y' - 2z = 2

(a)

y' + z = 2 cost z' - y = l

(b)

4.13. Evaluate following integrals, 1

(a)

1r/2

1 3 e x 0 ( X) dx

- 1

4.1 4

- 1r/2

Ans Keys 1

·1·

2

1 k + ko - i / T

1 k - ko - i / T

4.2. e- 4t(2 sin 4t - cos 4t) x4e- 2t

4.3. y 4.4 .

cos xo(sin X )dx

(b)

= -12

(

11

oo : 04: 4s

o4.2. e- 4 t(2 sin 4t - cos 4t) x4e- 2t

= --

4 .3. y

12

4.4.

y=

e- 2(x-xo) sin (x - xo)

for x > Xo

0

for x < x 0

4.5. : cosh 1 4.6.

f (x ) =

1 1 2 -+sin x-rr 2 1r

j [email protected]

cos 2x 22 - 1

cos 4x

+ 42 -

103

1

cos6x

+ 62 -

1

+ ...

physicsguide CSIR NET I GATE

@Sk J ahiruddin , 2020

Integral Transformat ions

4. 7. ; e-lxl 7r

4.8. -

8

4.9. 90

4.10. (a)

e- 2t -

te- 2t ;

(b)

cos(t - rr)

for

t>

1r

0

for

t


1r

for

t
0) we have f (z) = x - iy so that

av = - 1

A1,

11 oo : oo : 34 secon auc y- 1emann re at1on 1n .5 1s sat1s e everywhere. Turning to the first Cauchy-Riemann relation, in the first quadrant (x > 0, y > 0) we have J(z)

8u = l ax '

= x - iy so t hat

8v = - l ay

which clearly violates the first relation in (1.5) Thus J(z) is not analytic in the first quadrant. Following a similiar argument for t he other quadrants, we find

au -- -l or + 1 for x < 0 and x > 0, resp ectively ox and = - 1 or + 1 for y > 0 and y < 0, respectively.

t

Therefore 8u/8x and 8v/8y are equal, and hence J( z ) is analytic only in t he second and fourth quadrants. since x and y are related to z and its complex conjugate z* by

x=

~ (z + z*)

and

y =

;i (z -

z*)

we may formally regard any function f = u+iv as a function of z and z*, rather t han x and y. If we do t his and examine

j [email protected]

@Sk J ahiruddin, 2020

8f /8z* we obtain aJ aJ ax aJ ay - = -- + -8z* ax 8z* 8y oz*

13

physicsguide CSIR NET, GATE

Complex Analysis

00: 00: 37

aJ /az* we obtain aJ aJ ax aJ ay - = -- + -az* ax az* ay az* au .av 1 au .av ax + i ax 2 + ay +iay l au av av au + +2 ax ay 2 ax ay

1 -2i

(1.6)



1,

Now, if f is analytic then the Cauchy-Riemann relations must be satisfied, and these immediately give that af / az* is ident ically zero. Thus we conclude that if f is analytic then f cannot be a function of z* and any expression representing an analytic function of z can contain x and y only in the combination x + iy, not in the combination x - iy We conclude this section by discussing some propert ies of analytic functions that are of great practical importance in t heoretical physics. These can be obtained simply from t he requirement that the Cauchy-Riemann relations must be satisfied by t he real and imaginary parts of an analytic function. The most important of these results can be obtained by different iating the first Cauchy-Riemann relation with respect to one independent variable, and the second with [email protected]

@Sk J ahiruddin, 2020

14

physicsguide CSIR NET, GATE

Complex Analysis

respect to t he other independent variable, to obtain the two

11 oo : oo : 39 @Sk Jahiruddin, 2020

Complex Analysis

respect to the other independent variable, to obtain the two chains of equalities

a ax a ax

au ax av ax

a av a av a au ay ay ax ay ay ax a au a au a av ay ay ax ay ay ax

Thus both u and v are separately solutions of Laplace's equation in two dimensions, i.e.

a2u a2u ax2 + ay2 = 0

(1. 7)

and

Those functions satisfies the Laplace written above are called Harmonic function. Example: Consider the function

u(x, y)

= x2

-

y2.

Show t hat u(x, y) is a harmonic function. Find t he function v(x, y) such t hat u + iv is an analytic function of z Solution:

au au V u = ax2 + ay2 2

2

2

=

2- 2= 0

So u satisfies Laplace's equation hence u is a harmonic function . [email protected]

@Sk J ahiruddin , 2020

15

physicsguide CSIR NET, GATE

Complex Analysis

11 oo : oo : 42 15

[email protected]

physicsguide CSIR NET, GATE

@Sk J ahiruddin , 2020

Complex Analysis

Now by Cauchy - Riemann equations av = au = 2x 8y ax

Integrating part ially with respect to y, we get v (x, y)

= 2x y + g(x)

where g( x) is a function of x to be found. Differentiating partially with respect to x and again using the CauchyRiemann equations, we have av = 2y ax

I

+ g ( X)

OU

= - -

8y

=

2y

Thus we find g' (x) = 0,

or

g = const .

Then

f (z ) = u +iv= x

2

-

y

2

+ 2ixy +

const.

=

z

2

+

const

The pair of functions u, v are called conjugate harmonic functions.

j [email protected]

16

physicsguide CSIR NET, GATE

11 oo : oo : 4s

j [email protected]

16

physicsguide CSIR NET, GATE

@Sk J ahiruddin , 2020

1.2

Complex Analysis

Power series in a complex variable

We can expand a function of complex variable in a power series like 00

(1.8)

where z is a complex variable and the an are, in general, complex. We now consider complex power series in more detail. Expression (1.8) is a power series about t he origin and may be used for general discussion, since a p ower series about any other point z0 can be obtained by a change of variable from z to z - zo . If z were written in its modulus and argument form , z = r exp i 0, expression (1.8) would become 00

f (z ) =

L anrn exp(in0)

(1.9)

n=O

This series is absolutely convergent if (1.10) which is a series of positive real terms, is convergent. Thus tests for the absolute convergence of real series can b e used

11 oo:oo:47 (1.10)

which is a series of positive real terms, is convergent. Thus tests for the absolute convergence of real series can b e used [email protected]

17

physicsguide CSIR NET, GATE

Complex Analysis

@Sk J ahiruddin, 2020

in the present context, and of these t he most appropriate form is based on the Cauchy root t est. With the radius of convergence R defined by

1 R

lim an

(1.11)

l/n

n -tOO

t he series (1.8) is absolutely convergent if z < R and divergent if lz > R If lz = R then no particular conclusion may be drawn, and this case must be considered separately. A circle of radius R centred on the origin is called the circle of convergence of the series I: anzn. The cases R = 0 and R = oo correspond, resp ectively, to convergence at the origin only and convergence everywhere. For R finite the convergence occurs in a restricted part of the z - plane (the Argand diagram) . For a power series about a general point z 0 , the circle of convergence is, of course, cent red on t hat point. Find the parts of the z-plane for which t he following series are convergent: Example:

oo

00

(ii) ) . n! zn

(iii)

n

>. -z

11 oo : oo : so Find the parts of the z-plane for which t he following series are convergent: Example:

n

oo

00

(i) ~ _z L..t n !

(ii)

~

zn (iii) L..t n

L n!zn

n=O

n=O

[email protected]

18

n=l

physicsguide CSIR NET, GATE

@Sk J ahiruddin, 2020

Complex Analysis

Solution: (i) since (n !) 1fn behaves like n as n ➔ oo

we find lim(l/n!) 1/n convergent for all z .

0. Hence R

=

(ii) Correspondingly, lim(n!) 1/n t he series converges only at z = 0.

=

oo and t he series is

oo . Thus R

=

=

0 and

(iii) As n ➔ oo, (n) fn has a lower limit of 1 and hence lim(l/n) 1/n = 1/ 1 = 1. Thus the series is absolutely convergent if t he condit ion zl < 1 is satisfied. 1

Case (iii) in the above example provides a good illust ration of the fact t hat on its circle of convergence a power series may or may not converge. For this particular series, the circle of convergence is Iz = l , so let us consider the convergence of the series at two different points on t his circle. Taking z = l , t he series becomes 00

1

1

1

1

n

2

3

4

L -=l+-+-+-+ ··· n=l

which is easily shown to diverge . Taking z = - l , however, .

,

,

11 oo : oo : s3 1

1

1

1

n=l n

2

3

4

oc

L - = l+-+- + -+· ·· which is easily shown to diverge . Taking z = - l , however , t he series is given by

j [email protected]

19

physicsguide CSIR NET 1 GATE

Complex Analysis

@Sk J ahiruddin, 2020

which is an alternating series whose terms decrease in magnitude and which t herefore converges. The ratio test may also be employed to investigate the absolut e convergence of a complex power series. A series is absolut ely convergent if z ln+l lim _a_n_+_l _ __ n----+oo an z ln

(1.12)

and hence t he radius of convergence R of the series is given by _!_ = lim lan+l R n----+oo an For instance, in case (i) of the previous example, we have 1 R

=

n! 1 lim - - - = lim - n----+oo ( n + l ) ! n ----+oc n + l

=

0

Thus the series is absolut ely convergent for all (finite) z, confirming t he previous result.

11 oo : oo : 55 1 R

=

n! 1 lim - - - = lim - n➔ oo ( n + 1) ! n➔oo n + 1

=

0

Thus the series is absolutely convergent for all (finite) z, confirming t he previous result.

1.3

Some elementary functions

In the example at the end of t he previous section it was shown t hat the function exp z defined by [email protected]

20

physicsguide CSIR NET, GATE

@Sk J ahiruddin, 2020

Complex Analysis

oo

exp z

=

n

L zn.

I

(1.13)

n=O

is convergent for all z of finite modulus and is t hus, by t he discussion of t he previous section, an analytic funct ion over the whole z-plane. You may note that functions that are analytic in the whole z-plane are usually called integral or ent ire functions. Like its real-variable counterpart it is called the exponential function; also like its real counterpart it is equal to its own derivative. The multiplication of two exponential functions results in a further exponential function, in accordance with t he corresponding result for real variables.

11 oo : oo : sa in a further exponential function, in accordance with the corresponding result for real variables.

We will Show that exp z 1 exp z 2

= exp (z1 + z 2)

From the series expansion of exp z1 and a similar expansion for exp z2 , it is clear t hat the coefficient of z1z~ in t he corresponding series expansion of exp z1 exp z2 is simply 1/(r·!s!)

j [email protected]

21

physicsguide CSIR NET I GATE

@Sk J ahiruddin , 2020

Complex Analysis

From the definit ion of exp z we also have

In order to find the coefficient of z 1z~ in this expansion, we clearly have to consider the term in which n = r + s, namely (z 1

+ z2)r+s

(r+s) !

= (r +1 ) ! (r + s) ! (r 0 zr+s + ... +r+s C zr zs + ... +r+s C zr+s) 1 s l 2 r+s 2 8 The coefficient of z1z~ in this is given by 1

1

1

11 oo : 01 : oo 1

= ---(r + s) (r zr+s + . .. +r+s C zr zs + ... +r+s C zr+s) (r + ) ! ! 0 1 s I 2 r+s 2 8

The coefficient of z1z~ in this is given by

= (r + s) !

l

r+s c s

(r+s)!

1

s!r ! (r+s) !

1

r !s!

Thus, since t he corresponding coefficients on the two sides are equal, and all t he series involved are absolutely convergent for all z , we can conclude t hat exp z1 exp z2

= exp (z1+

z2) You can easily see exp z

= (exp x) (cos y + i sin y)

[email protected]

(1.14)

physicsguide CSIR NET ) GATE

22

Complex Analysis

@Sk J ahiruddin ) 2020

If we express z as

z

= rexp i0

where r is the (real) modulus of z and 0 is its argument (-1r

< 0 < 1r), t hen multiplying z b)r exp(2ik1r), where k is

an integer, will result in t he same complex number z. Thus we m ay write

z

= r exp [i(0 + 2k1r) )

where k is an integer. If we denote w in expw

=z

11 oo : 01 z

: 03

= r exp [i(0 + 2k1r)]

where k is an integer. If we denote w in exp w

=

z

by w

ln z

=

=

ln r

+ i (0 + 2k1r)

(1.15)

where ln r is t he natural logarit hm (t o base e ) of the real positive quantity r, then Ln z is an infinitely multivalued function of z . Its principal value, denoted by ln z is obtained by taking k = 0 so that its argument lies in the range -1r to

1r.

Thus

ln z

= ln r + i0,

with -

[email protected]

23

1r

< 0 < 1r

(1.16)

physicsguide CSIR NET, GATE

Complex Analysis

@Sk J ahiruddin , 2020

ft(# 0) and z are both complex, then the z th power oft is defined by tz = exp (z Ln t) (1.17) We will show that t here are exactly n distinct nth roots oft. t l fn

= exp

1 - Lnt n

On the RHS let us write t as follows: t

=

r

PYn

fi ( A + ? k1r) 1

11 oo : 01 : os t 1f n = exp - Ln t n

On the RHS let us writ e t as follows:

t = r exp[i(0 + 2k1r)] where k is an integer. We then obtain 1 (0 + 2k1r) t ln = exp - ln r + i - - - n n I/n .(0+2k1r) =r exp i n 1

where k = 0, 1, ... , n - 1; for other values of k we simply recover the roots already found. Thus t has n distinct n th roots.

j [email protected]

@Sk J ahiruddin, 2020

1.4

24

physicsguide CSIR NET, GATE

Complex Analysis

Multivalued functions and branch cuts

In the definition of an analytic function, one of the condit ions imposed was t hat the function is single-valued. However, as shown in the previous section, the logarithmic function, a complex power and a complex root are all multivalued. Nevertheless, it happens that the properties of analytic functions can still be applied to these and other multivalued

11 oo : 01 : oa ever, as shown in t he previous section , the logarithmic funct ion, a complex power and a complex root are all mult ivalued. Nevertheless, it happens that the properties of analytic functions can still be applied to these and other multivalued functions of a complex variable provided that suitable care is taken. This care amounts to identifying the branch points of the multivalued function f (z) in question. If z is varied in such a way that its path in the Argand diagram forms a closed curve that encloses a branch point, then, in general, f (z) will not return to its original value. For definiteness let us consider t he multivalued function 1 2 f (z) = z 1 and express z as z = r exp i0. From figure 1.1 (a), it is clear t hat, as t he point z traverses any closed contour C that does not enclose the origin, 0 will return to its original value after one complete circuit. However , for any closed contour C' that does enclose the origin, after one circuit 0 ➔ 0 + 21r (see figure 1.1 (b)) . Thus, for the function 25

[email protected]

physicsguide CSIR NET, GATE

Complex Analysis

@Sk J ahiruddin, 2020

f (z) =

1 2

z l

,

after one circuit

r 1 exp (i0 / 2) ➔ r 1 exp [i( 0 + 21r) / 2] = -r 1 exp (i0 / 2) 1 2

1 2

1 2

In other words, the value of f (z) changes any closed loop enclosing the origin; in t his case f (z) ➔ - f (z) . Thus z = 0 1 2 is a branch point of the function f (z) = z 1 We note in ,,

'

·r-

1

1

1









11 oo : 01

: 11

In other words, the value of f (z) changes any closed loop enclosing the origin; in t his case f (z) ➔ - f (z) . Thus z = 0 is a branch point of the function f (z) = z 1l 2 We note in t his case t hat if any closed contour enclosing the origin is 1 2 t raversed twice then f (z) = z 1 returns to its original value. The number of loops around a branch point required for any given function f (z) to return to its original value depends on the function in question, and for some functions (e.g. Ln z, which also has a branch point at the origin) t he original value is never recovered.

26

[email protected]

physicsguide CSIR NET, GATE

@Sk J ahiruddin, 2020

Complex Analysis

y

y

y

,. 0

0 X

X

X

11 oo : 01

: 13

y

y

y

,. 0 X

(a)

-

X

(b)

X

(c)

Figure 1.1: (a) A closed contour not enclosing the origin; (b) a closed contour enclosing the origin; (c) a possible 1 2 branch cut for f (z) = z / In order that f (z) may be treated as single-valued, we may define a branch cut in the Argand diagram. A branch cut is a line (or curve) in the complex plane and may be regarded as an artificial barrier t hat we must not cross. Branch cuts are positioned in such a way t hat we are prevented from making a complete circuit around any one branch point, and so t he function in question remains single-valued. For the function f (z) = z 112 , we may take as a branch cut any curve starting at the origin z = 0 and extending out to lz = oo in any direction, since all such curves would j [email protected]

@Sk J ahiruddin, 2020

27

physicsguide CSIR NET 1 GATE

Complex Analysis

equally well prevent us from making a closed loop around t he branch point at t he origin. It is usual, however, to take t he cut along either the real or the imaginary axis. For

11 oo : 01

: 16

@Sk J ahiruddin , 2020

Complex Analysis

equally well prevent us from making a closed loop around t he branch point at the origin. It is usual, however, to t ake t he cut along either the real or the imaginary axis. For example, in figure 1.1 (c) , we t ake the cut as t he positive real axis. By agreeing not to cross t his cut , we restrict 0 to lie in the range O < 0 < 21r, and so keep f (z) single-valued. These ideas are easily extended to functions with more t han one branch point. Example: Find the branch points off (z) = J z 2 + 1, and hence sket ch suitable arrangements of branch cuts.

Solution: We begin by writ ing

f (z)

=

✓z2 + l

=

f (z) as

✓(z - i)( z + i)

As shown above, the function g(z) = z1l 2 has a branch point at z = 0. Thus we might expect f (z) to have branch points at values of z that make t he expression under the square root equal to zero, i.e. at z = i and z = -i As shown in figure 1. 2 (a), we use the notation z - i = r 1 exp i01

We can therefore write

[email protected]

@Sk J ahiruddin , 2020

f (z)

and

z

+ i = r 2 expi02

as

28

physicsguide CSIR NET, GATE

Complex Analysis

11 oo : 01

: 1a

@Sk J ahiruddin , 2020

y.

i

l

Complex Analysis

z

rI

·· ·· 0I

y

y

l

l

r2

.,x

-,

.

02

.

- /

(b)

(a)

Figure 1.2:

X

-,. (c)

(a) Coordinates used in the analysis of the

112 2 branch points of f (z) = (z + 1) ; (b) one possible ar-

rangement of branch cuts; (c) anot her possible branch cut , which is finite. Let us now consider how f (z) changes as we make one complete circuit around various closed loops C in t he Argand diagram. If C encloses (i) neither branch point, t hen

01 -+ 01 , 02 -+ 02 and so f (z) -+ f (z) (ii) z = i but not z = - i, then 01 -+ 01 + 21r, 02 -+ 02 and so f (z) -+ - f( z) (iii) z = -i but not z = i, then 01 -+ 01, 02 -+ 02 + 21r and so f (z) -+ - f (z) (iv) both branch points, t hen 01 -+ 01+ 21r, 02 -+ 02+ 21r and so f (z) -+ f (z) Thus, as expected , f (z) changes value around loops containing either z = i or z = -i (but not both) . We must t herefore choose branch cuts that prevent us from making a complete Joop around j [email protected]

@Sk J ahiruddin, 2020

29

physicsguide CSIR NET, GATE

Complex Analysis

11 oo : 01 [email protected]

29

: 21

physicsguide CSIR NET, GATE

Complex Analysis

@Sk J ahiruddin , 2020

either branch point; one suitable choice is shown in figure 1.2 (b) For this f (z), however, we have noted t hat after traversing a loop containing both branch points t he function ret urns to its original value. Thus we may choose an alternat ive, finite, branch cut that allows t his possibility but still prevents us from making a complete loop around just one of the points. A suitable cut is shown in figure 1.2 (c)

1.5

Singularities and zeros of complex functions

1.5.1

Singular point

A singular point of a complex function f (z) is any point in t he Argand diagram at which f (z) fails to be analytic. We have already met one sort of singularity, the branch point , and in t his section we will consider other types of singularity as well as discuss the zeros of complex functions. If f (z ) has a singular point at z = z 0 but is analytic at all points in some neighbourhood containing z0 but no other [email protected]

30

physicsguide CSIR NET, GATE

11 oo : 01

: 23

If f (z) has a singular point at z = z 0 but is analytic at all points in some neighbourhood containing z0 but no other j [email protected]

physicsguide CSIR NET 1 GATE

30

Complex Analysis

@Sk J ahiruddin, 2020

singularities, t hen z = z 0 is called an isolated singularity. (Clearly, branch points are not isolated singularities.) The most impo1~tant type of isolated singularity is t he pole. If f (z) has the form

f (z) = (Z

g(z) -

)' n Zo

(1.18)

where n is a positive integer, g( z) is analytic at all points in some neighbourhood containing z = z 0 and g (z 0 ) =I- 0, then f (z) has a pole of order n at z = z 0 . An alternative (though equivalent) definition is that lim [(z - zo)n f (z)] = a

z --+ zo

(1.19)

where a is a finite, non-zero complex number. We note t hat if t he above limit is equal to zero, then z = z 0 is a pole of order less than n, or f (z) is analytic there; if the limit is infinite then the pole is of an order greater than n. It may also be shown that if f (z) has a pole at z = zo, then f (z ) ➔ oo as z ➔ z 0 from any direction in the Argand diagram.' If no finite value of n can be found such that equation (1.19) is satisfied, then z = z 0 is called an essential singularity.

11 oo : 01

: 26

0

diagram. ' If no finite value of n can be found such that equation (1.19) is satisfied, then z = z 0 is called an essential singularity.

31

[email protected]

physicsguide CSIR NET, GATE

@Sk J ahiruddin, 2020

Complex Analysis

Example: Find the singularities of the functions 1

1

(ii)f (z) = tanh z

(i)f (z) = 1- z - l +z'

f (z)

Solution: (i) If we write 1

f (z ) =

as 2z

1

1- z

1+ z

(1 -

z)(l + z)

we see immediately from (1. 19) t hat f( z) has poles of order 1 (or simple poles) at z = 1 and z = - 1 (ii) In this case we write

f (z) = tanh z =

sinh z = exp z - exp(-z) cosh z exp z + exp(- z)

Thus f (z) has a singularity when exp z = - exp (-z) or, equivalently, when exp z = exp[i(2n + l )1r] exp( -z) where n is any integer. Equating t he arguments of the exponent ials vve find z = (n + ½) 1ri for integer n Furthermore, using l'Hopital's rule we have ( r

I

1 \

.7



,

11 oo : 01

: 29

where n is any integer. Equating the arguments of the exponentials we find z = (n + ½ ) 1ri for integer n Furthermore, using l'Hopital's rule we have

[z - (n

lim

+ ½) 1ri] sinh z cosh z

z--+(n+½)1ri 32

j [email protected]

physicsguide CSIR NET I GATE

@Sk J ahiruddin , 2020

lim

Complex Analysis

[z - (n + ½) 1ri] cosh z + sinh z sinh z

z--+(n+~)1ri

= 1

Therefore, from (1.19) , each singularity is a simple pole. Another type of singularity exists at points for which the value of j(z) takes an indet erminate form such as 0/ 0 but limz--+zo f (z) exists and is independent of the direction from which z 0 is approached. Such points are called removable singularities.

Example: Show that j(z) singularity at z = 0

= (sin z)/z has a removable

Solution: It is clear that f (z) takes the indeterminate form O/ 0 at z = 0. However , by expanding sin z as a power series in z, we find 1

f( z) = z

z3 z - 3!

+

z5 5!

-

z2

...

z4

= 1--+--·•· 3! 5!

Thus limz--+O f (z) = 1 independently of t he way in which ,

, ,

.

,

..

11 oo : 01 1

f( z) = -

z

z3

: 31

z5

z4

3!

5!

= 1--+-- ·•·

z - - + - - ·•· 3!

z2

5!

Thus limz-+O f (z) = 1 independent ly of t he way in which z ➔ 0, and so f (z) has a removable singularity at z = 0.

[email protected]

physicsguide CSIR NET ) GATE

33

Complex Analysis

@Sk J ahiruddin ) 2020

1.5.2

Behaviour of function at infinity

An expression common in mathematics, is 'z tends to infinity' . For complex z t he behaviour off (z) at infinity is given by t hat off (1/ ~) at ~ = 0, where ~ = 1/ z Example: A Find the behaviour at infinity of (i) f (z) 2 2 a + bz - , (ii) f (z) = z (l + z ) and (iii) f (z) = exp z

=

Solution: (i) f (z) = a+ bz - 2 : on putting z = l / ~, f (l / ~) a+ b~ 2 , which is analytic at ~ = 0 thus f is analytic at z = oo

(ii) f( z) = z (l + z of order 3 at z = oo

2

) :

f( l / ~)

=

1/~ + 1/~

3

;

thus a pole

(iii) f (z) = exp z : f (1/ ~) = I:r(n!)- 1 ~ -ri; thus f has an essent ial singularity at z = oo .

1.5.3

Zeros of a function

=

11 oo : 01

: 34

(iii) f (z) = exp z : f (1/~) = I::r(n!)- 1 ~-n ; thus an essential singularity at z = oo.

1.5.3

f

has

Zeros of a function

We conclude t his section by briefly mentioning the zeros of a complex function. As the name suggests, if f (z0 ) = 0 t hen z = z0 is called a zero of t he function f (z) . Zeros are

[email protected]

34

physicsguide CSIR NET, GATE

@Sk J ahiruddin , 2020

Complex Analysis

classified in a similar way t o poles, in t hat if

where n is a positive integer and g (zo) -=/= 0, then z = zo is called a zero of order n off (z ). If n = l t hen z = z0 is called a simple zero. It may further be shown that if z = z 0 is a zero of order n of f (z) then it is also a pole of order n of the function 1/ f (z) We will return in to the classification of zeros and poles in terms of t heir series expansions after we discuss the series expansion of complex functions.

11 oo : 01

j [email protected]

: 36

35

physicsguide CSIR NET, GATE

Complex Analysis

@Sk J ahiruddin , 2020

1. 6

Exercises

1.1. Find an analytic function of z

= x +iy whose imaginary

part is

(y cos y + x sin y) exp x 1.2. (a) Find a function

f (z),

analytic in a suitable part of t he Argand diagram, for which

Ref=

sin2x cosh 2y - cos 2x

(b) Where are the singularit ies of J(z)? 1.3. Det ermine t he types of singularities (if any) possessed

by the following functions at z = 0 and z = oo : (b) ( 1 + z 3 ) / z 2 ( c) sinh ( 1/ z ) (a) ( z - 2)- 1 -



I)

,

'

1

/()



/

.

"\

1/2

(d)

11 oo : 01

: 39

(b) Where are the singularit ies of f( z )? 1.3. Determine t he types of singularities (if any) possessed

by the following functions at z = 0 and z = oo : (b) (1 + z 3 ) /z 2 (c) sinh(l/z) (a) (z- 2) - 1 ez/z3 (e) z1/2/ (1 + z2)1;2

(d)

1.4. Identify t he zeros, poles and essential singularities of

t he following functions: (a) tan z (b) [(z - 2)/z 2] sin[l/ (1 - z)] (c) exp(l /z) (e) z 2I 3 (d) t an (1/ z)

[email protected]

@Sk Jahiruddin, 2020 1.6.1

36

physicsguide CSIR NET, GATE

Complex Analysis

Ans keys

1.1. zez

1.2. (a) f( z ) = sin2x-isinh2y (b) y = 0 and x = n1r cosh2y-cos 2x ' 1.3. (a) Analytic, analytic; (b) double pole, single pole;

(c) essential singularity, analytic; (d) t riple pole, essent ial singularity; (e) branch point, branch point. 1.4. See solution

11 oo : 01

[email protected]

: 42

37

physicsguide CSIR NET, GATE

Complex Analysis

@Sk J ahiruddin , 2020

1.6.2

Solutions

Solution: 1.1. If the required function is

f (z) = u + iv,

with v = (y cosy + x sin y) exp x, t hen from the CauchyRiemann equations,

av . . au ax = ex(y COSY+ Sln y + Sln y) = - By X

Integrating with respect to y gives u = -ex

(ycosy

+ xsin y + siny)dy + f(x)

11 oo : 01

: 44

x( . . ) U -8-x = e y cosy + x Sln y + sin y = - 8y V

Integrating with resp ect to y gives

u = -ex

(ycosy

+ xsiny + siny)dy + f(x)



ys1n y -

sinydy - xcosy - cosy

+ j(x)

= -ex(y sin y + cosy - x cosy - cosy) + f (x) = ex(x cosy - y sin y) + f (x) We det ermine f (x) by applying t he second Cauchy-Riemann equation , which equates 8u/8x with 8v/8y : ~~ = ex(x cosy - y sin y +cosy)+ f'(x )

i~ = ex (cos y By comparison, [email protected]

y sin y + x cos y)

J'(x) = 0 38

f (x) = k physicsguide CSIR NET I GATE

Complex Analysis

@Sk J ahiruddin, 2020

where k is a real constant that can be t aken as zero. Hence, the analytic function is given by

j (z) = u +iv= ex(xcosy -ysiny + iycosy + ixsin y)

= ex [(cos y + i sin y) (x + i y)] . = exeiY(x + i y)

The fin al line confirms explicitly that this is a function of z

11 oo : 01 =

: 47

ex [ ( cos y + i sin y) (x

+ i y)]

The final line confirms explicit ly that this is a function of z alone (as opposed to a function of both z and z* ) .

Solution: 1.2. Let the required function b e f (z ) = u + iv, wit h u = sin 2x / (cosh 2y - cos 2x) since y appears less often t han x in t he given expression, it will probably be easier to consider 8u/8y rather than 8u/8x . This indicates that the relevant Cauchy-Riemann equation is

8u 8y

- 2 sin 2x sinh 2y (cosh 2y - cos 2x ) 2

av ax

Having differentiat ed w. r.t y , we now int egrate w.r.t x :

v=

2 sin 2x sinh 2y d X= (cosh 2y - cos 2x ) 2

[email protected]

39

sinh 2y f - - - - - - + (y) cosh 2y - cos 2x physicsguide CSIR NET, GATE

@Sk J ahiruddin , 2020

Complex Analysis

By insp ect ion, or by subst it ution in the other C - R equation , f (y) can b e seen to b e an ignorable constant. The required function is t herefore

f (z) =

sin 2x - i sinh 2y cosh 2y - cos 2x

To det ermine what funct ion of z t his is, consider it s form

11 oo : 01

: 49

required function is there£ore

f (z) =

sin 2x - i sinh 2y cosh 2y - cos 2x

To determine what function of z t his is, consider its form on the real axis where y = 0

f(x) =

2 sin x cosx -----=cotx 2 2 sin x

sin 2x 1 - cos 2x

f (z) =

cot z

This can be verified as follows. c~s(x + ~y) s1n(x + i y ) cos x cosh y - i sin x sinh y sin x cosh y + i sin x sinh y (cos x cosh y - i sin x sinh y) (sin x cosh y - i cos x sinh y)

f (z) =

sin x cosh y + x sinh y 2 2 2 _ sin x cos x (cosh y - sinh y) - i cosh y sinh y (cos x 2

2

2

cos2

2

cosh y - 12 - cos 2 x

+ sin x )

+ 12

sin 2x - i sinh 2y cosh 2y - cos 2x since f (z) = cot z the poles can only occur at t he zeros n1r where n is an integer ; cos n1r =I= 0 of sin z, i.e at z j [email protected]

@Sk J ahiruddin , 2020

40

physicsguide CSIR NET I GATE

Complex Analysis

and so there will be a (simple) pole at each such point. The same conclusion is reached by studying cosh 2y - cos 2x . since cosh 2y 2:: 1 and cos 2x ~ 1, this denomiator can only vanish if both t erms equal 1; this requires y = 0 and x = n1r

2

11 oo : 01 : s2 and so there will be a (simple) pole at each such point. The same conclusion is reached by studying cosh 2y - cos 2x . since cosh 2y ~ 1 and cos 2x :S 1, this denomiator can only vanish if both terms equal 1; t his requires y = 0 and x = n1r Solution: 1.3. 1

(a) Although (z - 2)- has a simple pole at z = 2, at both z = 0 and z = oo it is well behaved and analytic. (b) Near z = 0, J (z) = (1 + z3 ) / z 2 behaves like 1/ z 2 and so has a double pole there. It is clear t hat as z ➔ oof (z) behaves as z and so has a simple pole there; this can be made more formal by setting z = 1/~ to obtain g(~) = ~2 + ~- 1 and considering ~ -+ 0. This leads to the same conclusion . (c) As z ➔ oo, J(z) = sinh(l / z) behaves like sinh ~ as ~ -+ 0, i.e. analytically. However , the definition of the sinh function involves an infinite series - in t his case an infinite series of inverse powers of z . Thus, no finite n for which lim [zn f (z)] is finite

z ~O

can be found, and J (z) has an essent ial singularity at z = 0 (d) Near z = 0, J(z) [email protected]

@Sk J a hiruddin ) 2020

=

ez/ z3 behaves as 1/ z3 and has a 41

physicsguide CSIR NET ) GATE

Complex Analysis

pole of order 3 at t he origin. At z = oo it has an obvious essential singularity; formally, t he series expansion of e1/~

11 oo : 01

: 55

@Sk J ahiruddin , 2020

Complex Analysis

pole of order 3 at t he origin. At z = oo it has an obvious essential singularity; formally, t he series expansion of e1/( about ( = 0 contains arbitrarily high inverse powers of ( 2 112 z )

behaves as z 112 (e) Near z = 0, f( z) = z 1 / (1 + and t herefore has a branch point t here. To investigate its behaviour as z ➔ oo, vve set z = 1/ ( and obtain 1 2

f (z)

(-1

=

Hence

g(()

=

1/ 2

(

--

--

1+ (-2

f (z ) also has

1/ 2 f"'..J

(2 + 1

a branch point at z

(

1 2 /

as ( ➔ 0

= oo

Solution: 1.4. •

(a) This function tan z = sin z has zeroes when cosz 1 ( e ix-y . . ) . . sin z = 2i - e-ix+y = 0 ⇒ e 2 x e-y = e-1,XeY The two t erms can only be equal if t hey have equal mag. nitudes, i.e. e- YI = eY ⇒ y = 0. We also need eix = . e-ix ⇒ x = n 1r , where n is an integer. Thus t he zeroes of tan x occur at z = n1r The poles of tan z will occur at t he zeroes of cos z . By a similar argument to t hat above, t his needs y

= 0 and

e ix

=

jahir@physicsguide. in

@Sk J ahiruddin , 2020

-e- ix

=

e i( - x+1r+ 2n1r) ⇒ 42

2x

= (2n + 1)1r

physicsguide CSIR NET, GATE

Complex Analysis

11 oo : 01 : s1

@Sk J a hiruddin , 2020

Complex Analysis

Thus, the (simple) poles of tan z occur at z = (n

+ ~) 1r

We note that both sin u and cos u have Maclaurin series t hat contain arbitrarily large powers of u and that they are not mult iples of each other; we can conclude that their rat io will also have a Maclaurin series containing arbitrarily large powers of u. The same conclusion is reached by differentiating tan u and so constructing its Maclaurin series directly. Thus, when z ➔ oo is replaced by z = 1/ ~ with ~ ➔ 0, there will be arbitrarily large inverse powers of ~ in t he series expansion; this establishes that ~ = 0 (i.e. z = oo ) is an essential singularity of tan z (b) This function,

z- 2

z2

1 sin-- , has obvious zeroes at

1 -z

z = 2 and z = oo . Equally clearly, at z = 0 it has a 2 nd-order pole. Further zeroes will occur when the sine term factor is zero; from the analysis in part (a), this will be when (1- z)- 1 = n1r i.e. at z = 1- (n1r)- 1 . The remaining singularity to classify is that at z = l. By a similar argument to t hat given in part (a), the Laurent expansion of t he function about the point will have no largest negative power of 1 - z; t he point is therefore an essential singularity. (c) since exp(0) = 1, the function is well behaved and anj a [email protected]

(c)Sk J a,h ir11ddin. 2020

43

physicsguide CSIR NET, GATE

Comnlex Ana.lvsis

11 oo : 02 : oo j [email protected]

43

physicsguide CSIR NET I GATE

Complex Analysis

@Sk J ahiruddin , 2020

alytic at oo. The only nonanalytic point is the origin, z = 0, where t he defining series for t he exponential function generates a Laurent expansion with no largest negative power of z; the point is therefore an essent ial singularity. 1

(d) The singularities of tan (z- ) follow from those of tan z in part (a) . They are therefore zeroes at z = oo and 1 1 (n1r) - , simple poles at z = (n1r + t1r) - and an essential singularity at z = 0 (e) The origin, z = 0 is both a zero and a branch point of t he function z 213 To determine its behavior at oo we have 2 3 to consider 1/ ~ / near ~ = 0. There is clearly a singularity t here, and, since the function cannot even be expressed as a Laurent series, the singularity is an essential singularity.

[email protected]

44

physicsguide CSIR NET, GATE

11 oo : 02 : 03

44

[email protected]

physicsguide CSIR NET ) GATE

Complex Analysis

@Sk J ahiruddin ) 2020

Integration and Series Expansion

2 2.1

Integration of a Complex function

If a complex funct ion

J(z)

is single-valued and continuous

in some region R in the complex plane, t hen we can define t he complex integral of f (z) between two points A and B along some curve in R ; its value will depend, in general, upon t he path taken b etween A and B Let a particular path C be described by a continuous (real) paramet er (a :S t :S /3) t hat gives successive posit ions on C by means of t he equations X

= x(t) ,

y

= y(t)

wit h t = a and t = f3 corresponding to the points A and B , respectively. Then the integral along path C of a continuous function f (z) is written

J(z)dz C

And can be given explicit ly as a sum of real integrals as follows:

11 oo : 02 : os J(z)dz C

And can be given explicit ly as a sum of real integrals as follows: [email protected]

45

physicsguide CSIR NET , GATE

@Sk J a hiruddin , 2020

Complex Analysis

J(z)dz = C

(u

+ iv) (dx + idy)

C

udxC f3 a

v dy C

u

dx dt

+i

udy

+i

C f3

dta

dy V d dt + i t

vdx C

(3

u a

!yt dt + i

(3

a

dx V dt dt

(2.1) We will see how to evaluate integral of complex function manually from the definit ion of integration.

Example: Evaluate t he complex integral of f (z) = z- 1 along the three paths mentioned below and the figure (i) the circle z = R, starting and finishing at z = R : figure 2 .1 (a) (ii) the contour C 2 consisting of the semicircle z = R in the half-plane y > 0 : figure 2.1 (b) (iii) the contour C 3 made up of the two straight lines C3a and C3b :

figure 2 .1 ( c)

11 oo :02 : oa figure 2 .1 (c)

46

j [email protected]

physicsguide CSIR NET, GATE

@Sk J ahiruddin , 2020

Complex Analysis

y

y

y

C1

C2 C3b

R .·

R .· .

t

·,

-R

X

s= 1

{

R

(a)

Figure 2.1:

iR

X

(b)

t=O

R

-R

X

(c)

Different paths for an integral off (z)

= z- 1

Solution: (i) The path C 1 is circular. In t hat path t he distance from center is fixed . Hence t he path is parameterised as follows; see figure 2.1 (a)

z(t) = x(t)

+ iy(t)

and X(

t) = R cos t'

z(t) ,

. ..

= rl

x (t) \



y(t)

=

Rsin (t);

+ iy(t) = R cost+ iR sin t, •

,

0 < t < 21r

11 oo : 02 : 11 X(

y(t ) = R sin(t );

t ) = R cos t'

z(t) while

+ iy(t)

x (t)

=

f (z)

=

0 < t < 21r

R cost+ iR sin t,

is given by

f

z = ( )

l

X

+ iy

=

x 2 + y2

47

[email protected]

iy

X -

physicsguide CSIR NET , GATE

Complex Analysis

@Sk J a hiruddin, 2020

Thus the real and imaginary parts of

Rcos t R2

X

U=---

x2

+ y2

and

f (z)

are R sin t

-y

v=---= - --x2 y2 R2

+

Hence, using expression (2.1) 2 71"

1

- dz = C1

z

271"

cos t

R (-R sin t )dt -

-sint

R

R cos tdt

0

0 27!"

+i

o

cos t

- -R cos tdt + i R

2 71"

-sint R

o

(- R sin t) dt

= 0 + 0 + i1r + i1r = 21ri You could have done it directly 2

dz

1r

0

- R sin t + i R cos t d t= R eos t + iR sin t

271"

idt = 21ri (2.2) 0

(ii) This is just as in t he previous example, except that now O ::; t :::; 1r . Wit h this change, we have

11 oo : 02 : 13 0

idt = 21ri

- - - - - -dt = R cos t + i R sin t

2.2

0

(ii) This is just as in t he previous example, except that now O :S t :S 1r. Wit h this change, we have

dz

- = 7r'l •

(iii) The straight lines that make up the countour C3 may be parameterised as follows :

z = (l - t)R + itR

C3a,

for

48

[email protected]

O< t < 1

physicsguide CSIR NET , GATE

@Sk J a hiruddin , 2020

Complex Analysis

z = -sR + i(l - s) R

C3b ,

for

O< s < 1

With t hese parameterisations the required integrals may be written

1

dz 0

R + iR d ----. - t+ R +t(-R +iR )

1 0

- R - iR ( ) ds_ iR+ s - R -iR

Evaluation of these integral are time consuming. You can evaluate the first integral like this 1 0

- R + iR d t= R (l - t) + itR 1 0

1

2t - 1 1 - 2t+ 2t 2dt + i

= - rln (1 -

?.t + ?.t

2\ 1 1 1

(-1 + i) (1 - t - it) - - - - - -dt (1 - t)2 + t2 1 1 - - - - d2 t

1

0

0

1 - 2t + 2t

i.

+ -

r?.

t.::l.n

-1

(t -

2l

'11

11 oo : 02 : 16 i

0

-----dt = R (l - t) + itR 1 2t - 1 2 dt + i 1 2t + 2t 0

1 [ = ln ( 1 2

0 1

O

1 l - 2t + 2t 2 dt

2]1 i 2t + 2t ) + 0

2

.

i

(1 - t)2 + t 2

2 tan-

1

1 t--2

2

1

0



7r

7r

7r'l

2 2

2

2

= 0 +- - -

The second integral can also be shown to have the value wi/2. Thus dz

- = 7r'l •

49

j [email protected]

physicsguide CSIR NET 1 GATE

Complex Analysis

@Sk J ahiruddin , 2020

2.1.1

Cauchy's theorem

Cauchy's theorem states that if f (z) is an analytic function, and f' (z) is cont inuous at each point within and on a closed contour C, then

f (z) dz = 0

(2.3)

C

To prove t his theorem we will need the Green's t heorem in a plane . This says that if p and q are two functions with continuous first derivatives within and on a closed contour C (bounding a domain R ) in t he xy -plane, then

rr

8p "I

+

8q -"I-

dx dy =

£(pdy -

qdx)

11 oo : 02 : 1 a cont inuous first derivatives within and on a closed contour C (bounding a domain R ) in t he xy -plane, then 8p ax

R

With f (z) to

=

I=

u

+

8q ay

+ iv and dz

f( z )dz

=

dxdy =

dx

= c (pdy - qdx) + idy , this can be applied

(udx - vdy)

+i

C

C

(vdx

+ udy)

C

to give

I=

R

8(-u)

8(-v)

8y

8x

--+--

[email protected]

@Sk J ahiruddin, 2020

dxdy+i

50

R

8(-v)

au

8y

8x

--+-

dx dy

physicsguide CSIR NET, GATE

Complex Analysis

Now, recalling t hat f (z) is analytic and t herefore that t he Cauchy-Riemann relations apply, we see that each integrand is identically zero and t hus I is also zero; this proves Cauchy's theorem. Whenever you are asked to find to evaluate t he integration of a complex variable over a closed path (called a contour) you check that if the function is analytic over the bounded region or not (find whether the function diverges inside any point of the bounded region). If the function remains analytic then the integration over the closed contour is ZERO. That 's the Cauchy's theorem.

11 oo : 02 : 21 bounded region or not (find whether the function diverges inside any point of the bounded region). If the function remains analytic t hen t he int egration over the closed contour is ZERO. That 's the Cauchy's t heorem.

2.2

Cauchy's integral formula

Another very important theorem in t he theory of complex variables is Cauchy's integral formula, which st at es t hat if f (z) is analytic wit hin and on a closed contour C and z0 is a point within C t hen

f (zo) =

1

f (z) dz

.

27r1, C

Z -

(2.4)

Zo

This formula is saying t hat the value of an analyt ic funcj ahir@physicsguide. in

51

physicsguide CSIR NET I GATE

@Sk J ahiruddin , 2020

Complex Analysis

t ion anywhere inside a closed contour is uniquely determined by its values on the contour. We will skip t he proof. Consult any good text book. An extension to Cauchy's integral formul a can be made, yielding an integral expression for f' (z 0 ) :

f( z) c (z - zo)

2

d

z

(2.5)

11 oo : 02 : 23 f( z ) d 2 z c (z - zo )

(2. 5)

Furt her , it may be proved by induction that the n th derivative of f (z) is also given by a Cauchy int egral,

f( z) dz C ( Z - Zo )

(2.6)

n+ l

Example: Evaluate:

fi

·

2

+ cos 1rz c (z - l)(z - 2) is t he circle Iz = 3 (a)

sin 1r z

2

dz

52

[email protected]

e 2z

(b)

fc (z + l )4 dz where C

physicsguide CSIR NET ) GATE

@Sk J ahiruddin ) 2020

Complex Analysis

1

1 - -, we have

z -2 2

+ cos 1r z dz c (z - l)(z - 2) 2 2 sin 1rz + cos 1rz d zsin 1rT

c

z -l

2

z- 2

sin 1rT + cos 1rz dz c z - l 2

2

By Cauchy's int egral formula wit h a = 2 and a = l , respect ively, we have

11 oo : 02 : 26 sin 1rz

c

2

+ cos 1rz dz -

sin 1rT + cos 1r z dz z - 1

2

z- 2

2

c

2

By Cauchy's integral formula wit h a = 2 and a = l , respect ively, we have sin 1r z 2 + cos 1r z 2

c

------dz

z- 2

= 21ri {sin 1r(2)

sin 1r z 2 + cos 1r z 2

2

+ cos1r(2)

2

}

= 21ri

- - - - - -dz = 21ri { sin 1r(1) + cos 1r(1) } = - 21ri c z -1 2 2 since z = 1 and z = 2 are inside C and sin 1r1r z + cos 1r z is analytic inside C. Then , the required integral has the value 21ri - (- 21ri) = 41ri (b) Let

2

f (z) = e

2

z

and a

= -

2

l in t he Cauchy integral

formula

f (z)

dz

(z - a)n+l 111 and f ( - 1) = 8e- 2 . Hence the C

If n = 3, then

f

111

(

z) = 8e2z

above equation becomes 2

8e- = [email protected]

e2z

3! -

- - -dz 21ri C (z+ 1)4 53

@Sk J ahiruddin , 2020

physicsguide CSIR NET, GATE

Complex Analysis

from which we see that t he required integral has t he value

81rie- 2 / 3

2.2.1

Cauchy's inequality

11 oo : 02 : 2a

2.2.1

Cauchy's inequality

Suppose that f (z) is analytic inside and on a circle C of radius R centred on the point z = z0 . If If (z) < M on the circle, where M is some constant, we can show that

f

(n)

(z ) < Mn! o - Rn

(2.7)

We start from n! 2w

f (z)dz C (Z -

Zo )

n +l

We will state a result on complex integration without proof. Let is consider the integral of a function f (z) along some path C . If M is an upper bound on the value of f (z) on t he path, i.e. f (z) < M on C, and L is the length of the path C then

f( z) dz < M

f (z) dz < C

54

physicsguide CSIR NET, GATE

@Sk J ahiruddin , 2020

Complex Analysis

Using the above result we get

f

(n)

(2.8)

C

C

j [email protected]

dl =ML

(z ) < n ! M 2w R 0 - 2w Rn+l

= Mn! Rn

This result is known as Cauchv's ineoualitv

11 oo : 02 : 32 Using the above result we get

(n)(z ) < n ! M 21rR = M n ! f 0

-

21r Rn+l

Rn

This result is known as Cauchy's inequality

2.3

Taylor Series

Taylor's theorem for functions of a complex variable is the following.

If f (z) is analytic inside and on a circle C of radius R centred on the point z = zo, and z is a point inside C, t hen 00

f (z) =

Lan(z -

zo)n

(2.9)

n= O

where

The Taylor expansion is valid inside the region of analyticity and, for any particular z 0 , can be shown to be unique.

[email protected]

@ Sk J ahiruddin, 2020

55

physicsguide CSIR NET, GATE

Complex Analysis

To prove Taylor 's theorem we note that, since f (z) is analytic inside and on C, we may use Cauchy's formula to

.

.

,..

/

\

11 oo : 02 : 34 @Sk Jahiruddin, 2020

Complex Analysis

To prove Taylor's theorem we note t hat, since f (z) is analytic inside and on C, we may use Cauchy's formula to write f (z) as 1 (2.10) f (z) = . 21ri

where~ lies on C. Now we may expand the factor (~ - z)as a geometric series in (z - z0 ) / (~ - z0 ) Z -

1

n

Zo

~ - Zo

so

f (z)

becomes

f (z) =

!(~)

1

oc

21ri

1

= 2 . 1ri

z - zo

n

~ - Zo

f(~)

00

L (z -

zo )n

n=O

d

t )n+l ( C '::. - Zo

(n) ( ) . 1 """" ( 21ri 1 _ _ )n Zo -~ z-zo n.1 21ri oo

n=O

~

oo _ """" - ~

)n an z-z0 (

n=O

(2.11)

we have used equation (2.6) Example: Let j(z ) = ln(l + z), where we consider the branch that has the zero value when z = 0.

(a) Expand j(z) in a Taylor series about z = 0. [email protected]

56

physicsguide CSIR NET, GATE

Complex Analysis

@Sk J ahiruddin , 2020

1,

\

~

I

.

'

,

f'

'

,

11 oo : 02 : 37 [email protected]

physicsguide CSIR NET, GAT E

56

Complex Analysis

@Sk J ahiruddin, 2020

(b) Determine t he region of convergence for the series in (a).

(c) Expand ln(l + z/1 - z) in a Taylor series about z = 0 Solution: (a)

(a) f (z) = ln (1 + z) , 1 1 f (z) = = (1 + z) - , f I I ( z) = - (1 + z)- 2' 11 J (z) = (- 1)(- 2)(1 + z) - 3 ,

f (O) = 0 f '(O) = 1 1 f ' (0) = - 1 1 f ' ' (0) = 2!

• •



i!z







T hen

z2 z 3 z4 = z - - + - - -+ •·· 2

3

4

We could have proceed in another wayIf z < l 1

2

3

- - = l -z+z -z + ··· l +z j [email protected]

57

physicsguide CSIR NET, GAT E

1

A.

1



11 oo : 02 : 39 - - = l -z+z -z + ··· l +z [email protected]

57

physicsguide CSIR NET, GATE

Complex Analysis

@Sk J ahiruddin, 2020

Then integrating from O to z yields z2 z3 z4 ln(l + z) = z - - + - - - + · · · 2 3 4 (b) Then t h term is Un = (- l )n-Izn/n . Using t he ratio test, nz Un+l 1 · . 1Im - - = Im = z n➔ oo Un n----+oo n + l and the series converges for Iz I < 1. The series can be shown to converge for z = l except for z = - 1 This result also follows from t he fact t hat the series converges in a circle t hat extends to the nearest singularity (i.e., z = - 1 ) of

f( z) (c) From the result in (a) we have, on replacing z by - z z2

z3

z4

1nl ·· ( +z ) = z--+2 3 --+· 4 z2 z3 z4 1n ( 1 - z ) = -z - 2 - 3 - 4 - · · ·

both series convergent for z < 1. By subtraction, we have oo 2 z2n+l z3 z5 l +z ln =2 z+-+-+ ···

1 -z

3

5

which converges for z < 1. We can also show t hat t his series converges for z = 1 except for z = ± 1 [email protected]

58

physicsguide CSIR NET, GATE

11 oo : 02 : 42 n=O

n+

which converges for z < l. We can also show t hat t his series converges for z = l except for z = ± l [email protected]

58

physicsguide CSIR NET, GATE

Complex Analysis

@Sk J ahiruddin, 2020

2.4

Laurent Series

So far we have assumed that f (z) is analytic inside and on t he (circular) contour C. If, however, f (z) has a singularity inside C at t he point z = z0 , then it cannot be expanded in a Taylor series. Nevert heless, suppose that f (z) has a pole of order p at z = z 0 but is analyt ic at every other point inside and on C. Then the function g (z) = (z - z0)P f (z) is analytic at z = z 0 , and so may be expanded as a Taylor series about z = zo : 00

g(z) =

L bn (z -

zo)n

(2.12)

n=O

Thus, for all z inside C, f (z) will have a power series representation of the form

wit h a _ P i= 0. Such a series, which is an extension of the Taylor expansion, is called a Laurent series. By comparing

11 oo : 02 : 44

wit h a _ P #- 0. Such a series, which is an extension of t he Taylor expansion, is called a Laurent series. By comparing [email protected]

physicsguide CSIR NET, GATE

59

Complex Analysis

@Sk J ahiruddin , 2020

t he coefficients in (2 .12) and (2.13), we see that an = bn+p· Now, t he coefficients bn in the Taylor expansion of g(z) are seen from (2.11) to be given by

g(z) (Z -

Zo )

d n+l

Z

and so for the coefficients an in (2. 13) we have g(z)

dz

(z - zo)n+l+p

= I 21ri

f (z )n+ldz

(z - zo )

an expression t hat is valid for both positive and negative n. The terms in the Laurent series with n -> 0 are collectively called the analytic part , whilst the remainder of t he series, consisting of terms in inverse powers of z - z 0 , is called the principal part. Depending on t he natu1·e of t he point z = z 0 the principal part may contain an infinite number of terms, so that +oo

f (z) =

L

n = - oc

an (z - zo) n

(2.14)

11

00:02:47

+oo

f (z) =

L

an (z - zo)n

(2.14)

n = - oo

In this case we would expect the principal part to con1 verge only for (z - z 0 ) - less than some constant, i.e. outj [email protected]

physicsguide CSIR NET 1 GATE

60

Complex Analysis

@Sk J ahiruddin , 2020

side some circle centred on z 0 . However , t he analytic part will converge inside some (different ) circle also cent red on z 0 . If the latter circle has t he greater radius t hen the Laurent series will converge in t he region R between the two circles (see figure 2.2 below). Ot herwise it does not converge at all. y

R

• zo

X

Figure 2.2: The region of convergence R for a Laurent series off (z) about a point z = zo where f( z ) has a singularity.

11 oo : 02 : so X

Figure 2.2: The region of convergence R for a Laurent series off (z) about a point z == zo where f( z) has a singularity. In fact, it may be shown that any function f (z) that is analytic in a region R between two such circles C 1 and C2 centred on z == z 0 can be expressed as a Laurent series about z 0 that converges in R. We note that , depending on [email protected]

@Sk J ahiruddin, 2020

61

physicsguide CSIR NET, GATE

Complex Analysis

the nature of t he point z == z 0 , the inner circle may be a point (when the principal part contains only a finite number of terms) and t he outer circle may have an infinite radius. We may use the Laurent series of a function f (z) about any point z == z0 to classify the nature of t hat point. If f (z) is actually analytic at z == z 0 , t hen in (2.14) all an for n < 0 must be zero. It may happen that not only are all an zero for n < 0 but a0, a 1 , .. . , am- l are all zero as well. In t his case, t he first non-vanishing term in (2.14) is am (z - zo)m, with m > 0, and f (z) is then said to have a zero of order m at z == z0 If f (z) is not analytic at z == z0 , t hen two cases arise, as discussed above (p is here taken as positive): (i) it is possible to find an integer p such t hat but a-p-k == 0 for all integers k > 0

a _p -=I=

(ii) it is not possible to find such a lowest value of - p.

0

11

oo : 02 : s2

(i) it is possible to find an integer p such t hat a _p -=f. 0 but a-p-k = 0 for all integers k > 0 (ii) it is not possible to find such a lowest value of -p. In case (i), f (z) is of the form (2 .13) and is described as having a pole of order pat z = z0 ; the value of a _1 ( not a -p ) is called t he residue of f (z) at t he pole z = z0 , and will play an important part in later applications. For case (ii) , in which t he negatively decreasing powers 62

j [email protected]

physicsguide CSIR NET I GATE

@Sk J ahiruddin , 2020

Complex Analysis

of z - z0 do not terminate, singularity.

f (z)

is said to have an essent ial

Example:

Find t he Laurent series of 1

f (z) = z (z -

2)3

about the singularities z = 0 and z = 2( separately ) . Hence verify that z = 0 is a pole of -order 1 and z = 2 is a pole of order 3, and find the residue of f (z) at each pole.

Solution: To obtain t he Laurent series about z = 0, we make the factor in parentheses in t he denominator take the form (1 - a z ), where a is some constant, and t hus obtain

f (z ) = -

1 Q,.,/ 1

,., / ')\3

11 oo : 02 : 55 Solution: To obtain the Laurent series about z = 0, we make the factor in parentheses in t he denominator take the form (1 - a z ), where a is some constant, and t hus obtain 1

f (z ) = - 8z( l - z/2)3 z

= _ 1 l + (-3) - ~ + (-3)(-4) 8z

2

+ (-3)(-4)(-5) _ z

2! 3

2

2

+ ...

3!

2 2 1 3z 5z 3 - . .. 8z 16 16 32 since the lowest power of z is - 1, t he point z = 0 is a pole of order 1. The residue of f (z) at z = 0 is simply the 63

[email protected]

physicsguide CSIR NET ) GATE

@Sk J ahiruddin ) 2020

Complex Analysis

coefficient of z- 1 in the Laur ent expansion about that point and is equal to - 1/8 The Laurent series about z = 2 is most easily found by letting z = 2 + ( (or z - 2 = ( ) and substituting into the expression for f (z) to obtain

f (z) =

1 (2 + ()(3 1 12(3

=

1 1 2(3 - 4( 2 1

1

2(3(1 + (/2)

( 2

+

+

( 2

1 1 8( - 16 1

2

+

( 2 ( 32

3

+

( 2

...

1

1

4

-

...

11 oo :02 : s1 1-

~ 2

+

~ 2

2

~

-2

3

~

+ -2

1 1 1 1 ~ = 2~3 - 4~ 2 + 8~ - 16 + 32 - ... 1 1 1 1 = 2(z - 2)3 - 4(z - 2) 2 + 8(z - 2) - 16

4

+

- ···

z- 2 32 - · · ·

From this series we see t hat z = 2 is a pole of order 3 and t hat the residue of J(z) at z = 2 is 1/ 8

[email protected]

@Sk J ahiruddin , 2020

2.5

64

physicsguide CSIR NET, GATE

Complex Analysis

Calculation of Residue from Laurent Series

Finding the residue of a funct ion at a singularity is of crucial importance in t he evaluation of complex int egrals. Specifically, formulae exist for calculating t he residue of a function at a part icular (singular) point z = z 0 without having to expand the function explicitly as a Laurent series 1 about z 0 and identify the coefficient of (z - z 0 ) - . The type of formula generally depends on the nature of t he singularity at which the residue is required.

11 oo : 03 : oo a function at a particular (singular) point z = z 0 without having to expand the function explicitly as a Laurent se1~ies about z0 and identify the coefficient of (z - z 0 ) -

1

.

The type

of formula generally dep ends on the nature of the singularity at which the residue is required. Suppose that f (z) has a pole of order m at t he point z = zo. By considering t he Laurent series off (z) about zo . We will derive a general expression for the residue R (z0 ) of

f( z) at z = zo If f (z ) has a pole of order m at z series about t his point has t he form

= z0 , then its Laurent

a_m a-1 ! (z ) =---+ ··• +--(z - z0 )m (z - zo) 2

+ ao + a1 ( z - zo) + a 2 ( z - zo) + · •· which , on multiplying both sides of the equation by (z - z0 )m, j [email protected]

65

physicsguide CSIR NET, GATE

@Sk J ahiruddin, 2020

Complex Analysis



gives

(z - zo)m J(z) = a_m + a-m+l (z - zo) + · · ·

+ a_ 1 ( z - z0 )

m- 1

+ ···

Different iating both sides m - 1 t imes, we obtain ~

-1

00

dzm- l [(z - zo)m f (z) ] = (m - l )!a_1 +

L bn (z -

zo)n

n=l

for some coefficients bn. In the limit z



z0 , however , the

11 oo : 03 : 02 d m- 1

dzm- l [(z - zo)m f (z)] = (m - l )!a_1 +

oo

L bn (z -

zo )n

n=l

for some coefficient s bn. In the limit z ➔ zo, however , the t erms in t he sum disappear , and aft er rearranging we obtain t he formula

R (zo)

=

a-1

1 =

dm-1

(m - 1)! dzm-l [(z - zo)m f (z)]

lim

z➔ zo

(2. 15) which gives t he value of t he residue of f (z) at the point z = z0 . This is t he general residue formula for mth order pole. An import ant special case of (2.15) occurs when has a simple pole (a pole of order 1 ) at z residue at z 0 is given by [email protected]

66

f (z)

= z 0 . Then the

physicsguide CSIR NET, GATE

@Sk J ahiruddin, 2020

Complex Analysis

= lim [(z - z0 ) f (z) ]

R ( z0 )

(2 .16)

z ➔ zo

If f (z) has a simple pole at z = zo and, as is often t he case, has t he form g(z)/h(z), where g(z) is analytic and non-zero at z 0 and h (z0 ) = 0, then equation (2 .16) becomes . ( z - zo) g(z ) _ ( ) . ( z - z0 ) R (Zo) - 11m g Zo 1I m _

7

/

\

-

7

/

\

11 oo : 03 : os If f (z) has a simple pole at z = zo and, as is often the case, has t he form g( z) /h( z), where g(z) is analytic and non-zero at z 0 and h (zo) = 0, then equation (2. 16) becomes

. (z - z0 ) g(z) ( ) . (z - zo) R (Zo) = 11m = g Zo 11m z➔ zo

=

z➔zo

h(z)

. 1 g ( zo) z➔zo l1m hI ( z )

h(z )

g (zo)

(2.17)

( ) h' Zo

where we have used l'Hopital's rule. This result often provides t he simplest way " of det e1~mining the residue at a simple pole.

Example: Evaluate the residue of the function , by t he help of above prescription , at t he point z = i

f (z) =



exp iz (z2 + 1)2

Solution: •



exp iz

f( z) = exp iz (z2 [email protected]

@Sk J ahiruddin ,

2020

(z + i) 2 (z - i) 2

+ 1)2 67

physicsguide CSIR NET, GATE

Complex Analysis

we see immediately t hat it h as poles of order 2 (double poles) at z = i and z = -i. To calculate t he residue at (for example) z = i, we may apply the residue formula (equation (2.15)) with m = 2 P erforming the required different iation, we obtain

11 oo :03 : oa po es at z = i an z = -i. To ea cu ate t e resi ue at or example) z = i, we may apply the residue formula (equation (2.15)) with m = 2 P erforming t he required differentiation , we obtain

d

d

2

exp iz

dz[(z - i) f (z)]= dz (z+i) 2 1

= (

2

") 4 [(z + i) iexp iz - 2(exp iz)(z + i)]

z+i

Setting z

= i, we find the residue is given by 1 1 i 1 1 R (i) = 1! ( - 4ie- - 4ie- ) = - e 2 16

Example: Find R (-~) (Residue at-~) and R (5) (Residue at 5) for z f (z) = (2z + 1)(5 - z)

Solution: Multiply f (z) by (z + ½), [ Caut ion: not by (2z + 1)] , and evaluate t he result at z =

68

j [email protected]

physicsguide CSIR NET 1 GATE

Complex Analysis

@Sk J ahiruddin , 2020

-~ . We find

z+

1 2

f( z) = /

1\

1 z +2 1

z (2z + 1)(5 - z) 1

z 2(5 -

z)

11 oo : 03 : 1 o -~ - We find 1 z +2

R

1 z +2

J( z) =

(2z

+ 1)(5 - z)

- -1

1 -2

2(5 - z )

1 22

2

2 (5 +

z

z

½)

Similarly (z - 5)f (z)

=

z (z - 5) ( 2z + l )( 5 _ z)

Hen ce

z 2z+ 1

5

R(5) = - 11 Example: Find the residue of cot z at z

=0

Solution:

.

R (0) = l1m

z cos z .

z ---tO Sill Z

.

= cos O · l1m .

z

z ---tO Sill Z

Example: Find t he residue of (sin z )/

=

1·1= 1

(1 - z4 )

at z

=i

Solution: By (2. 17) we have, sin z R(i) = -4z3 .

z=i

jahir@physicsguide. in

@Sk J ahiruddin , 2020

sin i e- 1 - e 1 = - 4i3 = (2i)(4i) = 8 69

_1

(e - e ) =

1 .

4 s1nh 1

physicsguide CSIR NET, GATE

Complex Analysis

Example: Find the residue of the following functions

11 oo : 03 : 13 @Sk J ahiruddin , 2020

Complex Analysis

Example: Find t he residue of t he following functions

(i)

f (z)



__

Z Sln Z

(z - 1r) 3

at z

= 1r

(ii) 2

ez / sin z at z = m 1r Solution : (i) We know that the residue at p ole of order m , situated at a

a_1 = lim ( z➔a

1

dm- 1

_1 )1 m - 1 . dzm

{ (z

- a)m f (z) }

As z sin z is finit e at 1r, t he order of pole is 3 or m ay be less t han 3. Now applying t he formula of residue a1

1 d2 1 = !dz 2 (z sin z ) lz=7r = [- zsin z +2 cos z]z=1r = - 1 2 2 2

(ii) ez/ sin z has double poles at z = 0, ± 1r, ±21r, . .. Residue at z

=

m1r is

1 d lim - z➔m1r 1! dz j [email protected]

@Sk J ahiruddin , 2020

ez (z -m1r)2. 2 Sln Z

70

physicsguide CSIR NET, GATE

Complex Analysis

00 : 03 : 15 [email protected]

70

physicsguide CSIR NET , GAT E

@Sk J a hiruddin , 2020

=

Complex Analysis

ez [(z - m 1r) 2 sin z + 2 (z - m 1r) sin z - 2 (z - m 1r) 2 cos z]

.

l1m - - - - - - - - - - - - - - - - - - - - - - -

sin3 z

z -+ni

Put z - m1r = u

a1

=

2

lim eu+ m1r

u sin u

+ 2u sin u -

2

2u cos u

3

sin u

u -+0

. u 2 sin u + 2u sin u - 2u 2 cos u 1Im - - - - - - -3 - - - - - 11,---+0 sin u T he limit can be evaluated by using L 'Hospital's r ule. First, note t hat

u3 lim - 3- = lim u-+0 sin u u-+0

u •

3

Sln U

= 1

So

+ 2u sin u - 2u cos u u ------------- . --2

2

u sin u

3

u3 sin 3 u 2 2 u sin u + 2u sin u - 2u cos u = em1r 1n1r . = e 1Im - - - - - - -3 - - - - - u -+0 u

Alternat ive method - By expanding in Laurent Series [email protected]

71

physicsguide CSIR NET , GAT E

00 : 03 : 18 Alternative method - By expanding in Laurent Series 71

j [email protected]

physicsguide CSIR NET 1 GATE

Complex Analysis

@Sk J a hiruddin , 2020

f (z) = ez sin z in a Laurent 2

In this method, we expand

1

= m1r and obtain t he coefficient of ( ) z-m1r as required residue. Let z = u + m1r . Then, the funct ion to be expanded in a Laurent series about u = 0 is

series about

z

sin ( m1r + u) = em1r eu / sin pansions for eu and sin u, we find em1r+u /

2

(1 u 2

m1r

e

u2

= so t he residue is

[email protected]

6

+

u.

Using t he series ex-

u4 120 -

...

)2

u2

l +u+?T+ ··· ~-

(1 -

em1r

u2

2

1 u2

u2

3

+ 2u4 + ... ) 45

1 +u

5 u +- +6 3

+ ...

em1r

72

physicsguide CSIR NET, GATE

11 oo : 03 : 20

[email protected]

72

physicsguide CSIR NET , GATE

@Sk J a hiruddin , 2020

2.6

Complex Analysis

Residue theorem

Suppose t he function f (z) has a pole of order m at the point z = zo, and so can be written as a Laurent series about zo of the form 00 (2.18)

n=-m Now consider t he integral I of f (z) around a closed contour C that encloses z = z 0 , but no other singular points. Using Cauchy 's t heorem, t his integral has t he same value as the integral around a circle ry of radius p centred on z = z 0 since f (z) is analytic in the region between C and ry. On the circle we have z = zo + p exp i0 (and dz = ip exp i0d0) , and so

f (z)dz

I = 'Y

00

n=-m 21r

00

i pn+l exp[i(n + 1)0]d0

n=-m For every t erm in the series with n =/= - 1, we have 27r . /

i

p

n+l

f "(

exp i n

+

1)0ld0

=

ri pn+l exp [i (n .1

.

.. \

+ 1)0]l 27r =

0

11 oo : 03 : 23 exp i n

+

0

n=-m

For every term in the series with n =/= - 1, we have 21r

o

i

. n+ l

p

[ .(

exp i n

+

l )0]d0

j [email protected]

. n+l

=

ip

73

[ .(

exp i .(

+ 1)0] n

in+ l

21r

)

-_ 0

0

physicsguide CSIR NET I GATE

@Sk J ahiruddin , 2020

Complex Analysis

but for the n = - l term we obtain 21r

id0 = 21ri 0

1

Therefore only t he term in (z - zo)- contributes to the value of the integral around 'Y ( and t herefore C ) , and I takes the value

I=

J(z)dz = 21ria_1

(2.19)

C

Thus the integral around any closed contour containing a single pole of general order m (or, by extension, an essential singularity) is equal to 21ri times the residue of f (z) at z =

zo If we extend t he above argument to the case where f (z) is cont inuous wit hin and on a closed contour C and analytic, except for a finite number of poles, within C, then we arrive at t he residue theorem (2.20)

11 oo : 03 : 26 except for a finite number of poles, within C, then we arrive at the residue theorem (2.20)

[email protected]

74

physicsguide CSIR NET ) GATE

Complex Analysis

@Sk Jahiruddin) 2020

, • ,-,, , -





,,

' • ''

, ,,

I I

• C

(a)

,

,,

,

/

.

'

'

'

'• ,I' , , , , ,,

,,

C'

• ''

'

--- -·-------

---

,,

(b)

Figure 2. 3: The contours used to prove the residue theorem: (a) the original contour; ( b) t he contracted contour encircling each of t he poles.

In figure 2. 3, (a) shows the original contour C referred to in t he residue theorem (2.20) and (b) shows a contour C' giving the same value to t he integral, b ecause f is analytic between C and C'. Now t he contribution to the C' integral from t he polygon (a triangle for t he case illustrated) joining the small circles is zero. since f is also analvtic inside C' . Hence the whole value

11 oo :03 : 2a between C and C' . Now t he contribution to t he C' integral from t he polygon (a triangle for t he case illustrated) joining the small circles is zero, since f is also analytic inside C'. Hence the whole value of t he integral comes from t he circles and, by result (2.19) each of t hese contributes 21ri times the residue at the pole it encloses. All t he circles are traversed in their positive sense if C is thus traversed and so t he residue theorem follows. [email protected]

75

physicsguide CSIR NET, GATE

Complex Analysis

@Sk J ahiruddin , 2020

Formally, Cauchy's theorem is a particular case of (2.20) in which C encloses no poles.

2.6.1

General expression of Integral around a pole in terms of residue

Finally we prove another important result , for later use. Suppose that f (z) has a simple pole at z = z0 and so may be expanded as the Laurent series

f (z) = cp(z) + a _1 (z -

zo)-

1

where cp(z) is analytic within some neighbourhood surrounding z0 . We wish to find an expression for t he integral I of f (z) along an open contour C, which is the arc of a circle of radius p centred on z = zo given by

IZ

-

Zn I

=

0.

(} ,


3 (d) z < l

(c) 0 < z

+l
1 3.2. P rove t hat if J (z) has a simple zero at z0 then 1/ J(z)

has residue 1/ J' (zo) there. Hence evaluat e

11

oo : 04: s4

for a real and > 1 3.2. Prove t hat if

f (z)

has residue 1/ f' (z0 )

has a simple zero at z0 t hen 1/ f (z) t here. Hence evaluate 1r

-7r

sin 0 dB a - sin 0

where a is real and > 1 3.3. P rove t h at, for a > 0, the integral 00

0

t sin at dt 1 + t2

has t he value (1r / 2) exp ( -a) 3.4. Prove t hat 00

0

cos mx dx=1f - m/2 -m 4 e -e 4x 4 + 5x 2 + 1 6

j [email protected]

for m> 0

physicsguide CSIR NET, GATE

106

@Sk J ahiruddin , 2020

Complex Analysis

3.5. Evaluate 00

o

3.6. Evaluate

dx x6

+1

00

2

cos (x )dx 0

3. 7. Show t hat the principal value of t he integral 00

cos (x /a) dx

- oo x2 -

is - (1r /a) sin 1

a2

11 oo : 04: s6 00

cos(x/a) dx

- oo x2 -

a2

is - (1r /a) sin 1 3.8. Using a suitable cut plane, prove t hat if a is real and

0 0, all t he condit ions for Jordan 's lemma to hold are satisfied and the integral 4

11 oo : os : 11 z+i z-i around a semicircular contour like figure 3.2 . 4 As f (z) z - as z --+ oo and m > 0, all the conditions for Jordan's lemma to hold are satisfied and the integral f"'--1

j [email protected]

114

physicsguide CSIR NET I GATE

@Sk J ahiruddin , 2020

Complex Analysis

around the large semicircle cont ributes not hing. For t his integrand there are two poles inside the contour, at z = i and at z = ~i The respective residues are - 2ie-m/ 2

e-m/2

e-m

6

2i3ii

and

3i

2

(-i) 2i 2

The residue theorem therefore reads . 00 eimx ie- m dx + 0 = 21ri 4 2 _ 00 4x + 5x + 1 6

3

2ie- m/ 2

3

and the stated result follows from equating real parts and changing the lower integration limit, recognising that the integrand is symmetric about x = 0 and so the integral from O to oo is equal to half of t hat from -oo to oo. Solution: 3.5. We consider c

closed contour. Here, z 6

x

dx , where C is the 6

+1

+ 1 = 0 gives us the poles of t he function.

That are,

These are simple poles, amongst t hem only

00: 05: 20

That are,

These are simple poles, amongst t hem only • 1r

·

31r

· 57r

z = e 6 e e; e e; ' ' 2

2

2

lie inside t he contour.

[email protected]

physicsguide CSIR NET) GAT E

115

Complex Analysis

@Sk J ahiruddin ) 2020

Residue at

Similarly, Residue at .1m [- 1 ] 1 - 31r 6z 5 z ➔e 6

1

= -e

- i

151r

6

i

6

Residue at

So,

dz C

= =

'Tri

3 'Tri

3

[cos( [cos(

51r

6 51r

6

z6

+1

) +i sin( .

)-is1n(

51r

6 57r

1 ·51r = 21ri-[e-ie;

6

) + cos(

151r

6 1r

· 151r

+ e-1.

) +i sin( .

1r

151r

6

6

· 251r

+ e-i

) +cos( 57r

6 ]

251r

6

) +i sin( .

57r

251r

6

)]

27r

)+cos( )-isin( )-cos( )+is1n( )] = 2 2 6 6 6 3

11 oo : os : 23 = =

wi

3 wi

3

[cos( [cos(

5w 6

5w 6

) +i sin (

)-isin(

- oo

6

5w 6

dx

00

So,

5w

x6 + 1

) + cos(

15w

15w

6

6

) +i sin(

) +cos(

25w 6

) +i sin(

w w 5w 5w )+cos( )-i sin( )-cos( )+i sin( )] 2

2

6

dx

00

21r 3

6

o x6 + 1

25w 6 =

)]

2w 3

7r

3

Solution: 3.6. [email protected]

116

physicsguide CSIR NET, GATE

Complex Analysis

@Sk J ahiruddin, 2020

Consider t he contour drawn in t he figure. First con. 2

sider t he complex integral

e iz

dz . There is no pole in the

C

contour. So the value of t he integral would be zero. Now

y B

C

0

A

R

X

breaking t he integral along the lines of t he contour . . 2

e

2

z

dz

. 2

+

OA

eiz

AB

dz

+

e

. 2

2

z

dz

=0

BO

Now on OA , z = x( from x = 0 to x = R); on AB , z = R ei 0 ( from 0 = 0 to 0 = 1r / 4); on BO , z = re1ri/4 (from r = R to r = 0). Hence

00: 05: 25

Now on OA , z = x( from x = 0 to x = R); on AB , z = R ei8 ( from 0 = 0 to 0 = 1r / 4); on BO , z = re1ri/4 (from r = R t o r = 0). Hen ce

R

. 2

e ix

dx

1r/4

+

. i R2 e2iB

e

R i0 d0 i e .

0

+

e

1ri/4d 0 e r =

R

0

0

ir2e1ri/2

From h ere j [email protected]

117

physicsguide CSIR NET, GATE

Complex Analysis

@Sk J ahiruddin , 2020

R

( cos x

2

+ i sin x

2

)

dx

0 R

1r/4

2

e-r dr -

eiR2 cos 20- R2 sin 20 i R ei0 dB

0

0

First integral b ecomes oo

e1ri/4

e

- r2 d

r =

0



,Jir 1ri/ 4 1 - -e = 2

11"

'l

-2 + -2

2

11"

2

Second int egral b ecomes

1r/4

2

e'iR

2

cos 20- R sin 20 i R ei0 d0

7r

< 0

0

R f 1r12

0 2 ..,; .... ,J.. • •

/4

2

e- R

sin20 Rd0

11 oo : os : 2a 7i/4

eiR2 cos 20-R2 sin 20 i R ei0 d0

7i/ 4


.. , µ are constant real numbers. Show that C is an isotropic tensor. 3.12. Consider a coordinate syst em (u, v, w)

x=vw

'

'y

= uw,

Z

= UV

0 btain the metric in the new coordinate system (u , v, w) . 3.2.1

Ans Keys 2

3.3 (a) Principal moment of intertia's are 20ma and (20

2v'5)ma

2

±

11 oo : 02 : 33 3.2.1

Ans Keys

3.3 (a) Principal moment of intertia's are 20ma2 and (20 ±

2v'5)ma

2

(b)

2

1

- 1

- 1

2

-2 v'5

'

0

'

v'5

3.5 (a) 2

ds = du dA

2

+ h~dv

, hu = 1, hv = 112 2 v ) - dudv, ds = 2 112 j ( 2v - v ) , 2 112 v) j ( 1 - v)

u (2v -

=

2

eu = i (1 - v) + ev = - i ( 2v + au= eu = au, av = hvev, [email protected]

2

u (2v - v

)-

112

+ hvevdv

eudu

av = ev/ hv

55

physicsguide CSIR NET I GATE

@Sk J ahiruddin , 2020

Advanced Topics

(b) m

u-

aV = Fu au

uv2

v(2 - v)

uv 2(v - 1)

uii + 2uv m

[v(2 -v)]1/ 2

= -u-

1

[v(2 -

+ [v(2 -v)] 3/ 2

1 2 aV v)] l - =

Fv

av

(c) v u = euaU/ au

+ evu-

1

1

J v(2 - v) aU/ av

1

V. V = u - a (uVu) / au+ u - J v(2 - v) aVv / av n 2u v

=

! Aa 11,

11.

au uA 11.

1 ✓,----- a v(2 -v) A 2

+ 11.

V

✓,---- au

v(2 -v) ,,V ~

11 oo : 02 : 36 1

VU = euaU/ au + evu- J v(2 - v)au/ av 1 1 V. V = u- a (uVu) ; au+ u- J v(2 - v) a-vv; av n 2u V

=! a

au 1 ✓~-- a a Ua + v(2 -v) a 2 u u u u V

(d) u - 1 ,

l k u '

✓~-- au

v(2 -v) ;::i

uV

0

3.10

T'=

[email protected]

1 0 0 0 2 0 0 0 4

56

@Sk J ahiruddin ) 2020

4 4.1

physicsguide CSIR NET ) GATE

Advanced Topics

Green's function Basic reading and solved examples

This is again an advanced topic. I'm guiding you step by step to understand the topic. First, you need to underst and Laplace Transformation, Dirac Delta function and How to to solve differential equation by Laplace t ransform ation when t here is a delta function in t he RHS of t he equation. For

11 oo : 02 : 38 1s 1s again an a vance op1c. step to understand the topic. First, you need to understand Laplace Transformation, Dirac Delta function and How to to solve differential equation by Laplace t ransform ation when t here is a delta function in the RHS of the equation. For example you need to be able to solve differential equation like this

Now proceed to Boas Chapter 8, Section 12, A Brief Introduction to Green 's Functions. The section is brief, t here are only four pages. You need to understand the examples.

I'm writing t he general procedure to solve an equation by Green 's function. You will be given a general equat ion

d dx

dy p (x) dx

[email protected]

@Sk J ahiruddin, 2020

+ q(x )y = f (x) 57

physicsguide CSIR NET, GATE

Advanced Topics

This equation is to be satisfied on t he range within a boundary x 1 < x < x 2 . Boundary condit ion will be given.

Our Green 's function G(x, x') will be the solution of the equation

d dx

dG(x, x' ) ( ) p x dx

+ q(x)G(x, x') = b(x -

x')

11

00:02:41

Our Green's function G(x, x') will be the solution of the equation d dG(x, x' ) ( ) dx p x dx

+ q(x )G(x, x') = 6(x -

x')

G(x, x') needs to satisfy that boundary conditions. General solution of the equation will be X2

y(x) =

G(x.x')J (x') dx' X1

Getting the Green's funct ion may not be so easy. I'm giving you step by step procedure. Step - 1: Identify p(x) from t he given equation. Step - 2: Get the boundary condition (it will be given) Step - 3: Find t he solution of the equation (homogeneous part) d dy dx p(x) dx + q(x) y = O Step - 4: You get the solution as y(x) . Now write the same function as two different functions: y1( x) and Y2 (x) . j [email protected]

physicsguide CSIR NET, GATE

58

@Sk J ahiruddin , 2020

Advanced Topics

Step - 5: Apply given two boundary conditions on t hese two functions y 1 ( x) and y 2 ( x) and get two solutions. Ignore any common constants. Step - 6: Find G(x,x') by using (

.

I

'.

I

I '.

I

11 oo : 02 : 43 two functions y 1 x and y 2 x and get two solutions. Ignore any common constants. Step - 6: Find G (x,x') by using

G(x, x') =

Ay1 (x )y2 (x' ), Ay2 (x)y1(x'),

X X

< >

X X

1

' 1

(4.1)

'

Where

Step - 7: After you get the Green 's function you can easily get the general solution of t he equation by using X2

y(x) =

G(x .x') f (x')dx' X

= A Y2(x)

X2

Y1(x')j(x')dx'

+ Y1(x)

Y2(x')f(x')dx' X

Now follow t he next example carefully to find t he Green 's function.

Example 4.1. Consider t he ODE -~:;= f (x). W hat will be a suitable Green 's function for the equation with t he [email protected]

59

@Sk J ahiruddin, 2020

physicsguide CSIR NET, GATE

Advanced Topics

boundary condition y(O) = y( l ) = O? Solution: We are trying to find t he Green 's function for t he equation t-!

I

t-!,, ,\

11

00:02:47

boundary condition y(O) = y( l ) = O? Solution: We are trying to find t he Green 's function for t he equation

d~

as

d~

(- )

j~

=

p(x) :;

f (x) .

+ q(x)y = f( x)

So p( x) = ( - 1).

First we need to make solution of the homogeneous part

d:y

which is = 0. The general solut ion of this equation dx can be written as y = a+ bx, where a and bare constants. Now we need to apply t he boundary condit ions on the general solution t o get two separate solutions. let me explain in more details. Lets take Y1 (x) = c1 + c2x and Y2(x) = d1 + d2x . Bot h of Y1 and y 2 are solut ions of the homogeneous part. We will not apply both of t he boundary conditions on a single equation. R ather we will use one Boundary condit ion on one solution. Let 's say we apply y(O) = 0 on Y1 = c1 + c2x . As y(O) = 0 c1 must be zero. y 1 = c2 x. While constructing Green 's funct ion we ignore the common constant , so we get one solution [email protected]

@Sk J ahiruddin , 2020

Yl = x .

60

physicsguide CSIR NET, GATE

Advanced Topics

11 oo : 02 : 49 @Sk J ahiruddin, 2020

Advanced Topics

Yl = x .

Now we apply y(l) = 0 on y2 = d1 + d2x. As y( l ) = 0 d1 + d2 = 0. So y 2 = d1 - d 1 x . As mentioned before while constructing Green 's function we ignore the common constant, so we get t he other solut ion y 2 = 1 - x.

G(x, x') =

Ay1 (x )y2(x'), Ay2(x)y1(x'),

X

X

< >

X

X

1

' 1

(4.2)

'

Where

A=

l p( x') [y; (x')y1 (x') - y~ (x')y2 (x')]

Here, p(x') = - 1, So 1

A=--------= l - 1[- 1 X X - l X (1 - x)] Hence

G(x , x' ) =

x( l - x'),

0 < x < x',

(1 - x)x',

x' < x < l ,

(4.3)

Example 4.2. Find the solution of the equation

j [email protected]

@Sk J ahiruddin , 2020

61

physicsguide CSIR NET, GATE

Advanced Topics

11 oo : 02 : s1 61

[email protected]

physicsguide CSIR NET, GATE

@Sk J ahiruddin , 2020

Advanced Topics

(k is a constant) Solution: We have already got the Green 's function for t he general equation - ~;; = f (x) in t he previous problem

G(x , x') =

x( l - x'),

0 < x < x' ,

(4.4)

(1 - x)x' , x' < x < l ,

So we can directly formulate the solution X2

y(x) =

G(x .x') J( x')dx'

1

G(x.x') sin(kx')dx' 0

1

X

Y1(x') sin(kx')dx' + Y1(x)

= A y2(x) 0

X

1

X

x' sin(kx')dx' + x

= (1 - x)

Y2(x' ) sin(kx')dx'

0

(1 - x') sin(kx')dx' X

Example 4.3. Find the solution of the equation

d2y

- 2 = coskx d x

(k is a constant) Example 4.4. Construct Green's function for the differential equation

d2y 2 dx2 +w y = J(x) jahir@physicsguide. in

62

physicsguide CSIR NET, GATE

11 oo : 02 : s4 d y dx2

+w

2

y

(

=f x

62

[email protected]

)

physicsguide CSIR NET, GATE

@Sk J ahirudd in, 2020

Advanced Topics

given t he boundary condit ion y(O) = y( 1r /2) = 0 Solution: We are trying to find the Green 's function for the equation d dx

dy p (x) dx

d2y Our equation is d2 x

equation as d~

:~

+w

+w

2

+ q (x )y = f (x) 2

y

y =

= f (x) . We can write the j(x) . So p(x)

2

= 1, q(x) = w .

First we need to make solut ion of t he homogeneous part which is ~;; +w 2y = 0. T he general solution of this equation can be written as y = a sin x + b cos x, where a and b are constants. Now we need to apply the boundary conditions on the general solution to get two separate solut ions. Lets take YI (x) d2 cos x.

= CI

sin x

+ c2 cos x

and y2(x)

=

dI sin x

+

Let's say we apply y(O) = 0 on YI (x) = c I sin x + c2 cos x . As y(O) = 0 c2 must be zero. YI = cI sin x. While constructing Green 's function we ignore t he common constant , so we get one solution YI

= sin x .

Now we apply y(1r / 2) = 0 on y 2 (x) = dI sin x + d 2 cos x. j [email protected]

63

physicsguide CSIR NET, GATE

11 oo : 02 : s6 get one solut ion y 1

= sin x .

Now we apply y(1r/2) = 0 on y 2 (x) = d 1 sin x [email protected]

63

+ d2 cosx.

physicsguide CSIR NET, GATE

@Sk J ahiruddin , 2020

Advanced Topics

As y(1r / 2) = 0 d1 = 0. So y2 = d2 cos x . As mentioned before while constructing Green 's function we ignore the common const ant, so we get t he ot her solution y 2

G(x, x' ) =

A sin (x) cos (x'), A cos(x) sin(x' ),

X X

< >

X X

= cos x .

1

'

(4. 5)

1

'

Where

A= Here, p(x')

A=

1 p(x') [y~ (x')y1(x') -

Yi (x' )y2(x')]

= 1, So l

1 [- sin x' x sin x' - cos x' x cos x']

=

-1

Hence

G(x, x') =

- sin(x) cos(x'),

0 < x < x',

- cos(x) sin(x'),

x' < x ::; 1r / 2,

(4.6)

Example 4 .5. Solve

d2y 2 dx 2 +w y = cosec(x) by using Green 's function. Sol11tion: W P- 8.lrP-Fl,.. ,T ) =

81rhc )..5

1 ehc/>.kT _ 1

(2.7)

total energy density which is expressed in terms of Stefan- Boltzmann's total power per unit surface area is 00

81rh u(v, T )dv = 3 o c

00

O

) W:

hv = W + K

(2.9)

where K represents the kinetic energy of the electron leaving t he material

11 oo : oo : 29 hv = W + K

(2.9)

where K represents the kinetic energy of the electron leaving t he material The above equation gives the proper explanation to the experimental observation that the kinetic energy of t he ejected electron increases linearly with the incident frequency v,

K = hv - W = h (v - vo)

(2.10)

where v 0 = W / h is called the threshold or cutoff frequency of the metal. Moreover , this relation shows clearly why no electron can be ejected from t he metal unless v > v 0 : since the kinetic energy cannot be negative, the photoelectric effect cannot occur when v < v 0 regardless of the intensity of t he radiation. The ejected electrons acquire t heir kinetic energy from t he excess energ)' h (v - v 0 ) supplied by t he incident radiation. [email protected]

@Sk J ahiruddin, 2020

2.3

11

Physicsguide

Old Quantum Theory

Compton effect

Compton found that when X-rays are make incident on free electrons, the wavelength of t he scattered radiation is larger t han the wavelength of the incident radiation. How do we explain it? According to classical physics, the incident and scattered radiation should have the same wavelength. since

11 oo : oo : 32 Compton found that when X-rays are make incident on free electrons, the wavelength of t he scattered radiation is larger t han the wavelength of t he incident radiation. How do we explain it? According to classical physics, t he incident and scatt ered radiation should have the same wavelength. since t he energy of the X-ray radiation is too high to be absorbed by a free electron, the incident X-ray would then provide an oscillatory electric field which sets t he electron into oscillatory motion, hence making it radiate light with t he same wavelength as you have studied in radiation chapter in ED course. The experimental findings of Compton reveal that t he wavelength of the scattered X-radiation increases by an amount ~A , called the wavelength shift, and t hat ~A depends not on t he intensity of the incident radiation, but only on t he scattering angle. If we t reat t he incident radiation as a stream of particles which are named as photons - colliding elastically with individual electrons. In t hat scattering process, we use the laws of elastic collisio11s - t he conservation of energy and 12

[email protected]

@Sk J ahiruddin , 2020

Physicsguide

Old Quantum Theory

moment um. You can derive t he Compton scatt ering formula (hence . I ' m not der1v1ng . . .......''') ,\

\

\ I

\

h

i1

In

1 1 \

11 oo : oo : 34 momentum. You can derive the Compton scattering formula (hence .. I ' m notd er1v1ng . . .... ..''')

.6A = A

1

-

A= h (l - cos 0) = 2Acsin

2

ffieC

2.4

e2

(2.11)

Pair production

When high-frequency electromagnetic radiation passes through a foil, individual photons of this radiat ion disappear by producing a pair of particles consisting of an electron, e- , and a positron, e+ : photon This process is called pair production

j [email protected]

13

Physicsguide

Old Quantum Theory

@Sk J ahiruddin , 2020

e

E=hw

------Incoming



ffi

11 oo : oo : 36

e

E=hw

------Incoming photon



EB

Nucleus

Figure 2.2: Pair production: a highly energetic photon, interacting with a nucleus, disappears and p1~oduces an elect ron and a posit ron

Due to charge, momentum, and energy conservation, pair production cannot occur in empty space. For the process photon ➔ e- + e+ to occur, the photon must interact with an external field such as the Coulomb field of an atomic nucleus to absorb some of its momentum. In t he figure you see that an electron-positron pair is produced when the photon comes near (interacts with) a nucleus at rest; energy conservation dictates that

nw =

+ E e+ + E N 2 2 = (mec + ke-) + (mec + ke+) + K N Ee-

:::'. 2mec

2

(2.12)

+ ke- + ke+ 2

where hw is the energy of the incident photon, 2mec is the [email protected]

@Sk J ahiruddin, 2020

14

P hysicsguide

Old Qt1antum Theory

sum of the rest masses of t he electron and positron, and

11 oo : oo : 39

@Sk J ahiruddin, 2020

Old Quantum Theory

sum of the rest masses of t he electron and positron, and ke- and ke+ are the kinetic energies of the electron and posit ron, respectively. As for EN = K N, it represents the recoil energy of the nucleus which is purely kinetic. since t he nucleus is very massive compared to t he electron and t he positron, K N can be neglected to a good approximation. Note the photon cannot produce an electron or a a positron alone, for electric charge would not be conserved. Also, a massive object, such as the nucleus, must participate in t he process to t ake away some of t he photon's momentum. The minimum energy E min of a photon required to produce an electron-posit ron pair must be equal to the sum of rest mass energies of the electron and positron; t his corresponds to t he case where the l.. t he momentum of t his particle will be rather high. Formally, this means that if a particle is accurately localized (~ x -+ 0) , there will be total uncertainty about its moment um (i.e., ~Px -+ oo ) . To summarize, since all quantum phenomena are described by waves, we have no choice but to accept limits on our ability to measure simultaneously

11 oo : oo : s1 mally, this means that if a particle is accurately localized (~ x ➔ O) , t here will be total uncertainty about its moment um (i.e., ~ Px ➔ oo ) . To summarize, since all quantum phenomena are described by waves, we have no choice but to accept limits on our ability to measure simultaneously any two complementary variables. Heisenberg's uncertainty principle can be generalized to any pair of complementary, or canonically conjugate, dynamical variables: it is impossible to devise an experiment t hat can measure simultaneously two complementary variables to arbitrary accuracy (if this were ever achieved, the j [email protected]

21

@Sk J ahiruddin, 2020

Physicsguide

Old Quant11m Theory

t heory of quantum mechanics would collapse) . Energy and t ime, for inst ance, form a pair of complementary variables. Their simultaneous measurement must obey t he time-energy uncertainty relation:

n ~E~t -> -2

(4.2)

This relation states that if we make two measurements of the energy of a system and if t hese measurements are separated by a time interval ~ t, t he measured energies will differ by an amount ~E which can in no way be smaller than n/ ~t . If t he time interval between the two measurements is large, the energy difference will be small. We also have the ane::ular momentum and ane::le uncer-

11 oo : 01 : oo differ by an amount ~E which can in no way be smaller than Ii/ ~t . If t he time interval between the two measurements is large, the energy difference will be small. We also have the angular moment um and angle uncertainty

(4.3)

[email protected]

@Sk J ahiruddin , 2020

5

22

P hysicsguide

Old Quantum Theory

Bohr's Model

According to Rutherford the positive charges of an atom is centered in very small spaces, called nucleus, and t he negative charges are orbiting around the nucleus just as do planets do orbit around the sun. The main problem in t his model is according t he classical electromagnetics the negat ive charges do emit radiation as the are charged particle accelerating with acceleration v2 /r, so after some time the electrons will exhaust its energy and collapse on the nucleus. Bohr proposes solution to this problem by taking some

11 oo : 01

: 02

accelerating with acceleration v2 / r, so after some time the electrons will exhaust its energy and collapse on the nucleus. Bohr proposes solution to this problem by taking some extra assumptions. (1) Instead of a contimunm of orbits, which are possible in classical mechanics, only a dis- crete set of circular stable orbits, called stationary states, are allowed. Atoms can exist only in certain stable states with definite energies:

E1, E2, E3, etc. (2) The allowed (stationary) orbits correspond to those for which t he orbital angular momentum of t he electron is an integer multiple of n(n = h/21r) :

23

[email protected]

@Sk J ahiruddin, 2020

P hysicsguide

Old Quantum Theory

L

=nn

(5.1)

This relation is known as t he Bohr quantization rule of t he angular momentum (3) As long as an electron remains in a stationary orbit , it does not radiate electromagnetic energy. Emission or absorption of radiation can take place only when an electron jumps from one allowed orbit to another. The radiation corresponding to the electron's transition from an orbit of

11 oo : 01 : os 3 As long as an electron remains in a stationary orbit , it does not radiate electromagnetic energy. Emission or absorption of radiation can take place only when an electron jumps from one allowed orbit to another. The radiation corresponding to t he electron 's transition from an orbit of energy E n to another E m is carried out by a photon of energy

(5.2) So an atom may emit (or absorb) radiation by having t he electron jump to a lower (or higher) orbit. Now we use

e2

v2

- - - = m e47rcor2

(5.3)

r

and

nn

L = m evr = j [email protected]

(5.4) Physicsguide

24

@Sk J ahiruddin, 2020

Old Quantum Theory

to get

(5.5) where

47rcon

2

ao = - -

(5.6)

m ee2

is the Bohr radius, a0 = 0.053nm. The speed of the orbiting electron can also be derived

nn

(

2

e

~ 1

11 oo : 01

: 01 (5.6)

ao = is the Bohr radius, a0

=

m ee 2 0.053nm. The sp eed of t he orbiting

electron can also b e derived e2

nfi

Vn=--

1

(5.7)

41rco nfi

m ern

The ratio b etween the sp eed of the electron in the first Bohr orbit , v 1 , and the sp eed of light is equal to a dimensionless constant a, known as the fine structure constant: e2 41rc 0 fie 1

C

1

(5.8)

137

Total energy of electrons

E=

1

1 e2

2

(5.9)

-meV - - - -

41rc0 r

2

The kinetic energy, ~ m ev 2 , is equal to ~ e 2 / ( 41rc 0 r) Hence e2

1 E =-2 [email protected]

(5.10)

41rcor 25

Physicsguide

@Sk J a hiruddin, 2020

Old Q t1antum Theory

The total energy t h en , in quantized form m e

e2

2/i2

41rc 0

2

1

n2

R n2

(5.11)

known as t he Bohr energy, where R is the Rydberg constant:

11 oo : 01

: 1o

2

41rEo

2/i

n

2

n2

(5.11)

known as t he Bohr energy, where R is the Rydberg constant:

R

=

2

me 2n2

= 13.6eV

(5.12)

In deriving t he energy expression we have neglected the mass of the proton. If we include it, the expressions become 2

41rEori rn = µe2

E - - µ n 2/i2

1 + m e a 0n 2 , mp 2 1 1

n2 e2

n2

47rEo

R

1 + m e/m p n

(5.13)

2

where µ = mpme/ (mp + me) = m e/ (1 + m e/mp) is the reduced mass of the proton-electron system. We should note t hat rn and En, which were derived for t he hydrogen atom, can be gene1~alized to hydrogen-like ions where all electrons save one are removed. To obtain the radius and energy of a single electron orbiting a fixed nucleus 26

[email protected]

P hysicsguide

@Sk J ahiruddin , 2020

Old Quantum Theory

of Z protons, we need simply to replace e2 in (1.75) by Z e2

'

I_

m f'. \ ao

'>

z2

11 oo : 01

: 13

radius and energy of a single electron orbiting a fixed nucleus j [email protected]

26

@Sk J ahiruddin , 2020

Physicsguide

Old Quantum Theory

of Z protons, we need simply to replace e2 in (1.75) by Ze 2

'

m e ao

1 +M

2

z n,

Z2 R En = - - - - - -2 1 + m e/ M n

(5.14)

where M is the mass of the nucleus; when m e/ M we can just drop t he term m e/ M

j [email protected]

27

(x) is given by (1jJ, c/> ) =

'lj; * ( x) c/> (x) dx

(1.3)

In quantum mechanics we deal with square integrable vvave functions so t he integral converges. A function 1j; (x) is said tn h P ~n,,~rP intPo-r~hl P if th P ~r~l~ r nrnrl11rt. nf 1/, "\7iTith it~Plf

11 oo : oo : 29 ('lj; , cp) =

'lj; * ( x) cp (x) dx

(1.3)

In quantum mechanics we deal with square integrable wave functions so t he integral converges. A function 'lj; (x) is said to b e square integrable if t he scalar product of 'lj; wit h itself, 1.e 2 'lj;(x) dx = finite (1.4) ('lj; ' 'lj; ) = •

The linear vector space which contains square integrable funct ions and a posit ive inner product is a Hilbert space. We must note t hat the dimension of the Hilbert space of squareintegrable functions is infinite, since each wave function can be expanded in t erms of an infinite number of linea1~1y independent functions. The dimension of a space is given by t he maximum number of linearly independent basis vectors required to span t hat space.

[email protected]

@Sk J ahiruddin, 2020

2

11

Physicsguide

Jvi athematical Background

Dirac Notation: Bras and Kets

All the information about a syst em is stored in a stat e vector. That is called a ket. Kets belong to Hilbert space. There exits some elements which give real or complex numbers, while taking inner products with t he ket vetors.

11 oo : oo : 31 All the information about a system is stored in a stat e vector. That is called a ket. Kets belong to Hilbert space. There exit s some elements which give real or complex numbers, while taking inner products with t he ket vetors. These elements are said to belong to bra space. For every ket l'l/J) there exists a unique bra ('lf; . The scalar product (cp, 'lj;) is denoted by the bra-ket (cp 'lj;)

- >

(2.1)

(c/> 'lp)

The wave functions are basically t he projections of ket vectors on posit ion or momentum axis.

(r, t 'lfJ) = 'l/J (r, t)

(2.2)

In the coordinate representation , the scalar product (cp 'lj;) is given by ( c/>

'lj;) =

c/> * (

r, t )'lj; (r, t )d r 3

(2.3)

The momentum space wave function is t hen [email protected]

12

@Sk J ahiruddin , 2020

P hysicsguide

Nl athem atical Backg1·ound

(p, t 'lj;) = 'lj; (p, t )

2.1

Properties of Bra and kets

(2. 4)

11 oo : oo : 34 (p, t 7P) = 7P (p, t )

2.1

(2.4)

Properties of Bra and kets

(1) To every ket 1/;) , there corresponds a unique bra (1/;

(2. 5) (2) There is a one-to-one correspondence between bras and kets: a 7/J) + b q;) < > a* (7/J +b*(

(3.4)

or

V1

a11

a12

• • •

a1n

V1

V2

a ??

--

a22



• •

a2n

V2

• • •

• • •



• • •

• • •

>

• • •

Vn

V





• • •

Vn

(3.5) V

11 oo : oo : 55 >





• •







• •





• •



(3.5)

• • •



V

V

On the tensor product space, the same matrix can still act on the vectors, so t hat iJ 1--t Ail, but 1--t unt ouched . This matrix is written as A @ I , where I is t he identity mat rix. In the previous example of n = 2 and m = 3 ,

w w

t he matrix A is two-by-two, while A @ I is six-by-six, [email protected]

20

Physicsguide

Mathematical Background

@Sk J ahiruddin , 2020

A @I =

a11

0

0 0

a 11

a21

0 0

0 0

0 0 a11

a 21

0 0

0

a 21

a1 2

0

0 0

a12

a 22

0 0

0 0

0 0 a12

a 22

0 0

0

a22

(3.6)

The reason why this expression is appropriat e is clear once you act it on iJ @ w,

(A @I )(iJ@w) =

a 11

0

0 0

a 11

a21

0 0

0

a 21

0 0 a 11

0 0

a12

0

0 0

a12

a22

0 0

0

a22

0 0

V1W1

a12

V1W3

0 0

V2W1

V1W2

V2W2

11 oo : oo : ss

(A ® I )(v ® w) =

0 0

a1 1

0

0 0

a12

0

V1W2

0

a1 1

0

a 12

V1W3

a21

0

a22

0

0

a22

0 0

V2W1

a21

0 0

0 0

0

a 21

0

0

a22

V2W3

+ a12V2) W1 ( a11 v 1 + a12v 2) w2 ( a11 V1 + a 12V2) W3 ( a21 V 1 + a22V2) W1 ( a21 V 1 + a22V2) W2 ( a21 V 1 + a22V2) W3

V2W2

( a11 V1

j [email protected]

= (Av) ® w

(3.7)

Physicsguide

21

@Sk J ahiruddin , 2020

:Nl athematical Background

Clearly the matrix A acts only on w E W untouched.

v

E V and leaves

Similarly, the matrix B : W ➔ W maps w can also act on V ® W as I ® B , where

l ®B =

It acts on

b11

b 12

b13

0

0

0

b21

b22

b23

0

b31

b32

b33

0

0 0

0 0

0 0

0 0

0 0

b11

b12

b13

b21

b22

b23

0

0

0

b31

b32

b33

v ® w as

H

Bw. It

(3.8)

11 oo : 01 : oo 0 0

0 0

0 0

b11

b12

b13

b21

b22

b23

b31

b32

b33

0 0 0

0 0 0

0 0 0

It acts on v © was

b21

(I 0 B )(v 0 w) =

[email protected]

b 22

b 23

b31

b32

b33

0 0 0

0 0 0

0 0 0

b11

b1 2

V1W1 V1W2 V1W3

V2W1

b13

V2W2

b21

b22

b23 V 2W3

b31

22

@Sk J ahiruddin , 2020

+ b12 W2 + b13W3) V1 ( b21 W 1 + b 22W2 + b23W3) V1 ( b31 W1 + b32W2 + b33W3) V2 ( b11 W1 + b12W2 + b 13W3)

b32

b33

P hysicsguide

Jvi athematical Background

V1 ( b11 W 1

= v © (Bw)

(3.9)

+ b22W2 + b23W3) V2 ( b31 W 1 + b32W2 + b33W3) V2 ( b21 W1

In general, (A © I )(v ©w) = (Av) © w, and (I © B )(v© w) = v0 (B w) If you have two matrices, their multiplications are done on each vector space separately,

11 oo : 01

: 02

w) = v 0 (Bw) If you have two matrices, t heir multiplications are done on each vector space separately,

(A1

0 I ) (A2 0 I ) =

(A1A2)

0 I

(J 0 B 1) (I 0 B 2) = I 0 (B1B2) (A 0 J)(J 0 B ) = (I 0 B )(A 0 I ) = (A 0 B )

(3.10)

T he last expression allows us to write out A 0 B explicitly,

[email protected]

23

Nl athematical Backg1·ound

@Sk J ahiruddin, 2020

A 0 B=

P hysicsguide

a11b11

a11b12

a 11 b13

a 12 b 11

a12 b1 2

a12b13

a11b21

a1 1b22

a 11 b23

a12b21

a 12b22

a12b23

a11 b31

a11b32

a 11 b33

a12b31

a12b32

a12b33

a21b11

a2 1b12

a21 b13

a22b11

a22b12

a22b13

a21 b21

a21b22

a21 b23

a22 b21

a22b22

a22b23

a21 b31

a2 1b32

a21 b33

a22b31

a 22 b32

a22b33

(3.11)

It is easy t o verify t hat (A 0 B )(i10 w) = (Ail) 0 (Bw)

11 oo : 01 : os 22 23

a21b31

a21b32

a21b33

a 22b31

a22b32 a22b33

(3.1 1)

It is easy to verify t hat (A 0 B )(v0 w)

= (Av) 0 (B w)

Not e t hat not every matrix on V 0 W can be written as a t ensor product of a matrix on V and anot her on W . Ot her useful formul ae are

det (A 0 B ) = (

p2= f>

'

(4.14)

,,.,

The unit operator I is a simple ex ample of a projection ,,., ,,., operator, since J t = I , j2 = j A

The product of two commuting projection operators, P 1 ,,.,

and P 2 , is also a projection operator, since

(4.15) and

The surn of two projection operators is generally not a proj ection operator.

[email protected]

30

Physicsguide

11 oo : 01

: 23

projection operator.

j [email protected]

Physicsguide

30

lVIathem atical Background

@Sk J ahiruddin, 2020

T wo proj ection operators are said to be orthogonal if t heir product is zero "

"

"

For a sum of projection operators P 1 + P2 + P3 + · · · to be a projection operator, it is necessary and sufficient that t hese projection operators be mut ually orthogonal (i.e., the cross-product terms must vanish).

4. 7

Commutator of operators "'

"'

The commutator of two operators A and B , denoted by [A, B] , is defined by A

A

AA

AA

[A , B ] = AB - BA

(4.16)

and t he antiCommutator A

A

A

A

A

A

{ A , B} = AB + BA

(4.17)

Two operators are said to commute if t heir commutator is equal to zero and hence AB = BA . A

A

A

A

Any operator commut es with itself: "'

"'

[A, A] = 0

(4.18)

11 oo : 01 A

: 26

A

A

A

equal to zero and hence A B = B A . Any operator commutes with itself: (4.18) 31

[email protected]

P hysicsguide

@Sk J ahiruddin , 2020

Mathematical Background

If two operators are Hermit ian and t heir product is also Hermit ian, these operators commute: (4.19)

and since (A.B)t

=

AB we have AB = BA

The basic commutation relation between position and moment um operators A

A

A

in,

X , Px =

A

in,

Y , Py =

(4.20)

Some propert ies:

A

A

A

A

[A, B] = - [B , A]

(4.21)

[A., B + 6 + b + ... ] = [A., B] + [A., 6] + [A., b ] + . . . (4.22) [A., B]t = E t, At A

A

A

A

A

A

A

(4.23) A

A

+ B[A , C] [AB ,6] = A[B,6] + [A, C]B [A , BC]

=

[A , B ]C

J acobi ident ity r

A

..

A

r .....,,

A

~, ,

A

r .....,,

A

r ,,,-..,

A

..

11

A

r ,,,,.........

r

A

,.

A

T""""I, , ,

(4.24)

11 oo : 01

: 29

[A, BC] = [A, B]c + B[A, c ] [AB, 6] = A[B, 6] + [A, C]B

(4.24)

J acobi ident ity A

A.

A

A

A

A

[A , [B ,C]] + [B, [C,A]]

A

A

A

+ [C, [A ,B]] = 0

32

j [email protected]

Physicsguide

:Nl athematical Background

@Sk J ahiruddin , 2020

4.8

(4.25)

Uncertainty Relation between Two Operators

Let (A) and (B) denote t he expectation values of two Hermit ian operators A and B with respect to a normalized state vector 'l/J) = ('l/J A 'l/J) and (B) = ('l/JIB 'l/J) Introducing the operators ~ A and ~B , A

A

A

A

~A = A -

(A),

we have (~ A.)2 = A.2 - 2A(A) 2-8 (B) + (-8) 2, and hence

~B = B + (A.) 2

(B)

(4.26)

and (~ B) 2

=

-8 2

-

(4.27) and

(~ -8)2 =

_a2 -

(-8)2

where A. = 'ljJ A 'ljJ and -8 = uncertainties ~ A and ~ B are defined by 2

2

A

,1

_

flrA

-1 \? \ _

2

f I :i?\

'ljJ

A

/

(4.28)

,1 \

?

B2

'ljJ . The

11 oo : 01

: 31

where A. 1/; A 1/; and B uncertainties ~A and ~B are defined by 2

2

2

1/;

B

2

1/J . The

(4.29)

[email protected]

33

Physicsguide

Jvi athematical Background

@Sk J ahiruddin, 2020

We can prove after doing some algebra t hat (4.30)

Check using the formula that (4.31)

4.9

Functions of Operators

Let F(A ) be a function of an operator A. If A is a linear "' operator, we can Taylor expand F(A) in a power series of ,,.. A: 00 (4.32) n= O

where an is just an expansion coefficient. As an illustration of an operato1~ function, consider eaA, where a is a scalar ~

11 oo : 01

: 34 00

( 4.32) n=O

where an is just an expansion coefficient. As an illust ration of an operator function, consider eaA , where a is a scalar which can be complex or real. We can expand it as follows: ~

[email protected]

Physicsguide

34

@Sk J ahiruddin, 2020

Nl athematical Backg1·ound

Commutators of functions of operators: A

A

A

If A commutes with another operator B, t hen B corn" mutes with any operator function that depends on A :

[A, B] = o ~~> A

[F (A), B] = [A, F (B )] = 0

( 4.34)

A

A

F (A) commutes with A and with any other function, G(A), of A : A

[A, F (A)] =

o,

An, F (A)

=

o, [F (A) , G(A)] = o ( 4.35)

Hermitian adjoint of function of operators:

11 oo : 01 [A , F (A)] =

o,

: 36

o, [F (A) , G(A)]

An, F (A) =

=

o

(4.35) Hermitian adjoint of function of operators:

The adjoint of F (A) is given by

lF(A.)J t = F *(A.t) "

(4.36)

"

If A is Hermitian, F (A) is not necessarily Hermitian; F (A) will be Hermitian only if F is a real function and A

35

j [email protected]

Physicsguide

Niatl1ematical Backg1·ound

@Sk J ahiruddin , 2020

is Hermitian. Examples -

ei·11 t = e- iA.t

(4.37)

'

Commutators of function of operators: Non zero commutators: You must know "

"

[A, B] # 0

~;>

"

"

[B , F (A)] # 0

(4.38)

We can easily show by expanding the function in series t hat

(4.39)

11 oo : 01

: 39

t hat

(4.39)

We can prove (4.40)

The corollary of which is If A and B commute wit h t heir commutators ([A, [A, B]] = [B , [A, B]] = 0) then

(4.41) [email protected]

P hysicsguide

36

@Sk J ahiruddin, 2020

:Ni athematical Backg1·ound

We can also prove Baker-Campbell-Hausdorff Lemma A " -A

e Be

"

" "

l

"

" "

= B+ [A, B] +2! [A, [A, B]] +

1 "

"

" "

[A, [A, [A, B ]] I] + · · · 31 (4.42)

4.10

Inverse operator "

1

"

The inverse A- of a linear operator A is defined by the relation (4.43) "

where I is the unit operator

11

00:01 :41

1

The inverse A_- of a linear operator

A is

defined by the

relation (4.43) A

where I is the unit operator You can easily prove the following relations 1) - 1

(i) (A=A (ii) (AT) - 1 = (A-1) T

= (A- 1) t (iv) (AB) - 1 = B- 1 A- 1 (V) (AB · · · G)- 1 = G- l (iii) (At) - i

[email protected]

37

· · ·

B- 1A - l

Physicsguide

Mathematical Backg1·ound

@Sk J ahiruddin, 2020

4.11

(4.44)

Unitary operators

A linear operator () is said to b e unitary if its inverse ()- 1 is equal to its adjoint ()t :

or

(4.45)

There are few important results about unitary op erators you need to know

11 oo : 01

: 44

or

(4.45)

There are few important results about unitary operat ors you need to know

(1) The product of two unitary operators is also unitary as you can easily see for two op erators

(uv)(uv)t = (uv) vtut = u vvt ut = uut =

1

(4.46) This result can be generalized for any number of unitary operators A

A

(2) If

E

is real and G is Hermitian , t he operator

eicG

would b e unitary as you see A

-- e- icG --

(4.47)

(3) The det erminant of a unitary matrix has unit modj ahir@physicsguide. in

38

P hysicsguide

lVIathem atical Background

@Sk J ahiruddin , 2020

ulus. (4) A unit ary matrix represent s, in a part icular basis, a linear operator that leaves t he norms (lengt hs) of complex vectors unchanged.

5

Eigenvalues, Eigenvectors and ....

11 oo : 01

: 46

vectors unchanged.

5

Eigenvalues, Eigenvectors and Basis Transformation

5.1

Eigenvalues and Eigenvectors of operators

A state vector 'ljJ) is said to b e an eigenvector (also called an eigenket or eigenstate) of an operator A if the application "

of A to 'ljJ) gives

(5.1) "

where a is a complex number , called an eigenvalue of A . T his equation is known as t he eigenvalue equation, or eigenvalue "

problem , of the operator A. Its solutions yield t he eigenvalues and eigenvectors of A .

[email protected]

Physicsguide

39

@Sk J ahiruddin , 2020

Mathematical Background

We must know A n 'l/J)

= an 'l/J)

(5.2a) ~

"

F(A) 'l/J) = F (a) 'l/J)

=~►

;1- 1 'l/J) = ~l'l/J) ,

,

'ljJ) =

eia

'ljJ)

(5.2b) (5.2c)

a

,

eiA

. ,

.,

,

11 oo : 01

: 49

~

A

F(A) 1/;) = F(a) 1/;)

===>~>

eiA

1/J) =

eia

A-11/J) = ~ 1/J)

1/J)

(5.2b) (5.2c)

a

You already know t hese theorems about t he eigenvalues and eigenvectors of various operators (1) For a Hermitian operator, all of its eigenvalues are real and the eigenvectors corresponding to different eigenvalues are orthogonal.

If A_t

=A

'

(5.3)

(2) The eigenstates of a Hermitian operator define a complet e set of mutually orthonormal basis states. The operator is diagonal in this eigenbasis with its diagonal elements equal to the eigenvalues. This basis set is unique if t he operator has no degenerate eigenvalues and not unique (in fact it is infinite) if t here is any degeneracy. A

A

(3) If two Hermitian operators, A and B , commute and if A has no degenerate eigenvalue, then each eigenvector of A

j [email protected]

40

Physicsguide

:Nl athematical Background

@Sk J ahiruddin , 2020

A

A

A is also an eigenvector of B . In addition, we can const ruct a common orthonormal basis that is made of t he joint eigenvectors of A and B. "'

A

(4) The eigenvalues of an ant i-Hermitian operator are

11 oo : 01 : s2 A

A

A is also an eigenvector of B . In addition , we can construct a common orthonormal basis that is made of t he joint eigen,..,

A

vectors of A and B. (4) The eigenvalues of an ant i-Hermit ian operator are eit her purely imaginary or equal t o zero. (5) The eigenvalues of a unit ary operator are complex numbers of moduli equal to one; the eigenvectors of a unitary operator t hat has no degenerat e eigenvalues are mutually ort hogonal.

5.2

Parity operator

The space reflection about the origin of the coordinate system is called an inversion or a parity operation. This trans,.., formation is discrete. The parity operator P is defined by its action on t he kets f) of t he posit ion space: A

(5.4)

P r) = -r), so that

JJ'lj; (r) = 'lf; (-r)

[email protected]

(5.5)

41

@Sk J ahiruddin , 2020

The parity operator is Hermit ian ,

P hysicsguide

Jvi athematical Background

pt = P, since

11 oo : 01 : s4 @Sk J ahiruddin, 2020

lVIathem atical Background

A

A

The parity operator is Hermit ian , p t = P , since

d3rcp*(r) ['P'lj; (r)]

d3rcp*(r)'lj;( - r)

=

d3rcp*(- r) 'lj;(r)

=

d r[Pcp (r)] *'ljJ (r) 3

(5.6) (5.7) hence A

2

A

P =I

p = p-1

or

(5.8)

The parity operator is therefore unitary, since its Hermitian adjoint is equal t o its inverse:

(5.9) Now, since P2 = f , the eigenvalues of P are + 1 or - 1 with t he corresponding eigenstates A

A

P'lj;+ (r)

=

"P+ (- r)

=

"P+(r),

P'lj;-(r)

=

"P-(- r)

=

- 'lj;-(r) (5.10)

The eigenstate "P+) is said to be even and "P- ) is odd . Therefore, the eigenfunctions of t he parity operator have definite parity: t hey are eit her even or odd. Since "P+) and "P- ) are joint eigenstates of the same Hermit ian operator P but with different eigenvalues, t hese eigenstates must b e A

[email protected]

@Sk J ahiruddin, 2020

rir t

h f"IO'f"ln ~ l ·

42

Physicsguide

lVIathematical Backg1·ound

11 oo : 01 : s1

@Sk Jahiruddin, 2020

Mathematical Background

ort hogonal: 3

(1/J+ 1/J_) = =

d r'lj;~( - r)'l/J_(-r)

-

3

d r'ljJ~(rJ'l/J- (r)

- (1/J+ 1/J-) = 0 (5.11)

The states 1/J+) and 11/J- ) form a complet e set since any function can be written as 1/J(r) = 'l/J+(rJ + 1/J_(r), which leads to

'l/J- (r) =

1 2 ['ljJ (r) - 1P ( -r) ]

(5.12) Parity operator anti-commutes with position operator "'

"' -+

"'

PR r) = rP r) = f1 - f) "'

"'

RP r;

=

RI -

f)

=

(5.13)

- r1 - f)

hence we say "'

{P , R} = o

(5.14)

Similarly we can check t hat "'

{P , f} = o

j [email protected]

(c)Sk J a,h ir11ddin. 2020

43

(5.15)

Physicsguide

:Niatl1ema,tical Ba,cke:1·ound

11 oo : 01 : sg j [email protected]

43

Physicsguide

:Nl athematical Background

@Sk J ahiruddin , 2020

5.3

Basis transformation

In quantum mechanics we deal with unitary transformat ions. When the elements are real the unitary transformat ion becomes orthogonal transformation. Nature does not allow us non unitary transformation in our present discussion of non relativistic quantum mechanics. Kets 'l/J) and bras ('l/J transform under unitary transformation as follows:

l'l/J') = Ul'l/J),

(5.16)

The action of operator under old and new basis A

A

'l/J) =

A' 'l/J') =

)

')

In new basis we write

A' l'l/J') =

') ->

A'U 'l/J) = u ) = uA 'l/J) -> A'U = uA

Multiplying both sides of [![rt =[rt[!= i we have

A'U =

uA by ut and since

'

(5.17) [email protected]

44

Physicsguide

11 oo : 02 : 02

(5.17) [email protected]

44

Physicsguide

Jvi athematical Background

@Sk J ahiruddin, 2020

In summary we write (5.18)

and

11/J) = (;t 1/J'),

(1/J

=

(1/J' u, A= utA'u

(5.19)

Properties of unitary transformation: A

A

(1) If an operator A is Hermitian , its transformed A is also Hermitian, since

(5.20)

(2) The eigenvalues of

A and A' are same: (5.21)

(3) Commutators that are equal to (complex) numbers remain unchanged under unitary transformations, since the t ransform ation of [A, B] = a , where a is a complex number, is given by (5.22)

11 oo : 02 : os remain unchanged under unitary transformations, since t he t ransformation of [A, .BJ = a, where a is a complex number, is given by "' 1 "' 1 (5.22) A ,B = A,B A

[email protected]

A

P hysicsguide

45

Nl athematical Backg1·ound

@Sk J ahiruddin, 2020

A

(4) (1/J A x), remain unchanged under unitary transformations A

1/J' A1 X1

f)t f)

A.

f)tu

(5.23)

x = (1/J A. x)

1 1 (1/J x )

= (1/J x)

(5.24)

(1/J' 1/JI)

= (1P 7P)

(5.25)

(5) f) _Af)t n = f) _An(Jt

(5.26)

(5.27)

A unitary transformation does not change the physics of a system; it merely transforms one description of the system to another physically equivalent description.

11 oo : 02 : 01 scription of the system to another physically equivalent description.

46

j [email protected]

Physicsguide

Niatl1em atical Backg1·ound

@Sk J ahiruddin , 2020

5.4

Infinitesimal unitary transformation A

Consider an operator U which depends on an infinitesimally small real parameter E and which varies only slightly from t he unity operator I : A

Uc(G) = f + iEG

(5.28)

A

where G is called the generator of t he infinitesimal transformation. Clearly, Uf; is a unitary transformation only when the paramet er E is real and G is Hermitian, since A

A

where we have neglected t he quadratic t erms in E The transformat ion of a state vector l'lf') is

7P') = (i + icG) 'lf') = 1P) + bl'lf')

(5.30)

where A

(b*1 b*2 b*3

1/J)

A1 1 A1 2

A13

• • •

a1

A21

A22

A23

• • •

a2

A31

A32

A33

• • •

a3

• • •

• • •

• • •

• • •







• • •

(6.10)

6.4

Trace of matrix representation

The trace Tr(.A) of an operator A is given , within an ort honormal basis { 0 is little bit complicated. it is enough to remember is the reflection and t ransmission amplitude. Remember the reflection only. The t ransmission can be calculated by T = l - R R=

l

1 + (2n2 E /ma 2 )

( 5 15 )

·

The reflection and transmission coefficients for Delta potential barrier (V (x) = ab(x)) are same as delta function well scattering state. i.e for barrier

1 R = - -2 - - l + (2n E / ma 2 )

(5.16)

Do the step barrier and finite potential well problem from Griffiths/ Zettli. Due to lack of time I'm unable to cover these topics j [email protected]

37

Physicsguide

11 oo : oo : oo

Angular Momentum Sk J ahiruddin Assistant Professor Sister Nibedita Govt. College, Kolkata Author was the topper of IIT Bombay M.Sc Physics 2009-2011 batch He ranked 007 in IIT JAM 2009 and 008 (JRF) in CSIR NET J t1ne 2011 He has been teaching CSIR NET aspirants since 2012

1

@Sk J ahiruddin, 2020

Angular Momentum

11 oo : oo : 04 1

@Sk J ahiruddin, 2020

Angular JVIomentum

Contents 1 Angular momentum

4

1.1

Basic definition and commutation relation .

4

1.2

General formul ation of t he problem











5











8

Analytical expression of angular momentum operator in t hree dimension . . . . . . . . .

11

Matrix representation of angular momentum

14

Eigenvalues of J 2 and Jz ..

1.2.1 1. 3

1.4

1.5 Geometrical representation .



















15

1.6 Spin



















18













19













23



























1.6.1 Stern Gerlach experiment 1.6.2

General theory of S = 1/ 2

1.6.3

P auli Matrices . . . . . . . . .









28

1.6.4

Spin part icle in magnetic field









31

j [email protected]

2

Physicsguide

11 oo : oo : 06 1.6.4

Spin part icle in magnet ic field . . . .

j [email protected]

2

P hysicsgt1ide

@Sk J ahiruddin , 2020

2

31

Angular :NI01n entum

Addition of angular momentum 2. 1

Addition of t wo spin 1 / 2

2.2

Addit ion of t wo l

=1 . .

38























38























42

11 oo :oo :og

[email protected]

@Sk J ahiruddin , 2020

1 1.1

P hysicsguide

3

Angular JVIomentum

Angular momentum Basic d efinition and commutation relation

Angular moment um is defined as

L= R x P

(1.1)

You can evaluate t he commut ators like

[Lx, Ly] = [Y Pz - Z Py, Z Px - X Pz]

= [YPz,ZPx] + [Z Py, XPz] = Y [Pz, Z] Px + X [Z, Pz] Py

(1.2)

= -inY Px + inX Py = inLz Similarly evaluate the ot her commutators

[L x, Ly] = inLz [Ly, L z] = inLx [Lz, Lx] = inLy we write t his is

(1.3)

11 oo : oo : 11 [Lx, Ly] = inLz [Ly, L z] = inLx [L z, Lx] = inLy

(1.3)

we write t his is

(1.4) [email protected]

4

@Sk J ahiruddin , 2020

Physicsguide

Angular Moment um

Prove

(1.5) and

(1.6) where ~ are the position vectors, i,e x , y , z and Pi are the momentum operators, i.e Px, Py, Pz

1.2

General formulation of the problem

We call angular momentum operator a set of variables which satisfy [Jx, Jy] =iriJz

[Jy, Jz] = inJx

(1. 7)

[Jz, l x] = inly We define an operator

J2 = J2 + J2 + J2 X y Z

(1.8)

11 oo : oo : 14 We define an operator

=

J 2

J 2 X

+

J 2 y

+ 1 Z2

(1.8)

Prove that 2

[J , l x] = [J;+ J;+ J;, l x] [J;, lx] + [J;, l x] = 0 j [email protected]

(1.9) Physicsguide

5

Angular Niomentum

@Sk J ahiruddin, 2020

2

So t he 1 operator commutes with all components of angular momentum operators. So we can take any of the component to construct a simultaneously commuting observable or CSCO with 1 2 . We take

= j(j + l )fi k , j, m) l z k,j , m) = mri k , j, m) 2

2

J k,j, m)

(1.10)

Inst ead of taking j (j + 1), we could have taken any constant like a, f3 etc. But as know already know what will happen so to make our calculation little bit easier we write our eigenvalue like t his. Now define

1+ =Ix+ i l y

(1.11)

J_ = l x - i l y

Like t he operators a and at of the harmonic oscillator, J+ - ..........1

7

- .... -

--L

l l - .... -.-- :.4....:

- .... . .

J. 1..... -- .. ___.., _

_

...J..: - . : .._...,4... _

_

,e - - -1..... -.l.- 1.... - ....

11 oo : oo : 16 Now define

l + = l x + i Jy

(1.11)

J_ = l x - il y

Like t he operators a and at of the harmonic oscillator, l + and J _ are not Hermitian : t hey are adjoints of each other.

[email protected]

Physicsguide

6

@Sk J ahiruddin, 2020

Angular Momentum

Prove these commutation relations first

[Jz, J+] = nJ+ [Jz, J_] = -nJ_

(1.12a) (1 .12b)

[J+, J_] = 2nJz [J 2, l +] = [J 2, J_] = [J2, Jz]

(1.12c) =

0

(1.12d)

Calculate the products l +l- and J_J+ to get

l +J- = (l x + ily) (l x - il y)

= J;+ J;-i[Jx, l y] = J; + J; + n,Jz

(1.13)

J_J+ = (Jx - i Jy) (Jx + i J y)

= J;+ J;+i[Jx, l y]

= 1:. + J~~ - nJ

7

(1.14)

11 oo : oo : 19 J_J+ = (Jx - i Jy) ( Jx + i J y)

= J; + 1i + i [J1: , Jy]

=

1 X2 + 1 y2

_

(1.14)

nJ Z

So we can write J+ l J _J+

=J

2

=J

2

-

J; + nJz J; - n,Jz

(1.15)

and also

[email protected]

P hysicsguide

7

@Sk J ahiruddin , 2020

1 .2.1

Angular Moment um

Eigenvalues of J 2 and l z

As we have already calculat ed [Jz, l +]

=

[Jz, J _]

= - nJ _

hJ+

(1.17)

We easily see l zl + j, m)

and

= l +l z j , m) + nJ+ j, m) = m riJ+ j , m) + riJ+ j, m) = (m + l )riJ+ IJ, m)

(1.18)

11 oo : oo : 21 and

l zJ-lj, m) = J_JzlJ, m) - nJ_lj, m)

= mnJ_ lj, m) - nJ_ j, m) = (m - l )nJ_ j, m)

(1.19)

So l + j, m) is therefore an eigenvector of l z with the eigen-

+ 1) n and

J _ j, m) is there£ore an eigenvector of l z wit h the eigenvalue (m - l )n

value (m

So l + j, m) is proportional to j, m proportional to

lj, m

+ 1)

and J_ j, m) is

- 1) . So we write

j [email protected]

Physicsguide

8

Angular Niomentum

@Sk J ahiruddin , 2020

l + j , m)

= C+IJ, m + 1);

J_ j, m) =

c_ j, m -

1) (1.20)

Now, consider the vectors l + IJ , m) and J_ j, m) , and note t hat the square of their norms is posit ive or zero:

J+ j, m) I J_ j, m)

11

2

1 1

2

(j, m IJ_ J+ j, m) ~ 0 2 (j, m 1+ J_ j, m) ~ o

=

C+ 2

=

c_

=

We use the expression of l +J- and J_J+

(1.21)

11 oo : oo : 24 II J_

j, m) II =

c_

(j, m J+l- j , m)

We use t he expression of J+ J_ and J_ J+

(j,m J_J+ j,m)

= (j,m (J =

j(j

2

+ l)h

2

J; - fiJz) j , m)

-

2 2

m fi

-

-

2

mfi

=

2

C+ (1.22)

and (j,m lJ+J- j,m)

=

(k,j,m

2

(J

= j(j + l)fi

2

-

J; + fiJz) k,j,m) 2 2

m fi

-

+ mfi = 2

2

C_ (1.23)

Substituting these relations into t he expression of norms,

[email protected]

Physicsguide

9

@Sk J ahiruddin, 2020

Angular JVIoment um

equation (1. 21) , we get

+ 1) - m(m + 1) = (j - m)(j + m + 1) j(j + 1) - m(m - 1) = (j - m + l) (j + m) j(j

Hence

- (j + 1) -j

~ m ~

j

~

j

m

~

~

0

(1.24)

(1.25)

+l

We have calculated the values of C+ and write J+ j, m)

~ 0

IC- .

= nJj(j + 1) - m(m + l )lj, m + 1)

So we

(1.26)

11 oo : oo : 26 We have calculated the values of C+ and write

J+ j, m) = nJj(j

IC- .

So we

+ 1) -

m(m + l )lj, m

+ 1)

(1.26)

+ 1) -

m(m - 1) j, m - 1)

(1.27)

And

J_l j, m) = nJj (j

As you see the value of m increases as st ep of ONE, and t he value of m varies from -l to l, then we must have

- l +n= l;

(1.28)

Where n is an integer

So

2l = n

(1.29)

Hence l is integer or half integer And so m is also an integer or half integer j [email protected]

10

Physicsgt1ide

Angular :NI01nentum

@Sk J ahiruddin , 2020

1.3

Analytical expression of angular momentum operator in three dimension L = (n/i)(r x V )

(1.30)

and as

r

= rf·' and·'

a V = f 8r

A1 a 0 + ~a0

1 a + cp rsin08cp A

3 (l. l )

11 oo : oo : 29 and as r

" 8 0" 1 8 ;, 1 8 = r - + -- + 'f' ar r 80 r sin 0 8cp

n

= rf·' and·,

V

(1.31)

So L

n (

i

=

a r 8r

A

rr

")

X

( + r A

X

a 0") 80

( + r A

1 a '+' Sin 0 8cp ;,)

X

(1.32)

We know A

(f x f)

A

A

0, (f x 0) = cp, and (f x cp) = - 0

=

Hence

L = ~ i A

A

J a_ 0 80

1

a

(1.33)

(1.34)

sin 0 8cp

A

Write 0 and cp in cartesian components:

A

A

0 = (cos 0 cos cp)i + (cos 0 sin cp) j - (sin 0)k A

cp = -(sin cp)i + (cos cp)j jahir@physicsguide. in

(1.35)

Physicsguide

11

@Sk J ahiruddin , 2020

Angular JVIomentum

Hence

Ii

L =•

'l

(1.36)

So we get

n I

A\

00: 00: 32 •

i

(1.36)

So we get (1.37)

n

(1.38)

L y= -. i

(1.39) You can also write

L± = Lx ± iLy

(1.40) As cos cp

± i sin cp = e±i 8

8

( 1.41)

80 ± i cot 0 8cp You can easily calculate

8

2

8

2

+ cot 0 802 80

[email protected]

12

L +L-

= - ri

2

8 8 + cot 0 O

4

-

½

2 (2V0 -

2 E

Va ) 2

2

=

~ [3 ± J1 + 4c ] ::::; 2

~o [3 ± (1 + 2c

2

)]

So the 2nd and 3rd eigenvalues are

[email protected]

19

Physicsguide

@Sk J ahiruddin, 2020

Perturbation and Variational

(b) The unperturbed Hamiltonian is

Ho = Vo

(1) 0 0 O l 0

0

0 2

And t he p ert urbation

H'

= EVo

(- 1) 0 0 0 0 1 0 1 0

There are t hree states

0

1

x1=

0 0



'

x2 =

1

0



'

X3

=

0 0 1

11 oo : oo : s6 There are t hree states

0

1

x1 =

0 0



'

x2 =

1



X3

'

0

0 0 1

=

First order correction on state 3 is ZERO as you know the matrix element of the perturbation matrix (X3 H' X3) = 0. So the result is zero. However if you want to do the matrix

[email protected]

P hysicsguide

20

@Sk Ja.hiruddin, 2020

Perturbation and Variational

multiplication explicitly then ... - 1 0 0

Ej = (X3 H' X3) = EVo(O O 1)

=

EVo (O O 1)

0 1 0

0 0

=

0 0 1

0 1 1 0

0

The second order correction is

and hence TT

T

T

11 oo : oo : s1 m =l ,2

and hence

So

2

Ej = (x1 IH' X3) 1 + (x2 H' X3) Vo

2

Vo

You know the matrix elements, right???

[email protected]

21

Physicsguide

Perturbation and Variational

@Sk J ahiruddin, 2020

So the result 2 _

E3

-

(x1

IH'

X3)

2

+ l(x2 H'

Vo

X3)

2 _

-

Vo

( 17)2 ; 17 _ E vo

vo -

217

E

vo

So t he energy aft er correction

E 3 = Eg

+ E~ + E ~ =

2V0 + 0 +

This is correct upto the order values of t he matrix .

2

E

2

E

Vo

=

V0 (2 +

2

E )

as found in t he exact eigen-

(c) The state 1 and state 2 are degenerate. So we need to find the matrix element of t he perturbation Hamiltonian first . And as t he matrix elements are already given. You have - 1 0 0 (\

(\

1

11 oo : 01

: 01

find the matrix element of the perturbation Hamiltonian first. And as t he matrix elements are already given. You have

-1 0 0 0 0 1 0 1 0

H' = cV0 Hence

(x1

H'I x1) =

- E½ ;

(x2 H' x2) = O; (x1 H ' x2)

=

(x2 H' x1)

=

o

So the W matrix will be - 1 0

W = cV0

0 0 22

[email protected]

P hysicsguide

Perturbation and Variational

@Sk Jahiruddin, 2020

This is already diagonalized. So the eigenvalues are -E½, So to the first order the energy correction is

0.

A schematic diagram of t he problem is l3>

2~

~o

r

ll,)

,J

12>

l

l>

t2,>

'')

-2-Vo ~Vo

e1-

\lo Vo-Vo t:=-

Figure 1.2: Energies of t he Harmonic oscillator after t he perturbation

11 oo : 01

: 03

Figure 1.2: Energies of t he Harmonic oscillator after the perturbation

Exercise:

Do t he same problem for

(1 -E) E 0 H = Vo

1.2

E

1

E

0

E

2

Feynman Hellman theorem

Suppose t he Hamiltonian H , for a particular quantum syst em , is a function of some paramet er .\; let En (A) and 1/Jn (A) be [email protected]

@Sk J ahiruddin, 2020

23

P hysicsguide

Perturbation and Variational

the eigenvalues and eigenfunctions of H (A) . The FeynmanHellmann theorem states t hat

(1.12) We will apply it t o t he one-dimensional harmonic oscillator, (i) using A = w (this yields a formula for the expect ation value of V )

11 oo : 01

: 06

tor ,

=w

(i) using ,X

( t his yields a formula for the expectation

value of V ) (ii) using ,X

= n (this

(iii) using ,X

= m - this yields a relation b etween (T )

yields (T )), and (V) .

For Harmonic oscillator

En=

1

+ 2 nw;

n

n

2

H

d

2

1

= - - -2 + -mw x 2m dx

2 2

2

(1.13)

=w

(i) Take .X

8En

n

aw

8H

1

+ - n· 2

-

aw

'

=mwx

2

' ·

so Feynmann Helman theorem gives

1 n +2

ri

[email protected]

2

= \ n mwx n) Physicsguide

24

@Sk Jahiruddin, 2020

Perturbation and Variational

2 2

As V = ! mw x 2 ' (V) =

1

2 2

n -mw x 2

n

=

n+

1 2

so

1 -w n+2 2 1

Hence

(V) = (ii) Take .X

=n

!2

(1.14)

11 oo : 01 : os (V) = (ii) Take ,,\

1

!2

n+ -

(1.14)

2

=n

aEn

1

an

2

- - = n +-



'

aH an

n

d2

mdx 2

2

n2 d2

n

2mdx 2

-

So Feynman Hellman t h eorem gives 1

n+2

(T ) =

!2

n+

1 2

nw

(1.15)

(iii) Take ,,\ = m

aEn = O

am

and

[email protected]

aH am

n2 d2 1 ----+-w2x2 2m2 dx 2 2 25

P hysicsguide

Perturbation and Variational

@Sk Ja.h iruddin, 2020

n2 d2 1 1 + 2 m 2mdx m So Feynman Hellman Theorem gives

0 = - l (T) m

+

l (V) m

(1.16)

or ( 1.17)

11 oo : 01

: 11

So Feynman Hellman Theorem gives

+

0 = - l (T) m

l (V) m

(1. 16)

or

(T ) = (V)

(1.17)

Feynman Hellman theorem can also be used to determine the expectation values of 1/ r and 1/r 2 for hydrogen. The effective Hamiltonian for the radial wave functions is

n d n l(l + 1) e 1 --- +--- ---2

H =

2

2m dr 2

2

2

2m

r2

41rE0 r

(1.18)

The energy is 4

me E - - - - - - - - -2 n 321r 2 E5n2 (Jmax + l + l )

(1.19)

(i) We take A = e

and

8H

2e 1 --41rEo r

ae 26

[email protected]

Physicsguide

@Sk J ahiruddin, 2020

Perturbation and Variational

So the t heorem says

4E _ _

e Hence

n-

e

1

21rEo

r

11 oo : 01

: 13

So t he t heorem says 1

4 E __ e ne 21rEo

r

Hence 1

81rEo E _ _ 81rEoE1 n e2 e2n2

r

87rEo e2

m

e2

2/i2

47rEO

2

e2m 1 41rE0 1i2 n 2

1

n2

and as

- - 2- = a me then 1

(1.20)

r

(ii) We take A = l then 8En

az -

2me

4

3 2 2 E ~li 321r (Jmax + l + 1)

and

8H

az

/i2 -2m_r_2 (2l

-

2En n

+ 1)

So from the t heorem

2En n [email protected]

2

/i (2Z + l ) 2m 27

@Sk J ahiruddin, 2020

or

1 r2

1 r2 P hysicsguide

Perturbation and Variational

n (2Z + l )/i2

n 3 (2l

+ 1)/i2

11 oo : 01

: 16

@Sk Jahiruddin, 2020

or

1 r2

Perturbation and Variational

4m E n n (2l + l )n2

as hence

4mE1

2

n,2

a2

1 r2

[email protected]

4m E 1 n 3 (2l + l )n2

n3

1 (l +

t) a2

28

@Sk Ja.h iruddin , 2020

2

Variational Principle

(1.21)

P hysicsguide

Perturbation and Variational

11 oo : 01

: 1s

@Sk Ja.h iruddin, 2020

2

Perturbation and Variational

Variational Principle

We want to calculate the ground state energy,Egs, for a system described by the Hamiltonian H, but we are unable to solve t he (time-independent) Schrodinger equation. The variational principle will get us an upper bound for E 98 which is sometimes all we need, and if we guess the wavefunction properly very close to the exact value. Here's how it works: Pick any normalized function 'l/; and we will get

(2.1 ) In t he exam we will be given a potential and t he trial wavefunction 'ljJ

[email protected]

@Sk Jahiruddin, 2020

29

P hysicsguide

Perturbation and Variational

11 oo : 01 [email protected]

: 21

Physicsguide

29

@Sk Jahiruddin, 2020

Perturbation and Variational

You will need to remember few integrals +oo

e -oo CX)

2

- ax +bxd

b2

1r

x=

- exp a

4a

an ? 2 2 X'"'ne - a x dx = (-l)n __ 8a2n

-oo

00

-oo

fi(2n - 1) !! 2na 2n+l CX)

_

00

(- l) n an

dx (x2 + a2)n+l -

a>O

' CX)

n ! 8a2n _ 00 x2

1r(2n - 1)!! 2nn!a2n+l '

+ a2

a>O

bl -----7r - J (x - a)(b - x)dx =-(a+ b - 2vioJy), 2

a X

dx

O 0) in the new potent ial well; it s energy levels and wave function are now given by [email protected]

66

P hysicsguide

Perturbation and Variational

@Sk Ja.h iruddin, 2020

n21r2n 2

E' = - - -2 n ~ 2m(8a)

2 . - Sln 8a

n1rx

8a

(0
a.

(a) Calculate t he total cross section in the low-energy limit. (b) Calculate the total cross section in t he high-energy limit. Solution:

(a) As the scattering is dominated at low energies by s-waves, l = 0, t he radial Schrodinger equation is

11 oo : 01 : os Solution: (a) As t he scattering is dominated at low energies by s-waves, l = 0, t he radial Schrodinger equation is

d2u(r) = Eu(r) 2m dr 2

-

where u(r)

n,2

( ) r>a

= r R (r) . T he solut ions of t his equation are

u(r) =

r a

2m E /n2 . The cont inuity of u(r) at r = a

where k 2 leads to sin ( ka

u1(r) = 0,

+ 60) = 0

~~>

[email protected]

tan 50

= - tan (ka)

23

@Sk J ahiruddin , 2020

P hysicsguide

WKB and Scattering

since sin a = 1/ ( 1 + cot a) . The lowest value of the phase shift is 50 = -ka; it is negative, as it should be for a 2 2 repulsive potential. Insert sin 50 = sin (ka) into t he total scattering cross section formula 2

2

41r . 2 41r . 2 o-o = k 2 sin 50 = k 2 sin (ka)

For low energies, ka

1, the number of part ial waves contribut ing to the scattering is large. Assuming t hat lrriax ~ ka, we write t he total scattering cross section formula

since so many values of l cont ribute in t his relation, we 2 may replace sin bz by its average value, ~; hence

j a [email protected]

@Sk J a hiruddin , 2020

Physicsguide

24

WKB and Scattering

where we have used I:~0 (2l+ 1) = (n+ 1) . since 1 we have 2

lmax

>>

(2.9)

11 oo : 01 where we have used 1 we have

: 1o

I:~0 (2l + 1) =

2

(n+ 1) . since lmax

21r 2 21r 2 2 a~ k2 l max = k2 (ka) = 21ra

>>

(2.9)

In conclusion, the cross section from a hard sphere potent ial is ( a) four times t he classical value, 1ra 2 , for lowenergy scattering and (b) twice the classical value for highenergy scattering. Example: Consider t he elastic scatt ering of 50 MeV neutrons from a nucleus. The phase shifts measured in

t his experiment are 60 = 95°, 61 = 72°, 62 = 60°, 63 = 35°, 64 = 18°, 65 = 5°; all other phase shifts are negligible ( i.e., 6t ~ 0 for l > 6) (a) Find the total cross section. (b) Estimate the radius of the nucleus.

Solution: (a) As 6z ~ 0 for l > 6 25

[email protected]

Physicsguide

WKB and Scat tering

@Sk J ahiruddin, 2020

4

6

k: L(2l + 1) sin 81 = 1: (sin 80 + 3 sin 81 + 5sin 8 + 7 sin 8 +9 sin 8 + 11 sin 85) = 1: x 10.702 2

a=

l=O

2

2

2

4

2

2

2

2

3

11 oo : 01 a = k2

: 12

2l + 1 sin Ol

..__

l=O

=

47f 2 2 2 2 ? (sin 80 + 3 sin 81 + 5sin 82 + 7 sin 83 k+9sin2 04

+ 11 sin

2

05)

=

1: x 10 .702

2

To calculate k , we need simply to use the relation E = n 2 k 2 / (2mn) = 50MeV, since the neutrons move as free part icles before scattering. Using mnc2 197.33MeV fim , we have

= 939.57MeV and

nc =

2

k 2 = 2mnE = 2 (mnc ) E = 2(939.57MeV) (50MeV) n2 (nc) 2 (197.33MeVfm) 2

= 2.41fm - 2 hence 47f a = _ lfm_ 2 x 10.702 24

2

= 55.78fm = 0.558barn

(b) At large values of l , when t he neutron is at its closest approach to t he nucleus, it feels mainly the effect of t he centrifugal potential l(l + l )n2 / (2m nr 2 ); t he effect of the nuclear potential is negligible. We may thus use the approximations E ~ l (l + l )n jahir@physicsguide. in

@Sk J ahiruddin , 2020

2

/

(2m nr~) ~ 42n

2

26

/

(2mnr~) where Physicsguide

"\i\'KB and Scattering

we have taken l ~ 6, since Oz ~ 0 for l > 6. A crude value of the radius of t he nucleus is t hen given by 21(197.33MeVfm) 2 (939.57MeV) (50MeV )

11 oo : 01 j [email protected]

26

: 16

Physicsguide

WKB and Scat tering

@Sk J ahiruddin , 2020

we have taken l ~ 6, since bz ~ 0 for l > 6. A crude value of the radius of t he nucleus is t hen given by

21/i2 mn E

21 (lie ) 2 (mn,c2 ) E

21 (197.33MeVfm)2 (939.57Me V) (50MeV)

= 4. 17fm

j [email protected]

27

Physicsguide

11 oo : 01 j [email protected]

26

: 11

Physicsguide

WKB and Scat tering

@Sk J ahiruddin , 2020

we have taken l ~ 6, since bz ~ 0 for l > 6. A crude value of the radius of t he nucleus is t hen given by

21/i2 mn E

21 (lie ) 2 (mn,c2 ) E

21 (197.33MeVfm)2 (939.57Me V) (50MeV)

= 4. 17fm

j [email protected]

27

Physicsguide

11 oo : 01 j [email protected]

26

: 20

Physicsguide

WKB and Scat tering

@Sk J ahiruddin , 2020

we have taken l ~ 6, since bz ~ 0 for l > 6. A crude value of the radius of t he nucleus is t hen given by

21/i2 mn E

21 (lie ) 2 (mn,c2 ) E

21 (197.33MeVfm)2 (939.57Me V) (50MeV)

= 4. 17fm

j [email protected]

27

Physicsguide

11 oo : oo : oo

Basic Therillodynaillics Sk J ahiruddin Assistant Professor Sister Nibedita Govt. College, Kolkata Author was the topper of IIT Bombay M.Sc Physics 2009-2011 batch He ranked 007 in IIT J AM 2009 and 008 (J RF) in CSIR NET J t1ne 2011 He has been teaching CSIR NET aspirants since 2012

1

@Sk J ahiruddin, 2020

Basic Thermodynamics

11 oo : oo : 03 1

@Sk J ahiruddin, 2020

Basic Thermodynamics

Contents 1

Introduction

4

2

Basic concepts

7

3

4

2. 1

System . . .





































9

2.2

P arameters





































11

2.3

Thermodynamic State

























13

2.4

equilibrium

. . .































14

2.5

Equation of State































17

Thermodynamic transformation

18

3.1

irreversible process





























19

3.2

reversible process .





























20

Work Heat Internal energy

j [email protected]

2

22

Physicsguide

11 oo : oo : 06

j [email protected]

2

P hysicsguide

@Sk J ahiruddin , 2020

5

Basic Thermodynamics

4. 1

Work.











































22

4.2

Heat .











































25

4.3

Internal energy

































27

Laws of thermodynamics

29

5. 1

First Law . .



































29

5.2

Carnot engine



































33

5.3

Second Law





































38

5.4

Third Law .





































41

11 oo :oo :oa

[email protected]

@Sk J ahiruddin , 2020

1

3

Physicsguide

Basic Thermodynamics

Introduction

Statistical mechanics is the art of turning the microscopic laws of physics into a description of Nature on a macroscopic scale. Suppose you've got theoretical physics cracked. Suppose you know all t he fundamental laws of Nature, t he properties of the elementary particles and the forces at play between t hem. How can you t urn this knowledge into an understanding of t he world around us? More concretely, if I give you a box containing 1023 particles and tell you t heir mass, t heir charge, t heir interactions, and so on, what can you tell me about t he stuff in the box? There's one strategy t hat definitely won't work: writ2 ing down the Sehr·· odinger equation for 10 3 particles and solving it. That's typically not possible for 23 particles, let alone 102 3. What's more, even if you could find the wavefunction of the system , what would you do with it? The posit ions of individual particles are of little interest to anyone. We want answers to much more basic, almost childish, questions about t he cont ents of the box. Is it wet ? Is it hot? What colour is it? Is the box in danger of exploding? Wh:::i.t h:::i.n nPn ~ if wP ~n11 PP'ZP it. n1111 it . h P.:::i.t it 11n? Hnw r :::i.n

11 oo : oo : 11 positions of individual particles are of little interest to anyone. We want answers to much more basic, almost childish, questions about the contents of the box. Is it wet? Is it hot? What colour is it? Is the box in danger of exploding? What happens if we squeeze it, pull it, heat it up? How can

[email protected]

@Sk J ahiruddin, 2020

4

Physicsguide

Basic Thermodynamics

we begin to answer these kind of questions starting from the fundamental laws of physics? For centuries from the 1600s to the 1900s scientists were discovering laws of physics that govern different substances. There are many hundreds of these laws, mostly named after their discovers. Boyle's law and Charles's law relate pressure, volume and temperature of gases (t hey are usually combined into the ideal gas law); the Stefan-Boltzmann law tells you how much energy a hot object emits; Wien's displacement law tells you t he colour of that hot object; the Dulong-Petit law tells you how much energy it takes to heat up a lump of stuff; Curie's law tells you how a magnet loses its magic if you put it over a flame ; and so on and so on. Yet we now know t hat these laws aren't fund amental. In some cases they follow simply from Newtonian mechanics and a dose of statistical thinking. In other cases, we need to throw quant um mechanics into the mix as well. But in all cases, we're going to see how derive them from first principles A large part of this course will be devoted to figuring

11 oo : oo : 13 1n 1ng. n o er cases, we nee quantum mechanics into the mix as well. But in all cases, we're going to see how derive them from first principles A large part of this course will be devoted to figuring out the interesting things that happen when you throw 102 3 particles together. One of t he recurring themes will be that 2 10 3 =f. 1. More is different: there are key concepts t hat j [email protected]

@Sk J ahiruddin , 2020

5

Physicsguide

Basic Thermodynamics

are not visible in the underlying laws of physics but emerge only when we consider a large collection of particles. One very simple example is temperature. This is not a fundamental concept: it doesn't make sense to talk about the temperature of a single electron. But it would be impossible to talk about physics of the everyday world around us without mention of temperat ure. This illustrates the fact that the language needed to describe physics on one scale is very different from that needed on other scales. We'll see several similar emergent quantities in this course, including the phenomenon of phase transitions where the smooth continuous laws of physics conspire to give abrupt, discont inuous changes in the structure of matter . Historically, the techniques of statistical mechanics proved to be a crucial tool for understanding the deeper laws of physics. Not only is the development of the subject int imately tied with the first evidence for the existence of atoms, but quantum mechanics itself was discovered by ap-

11 oo : oo : 11 to be a crucial tool for understanding t he deeper laws of physics. Not only is the development of t he subj ect int imat ely t ied with t he first evidence for the existence of atoms, but quantum mechanics itself was discovered by applying statistical methods to decipher the spectrum of light emitted from hot objects. (We will study t his derivation in Section 3). However, physics is not a finished subject. There are many important systems in Nature from high temperature superconductors to black holes which are not [email protected]

6

@Sk J ahiruddin, 2020

Physicsguide

Basic Thermodynamics

yet understood at a fundamental level. The information t hat we have about t hese systems concerns their macroscopic properties and our goal is to use these scant clues to deconstruct t he underlying mechanisms at work. The tools t hat we will develop in t his cour·se will be crucial in t his task.

2

Basic concepts

Few definit ions to start wit h 1. A system that is completely isolated from all outside influences is said t o be contained in adiabatic walls. We will also refer to such syst ems as insulated 2. Walls that are not adiabat ic are said to be diathermal

11 oo : oo : 19 1. A system that is completely isolated from all outside influences is said to be contained in adiabatic walls. We will also refer to such systems as insulated 2. Walls that are not adiabatic are said to be diathermal and two systems separated by a diathermal wall are said to be in t hermal contact . A diathermal wall is still a wall which means that it neither moves, nor allows part icles to transfer from one system to the other. However, it is not in any other way special and it will allow heat to be transmitted between systems. If in doubt , think of a thin sheet of metal. [email protected]

@Sk J ahiruddin , 2020

7

Physicsguide

Basic Thermodynamics

3. An isolated system , when left alone for a suitably long period of t ime, will relax to a state where no further change is noticeable. This state is called equilibrium For a gas, the only two variables t hat we need to specify are pressure P and volume V : if you know the pressure and volume, then all other quantities - colour, smell, viscosity, t hermal conductivity all are fixed. For other syst ems, furt her (or different) variables may be needed to describe t heir macrostate. Common examples are surface tension and area for a film; magnetic field and magnetization for a magnet ; electric field and polarization for a dielectric. In what follows we'll assume that we're dealing with a gas and use P and V to specify the state. Everything t hat we say can be

11 oo : oo : 22 for a film ; magnetic field and magnetization for a magnet ; electric field and polarization for a dielect ric. In what follows we '11 assume that we 're dealing with a gas and use P and V to specify the state. Everything that we say can be readily extended to more general settings. In t he case of a magnetic solid the appropriate variables are t he magnetic field H, the magnetization M, and the temperature T . In more complicated situations, such as when a liquid is in contact wit h its vapor, addit ional variables may be needed: such as the volume of both liquid and gas VL , Va, the interfacial area A, and surface t ension (j. If t he t hermodynamic variables are independent of t ime, the system is said to be in a steady state. j [email protected]

@Sk J ahiruddin, 2020

2.1

8

Physicsguide

Basic Thermodynamics

System

Any macroscopic material body could be considered as a t hermodynamic system. Macroscopic system means a system composed of atoms or molecules of t he order of one Avogadro number (NA ~ 6.022 x 10 23 ) per mole. The examples of t hermo- dynamic system could be a wire under tension, a liquid film, a gas in a cylinder, radiation, a solid material, magnetic material, dielectrics, and many others. The thermodynamic systems should have a boundary which separates the systems from t he surroundings. Consider a drop of liquid as a thermodynamic system. The surface of t he liauid is the boundarv between the liauid and air. In t he

11 oo : oo : 24 material, magnetic material, dielectrics, and many others. The thermodynamic systems should have a boundary which separates the systems from t he surroundings. Consider a drop of liquid as a thermodynamic system. The surface of t he liquid is the boundary between the liquid and air. In t he language of t hermodynamics, the boundary is considered as a wall. This has been demonstrated in t he following Figure.

[email protected]

9

@Sk J ahiruddin, 2020

Physicsguide

Basic Thermodynamics

SYSTEM

Figure 2.1: Schematic representation of a thermodynamic system. The shaded area is the surroundings or the universe. The thick line represents the boundary. The cent ral white space is the system.

11 oo : oo : 21 Figure 2.1: Schematic representation of a t hermodynamic system. The shaded area is the surroundings or the universe. The thick line represents the boundary. The central whit e space is the system.

The nature of the wall classifies t he thermodynamic system in different categories. (i) If the wall is such t hat energy or matter (atoms or molecules) cannot be exchanged between t he system and its surroundings, t hen the system is called isolated . Total energy E , and total number of part icles N are conserved for t his system. (ii) The wall is such t hat only energy could be exchanged between the system and t he surroundings. If the system is in thermal contact with a heat bath in t he surroundings, j [email protected]

@Sk J ahiruddin , 2020

10

P hysicsguide

Basic Thermodynamics

heat energy will be exchanged however total number of part icles will remain constant. This syst em is known as closed system. (iii) If t he wall is porous, t hen, beside energy, matter (atoms or molecules) can also be exchanged between t he system and the surroundings. If the system is in contact wit h a heat bath as well as with a part icle reservoir, heat energy and number of part icles both will be exchanged. Neither

11 oo : oo : 29 (atoms or molecules) can also be exchanged between t he system and the surroundings. If the system is in contact with a heat bath as well as with a particle reservoir, heat energy and number of particles both will be exchanged. Neither energy nor number of particles is conserved in this system. The system is called an open system.

2.2

Parameters

Thermodynamic paramet ers are measurable macroscopic physical quantities of a system. Consider a gas in a cylinder. Measurable physical quantities are t he pressure (P ) , t emperature (T ) and the volume (V ) of t he gas. These physical quant ities are called thermodynamic parameters or thermodynamic variables. The t hermodynamic variables are macroscopic in nature. They are divided in two categories, intensive and extensive parameters. [email protected]

@Sk J ahiruddin, 2020

11

P hysicsguide

Basic Thermodynamics

Extensive quantit ies are proportional to the amount of matter present - V , U, N, total magnetization etc Intensive quantities are independent of t he amount of matter present - P , T , µ magnetic field etc. Every intensive parameter has a corresponding indepen-

11 oo : oo : 32 Intensive quant ities are independent of t he amount of matter present - P , T , µ magnetic field etc. Every intensive parameter has a corresponding independent extensive parameter. They form a conjugate pair of t hermodynamic variables. Since they are independent of each other , one could be changed wit hout effecting the other. Keeping the pressure constant the volume of the gas can be changed and vice versa. A partial list of conjugate thermodynamic parameters are given in Table Syst em Wire Liquid film Fluid Charged particles Magnetic mat erial Dielectrics

Intensive paramet er Tension (T) Surface tension (,) P ressure (P) Electric pot ential ( cp) Magnetic field (B) External electric field (E)

Extensive paramet er Length (L) Surface area (A) St1rface area (A) Electric charge ( q) Total magnetization (M: Elect ric polarization (P)

For each system, there always exists one more pair of conjugate intensive and extensive parameters. They are temperature (T ) and ent ropy (S). Temperature is the ot her [email protected]

12

@Sk J ahiruddin, 2020

P hysicsguide

Basic Thermodynamics

intensive parameter and ent ropy is the corresponding conjugat e extensive parameter.

2.3

Thermodynamic State

11 oo : oo : 34 intensive parameter an entropy 1s t e correspon 1ng conjugate extensive parameter.

2.3

Thermodynamic State

Position and momentum coordinates are used to specify the state of a particle in mechanics. Similarly, the state of a t hermodynamic system can be specified by the given values of a set of thermodynamic parameters. For example, the state of a fluid system can be specified by t he pressure P, volume V, and t emperature T and specified as (P, V, T ). For an dielectric of polarization P at temperature T under an external electric field E , t he state is defined by (E , P , T). For a magnetic system t he state can be given by ( M , B , T ). For every thermodynamic systems there always exists t hree suitable thermodynamic parameters to specify t he st ate of t he system. It is important t o notice that thermodynamic parameters are all macroscopic measurable quantities. On t he other hand, microscopic quantities like position or moment um of the constit uent particles are not used for specifying t he state of a thermodynamic system.

j [email protected]

@Sk J ahiruddin, 2020

2.4

13

Physicsguide

Basic Thermodynamics

equilibrium

The equilibrium condition in mechanics is defined as: in

00: 00: 37 Basic Thermodynamics

2.4

equilibrium

The equilibrium condition in mechanics is defined as: in absence of external forces, if a particle is slightly displaced from its st able equilibrium posit ion it will come back to its original position aft er some time. Consider a thermodynamic system like gas in a cylinder. Suppose the gas is in a state defined by the given thermodynamic parameter values (P, V, T ) . The downward force (W ) due to the weight of the piston is just balanced by the upward force exerted by the pressure (P ) of the gas, and the system is in equilibrium. If the piston is slightly depressed and released , it will oscillate around the equilibrium posit ion for some t ime and slowly come to rest at t he original equilibrium position. It means that if a small external force is applied to the system and released , the system would come back to the thermodynamic st at e it was in originally, i.e., the values of all the extensive and intensive parameters would recover. This definition is very similar to the definition of equilibrium given in mechanics and known as mechanical equilibrium of a thermodynamic system. Apart from mechanical [email protected]

@Sk J ahiruddin, 2020

14

Physicsguide

Basic Thermodynamics

equilibrium, the syst em should have thermal and chemical

11 oo : oo : 39 @Sk J ahiruddin, 2020

Basic Thermodynamics

equilibrium, the system should have thermal and chemical equilibrium as vvell to achieve thermodynamic equilibrium of a system. Consider an isolated system with two partial systems. Initially each of them are in equilibrium at different t emperatures. Temperature at all points of each system are the same. They are now taken into thermal contact, only exchange of heat and no exchange of matter, with each other. Heat would flow from the system of higher temperature to t he system of lower temperature until uniform temperature is attained t hroughout t he combined system. The system is then in thermal equilibrium. Experience shows, all systems which are in thermal equilibrium with a given system are also in thermal equilibrium with each other. This principle defines t he temperature of a t hermodynamic system and known as zeroth law of thermodynamics. Hence systems which are in t hermal equilibrium with each other have a common intensive property, i.e., t emperature. Suppose the system is a mixture of several different chemical components. When t he composition of t he syst em remain fixed and definite, t he system is said to be in chemical equilibrium. Generally, chemical equilibrium takes a long [email protected]

@Sk J ahiruddin , 2020

15

P hysicsguide

Basic Thermodynamics

11 oo : oo : 43 [email protected]

15

@Sk J ahiruddin , 2020

Physicsguide

Basic Thermodynamics

t ime to achieve. Sometimes the system appears to be in chemical equilibrium, having fixed amount of components but the chemical reaction may continue with an extremely slow reaction rate. The mechanical equilibrium t herefore refers to uniformity of pressure, t he thermal equilibrium refers to uniformity of t emperature and the chemical equilibrium refers to t he constancy of chemical composit ion. If t here exist in t he system gradients of macroscopic parameters such as pressure, temperature, density, etc such a state of t he system is referred as a non-equilibrium state. A system which satisfies all possible equilibrium conditions is said to be in thermodynamic equilibrium. Thermodynamic equilibrium is thus correspond to the sit uation when t he t hermodynamic state does not change wit h time.

T = const. P = const. µ = const.

j [email protected]

Thermal equilibrium Mechanical equilibrium Chemical equilibrium

16

Physicsguide

11 oo : oo : 4s

j [email protected]

16

@Sk J ahiruddin , 2020

2.5

Physicsguide

Basic Thermodynamics

Equation of State

In equilibrium t he stat e variables are not all independent and are connected by equations of state. The role of statist ical mechanics is the derivation, from microscopic interact ions, of such equations of state. Simple examples are the ideal gas law,

where N is the number of molecules in the system and kB is Boltzmann's constant; the van der Waals equation,

where a , b are constants; the virial equation of state

1+

N B2(T) N 2 B3(T ) V + v2 + ... = 0

where the functions B 2 (T ), B 3 (T ) are called virial coef-

11 oo:oo:47 1+

NB2(T ) V

2

+

N B3(T ) v2

+ .. .

= 0

where t he functions B 2 (T ), B 3 (T ) are called virial [email protected]

17

Physicsguide

Basic Thermodynamics

@Sk J ahiruddin, 2020

ficients; and in the case of a paramagnet, the Curie law, M- C H = 0

T

where C is a constant called t he Curie const ant. These equations of st ates are approximations, and we shall use t hem primarily to illust rate various principles. The virial equation of state is, in principle, exact, but calculation of more t han a few of the virial coefficients is very difficult.

3

Thermodynamic transformation

A thermodynamic transformation is a change of state. If one or more of t he paramet ers of a syst em are changed, the state of the system changes. It is said t hat the system is undergoing a transformation or process. The transformat ion is generally from an initial equilibrium state to a final equilibrium st ate. T hermodynamic processes are classified into two groups (i) irreversible and (ii) reversible.

11 oo : oo : so t ion is generally from an initial equilibrium state to a final equilibrium state. Thermodynamic processes are classified into two groups (i) irreversible and (ii) reversible.

[email protected]

18

@Sk J ahiruddin, 2020

3.1

Physicsguide

Basic Thermodynamics

irreversible process

The water from the slopes of the Himalayas flows down the Ganges into the Indian Ocean. The water in the Indian Ocean will never go back to the hill on itself even if t he total energy loss during the down flow producing heat and sound energy is supplied back to the water at the Ocean. This means t hat the work done in the forward process is not equal to the work done in the backward process. Such natural flow of liquid downward is spontaneous and is irreversible. Almost all natural spontaneous processes are irreversible, just reversing the direction of the process it is not possible to get back the initial state. Consider free expansion of a gas. It does no work during t he free expansion however to compress it back to the original volume a large amount of work has to be performed on t he gas. A pendulum without a driving force will by itself cease to swing after some t ime, since its mechanical energy is transformed into heat by friction. The reverse process, that a pendulum starts swin~ bv itself while the surroundin~s cool. has never been

11 oo : oo : s3 work has to be performed on t he gas. A pendulum without a driving force will by itself cease to swing after some t ime, since its mechanical energy is transformed into heat by friction. The reverse process, that a pendulum starts swing by itself while the surroundings cool, has never been occurred. It is characteristic of irreversible processes that they proceed over non-equilibrium states dissipating energy in various forms during the transformation from one state j [email protected]

19

@Sk J ahiruddin, 2020

Physicsguide

Basic Thermodynamics

to t he other. Ferromagnets are magnetized by applying external magnetic field. If the external filed is reduced the magnetization curve does not follow the original path and forms a hysteresis loop because during magnetization the system dissipates energy in the form of heat and sound.

3.2

reversible process

In a reversible process, the change of states occurs only over equilibrium intermediate states. That is to say, all steps between the final and init ial states are in equilibrium during a reversible process. A reversible process is t hen an idealizat ion. Because, if a system is in thermodynamic equilibrium, t he parameters should not change with time. On t he contrary, in order to change the state one needs to change the parameter values. However, a reversible process could be realized in a quasi static manner.

11 oo : oo : 55 t he parameters should not change with time. On t he cont rary, in order to change the state one needs to change the paramet er values. However , a reversible process could be realized in a quasi static manner. In a quasi static process, infinitesimal change in t he paramet er values are made sufficient ly slowly compared to the relaxation time of t he system . Relaxation t ime is t he t ime required for a system to pass from a non-equilibrium state to an equilibrium state. Thus, if t he process rate is [email protected]

@Sk J ahiruddin , 2020

20

P hysicsguide

Basic Thermodynamics

erably less t han t he rate of relaxat ion, there will be enough t ime for the parameters to equalize over the entire system and t he system could be considered at equilibrium. The process will represent a cont inuous succession of equilibrium states infinitely close to each other and could be considered as a reversible process, reversing t he direction of t he process one could reach to the init ial state from the final state following the same path. The reversible change could be performed under different conditions. Consider a t hermally insulated system where no heat exchange is possible, any process under this condition is an adiabatic process. Reversible adiabatic process are also known as iso-entropic process. If a system undergoes a change keeping temperature constant , it is called an isothermal process, it is an isochoric process if volume kept constant and it is an isobaric process if pressure remains constant.

11 oo : oo : sa Reversible adiabatic process are also known as iso-entropic process. If a system undergoes a change keeping temperature constant, it is called an isothermal process, it is an isochoric process if volume kept constant and it is an isobaric process if pressure remains constant.

j [email protected]

Physicsguide

21

@Sk J ahiruddin, 2020

4 4.1

Basic Thermodynamics

Work Heat Internal energy Work

Work app ears during a change in state. The definition of work in t hermodynamics is borrowed from mechanics and it is given by -+ ➔ bW=-F-dR. (4.1) -+

where F is the force acting on the syst em during a small ➔ displacement dR.. The negative sign is a convention in t hermodynamics and it is decided by the fact that: work done by t he system is negative and work done on the system is positive. In mechanics, doing work t he potential or kinetic en_ __ ......,__ _ £J..L ,....

- --- J.. - . . . . -- .: _

- 1--- .. -- ............. .-l

Q ,: _____ .:l - . ..... 1 __

___ _ ___ , _ ,: _

---· -- =--- 1 - .... . . J..

11 oo : 01 : oo ➔

displacement dl!, . The negative sign is a convention in thermodynamics and it is decided by the fact that: work done by t he system is negative and work done on the system is posit ive. In mechanics, doing work t he potential or kinetic energy of the system is changed. Similarly, work is equivalent to energy exchange in t hermodynamics. Energy exchange is positive if it is added to a system and it is negative if it is subtracted from a system. Not e t hat, only macroscopic work is considered here, and not on an atomic level. Consider a gas enclosed in a cylinder at an equilibrium t hermodynamic state (P, V, T ). Assuming that there is no friction between the piston and the cylinder, the force acting on the gas, i.e., the weight on the piston, is F = PA [email protected]

22

Physicsguide

@Sk J ahiruddin, 2020

Basic Thermodynamics

where A is the cross sectional area of the piston. In order to compress the volume by an infinitesimal amount dV, the piston is pushed down by an infinitesimal amount di!,. The corresponding work done is bW

-I

=



- F · df

=

PAdi!,

=

- PdV

(4.2)

since pressure is acting in a direction opposite to the displacement and Adi!, = -dV during compression. The same definition is also valid for expansion. In case of expansion, t he pressure will act in the same di1·ection of the displacement and Adi!, = dV

11 oo : 01

: 03

definition is also valid for expansion. In case of expansion, the pressure will act in the same direction of the displacement and AdR = dV Thus, work is the product of an intensive state quantity (pressure) and the change of an extensive state quantity (volume) . One could easily verify that the same definition can be applied to t he other thermodynamic syst ems. For example, in case of dielectrics and magnetic mat erials, in order to change the electric polarization P or magnetization M by a small amount dP or dM in presence of electric field E or magnetic field H , the amount of work has to be performed on the syst ems are

JW

= E · dP

or

[email protected]

23

@Sk J ahiruddin, 2020

JW

= B · dM

(4.3) Physicsguide

Basic Thermodynamics

where E and B are the intensive parameters and P and M are ext ensive parameters. In order to change t he particle number by dN, one should add particles those have energy comparable to the mean energy of other particles otherwise equilibrium will be lost. Let us define

JW

= µdN

(4.4)

as the work necessary to change the particle number by dN. The intensive field quantity µ is called the chemical

11 oo : 01 : os (4.4)

V1 , t hen L).Q 1 > 0, i.e., the amount of heat L).Q 1 is absorbed by the gas from t he surroundings. Step 2: Adiabatic expansion of t he gas from ½ to V3 . The temperature decreases from T1 to T2 (T1 > T2 ). The 312 equation of state is ½/V2 = (T1 / T2) . since L).Q = 0

(5.10)

Step 3: Isothermal compression from ½ to ¼ at temperature T 2 . The equation of state is: P3 V3 = P 4V4 = RT2 . Again one has,

!1E3 = \

A /"'I

L). W 3 A

r ;r ,

+ L).Q2 = nrri

1 • ·~

0 (

V3\

11 oo : 01 : 3s Again one has,

(5.11)

since ½ > V4 , ~Q 2 < 0, the amount of heat is released by t he gas. Step 4: Adiabatic compression from V4 to V1 . Here temperature increases from T2 to T1 and the equation of state [email protected]

34

@Sk J a hiruddin , 2020

Physicsguide

Basic Thermodyna mics

(5.12)

The net change in internal energy ~E = ~E1 + ~E2 + ~E3 + ~E4 = 0 as it is expected. Consider the amount of heat exchanged during the isothermal processes:

and

½ ~Q2 = - RT2ln ¼

(5.13)

During t he adiabatic processes, one has combining t he equation of st ates

11 oo : 01

: 37 (5.13)

During t he adiabatic processes, one has combining t he equation of states (5.14)

This implies (5.15)

If the Carnot 's cycle is made of a large number of infinitesimal steps, the above equation modifies to 6Q

T j [email protected]

35

@Sk J ahiruddin, 2020

=0

(5.16) Physicsguide

Basic Thermodynamics

This is not only true for Carnot 's cycle but also t rue for any reversible cyclic process. Suppose that t he state of a t hermodynamic system is changed from state 1 to stat e 2 along a path C1 and the system is taken back to t he initial state along another reversible path C2 , as shown in Figure 5.2. Thus, 1 - C 1 - 2C2 - 1 forms a closed reversible cycle and one has

X

1

11 oo : 01

: 40

1

y Figure 5.2: A reversible cyclic process on a XY diagram where X and Y form a conjugate pair of thermodynamic variables. 2

or 1

[email protected]

8Q T

+

l

2

8Q T

=0

P hysicsguide

36

@Sk J ahirudd in, 2020

Basic Thermodynamics

Since, the paths are reversible, one also has

8Q T

l 2

so hence 2 1

8Q T

2 1

2

1

8Q T 8Q T

(5.18)

(5.19)

Thus t he integral J 8Q/ T is path indep endent, i .e., independent of t he process of heating or cooling the system. The integral depends only on the init ial and final states of t he syst em and t hus represents a state function whose total differential is 8Q / T. since heat is an extensive quantity,

11 oo : 01

: 42

pendent of the process of heating or cooling the system. The integral depends only on the initial and final states of t he system and t hus represents a state function whose total differential is bQ /T. since heat is an extensive quantity, t his st ate function, say S, is also extensive whose conjugate intensive parameter is temperature T. This extensive state function is t he entropy S and defined as dS

= bQ T

(5.20)

and

Note that, only ent ropy difference could be measure, not the absolute entropy. The statistical mechanical definit ion of entropy will be given in t he section of st atistical mechanics.

[email protected]

Physicsguide

37

@Sk J ahiruddin, 2020

5.3

Basic Thermodynamics

Second Law

The second law of thermodynamics introduces the entropy S as an extensive state variable and states that for an infinitesimal reversible process at temperature T , t he heat given to t he system is

aQ rev

=

TdS

11 oo : 01 : 4s

aQ rev

=

TdS

while for an irreversible process

a Q irrev

~ TdS

If we are only interested in thermodynamic equilibrium states we can use a Q rev = T dS and t reat the ent ropy S as t he generalized displacement which is coupled to the 'force' T. The above formulation of t he second law is due to Gibbs We present next two equivalent statements of the second law of t hermodynamics. The Kelvin version is: There exists no thermodynamic process whose sole effect is to extract a quantity of heat from a j [email protected]

@Sk J ahiruddin, 2020

Physicsguide

38

Basic Thermodynamics

system and to convert it entirely to work.

The equivalent stat ement of Clausius is:

No process exists in which the sole effect is that heat flows from a reservoir at a given temperature to a reservoir at a higher temperature.

11 oo : 01

: 47

No process exists in which the sole effect is that heat flows from a reservoir at a given temperature to a reservoir at a higher temperature.

A corollary of t hese statements is that the most efficient engine operating bet ween two reservoirs at temperatures T1 and T2 is the Carnot engine. The first law of t hermodynamics tells us about t he conservation of energy in a t hermodynamic process during its change of st at e. The second law tells us about t he direction of a natural process in an isolated system. The ent ropy S = 8Qrev/T is t he amount of heat reversibly exchanged with the surroundings at temperature T . since t he amount of heat 8Qirr exchanged in an irreversible process is always less than that of 8Qrev exchanged in a reversible process, it

[email protected]

39

@Sk J ahiruddin , 2020

Physicsguide

Basic Thermodynamics

is t hen always true that 8Qirr < 8Qrev = T dS

(5.21)

For an isolated system, 8Qrev = 0. Therefore, in an isolated system t he ent ropy is const ant in t hermodynamic equilibrium and it has an extremum since dS = 0.

11 oo : 01 : so 6Qirr < 6Qrev = TdS

(5.21)

For an isolated system, 6Qrev = 0. Therefore, in an isolated system t he ent ropy is constant in t hermodynamic equilibrium and it has an extremum since dS = 0. It is found t hat in every sit uation this extremum is a maximum. All irreversible processes in isolated system which lead to equilibrium are t hen governed by an increase in ent ropy and t he equilibrium will be reestablished only when t he entropy will assume its maximum value. This is t he second law of thermodynamics. Any change of state from one equilibrium st at e to another equilibrium state in an isolated syst em will occur naturally if it corresponds to an increase in entropy. For reversible process the ent ropy change is zero.

dS= 0

(5.22)

and for irreversible processes the entropy change is al-

j [email protected]

40

@Sk J ahiruddin , 2020

P hysicsguide

Basic Thermodynamics

ways posit ive

dS > 0

(5.23)

Note t hat, entropy could be negative if there is heat exchange wit h the surroundings i.e., t he system is not an iso-

11 oo : 01 : s2 ways posit ive dS > 0

(5.23)

Note t hat, entropy could be negative if there is heat exchange with the surroundings i.e., the system is not an isolated system. It is posit ive only for an isolated system. The first law for reversible changes now can be rewritten in terms of entropy: Fluid system: Surface film:

dE = T dS - P dV dE = T dS + ,ydA

Strained wire: Magnetic materials:

If there is exchange of energy of several different forms, the first law should take a form

dE = TdS - P dV

5.4

+ H dM + µdN + · · ·

(5.24)

Third Law

The third law of thermodynamics deals with t he entropy of a system as the absolute t emperature tends to zero. It is [email protected]

@Sk J ahiruddin , 2020

already seen t hat

41

Physicsguide

Basic Thermodynamics

c

c

11 oo : 01

: 55

@Sk J ahiruddin, 2020

Basic Thermodynamics

already seen t hat

(5.25) and one can only measure the ent ropy difference between two states. The absolute value of ent ropy for a given thermodynamic states remains undetermined because of the arbitrary additive constant depending on the choice of the initial state. The t hird law enables us to determine t he additive constant appearing in the definition of entropy. It states t hat: t he ent ropy of every system at absolute zero can always be taken equal to zero,

lim S=O

T➔ O

(5.26)

The zero temperat ure ent ropy is t hen independent of any other properties like volume or pressure of the system. It is generally believed t hat the ground state at T = 0 is a single non-degenerate state. It is therefore convenient to choose this nondegenerate state at T = 0 as the standard initial state in the definition of entropy and one could set t he entropy of t he standard state equal to zero. The entropy of any state A of t he system is now defined, including the [email protected]

@Sk J ahiruddin, 2020

42

P hysicsguide

Basic Thermodynamics

11 oo : 01 : s1 j [email protected]

42

Physicsguide

@Sk J ahiruddin , 2020

Basic Thermodynamics

additive constant , by the integral

S(A) =

A

T =O

,SQ T

(5.27)

where the integral is taken along a reversible transformation from T = 0 state (lower limit) to the state A . since dQ = G(T) dT , the entropy of a system at temperature T can also be given as T

Gv(T ) dT

T

0

T

or

Gp(T ) dT

T

0

(5.28)

when the system is heated at constant volume or constant pressure. As a consequence of the third law S (0) = 0, the heat capacities Gv or Gp at T = 0 must be equal to zero otherwise the above integrals will diverge at the lower limit. Thus, one concludes

Gv

or

Gp



0

as

T



0

(5.29)

The results are in agreement with the experiments on t he sp ecific heats of solid

j [email protected]

43

Physicsguide

11 oo : 02 : oo j [email protected]

42

Physicsguide

@Sk J ahiruddin , 2020

Basic Thermodynamics

additive constant , by the integral

S(A) =

A

T =O

,SQ T

(5.27)

where the integral is taken along a reversible transformation from T = 0 state (lower limit) to the state A . since dQ = G(T) dT , the entropy of a system at temperature T can also be given as T

Gv(T ) dT

T

0

T

or

Gp(T ) dT

T

0

(5.28)

when the system is heated at constant volume or constant pressure. As a consequence of the third law S (0) = 0, the heat capacities Gv or Gp at T = 0 must be equal to zero otherwise the above integrals will diverge at the lower limit. Thus, one concludes

Gv

or

Gp



0

as

T



0

(5.29)

The results are in agreement with the experiments on t he sp ecific heats of solid

j [email protected]

43

Physicsguide

11 oo : oo : 01

Thermodynamic potentials and applications Sk J ahiruddin

Assistant P rofessor· Sister Nibedita Govt. College, Kolkata Author was the topper of IIT Bombay lVI. Sc Physics 2009-2011 batch He ranked 007 in IIT JAlVI 2009 and 008 (JRF) in CSIR NET June 2011 He has been teaching CSIR NET aspirants since 2012

1

@Sk J ahiruddin, 2020

Thermodynamic potentials and applications

11 oo : oo : 04 1

@Sk J ahiruddin, 2020

Thermodynamic potentials and applications

Contents 1 Thermodynamic Potentials

4

1.1

Entropy as a t hermodynamic potential: . . .

5

1. 2

Enthalpy as a t hermodynamic potential: . .

6

1.3

Helmholtz free energy as a t hermodynamic potential: •



8

Gibbs free energy as a thermodynamic pot ential: . . . . . . . . •

9



1.4

























































1.5

Grand potential as a thermodynamic potential: 11

1.6

Maxwell's relations





























13

1.7

Response functions





























14





























14

1.8

1.7.1

Definit ions .

1.7.2

Relations between t he response function 15

Thermodynamics of a magnetic system 1.8.1

j [email protected]

Definition of potentials 2

















17







18

Physicsguide

11 oo : oo : 06 1.8.1

Definition of potentials

j [email protected]















18

Physicsguide

2

@Sk J ahiruddin, 2020

2



Thermodynamic potent ials and applicat ions

1.8.2

Maxwell relations for magnetic systems 20

1.8.3

Response functions for magnetic systems . . . . . . . . . . . . . . . . . .

Some applications of thermodynamics

20

22

2.1

TdS equations . . . . . . .





















22

2.2

Internal energy equations .





















23

2. 3

Heat transfer of isothermal expansion of van der Waals gas . . . . . . . . . . . . . . . . .

24

2.4

Reversible isothermal change of pressure . .

25

2.5

Internal energy of van der Waal gas











27

2.6

Heat capacities of ideal gas .



















28

2.7

Gibbs paradox:

































30

2.8

Radiation: . . .

































31

11 oo :oo :oa •









[email protected]





























Physicsguide

3

@Sk J ahiruddin, 2020

1



Thermodynamic potent ials and applications

Thermodynamic Potentials

By the second law of thermodynamics, an isolated system during a spontaneous change reaches an equilibrium st ate characterized by maximum entropy:

dS = 0,

S = Smax

(1.1)

On the other hand, it is known from mechanics, electrodynamics and quantum mechanics t hat a syst em which is not isolated minimizes its energy. An interacting t hermodynamic syst em always exchanges heat or perform work on t he surroundings during a spontaneous change to minimize its internal energy. However, t he entropy of the system plus the surroundings, which could be thought as a whole an isolated syst em, always increases. Thus, a non-isolated system at constant entropy always leads to a st at e of minimum energy.

11 oo : oo : 11 energy.

[email protected]

@Sk J ahiruddin, 2020

1.1

P hysicsguide

4

Thermodynamic potentials and applications

Entropy as a thermodynamic potential:

Both entropy and t he internal energy are state funct ions. If t hey are known as function of state variables of an isolat ed syst em then all ot her thermodynamic quantities are complet ely known. Consider t he internal energy E = E (S, V, N) t hen , t he different ial form of energy is

dE = TdS - P dV

+ µdN

(1.2)

So, t he temperat ure and pressure are known as fun ctions of other state variables

T=

oE as

V,N

'

- P=

oE av

S ,N

'

µ=

oE 8N s,v

(1.3)

Similarly, consider t he entropy S

TdS = dE

= S(E, N , V ), then

+ P dV

- µdN

and t he t emperature and pressure can be found as

(1.4)

11 oo : oo : 14 Similarly, consider the entropy S = S(E, N , V ), then

TdS = dE + P dV - µdN

(1.4)

and t he temperature and pressure can be found as 1

T

as aE

V,N

'

P=T

as av

E ,N

'

µ=- T

as aN

EV '

(1.5) j [email protected]

@Sk J ahiruddin, 2020

5

Physicsguide

Thermodynamic potentials and applications

The entropy and the internal energy then can be calculated as functions of t he state variables form t he equation of state. Since t he equilibrium state of the system is given by a maximum of the entropy as a fun ction of (E , V), it gives information about the most stable equilibrium state of the system as potential energy does in mechanics. As the difference in potential energy defines the direction of a natural process in mechanics, t he entropy difference determines the direction of a spontaneous change in an isolated system. Thus, the ent ropy can be called as a t hermodynamic potential.

1.2

Enthalpy as a thermodynamic potential:

The enthalpy of a syst em is defined as

11 oo : oo : 16 tial: The enthalpy of a system is defined as

(1.6)

H =E+PV

t he differential form is dH = dE+ P dV + VdP

=:>~>

dH = TdS

+ V dP + µdN (1.7)

[email protected]

6

@Sk J ahiruddin , 2020

Physicsguide

Thermodynamic potentials and applications

Knowing t he enthalpy H = H (S, P , N) , the state variables can be calculated as T =

8H

as

P ,N

'

V =

8H BP

, S,N

µ=

8H 8N

S,P

(1.8) Consider an isolated system at constant pressure. Process at constant pressure are of special interest in chemistry since most of the chemical reactions occur under constant atmospheric pressure. In an isolated-isobaric system, bQ = 0 and P is constant , thus

dE+PdV = 0

=:>~>

dH

=0 (1.9)

In a spontaneous process of an adiabatic-isobaric system, t.h P Pn11ilihri11m rnrrPsnnnrls t.n t.h P minim11m nf t.hP Pnt.h::i.lnv

11 oo : oo : 19 ===>~►

dE+PdV = 0

+ PV ) = 0

d( E

===>~►

dH = 0 (1.9)

In a spontaneous process of an adiabatic-isobaric system, t he equilibrium corresponds to t he minimum of the enthalpy

dH = 0,

H (S, P ) =

[email protected]

7

@Sk J ahiruddin, 2020

1.3

(1.10)

H min

P hysicsguide

Thermodynamic potent ials and applications

Helmholtz free energy as a thermodynamic potential:

The Helmholtz potential (free energy) is defined as

F=E-TS

(1.11)

t he differential form

dF = dE-SdT-TdS

dF = -SdT - P dV +µdN (1.12) since dE = T dS- P dV +µdN . Thus, knowing F = F (T , V, N), S, P and µ could be determined as

8F 8T

V,N

'

~~►

- P=

8F

av

T ,N

'

µ

=

8F 8N

T ,V

11 oo : oo : 21 since dE = T dS - PdV +µ dN . Thus, knowing F S, P and µ could be determined as

- S=

8F 8T

V,N

'

8F

- P =

av

T ,N

'

µ=

=

F (T , V, N),

8F 8N

TV

'

(1.13) The Helmholtz potential is useful in defining t he equilibrium

of a non-isolated system in contact with heat bath at constant temperature T . The system is interacting wit h t he heat bath through heat exchange only. Consider an arbit rary isothermal t ransformation of this system from a state A to state B. By t he second law , one have (1.14) j [email protected]

@Sk J ahiruddin, 2020

8

P hysicsguide

Thermodynamic potentials and applications

Since T is constant (1.15)

where ~Q is the amount of heat absorbed during the transformat ion and ~ S = S(B) - S(A). Using t he first law, the inequality could be written as (1.16)

where ~ W is the work done by t he system. Thus, the equilibrium of an isothermal system which does not perform work (mechanically isolated) always looks for a minimum of Helmholtz potential. Irreversible process happen spontaneously, until t he minimum

11 oo : oo : 24 librium of an isothermal system which does not perform work (mechanically isolated ) always looks for a minimum of Helmholtz potential. Irreversible process happen spont aneously, until t he minimum dF

=

0,

F

(1.17)

= Fmin

is reached.

1.4

Gibbs free energy as a thermodynamic potential:

The Gibb's potential (free energy) is defined as G = E - TS + PV = F + PV [email protected]

Physicsguide

9

@Sk J ahiruddin , 2020

(1.18)

Thermodynamic potentials and applications

t he differential form dG = - SdT + V dP

+ µd N

(1.19)

since E = T S - PV + µ N for a system attached wit h heat bath as well as wit h a bariost at . System exchanges heat and does some work due to volume expansion at constant pressure. The thermodynamic variables can be obtained in terms of G (P, T , N) as

- S=

ac 8T

P,N

'

V=

ac 8P

T ,N

'

µ=

ac 8N

T ,P

11 oo : oo : 21 pressure. The thermodynamic variables can be obtained in terms of G(P, T , N) as

ac

-S=

8T

'

P,N

ac

V=

8P

T ,N

'

µ=

ac 8N

T ,P

(1.20) Notice that the chemical potentialµ can be defined as Gibb's free energy per particle. Consider a system at constant pressure and temperature. For isothermal process, (1.21)

as it is already seen. If the pressure remain constant P~V, t hen

~W

=

(1.22)

Thus, a syst em kept at constant temperature and pressure, t he Gibb's free energy never increases and the equilibrium j [email protected]

10

@Sk J ahiruddin , 2020

Physicsguide

Thermodynamic potent ials and applicat ions

state corresponds to minimum Gibb's potent ial. Irreversible spontaneous process in an isothermal - isobaric system dG

=

0 G

= G roin

(1.23)

are always achieved.

1.5

Grand potential as a thermodynamic J

J



1

11 oo : oo : 30 are always achieved.

1.5

Grand potential as a thermodynamic potential:

The grand potential is defined as

qJ

=

E - T S - µN

=

F - µN

= -

PV

(1.24)

since E = TS - PV + µN . The syst em attached with heat bath as well as with a part icle reservoir. System exchanges heat with heat bath and exchanges particle with the particle reservoir. Differentially the grand potential can be expressed as

dqJ = dE - TdS - SdT- µdN - Ndµ

(1.25)

= -SdT - P dV - Ndµ

[email protected]

Physicsguide

11

@Sk J ahiruddin, 2020

Thermodynamic potentials and applications

since dE = TdS - PdV + µdN. The thermodynamic variables are then obtained in terms of qJ (V, T , µ) as

-S =

8T

, -P = V ,µ

av

T ,µ

'

-N=

aqJ



TV '

(1.26)

Consider an isothermal system at constant chemical potent ial. For an isothermal svstem

11 oo : oo : 32 -S=

ar

, V,µ

-P=

av

, -N= T ,µ

a 8µ

TV ,

(1.26) Consider an isothermal system at constant chemical potent ial. For an isothermal system (1.27)

and ~ W = - µ~N since µ is constant and the inequality leads to (1.28) Thus, a system kept at constant temperature and chemical potential, the grand potential never increase and the equilibrium stat e corresponds to minimum grand potential. Irreversible spontaneous process in an isothermal system with constant chemical potential correspond to (1.29)

[email protected]

@Sk J ahiruddin, 2020

1.6

12

Physicsguide

Thermodynamic potentials and applications

Maxwell's relations

A number of relations between the thermodynamic state variables can be obtained since the thermodynamic potent ials E, H , F and G ( also ) are state functions and have

11 oo : oo : 3s axwe A number of relations between the thermodynamic state variables can be obtained since the thermodynamic potent ials E , H , F and G ( also ) are state functions and h ave exact different ials. We know for exact differentials dcp

aM ay

= Mdx + Ndy

aN ax

X

(1.30) y

• From dE

aT av

= T dS- P dV s



aT aP

dH = TdS + VdP

• dF

= -SdT - PdV

av as

8

as aP

= -SdT + VdP

(1.31)

v

as av

• dG

aP as

p

aP aT

r

r

(1.32)

v (1.33)

av aT

p

(1.34) j [email protected]

@Sk J ahiruddin, 2020

1. 7 1. 7 .1

13

Thermodynamic potent ials and applications

Response functions D efinit ions

Physicsguide

00: 00: 37 Thermodynamic potent ials and applications

1. 7 1. 7 .1

Response functions Definitions

A great deal can be learned about a macroscopic system t hrough its response t o various changes in externally cont rolled parameters. Important response functions for a PVT syst em are t he specific heats at constant volume and pressure,

Cv= Gp=

aQ 8T

as

=T

8T

V

aQ BT p

as

=T

V

(1.35)

8T p

The isothermal and adiabatic compressibilities, 1 K T= - -

V

av 8P

av

1 K s=- -

T

(1.36)

8P s

V

and the coefficient of thermal expansion 1

av

v

fJT

O'.= -

[email protected]

@Sk J ahiruddin, 2020

t his can be writ t en as

14

(1.37) P,N P hysicsguide

Thermodynamic potent ials and applications

11 oo : oo : 39 @Sk J ahiruddin, 2020

Thermodynamic potentials and applications

t his can be written as a,p

1. 7.2

=

1 V

av aT

P

1 V

(1.38)

Relations between the response function

Intuit ively, we expect the specific heats and compressibilit ies to be posit ive and Gp > Gv, K r > K 8 . In this section we derive relations between t hese response functions. The intuition that the response functions are positive will be justified in the following section in which we discuss t hermodynamic stability. We begin with t he assumption t hat the entropy has been expressed in t erms of T and V and t hat the number· of particles is kept fixed . Then

as as dS = ar V dT+ av dV T as as as av T aT p =T aT +T av r fJT V as av Gp - Gv = T fJT p av

p

(1.39)

T

We now use t he Maxwell relations and the chain rule [email protected]

@Sk J ahiruddin , 2020

15

Physicsguide

Thermodynamic potentials and applications

11 oo : oo : 43 15

[email protected]

@Sk J ahiruddin , 2020

Physicsguide

Thermodynamic potentials and applications

az ax

y

ay az

X

ax ay

=-1

(1.40)

z

We get

as av r Gp -

aP ar v aP Cv = -T av

T

aP av av r ar p 2 av = -rv 0'.2 aT P Kr

(1.41)

In a similar way we obtain a relation between the compressibilit ies and Assume that t he volume has been obtained as function of S and P.

Kr

Ks .

V

Then we get

-

av av dV= aP dP+ as p dS s 1 av 1 av 1 av as - - V aP T V aP s V as p aP 1 av as Kr- Ks=- -V as p aP

(1.42) T

T

The Maxwell relation and

av as j [email protected]

av p

8T 16

p

as

- l

aT

p Physicsguide

11 oo : oo : 4s av

av as

8T

p

j [email protected]

p

as aT

-i

p

16

@Sk J ahiruddin, 2020

Physicsguide

Thermodynamic potent ials and applications



gives

TV 2 Kr -Ks = Gp a

(1.43)

Now we have useful relations

Gp (K r - K s) And

1.8

=

Kr (Gp - Gv)

Gp Gv

Kr Ks

=

TV a 2

(1.44)

(1.45)

Thermodynamics of a magnetic system

In order to study magnetic properties of matter one requires t he expression for the work of magnetizing a m at erial. One needs to be careful in defining precisely the system and the processes in order to calculate m agn etic work done. Let us assume that the effect s of pressure and volume on a magnetic system is negligible.

11 oo:oo:47 processes In or er o ea cu a e magne IC wor e us assume that t he effects of pressure and volume on a magnetic system is negligible.

[email protected]

@Sk J ahiruddin, 2020

1.8 .1

17

P hysicsguide

Thermodynamic potentials and applications

Definition of potentials

T he thermodynamic parameters of a magnetic system are going to be t he external magnetic induction B, total magnetization M and t emperature T instead of P, V and T of a fluid system. The first law of t hermodynamics: t he differential change in internal energy E for a reversible change of state can be writ t en in two different but equivalent forms as

dE = T dS-MdB

(1.46)

dE = TdS+BdM

(1 .47)

or

we will use t he SECOND form We define other state functions and t hermodynamic p otentials such as ent halpy H (N, S, B) , the Helmholtz free energy F (N, M , T) and t he Gibbs free energy G(N, B , T ). T he definit ions of t hese t hermodynamics state functions and differential change in a reversible change of state are given by

11 oo : oo : so energy F (N , M , T) and the Gibbs free energy G(N , B , T ). The definit ions of t hese t hermodynamics state functions and differential change in a reversible change of state are given by

[email protected]

18

@Sk J ahiruddin , 2020

P hysicsguide

Thermodynamic potent ials and applicat ions

H (N , S , B ) = E - M B F (N , M , T ) = E - TS G(N , B , T ) = E - TS - MB

and and and

dH = TdS-MdB dF = -SdT + BdM dG = - SdT - MdB (1.48)

where explicit N dep endence is also avoided. If one wants to take into account of number of part icles t here must be another t erm µdN in all different ial forms of t he state functions. It can be noticed t hat the thermodynamic relat ions of a magnetic syst em can be obtained from t hose in fluid system if V is replaced by - M and P is replaced by

B. T he thermodynamic parameters can be obtained as

T =

S = -

aE as

or

T =

NI

aF

ar

( aE\

or M

S= -

aH as

ac ar ( aF\

(1.49) B

(1.50) B

11 oo : oo : s3 as

or

as

NI

(1.49) B

aF ac or S =S =aT M aT B aE aF or B = B= aM s aM T aH ac or M =M =aB s aB T j [email protected]

@Sk J ahiruddin , 2020

1.8.2

(1.50) (1.51) (1.52) Physicsguide

19

Thermodynamic potentials and applications

Maxwell relations for magnetic systems

• dE = TdS + BdM

aT aM s

aB

as

M

(1.53)

• dH = TdS-MdB

• dF = -SdT + BdM

aT aB

aM 8

as

B

(1.54)

as

aB

aM r

ar

M

(1.55)

• dG = -SdT - MdB

as

aM

aB r

ar

B

(1.56)

11 oo : oo : 55 •

as

dG = -SdT - MdB

f)B

8M T

f)T

B

(1.56)

1.8.3

Response functions for magnetic systems

The specific heats CM and CB are t he measures of the heat absorption from a temperature stimulus at constant magne20

[email protected]

P hysicsguide

Thermodynamic potentials and applications

@Sk Jahiruddin, 2020

tization and constant external magnetic field respectively. The definition of heat capacities are :

as 8T

= -T JVI

as 8T

(1.57)

=-T P

f)T2 B

In case of magnetic systems, instead of isothermal and adiabatic compressibilities one have the isothermal and the adiabatic magnetic susceptibilities

xr= and Xs

=

8M f)B

(1.58)

T

8M 8B

s

f)B2

s

(1.59)

where M is the total magnetization and B is the external magnetic field. Note that t he normalizing factor of 1/V is

11 oo : oo : sa 8M (1.59) Xs == 8B 8 8B 2 8 where M is the total magnetization and B is the external magnetic field. Note t hat t he normalizing factor of 1/V is absent here. T he change of magnetization M with respect to temperature T under constant external magnetic field a.B, is defined as

8M

ar j [email protected]

(1.60) B 21

@Sk J ahiruddin , 2020

P hysicsguide

Thermodynamic potentials and applications

The response functions are not all independent of one another. It can be shown t hat (1.61)

T wo other relations you can prove are XT (CB - CM)== T a.~

2

and

CB (XT - xs) == T a.1 (1.62)

Some applications of thermody•

nam1cs .

.

11 oo : 01 2

: 01

Some applications of thermody•

nam1cs 2.1

TdS equations

The entropy of a pure subst ance can be considered as a function of any two variables, such as T and V; thus,

__ as d as d ds T + V ar v av r as dT T as dV Tds = T ar + av V T [email protected]

@Sk J ahiruddin, 2020

since TdS follows that

22

(2.1)

Physicsguide

Thermodynamic potentials and applications

aQ for a reversible isochoric process, it

T as ar

= Cv

v and from t he Maxewells relation (

TdS = CvdT + T

(2.2)

gt )T = (if)v aP

ar v

dV

(2.3)

This is t he firs TdS equation. Similarly t ake entropy as a function of T and P and derive the second TdS equation /f"\yr'\

11 oo : 01

: 03

This is t he firs TdS equation. Similarly t ake ent ropy as a function of T and P and derive the second TdS equation TdS

2.2

== CpdT - T

av aT

dP

(2.4)

p

Internal energy equations

For the internal energy equation we start from dU

== TdS - PdV

(2.5)

and take partial derivative with respect to V keeping T constant

au

av

T

jahir@physicsguide. in

== T

as av

- P T

23

@Sk J ahiruddin , 2020

(2.6) Physicsguide

Thermodynamic potentials and applications

Using Maxwell's t hird relation, (aS/ 8V )T == (aP/ aT)v, we get

au

av

== T aP T

ar

_P v

(2.7)

This equation is very useful. Similarly t aking partial derivative wit h respect to P both side of the equation dU == T dS - P dV prove that I

~ TT\

11 oo : 01

: 01

This equation is very useful. Similarly t aking partial derivative wit h resp ect to P both side of the equation dU = T dS - P dV prove that

au

av

=-T

8P r

2.3

8T

_ P av 8P r

P

(2. 8)

Heat transfer of isothermal expansion of van der Waals gas

Start with

TdS = Cv dT + T

8P BT

dV V

From van der Waal's equation

P = RT V-b j [email protected]

@Sk J ahiruddin , 2020

24

(2.9) P hysicsguide

Thermodynamic potentials and applications

8P 8T v Hence

a V2

R V - b

dV T dS = CvdT + RT V _ b

(2.10)

since T is constant, Cv dT = O; and , since the process is reversible, q = J Tds Therefore, V1

q = RT

½ ?.

dV V-b

(2.11)

11 oo : 01

: 01

This equation is very useful. Similarly t aking partial derivative wit h resp ect to P both side of the equation dU = T dS - P dV prove that

au

av

=-T

8P r

2.3

8T

_ P av 8P r

P

(2. 8)

Heat transfer of isothermal expansion of van der Waals gas

Start with

TdS = Cv dT + T

8P BT

dV V

From van der Waal's equation

P = RT V-b j [email protected]

@Sk J ahiruddin , 2020

24

(2.9) P hysicsguide

Thermodynamic potentials and applications

8P 8T v Hence

a V2

R V - b

dV T dS = CvdT + RT V _ b

(2.10)

since T is constant, Cv dT = O; and , since the process is reversible, q = J Tds Therefore, V1

q = RT

½ ?.

dV V-b

(2.11)

11 oo : 01

: 1o

since T is constant, CvdT = O; and, since the process is reversible, q = J T ds Therefore, q = RT

(2.1 1)

V-b

½

Finally V1- b q= RTlnV:-b

(2.12)

'l

2.4

Reversible isothermal change of pressure

When T is constant TdS

=

-T

oV 8T

dP

and so

Q

=

p

-T

av 8T

dP p

(2.1 3)

The volume expansivity is

av 8T [email protected]

p

25

@Sk J ahiruddin , 2020

Physicsguide

Thermodynamic potentials and applications

t hen Q

=-T

V f3dP

(2.14)

In t he case of a solid or liquid, V and /3 do not change much with pressure. So we can take t hem out of the integration to get ,........

,



-

___ ,

11 oo : 01

: 13

In t he case of a solid or liquid , V and f3 do not change much with pressure. So we can take t hem out of the integration to get

It is seen from t his result that, as t he pressure is increased isothermally, heat will flow out if /3 is insositive but t hat , prossunce with a negative expansivity (such as water between O and 4°C, or a rubber band) , an isothermal increase of pressure causes an absorption of heat. If the pressure on 15cm3 of mercury at 20°C is increased reversibly and isothermally from O to lOOOatm, the heat t ransferred will be approximately

where T = 293K, V = 2 x 10- 5m 3 , f3 and P1 = 1.01 x 108 Pa Hence, [email protected]

1.81 x 10- 4 K- 1 ,

26

@Sk J ahiruddin , 2020

Physicsguide

Thermodynamic potent ials and applicat ions

5

Q = - (293K) (1. 5 x 10- m

= -80.3N · m = -80.3J

=

3

)

4

(1.81 x 10- K-

1

)

(1.01 x 108 Pa)

11 oo : 01 Q

= - (293K) (1.5 x = -80.3N · m = -80.3J

2.5

: 1s

5 3 10- m )

4

(1.81 x 10- K-

1

)

(1.01 x

8 10 Pa)

Internal energy of van der Waal gas P = RT _ a V-b V 2

and

8P 8T

R v

V - b

so

au

av

R

T

=

RT

TV - b - V - b +

so dU

=

a V2

=

a V2

a CvdT + V 2 dV

(2.16)

(2.17)

Finally

U= j ahir@physicsguide. in

@Sk J ahiruddin , 2020

2.6

a CvdT - V

+ const .

27

(2.18) Physicsguide

Thermodynamic potentials and applications

Heat capacities of ideal gas

Th P. rliffP.rP.nr.P. hP.t,wP.P.n t.h P. hP.::i.t. r.::i.n::i.r.it.i P.s is Q'ivP.n hv

11 oo :01

:1

@Sk J ahiruddin , 2020

2.6

a

Thermodynamic potentials and applications

Heat capacities of ideal gas

The difference between t he heat capacit ies is given by

Gp -

2 Cv = TVn /""T

(2.19)

t

where a = (i~) P is the volume expansion coefficient and ""T = ~~) T is the isothermal compressibility. Using t his relation, t he difference between heat capacities of an ideal gas could be easily obt ained. For one mole of an ideal gas, t he equat ion of state is P V = RT where R is the universal gas constant . T hus,

-t (

1

Q=-

V

av aT

l R -VP

p

1

(2.20)

-

T

and 1 ""T = - -

V

av 8P

1

--

V

T

V -p

1 -

p

(2.21)

PV =R T

(2.22)

so the differences

Cp- Cv

=

TV

-

1

T

2

1

1/ P

--

As T -+ 0, by third law of t hermodynamics, the ent ropy S -+ S0 = 0 becomes independent of all parameters like [email protected]

@Sk J ahiruddin , 2020

28

Physicsguide

Thermodynamic potentials and applications

pressure, volume and temperature. T herefore, in t he limit

11 oo : 01

@Sk J ahiruddin, 2020

: 21

Thermodynamic potent ials and applications

pressure, volume and temperature. Therefore, in the limit T --+ 0 the heat capacities also tend to zero,

Cv=T

as ar

v

--+ 0

and

Cp=T

as aT

p

--+ 0 (2.23)

This is because of the fact that as T --+ 0, the system tends to settle down in its nondegenerate ground state. The mean energy of t he system then become essentially equal to its ground state energy, and no further reduction of temperature can result in a further reduction of mean energy. Not only t he individual heat capacity goes to zero but also t heir difference goes to zero as T --+ 0. Because, the volume expansion coefficient a also goes to zero 1 a=V

av 8T

as aP

P

r

--+ 0

(2.24)

However, the compressibility ""T, a purely mechanical property, remains well-defined and finite as T--+ 0. Thus,

j [email protected]

@Sk J ahiruddin, 2020

29

Physicsguide

Thermodynamic potent ials and applicat ions

11 oo : 01

: 23

[email protected]

29

@Sk J ahiruddin , 2020

P hysicsguide

Thermodynamic potent ials and applications

as

T



Gp - Gv

0



0

(2.25)

This is not in contradiction to the ideal gas result Gp Gv = R because as T ➔ 0, the system approaches its ground state and quantum mechanical effects become very important. Hence the classical ideal gas equation PV = RT is no longer valid as T ➔ 0.

2. 7

Gibbs paradox:

An isolated syst em with two parts of equal volume V each contains N number of molecules of the same monatomic perfect gas at the same t emperature T , pressure P . Init ially, t he two parts were separated by a membrane and t hen the membrane was removed. The syst em is allowed to equilibrate. The change in entropy is given by

oQ T

~

T

oQ

(2.26)

From first law, for a perfect gas dE(T) = oQ + oW = 0 and t hus oQ = -oW = P dV. The volume changes from V to

[email protected]

30

Physicsguide

11 oo : 01

: 26

t hus bQ = -bW = P dV. The volume changes from V to

j [email protected]

Physicsguide

30

@Sk J ahiruddin, 2020

Thermodynamic potentials and applications

2V for the each part . Therefore ,

b,.S = 2_ T

PdV 2V

= NkB

V

dV V

2V

+ NkB

dV

V

(2.27)

V

= 2NkB ln 2 > 0 The entropy of the system then may increase indefinitely by putting more and more membranes and removing them. However , the process is reversible. By putting back the membranes one would recover the initial state. According to Clausius t heorem , the change in entropy must be b,.S = 0 in a reversible process. This discrepancy is known as Gibb's paradox. The paradox would be resolved only by applying quant um statistical mechanics. The same problem will b e discussed again in the next section.

2.8

Radiation:

According to electromagnetic t heory, the pressure P of an isotropic radiation field is equal to 1/ 3 of the energy density:

P

=

u(T )/3 = U(T )/3V

(2.28)

11 oo : 01 : 2a According to electromagnetic theory, the pressure P of an isotropic radiation field is equal to 1/ 3 of the energy density:

(2.28)

P = u(T )/3 = U(T )/3V 31

[email protected]

@Sk J ahiruddin , 2020

Physicsguide

Thermodynamic potentials and applications

where V is the volume of t he cavity, U is the total energy. Using the thermodynamic principles, one could obtain Stefan 's law: u = aT4 , where a is a const ant. To do t hat we will start from t he radiation pressure and also t ake t hat the total energy of photon gas is proport ional t o volume. By the second law dU = TdS - PdV, thus

au

av

=T r

as

av

au

av

- P

r

av

r

(if)v

· (av 8S )T since u(T )V

au

= u (T)

=T

r

and

ar

v

ar

-P

v (2.29)

by Maxwell equation.

aP

aP

1 du 3dT

Since U

=

(2.30)

Hence

u(T ) =

T du -3 dT

-

l -u 3

where a is a const ant.

T du = 4 dT u

4

u = aT (2.31)

11 oo : 01

: 31 (2.28)

P = u(T)/3 = U(T )/3V 31

j [email protected]

@Sk J ahiruddin, 2020

Physicsguide

Thermodynamic potentials and applications

where V is the volume of t he cavity, U is the total energy. Using the thermodynamic principles, one could obtain Stefan's law: u = aT4 , where a is a const ant. To do t hat we will start from the radiation pressure and also take that t he total energy of photon gas is proport ional to volume. By the second law dU = TdS - P dV, thus

au = T as - P av r av r

au = T aP - P av r ar v (2.29)

(if) v

· (av 8S )T since u(T )V

au = u(T ) av r

by Maxwell equation.

aP ar

and

v

l du 3dT

Since U

=

(2.30)

Hence u(T)

=

Tdu -3 dT

-

l -u

3

Tdu =4 dT u

(2.31)

where a is a constant.

j [email protected]

32

Physicsguide

11 oo : 01

: 33 (2.28)

P = u(T)/3 = U(T )/3V 31

j [email protected]

@Sk J ahiruddin, 2020

Physicsguide

Thermodynamic potentials and applications

where V is the volume of t he cavity, U is the total energy. Using the thermodynamic principles, one could obtain Stefan's law: u = aT4 , where a is a const ant. To do t hat we will start from the radiation pressure and also take that t he total energy of photon gas is proport ional to volume. By the second law dU = TdS - P dV, thus

au = T as - P av r av r

au = T aP - P av r ar v (2.29)

(if) v

· (av 8S )T since u(T )V

au = u(T ) av r

by Maxwell equation.

aP ar

and

v

l du 3dT

Since U

=

(2.30)

Hence u(T)

=

Tdu -3 dT

-

l -u

3

Tdu =4 dT u

(2.31)

where a is a constant.

j [email protected]

32

Physicsguide

11 oo : oo : 01

Basic concepts of Statistical Mechanics Sk J ahiruddin

Assistant P rofessor· Sister Nibedita Govt. College, Kolkata Author was the topper of IIT Bombay lVI. Sc Physics 2009-2011 batch He ranked 007 in IIT JAlVI 2009 and 008 (JRF) in CSIR NET June 2011 He has been teaching CSIR NET aspirants since 2012

1

@Sk J ahiruddin, 2020

Basic concepts of Stat Mech

11 oo : oo : 03 1

Basic concepts of Stat Mech

@Sk J ahiruddin, 2020

Contents 1

Basic concepts of Statistical Mechanics 1.1

Specification of states: .

1.2

3





















4

Counting of Microstates: .





















5

1.3

Equal a priori probability:





















8

1.4

Statistical ensembles:



























9

1.5

Phase point density:



























11

1.6

Statistical average and mean values: .









12

1. 7

Condition of Equilibrium:









13

j [email protected]

2















Physicsguide

11 oo : oo : 06

j [email protected]

2

@Sk J ahiruddin , 2020

1

P hysicsguide

Basic concepts of Stat Mech

Basic concepts of Statistical Mechanics

Consider a physical system composed of N identical pa1·t icles confined in a volume V. For a macroscopic syst em, N is of the order of Avogadro number NA~ 6.022 x 1023 per mole. In this view, all the analysis in statistical mechanics are carried out in the so-called thermodynamic limit. It is defined as: both number of particles N and volume V of t he syst em t ends to infinity whereas the density of particles p = N / V remains finite.

N



oo,

V



oo ,

p

= N / V = finite.

(1.1)

In t his limit , the extensive properties of t he system become directly proportional to t he size of the system (N or V ), while the intensive properties become independent of the size of the system. The particle density becomes an important parameter for all physical properties of the system. In order to develop a microscopic t heory of a macroscopic syst em, it is necessary to specify t he state of micro-particles, atoms or molecules as a first step. Next is to construct the macro-st ate from t he micro-states of N number of parti-

11 oo :oo :oa In order to develop a microscopic theory of a macroscopic system, it is necessary to specify t he st ate of micro-part icles, atoms or molecules as a first step. Next is to construct the macro-state from t he micro-states of N number of [email protected]

P hysicsguide

3

Basic concepts of Stat Mech

@Sk J ahiruddin, 2020

cles when N is very large. Finally, one needs to extract t he macroscopic properties in terms of the micro-states of a macroscopic system. In this section, all t hese essent ial definitions will be given.

1.1

Specification of states:

The specification of t he state of a part icle depends on t he nature of the particle, i .e. whether the particle's dynamics is described by classical mechanics or by quantum mechanics. The dynamics of a classical system is determined by its Hamiltonian 1-l(p, q) where q and P are the generalized posit ion and momentum conj ugate variables. The motion of a particle is described by the canonical Hamilton 's equation of motion

and

.

Pi = -

81-l(p, q)

aqi ;

i = 1 2 ··· N

' '

' (1.2)

The state of a sine:le oarticle at anv time is t hen e:iven bv

11 oo : oo : 11 and

.

Pi = -

81i(p, q)

aqi ;

i = 1 2 ··· N

' '

' (1.2)

The state of a single particle at any time is t hen given by t he pair of conjugate variables (qi, Pi ) . The state of a system [email protected]

@Sk J ahiruddin, 2020

Physicsguide

4

Basic concepts of Stat Mech

composed of N particles is then completely and uniquely defined by 3N canonical coordjnates q1 , q2 , · · · , q3N and 3N canonical momentum P1 , P2, · · · , P3N. These 6N variables constitute a 6N -dimensional r space or phase space of the system and each point of t he phase space represents a state of t he system. Each single particle constitutes a 6 -dimensional µ -space. r -space is evident ly built up of N such 6 -dimensional µ -space of each particle. The locus of all t he points in r -space satisfying the condition 1i(p, q) = E , total energy of t he system , defines a surface called energy surface. Specification of states of quantum particles will be discussed later.

1.2

Counting of Microstates:

It is important to enumerate t he number of microstates for a given macrostate (N, V, E ) of a system. For a classical

11 oo : oo : 14 1.2

Counting of Microstates:

It is important to enumerate t he number of microstates for a given macrostate (N, V, E) of a system. For a classical particle, t he microstate is specified by a phase point . Consider an infinitesimal change in t he position and moment um coordinates in the phase space. T he phase point of this parj [email protected]

Physicsguide

5

@Sk J ahiruddin , 2020

Basic concepts of Stat Mech

t icle then under go a small displacement in the phase space and the microscopic state of t he system will be modified. However, if t he change in t he microstate is so small that it is not measurable by the most accurate experiment , then it can be assumed that t here is no change in t he macroscopic state of the system. T hus, t he state of a syst em is given by a small volume element Jn of t he r -phase space. If bqbp = h , t hen

By Heisenberg's uncertainty principle in quantum mechanics it can be shown that h is the P lanck's constant. Thus, t he number of states of a system of N particles of energy < E is given by

r=

1 h 3N

dn =

1 h3N

d3N qd3Np

(1.4)

11 oo : oo : 16 energy < Eis given by

r=

1 h3}l

dD.

=

1 h3N

d3N qd3Np

(1.4)

The number of microst at es is then proportional to the volume of the phase space. The counting of number of microstates however depends on the distinguishable and indistinguishable nature of t he [email protected]

Physicsguide

6

Basic concepts of Stat Mech

@Sk J ahiruddin, 2020

particles. If the constituent particles are distinguishable, t he two microst ates corresponding to the interchange of two particles of different energy a1~e distinct microstates. On the other hand, if t he constituent particles are indistinguishable, these two microstates are not distinct microstates. Thus, for a syst em of N indistinguishable, identical classical particles having different energy states, the number of microstates is then given by

r=

1 h3NN!

d3N qd3Np

(1.5)

Consider a system of N part icles of t otal energy E and corresponds to a macrostate (N , V, E ). If the particles are

11 oo : oo : 19 r ==

d3N qd3Np

h3N N!

(1.5)

Consider a system of N particles of total energy E and corresponds to a macrostate (N, V, E ). If the particles are distributed among t he different energy levels as, ni particles in t he energy level Ei, the following conditions has to be satisfied

(1.6)

and .

.

'l

'l

The total number of possible distributions of N such [email protected]

7

Physicsguide

Basic concepts of Sta t lVIech

@Sk J a hiruddin , 2020

cles is then given by

(1. 7) If the particles are distinguishable, then all t hese permutations would lead to distinct microstate whereas if the particles are indistinguishable, these permutations must be regarded as one and t he same microstate.

1.3

Equal a priori probability:

As it is already seen that for a given macrostate (N, V, E) , t here is a large number of possible microstates of t he system. In case of classical non-interacting system, t he total T7

1

1•

,

.,

i

1

I

1

1\ T

, •

1



11 oo : oo : 21 As it is already seen that for a given macrostate (N, V, E ), t here is a large number of possible microstates of t he system. In case of classical non-interacting system, t he total energy E can be distributed among the N particles in a large number of different ways and each of these different ways corresponds to a microstate. In case of a quantum system, the various different microstates are identified as the independent solutions \JI (r 1 , r 2 , · · · , r N) of the Schrodinger equation of the system, corresponding to an eigenvalue E . In any case, to a given macrostate of the syst em there exists a large number of microstates and it is assumed in statistical mechanics that at any time t the system is equally likely j [email protected]

8

@Sk J ahiruddin, 2020

Physicsguide

Basic concepts of Stat Mech

to be in any one of these microstates . This is generally referred as the postulate of equal a priori probability for all microstates of a given macrostate of the syst em. Equal a priori probability postulate is one of t he two fundamental postulates in Statistical Mechanics.

1.4

Statistical ensembles:

The microstate of a macroscopic system is specified by a point in 6N -dimensional phase space where N is of the 23 order of Avogadro number ( 10 ) . At any time t the system

11 oo : oo : 24 The microstate of a macroscopic system is specified by a point in 6N -dimensional phase space where N is of the order of Avogadro number ( 1023 ) . At any t ime t the system is equally likely to be in any one of the large number of microstates corresponding to a given macrostate ( N, V , E) . As t he dynamical system evolves with time, the system moves from one microst ate to another. After a sufficiently long time, the syst em passes through a large number of microstates. The behaviour of the system t hen can be obtained as averaged over those microstates through which the system passes. Thus, in a single instant of time, one could consider a

[email protected]

@Sk J ahiruddin, 2020

9

Physicsguide

Basic concepts of Stat Mech

collection of large number of replicas (mental copies) of the original system characterized by the same macrostate but could be in any of t he possible microstates to obtain the average behaviour of the system. This collection of large number of copies of the same system is called an ensemble. It is expected t hat t he ensemble averaged behaviour of a system would be identical with the time averaged behaviour of the given system. Since the values of (q, p) at any instant are different for

11 oo : oo : 21 averaged behaviour of a system would be identical with the time averaged behaviour of the given system. Since the values of (q, p) at any instant are different for a syst em of an ensemble, they are represented by different points in the phase space. Thus, the ensemble will be represented by a cloud of phase points in the phase space. As t ime passes, every member of the ensemble undergoes a continuous change in microstates and accordingly the system moves from one place to another on the phase space describing a phase trajectory. The density of the cloud of phase points at a particular place t hen may vary with time or at a given time the density may vary place to place. It must be emphasized here t hat the systems of an ensemble are independent systems, t hat is, there is no interaction between them and hence the

j [email protected]

10

@Sk J ahiruddin , 2020

Physicsguide

Basic concepts of Stat Mech

trajectories do not intersect.

1.5

Phase point density:

Since a phase point corresponds to a microstate of a system and its dynamics is described by the phase trajectory, the density of phase points p(p, q) is t hen determining the number of microstates per unit volume, that is, the probability

11 oo : oo : 29 Since a phase point corresponds to a microstate of a system and its dynamics is described by t he phase t rajectory, the density of phase points p(p, q) is t hen determining the number of microstates per unit volume, that is, the probability to find a state around a phase point (p, q). The phase point density p(p, q) is given by Number of states pp ( q) = - - - - - ' Volume element

(1.8)

At any t ime t , t he number of representative points in the volume element d3N qd3N p around the point (p, q) of t he phase space is then given by p(p, q)d3JVqd3Np

(1.9)

The density function p(p, q) then represents the manner in which t he members of t he ensemble are distributed over all possible microstate at different instant of t ime.

[email protected]

@Sk J ahiruddin, 2020

1.6

11

Physicsguide

Basic concepts of Stat Mech

Statistical average and mean values:

Consider any physical p1~operty of t he system X(p , q), which may be different in different microstates for a system. The macroscopic value of X must be t he average of it over all possible microstates corresponding to a given macrost ate. The ensemble average or t he statistical average (X ) of the

11 oo : oo : 32 Consider any physical property of t he system X (p , q), which may be different in different microstates for a system. The macroscopic value of X must be the average of it over all possible microstates corresponding to a given macrostate. The ensemble average or the statistical average (X) of the physical quantity X at a given instant of time, is defined as (1.10)

where p(p, q) is the density of phase points. Note that the integration is over the whole phase space. However , it is only t he populated region of phase space that really contribute. -

The mean value X of the physical quantity X depends how it evolves with time t In a sufficiently long time, the phase trajectory passes through all possible phase points. If t he duration of time is T , t he mean value is defined as

-

X

1

= T--+oo lim T

T

o

X (t)dt

(1.11)

The statistical average (X ) and the mean value X are [email protected]

@Sk J ahiruddin, 2020

12

Physicsguide

Basic concepts of Stat Mech

equivalent. This is known as ergodic hypothesis. Ergodic hypothesis is the second fund amental postulate of statistical mechanics.

11 oo : oo : 34 equiva ent . Ergodic hypothesis is the second fund amental postulate of statistical mechanics.

1. 7

Condition of Equilibrium:

By Liouville's t heorem, t he total t ime derivative of t he phase point density p(p, q), in absence of any source and sink in the phase space, is given by

dp dt where

=

8p 8t

+ {p , 1-l} =

(1.12)

O

3N

{p , 1-l} =

L

(1.13)

i =l

is known as Poisson bracket of t he density function p and t he Hamiltonian 1-l of the system. Thus, the cloud of phase points moves in the phase space like an incompressible fluid . T he ensemble is considered to be in statistical equilibrium if p(p, q) has no explicit dependence on t ime at all points in the phase space, i .e.,

(1.14) j [email protected]

13

@Sk J ahiruddin , 2020

Physicsguide

Basic concepts of Stat Mech

Under the condition of equilibrium is then 3N

{p, 1-l} = ),

8p 81-l

8p 81-l

=0

(1.15)

11 oo : oo : 37 -at =

(1.14)

0

13

j [email protected]

@Sk J ahiruddin , 2020

Physicsguide

Basic concepts of Stat Mech

Under the condition of equilibrium is then 3N

{p,1-l} =

L

=0

(1.15)

i =l

and it will be sat isfied only if p is independent of P and q. That is p(p, q) = constant (1.16) which means that the representative points are distributed uniformly over the phase space. The condition of statistical equilibrium then requires no explicit time dependence of t he phase point density p(p, q) as well as independent of the coordinates (p, q) .

j [email protected]

14

Physicsguide

11 oo : oo : oo

Random -walk and Probability distributions Sk J ahiruddin

Assistant P rofessor· Sister Nibedita Govt. College, Kolkata Author was the topper of IIT Bombay lVI.Sc Physics 2009-2011 batch He ranked 007 in IIT J AlVI 2009 and 008 (JRF) in CSIR NET J une 2011 He has been teaching CSIR NET aspirants since 2012

1

@Sk J ahiruddin, 2020

Random walk and P robabilities

11 oo : oo : 03 1

@Sk J ahiruddin, 2020

Random walk and P robabilities

Contents 1 Introduction

3

2 Random Walk and Probabilities

4

2.1

Random walk in one dimension















5

2.1.1

Mean value





























7

2.1.2

Dispersion .





























8

3 Probability distributions

9

3. 1

Gaussian Distribut ion

























9

3.2

Poisson's distribution .

























10

j [email protected]

2

Physicsguide

11 oo : oo : 06

j [email protected]

@Sk J ahiruddin , 2020

1

2

P hysicsguide

Random walk and P robabilities

Introduction

Thermodynamics and statistical mechanics deals with many particle system. In thermodynamics, for a system in equilibrium, we try to make some general statements about t he relationship between different macroscopic parameters of these systems. Thermodynamics requires very few number of postulates without any detailed assumption about t he microscopic properties of the system. Statistical Mechanics, for a system in equilibrium, starts with the microscopic properties of t he particles in t he system and the laws of classical and quantum mechanics Statistical mechanics requires input from the laws of statistics.Statistical Mechanics reproduces all t he results of t hermodynamics, plus a large number of general relations for calculating the macroscopic properties of system from the knowledge of its microscopic elements. For more details on probabilities please go to the mathematical physics part

11 oo :oo :oa mathematical physics part

[email protected]

@Sk J ahiruddin , 2020

2

Physicsguide

3

Random vvalk and Probabilities

Random Walk and Probabilities

The problem of random walk is one of the classic problems in physics having a wide range of application in various fields. A drunk start out from a lamppost located on a street. Each steps he takes is of equal size of length l. The man is however so drunk t hat t he direction of each step he takes , whet her it is left or right, is completely independent of t he preceding step. All we can say, t hat the probability of taking a step to right is p and t he probability of taking a step to left is q = l - p. Let's choose x-axis is t he road and t he lamppost is located at x = 0. The question we ask, after t he man takes N steps, what is the probability of t he man being located at x = ml? The problem can be reformulated in terms of N man rather than N steps. One can also study t he problem in more t han one dimension. The random walk problem illust rate some very elementary concepts of probability theory. It relates to many problem in physics. Think of two examples from Magnetism and Diffusion. .

.

1

11 oo : oo : 12 more t han one dimension. The random walk problem illust rate some very elementary concepts of probability t heory. It relates to many problem in physics. Think of two examples from Magnet ism and Diffusion. Magnetism: An atom has a spin [email protected]

@Sk J ahiruddin , 2020

i and magnetic moPhysicsguide

4

Random walk and P robabilities

ment mu. In accordance with quant um mechanics t he spin can be either '' up'' or '' down'' with respect to a given direct ion. If bot h t his probabilities are equal likely,what is the net total magnetic moment of N such atoms.? Diffusion of a molecule in a gas: A given molecule t ravels in three dimension a mean distance l between colli-

sions with other molecules. How far it is likely to have gone after N collisions?

2.1

Random walk in one dimension

We are going to calculate the probability of finding the walker at distance x = m l after N steps wit h

-N 6 . So U ( ) becomes (under t he infinitesimal change of )

U ( ) + 6U () = U ()

+

dU de/> 6

d = [1 + 6 de/> ] U ()

= (1 + X(b) )U() -+ As given in t he problerr

j [email protected]

24

@Sk J ahiruddin, 2020

P hysicsguide

Advanced topics in Mathematical Physics

By comparing last two lines we get

Now you need to apply your knowledge of Quantum mechanics to ident ify a relation between t he X operator and . d £ 2 operator. You know £ 2 = -in de/> rl



'I

I

rl \



'I

11 oo :01 : oa ow you nee to app y your now e ge o uantum mechanics to identify a relation between t he X operator and . d L 2 operator. You know L 2 = -in dcp . d -in dcp •

x t(81>) = - !_81>i1 n

So the X operator is Anti-Hermitian. Solution 2.8. According to the group property each element must have an inverse. a and b has self Inverse. And t he group also follows the closure rules - multiplication of any two elements belongs to the group. Hence a 0 b also belongs to t he group. Now the group has been given as abelian. So a 0 b = b 0 a. Now a 0 b 0 b 0 a= a(0 b 0 b) 0 a= I. Hence a 0 b has self inverse too. Then the smallest group contains a, b, ab, I . So t he order is FOUR.

[email protected]

25

@Sk J ahirudd in, 2020

3

Physicsguide

Advanced t opics in :Nlathem atical Physics

Tensors

Prob 3.1. Under a certain rotation of coordinate axes, a rank- 1 tensor va(a = 1, 2, 3) transforms according to the orthogonal transformations defined by the relations v{ =

fi(v1 + v2); v~ = fi(-v1 + v2); v~ = v3 . Under the same rotation, a rank-2 tensor T ab would transform such t hat: l

11 oo : 01

: 1o

. . n er a certain rotation o coor inate axes, a rank- 1 tensor va(a = l, 2, 3) transforms according to t he orthogonal transformations defined by the relations vf = fi(v1 + v2); v~ = J2(-v1 rotation, a rank-2 tensor

+ v2); T ab l

v~ = v3. Under the same would transform such that:

(GATE 2008] (a) T{,1

= T1,1T1,2

(c) T{,1

= T1,1 + 2T2,2 - T2,1

T1,2 + T2,1)

Prob 3 .2 . Let

T ij =

L

Eijk a k

and

f3k =

L

Eijk1ij,

where

IJ

k

is the Levi-Civita d ensity, defined to be zero if two of t he indices oincide and +1 and - 1 depending on whether Eijk

i,j, k is even or odd permutation of 1, 2, 3. The

/33

is equal

[GATE 2009]

to:

(a) 2a 3

(d)

-a3

Prob 3 .3 . Consider an ant i-symmetric tensor P ij wit h indices i and j running from 1 to 5. The number of independent component of t he tensor is: (a) 3 (b) 10 (c) 9 (d) 6 [GATE 2010]

[email protected]

@Sk J ahiruddin, 2020

26

P hysicsguide

Advanced topics in l\llathematical Physics

Prob 3 .4 . The no. of independent components of the symmetric tensor A ij with indices i, j = l , 2, 3 is [GATE

2012]

(a) 1

(b) 3

(c) 6

(d ) 9

11 oo : 01

: 13

Prob 3.4. The no. of independent components of the sym[GATE metric t ensor A ij with indices i, j = l , 2, 3 is 2012] (a) 1 (b) 3 (c) 6 (d) 9 Prob 3.5. In t he most general case, which one of the following quantities is NOT a second order t ensor? [GATE 2013] (a) Stress (b) Strain (c) Moment of Inertia (d) Pressure Prob 3. 6. Let ¼ be the i th component of a vector field V , which has zero divergence. If

ai = aa , the expression for X·

.7

(c) 8J¼

[GATE 2016] (d) -8}¼

Prob 3. 7. The rank 2 tensor X i Xj where X i are the Cartesian coordinates of t he position vector in three dimensions, has 6 indep endent elements. Under rotation, these 6 elements decompose into irreducible sets (that is, the elements of each set transform only into linear combinations of ele[NET June 2015] ments in that set) containing (a) 4 and 2 elements (b) 5 and 1 elements (c) 3,2 and 1 elements (d) 4,1 and 1 elements

j [email protected]

@Sk J ahiruddin , 2020

27

Physicsguide

Advanced topics in :Nl athematical P hysics

Prob 3. 8. The lengt h element ds of an arc is given by

11 oo : 01 @Sk J ahiruddin , 2020

: 16

Advanced topics in Mathematical Physics

Prob 3. 8. The lengt h element ds of an arc is given by

The metric t ensor

(a)

2

v'3

v'3

1

[GATE 2014]

is

(b)

2 v'3 2 v'3 1 2

(c)

2

1

3

1

(d)

9 ij

2

2 Prob 3. 9. The scale factors corresponding to t he covariant

metric tensor 2018] 2

2

9 ij

2

(a) 1, r ,r sin 0

[GATE

in spherical coordinat es are 2

2

(d) l ,r,rsin0

(b) 1, r ,sin 0 (c) 1, 1, 1

Prob 3.10. A fourt h rank Cartesian tensor t he following identities

~jkl

satisfies

= T jikl (ii) ~ jkl = ~ jlk (iii) ~ jkl = T klij

(i)

~ jkl

Assuming a space of t hree dimensions (i.e i, j, k = l , 2, 3) what is the number of independent components of T ijkl [TIFR 2018] Prob 3.11. An array Thas elements [email protected]

@Sk J ahiruddin, 2020

1, 2, 3, 4. It is given that

28

~ jkl

where i , j , k, l

=

Physicsguide

Advanced topics in Mathematical P hysics

00 : 01 : 18

@Sk J ahiruddin , 2020

Advanced topics in :Nlathematical P hysics

1,2 , 3, 4. It is given that Ti jkl = T jikl = Ti jlk = - T klij

for all values of i, j, k, l. The number of independent components in this array is [TIFR 2019] (a) 256 (b) 1 (c) 45 (d) 55

j [email protected]

@Sk J ahiruddin , 2020

29

Physicsguide

Advanced topics in Mathematical P hysics

11 oo : 01 [email protected]

29

@Sk J ahiruddin , 2020

3.1

: 21

P hysicsguide

Advanced topics in l\llathematical P hysics

Ans keys

3.1. d

3.5. b

3.2. a

3.6. d

3.9. d

3.10. 21 3.3. b

3.7. b

3.4.

3.8. b

3.2

C

3.11.

C

Solutions

Solution 3.1. First we write the transformation equation

for vector in matrix form v'1 v'2 v'3

1

1

J2

J2

1

../2 0

0

J2

0

0

1

Transformation equation for t ensor is Tlt

= akiazj'Ti,j

1

Ji

For this problem our a matrix is a =

~

1

Ji

~ 0

0 0 So t he Tensor t ransformation equation is

[email protected]

30

0 •

1

Physicsguide

11 oo : 01

j [email protected]

Physicsguide

30

@Sk J ahiruddin, 2020

T{ 1

: 24

Advanced topics in :Nlathematical Physics

=

a11a11T11

+ a11a12 T 12 + a11a13T13

+

a12a11T21

+

a12a12T22

+

a12a13T23

+

a13a11T31

+

a13a12T32

+

a13a13T33

Now put the elements of the a matrix in the equation

,_ llT + llT 1 T11 11 12 + /n 2 J2J2 J2J2 VL1

+

1

J2 J2T21

+ 0

X

1

+

O T31 + 0

1

J2 J2T22

1 X v'2T32

X

OT

13

1

+

v'2 x

+ 0

X

O T23

O T 33

So we get , T 11

=

1 1 1 1 T11 + T12 + T21 + T22 2 2 2 2

Solution 3.2. /33 = L i,j Eij3~j Now from the definition of Levi-Civita Tensor Eij3 will give you ZERO when any of i,j will b e THREE. Now when i,j will both be ONE or TWO then also the Tensor give you ZERO. So the only non ZERO values are

11 oo : 01

: 26

i , j will be THREE. Now when i, j will both be ONE or

TWO then also the Tensor give you ZERO. So the only non ZERO values are

31

[email protected]

@Sk J ahiruddin , 2020

Physicsguide

Advanced topics in Mathematical P hysics

Now wit h t he same logic ~ j

=

L

Eijk a k k

Solution 3.3. Second Order Tensor with dimension n is a

n x n matrix. You can easily verify t hat the no of indepenn n

.

.

.

+1

. n n - 1

matrix is anti-symmet ric and n = 5. Hence t he ans is 10. Solution 3.4. As t he general formula is given in the previous problem, here the matrix is symmetric and n = 3. Hence t he ans is 6. Solution 3.5. The direct product of two vectors and division of two vectors is a Tensor. (Refer to Quotient rule) . We all know Moment of Inertia is a tensor (L = I w). St ress and Force . .. pressure are A . So as d1v1s1on of two vectors they are r ea tensors. Strain is division of two scalars (elongation/length).

So Strain can't be a t ensor.

11 oo : 01

: 29

all know Moment of Inertia is a tensor ( L = I w). Stress and Force . .. pressure are A . So as d1v1s1on of two vectors they are rea tensors. Strain is division of two scalars (elongation/length). So Strain can't be a tensor.

Solution 3.6. Levi Civita symbol index can be changed in cyclic permutation . Eijk = Ejki = Ekij . Similarly Ezmk = [email protected]

32

@Sk J ahiruddin , 2020

Emkl

=

Physicsguide

Advanced topics in Mathematical P hysics

Eklm

Now we write t he given identity as

We know t he relation between Levi-Civita and Kronekar Delta function

Hence Ekij Eklm Oj Ol V m

= (Oil Ojm - OimOjz) Oj Ol V m = 8 rri 8 zV m - 8z8z V m = 8 z8 m V m - of V m = - 8[Vm ➔

as

1.v =

0

a

- - Vm =O OXm

Solution 3.7. Any three dimensional vector in cartesian coordinate has t hree independent elements. As we have already seen that the direct product of two vectors is a tensor, so the most generalized rank 2 tensor XiXj where Xi are the .

.

.

.

11 oo : 01

: 32

Solution 3. 7. Any t hree dimensional vector in cartesian coordinate has t hree independent elements. As we have already seen that t he direct product of two vectors is a tensor, so t he most generalized rank 2 tensor X i Xj where Xi are the Cartesian coordinates of the position vector in t hree dimensions should have nine independent elements. The given case in the problem is a special case as t he t ensor has 6 independent element. The main catch is to identify from t he [email protected]

P hysicsguide

33

@Sk J ahiruddin, 2020

Advanced topics in 1\ll athematical Physics

ment ion of 6 independent elements is t he tensor as SYMMETRIC tensor. Now from our previous knowledge we get that tensor product of two spaces of dimension 3 can be seen as direct sum of two spaces of dimension 1, 3, 5 wit h j values 0, 1, 2 respectively. Here again we need to understand t hat the j value of 1 which leads to dimension 3 is ANTI-SYMMETRIC. The given tensor is symmetric. So we omit the ant i-symmetric combination and conclude t he direct sum to contain dimension 1 and 5 only ( j values O and 2 ) . Solution 3.8. Metric Tensor is a symmetric tensor defined as a property of length element of a space. 2

ds = du dv

E F FE

du dv

To explain more lets your length element is c1 dx I



I

2

+ c2dx~ + c3dx1dx2

., , ,

11 oo : 01

: 34

FE

To explain more lets your length element is 2 c1 dx + c2dx~

+ c3dx1 dx2

Your metric tensor will be

In this problem 1 2 2 2 2 ds = 2(dx ) + (dx )

[email protected]

1 2 + -/3dx dx P hysicsguide

34

@Sk J ahiruddin, 2020

Advanced topics in Mathematical Physics

So = 2, C2 =

C1

1,

C3

=

,/3

Hence the ans will be 2 v'3 2 v'3 1 2

You can easily generalize t he formulation in three and four dimensions. Solution 3.9. Obvious from the definit ion of scale factors you have studied. Solution 3.10. (i) and 2nd indices

1ijkl

(ii) T ijkl = T tjlk (iii) T ijkl = T klij of indices

~►

1'T

I



I



=

T jikl ===;:~>

Symmetric under l st

Symmetric under 3rd and 4th indices ~> Symmetric under l st and 2nd pair 1

• 1



1

11 oo : 01

: 37

and 2nd indices (ii) ~Jkl = ~ j lk ~► Symmetric under 3rd and 4th indices ~► Symmetric under 1st and 2nd pair (iii) T i.ikl = T ktij of indices Now we can partition ~ jkl as T (i j) (kl) where we now t hink of T (ij )(kl ) as a m x m symmetric matrix with ~m(m + 1) independent components. We can also t hink the pairs (ij) and ( kl) as individual matrices which are symmetric wit h ½n(n + l ) independent components.

j [email protected]

35

@Sk J ahiruddin, 2020

Physicsguide

Advanced t opics in Mathematical Physics

So we can write. 1

m (m + 1) =

2

1

1

n (n + 1)

2 2

(As n = 3) 1 1 - . - ♦ 2 2 1

1

-

=

2 1 2

-

2

X

6

X

X

3 ♦ 4

1 - ·3·4 + 1 2

1 12 - X 12 + 1 2 42 7 = - = 21 2

Solution 3.11. All we need to do some count ing. For a general rank-4 t ensor in four dimensions, since each index can take any of four possible values, the number of indep endent components is simply

11 oo : 01

: 39

Solution 3.11. All we need to do some counting. For a general rank-4 tensor in four dimensions, since each index can take any of four possible values, the number of independent components is simply 4

independent components = 4 = 256 Taking into account the first symmetry relation, the first part ~.ikl = T jikl indicates t hat the tensor is antisymmetric when the last pair of indices is switched. Thinking of t he last pair of indices as specifying a 4 x 4 symmetric matrix, t his means instead of having 42 = 16independent elements, we actually only have 4( 4 + 1) / 2 = 10 independent choices for the last index pair (t his is the number of elements in an symmetric 4 x 4 matrix). Similarly, the second part of the [email protected]

@Sk J ahiruddin, 2020

36

P hysicsguide

Advanced topics in :Nlathematical P hysics

first relation 1iJkl = T i.i lk indicates that the tensor is symmetric when the last pair of indices is switched. Hence by similar argument we get 10 independent choices. So with the three relations we have 10 x 10 = 100 independent choices and e can take it as 10 x 10 matrix. We are now able to handle the last condit ion 1iJkl = - T klij

By now, it should be obvious that t his statement indicates that the tensor is ant isymmetric when the first index pair is interchanged with the second index pair. The counting of independent components is t hen the same as t hat for 10 x 10 antisymmetric matrix. So we get t he final ans

11

00:01 :41

cates that the tensor is antisymmetric when the first index pair is interchanged with the second index pair. The counting of independent components is t hen the same as t hat for 10 x 10 antisymmetric matrix. So we get t he final ans 10 x (10 - 1)/2 = 45 independent choices.

[email protected]

@Sk J ahiruddin , 2020

4

37

P hysicsguide

Advanced topics in l\llathematical P hysics

Green's functions

8(x), where k is a constant. Which of t he following state[JEST 2013] ments is true?

(a) Both G(x) and G'(x) are continuous at x = 0 (b) G(x) is continuous at x = 0, but G' (x) is not (c) G' (x) is discontinuous at x = 0 (d) The continuity properties of G (x) and G' (x) at x = 0 depend on the value of k

11 oo : 01

: 44

(b) G(x) is continuous at x = 0, but G' (x) is not (c) G' (x) is discont inuous at x = 0 (d) The cont inuity propert ies of G (x) and G' (x) at x depend on the value of k

=

0

o(x - x 0 ) with the boundary conditions g(- L , x 0 ) = 0 = g(L, x 0 ) is [NET June 2017]

(a)

iL(xo - L) (x ½,(xo

(b)

(c)

+ L ),

+ L )(x -

L ),

- L < x < x0 x0 < x < L

2i(xo+L)(x+L) ,

-L ~x:Sx 0

ft (xo - L ) (x - L) , 2i (L - x0 )(x + L), 2i (xo + L ) (L - x ),

x0 < x < L - L

< x < x0

x0 < x < L

(d) 2i(x- L )(x+ L ), - L < x < L - - - - - - - j [email protected]

38

@Sk J ahiruddin , 2020

Physicsguide

Advanced t opics in :Nl athematical Physics

Prob 4.3. The Green 's function G (x, x') for t he equation

a2 y

X

with t he boundary values y(O) = y (; ) = 0, is June 2018]

[NET

11 oo : 01

: 47

dx with t he boundary values y(O) = y (; ) June 2018]

(a) G (x, x') =

(b)G(x ,x') =

(x' - ;) , (x - ; ) x',

X

0
E 1 , if the particle crosses t hose points then from above eqution we will get r 2 = -ve. From the picture t here is only such point , the particle has only one turning point. From point r = a the potential energy start to decrease,

11 oo : oo : 55 From the picture there is only such point , the particle has only one turning point. From point r == a the potential energy start to decrease, so K.E start to increase, hence the velocity also start to increase till t he t he particle reaches point c, where the potential in minimum. At that point velocity and K.E become maximum. •

jahir@physicsguide. in

20

physicsguide CSIR NET, GATE

@Sk J ahiruddin , 2020

Basic Newtonian Niechanics: Part-2

At points r = a and b the potential energies are equal, so are velocities and K.E. Sol 1.12.

x(t) = y(t) x(t) = iJ = -x(t) and,

y(t) = -x(t) y(t) = -x == -y(t) The equation of motion,

F

=

m(xx + yf))

=

- (xx + yf))

If we take t he potential as V(x, y) = ~ (x 2 + y 2 ) then, 1 . ,.,

,., .

11 oo :oo : s1 F

=

m(xx + yy)

=

- (xx + yy) 2

If we take the potential as V(x,y) = ~(x +y

2

)

then ,

Sol 1.13. The equation of the surface on which the bike is moving 1s, •



j [email protected]

21

physicsguide CSIR NET I GATE

Basic Newtonian 1\/[echanics: Part-2

@Sk J ahiruddin , 2020

A unit normal vector to the surface at (x, y, z)is,

" V cp(x, y,z) n = ---cp(x, y,z) 2 2 v' [z - a(x + y ) ] v' [z - a(x 2 + y 2 ) 2a(xx + yy) + z 2 2 2 J4a (x + y ) + 1

TcosB

" (\

sin8

p

'-------+ . . . . . . . . . . mg

Zo

11 oo : 01 : oo mg

Switching to a cynlindrical coordinate syst em , x 2 + y 2

=

p2 .

At z = zo, p

2

[email protected]

=

Zo a

22

physicsguide CSIR NET ) GATE

Basic Newtonian Nlechanics: Part-2

@Sk J ahiruddin ) 2020

if n makes an angle 0 with z axis , then

n can be written

as, A

n

2app

2

✓4a p + 1

✓4a p + 1

= ----::========== + ----::========== 2 2 2 2 = sin 0p + cos 0z

So, tan0

= 2ap

The Normal Force at z 0 acts in the direction of the normal vector to the surface at that point. The z direction of t he Normal Force N z

=

.....

N cos 0, bal-

11 oo : 01

: 02

The Normal Force at z 0 acts in the direction of the normal vector to the surface at that point. .....

The z direction of t he Normal Force N z = N cos 0, balances the downward gravitational force -mgz , so t hat the bike doesn 't slip down. And the radial component of the ..... Normal Force N r = N sin 0, produces the required centripetal force for the bike to circle around. So, Ncos0 Nsin0

= mg mv

2

= -p

Now we have got, 2

mv 2ap = - pmg v

=

2zog

23

[email protected]

physicsguide CSIR NET , GATE

@Sk J a hiruddin, 2020

B asic Newtonia n Nlechanics: P ar t-2

Sol 1.14.

1

E = -mv 2

2

1

+ -mw

2

2

Then 1

- m4 2

2

1

+ - mw

2?

1

?

1

22

2- = -m2~ + - mw 6 2 2 2 2 2 16 + 4w = 4 + 36w 2 3 w = 8

11 oo : 01 : os 16 + 4w = 4 + 36w 2 3 w = 8 2

2

Equation for S.H.O

x

= a cos (wt - cp)

v = -aw sin(wt - cp)

At any point

a= j [email protected]

@Sk J ahiruddin, 2020

2

24

140 3 physicsguide CSIR NET, GATE

Basic Newtonian lVIechanics: Part-2

Momentum

Prob 2.1. A jet of gas consists of molecules of mass m, speed v and number density n all moving co-linearly. This jet hits a wall at an angle 0 to t he normal. The pressure exerted on the wall by t he jet assuming elastic collision will be [JEST 2012] (a) p = 2mnv 2 cos20 (b) p = 2mnv 2 cos 0 ( c) v

= " fim nvcos20

(d )v=mnv 2

11 oo : 01

: 01

jet hits a wall at an angle 0 to t he normal. The pressure exerted on the wall by t he jet assuming elastic collision will be [JEST 2012] (a) p = 2mnv 2 cos 20 (b) p = 2mnv 2 cos 0

(c) P =

3 mnvcos 20 2

(d) p = mnv 2

Prob 2.2. A cylindrical rod of length L has a mass density

distribut ion given by p(x) = Po( l +~),where x is measured from one end of t he rod and p0 , is a constant of appropriate dimensions. The center of mass of the rod is [JAM 2016]

5 (a) L 9

4 (b) L 9

(c)

!9 L

(d)

!2L

Prob 2.3. A block of mass M is free to slide on a frictionless horizontal floor. The block has a cylindrical cavity of

radius R in the middle of it. The center of mass (CM) of the block lies on t he dashed line passing t hrough t he center of t he cavity (see figure) . Initially t he CM of t he block is at a horizontal distance X 1 from t he origin, Now a point part icle of mass m is released from point A into t he cavity. There is negligible friction bet~reen t he particle and t he cavity [email protected]

@Sk J ahiruddin, 2020

25

physicsguide CSIR NET, GATE

Basic Newtonian :tvlechanics: Part-2

face. When t he particle reaches point B, t he CM of the block is at a distance X 2 from t he figure. Find (X 2 - X 1 ) [JAM 2009]

11 oo : 01

: 11 [JAM 2009]

A

B

I I

I I

I I

X1

(Cent er of mass of the t otal system should be conserved as there is no external force) Prob 2.4. A homogeneous semi-circular plate of radius R = 3m is shown in t he figure. The distance of t he COM of t he

plate (in meter) from the point O is?

[email protected]

26

[JAM 2010]

physicsguide CSIR NET, GATE

Basic Newtonian l\/Iechanics: Part-2

@Sk J ahiruddin , 2020

O

3m

11 oo : 01

: 13

O

3m

(If you are getting 2R/3 then you are totally wrong on t he definition of cent er of mass) Prob 2.5. A chain of mass M and length L is suspended vert ically with its lower and touching a weighting scale. The chain is released and falls freely onto the scale. Neglecting t he size of the individual links, what is the reading of t he scale when a length x of the chain has fallen? [JEST 2015]

(a) Mix

(b) 2~gx

(c) 3~gx

(d) 4~gx

(Falling chain prob is very interesting dude!) Prob 2.6. Sand falls on a conveyor belt at the rate of 1.5 kg/s. If the belt is moving with a constant speed of 7m/ s, t he power needed to keep t he conveyor belt running is: [JAM 2017] (Specify your answer in Watts to two digits after t he decimal j [email protected]

@Sk J ahiruddin, 2020

27

physicsguide CSIR NET 1 GATE

Basic Newtonian l\llechanics: Part-2

point .) Prob 2. 7. A water cannon starts shooting a jet of water

11 oo : 01

: 16

@Sk J ahiruddin , 2020

Basic Newtonian l\llechanics: Part-2

point .) Pro b 2. 7. A wat er cannon start s shooting a jet of water horizont ally, at t = 0, into a heavy trolley of mass M placed on a horizontal ground . The nozzle diameter of the water cannon is d, t he density of water is p , and the speed of water coming out of the nozzle is u . Find t he sp eed of t he trolley as a function of t ime. Assume t hat all t he water from the jet is collected in t he t rolley. Neglect all frictional losses. [JAM 2012]

I

\VATERCANNON

I>---..,

M

-------- ----- - -- ---

-- - - - - -0



0

Truly speaking t he problem is incomplet e. It is not given whether t he cannon is also moving wit h t he t rolley. If you assume t hat the cannon is also moving with t he t rolley and maintain a constant difference wit h t he t rolley then you get t his ans. Prob 2.8 . A rain drop falling vertically under gravity [email protected]

@Sk J ahiruddin , 2020

28

physicsguide CSIR NET, GATE

Basic Newtonian l\llechanics: Part-2

ers moisture from t he atmosphere at a rate given by

d;: =

11 oo :01

:1

a Basic Newtonian :tvlechanics: Part-2

@Sk J ahiruddin, 2020

ers moisture from the atmosphere at a rate given by dd"; = 2 kt , where m is the instantaneous mass, t is time, and k is a constant. The equation of motion of the rain drop is m~ + vd:; == mg. If the drop starts falling at t == 0, with zero init ial velocity and initial mass m 0 (given m 0 == 2gm, k == l2gm/s 3 and g == lOOOcm/s 2), the velocity (v) of the drop after one second is [JAM 2011] (a) 250cm/ s (b) 500cm/ s (c) 750cm/ s (d) 1000cm/ s Prob 2.9. A particle with time-varying mass m(t) == m 0 (1;), where m 0 and T are positive constants, moves along the x-axis under the action of a constant positive force F for 0 ::; t < T. If t he particle is at rest at time t == 0, then at time t == t, its velocity v will be [TIFR 2013] TF t Ft t (a)- -log 1 - (b)-- log-

mo

Ft

t

(c)- 1 - m0

mo

T

T

-i

(e) r F mo

T

t

l -T

Prob 2.10. An aircraft, which weighs 12000 kg when unloaded, is on a relief mission, carrying 4000 food packets weighing 1 kg each. The plane each is gliding horizontally with its engines off at a uniform speed of 540 km/h when t he first food packet is dropped. Assume t hat the horizon-

j [email protected]

@Sk J ahiruddin , 2020

29

physicsguide CSIR NET, GATE

Basic Newtonian 1\/[echanics: Part-2

11 oo : 01 [email protected]

: 21

29

@Sk J ahiruddin, 2020

physicsguide CSIR NET, GATE

Basic Newtonian l\/Iechanics: Part-2

tal air drag can be neglected and t he aircraft keeps moving horizontally. If one food packet is dropped every second, t hen the distance between the last two packet drops will be [TIFR 2016] (a) l .5 km (b )200 m (c) 150 m (d)lOO m

[email protected]

30

physicsguide CSIR NET, GATE

11 oo : 01

: 23

j [email protected]

30

@Sk J ahiruddin, 2020

2.1

Basic Newtonian l\llechanics: Part-2

Ans Keys

2.1. a

2.5.

2.2. a

2.6. 73.5

2.3. 2.4.

physicsguide CSIR NET 1 GATE

-2mR

m+M 4 1r

2. 7. u ln

C

V

M+kt M

where k 2.8. b 2.9.

C

2.10.

C

=

1rd2

4 up

11 oo : 01

[email protected]

@Sk J ahiruddin, 2020

2.2

: 26

31

physicsguide CSIR NET, GATE

Basic Newtonian Niechanics: Part-2

Hints

2.1. what is the change in momentum Px? .....

2.2. definit ion of c. o. m for a continuous rigid body is R

=

ITrdm ITdm . 2.3. 2.4.

2.5. 2.6. what is the required energy ~ W to keep the belt mov-

ing at t he same speed for t ime ~ t? 2. 7. use conservation of momentum. 2.8. you are already given all information, so just solve the

differential equation. 2.9. after doing t he last two problem, this one should be

easy.

11 oo :01 : 2a differential equation.

2.9. after doing the last two problem , this one should be easy.

2.10. Ans key is right .... !!

32

j [email protected]

@Sk J ahiruddin , 2020

2.3

physicsguide CSIR NET I GATE

Basic Newtonian 1\/[echanics: Part-2

Solutions

Sol 2.1. For a single particle t he change in x component of moment um, ~P.'[; = 2mv cos 0 •

~

Foi~ce Fx apllied on the surface

F _ ~Px _ 2mvcos 0 X -

~t -

~t

11 oo : 01

: 31

Fo1·ce Fx apllied on t he surface

F _ ~ P.x _ 2mvcos 0 ~t -

X -

~t

The number of particle striking in t ime ~ t on t he area

A is N = nv~ tA cos 0. [email protected]

physicsguide CSIR NET ) GATE

33

@Sk J ahiruddin ) 2020

Basic Newtonian Nlechanics: Part-2

Hence t he pressure

P = N ·

i

2

2

= 2nmv cos 0

Sol 2 .2 .

f xdm X =-T_ _ f dm f0L p(x )xdx 7

foL p(x) dx Po foL (1+ f ) xdx -

L

Po fo (1 + I,) dx 5L 9

Sol 2 .3 . Center of m ass of t he total system should be conserved as there is no external force. The COM of t he combined syst em init ially is

MX1

+ m (X1 - R) '/\If+ rn

11 oo : 01

: 33

Sol 2.3. Center of mass of the total system should be conserved as there is no external force.

The COM of t he combined system initially is MX1

+ m(X 1 - R) M+m

After the ball reaches t he other side the COM is MX2 + m(X2 + R)

m+M Equating t he initial and final coordinate of COl\!l

[email protected]

physicsguide CSIR NET, GATE

34

@Sk J ahiruddin, 2020

Basic Newtonian Nlechanics: Par t-2

Hence we get

Sol 2.4 .

COM

=

R = L i miri mi

Center of mass is a vector. keep t his t hing in mind while calculating t he COM. You need to find x coordinate and y coordinate of the COM separately in some types of problems particularly where t he are no spherical / cylindrical symmetry. x coordinate of t he COM is at ZERO (taking t he center of the semicircular plate as origin ) . Now y coordinate is

" \/

f ydm

I. CM=

Now, substituting the values y

f dm =

r sin 0 and dm

=

e7rdrd0 ,

11 oo : 01

: 36

of the semicircular plate as origin ) . Now y coordinate is y;

c1v1

J ydm J dm

=

Now, substituting the values y = r sin 0 and dm = ardrd0, we get YcM

=

f R f 1r ar 2 sin 0d0dr 0 0 R 0 01r ar d0dr

f J

R

_

fr0

2

(2)r dr

f oR ( 1r )rdr

_

R

(J0 sin 0d0) r 2 dr R f0 (J0 d0) rdr

f R 0

=

1r

fr0 r dr 1r JR rdr 0

2

1r

2

2

_

r

3

3

R 0 _

2

1r [ r 2

] :

3 4R -31r R

2

4R 31r

4

Put R = 3 and get ans== -

1f

35

j [email protected]

@Sk J ahiruddin , 2020

physicsguide CSIR NET, GATE

Basic Newtonian lVIechanics: Part-2

Sol 2.5. The reading of t he weight machine will come from two parts. The first is obviously the weight of the chain accumulated on the machine. As the chain has fallen t he distance x on the weight machine, t he weight due to t his part is

W1 = Mgx/ L The second component contributing to t he weight is due to t he momentum delivered by the chain to the weight machine. As the chain is falling freely the velocity of t he chain parts at t he instant of hitting t he scale is simply found from 2

v = 2gx

So the momentum dp delivered in t ime dt

11 oo : 01

: 39

at t he instant of hitting t he scale is simply found from v

2

=

2gx

So t he momentum dp delivered in t ime dt dp

=

d(mv)

=

vdm

where t he v has taken to be constant as the momentum is delivered in very small time dt. The chain is moving in velocity v, the length of the chain falls down to t he machine in t ime dt is dx = v dt . The mass per unit length is M/ L. So t he mass dm which falls on the weight machine in time dt is nothing but

M

M

L

L

dm = - dx = - vdt [email protected]

36

@Sk J ahiruddin , 2020

physicsguide CSIR NET, GATE

Basic Newtonian :tvlechanics: Part-2

So t he momentum dp delivered in t ime dt M M 2 dp = d(mv) = vdm = v L vdt = L v dt

So the force which will be exerted by t he chain to the weight machine is the rate of change of this momentum delivered dp = lim fl.p = M v 2 dt ~t➔O i:l.t L

As v

2

=

2gx, so t he force due to momentum delivered be-

comes W _ dp 2 - dt

M

X

L v2=2MgL

The total force on t he scale during the fall of t he chain

11 oo : 01 As v 2

= 2gx,

: 42

so t he force due to momentum d elivered b e-

comes W - dp 2 -

dt

The total force on t he scale during the fall of the chain is t her e£ore X

X

X

L

L

L

Mg -+ 2Mg - = 3Mg -

Sol 2.6. Say, l:!,,.m m ass has been added on t h e already existing sand of mass m, in t ime 1:!,,.t. According to t he question we have to keep the b elt moving at the same speed. So t he reqired extra energy in t ime 1:!,,.t I:!,. W

1

= - (m + l:!,,.m) v 2

=

2

1

- - mv

2

2

1 - l:!,,.mv 2 2

[email protected]

37

physicsguide CSIR NET, GATE

Basic Newtonian l\/Iechanics: Part-2

@Sk J ahiruddin , 2020

Hence the required p ower ,

P = lim

I:!,.

w

b.t➔O I:!,.

t

. 1 l:!,,.m 2 = 11m ---v b.t➔O 2 i:!,,.t l dm 2 = --v 2 dt

=

!2 · 1.5 · 7

=

73.5

2

kg • m

2



s-

3

Watt

Sol 2. 7. Let us fo cus on t he short time interval t and t + 1:!,,.t.

11 oo : 01

: 44

-

2 dt

= !2 · 1.5 · 72 kg • m 2 • s- 3 = 73.5 W att Sol 2. 7. Let us focus on t he short t ime interval t and t + ~ t . Let water of mass ~ m added to t he t rolley in t ime ~ t. At any t ime t he system consist of the mass of the t rolley M (t) and ~ m. The initial momentum is

P (t) = M(t)v + ~mu. T he final momentum is

P (t + ~ t) = (M(t) + ~ m) · (v + ~v) . The change in momentum is (ingnoring the negligible products like ~ m~v) ~p

= P (t + ~ t) - P (t) = M(t) ~v + (v - u )~ m.

j [email protected]

physicsguide CSIR NET 1 GATE

38

@Sk J ahiruddin, 2020

Basic Newtonian l\llechanics: Part-2

The rate of change of momentum is approximately ~p ~t

In t he limit

~m

~v

= M(t) ~ t + (v - u) ~ t

~t ➔

0, we have

dP dv dt = M (t) dt

+ (V

-

dm u) dt

11 oo : 01 .6.t ➔

In t he limit

dP dt

: 47

0, we have dv M (t ) dt

=

+ (V

-

dm u) dt

In this case since there are no external force cZJ: = 0. dm dM 1rd2 = = up = k (say ). dt dt 4

So t

M(t)

=

dM' M

kdt' 0

M (t)

[email protected]

= kt + M

39

physicsguide CSIR NET, GATE

Basic Newtonian Niechanics: Part-2

@Sk J ahiruddin , 2020

Then, (u - V)k

k

dt

=

dv (kt + M) dt dv

kt + M (u - v) M + kt (u - v) ln - - - = - ln M u

11 oo : 01

: 49

(u - V)k

= (kt + M) dt dv (u - v)

dt

k

kt + M

In M

+ kt = -

In (u - v)

M

u

M+kt M

u

(u -v) M

V

= u 1- M + kt

A frequent error!! If u start with F = (d/dt)(mv ) you would get m(dv/dt) + v(dm/ dt). Which is simply wrong! Because in that case you are not considering the moment um change of the water. Sol 2.8. The mass of t he water droplet at any time t is,

m

dm = kt2 dt t

kt'2 dt'

dm' = mo

0

1 3 m = kt + mo 3 From the eq. of motion of t he rain drop, j [email protected]

@Sk J ahiruddin , 2020

40

physicsguide CSIR NET I GATE

Basic Newtonian 1\/[echanics: Part-2

dv dm md +v = mg t dt dv 2 m dt + v kt = mg

11 oo : 01 : s2 dv dm m dt + v dt = mg dv 2 m-+vkt = mg dt kt 2 dv - + v - =g dt m kt 2 dv - + v - -3 - - = g dt 1kt + m 0 We can solve t his linear first order ODE by multiplying, exp

kt' 2 - - - -dt' }kt'3 + m 0

d (1kt'3 + m )

= exp

o dt'

3

}kt'3 + m 0

= exp In

1 3

kt

3

+ mo

on both side, thus turning it into an exact differential. Hence,

[email protected]

@Sk J a hiruddin ) 2020

41

physicsguide CSIR NET ) GATE

Basic Newtonian Nlechanics: Part-2

11 oo : 01 : s4 @Sk J ahiruddin, 2020

Basic Newtonian l\llechanics: Par t-2

dv kt 2 . -+v---- · dt ½kt3 + mo

or,

1 3 kt + mo g 3 d 1 3

or,

d V

dt v

Integrating

on

kt + m0 3

both

sides,

v,t

t

d

V

0

0 ,0

1 3

kt + m 0 gdt 3

4

t2kt + mat

·g

or,

v=

or,

+ mat v = -3 - - - ·g ½kt + mo

tkt3 + mo 112kt4

Putting the values k t = l s, we get,

=

12gm/s

v

3

,

m0

=

2gm, g

=

lOOOcm/s

2

= 500cm/s



[email protected]

@Sk J ahiruddin, 2020

42

physicsguide CSIR NET, GATE

B asic Newtonian l\llechanics: Part-2

,

11 oo :01 : s1 j [email protected]

physicsguide CSIR NET, GATE

42

@Sk J ahiruddin, 2020

Basic Newtonian Nlechanics: Part-2

Sol 2.9. Using t he same principle of problem Prob 2. 7. , we get

dm dv F = dt v + m dt mo dv or, F = --v + m(t) d

t

T

or,

t mo F +-v=mo 1 - T

T

dv dt

or,

Int egrating bot h side, we get Ft

v= mo

t

- 1

1- -

T

Sol 2.10. 540 km/ h is 150 m/s

Well you have thought quite right . The starting t hought would be the variable mass problem . But, t he velocity in horizontal direction do not change. The horizontal velocity of food packet is, when released, t he same before and after release. So t he momentum change is in t he vert ical direction only. Horizontal motion do not get effected. Please go t hrough the previous solutions of variable mass problems once again. j [email protected]

43

physicsguide CSIR NET, GATE

11 oo : 01 : sg j [email protected]

physicsguide CSIR NET, GATE

42

@Sk J ahiruddin, 2020

Basic Newtonian Nlechanics: Part-2

Sol 2.9. Using t he same principle of problem Prob 2. 7. , we get

dm dv F = dt v + m dt mo dv or, F = --v + m(t) d

t

T

or,

t mo F +-v=mo 1 - T

T

dv dt

or,

Int egrating bot h side, we get Ft

v= mo

t

- 1

1- -

T

Sol 2.10. 540 km/ h is 150 m/s

Well you have thought quite right . The starting t hought would be the variable mass problem . But, t he velocity in horizontal direction do not change. The horizontal velocity of food packet is, when released, t he same before and after release. So t he momentum change is in t he vert ical direction only. Horizontal motion do not get effected. Please go t hrough the previous solutions of variable mass problems once again. j [email protected]

43

physicsguide CSIR NET, GATE

11 oo : 02 : 02 j [email protected]

physicsguide CSIR NET, GATE

42

@Sk J ahiruddin, 2020

Basic Newtonian Nlechanics: Part-2

Sol 2.9. Using t he same principle of problem Prob 2. 7. , we get

dm dv F = dt v + m dt mo dv or, F = --v + m(t) d

t

T

or,

t mo F +-v=mo 1 - T

T

dv dt

or,

Int egrating bot h side, we get Ft

v= mo

t

- 1

1- -

T

Sol 2.10. 540 km/ h is 150 m/s

Well you have thought quite right . The starting t hought would be the variable mass problem . But, t he velocity in horizontal direction do not change. The horizontal velocity of food packet is, when released, t he same before and after release. So t he momentum change is in t he vert ical direction only. Horizontal motion do not get effected. Please go t hrough the previous solutions of variable mass problems once again. j [email protected]

43

physicsguide CSIR NET, GATE

11 oo : 02 : 04 j [email protected]

physicsguide CSIR NET, GATE

42

@Sk J ahiruddin, 2020

Basic Newtonian Nlechanics: Part-2

Sol 2.9. Using t he same principle of problem Prob 2. 7. , we get

dm dv F = dt v + m dt mo dv or, F = --v + m(t) d

t

T

or,

t mo F +-v=mo 1 - T

T

dv dt

or,

Int egrating bot h side, we get Ft

v= mo

t

- 1

1- -

T

Sol 2.10. 540 km/ h is 150 m/s

Well you have thought quite right . The starting t hought would be the variable mass problem . But, t he velocity in horizontal direction do not change. The horizontal velocity of food packet is, when released, t he same before and after release. So t he momentum change is in t he vert ical direction only. Horizontal motion do not get effected. Please go t hrough the previous solutions of variable mass problems once again. j [email protected]

43

physicsguide CSIR NET, GATE

11 oo : 02 : 01 j [email protected]

physicsguide CSIR NET, GATE

42

@Sk J ahiruddin, 2020

Basic Newtonian Nlechanics: Part-2

Sol 2.9. Using t he same principle of problem Prob 2. 7. , we get

dm dv F = dt v + m dt mo dv or, F = --v + m(t) d

t

T

or,

t mo F +-v=mo 1 - T

T

dv dt

or,

Int egrating bot h side, we get Ft

v= mo

t

- 1

1- -

T

Sol 2.10. 540 km/ h is 150 m/s

Well you have thought quite right . The starting t hought would be the variable mass problem . But, t he velocity in horizontal direction do not change. The horizontal velocity of food packet is, when released, t he same before and after release. So t he momentum change is in t he vert ical direction only. Horizontal motion do not get effected. Please go t hrough the previous solutions of variable mass problems once again. j [email protected]

43

physicsguide CSIR NET, GATE

11 oo : 02 : 1o j [email protected]

physicsguide CSIR NET, GATE

42

@Sk J ahiruddin, 2020

Basic Newtonian Nlechanics: Part-2

Sol 2.9. Using t he same principle of problem Prob 2. 7. , we get

dm dv F = dt v + m dt mo dv or, F = --v + m(t) d

t

T

or,

t mo F +-v=mo 1 - T

T

dv dt

or,

Int egrating bot h side, we get Ft

v= mo

t

- 1

1- -

T

Sol 2.10. 540 km/ h is 150 m/s

Well you have thought quite right . The starting t hought would be the variable mass problem . But, t he velocity in horizontal direction do not change. The horizontal velocity of food packet is, when released, t he same before and after release. So t he momentum change is in t he vert ical direction only. Horizontal motion do not get effected. Please go t hrough the previous solutions of variable mass problems once again. j [email protected]

43

physicsguide CSIR NET, GATE

11 oo : 02 : 12 j [email protected]

physicsguide CSIR NET, GATE

42

@Sk J ahiruddin, 2020

Basic Newtonian Nlechanics: Part-2

Sol 2.9. Using t he same principle of problem Prob 2. 7. , we get

dm dv F = dt v + m dt mo dv or, F = --v + m(t) d

t

T

or,

t mo F +-v=mo 1 - T

T

dv dt

or,

Int egrating bot h side, we get Ft

v= mo

t

- 1

1- -

T

Sol 2.10. 540 km/ h is 150 m/s

Well you have thought quite right . The starting t hought would be the variable mass problem . But, t he velocity in horizontal direction do not change. The horizontal velocity of food packet is, when released, t he same before and after release. So t he momentum change is in t he vert ical direction only. Horizontal motion do not get effected. Please go t hrough the previous solutions of variable mass problems once again. j [email protected]

43

physicsguide CSIR NET, GATE

11 oo : 02 : 1 s j [email protected]

physicsguide CSIR NET, GATE

42

@Sk J ahiruddin, 2020

Basic Newtonian Nlechanics: Part-2

Sol 2.9. Using t he same principle of problem Prob 2. 7. , we get

dm dv F = dt v + m dt mo dv or, F = --v + m(t) d

t

T

or,

t mo F +-v=mo 1 - T

T

dv dt

or,

Int egrating bot h side, we get Ft

v= mo

t

- 1

1- -

T

Sol 2.10. 540 km/ h is 150 m/s

Well you have thought quite right . The starting t hought would be the variable mass problem . But, t he velocity in horizontal direction do not change. The horizontal velocity of food packet is, when released, t he same before and after release. So t he momentum change is in t he vert ical direction only. Horizontal motion do not get effected. Please go t hrough the previous solutions of variable mass problems once again. j [email protected]

43

physicsguide CSIR NET, GATE

11 oo : oo : oo

Problellls and Solutions in Angular Molllentulll and Rigid body Mechanics : Part-1 Sk J ahiruddin * Sandip Biswas

*Assistant Professor Sister Nibedita Govt. College, Kolkata Author was the topper of IIT Bombay iVI.Sc Physics 2009-2011 batch He ranked 007 in IIT JAM 2009 and 008 (JRF) in CSIR NET June 2011 He has been teaching CSIR NET aspirants since 2012

1

@Sk J ahiruddin , 2020

Angular momentum & Rigid body: Part-1

11 oo : oo : 03 1

Angular momentum & Rigid body: Part-1

@Sk J ahiruddin , 2020

Contents 1 Simple motion of rigid bodies 1.1

3

Angular Momentum and torque

1.2 Moment Of Inert ia . . . . . . .















3















9









16

1.3

Rotational and transnational motion

1.4

Ans Keys







































29

1.5

Hint s . . .







































31

1.6

Solutions .







































35

j [email protected]

2

physicsguide CSIR NET, GATE

11 oo : oo : os

j [email protected]

2

@Sk J ahiruddin , 2020

Angular momentum & Rigid body: Part-1

physicsguide CSIR NET I GATE

Problems from NET, GATE, JEST, TIFR & JAM papers

1 1.1

Simple motion of rigid bodies Angular Momentum and torque

Prob 1.1. A solid sphere of mass m and radius a is rolling

with a linear speed v on a flat surface without slipping. The magnitude of angular momentum of t he sphere in t he surface is [JAM 2005] 2 (a) mav 5

7 (b) mav 5

3 (c) mav 2

(d) mav

Prob 1.2. A particle of mass 2kg is moving such t hat at a time t, its position in meter, is given by r(t) = 51 - 2t 23. The angular moment um of the particle at t = 2sec about 2 2 t he origin in kgm s- , is [GATE 2006]

(a) - 40k

(b) - 80k

(c) 80k

(d) 40k

Prob 1.3. A mass m is constrained to move on a horizontal frictionless surface. It is set in circular motion wit h radius r 0 and angular speed w0 by an applied force F communi-

cated through an inextensible thread that passes t hrough a

11 oo : oo : 09 mass m 1s cons ra1ne frictionless surface. It is set in circular motion with radius r 0 and angular speed w0 by an applied force F communicated through an inextensible thread that passes t hrough a [email protected]

3

@Sk J ahiruddin, 2020

Angular momentum & Rigid body: Part-1

physicsguide CSIR NET, GATE

hole on t he surface as shown in the fig. This force is t hen suddenly doubled. The magnitude of t he radial velocity of [GATE 2008] t he mass

F

(a) increases t ill the mass fall into the hole. (b) decreases t ill t he mass falls into the hole. (c) remains constant. (d) becomes zero at radius r 1 where O < r 1 < r 0 . Prob 1.4. Two bodies of equal mass m are connected by t he massless rigid rod of length l lying in the xy-plane with

t he center of the rod at the origin. If the system is rotating about the z-axis wit h a frequency w, its angular momentum is [NET Dec 2012] 2w 2w (a) ml -;; (b) ml ~ (c) ml 2w (d) 2ml 2w

11 oo : oo : 11 the massless rigid rod of length l lying in the xy-plane with the cent er of t he rod at the origin. If the system is rotating about the z-axis wit h a frequency w , its angular moment um [NET Dec 2012] is

(a) ml 2 ~

(b) m l2 ~

(c) ml 2w

(d) 2ml 2w

[email protected]

4

@Sk J ahiruddin , 2020

Angular momentum & Rigid body: Part-1

physicsguide CSIR NET, GATE

Prob 1.5. A t hin massless rod of length 2l has equal point masses m attached at its ends (see fig) . T he rod is rotat-

ing about an axis passing through its center and making an angle 0 with it. The magnitude of the rate of change of its angular momentum

2

2

-

~f (only absolute value) is

(a) 2ml w sin 0 cos 0 2

2

(c) 2ml w sin 0

2

2

[JAM 2009]

2

(b) 2ml w sin 0 2 2

2

(d) 2ml w cos 0

Prob 1.6. At an instant shown, t hree point masses m , 2m

11 oo : oo : 14 2

2

(a) 2ml w sin 0 cos 0 2

2

2

(b) 2ml w sin 0 2

(c) 2ml w sin0

2

2

(d) 2ml w cos0

2

2

Prob 1.6. At an instant shown, three point masses m , 2m

and 3m rest on a horizontal surface, and are at the vertices of an equilateral triangle of unit side length. Assuming that G is t he gravitational constant, t he magnit ude and direction j [email protected]

5

@Sk J ahiruddin , 2020

Angular momentum & Rigid body: Part- I

physicsguide CSIR NET, GATE

of t he torque on t he mass 3m, about the point O, at t hat instant is [JAM 2014]

m

0

(a) zero 2 (b) ~ Gv'3m , going into t he paper 2 (c) 3Gv'3m , coming out of the paper 2 (d) !G-/3m , going into the paper

Pro b 1. 7. A spherical ball of ice has radius Ro and is rotating with an angular speed w about an axis passing t hrough its center. At time t = 0, it starts acq uiring mas because

11 oo : oo : 16

Prob 1. 7. A spherical ball of ice has radius Ro and is rotating with an angular speed w about an axis passing t hrough its center. At time t = 0, it starts acquiring mas because t he moisture (at rest ) around it starts to freeze on it uniformly. As a result its radius increases as R (t ) = Ro + at, where a is a constant. The curve which best describes its angular speed with t ime is [JAM 2014]

[email protected]

6

@Sk J ahiruddin, 2020

Angular moment um & Rigid body: Part-1

(a)

physicsguide CSIR NET, GATE

(b)

t (c)

0>1....o--

t

I

(d)

(I).,___ t

Prob 1.8. MSQ: A particle moves in a circular path in t he xy-plane centered at the origin. If t he speed of the particle [JAM 2016] is constant, then its angular momentum (a) about t he origin is constant both in magnitude and direction (b) about (0,0 ,1) is constant in magnitude but not in direct ion (c) about (0,0,1) varies both in magnitude and direction

11 oo : oo : 19 rection (b) about (0,0,1) is constant in magnit ude but not in direction (c) about (0,0,1) varies both in magnit ude and direction (d) about (0,0,1) is a const ant in direction but not in magnit ude

Prob 1.9. A uniform circular disc of radius R and mass

M is rotating wit h angular speed w about an axis, passing t hrough its center and inclined at an angle 60 · with respect [email protected]

7

@Sk J ahiruddin, 2020

Angular momentum & Rigid body: Part-1

physicsguide CSIR NET, GAT E

to its symmetry axis. The magnit ude of t he angular mo[GATE 2013] ment um of the disc is

(a) J}wM R

(c)

f wMR

2

2

(b) 1wMR (d)

f wMR

2

2

Prob 1.10. What is the change in the kinetic energy of

rotation of t he earth if its radius shrinks by 1%? Assume t hat the mass remains the same and the density is uniform. [JEST 2019] (a) increases by 1% (b) increases by 2% (c) decreases by 1%

(d) decreases by 2%

Pro b 1.11. If t he diameter of t he Eart h is increased by 4 % without changing t he mass, t hen t he length of t he day is

11 oo : oo : 21 (a) increases by 1%

(b) increases by 2%

(c) decreases by 1%

(d) decreases by 2%

Pro b 1.11. If t he diameter of t he Earth is increased by 4 % without changing t he mass, then t he length of the day is _____ hours. [JAM 2019]

(Take t he length of the day before the increment as 24 hours. Assume the Earth to be a sphere with uniform density.) (Round off to 2 decimal places) Prob 1.12. A wheel is rotating at a frequency Jo Hz about

a fixed vertical axis. The wheel stops in t 0 seconds, with constant angular deceleration. The number of t urns covered by the wheel before it comes to rest is given by: [JAM j [email protected]

8

@Sk J ahiruddin, 2020

Angular momentum & Rigid body: Part-1

physicsguide CSIR NET, GATE

2020]

(A) Joto Prob 1.13. The angular momentum of a particle relative

to origin varies wit h t ime ( t ) as L = (4x + at 2 y) kgm 2 /s, where a = l kgm 2 /s 3 . The angle between L and the t orque acting on t he particle becomes 45° after a time of - - - - - - - - - - - - - - s [JAM 2020] --+

1.2

Moment Of Inertia

Prob 1.14. The moment of inertia of uniform radius r about ')

11 oo : oo : 24

1.2

Moment Of Inertia

Prob 1.14. The moment of inertia of uniform radius r about 2 5 an axis passing through its center is given by -~r p. A rigid sphere of uniform mass density p and racfius R has

two smaller sphere of radius ~ hollowed out of it , as shown in fig. The moment of inertia of t he resulting body about t he Y axis is [GATE 2007]

[email protected]

9

@Sk J ahiruddin , 2020

Angular momentum & Rigid body: Part-1

physicsguide CSIR NET, GATE

y

X

11 oo : oo : 26 X

(b) 51rpR5 12

( C)

5 71rpR

12

(d) 31rpR5 4

Prob 1.15. Consider a uniform thin circular disk of radius R and mass M. A concentric square of side ~ is cut out from the disk (see fig.). What is moment of inertia of t he resultant disk about an axis passing t hrough the center of [JAM 2017] t he disk and perpendicular to it?

j [email protected]

10

@Sk J ahiruddin , 2020

Angular momentum & Rigid body: Part-1

physicsguide CSIR NET I GATE

R




11 oo : oo : 29 R




(b) I

=

M R2 2

(d)I =MR2 1 2

1 1 - 481r 1

241r

Prob 1.16. A cylindrical shell of mass m has an outer radius b and an inner radius a. The moment of inertia of the shell about the axis of the cylinder is [JEST 2016]

(a) lm(b 2 (c) m(b

2

2

-

a

+a

2

2

)

)

(b) lm(b 2 ( d)

m(b

2

2

-

+a a

2

2

)

)

Prob 1.17. The three-dimensional object sketched on the [email protected]

11

@Sk J ahiruddin, 2020

Angular momentum & Rigid body: Part-1

physicsguide CSIR NET, GATE

right is made by taking a solid sphere of uniform density (shaded) wit h radius R , and scooping out a spherical cavity (unshaded) as shown, which has diameter R. If this object has mass M, its moment of inertia about the tangential axis passing through the point where the spheres touch (as shown in the figure) is [TIFR 2019]

11 oo : oo : 31 unsh aded as shown, which has diameter R. If this object has mass M, its moment of inertia about t he tangential axis passing through the point where the spheres touch (as shown in the figure) is [TIFR 2019]

3 2 (a) MR 16

(c) 31 MR2 70

(b) 62 MR2 35

(d) 31 MR2

20

Pro b 1.18. Seven uni£orm disks, each of mass m and radius r, are inscribed inside a regular hexagon as shown. The moment of inertia of t his syst em of seven disks, about an axis passing through t he central disk and perpendicular to t he plane of the disks, is [JAM 2015]

[email protected]

12

@Sk J ahiruddin, 2020

Angular moment um & Rigid body : Par t-1

physicsguide CSIR NET, GATE

11 oo : oo : 3s

(a) ;mr

2

(b) 7mr

1

2

(c) }mr2

(d)

55 2 mr 2

Prob 1.19. Consider the uniform solid right cone depicted in t he figure. This cone has mass M and a circular base of radius r . If the moment of inert ia of the cone about an axis parallel to t he X axis passing through t he center of mass Oc.m. (see fig) is given by, s15M( 4r 2 + h2 ) then the moment of inert ia about another axis parallel to t he X axis, but passing [TIFR 2013] t hrough t he point Or(see fig) , is

physicsguide CSIR NET, GATE

j a [email protected]

13

@Sk J a hiruddin , 2020

Angular momentum & Rigid body: Part-1

.

"

11 oo : oo : 37

--

••

T Soffd COfl•_. Man •M

(a) /o M (4r (c)

1 20 M

2

+h

2

(23r 2 + 2h2 )

)

+h ) 2 2 ~M(15r + 4h ) 3

(b) foM(2r

(d)

2

2

Prob 1.20. The mass density of a disc of mass m and radius R varies as

p(r)

r

p( l - R) for (0 < r < R)

0 (for r > R)

Find t he moment of inertia of t he disc about t he axis perpendicular to the plane of the disc and passing t hrough the [email protected]

14

@Sk J ahiruddin , 2020

Angular momentum & Rigid body: Part-1

physicsguide CSIR NET, GATE

center of t he disc in t erms of m and R.

[JAM 2013]

11 oo : oo : 40 Angular momentum & Rigid body: Part-1

@Sk J ahiruddin, 2020

center of t he disc in t erms of m and R.

[JAM 2013]

Prob 1 .21. The moment of inert ia of a disc about one of its diameters is IM. The mass per unit area of the disc is proportional to the distance from its cent er . If t he radius of t he disc is R and it s mass is M , the value of IM is [JAM 2014]

(a) 1MR

2

(b) ~M R

2

(c)

2 3 10 M R

(d) ~M R

2

Prob 1.22. T wo uniform t hin rods of equal length, L and masses M 1 and M 2 are joined toget her along t he length. The moment of inert ia of the combined rod of length 2£ about an axis passing through the mid-point and perpendicular to t he length of t he rod is [GATE 2012]

(a) (M1 + M2) f;

(c) (M1

+ M2 )!f

2

(b) (M1 + M2)~ (d) (M1

+ M2)¥

Prob 1 .23. The mass per unit length of a rod (length 2 m) varies as p = 3x kg/m. The moment of inert ia (in kg m 2 ) of t he rod about a perpendicular-axis passing the t ip of the [JAM 2019] rod (at x = 0) (A) 10 (B) 12 (C) 14 (D) 16

[email protected]

15

@Sk J ahiruddin , 2020

Angular momentum & Rigid body : Part-1

physicsguide CSIR NET, GATE

11 oo : oo : 42 [email protected]

15

@Sk J ahiruddin , 2020

Angular momentum & Rigid body : Part-1

1.3

physicsguide CSIR NET, GATE

Rotational and transnational motion

Prob 1.24. A uniform cylinder of radius r and length l ,

and a uniform sphere of radius R are released on an inclined plane when their centers of mass are at t he same height . If t hey roll down without slipping, and if t he sphere reaches t he bot tom of the plane with a speed V, then the speed of [NET June t he cylinder when it reaches the bottom is 2013]

(a) V

l4rl 15r 2

(c) ~

(b) 4V ~

(d) V

14 15

Prob 1.25. Moment of inertia of a solid cylinder of mass

m , height h and radius r about an axis (shown in fig by dashed line) passing through its center of mass and perpen[JAM 2009] dicular to its symmetry axis is

j [email protected]

16

physicsguide CSIR NET, GATE

11 oo : oo : 44

j [email protected]

16

@Sk J ahiruddin , 2020

Angular momentum & Rigid body: Part-1

physicsguide CSIR NET, GATE

'I I I

•i • •t I

r J



I

h:

~

• ''I I I I I

I

2 1 (a) 4 mr 2 c) lmr ( 2

+ l12 mh

2 + lmh 12

2

(b) ~mr

2

+ ~mh

2

2 2 (d) lmr + lmh 2 4

Pro b 1. 26. A uniform rod of mass m and length L is hinged at one of its ends O and is hanging vertically. It is hit at

its midpoint with a very short duration impulse J so that it starts rotating about O. Find the magnitude and the direction of t he horizontal impulse t hat O applies on the rod [JAM 2014] when it is hit.

11 oo:oo:47

[email protected]

17

@Sk J ahiruddin, 2020

Angular momentum & Rigid body: Part-1

physicsguide CSIR NET, GATE

0

J

Prob 1.27. A mass m t ravels in a straight line with velocity v 0 perpendicular to a uniform stick of mass m and length L , which is initially at rest. The distance from t he center of the stick to the path of the t raveling mass is h (see fig). Now the traveling mass m collides elastically wit h the stick, and t he center of t he stick and t he mass m are observed to move with equal speed after the collision Assuming that t he t raveling mass m can be treated as a point mass, and t he 2 moment of inertia of the stick about its cent er is I = , it follows that t he distance h must be [TIFR 2010]

~t

m

L

11 oo : oo : 49 m

L

[email protected]

18

@Sk J ahiruddin , 2020

Angular momentum & Rigid body: Part-1

(a)

t

(b)

i

(c)

}e

(d)

physicsguide CSIR NET, GATE

!h,

(e)

f

(f) zero

Prob 1.28. A scient ist is given two heavy spheres made of same metal, which have t he same diameter and weight , and is asked to distinguish and spheres, without damaging t hem in any way. Though the spheres look ident ical, one of t hem is actually a hollow spherical shell, while t he other is a set of concentric shells mounted on four thin rods of the same metal (see fig). [TIFR 2011]

To make t he distinction, the scientist must perform an experiment where each sphere is (a) set rotating under the action of a constant torque (b) made into t he bob of a long simple pendulum and set oscillating

11 oo : oo : s2 experiment where each sphere is (a) set rotating under t he action of a constant torque (b) made into t he bob of a long simple pendulum and set oscillating (c) immersed fully in a non-corrosive liquid and then weighed (d) given t he same electric charge Q and t he potential is measured

j [email protected]

19

@Sk J ahiruddin, 2020

Angular momentum & Rigid body: Part-1

physicsguide CSIR NET, GATE

Prob 1.29. A ring of mass m and radius R rolls (wit hout slipping) down an inclined plane starting from rest. If the

center of the ring is initially at a height h, t he angular velocity when the ring reaches the base is [NET December 2013]

h

(a)

(c)

g

(b)

h _ R tan0

g(h - R) R2

(d)

g

h- R

2g h-R

11 oo : oo : 55 (a)

(c)

g

(b)

h _ R tan0

g(h - R)

R2

g

h-R

2g h-R

(d)

Prob 1.30. MSQ: A rod is hanging vertically from a pivot. A particle traveling in horizontal direction, collides with the rod as shown in fig. For the rod-particle syst em, consider t he linear momentum and t he angular momentum about the [email protected]

20

@Sk J ahiruddin, 2020

Angular momentum & Rigid body: Part-1

physicsguide CSIR NET, GATE

pivot. Which of the following statements are NOT t rue? [JAM 2015]





(a) Both linear momentum and angular momentum are conserved (b) linear momentum is conserved but angular momentum is not (c) linear momentum is not conserved but angular moment um is conserved (d) Neither linear momentum nor angular momentum are

11 oo :oo : s1 is not (c) linear momentum is not conserved but angular moment um is conserved (d) Neit her linear moment um nor angular momentum are conserved

Prob 1.31. A hoop of radius a rotates with constant angular velocity w about the vert ical axis as shown in the fig. A bead of mass m can slide on the hoop wit hout fiction. If g < w 2 a at what angle 0 apart from O and 1r is t he bead stationary (i.e.,!~=~:i =0)?

[JEST 2016]

j [email protected]

21

@Sk J ahiruddin, 2020

Angular momentum & Rigid body: Part-I

(a) tan 0 = (c) cos 0 = ~

9

; 2a

physicsguide CSIR NET I GATE

(b) sin 0 = ( d)

tan 0 =

w·~a

1r!2a

11 oo : 01 : oo

(a) tan0 = C:fa

(b) sin0 = w.ga

(c) cos 0 = w~a

(d) tan 0 = 1r32a

Prob 1.32. Two cylinders A and B of t he same length L and outer radius R were placed at t he same height h on an inclined plane at an angle ef> with the horizontal (see fig) . Starting from rest, each cylinder was allowed to roll down t he plane without slipping. It was found t hat A reached t he end of t he inclined plane earlier t han B . which of the [TIFR 2015] following possibilit ies could be true.

[email protected]

22

@Sk J ahiruddin, 2020

Angular momentum & Rigid body: Part-1

physicsguide CSIR NET, GATE

h I

I

(a) A is solid and made of copper; B is hollow and made of copper. (b) A is solid and made of copper; B is solid and made of

11 oo : 01

: 03

(a) A is solid and made of copper; B is hollow and made of copper. (b) A is solid and made of copper; B is solid and made of aluminium. (c) A is hollow and made of aluminium; B is solid and made of aluminium. (d) A is hollow and made of copper; B is hollow and made of copper ; B is heavier than A. Prob 1.33. A t hin rod of length 2l and mass M is pivoted at one end P on a horizontal plane (see fig). A ball of mass m < < M and speed v 0 strikes t he free end of t he rod perpendicularly and bounces back with velocity VJ along the original line of motion as shown in the fig. If t he collision is [email protected]

23

@Sk J ahiruddin, 2020

Angular momentum & Rigid body: Part-1

physicsguide CSIR NET, GATE

[TIFR 2015]

perfectly elastic the magnitude of v f is

1't _...,. Vo _____________ _

• I I I I I

M

21 I

t I I

I

t n

11 oo : 01 : os I I I I

I



p

M (a ) M

+ 3m

- 3m M-4m (c) M + 4m Vo

b) M 3mv ( M + 3m o

Vo

(d) M + 4m vo M-4m

Prob 1 .34. A uniform solid wheel of mass M and radius r is halted at a step of height h as shown in t he fig . The

minimum force F , applied horizontally at the center of t he wheel, necessary to raise t he wheel over t his step is [TIFR 2016]

j [email protected]

24

@Sk J ahiruddin , 2020

Angular momentum & Rigid body: Part- I

physicsguide CSIR NET, GATE

F

r _ _.....:::::......a...:::::.........i.. - - - - - - -

(a ) (C)

M ✓h(2r-h) g

r+h

M ✓h(r+h) g

r- h

_t !i

b) M ✓h(2r+h) ( g r- h

d) M ✓h(2r-h) ( g r- h

11 oo :01 : oa

(a)

J h(2r- h) M g r+h

c) M J ( g

h(r+h) r- h

b) M J h(2r+h) ( g r- h d) M J h(2r- h) ( g

r- h

Prob 1.35. A uniform rigid met er-scale is held horizontally

with one of its end at the edge of a table and t he other supported by hand. Some coins of negligible mass are kept on [JAM 2017] t he meter scale as shown in the fig.

As t he hand support ing t he scale is removed, t he scale starts rotating about its edge on the table and the coins start moving. If a photograph of the rotating scale is taken soon after, it will look closest to:

[email protected]

25

@Sk J ahiruddin, 2020

Angular momentum & Rigid body: Part-1

physicsguide CSIR NET, GATE

tA)

(B)

(C)

(D)

Prob 1.36. A pendulum is made of a massless string of

11 oo : 01

: 1o (0)

(C)

Prob 1.36 . A pendulum is made of a massless string of

lengt h L and a small bob of negligible size and mass m . It is released making an angle 00 ( r t hen v > vo .

Sol 1.4.

I =

L m ir; Z

2

+m

=m 2

l

2

2

ml2 2 hence,

z2

L = I w =m-w 2

Sol 1.5. In this case w is constant. Then from t he definition

[email protected]

37

@Sk J ahiruddin , 2020

Angular momentum & Rigid body: Part-1

of angular momentum .

.

1,

.

'l

.

'l

dL

-

dt

=m

L. r -w -r· 2-

i

1,

- (r· i

i

-) ·w

physicsguide CSIR NET, GATE

11 oo : 01

: 44 .

1,

_,

L

dL 2 _, _, ( _, _,) -dt =m . r-i w-r·,,, r·,,, · w 'l,

.

'l,



1,

Now iJ =

w X r'

t hen

_, _, V ·W Therefore,

=

(_, W

X

;::;' \ . . . T; · W

=

0

_,

dL ~ _, ( _, _,) dt = -m ~ V i r i · w 'l,

j [email protected]

38

@Sk J ahiruddin, 2020

Angular moment um & Rigid body: Par t-1

physicsguide CSIR NET, GATE

Using (r, 0. Which of t he following statements is t rue for the coordinates of t he centre of mass of t he disc in t he reference frame of t he car? (a) Only t he x and the z coordinat es change (b) Only t he y and t he z coordinates change (c) Only t he x and the y coordinates change (d) all the t hree coordinates change. j [email protected]

21

@Sk J ahiruddin , 2020

Angular moment um & Rigid body: Part-2

physicsguide CSIR NET I GATE

Prob 2.12. T wo objects of unit mass are thrown up vert i1 cally wit h a velocity of 1m s- at latitudes 45°N and 45°S,

respectively. T he angular velocit y of t he rotat ion of Earth is given to be 7.29 x 10- 5 s- 1 . In which direction will t he objects deflect when t hey reach t heir highest point (due to Coriolis force)? Assume zero air resistance. [JEST 2019] (a) to t he east in Nort hern hemisphere and west in Sout hern Hemisphere (b) to the west in Northern hemisphere and east in Sout hern Hemisphere (c) to t he east in both hemisphere and east in Sout hern Hemisphere (d) to t he west in bot h hemispheres Prob 2.13. Consider an obj ect moving wit h a velocity v in

11 oo : 01 : oo (c) to the east in both hemisphere and east in Southern Hemisphere (d) to the west in both hemispheres Prob 2.13. Consider an object moving with a velocity iJ in

a frame which rotates with a constant angular velocity w. The Coriolis force experienced by the object is [JAM 2019] (a) along iJ (b) along w (c) perpendicular to both iJ and w (d) always directed towards t he axis of rotation Prob 2.14. A turn-table is rotating with a constant angu-

lar velocity w 0 - In the rotating frame fixed to the t urntable, [email protected]

22

@Sk J ahiruddin, 2020

Angular momentum & Rigid body: Part-2

physicsguide CSIR NET, GATE

a particle moves radially outwards at a constant speed v 0 . The acceleration of the particle in t he r0 -coordinates, as seen from an inert ial frame, t he origin of which is at the centre of the turnable is [NET June 2019]

(a) -rw5f (b) 2rw5f 2 ,. _ (d) -rw0 f + 2v 0w0 0

+ vowo0

(c) rw5f

+ 2vowo0

11 oo : 01

: 02

[email protected]

23

@Sk J ahiruddin, 2020

Angular momentum & Rigid body: Par t-2

2.1

physicsguide CSIR NET, GATE

Ans Keys

2.1. b

2.8. a

2.2. b

2.9. b

2.3. 0.34

2.10. b , c

2.4.

C

2.11. d

2.5.

C

2.12. d

11 oo : 01 : os 2.4. c

2.11. d

2.5. c

2.12. d

2.6. b

2.13. c

2.7. c

2.14. d

j [email protected]

24

@Sk J ahiruddin , 2020

Angular momentum & Rigid body: Part-2

2.2

physicsguide CSIR NET, GATE

Hints

2.1. There can be no hints for this one. 2. 2. Use the expression of Coriolis force and centrifugal

force .

2.3. The weight are different in pole and equator due to different centrifugal force.

11 oo : 01

: 01

force . 2.3. The weight are different in pole and equator due to

different centrifugal force. 2.4. Use the expression of Coriolis force and centrifugal

force to find out the magnitude of t he forces. 2.5. Use t he expression of Coriolis force to find out the di-

rection and the magnitude of the force. 2.6. Use t he expression of Coriolis force to find out which direction t he particle goes. 2. 7. Use the expression of Coriolis force and centrifugal

force , then cornpare t he two. 2.8. Use t he expression of Coriolis force to find out the direction and the magnit ude of the force. 2.9. What is the direction of Coriolis force in southern [email protected]

25

@Sk J ahiruddin, 2020

Angular moment um & Rigid body: Part-2

physicsguide CSIR NET, GATE

sphere? 2 .10. Use t he expression of Coriolis force and centrifugal

force to find out the magnitude and t he direction of the forces . 2.11. The sit uation is analogous to spinning top.

11 oo : 01

: 1o

2.10. Use t e expression o

orce an centri uga force to find out the magnitude and t he direction of the forces . 2.11. The sit uation is analogous t o spinning top. 2.12. Same problem of a previous problem 2.13. Use the expression of Coriolis force and centrifugal force to find out the direction of the forces. 2.14. This one is easy, you don't need hints.

[email protected]

26

@Sk J ahiruddin, 2020

Angular momentum & Rigid body : Part-2

2.3

physicsguide CSIR NET, GATE

Solution

Sol 2.1. Foucault 's pendulum is designed such a way so t hat it can swing back and forth in any direction. As earth rotate t he plane of the pendulum rotates. From which we

11 oo : 01 2.3

: 13

Solution

Sol 2.1. Foucault 's pendulum is designed such a way so t hat it can swing back and forth in any direction. As earth rotate t he plane of t he pendulum rotates. From which we can say that earth is not an inertial frame.

Sol 2.2. Direction of coriolis force

x

is along axis at Q. Then w x Similarly centrifugal force -w x ward.

2w x v-;, where w and v-;

v-; is along y axis. (w x r)

points radially out-

Sol 2.3. The centripetal force on the person at t he pole mwz x (w z x R z)

= 0,

at equator mwz

X

(wz

X

Rf)

= -mw

2

Rf.

So, t he force felt by t he person at equator We

GMm = - R2

=

9

+ mw= R 2

-mg + mw R

at the pole Wp

=

-mg

j [email protected]

27

@Sk J ahiruddin , 2020

Angular momentum & Rigid body: Part-2

physicsguide CSIR NET, GATE

Therefore

lOO Wp

I)

I!

We

= lOO -mg+ mg - mw _

/YYl

r,

2

R

11 oo : 01

: 16 Angular momentum & Rigid body: Part-2

@Sk J ahiruddin , 2020

Therefore

lOO Wp

-

We

= lOO -mg + mg -

2

mw R

- mg

Wp

= lOO w2R g

= 100

2

21r 86400

6400 X 10 3 10

= .338 ~

.34 •• ......

Sol 2.4. The acceleration of the ovserver R = 0. The coriolis force F cor

= m2w

x iJ

= .01 · 2 · 22 X .06( -f) " = .0024. So, t oward his right. Sol 2.6. For P the coriolis force F corp

= - m 2w x iJ = - 2mwvcf> A

i.e t oward west. For Q t he coriolis force F corQ

= - m 2w x iJ = - 2m wv cf> A

i. e also toward west. Sol 2. 7. Coriolis force F co

= - 2mwz x (-vJ) 2 = - 2mw rf

j [email protected]

29

@Sk J ahiruddin , 2020

Angular momentum & Rigid body: Part-2

physicsguide CSIR NET, GATE

11 oo : 01

: 21

[email protected]

29

@Sk J ahiruddin , 2020

Angular momentum & Rigid body: Part-2

physicsguide CSIR NET, GATE

Centrifugal force

Fe = -mwz

= mw

2

X

(w z

X

rf)

rf

Sol 2.8. Coriolis force on t he bullet ,...

Fcor = - 2mDz x vcp 2

= - 2mD rf So, to his right. Sol 2.9. Cyclone rotates clockwise in southern hemisphere due to coriolis force. Sol 2.10. Coriolis force

Fcor = -m2w x iJ ,...

= -2mwz x (-wRcp) 2 = -2mw Rf Centrifugal force

Fcjg =

-mw X

(w

~

X

R)

= -mwz X (wz X Rf ) 2 = -mw Rz X J) 2 = mw Rf [email protected]

30

physicsguide CSIR NET, GATE

11 oo : 01

: 23 2

= -mw R z 2 = mw Rf

X

J)

j [email protected]

30

@Sk J ahiruddin, 2020

Angular momentum & Rigid body: Part-2

physicsguide CSIR NET, GATE

Hence -+

2

-+

F ear + F cfg

A

= -mw R r

Sol 2 .11. Due to the pseudo force the disc will move toward -x direction at first . Since the body coor·dinate is rotating about t he fixed car coordinate, t here will be a torque acting

on the body which will help it rotate in xy plane. Sol 2.12. Same problem of 3.6, so west in both hemisphere. Sol 2.13. The expression of corriolis force 2mw

always perpendicular to bot h

xv. So it's

w and v.

Sol 2.14. The acceleration in t he fixed coordinate •

ap = aM + w x

r + 2w xv+ w x

(w x r) .



Since v and w are constant aM and w are zero. Hence

ap = 2woz x vof + Wo Z x (wo z x rf)

=

A

2

2wovo - w0 rf

11 oo : 01

: 26 2

= mw Rf j [email protected]

30

@Sk J a hiruddin , 2020

Angular momentum & Rigid body: Part-2

physicsguide CSIR NET, GATE

Hence -+

-+

F ear

+ F cj9 =

2

A

-mw R r

Sol 2 .11. Due to the pseudo force the disc will move toward -x direct ion at first . Since t he body coordinate is rotating about t he fixed car coordinate, there will be a torque acting

on the body which will help it rotat e in xy plane. Sol 2.12. Same problem of 3.6, so west in both hemisphere. Sol 2.13. The expression of corriolis force 2mw x iJ. So it 's

always perpendicular to both w and iJ. Sol 2.14. The acceleration in t he fixed coordinate •

ap = aM+ w x r + 2w x iJ + w x (w x r) . Since

v and w



are constant

ap = 2woz

j a [email protected]

X

aM and w are zero.

Vof

+ Wo Z X

31

(woz

X

Hence

rf)

physicsguide CSIR NET , GATE

11 oo :01 : 2a 2

= mw Rf j [email protected]

30

@Sk J a hiruddin , 2020

Angular momentum & Rigid body: Part-2

physicsguide CSIR NET, GATE

Hence -+

-+

F ear

+ F cj9 =

2

A

-mw R r

Sol 2 .11. Due to the pseudo force the disc will move toward -x direct ion at first . Since t he body coordinate is rotating about t he fixed car coordinate, there will be a torque acting

on the body which will help it rotat e in xy plane. Sol 2.12. Same problem of 3.6, so west in both hemisphere. Sol 2.13. The expression of corriolis force 2mw x iJ. So it 's

always perpendicular to both w and iJ. Sol 2.14. The acceleration in t he fixed coordinate •

ap = aM+ w x r + 2w x iJ + w x (w x r) . •

Since v and w are constant aM and

ap = 2woz

j a [email protected]

X

Vof

w are zero.

+ Wo Z X

31

(woz

X

Hence

rf)

physicsguide CSIR NET , GATE

11 oo : 01

: 31 2

= mw Rf j [email protected]

30

@Sk J a hiruddin , 2020

Angular momentum & Rigid body: Part-2

physicsguide CSIR NET, GATE

Hence -+

-+

F ear

+ F cj9 =

2

A

-mw R r

Sol 2 .11. Due to the pseudo force the disc will move toward -x direct ion at first . Since t he body coordinate is rotating about t he fixed car coordinate, there will be a torque acting

on the body which will help it rotat e in xy plane. Sol 2.12. Same problem of 3.6, so west in both hemisphere. Sol 2.13. The expression of corriolis force 2mw x iJ. So it 's

always perpendicular to both w and iJ. Sol 2.14. The acceleration in t he fixed coordinate •

ap = aM+ w x r + 2w x iJ + w x (w x r) . •

Since v and w are constant aM and

ap = 2woz

j a [email protected]

X

Vof

w are zero.

+ Wo Z X

31

(woz

X

Hence

rf)

physicsguide CSIR NET , GATE

11 oo : 01

: 33 2

= mw Rf j [email protected]

30

@Sk J a hiruddin , 2020

Angular momentum & Rigid body: Part-2

physicsguide CSIR NET, GATE

Hence -+

-+

F ear

+ F cj9 =

2

A

-mw R r

Sol 2 .11. Due to the pseudo force the disc will move toward -x direct ion at first . Since t he body coordinate is rotating about t he fixed car coordinate, there will be a torque acting

on the body which will help it rotat e in xy plane. Sol 2.12. Same problem of 3.6, so west in both hemisphere. Sol 2.13. The expression of corriolis force 2mw x iJ. So it 's

always perpendicular to both w and iJ. Sol 2.14. The acceleration in t he fixed coordinate •

ap = aM+ w x r + 2w x iJ + w x (w x r) . •

Since v and w are constant aM and

ap = 2woz

j a [email protected]

X

Vof

w are zero.

+ Wo Z X

31

(woz

X

Hence

rf)

physicsguide CSIR NET , GATE

11 oo : oo : 01

Advanced Problems and Solutions in Ne-wtonian Mechanics Sk J ahiruddin* Sandip Biswas

*Assistant Professor Sister Nibedita Govt. College, Kolkata Aut hor was the topper of IIT Bombay NI.Sc Physics 2009-2011 bat ch He ranked 007 in IIT JAM 2009 and 008 (JRF) in CSIR NET June 2011 He has been teaching CSIR NET aspira11ts since 2012

1

@Sk J ahiruddin, 2020

Advanced Problems in Newtonian Mechanics

11 oo : oo : 03 1

@Sk J ahiruddin, 2020

Advanced P roblems in Newtonian Mechanics

Contents 1 Advanced Problems on Newtonian Mechanics •





































18

1.2 Hints . . .







































19

Solutions .







































21

1.1

1.3

Ans Keys

3

j [email protected]

2

physicsguide CSIR NET, GATE

11 oo : oo : 06

j ahir@physicsguide. in

@Sk J ahiruddin , 2020

2

physicsguide CSIR NET I GATE

Advanced Problems in Newtonian Mechanics

Problems from NET, GATE, JEST, TIFR & JAM papers

1

Advanced Problems on Newtonian Mechanics

Do these problems only if you are a serious aspirant of JEST and TIFR. CSIR NET and GATE aspirants may skip this section

Prob 1.1. Two masses M 1 and M 2 (M1 < M 2 ) are suspended from perfect ly rigid horizontal support by a system of t hree taut mass-less wires W1 , W 2 and W 3 , as shown in t he figure. All t he three wires have identical cross-sections and elastic properties and are known to be very strong. If the mass M2 is increased gradually, but wit hout limit, we should expect t he wires to break in the following order: [TIFR 2012]

11 oo : oo : 09 •

[email protected]

3

Advanced P roblems in Newtonian Mechanics

@Sk J ahiruddin , 2020

(a) first W 2 , t hen W 1 (c) first W 2 , then W 3

physicsguide CSIR NET, GATE

(b) first W 1 , t hen W 2 ( d)

first W 3

Prob 1.2. A particle slides on t he inside surface of a frict ionless cone. T he cone is fixed wit h its t ip on the ground and its axis vert ical, as shown in t he figure. The semi-vertex angle of t he cone is a. If the particle moves in a circle of radius r 0 , without slipping downwards, the angular frequency w of this motion will be [TIFR 2015]

11 oo : oo : 11 1us r 0 , wit out s 1pp1ng w of this motion will be

e angu ar

[TIFR 2015]

[email protected]

Advanced P roblems in Newtonian Mechanics

_.

(a)

(c)

g

(b)

ro tan a g •

r 0 sin a

physicsguide CSIR NET, GATE

4

@Sk J ahiruddin , 2020

equency

(d)

----

g

ro cot a g

r 0 cos a

Prob 1.3. A stone is dropped is vertically from t he top of a tower of height 40 rn. At the same t ime a gun is aimed directly at the stone from the ground at a horizontal distance 30 m from t he base of the tower and fired. If the bullet from the gun is to hit t he stone before it reaches t he ground, t he minimum velocity of the bullet must be, approximately, [TIFR 2013] /,...\ 17 '7,yy-,,., -1

11 oo : oo : 13 tance 30 m from t he base of the tower and fired. If t he bullet from the gun is to hit t he stone before it reaches t he ground, t he minimum velocity of t he bullet must be, approximately, [TIFR 2013] (a)57.4ms- 1 (b)27.7ms-1 (c) 17.7ms -1 (d)7.4ms-1 Prob 1.4. A car st arts from rest and accelerates under a force F increasing linearly in time as F = at at where j [email protected]

5

physicsguide CSIR NET, GATE

Advanced Problems in Newtonian Mechanics

@Sk J ahiruddin , 2020

a is const ant . At t ime t 1 > 0, t he force F is suddenly switched off. At a later time t 2 > t 1 , brakes are applied resulting in a force F' whose magnit ude increases with time, F' = -a(t - t 2 ) where a is t he same constant as before . which of t he following graphs would best represent the [TIFR change in the posit ion of the car x(t) wit h time? 2015]

(a)

(b)

x(t)

x(t)

(c)

(d)

x(t)

0

x(t)

I I I I I I I I I I

t,

l2

12+ 11

t

0

,,

'2

f 2 +t ,

t

11 oo : oo : 16 x(t)

x6)

Prob 1.5. A ball is dropped vertically from the height H on to a plane surface and permitted to bounce repeatedly along a vertical plane. After every bounce, its kinetic energy

becomes a quarter of its kinetic energy before the bounce. The ball will come to rest after time [TIFR 2016] [email protected]

6

@Sk J ahiruddin, 2020

(a) infinity

physicsguide CSIR NET, GATE

Advanced Problems in Newtonian Mechanics

1

1

(b) 2H 2

(c) 2 2H 2

g

g

(d)3

1 2H 2 g

Prob 1.6. In a moving car , the wheels will skid if the breaks are applied too suddenly. This is because [TIFR 2016]

(a)the inertia of the car will carry it forward . (b )the momentum of the car must be conserved. (c)the impulsive retarding force exceeds the limiting force of static friction. (d )the kinetic friction will suddenly get converted to static friction.

3

Prob 1.7. A ball of mass 0.1 kg and density 2000 kg/m is suspended by a massless string of length O.5 m under water having density 1000 kg /m3 . The ball experiences a drag

force, Fd = -0. 2(vb -

ilw), where Vb and Vw are t he veloc-

11 oo : oo : 19 3 kg/m

Prob 1.7. A ball of mass 0.1 kg and density 2000 is suspended by a massless string of length O. 5 m under water having density 1000 kg /m3 . The ball experiences a drag _, force, Fd = - 0.2 (vb - vw), where Vb and Vw are t he velocit ies of the ball and water respectively. What will be t he frequency of small oscillations for the mot ion of pendulum, [JEST 2017] if t he water is at rest ?

Prob 1.8. A small elastic ball of mass m is placed at the apex of a 45 ° inclined plane as shown in the figure below. [email protected]

@Sk J ahiruddin , 2020

7

physicsguide CSIR NET, GATE

Advanced Problems in Newtonian Mechanics

[TIFR 2017]

The ball is allowed to slip wit hout friction down t he plane (along t he dotted line), hit t he ground (as shown) and bounce along it. If the height of t he inclined plane in h and the coefficient of restit ution between the ball and t he ground is 0. 5, then t he distance AB , as marked on t he figure, will be

(a) 3h

(b) 2h

(c) (1 + v2)h

(d) 3v2h

11 oo : oo : 21 h and the coefficient of restitution between the ball and the ground is 0.5, t hen the dist ance AB , as marked on t he fig-

ure, will be

(a) 3h

(b) 2h

(c) (1

+ v'2)h

(d) 3\1'2h

Pro b 1. 9. A toy car is made from a rect angular block of mass M and four disk wheels of mass m and radius r . The car is attached to a vert ical wall by a massless horizontal spring wit h spring constant k and constrained to move

perpendicular to the wall. The coefficient of static friction between the wheels of the car and the floor is µ. The maximum amplitude of oscillations of the car above which the wheel start slipping is [JEST 2017] j [email protected]

@Sk J ahiruddin, 2020

(a)µ g(M

Advanced Problems in Newtonian Mechanics

+ 2m)(M + 4m)

mk 2 µg (M + m) ( ) c 2m k

physicsguide CSIR NET, GATE

8

(b)µg (M

2

-

m

2

)

Mk (d)µg (M + 4m)(M + 6m) 2mk

Prob 1.10. Consider a point part icle A of mass mA colliding elastically with another point particle B of mass mB at

rest , where m B/mA = ry . After collision, the ratio of the kinetic energy of particle B to t he initial kinetic energy of particle A is given by [JEST 2017]

,

,

(c)

2 i ry+ 2 - ,

(d).!_ "'(

Prob 1.11. A cyclist , weighing a t otal of 80 kg with the hir,rrlP n Prl~.1~ ~-t ~- ~n PPli nf 1 n m /~

ShP ~t n n ~ n P 0). What is the distance of the closest approach d? [JEST 2019] k (a) d = b2 + mv~

(c) d = b

1.4

1 2

1

(b) d = b2 -

(d) d =

k mv02

Inverse square Law

k mv~

z

11 oo : oo : 41 (c) d = b

1.4

(d) d =

k

m v02

Inverse square Law

Prob 1.29. A space station is moving in a circular orbit around the Earth goes into a new bound orbit by firing its engine radially outwards. This orbit is [GATE 2007] (a) a larger circle (c) an ellipse

(b) a smaller circle

(d) a parabola

Prob 1.30. A satellite moves around a planet in a circular orbit at a dist ance R from its center. The time period of revolution of the sat ellite is T. If the same satellite is taken to an orbit of radius 4R around t he same planet, t he t ime [JAM 2007] period would be

(a) 8T

(b) 4T

Prob 1.31. A satellite is moving in a circula1~orbit around t he Earth. If T , V and E are its average kinetic, average [email protected]

@Sk J ahiruddin , 2020

15

physicsguide CSIR NET, GATE

Central force

potential and total energies, respectively, then which one of [GATE 2015] t he following options is correct?

(a) V

- 2T; E = - T (c) V = - T / 2; E = T / 2 =

(b) V =-T; E = O (d) V = - 3T/ 2; E = - T / 2

11 oo : oo : 44 [GATE 2015]

t he following options is correct ?

(a) V = - 2T; E = -T (c)V=-T/2; E=T/2

(b)V =-T; E=O (d) V = -3T/ 2; E = - T/ 2

Prob 1.32. Consider a classical p article subj ected to an attractive inverse-squ are force field. The tota energy of the particle is E and t he eccentricity is

follow a p arabolic orbit if (A) E > 0 and E = 1 (B) E < 0 and (C) E = 0 and

E

=

1

The p article will

E .

(D) E < 0 and

[JAM 2019] E E

0 is a constant. If a system of such particles has reached virial equilibrium, the ratio of the kinetic to the total en er gy of the system is

(a) 1/ 2

(b) 1/3

(c) 3/ 4

[NET Dec 201 7] (d) 2/ 3

Prob 1.54. A p article of mass m is p laced in a potential well U(x)

= cxn ,

where c is a positive constant and n is

an even positive integer. If t he particle is in eq uilibrium at

11 oo : 01 (a) 1/ 2

(b) 1/ 3

: 1o (c) 3/ 4

(d) 2/ 3

Prob 1.54. A part icle of mass m is placed in a potential

well U(x ) = cxn, where c is a posit ive constant and n is an even positive integer. If t he part icle is in equilibrium at constant temperature, which one of the following relations between average kinetic energy (K ) and average potential energy (U) is correct? [JEST 2020]

(A) (K)

=

2 (U) n

(C) (K ) = ; (U)

(B) (K )

=

(U)

(D) (K ) = 2(U)

[email protected]

25

physicsguide CSIR NET ) GATE

Central force

@Sk J ahiruddin ) 2020

1.6

Ans Keys

1.1. A

1.12. a

1.23. d

1.2. a

1.13.

C

1.24. 4

1.14. d

1.25. d

1.15. d

1.26. b

1.4.

C

-8l 2 a 2

1.5. (a) - - r5 , m

(b) 0

1.16. (a) L 3

rt

1

= lO , 1.27.

C

E = soo· (b) 1.28. a L 2 = unchanged , 1

1

v15

E2 = , ,. .,. .

1.29.

C

11 oo : 01 1.4.

: 12

C

2 2

-8l a 5 1.5. (a) - - r , m

(b) 0

1.16.

1.7. a 1.8. b 1.9. a,c,d 1.10. a 1.11. d

1.27.

C

3

E = soo· (b) 1.28. a L 2 = unchanged , 1 400

1.29.

C

1.17. (a) 1 m

1.30.

C

1.18. d

1.31. a

E2 =

1.6. d

J5 (a) L = lO ,

1.19.

1.32.

C

C

1.20. B

1.33. b

1.21.

1.34.

C

1.35. 2

1.22. a 26

[email protected]

C

physicsguide CSIR NET, GATE

@Sk J ahiruddin , 2020

Central force

1.43. b

1.50.

1.37. d

1.44. 121

1.51. b

1.38. b

1.45. 85

1.52. C

1.39. d

1.46.

1.53. b

1.40.

C

1.47. d

1.41.

C

1.48.

1.36.

C

1 .42. d

C

C

1.49. b

C

1.54. C

11 oo : 01

: 1s

1.40.

C

1.47. d

1.41.

C

1.48.

1.42. d

j [email protected]

1.54. C

C

1.49. b

27

physicsguide CSIR NET, GATE

@Sk J ahiruddin , 2020

1.7

Central force

Hints

1.1. This is a standard basic conceptual question. Try to

prove t hat the motion is indeed restrict ed in a plane for central force. 1.2. You should be able to do it, since the trajectory is very

familiar. 1.3. Write the energy equation replacing

r by

1~-

11 oo :01

:1

a

1.2. You should be able to do it, since the trajectory is very

familiar. 1.3. Write the energy equation replacing

r by j; .

1.4. (You are given r, put it into the equation of motion to

get J(r )) 1.5. You don't need hint for this problem. 1.6. Use the concept of problem ??. Although be careful in

using the given equation as it is not correct!! 1. 7. Almost a same as the previous one.

1.8. The locus would be lissajous figure. 1.9. Same problem as the previous one. 1.10. You should be able to do it.

[email protected]

28

physicsguide CSIR NET, GATE

@Sk J ahiruddin, 2020

1.11. Almost a same as the previous one. 1.12. You don't need hint for t his problem. 1.13. Conservation of angular momentum.

1.14. We have done this kind of problem before.

Central force

11 oo : 01

: 21

1.13. Conservation of angular momentum. 1.14. We have done this kind of problem before. 1.15. Very easy problem. 1.16. Use conservation of energy and angular moment um. 1.17. Write t he equation of centripetal force and find out r

from t here. 1.18. Write t he equation of centripetal force and find out l

from t here. 1.19. Find out w from cent ripetal force. 1.20. Find out radius from cent ripetal force. 1.21. 1.22. 1.23. Find if r 0 is stable equilibrium. [email protected]

29

physicsguide CSIR NET, GATE

@Sk J ahiruddin , 2020

1.24. 1.25. You don 't need hint for t his one. 1.26.

Central force

11 oo : 01

: 23

1.25. You don 't need hint for t his one. 1.26. 1.27. Repulsive potential do not attract. 1.28. Use conservation of energy 1.29. 1.30. Use Kepler's law. 1.31. Use virial t heorem. 1.32. What is t he eccentricity and energy for a parabolic

orbit? 1.33. Use Kepler's law. 1.34. You don't need hint for t his one. 1.35. Use conservation of energy and angular moment um. 1.36. You don't need hint for t his one.

j [email protected]

@Sk J ahiruddin , 2020

30

physicsguide CSIR NET 1 GATE

Central force

1.37. Use conservation of energy and angular momentum . 1.38. Use conservation of energy and angular momentum .

11 oo : 01 : 2s 1.37. Use conservation of energy and angular momentum. 1.38. Use conservation of energy and angular momentum. 1.39. Use conservation of energy and angular moment um. 1.40. You don't need hint for t his one. 1.41. Use Kepler's law. 1.42. You don't need hint for t his one. 1.43. Find out radius from centripetal force. 1.44. You don't need hint for this one. 1.45. gravitational force on t he man will be due to t he imaginary sphere of radius r . 1.46. What is gravitational self energy? The energy the attractive system t ake to assemble itself .. .. ! 1.47. Use the result of Sol 1.45 .. 1.48. Use conservation of energy 1.49. [email protected]

@Sk J ahiruddin , 2020

31

physicsguide CSIR NET, GATE

Central force

1.50. See when condition of stable equilibrium are met.

00 : 01 : 29

Central force

@Sk J ahiruddin, 2020

1.50. See when condition of stable equilibrium are met. 1.51. 1.52. Use Kepler's law. 1.53. Use virial t heorem. 1.54. Use virial t heorem.

j [email protected]

@Sk J ahiruddin , 2020

32

physicsguide CSIR NET, GATE

Central force

11 oo : 01

: 31

[email protected]

32

physicsguide CSIR NET, GATE

Central force

@Sk J ahiruddin, 2020

1.8

Solutions .....

Sol 1.1. The form of a central force is F Hence the equation of the motion

diJ

= f (r )f.

...

m-=F dt

!( ) rxmdt=rx rr ...

diJ

...

A

(multiplying bot h side by rx)

d (..... .....) 0. . . dt r x mv = ..... .r. . x mv. . . = L .....

where L is angular moment um vector and is a constant. Multiplying both side by f .,

r

So we see that is always perpendicular to contant angular ..... moment um vector L , so the motion takes place in a plane because angular momentum is conserved/ constant. Sol 1.2.

Hence

mi= m [email protected]

(xx + yy) = m (-wfxx + -w~yy) 33

physicsguide CSIR NET, GATE

11 oo : 01

: 33

j [email protected]

33

physicsguide CSIR NET 1 GATE

@Sk J ahiruddin , 2020

Central force

So we can see t hat only if w1 was equal to w2 we could write it as F = = -wf i.e in the form of a central force. -+

••

mr

r,

Sol 1.3. The total energy eqation,

1 . 2 2 2 -m r +r 0 + V(r) =E 2 For a central force the angular moment um is conserved,

Using chain rule

r

=

dr . d0 = d0 dt

0dr

d0



replacing 0 , we get, . dr L r = - • - -2 d0 mr

replacing

r in energy equation we get

L dr 2 2 mr 4 (d ) + r + V(r) = E 2 0

(you have to remember this eqt

We are given that E=O and r = r 0 exp( k0) . Hence

dr de = kro exp(k 0) = kr.

11 oo : 01

: 36

We are given that E=O and r = r 0 exp ( k0) . Hence

dr dB = kro exp( k0) = kr.

[email protected]

34

physicsguide CSIR NET, GATE

@Sk J ahiruddin , 2020

Central force

Replacing this to the energy equation

L 2 2 - -4 [k r 2mr

or, Sol 1 .4 . Use

V (r) = d2u d02

+r ] = 2

L 2mr 2

(k

2

-V(r)

+ 1).

m + u = - z2u2 f

There are lot's of examples in the cent ral force notes. Solve t hem first.

Sol 1 .5 . (a) From fig.

u

=

r = 2acos 0 1 1 - = -sec 0 r 2a

Hence,

du l - sec 0 tan 0 d0 2a d 2u l 2 3 or, dB 2 = a [sec 0 tan 0 + sec 0] 2

11 oo : 01 or,

: 39

d2u l 2 3 d02 = a [sec 0 tan 0 + sec 0] 2

35

j a hir@physicsguide. in

physicsguide CSIR NET I GATE

Centra l force

@Sk J a hiruddin , 2020

Replacing ~~~ and u in t he given orbit equation 1

2

2

2

a sec 0 [t an 0 + sec 0]

1

+ 2a sec 0 = -

1 2 2 a sec 0 [tan 0 + sec 0 + 1] 2

1

2

m

l2 u 2 f

= - l~ 2 f m

a sec 0 · 2 sec 0 = l 2u 2 f 2 (putting expression of u) gz2a2

f = - mr5 (b) r

f (r' )dr'

V(r) = 00

2 2

8l a m

1

r 00

r

,5 dr'

2l 2a 2 mr 4 Putt ing t he expression of r, and ~ in the energy equa-

11

00:01 :41 mr 4

Putting the expression of r, and ~ in the en ergy equa-

[email protected]

physicsguide CSIR NET ) GATE

36

Central force

@Sk J ahiruddin ) 2020

t ion,Sol 1.3.

L dr 2mr4 (d0)2 + r2 + V(r) = E L

sin 0) [ (2a 2mr 4

2

+ (2a cos 0)

2 2

2

2l a ] -

= E mr 4 2 2

L 2l a 2 - - 4(2a) - - 4 = E 2mr mr 2l 2a 2

2l 2a 2

--=E 4

mr 4

mr

:. E = 0 Sol 1.6. R educing the problem into equivalent one body problem , we can say t hat a massµ= 1; moving in a circular 2

orbit of radius 2R under the influence of forcce (~ ~ 2 • Hence to find the angular velocity we equate, 2

µw R = 1

w=-

2R

Gm2 (2R )2

Gm R

11 oo : 01

: 44 Gm2 µw R = (2R)2 2

1 w=2R

Gm R

The t ime period T

R Gm

21r

= = 41rr w

When the two masses stop orbiting and start to fall onto each other. the equation of motion gives us, [email protected]

physicsguide CSIR NET, GATE

37

@Sk J ahiruddin, 2020

Central force

dv Gm 2 µv dr = - r 2 dv 2Gm v - = - -2dr r V

r

v'dv' = -2Gm

2r

0

v2

l dr' r'2

1

1

-=2Gm - - 2 r 2R 2R -r 1 2 v = (2Gm) / Rr dr = (2Gm )1; 2 2R-r dt R r

t= Now replace r by x 2 ,

R

2Gm

0

r'l/2 ----:::========== dr'

2R

J2R - r'

11 oo : 01

: 46

t =

Now replace r by x

2

0

R 2Gm

r'l / 2

--;:========= dr' 2R

J2R - r'

,

dr = 2x dx



• •

But we will use the formula (which we will prove in t he next problem) ,2

d X ' = -a Sln . Ja - x'2 2 x

j [email protected]

X

1

~

physicsguide CSIR NET, GATE

38

@Sk J ahiruddin , 2020

Central force

o x2 R -----;:::===== dx = t 2Gm -/2R, J2R- x 2

R a . - Slll 2Gm 2

1

X

'1m,

,----- 0 -J2R -x 2 X

-

2

=t

v'2R R 2Gm =t T

t= 4✓2

Sol 1. 7. The equation of motion gives us dv GMm m v dr = - r2 dv GM v dr r2

11 oo : 01

: 49

dv GMm mvd = 2 r r dv GM v - = - -2dr r V

v'dv' = -Gm

/2 r 2R

0

v2

l dr'

r

1

1

- = Gm - - 2 r 2R v = (Gm) 1; 2

2R - r

R

r 2R-r r 0

t=

Gm

[email protected]

r'l/ 2 -----;::==== dr'

2R ✓2R-

39

r'

physicsguide CSIR NET, GATE

@Sk J ahiruddin , 2020

Central force

Now replace r by x

2

,

• • •

2

dr

= 2xdx

o x2 R ----;::::========== dx = t Gm -/2R, ✓2R-x 2

Let

= vl2R,sin0 :. dx = ✓2R, cos 0d0 x

Substit uting t=2

R Gm

2

2R sin 0\/2Rcos 0 dB

\/2Rcos0

11 oo : 01 : s1 Substituting

t = 2 = 2

2

R Gm

2R sin 0-/2R cos 0 dB -/2Rcos0

RR Gm

(1 - cos 20) d0

v'R

=2

R R Gm

=2

R3 Gm

R3

sin- 1

1r

4+

X

-/2R 1 2

1r

Gm 2+l

Sol 1.8. The coordinate of the particle at any time t can 40

[email protected]

physicsguide CSIR NET, GATE

Central force

@Sk J ahiruddin , 2020

be written as

= a cos(wt) y = bsin(wt + c5)(where c5 is the phase different between

x

Therefore,

t = cos(wt) cos 8 y

X

b

a

- = -cosc5 From there we get

sin(wt) sin 8 x2

1 -a2

x and 1,

11 oo : 01 : s4 b

-

y

X

b

a

x2

- = -cosb -

l -a2

From there we get

y - +a b X

-

2xy ab

COS ()

.

= Sln

2 ()

Which is equation of an ellipse centered at t he origin. Sol 1 .9 . (a)we prooved it in the previous problem. Only in .

t his case 8 =

7r

2

(b)

v=

-wA sin(wt)i + wB cos(wt)J

So speed of the particle is not constant. (c) Force

F = ma= -w

2

A cos(wt)i + B sin(wt)J

j [email protected]

41

physicsguide CSIR NET 1 GATE

@Sk J ahiruddin , 2020

Central force

So the force is toward the origin. (d)Angular moment um _, _, L =r x mv _,

= mw A cos(wt)i + B sin(wt)J x - w A sin(wt)i + wB cos(wt)J

= mw AB cos (wt)k + AB sin (wt)k 2

A

=mwABk

2

11 oo :01 : s1 - w A sin(wt)i + wB cos(wt)3

= m w AB cos2 (wt )k + AB sin 2 (wt )k A

=mwABk

Sol 1.10. Cent ripetal force acting on the first particle

Similarly for the second particle 2

4

L1 = mr2

[email protected]

physicsguide CSIR NET , GATE

42

Central force

@Sk J a hiruddin , 2020

Therefore L1

2

r1

L1

4

r2

22

=

r1 r2

V2 =

r1 r2

4

11 oo : 01 : sg r2 4

r1

22 =

r2

~=

r1 r2

Sol 1.11. If we take corn as t he origin, then mr 1f

+ 2mr2 f = 0

3m r1 + 2mr2 And we are given that r 1

-

r2

= r Then

r2

=

r1

=0

2r 3

r

= --

3 Since same force is acting on the part icle we can write

mvr

2mv~

43

j [email protected]

physicsguide CSIR NET I GATE

Central force

@Sk J ahiruddin , 2020

Sol 1.12. Equating the cent ripetal force 2

mw R w2

= 4kR 3 V"\

R2

11 oo : 02 : 02 Sol 1.12. Equating the centripetal force 2

mw R

w2

= 4kR V"\

3

R2

Sol 1.13. Conservation of angular momentum gives us

0 = constant k0 80 = constant r

2

08 d0 = constant 1 9 -0 t 9 0 -1i

dt

V"\

V"\

Sol 1.14. Writing the total energy equation

1 2

.2

mr

!mr 2

2

+ U(r ) = E = E - U(r)

Since t he K.E > 0, the particle is bounded by the condition E > U(r) . So, region I and III are forbidden for the particle.

44

[email protected]

physicsguide CSIR NET ) GATE

Central force

@Sk J ahiruddin ) 2020

Sol 1.15.

V (r) = -

r

.

()()

k - -dr r3

11 oo : 02 : 04 @Sk J ahiruddin , 2020

Central force

Sol 1.15. r

V (r) = -

00

k - - dr r3

k

2r 2 Sol 1.16. T he force acting on t he body

-F =-V-V 3 (- ; )

-exp = V---

5r2

-

1

1

5r

5

3r

= v e- 2

-

2 + -e2

1

3r -

V

3 e - ?f

2 e- ?f

10 r

5 r

r2

" - ---r " = -----r 2 3

(a) T he equation of motion 3r

or,

e- 2

?

[email protected]

45

@Sk J ahiruddin , 2020

=

32r

- 3r -

L- = m (3r + 4)

or,

putting r

2e _

3 -mw R = 10 r 2 5 r3 L2 3 e- ?f 2 e- ?f mr 2 10 r 2 + 5 r 3 2

2 and m

e

2

10

physicsguide CSIR NET, GATE

Central force

=

1 we get,

11 oo :02 : oa Central force

@Sk J ahiruddin , 2020

putting r

=

2 and m

=

1 we get, -3

£ 2 = (6 + 4) elO

L=

or,

l

Jw

J5

or,

10

The total energy equation

E=

1

£ 2

2 mr 2 1

-

.

+ V (r) e- 3

1

2 20 · 4 1 1

160

5·4

400

3 800

(b )Angular momentum of the body which falls toward the center , -+

-+

L 1 = r 1 x mv1

= r1f1 =0

x

mv(-r1)

Using conservation of angular momentum before and afj [email protected]

@Sk J ahiruddin, 2020

46

physicsguide CSIR NET, GATE

Central force

11 oo : 02 : 1o 46

j [email protected]

physicsguide CSIR NET I GATE

Central force

@Sk J ahiruddin , 2020

ter parting away

~ lO = L1

+ L2

~ - L2 10 -

Let T1 and E 2 be the K.E of the 1st and t otal energy of the 2nd body respectively. Then conservation of energy gives us •

E = T 1 + E2 +

1 V (r) 2

3 1 2 800 = 2 . .5 x . l + E 2 1 400 = E 2

e- 2 -4-0

-

Sol 1.17. (a) The cent ripetal force on t he part icle mv 2 r

=

3 r2

£2

+

1 r

- = 3r + r m 2 r + 3r - 4 = 0

or, or,

2

r = l

or, •

(b) For a circular iJ = r0. So, t he t angential equation of mot ion r0 + 2r0 = Ar 0 ••

[email protected]



47



physicsguide CSIR NET, GATE

11 oo : 02 : 12 motion ••

r0





+ 2r0 = >..r0

jahir@physicsguide. in

47

physicsguide CSIR NET ) GATE

Central force

@Sk J ahiruddin ) 2020

multiplying both sides by mr

m(r

2

or,

0 + 2rr0) = >..mr

0 d . . 2 2 dt (mr 0) = >..mr 0 dL = >..L dt dL = >..dt L L ln = >..t Lo

or, or, or, or, or, Sol 1.18. Force

F

= _

av

ar

= -~

r

Then from equation of motion

L2

k

mr3 r~ ~l = mkro Sol 1.19. Force F = _ 8V = _ nk

8r

2

rn+l

11 oo : 02 : 1 s Sol 1.19. Force

F

[email protected]

= _

8V 8r

nk rn+I

= _

48

physicsguide CSIR NET, GATE

@Sk J ahiruddin, 2020

Central force

Then from equation of motion

1

2

nk m

1 R ~+I 1 2

R~+l

Similarly -1 2

t herefore

Sol 1.20. Equation of motion

(2R) ;+1

11 oo :02 : 1a

49

j [email protected]

physicsguide CSIR NET, GATE

@Sk J ahiruddin , 2020

Central force

L2

av

mrg L2

ar

3

mr0

L2

-

mr03

e - oro

= - k a--+ r 0

=k

e-oro 2

r0

L2

-- = mr0

e - or0

ke - oro

2

r0

(1 + ar0 ) (1 + aro)

Sol 1.21. This is a inverse square force where the effective potential looks like t he the figure Sol 1.21. below.

00: 02: 20

[email protected]

physicsguide CSIR NET, GATE

50

@Sk J ahirudd in, 2020

Central force

I

' ,2

'

.., \ 2mr" \ \

V'

',

\

V'

,- -

--

\.. /~

I

/

I _

k

I V- - , I

I

Figure 1.1: Effective potential for inverse square force

If we ignore the effect of atmosphere and the energy loss due to friction, as t he initial energy is positive for the approaching meteor there is no capture like the previous problem ( ~ potential ) . But still the meteor can hit the earth,

11 oo : 02 : 23 If we ignore t he effect of atmosphere and t he energy loss due to friction, as t he initial energy is positive for the approaching met eor t here is no capt ure like t he previous problem ( -fs potential ) . But still the meteor can hit the earth, rather touch t he earth in a hyperbolic orbit . When it is at a large distance, it has velocity v00 and impact paramet er b. So if we take the mass of t he met eor 1 as m , t hen initial energy to be mv~ . 2 The angular momentum L is mbv00 which remains [email protected]

51

@Sk J ahiruddin, 2020

physicsguide CSIR NET, GATE

Central force

stant in due course of motion. The distance b between the initial t rajectory and line aa' is called t he impact paramet er of the t rajectory. The largest value of b for which t he trajectory hits the planet is indicated by b' in the figure.

b' --

a

----

a

Figure 1.2: capture cross section for inverse square force

So if t he particle touches the earth (in case of b' impact oaramet er ) t hen t he kinetic enere:v when touched is ZERO.

11 oo : 02 : 2s Figure 1.2: capture cross section for inverse square force

So if t he particle touches t he earth (in case of b' impact parameter ) t hen t he kinetic energy when touched is ZERO . So the energy when t he particle touches the eart h is

GmM Re

+

L2 2mR~

This equals to the total energy when the particle was at long distance.

j [email protected]

52

physicsguide CSIR NET 1 GATE

@Sk J ahiruddin, 2020

Central force

Substitute L = mv00 b and get 1

b'

2GM

= R e (1 + R 2 voo

)

2

Now use escape velocity to be ~w you get

For the particle to be captured this b' is the maximum value of b. Hence

11 oo : 02 : 2a of b. Hence

'Veff

r)

-----------£ - - -£ -+---+-----+--►

[email protected]

r

physicsguide CSIR NET, GATE

53

Central force

@Sk J ahiruddin , 2020

Sol 1.22. See figure above. The effective potential is

z2

Ve11(r) = 2 2 mr

k 3r 3

Take derivative make it to zero. Check double derivative is negative. v max e ff -

z6

6m3k2

(b) If the energy E of t he particle is less than ½,jJx, then t he particle will reach a minimum value of r , and t hen head back out to infinity (see fig). If E is greater than ½,jJx, t hen the particle will head all t he way in to r = 0, and will never return. The condition for capture is t herefore Veffx < r

I



1

l

1 •

I

r

11 oo : 02 : 30 t he part icle will reach a minimum value of r , and t hen head back out to infinity (see fig). If E is greater than ½7Jx, t hen the particle will head all t he way in to r = 0, and will never return. The condition for capt ure is t herefore Veffx < E. Now for a particle at very large distance from cent er of attraction l = mub and E = becomes

½mu . 2

So the condition

Extra: The cross section for capt ure is

(J"

= 1rb~ax = 1r

j [email protected]

54

physicsguide CSIR NET I GATE

Central force

@Sk J ahiruddin , 2020

Sol 1.23. For stable orbit In this case

82V:

areft > 0.

82½11 - 3L2 12k r5 8r 2 mr6 0 3L l0 12kL 10 (3km) 4m (3km) 5 L lO which is less t han zero, i.e r 0 is unstable equilibrium point. So the particle will never move toward r 0 .

11 oo : 02 : 34 L IO

(8 1k 4 m 5

which is less t han zero, i.e r 0 is unstable equilibrium point. So the particle will never move toward r 0 . Sol 1.24. Sol 1.25. If N is the flux of the incoming particle then the total number of scattered part icle.

Ntotal =

Nda

N(a

2

+ b cos 2

2

0)dw

b2

=41rN a2+-

3

. da Sol 1. 26. The formula you know 1s for dD,, here you are dN . . da asked dB which is proportional to dB . Read the Rut herford [email protected]

55

@Sk J ahiruddin ) 2020

physicsguide CSIR NET ) GATE

Central force

scattering carefully once again. Sol 1.27. For a repulsive central potent ial closed orbit can

not form . So (b) and (d) are wrong. In case of (a) the part icle moved toward the origin, which will not happen for a repulsive cent ral potent ial. In case of (c) t he particle moved away from t he origin , which will happen for a repulsive cent ral potential.

11 oo : 02 : 3s t icle moved toward t he origin, which will not happen for a repulsive central potent ial. In case of (c) t he particle moved away from the origin, which will happen for a repulsive cent ral potential. Sol 1.28. The angular momentum of the proj ectile L mv0 b. For distance of closest approach r = 0. The central potent ial V (r) = ~

The conservation of energy gives us 1

2

2mvo =

L2 2md2

k

+d

0 = !mv2d2 - kdmv5b2 2 2 k b2 + k2 d =-+ 2 mv 0 mv5

°

Sol 1.29. Angular momentum remains same and energy is decreased. Obviously it will b e a ellipse.

[email protected]

physicsguide CSIR NET, GATE

56

@Sk J ahiruddin , 2020

Central force

Sol 1.30. From Kepler 's law 2

T ex R 2

3 3

T = kR T2 k = R3 If R becomes 4R, then

11 oo : 02 : 39 2

3

T = kR T2 k= -R 3 If R becomes 4R, t hen 2

T' = k(4R)

3

T ' = ST Sol 1.31. From virial theorem

n+l T = - -V. 2 In case of gravit at ional potent ial n = - 2. Then, -

- 1T = V 2 V = -2T -

-

And E = T

-

+V

=

-

-T

Sol 1.32. For attractive inverse square force field -~ ec-

centricity is given by (you have to remember it). E j [email protected]

1+ mK2 57

@Sk J ahiruddin , 2020

For a parabola

=

2EL2

E

physicsguide CSIR NET, GATE

Central force

= 1. Then t he energy E becomes O.

Sol 1.33. The major axis of t he second sat ellite orbit is SR

t hen semi major axis is 4R. Then t he problem becomes same as Sol 1.30. , and t he t ime period of the second orbit

11

00:02:41

For a parabola

E

= 1. Then t he energy E b ecomes O.

Sol 1.33. The m ajor axis of the second satellite orbit is 8R t hen semi major axis is 4R. Then the problem b ecomes sam e as Sol 1.30. , and t he t ime period of the second orbit is 8days. Sol 1.34. The gravitational force at height h

99h

2

GM

99 GM

(R + h) 2

100 R 2

+ 2 · 99hR -

R

2

h

=0 2

= - 2 · 99 + J (2 · 99) + 4 · 99 R 2 · 99 - 2 · 99 + 2 · 99

h=

h=

2 · 99 100 _

99

R 1

1 -1 h= (1 - 100 ) T

-

1 R

1 h ~ 200R

h = 32km [email protected]

@Sk J ahiruddin, 2020

58

physicsguide CSIR NET, GATE

Central force

Sol 1.35. Conservation of angular momentum gives us

11 oo : 02 : 44 Central force

@Sk J ahiruddin, 2020

Sol 1.35. Conservation of angular momentum gives us mvar a = mvprP rp ra Vp

ra rp

=8

ra

The cent ripetal force on t he eart h

T he conservation of energy gives us 1

2

Gm M s

-mv - - - 2 a ra

=

l

Gm Ms

2

-mv - - - 2 P rp

Sol 1.36. T he escape velocity on a planet of radius R and [email protected]

59

physicsguide CSIR NET, GATE

Central force

@Sk J ahiruddin , 2020

acceleration due to gravity _q is

Vescave

= ,J2iiR. Hence

11 oo : 02 : 46

@Sk J ahiruddin, 2020

Central force

acceleration due to gravity g is

V escape

= v12iR,. Hence

V escapeearth

9 earth R earth

V escapeniars

9 mars R mars

Sol 1.37. The aphelion distance of t he probe is RE and

t he perihelion distance is RM. The conservation of energy at t hese two distances gives us

£2

GmM8

2mR1 L2 1 2m R2E

RE

E =--

1 - 2-

RM L2

£2 2m Ri

= GmMs

1 RE

RM

2

= 2Gm M sRERM RM+ RE

j [email protected]

@Sk J ahiruddin, 2020

60

physicsguide CSIR NET, GATE

Central force

11 oo : 02 : 49 [email protected]

physicsguide CSIR NET, GATE

60

@Sk J a hiruddin , 2020

Central force

The conservation of energy at ant distance r gives us 2

2Gm MsRERM GmMs ------ - --2m(R M + RE) RE

=

1 2 GmMs - mv - --2 r 1 2 -v 2

2GMsRERJ\IJ GMs GMs ----- - -- + -- = 2(R1V1 + RE) RE r 2GMsRM + RE-r =v2 (RM+ RE)r

Sol 1.38. For attractive inverse square force field - ~ eccentricity is given by (you have to remember it). E=

2EL 2 1+ -mK2

Conservation of angular moment um reads

If v 0 is the initial velocity t hen

L 0 = mv0 R 0

jahir@physicsguide. in

61

physicsguide CSIR NET , GATE

11 oo : 02 : s1

61

[email protected]

physicsguide CSIR NET, GATE

@Sk J ahiruddin, 2020

Central force

Total initial energy

Gm M - --- = 0 Ro 2GM 2 or, v 0 = Ro

1 Ea = -mv02 2

So,

2 GM Ro Lo= m Ro = mJ2GMRo Now, L1 = mv1R 1 = m J2GMRo VJ=

1

2KRo

R1

m

Total final energy

1 GmM 2 E1 = -mv1 - - - - = 0 2 R1 For circular orbit

E

= 0. So, from the first equation mK 2 E1 = - 2L2

11 oo : 02 : s4 For circular orbit

= 0. So, from the first equation

E

E1 =

-

mK2 2£2

62

[email protected]

physicsguide CSIR NET, GATE

@Sk J ahiruddin , 2020

Central force

So, 1 2 Gm M - mv1 2 R1 2 2K K 2 V --f - m R _r £ 2

mK

2

2£2

Putting t he expression of v I and L as we have got previously 1 2KRo

R}

m

1 2KR0

R}

m 2R0 R}

2K K2 m R 1 2GMRo K 2 1 m R1 2Ro 2 1 R 1 2Ro

4R6 = 4R oR 1 - R 1 R 1 = 2Ro Sol 1.39. Let the equation of mot ion of the planet around t he st ar is 1

- = B + A cos0 r

where B =- GMm £ 2

11 oo : 02 : s6 1 r

= B + Acos0

where

B =-GMm

£2 B2 2mE + £2

A=

and, j [email protected]

63

physicsguide CSIR NET 1 GATE

@ Sk J ahiruddin, 2020

Cent ral force

We can write it as l

r=----

l l

wher e,

l= B A E=B

and, For a parabola

E

= 1. So, l r= - - - l - cos 0.

at pericenter distance 0

For a parabola

E

= 1r. Hence

= 0. So, re=

Therefore

l.

ECOS

0

L2 GMm

11 oo : 02 : sg re = l. Therefore

Sol 1.40. Weight GMm

mg= [email protected]

R2 physicsguide CSIR NET, GATE

64

Central force

@Sk J ahiruddin, 2020

if R



2R , t hen

l GMm ---➔---R2 4 R2 GMm

w

w

➔ 4

decreases by factor of 4.

Sol 1.41. From Kepler 's law Tmax - Tmin

E=-----

+

Tmax Tmin Tmax - Tmin

2a Tm ax - Tmin

2T ~ And,

+1

Tmax

1

Tmin

- -Tmin

= Tmax

E

E-

E+ l

E- 1 For B this equation reads

11 oo : 03 : 01 €

+1

Tmax

1

T min

€ -

c+ l r rrii n E- 1 For B t his equation reads E+ l E - 1R

= r m ax

= r B ma,x

And for A E- 1 1 - --

2

3

2TA E=TAmax

c+ l

2

Tjc(E + 1) = j [email protected]

TAmax

physicsguide CSIR NET I GATE

65

Central force

@Sk J ahiruddin, 2020

The m aximum possible distance between t he planets A and B

-2

Tj ( 2 TBmax 2TB

-

R)

+

R

l

E-

2

Tj (E + 1 R _ R ) + R 2 2T3 E - 1 E - 1 B

2

c+ l

E-1

T3

1 +1 T3 B

Sol 1.42. We are given QPn1111fnr

=

1 ~ O r 1nlP

00: 03: 04 E-1

1 + 1 T -3 B

Sol 1.42. We are given 9 eq·uator

or,

=

1 9pole 2

GMm

2 Vequator

GMm

R2

R2

2R2

GMm

2

or,

R 2

= Vequator

GMm - R

or,

2

-

2 v equator

[email protected]

66

physicsguide CSIR NET ) GATE

Central force

@Sk J ahiruddin ) 2020

Then 9 pole

=

GMm

2v;quator

R2

R

So, t he escape velocity at pole

4v;quator Vesc

=

2 Vequator

Sol 1.43. The frequency of the planet relative to earth year •

1S

0 w=-

t

4.2

X

lQ - 3

11 oo : 03 : 01 •

1S

0 w=-

t

4.2 X 10- 3 l year The centripet al force on 2 m ewe R e

GM m e

=

R2 e

R3 = GM w2

e

e

67

[email protected]

physicsguide CSIR NET, GAT E

@Sk J ahiruddin , 2020

Central force

The centripet al force on the planet

rnw2 R = G Mm R2 R3 = W e R3 w2

e

R = 121Re

Sol 1.44. Weight at equator ,

GMm

W equator = 2

R2

-

mw

2

R =

0

GM

w =--

R3

putt ing R ~ 6370 ~ 6400km and g

lOms- 2 , the time

11 oo : 03 : 09 2

W'equator= - R2- m w R = O GM 2 w =-R3 putting R ~ 6370 ~ 6400km and g period of the planet

21r

GM

T 21r T

R3

lOms- 2 , the time

g

-

R

T = 21r

R g

= 5026s ~

84min

Sol 1.45. If r is t he distance of the p erson at any time from t he center of earth. Then the gravitational force on t he man j [email protected]

@Sk J ahiruddin , 2020

68

physicsguide CSIR NET, GATE

Central force

will be due to the imaginary sphere of radius r. if p is the mass density of earth t hen t he mass of the sphere of radius r

or, or, Equation of motion

11 oo : 03 : 12 - - -1rr 11rR 33 3 r3 = MR3

or,

Equation of motion r3 m

mx = - GM R3 r 2 sin 0 .. X

=-

Hence t he frequency w =

GM R3 X

~If .and t he t ime period

T = 21r w

= 21r

R3 GM

Sol 1.46. Imagine a mass dm is brought in from infinity,

where its potent ial energy is zero, to t he surface of a part ially const ructed star, where its potent ial energy is - Gm (dm )/r. [email protected]

@Sk J ahiruddin, 2020

69

physicsguide CSIR NET, GATE

Central force

The star 's gravitational potential self energy is t he sum (integral) of all such mass t ransfers in building up t he star. T he total change in Uz is t he integral of dU8 over all shells dm : rnd m = - G m pdV Ua = - G r r R m (r) 2 =-G - - p(r) 41rr dr o r Here, p(r) is the density of t he sun at a dist ance r from its

11 oo : 03 : 1 s rnd m = - G mpdV Ua = - G r r R m(r) =-G --p(r)41rr 2 dr o r Here, p(r) is the density of t he sun at a distance r from its center , m(r) is the mass inside a sphere of radius r, 41rr 2 dr is the volume of the spherical shell of mass dm , and R is the radius of t he star. Doing the integral requires knowing the functions m (r) and p(r) . We can calculate m (r) if we know p(r) because r

m(r) =

2

dm =

41rr' p (r') dr' 0

If we imagine t hat the sun is layered like an onion, each 2

layer has a volume dV = 41rr' dr' and a mass dm = p (r') dV = 41rr' 2 p (r') dr' . The mass m( r) is t he sum of dm for all layers up to t he radius r . Now, do the rest of the integrals yourself.

[email protected]

70

physicsguide CSIR NET, GATE

@Sk J ahiruddin , 2020

Central force

Sol 1.47. From problem Sol 1.45. we get the t ime p eriod

T = 21r Now

R3 GM

11 oo : 03 : 11 T = 21r

GM

Now 4

M = 31rR3p

R

3

or,

3 41rp

M

So,

T = 21r

1 3 G41rp

31r pG

Sol 1.48. As t he energy is conserved

E =

~mv2-

Gm M

2 1 2GM =-m-2 R

R

GmM

R

=0 Therefore

E

= 1, hence t he orbit is a parabola.

Sol 1.49. Solution: We know K.E + P.E = E nergy, So . k we get from t he potential t hat K.E - - = E. Hence K.E =

r

k E + -. r j [email protected]

@Sk J ahiruddin, 2020

71

physicsguide CSIR NET 1 GATE

Central force

k Now when K.E will be m aximum? When - will be maxr imum, i.e when r will be minimum. Now we a11~eady know (you can just derive from the given solution of r( 0) ) t hat t he m aximum and minimum r for el-

11 oo : 03 : 20 Now when K.E will be m aximum? When k will be m axr imum, i.e when r will be minimum. Now we already know (you can just derive from the given solut ion of r( 0)) that the m aximum and minimum r for ellipt ic orbit is

Tmax =

a( l

Just put t he value of k

+ e)

and

r min ·

Tmin =

a(l - e) .

So we get

K.Emax

=

E

+

a( l - e)

k

Now as E =

we get - = - 2E. 2a a

So K.Emax

=

k

E -

2E -1- e

K .E max

=

e+ l

E --

e- l

Sol 1.50. If

V (r) = -k - /3

r3

r

t hen t he effective potent ial

k /3 Vef f = 2mr2 - r - r3

L2

[email protected]

72

physicsguide CSIR NET , GATE

@Sk J a hiruddin , 2020

Central force

hence, 8Veff

L

2 -i..

k

-i..

3/3 = n

11 oo : 03 : 22 Central force

@Sk J ahiruddin, 2020

hence, 2

8 Veff

_L_

ar

mr 3

kr

+ _k + _3/1 = O

£2

2

r2

r4

- r + 3/1 = 0

-

m

£2

-m +

4

L m2

- 4· k3ffJ-
·2

(d) m(~

·2

+ rJ)(~ + ~)

Prob 1.26. The parabolic coordinates (~, rJ) are related to t he Cartesian coordinates (x, y) by x = ~rJ and y = ~ ( ~ 2 rJ 2 ). The Lagrangian of a two-dimensional simple harmonic [NET oscillator of mass m and angular frequency w is

Dec 2016]

[email protected]

@Sk J ahiruddin, 2020

14

physicsguide CSIR NET, GATE

Lagrangian iVIechanics

11 oo : oo : 39 @Sk J a hiruddin , 2020

(a) (b) (c) (d)

Lagra ngia n Mechanics

tm[~2 + iJ2 - w2(~2 + 772)] ½m(~2 + 77 2) [~2 + 7]2 _ ¼w2(~2

+ 77 2)]

~m(~2 + 772)[~2 + 7]2 - ~w2~77] ½m(~2 + 772) [~2 + 7]2 - t w2]

Prob 1.27. The dynamics of a particle governed by t he La•

grang1an 1

·2

1

2

.

L = - mx - - kx - kxxt 2 2 [NET Dec 2016]

describes

(a) an undamped simple simple harmonic oscillator (b) a dumped harmonic oscillator with a time varying damping factor (c) an undamped harmonic oscillator with a t ime dependent frequency (d) a free part icle

Prob 1.28. A double pendulum consists of two equal masses m susp ended by two strings of length l. What is the Lagrangian of t his system for oscillations in plane? Assume t he angles 01 , 02 made by t he two strings are small (you can use cos 0 = 1 (given , wo

0 ;)

[JEST 2014]

= Jg/l)

[email protected]

@Sk J a hiruddin , 2020

15

physicsguide CSIR NET , GATE

Lag1·angia n l\/Iechanics

11 oo : oo : 42 [email protected]

15

physicsguide CSIR NET , GATE

Lagrangian Mecl1anics

@Sk J a hiruddin , 2020

(a) L

r--.J r--.J

m l2(02 + 1.02 _ 1w202) 1 22 _ w202 01 2 02

(b) L ~ ml

(0f + ~0~ + 0102 - w50i - ~w50~) 2 (c) L ~ m l (0i + ~ 0~ - 0102 - w50r - ~w50~) 2 (d) L ~ ml (0t + -;0~+ 0102 - w50r - w50~) 2

Prob 1.29. A classical particle with total energy E moves 2 3 under the influence of a potential V (x, y) = 3x + 2x y + 2 2xy + y 3 . The average potential energy, calculated over a long time is equal to [JEST 2015]

Prob 1.30. The Conservation Principles for energy, linear momentum and angular momentum arise from t he necessity t hat [TIFR 2014] (a) the laws of physics should not involve infinite quantit ies. (b) internal forces on a body should cancel out, by Newton's (third) law of action and reaction. (c) physical measurements should be independent of the origin and orientation of the coordinate system. (c) t he laws of physics should be independent of the state of rest or motion of the observer. Prob 1.31. If t he Lagrangian Lo j [email protected]

16

= ~m(!~)

2

-

2 2

~mw q is

physicsguide CSIR NET , GATE

11 oo : oo : 44 2 2 2 Prob 1.31. If t he Lagrangian Lo = tm(!j) - ~mw q is j [email protected]

@Sk J ahiruddin , 2020

16

physicsguide CSIR NET, GATE

Lagrangian JVIechanics

modified to L = Lo + aq( !i), which one of t he following is TRUE? [GATE 2017] (a) Both t he canonical momentum and equation of motion do not change (b) Canonical momentum changes, equation of motion does not change (c) Canonical momentum does not change, equation of motion changes (d) Both the canonical momentum and equation of motion change Prob 1.32. A rob of mass m and length l is suspended from two massless vertical springs with a spring constants k1 and K 2 . What is the Lagrangian for the system, if x 1 and x 2 be t he displacements from equilibrium posit ion of the two ends of the rod? [JEST 2017]

(a) ?f (xi + 2x1x2 + X§) - ½k1Xi - ½k2X§ (b) ~(xi + x1x2 + x~) - ! (k1 + k2)(xr + x~) (c) W (xr + X1X2 + X§) - ½k1Xi - ½k2X§ (d) 7(xi - 2x1x2 + x~) - !(k1 - k2)(xi + x~) Prob 1.33. A possible Lagrangian for a free particle is [JEST 2017] 2 2 2 (a) L = q - q (b) L = q - qq

11 oo:oo:47 Prob 1.33. A possible Lagrangian for a free part icle is [JEST 2017] (a) L = q2 - q2 (b) L = q2 - qq [email protected]

17

@Sk J ahirudd in, 2020

2

(c) L = q

-

q

physicsguide CSIR NET, GATE

Lagrangian iVIechanics

(d) L = q2 - .!q

Prob 1.34. A particle of mass m moving in one dimension x is subject ed to t he Lagrangian L = 12 m(x - Ax) 2 where A is a real constant. If it starts at the origin at t = 0, its motion corresponding to t he equation (a is constant) [TIFR 2018] (a) x = a exp At (b) x a sin At (c) x (d) x

a( l - exp(- At)) a sin hAt

Prob 1.35. A block of mass M is moving on a friction-less inclined surface of a wedge of mass m under the influence of gravity. The wedge is lying on a rigid frictionless horizontal surface. The configuration can be described using t he radius vectors ri and r2 shown in the figure. How may [JEST constrains are present and what are t he typ es? 2018]

11 oo : oo : so t he radius vectors ri and r2 shown in t he figure. How may [JEST constrains are present and what are the types? 2018]

[email protected]

18

physicsguide CSIR NET, GATE

Lag1·angian l\/Iechanics

@Sk Jahiruddin, 2020

(a) One constrain: holonomic and scleronomous (b) Two constrains: Bot h are holonomic; one is scleronomous and rheonomous (c) Two constrains: Bot h are scleronomous: one is holonomic and t he other is non-holonomic (d) T wo constrains: Both are holomic and scleronomous Prob 1 .36 . Consider t he Lagrangian

L=

~. q2 l - ✓1 - q2 - 2

of a particle executing oscillations whose amplitude is A. If p denotes t he momentum of the particle, t hen 4p2 is [JEST ?01

~l

11 oo : oo : s2 L = l - ✓1 - q2

2 -

q

2

of a particle executing oscillations whose amplitude is A. If p denotes the momentum of the particle, then 4p 2 is [JEST

2018]

(a) (A2 - q2)(4 + A2 - q2) (c) (A2 - q2)(4 + A2 + q2 ) j ahir@physicsguide. in

(b) (A 2 + q2 ) (4 + A 2 - q2 ) (d) (A2 + q2)(4 + A2 + q2) 19

physicsguide CSIR NET 1 GATE

@Sk J ahiruddin , 2020

Lag1·angian Mechanics

Prob 1.37. The motion of a p article in one dimension is 2 2 described by the Lagrangian L = ½ ( in suitable - x

¥t)

units. The value of the action along the classical path from

x = 0 at t = 0 to x = x 0 at t = t 0 , is

[NET Dec 2018] 2

Xo

2 cos 2 to

Prob 1.38. A p endulum is created by hanging a heavy bob of mass m from a rigid support (taken as zero level of potent ial! symmetrically using two massless inextensible strings, each of length 1!, making an equilateral triangle as shown in the figure below.

[TIFR 2019]

11 oo : oo : s6

A correct Lagrangian for the angular oscillations of the bob in t he plane perpendicular to t he paper is (a) L = ~mf

0

2 2

2 2

(b) L = v0,mR 0

-

v0,mgRsin ~ 2

+ 4mgf sin

[email protected]

2

~

20

physicsguide CSIR NET, GATE

@Sk J ahiruddin , 2020

2

(c) L = !m0 (d) L = ~mf

-

Lagrangian Nlechanics

)3~- sin ~ 9

2

0 + mgf 0 sec 0 + v0,mgRsin

2 2

2

2

~

Prob 1.39. Which of the following terms, when added to t he Lagrangian L(x, y, x, y) of a system with two degrees of

freedom , will not change the equations of motion? Dec 2019] (A) XX - yy (B) xy - yx (C) xy - yx (D) yx 2 + xy 2

[NET

Technically t he qs is wrong. If you use gauge invariance of Lagrangian, L' = L + A C,

a\

11 oo : 02 : 1o

Sol 1.38. The radius of oscillation

.J,f'z. Lagrangian

1 ·2 v'3 L = I 0 - mg l ( 1 - cos 0) 2 2 1 32 2 . 2 2 0 = -m-l 0 - v'3mgl sin 2 4 2

= [email protected]

@Sk J ahirudd in, 2020

3

2 ·2

- ml 0 8

-

V

47

~

3mgl sin

2

0

-

2

physicsguide CSIR NET, GATE

Lagrangian iVIechanics

Sol 1.39. Technically the qs is wrong. If you use gauge invariance of Lagrangian , L' = L + df:, you get option B , but L can't b e function of acceleration.

00 : 02 : 13 3mgl sin j [email protected]

47

@Sk J ahiruddin , 2020

2

physicsguide CSIR NET, GATE

Lagrangian Mecl1anics

Sol 1.39. Technically the qs is wrong. If you use gauge invariance of Lagrangian, L' = L + O

= oo

for x < 0

Which of the following schematic diagrams best represents t he phase space plot of the ball? [GATE 2019]

[email protected]

26

Hamiltonian Mechanics & Phase Space

@Sk J ahiruddin , 2020

+.J21nE

physicsguide CSIR NET, GATE

(a)

+.J2mE r---_ E rng

r--------it--"-- X

- ../2mE

- ../2mE ~

00 : 01 : 13 (a) E

E mg

mg

r-----~- x

X

-✓2mE

- ✓2111£ ~

(c)

+,/2m£

(d) E mg

!-----3==---_:::.._- x

E

E mg

mg

-+----1-----+-...;a..._- x

- ,/2m£

- ,/2m£

j [email protected]

27

@ Sk J ahiruddin, 2020

1.3 1.1. b

physicsguide CSIR NET, GATE

Ham iltonian :Nlechanics & Phase Space

Ans Keys 1.12. a

1.23. a

1.34.

C

11 oo : 01

: 1s Hamiltonian :Nlechanics & Phase Space

@Sk J ahiruddin, 2020

1.3

Ans Keys

1.1. b

1.12. a

1.23. a

1.34.

1.2. b

1.13. d

1.24. a

1.35. d

1.3. b

1.14. b

1.25.

C

1.36. a

1.4. b

1.15. a

1.26. b

1.37. d

1.5. a

1.16.

1.27.

C

1.38. b

1.6. b

1.17. a

1.28. a

1.39. a

1. 7.

C

1.18. a

1.29. a

1.40.

1.8. a

1.19. a

1.30. a

1.41. b

1.9. d

1.20. b

1.31.

C

1.42. a

1.10. a

1.21. a

1.32. a

1.43. b

1.11. d

1.22. b

1.33. b

[email protected]

@Sk J ahiruddin, 2020

1.4

Solutions

C

28

C

C

physicsguide CSIR NET, GAT E

Hamiltonian :Nlechanics & Phase Space

11 oo :01

:1

a Hamiltonian Mechanics & P hase Space

@Sk J ahiruddin, 2020

1.4

Solutions

Sol 1.1. The electric field .....

E =-V cp- 8A 8t and

.....

.....

.....

B =V x A The Lorentz force

can be derived from a velocity dependent potential .....

U = qcp - qA · v. So the Lagrangian

L=

1 2 .......... -mv - qcp + qA · v . 2

conjugate momentum

. . . 8L ... p = oil = miJ + qA .....

=p+qA

j [email protected]

@Sk J ahiruddin , 2020

29

physicsguide CSIR NET, GATE

Hamiltonian Mechanics & Phase Space

11 oo : 01

: 21

[email protected]

29

physicsguide CSIR NET, GATE

Hamiltonian Mechanics & Phase Space

@Sk J ahiruddin , 2020

Therefore the Hamiltonian _,

H = P· v- L _,

_,

P -.... l _, -- 2 A .... -= - · (P - qA) - - (P - qA) + q..qq .

q=

p m-

>..q

The Hamiltonian

H =pq-L p2 1 p2 ).. p2 =-----m----+ -q--2 m - >..q 2 (m - >..q) 2 m - >..q p2 2(m - >..q) Sol 1.15. Volume of the phase space I'

11 oo : 01

: 39

q

mp2

2(m - >..q)

Sol 1.15. Volume of the phase space

V

If E 0 is the m aximum available energy t hen p

= ✓2m(Eo - a q4 ).

Hence

✓2m(E0

V =

aq4 )dq

-

V

aq4 l - 2mE )dq

2mEo

0

V

[email protected]

36

physicsguide CSIR NET, GATE

Hamiltonian Mechanics & P hase Space

@Sk J ahiruddin , 2020

1

If we take

a 2mEo

4

q

= t , then --1 4

dq =

dt

2mEo

Hence t he volume of the phase space

V

1

1

= (2mE0 ) 2 (2mE 0 ) 4

V

Sol 1.16. T he conjugate moment um Px

= mx+miJ

11 oo : 01

: 42

Sol 1.16. The conjugate momentum Px

= mx+miJ •

Py =mx

The Hamiltonian

H = PxX + PyY -

=

Py Pxm

Py ( ) + x- Px - Py m

p2 -m Y2 2 m 1

p

+ mY (Px -

P) Y

2

PxPy l P y ----

2m

m

[email protected]

physicsguide CSIR NET, GATE

37

Hamiltonian Mechanics & Phase Space

@Sk J ahiruddin, 2020

Sol 1.17. From Hamilton 's equation

.q =oH op

.p= - -oH oq

CJ. = 2pq2

= -2p2q

Using separation of variables •

~=-P=k or,

q

q





p

q

- = -- =

k

(say)

11 oo : 01

: 44

Using separation of variables •

p -



q p p

--

p

- -

q q



or,

-k (say)



--

- - -k -

q

Solving p

= Poekt

and q = qoe-kt_ But

p = -2p5e2ktqoe-kt Pokekt = - 2p5e2kt qoe-kt Pokekt = - 2p5qoekt equating the coefficient

k Po = 2qo j [email protected]

Hamiltonian :Nlechanics & Phase Space

@Sk J ahiruddin , 2020

If we take -

k 2-qo

physicsguide CSIR NET, GATE

38

=Band

_k 2 A

= A, then

p = B e-2At and

A q = -e2At _ B

Sol 1.18. The conjugate momentum 'Pl

= 2d,

11 oo : 01

: 47 =

q

A

-e2At _

B

Sol 1.18. The conjugate momentum

= 2q1 P2 = 2q2

Pl

The Hamiltonian H

= P1 ri1 - P2ri2 - L

PI

p~ 2

2

PI

p~ 4

4

PI +P2 4

Sol 1.19. The Lagrangian

L = >..q1 ri2 The conjugate momentum Pl P2 [email protected]

= Aq2 = >..q1 physicsguide CSIR NET, GATE

39

@Sk J ahiruddin, 2020

Hamiltonian :Nlechanics & P hase Space

The Hamiltonian H

= P1ri1 = Pl ri1

- P2ri2 - L

- P2ri2 - Aq1 ri2 2P1P2 P1P2 )..

P1P2 )..

)..

11 oo : 01

: 49

= Pl ci1 - P2 ci2 2P1P2

P1P2

;\

;\

Aq1 ci2

P1P2

A Sol 1.20. The K.E 1 2 T = -mv 2 1 • 2 = m rf + rsin0 + r00 2 1 2 2 2 2 2 2 = -m r + r sin 0~ + r 0 2 The potential energy A



A

k V (r) = -r

The Lagrangian

L = T-V =

~m

r +r 2

2 The conjugate moment um Pr = mr

r

= Pr

m j [email protected]

@Sk J ahiruddin , 2020

2

2

sin 0~

2 .

P0 = mr 0

B=

2

P0

mr

2

40

+r 0

2 2

P

k +-r 2

2

= mr sin 0~ .

P

= - 2 mr2 sin

0

physicsguide CSIR NET I GATE

Hamiltonian Mechanics & P hase Space

The Hamiltonian

H = T+V 1

= -m 2

k r

11 oo : 01 : s2 The Hamiltonian

H = T+V k r

1 =-m 2

P~

P~

P~

k 2m + -2_ mr_2 + -2_ mr_ 2 _0 - r 2 -si_n_

Sol 1.21. We see that sin2 q

8L

p

= 8q

2

. pq



'

2

. sin q q 2

=

hence we see

H =pq -L = O Now the question is can t he Hamiltonian of a physical system be ZERO? Ans is NO! To explore more let's t ry to calculate the Lagrangian equation of motion of t he given Lagrangian. 2

8L

p

d dt

= 8q =

8L 8q

=



2·q .

8L

sin q 2 ;

2

8q



q Sln q COS q;

Sln

q COS q

d 8L _ 8L = O dt

8q

8q

So we did not get a valid Lagrangian equation of motion of the particle. [email protected]

@Sk J ahiruddin, 2020

41

physicsguide CSIR NET, GATE

Hamiltonian Mechanics & Phase Space

Hence we conclude that the Hamiltonian is NOT DEFINED . And we see that t here may not always b e a valid

11 oo : 01

: 55 Hamiltonian :Nlechanics & Phase Space

@Sk J ahiruddin, 2020

Hence we conclude that the Hamiltonian is NOT DEFINED . And we see that t here may not always be a valid Hamiltonian for a given Lagrangian. Sol 1.22. If t he Hamiltonian is independent of t ime

Thus H is a constant. By Euler 's theorem on homogeneous functions the K.E

If the potential V does not contain velocity dependent term, d we h ave .. , t hen Pi = 8f}LQi.. = 8f}T an Qi H

=

L

PiQi - L

=

2T - (T - V ) = T

+V

=

E

Sol 1.23. From Sol 1.9. we se that under Gauge transfor-

mation the Lagrangian changes by a total derivative.

[email protected]

@Sk J ahiruddin, 2020

42

physicsguide CSIR NET, GATE

Hamiltonian Mechanics & Phase Space

11 oo :01 : s1

Hamiltonian :Nlechanics & Phase Space

@Sk J a hiruddin , 2020

So the action which is defined as S =

S

➔ S' = S' = S'

=

S' =

J L dt changes as

(L + !~)dt Ldt + S+

!;dt

dx

s + X (t) I~~

The last t erm is a constant and a surface element term i.e depends only on t he end points. Sol 1.24. Let

x+y

X+=--

\1'2

x-y

and,

X-

= y'2

Therefore

+x_ x=--X+

v'2 X+ - X_ y = v'2

and,

j a [email protected]

(c)Sk J a,h ir11ddin. 2020

43

physicsguide CSIR NET, GATE

Ha.miltonian Nlechanics & Phase Sna.ce

11 oo : 02 : oo 43

j ahir@physicsguide. in

physicsguide CSIR NET I GATE

Hamilt onian Mechanics & P hase Space

@Sk J ahiruddin , 2020

t hen x2

+

(x+

= =

+ y2

xy

+ x_)

2

+

2

3

2

+

x_)

(x+ -

2

1

+ -x2 + 2 -

-x2

2

xt - x~

2

Hence 1 2

\x

+ xy + y

J e- f3H (x 2 + x y + y 2 )dpxdpydx dy J e- f3HdPxdPyd dY 3 x 2 + lx 2 )dx dx ef3H ( J 2 + 2 + -

2) _

-

X

Sol 1.25. We write the equations in matrix form

d dt

q

0

p

-3 - 4

1

q p

Eigenvalues of the matrix are -3, - 1 Eigenvectors (1, -3) and (1, -1) for the eigenvalues -3 and - 1 respectively. General solution will be

q(t ) p(t )

1

=a

e- 3t

-3

+ /3

1 - 1

e

-t

or q [email protected]

=

ae- 3t 44

+ f3e - t physicsguide CSIR NET, GATE

11 oo : 02 : 03 or

q = ae-3t

+ f3e - t

44

[email protected]

physicsguide CSIR NET, GATE

Hamiltonian Mechanics & P hase Space

@Sk J ahiruddin , 2020

and

p = -3ae- 3t - f3e - t a and

/3

can be found :&·om the initial conditions which has not been given. Area is

dA = dqdp dq = -3ae- 3tdt - f3e - tdt dp = 9ae- 3t dt

+ f3e - tdt

dA = dqdp

= (-3ae- 3t - f3 e- t) (9ae- 3t + f3e - t) dtdt Till now is OK. Without t he initial conditions we can 't proceed further in t his way. Fortunat ely there is another way in which we do not solve p q explicitly, rather we go for the increment.

oq = O·

oq

,

Elementary are in phase space

dA = dqdp They are all funct ions of time. dq = dq(t). Now suppose at t ime t = 0 dq = dq(O ). What will be dq at time dt ? The

11 oo : 02 : os dA = dqdp They are all funct ions of t ime. dq = dq(t ). Now suppose at time t = 0 dq = dq(O ). What will be dq at t ime dt ? The [email protected]

physicsguide CSIR NET, GATE

45

Hamiltonian Mechanics & P hase Space

@Sk J ahiruddin , 2020

increment in differential elements will be dq'(dt ) dp' (dt)

= dq(O) + = dp(O) +



a! dqdt a·

a:dpdt

dA = dqdp dA' (dt )

= (dq (O) +



a! dqdt )

X

(dp(O)

8q

+ 8p



a: dpdt)

= dq (O)dp (O) + Bq dqdtdp + Bp dpdtdq Where we ignored the second order term. We get

dA' = dA

+

a· a· _q + p 8q

8p

dA · dt

Using the results

dA' = dA - 4dtdA

= dA (l - 4dt) The increment in time is dt . Total t ime is t. Say we have N number of increments. So we can write t = N dt or

11 oo :02 : oa dA = dA - 4dtdA

= dA (l - 4dt) The increment in time is dt . Total time is t. Say we have N number of increments. So we can write t = N dt or dt = t / N or

46

j [email protected]

physicsguide CSIR NET, GATE

Hamiltonian :Nlechanics & P hase Space

@Sk J ahiruddin , 2020

This is for one increment from O t o dt . For t he next increment it will be

t l- d N

dA'' = dA'

2

= dA 1 - d t

N

and so on .... For N increment

t

N

l -dN At limit N ➔ oo t he limit in t he RHS becomes e- 4t. Hence

A(t) = A(O)e-4t Sol 1.26. We are givenF(x, y) = x 2 + y 2

fJF

U=-

OV

= 2x+y

and and

+ xy .

Then

fJF v=fJy

=

2y+x

11 oo : 02 : 1o o 1.26. We are givenF x, y

oF u=ov = 2x+y

and

oF v=-

and

= 2y + x

oy

From there we get x = ½(2u - v)and y = -½(u - 2v) .

47

[email protected]

physicsguide CSIR NET, GATE

Hamiltonian Mechanics & Phase Space

@Sk J ahiruddin, 2020

From Legendre's t ransformations

G (u, v) = F (x , y) - ux - vy

=

1

3 2 (2u

l

-

32

7

- v )2

( 2u

1

+ 32 ( u -

2v )2

- v) (u - 2v) -

2

=- (u -uv+v 9

2

u

( 2u

3

- v) +

)

Sol 1.27. From Hamilt on's equation

.

8H X=Op m . p = -x X

The Lagrangian

L = px- H m ,:,. ;.,.

( x ( m ;.,. \

2

,

1 ,A""\

V 3

(u

- 2v)

11 oo : 02 : 13 The Lagrangian L = px- H m



-xxx 2 mx -- -

m.

X



2

-X

2m

X

lk +-2 X

1 -kx

2x

2

48

[email protected]

physicsguide CSIR NET, GATE

Hamiltonian Mechanics & Phase Space

@Sk J ahiruddin , 2020

Hence

8L

ax

d dt

mx

8L



X

ax

..

mx mx 2

fJL

ax

=

1 - -k -

2

mx2 2x 2

x2

X

Euler-Lagrange equation in this case

..

mx X

8L

d dt

mx

ax

2

1

8L

-

ax mx

= 0

2

+-k + = 0 2 x2 2 2x 2 2 2mxi - mx + kx = 0

Sol 1.28. The total time derivative of a dynamical system Q is defined as dQ _ rlf -

{ Q,

H

}+

8Q At

11 oo : 02 : 1 s Sol 1.28. The total time derivative of a dynamical system Q is defined as

dQ _

8Q

H

dt - {Q, } + 8t

So, in t his case dp + v'2x = {

0 H} p + V L-X,

dt

= {p , H} + v'2{ x, H} = _ 8p8H + v'28x8H 8p ax

ax 8p

= -x + v'2p 49

j [email protected]

physicsguide CSIR NET, GATE

Hamiltonian :Nlechanics & Phase Space

@Sk J ahiruddin , 2020

Sol 1.29. From Hamilton's equation .

8H 4 q= = aq p 8p •

q p= aq4

The Lagrangian

L =pq- H

q . aq4

q-



aq4

q

2

a q4

q2

/3

2aq4

q2

Sol 1.30. Since t he force is conservative, t he Hamiltonian

11 oo :02 : 1a •

-

q-

aq4

+ q2

2

q2

/3

2aq4

q2

Sol 1.30. Since t he force is conservative, the Hamiltonian

From Sol 1.20. for spherical coordinate 2

T = Pr

2m in t his case r = a, and The potential energy

V

+ r

2

p2

P0 + d

-

Q 2f 41rcor

--

= 0 : a d ~ 41rc r 2

r < a·'

: b < r < c;

- Q f 41rc 0 r 2 •

0

9

physicsguide CSIR NET, GATE

Electrostatics: Part-1

(d)E = 0 : r < a· '

E = 4ncor Q r 2

=0 : a d - 41rcor2 •

Prob 1.16. In order to have equal surface charge densit ies on t he outer surface of the bot h shells, the following conditions should be satisfied

(a) d = 4b and c = 2a (c) d = v"2,b and c > a

(b) d = 2b and c =-J2a (d) d > b and c = v"ia

00: 00: 26 t ions should b e satisfied (b) d = 2b and c =0a (d) d > b and c = v'2,a

(a) d = 4b and c = 2a (c) d = v'2,b and c > a

Prob 1.17. A thin hollow cylinder of radius and length

bot h equal L is closed at the bottom. A disc of radius L / 2 is removed from t he bottom as shown in t he figure. [JAM 2008]

p L

-------L

I I

- : L/2 'I

j [email protected]

I

L

i+I

10

physicsguide CSIR NET I GATE

@Sk J ahiruddin , 2020

Electrostatics: Part-1

This object carries a uniform surface chaarge density a. Calculat e the electrostatic potential at t he point P on the axis of the cylinder as shown in the figure. (use

dx

J ✓x2 + a2

= ln (x + ✓x2 + a2))

Pro b 1.18. Consider a charge Q at the origin of 3-dimensional coordinate syst em. The flux of the electric field through the curved surface of a cone that has a height h and a circular

11 oo : oo : 29

Prob 1.18. Consider a charge Q at the origin of 3-dimensional coordinate syst em. The flux of the electric field through the curved surface of a cone t hat has a height h and a circular

base of radius R (as shown in figure) is [NET Dec 2015]

h ---'Q

R

(b)

(c)

Q

2c 0

h,Q

Rea

E=

Prob 1.19. For an electric field k,Jxx where k is a non-zero constant, total charge enclosed by the cube as

[JEST 2017]

shown below is

[email protected]

11

physicsguide CSIR NET ) GATE

Electrostatics: Part-1

@Sk J ahiruddin ) 2020

z l

l

y l "' •

-----------

11 oo : oo : 31 l

o: - --

l

--- 4 - - - - 4

y l "

-----------



l

X

(a) 0 (b) kc0 l~(v'3 - 1) kcal ~( v'2 - 1)

(c) kc0 l~(vb - 1)

(d)

Prob 1.20. Five sides of a hollow metallic cube are grounded and the sixt h side is insulated from the rest and is held at a potential (see figure) [TIFR 2012]

--

The potential at the centre O of t he cube is

(a) 0

(b)

! (c) ! (d) f

[email protected]

2

12

physicsguide CSIR NET, GATE

@Sk J ahiruddin , 2020

Electrostatics: Part-1

Prob 1.21. A point charge q sits at a corner of a cube of side a, as shown in t he figure below. The flux of the electric

[TIFR 2013]

field through the shaded side is

71

11 oo : oo : 34 1.21. point c arge q sits at a corner o side a, as shown in t he figure below. The flux of t he electric [TIFR 2013]

field through the shaded side is

q

q

(a) 8 co

q

(b) 16co

q

(c) 24c

(d) 0

q

6co

Prob 1.22. A charge q is at the cent re of two concent ric sp heres. The outward electric flux through t he inner sphere is 4? while t hat t hrough t he outer sphere is 4?. The am ount of charge contained in t he region between t he two spheres

[JAM 2015]

is

(a) 2q

(b) q

(c) -q

(d) -2q

Prob 1.23. A closed Gaussian surface consisting of a hemisphere and a circular disc of radius R , is placed in a uniform electric field ,as shown in the figure.The circular disc

£

makes an angle 0 = 30° with the vertical. The flux of the electric field vector coming out of the curved surface of the j [email protected]

13

physicsguide CSIR NET, GATE

@Sk J ahiruddin , 2020

Electrostatics: Part-1

[JAM 2011]

hemisphere is , , ,

00: 00: 36 Electrostatics: Part-1

hemisphere is

[JAM 2011] , , ,

,

, , ,

, , ,

,

, , ,

E

I , I , I ,

Prob 1.24 . A cube has a const ant electric potent ial V on its surface. If t here are no charges inside t he cube, t he potential at t he centre of t he cube is [JEST 2012]

(a) V

(b)

r (c) 0

(d)

r

Prob 1.25. T he charge per unit length of a circular wire of radius a in t he xy plane, with its cent re at t he origin, is A= ,X. 0 cos0, where ,X. 0 is a constatnt and t he angle 0is measured from t he posit ive x-axis. T he electric field at t he cent re of t he circle is [NET Dec 2016] (a) = - 4 2 (b) 15 = 4 2 (C) £ = - 4 d)

15 ;~a 15=41rcoa Ao k

;~a

[email protected]

14

@Sk J ahiruddin, 2020

;~a]

(

physicsguide CSIR NET, GATE

Electrostatics: Part-1

Prob 1.26. T wo uniformly charged insulat ing solid spheres

11 oo : oo : 39

@Sk J ahiruddin , 2020

Electrostatics: Part-1

Prob 1.26. Two uniformly charged insulating solid spheres A and B, both of radius a, carry total charges + Q and -Q , respectively. The spheres are placed touching each other as [NET Dec 2016] shown in the figure.

+

+ +

A

-

+

-

B

-

+ +

If t he potential at t he centre of the sphere A is VA and t hat at t he centre of B is VB, then t he difference VA-VB is

(a)

4~a

(b)

2--;~a

( C) 2~a

( d) 4--;~a

Prob 1.27. Consider a sphere S 1 of radius R which carries

a uniform charge of density p. A smaller sphere S2 of radius a < ~ is cut out and removed from it. The centres of the two spheres are seperated by t he vector b = nf ,as shown in [NET June 2016] t he figure. [email protected]

@Sk J ahiruddin, 2020

15

physicsguide CSIR NET, GATE

Electrostatics: Part-1

11 oo : oo : 42 15

[email protected]

physicsguide CSIR NET, GATE

@Sk J ahiruddin , 2020

Electrostatics: Part-1

The electric field at a point P inside S 2 is (b) pR (7 -na) (c) pRn (a) pRn 3 £o

3£oa

6 £0

(d)

7 3£o R pa

Prob 1.28. Four equal charges of + Q each are kept at the

vertices of a square of side R. A part icle of mass m and charge + Q is placed in t he plane of the square at a short distance (a < < R) from t he centre. If the mot ion of the particle is confined to the plane, it will undergo small oscillations with an angular frequency [NET June 2016]

(a)

21r£~~ 3m

(b)

1r£o~3m

(

C)

1r~~~

(

d)

Q2

j [email protected]

16

physicsguide CSIR NET, GATE

11 oo : oo : 44 Q2

j [email protected]

16

@Sk J ahiruddin , 2020

physicsguide CSIR NET, GATE

Electrostatics: Part-1

Prob 1.29. A sphere of radius R has a unifor charge density p. A sphere of smaller radius ~is cut out from the original sphere, as sho,vn in t he figure below. The centre of

t he cut out sphere lies at z = ~. After the small sphere has been cut out, the magnitude of t he electric field at z = - ~ is .e!i. What is the value of integer n? [JAM 2017] nt0

R/2

R

Prob 1.30. A hollow, conducting spherical shell of inner

radius R 1and outer radius R 2 encloses a charge q inside, which is located at a distance (d < R 1 ) from t he cent re of t he spheres. The potential at the cent re of the cell is [JAM 2015]

11 oo:oo:47

[email protected]

17

physicsguide CSIR NET, GATE

@Sk J ahiruddin, 2020

(a) Zero

(b)

Electrost atics: Part-1

i 41rt 0

q

d

Prob 1.31. A solid, insulating sphere of radius 1 cm has 7 charge 10- C distributed uniformly over its volume . It is surrounded concent rically by a conducting thick spherical shel of inner radius 2 cm , outer radius 2.5 cm and is charged with -2x10- 7 C. What is the electrostatic potential in Volts on t he surface of t he sphere? [JEST 2017]

Prob 1.32. Consider an infinitely long cylinder of radius R , nlA.c:P-o 2r 02

[NET

(d) _ 2ecoc/>o

(c) _ Eoc/>o er02

r2 0

Prob 1.38. A solid sphere of radius R has a charge density, given by p(r) = p0 (1 - ~) where r is the radial coordinate

and p 0 , a and R are positive constants. If the magnitude of [email protected]

20

physicsguide CSIR NET, GATE

@Sk J ahiruddin, 2020

t he electric field at r t he value of a is (a) 2

Electrost atics: Part-1

=

(b) 1

R/ 2 is 1.25 t imes that ar r = R , the [NET Dec 2014] (c) 1/2

(d) 1/4

Prob 1.39. Consider a hollow charged shell of inner radius a and outer radius b . The volume charge density is

p(r) = r\(k is constant) in the region a< r < b. The magnitude of the electric field produced at a distance r > a is [NET Dec 2012] (a) (b) (c) (d)

k (b- 2a)

for all r > a

k (b~i) k (r - 2a )

for a < r < b and Eor kb2 for r > b fora < r < b and k(b- 2a ) for r > b

k (r- a)

for a < r < b and

Eor

Eo

Eor

Eo a 2

Eor

k(b- a)

Eor 2

for r > b

Prob 1.40. The shape of a dielectric lamina is defined by t he two curves y = 0 and y = 1- x 2 . If the charge density

11 oo :oo : s1 (d)

k(r - 2a) Eo a

for a < r < b and

k(b- 2a) b

Prob 1.40. The shape of a dielectric lamina is defined by t he two curves y = 0 and y = 1- x 2 . If the charge density

of t he lamina a = l 5y C / m 2 , then the total charge on t he [JAM 2016] lamina is ___________ C.

Prob 1.41. A sphere of inner radius 1 cm and outer radius 2 cm , centered at origin has a volume charge density

Po =

~r,

4

where K is a non zero constant and r is the radial

j [email protected]

21

physicsguide CSIR NET I GATE

@Sk J a hiruddin , 2020

Electrostatics: Part-I

distance . A point charge of magnitude 10- c is placed at 2 t he orgin. For what value of Kin units of C/ m , the electric field inside the shell is constant? [JEST 2017] 3

Prob 1.42. A point particle of mass m carrying an electric

charge q is attached to a spring of stiffness const ant k. A constant electric field E along the direction of the spring is switched on for a time interval T ( where T i. l Eo

11 oo : 01 : s1 Sol 1.13. We can solve this by applying Gauss' Law. We consider a cylindrical Gaussian surface or length land radius r :

EdS = Al

Eo



A E · 21rr l = -l

Eo

A l " ⇒ E = --r 21rEo r We can now note f =~and r= xx+yy with r = ✓x 2 + y 2 . To find the field the value of t he field at (a, b), we plug in t he values of (x, y) in the expression for f and obtain

E= A ax+by 21rEo a2

+ b2

Sol 1.14. The electric field on t he axis is

E=

z

A(21rR)z

1

41rEo ( R 2 + z2)3/2

This will be maximum when the derivative of the electric field with respect to z will be zero.

8E

3z 2

1

az

(R2 + z 2 )3/ 2 This will equals to ZERO , hence 47rEo

3z

2

=

[email protected]

R

2

+z

2

z= -

R

v'2

41

physicsguide CSIR NET ) GATE

@Sk J ahiruddin ) 2020

Electrostatics: Part-1

Sol 1.15. Fi1·stly, for r < a , No charge is enclosed , and, as we know, inside the conductor all charges reside at t he C' 11rft'.l d, Total charge Enclosed

2Q, so,

Applying Gauss Law,

E

2 = Q 2r 41rc 0r

. So, option (d) is correct.

Sol 1.16. In order to have,

ab

=

ad ,

where

charge density of the shell of radius b and, charge density of t he shell of radius d Q 41rb2

ab

=surface

ad

=surface

2Q 41rd2

Hence,

Hence,option (c) is correct. [email protected]

@Sk J ahiruddin, 2020

42

physicsguide CSIR NET, GATE

Electrost atics: Part-1

11 oo :01 : s1

@Sk J a hiruddin , 2020

Electrostatics: Part-1

Sol 1.17. For the disc, t he electrostatic potent ial at point P, dV = dq 41rc:0Jr2 + 4£2

d-V =

a(21rrdr) 4nc:o J r 2 + 4£ 2 1

or, L

rdr

J r + 4£ Hence, 2

L/ 2

Now consider, r 2

+ 4£ 2 = z 2 .

v = 2c:o a

~ L

m2

V = o-L (v'5 -

2so Now for t he curved path, dV =

2

dz

v'17) 2

dq 4nsoJ (2L - r) 2 + L2

[where we considered a ring at a distance r from t he bot tom of the cylinder.] Here, dq = a(21rrdr )

V= j a [email protected]

(c)Sk J a,h ir11ddin. 2020

a (21rLdr) 4nc:o o J (2L - r) 2 + £ 2 l

L

43

physicsguide CSIR NET, GATE

Electrosta,t ics: Part-1

11 oo : 01 : sg 43

j [email protected]

physicsguide CSIR NET I GATE

@Sk J ahiruddin , 2020

Electrostatics: Part-1

L

dr

Now, if we consider (2£- r) t he question, we get,

V=

= z From the formula given in

- o-L [ln((2 L -r)+ J (2L-r )2 + L 2 )] 2co

in t he limit Oto L. Hence,

1/ = ;c~ln [(2 + v'5) /( 1 + v/2)]

Sol 1.18. We know from Gauss law, Electric flux t hrough

a closed surface

ff E. dS =

Q enc co Now make it a close surface imagining another cone below the original one. Through that closed surface, electric flux coming out will be :- co Q Hence, flux coming out through

one cone is nothing other t han, Q . Hence, option (b) is 2co correct. ~

Sol 1.19. Since the electric field is given, we can use v7 · E .P... to determine the volume charge density p €Q

=

Now, we can calculate the total charge enclosed by t he cube [email protected]

44

physicsguide CSIR NET, GATE

11 oo : 02 : 02 = kEo-Jx =

ax

o

2,Ji

Now, we can calculate the total charge enclosed by t he cube 44

[email protected]

physicsguide CSIR NET ) GATE

@Sk J a hiruddin ) 2020

Electrost atics: Part-1

be performing the following volume integral with appropriate as indicated by the figure. Look carefully at the limit of x axis. •

Q=

pdxdydz V

2z dx

kEo 2

= = =

k Eo l

l 2

2

Jx

vx

l

l

dy 0

dz 0

2l

2 l 2 kEol +~(V2- 1) kEol~ ( V2 - 1)

Sol 1.20. There are two ways to solve the problem. One

is the longer way where you need to apply some advance techniques called boundary value problem. The other is t he short way which I will discuss now. We will apply superposition principle of electric field and potential plus the idea of conductor. First consider what if all the sides have potential q> ? Consider two arbitrary points inside the cube. Let's say t he points are a and b. The potential difference between a

11 oo : 02 : os Consider two arbitrary points inside the cube. Let's say the points are a and b. The pot ential difference between a

[email protected]

physicsguide CSIR NET, GATE

45

@Sk J ahiruddin , 2020

Electrostatics: Part-1

and b will be equal to b

(b) -(a)= -

-

-

E-dl a

As t here is no charge inside the cube, application of Gauss law gives that t he field inside t he cube will be ZERO. Hence

cp(b) - (a) = 0

⇒ (a) =

(b)

There is no harm if one of t he points a or b touches the surface. As the potential at the surface is , potential at all the points inside the cube will be same which will be . From symmetry of the problem we see that at t he center of t he cube all sides will have equal influences. If we say t hat the contribut ion to t he potent ial at t he center from each side is c, then the potential at the center will b e

We have already proved t hat the pot ent ial everywhere inside the cube is same. Hence,

= 6c·

'

C

1

=-

6

11 oo : 02 : 01 We have already proved t h at the potent ial everywhere inside the cube is sam e. Hence,

=

1

6c ·

C

'

=-

6

Now in our problem a single side is at potential , all other sides are grounded.

So the contribut ion to t he cent er is 46

j [email protected]

physicsguide CSIR NET, GATE

@Sk J ahiruddin , 2020

Electrostatics: Part-1

coming from one single side. Hence the potential at t he center is ½ enter

= c = 6

Sol 1.21. To m ake the charge closed , imagine 8 more cubes and electric flux coming out of these 8 cubes Q enc

Eo

Hen ce, flux coming out of 1 cube

Q 8Eo

This flux can be out only through 3 sides as other 3 sides are p erpendicular to electric field lines. Hence electric flux coming out of shaded surface

Q 24c 0

Hen ce, option (c) is correct

11 oo : 02 : 1o Q 24c: 0

Hence, opt ion (c) is correct Sol 1.22. Electric flux t hrough a closed surface is defined as

E . dS =

Qenc

(1.1)

Eo 47

[email protected]

physicsguide CSIR NET, GATE

Electrostatics: Part-1

@Sk J ahiruddin, 2020

Let q2 be t he amount of charge in the region between t he two spheres. Then t he total flux t hrough t he out er sphere 2

= q + q2 EO

2 = + q2 EQ

q

q2

Eo

Eo

q

= q2

Sol 1.23. Since there are no enclosed in t he volume , so from 1.1 t he total flux coming out of t he volume is zero. Hence

-+

-+

E · dS = 0 -I-

-I

~

E · dScurved surface

+

~

E · dSflat surface

=0

f E · d S curved surface + f E · ndSflat surface = 0

11 oo : 02 : 12 _,

_,

E · dS = 0 _,

_,

E · dScurved surface _,

_,

+

_,

E · dSflat surface

=0

_,

+

E · dScurved surface

where

_,

E · ndSflat surface

=0

n is the normal unit vector to the fl at surface.

48

[email protected]

So

n

physicsguide CSIR NET, GATE

Electrostatics: Part-1

@Sk J ahiruddin , 2020 _,

makes an angle 150° with E. Hence _,

_,

E · dScurved surface +

_,

E

COS

150°dSflat surface

_,

E · dScurved surface

v'3 -

_,

2

7r R

2

=0

E = 0

_,

E · dScurved surface

=

v'3 2

7r R

2

E

Sol 1.24. Since there are no charges inside t he cube, Laplace's equation gives us

(1.2) Since cp = V at the surface. So, one solution of t he Laplace's equation is cp = V inside. Uniqueness theorem warrant us t hat this is t he only solut ion. -

......

...

..

.....

.

.

..

-

..

......

..

11 oo : 02 : 1 s Since = V at the surface. So, one solution of t he Laplace's equation is = V inside. Uniqueness theorem warrant us t hat this is the only solut ion. Sol 1.25. The electric field for a line charge A is defined as -

R =r-r

where

49

j [email protected]

-'I

(1.3)

physicsguide CSIR NET 1 GATE

@Sk J ahiruddin, 2020

Electrostatics: Part-1

Hence, in this case

-

21r \

1

/\ QCOS :;, 2

E = - --

41rEo

r

0

A

41rEo r'

0 21r

Ao Ao

'd0' , . , r r

21r

1 Ao

4nE0 a

0'

A

cos 0'i

+ sin 0' j cos0' d0'

cos2 0'i

+ cos 0' sin 0'3 d0'

0

~

--i

4Eoa Sol 1.26. In t his problem , we can first find the potential at

t he center of A due to t he charge + Q distributed over the first sphere A and the charge - Q distributed ove1~ the second sphere. We can do a similar calculation for the potential at t he center of B and subtract the two to obtain the required

11 oo : 02 : 11 Sol 1.26. In this problem, we can first find t he potential at

t he center of A due to the charge + Q distributed over the first sphere A and t he charge - Q distributed over the second sphere. We can do a similar calculation for the potential at t he center of B and subtract t he two to obtain the required potential difference. (In fact, we shall see that we don't have explicitly calculate the potential at B ). First , we consider t he center of the sphere A as our origin and calculate t he electric field at a point r > a and at a point r < a using Gauss' law. These come out to be

for r > a, and

E = Q r 41rEo a 3 [email protected]

physicsguide CSIR NET, GATE

50

@Sk J ahiruddin, 2020

Electrost atics: Part-1

for r < a We can now calculate t he potential at the center of A (i.e., at t he origin ) using t he above fields. 0

Vi = -

Edr 00

a

o Q

Qrdr

00

+ 41rEoa3 r2

a

a

1

Qdr 47rEor2 00

47rEO 2a 3 0 r a 3 Q --2 41rEoa Now the sphere B will also have a contribution to the potential at the center of A. Here, if we calculate the field or

11 oo : 02 : 20 47rEO 2a 3 0 r a 3 Q 2 41rEoa Now the sphere B will also have a contribution to the potential at the center of A. Here, if we calculate the field or potential for a point outside the sphere, it would appear as if t he ent ire charge is located at the center of t he sphere (and not just distributed t hrough the entire volume of the sphere) . Now the center of t he sphere B is at a distance of 2a from the center of of the sphere A. So we imagine that t he charge -Q is placed at a distance 2a from the origin, and calculate the potential at t he origin. We can directly

right is off as

4

1r~%a), and so adding t his to Vi 3 2

-

1 2

we get

Q 41rE0 a

Now we have to repeat the same calculation of the potent ial j [email protected]

51

physicsguide CSIR NET I GATE

@Sk J ahiruddin , 2020

Electrostatics: Part-I

at t he center of the sphere B but wit h charge - Q instead of + Q , so we merely flip t he sign of Q in the above expression to obtain VB = 4-7rcoQa So the potential difference becomes

VA - VB= 2 Q 41rE0 a

Q 21rE0 a

Sol 1.27. In this problem, a portion of the sphere has been

removed. Since t he portion that has been removed is a sphere as well, we can think of solving t he problem in the following manner: We can first find the field due to the hi O'O'Pr l cm physicsguide CSIR NET, GATE

Electrostatics: Part-1

@Sk J ahiruddin, 2020

So, t he electrostatic p otential

1 T

T

{ 2.5x 10- 2

dr

1

{1 x 10- 2

dr

11 oo : 02 : 33 @Sk J ahiruddin, 2020

Electrostatics: Part-1

So, t he electrostatic potential V

=

l

2.5x 10- 2

41rEo

=9

=9

X

X

oo

10

9

10

4

X

10-

7

dr

1

r2

41rEo 10

X

1

1

2

2.5

--

1x 10- 2 2 x 10-

1

2

2.5

dr

r

2

+9 X

2

10

9

X

10-

5

1 1 -2

= 9000 Sol 1.32. Using Gauss's law the electric field inside the cylinder at distance r, E · 21rrh

=

r

1 -21rh

p(r')r' dr'

EQ r

0

E= k r' 2dr' Ear o E

=

k r2

3Eo

So, the electrost atic potential V = - k

R

r' 2dr

3Eo r k ex - (r 3 - R3 ) 3Eo k r3 ex - - - 1 3Eo R 3 [email protected]

56

physicsguide CSIR NET, GATE

Electrostatics: Part-1

@Sk J ahiruddin , 2020

I

,

,

.,

11 oo : 02 : 36 [email protected]

56

physicsguide CSIR NET, GATE

@Sk J ahiruddin, 2020

Electrostatics: Part-1

Sol 1.33. Let t he sheets are parallel to xy plane , and the posit ive charged plate is at z = 0 and the negative charged

on is at z = l m. Then , t hen t he electric field in the region between the sheets for t he positive charged plate is ;~ z and for the negative charged one is ; EQ- z. Using super position principle a+

a+

E=-

+ 2Eo 2E0

6.8

X

10- 6

4.3

X

10- 6

----+---2Eo

~ 6.27

X

2Eo

10 5

Sol 1.34. Using Gauss's law the electric field for electron

j [email protected]

57

physicsguide CSIR NET, GATE

T""l

,

I

,



T""\



-1

00: 02: 38

57

[email protected]

physicsguide CSIR NET, GATE

@Sk J ahiruddin , 2020

Electrostatics: Part-1

distribut ion , at a dist ance a a

Ee41ra2 = - qo

41r

Eo

1ra3

2r

2

e- 0 r dr o

a

Ee = -

E e-- E e-- Ee-

Ee =

qo

Eo1ra 5

r2

r

dr

0

qo Eo7ra5

qo Eo7ra5

qo Eo7ra5

5qo 41rE0a 2 e 2

5a 3 - 4e 2

+

a3 4

1 qo 41rEo a2

The electric field for the proton at a distance a is E _ P -

1 qo 41rEo a 2

Hence the total electric field at a distance a is

[email protected]

58

physicsguide CSIR NET, GATE

11

00:02:41

[email protected]

physicsguide CSIR NET , GATE

58

@Sk J ahiruddin, 2020

Electrostatics: Part-1

Sol 1.35. _,

_,

p

p

_,

V-E = Eo -= V· Eo A

=O'.

A

_, ( r) r ( r) _, r V 1 - e- R • r 2 + 1 - e- R V · r 2 r

1 e- R ( r) 3 + 1 eR 41rb (r) =a - R r2 r aEo e - R P = R r2

Hence the charge contained wit hin a sphere of radius R , centred at t he origin is R

q

2

=

p41rr dr 0

41r0'.Eo q= R 41raEo q= R q

0

= 41raE0 (1 -e- 1 )

11 oo : 02 : 43

[email protected]

physicsguide CSIR NET, GATE

59

@Sk J ahiruddin , 2020

Electrostatics: Part-1

Sol 1.36. Using Poisson's equation, charge density

n n = -Eo V

·

V

n = - A Eo v •

-Ar

Ae r ,e

-Ar

- /\

rA

r

e

- Ar

r2

r A

A r e --r r

Ar n e - v · - -2 r r 1 8 e- Ar (re - Ar)+ A - - e- Arv. = -Aco -A-2 2 A

r

ar

A

r

A

r r2

1 e -Ar 3 (r) A-(eAr Are-Ar)+ A 41ro = - Aco r2 r2

The total charge enclosed within a sphere of radius of

½,

11 oo : 02 : 46

j [email protected]

physicsguide CSIR NET 1 GATE

60

@Sk J ahiruddin, 2020

Electrostatics: Part-1

with its origin at r = 0 is 1

>.

q=

2

p41rr dr 0

= - A E0 41r 1

>.

= - A Eo 41r

re- >.r dr - 1

0

e

Sol 1.37. Using Poisson's equation, charge density

_£__ = V2 R2)

0

12

[email protected]

physicsguide CSIR NET, GAT E

@Sk J ahiruddin, 2020

Electrostatics: Part-2

The electrost atic energy

allspace

R2

EO

-

')

n

.\ 1

2

21rsds

11 oo : oo : 34

allspace

R2

,\ 1 21rc0 s

EQ 2 Ri 1 ,\2

2

21rsds

R2 1

---

-ds

2 27rEo

Ri

S

,\2

=-

47rEQ

ln(R2/R1 )

Prob 1.9. For a conductor all charges will be on the sur-

face. So, the electrostatic energy in that case

q2 1

E =-81rco R

9

X

109 100

2

9 = 7

X

8.5

X X

10- 6 10- 2

1000

= 529 Prob 1.10. For a uniformly charged spherical shell the elec-

j [email protected]

13

physicsguide CSIR NET, GATE

@Sk J ahiruddin , 2020

Electrostatics: Part-2

t rostatic energy is given by q2 1 E =-R7rFn

R

00: 00: 36 Electrostatics: Part-2

t rostatic energy is given by q2 1

£ =--

=

81rco R 2 15 24 25 8 3.125

Prob 1.11. This problem is similar to Prob 1.3. . The electrostatic potential energy of the configuration is then in

this case £

= Ll1 + Ll2 + L'.l3 q2 1

2q 2 1 = 0+ + 41rco L 41rc0 L 3q 2 1 41rco L

[email protected]

@Sk J ahiruddin , 2020

2

Conductors

14

physicsguide CSIR NET, GATE

Electrostatics: Part-2

11 oo : oo : 39 @Sk J ahiruddin, 2020

2

Electrostatics: Part-2

Conductors

Prob 2.1. A solid spherical conductor encloses 3 cavities, a cross-section of which are shown in the figure. A net charge +q resides on the outer surface of t he conductor. Cavities

A and C contain point charges +q and -q respectively. +q

The net charges on the surfaces of t hese cavities are [TIFR 2015] (b) A = - q, B = q, C = 0 (a)A = - q, B = 0, C = -q (c) A= +q, B = 0, C = -q (d) A= -q, B = O,C = + q Prob 2.2. An arbitrarily shaped conductor encloses a charge q and is surrounded by a conducting hollow sphere as shown in t he figure. Four different regions of space, 1,2,3,4 are indicated in t he figure.W hich one of t he following statements is correct? [JAM 2016]

[email protected]

@Sk J ahiruddin , 2020

15

physicsguide CSIR NET, GATE

Electrostatics: Part-2

11 oo : oo : 41 15

[email protected]

physicsguide CSIR NET, GATE

@Sk J ahiruddin, 2020

Electrostatics: Part-2

-

1

3 4

')

(a) The electric field lines in region 2 are not affected by t he posit ion of the charge q (b) The surface charge density on the inner wall of the hollow sphere is uni£orm (c) The surface charge density on the outer surface of the sphere is always uniform irrespective of t he position of charge q in region 1 (d) T he electric field in region 2 has a radial symmetry Prob 2.3. A solid spherical conductor has a conical hole

made at one end, ending in a point B , and a small conical projection of the same shape and size at the opposite side, ending in a point A. A cross-section t hrough the center of t he conductor is shown in the figure below.

j [email protected]

16

physicsguide CSIR NET, GATE

11 oo : oo : 44

j [email protected]

16

@Sk J ahiruddin , 2020

physicsguide CSIR NET, GATE

Electrostatics: Part-2

A

If, now a posit ive charge Q is t ransferred to t he sphere, t hen [TIFR 2014] (a) the charge density at bot h A and B will be undefined (b) t he charge density at A will be the same as t he charge density at B. (c) the charge density at A will be more than the charge density at B. (d) the charge density at B will be the more than t he charge density at A.

Prob 2.4. A spherical conductor, carrying a total charge Q, spins uniformly and very rapidly about an axis coinciding with one of its diameters. In t he diagrams given below, t he equilibrium charge density on its surface is represented by the t hickness of t he shaded region. Which of these dia[TIFR 2014] grams is correct?

11 oo:oo:47 ing with one of its diameters. In t he diagrams given below, t he equilibrium charge density on its surface is represented by the t hickness of t he shaded region. Which of these dia[TIFR 2014] grams is correct? [email protected]

17

@Sk J ahiruddin, 2020

physicsguide CSIR NET, GATE

Electrostatics: Part-2

(b)

(c)

Prob 2.5. A point charge q < 0 is brought in front of a

grounded conducting sphere. If t he induced charge density on t he sphere is plotted such t hat the thickness of the black shading is proportional to the charge density, t he correct [TIFR 2019] plot will most closely resemble

11 oo : oo : 49

[email protected]

18

physicsguide CSIR NET, GATE

@Sk J ahiruddin, 2020

Electrostatics: Part-2

(a) 0

q

(c)

0

q

0

q

--

(b)

0

q

(d)

Prob 2.6. MSQ: Out of the following statements, choose [JAM t he correct option(s) about a perfect conductor. 2019] (a) The conductor has an equipotential surface (b) Net charge, if any, resides only on the surface of conductor (c) Electric field cannot exist inside the conductor (d) Just outside the conductor, t he electric field is always perpendicular to its surface Prob 2. 7. A conducting sphere of radius lm is placed in air. The maximum number of electrons that can be put on

11 oo : oo : s2 perpendicular to its surface Prob 2. 7. A conducting sphere of radius l m is placed in air. The maximum number of electrons that can be put on t he sphere to avoid electrical breakdown is about 7 x 1on,

where n is an integer. The value of n is

[GATE 2020]

Assume: j a [email protected]

@Sk J a hiruddin , 2020

19

physicsguide CSIR NET 1 GATE

Electrostatics: P art-2

-+

Breakdown electric field strength in air is E 6

10 V / m Permitti,rity of free space co = 8.85 x 10- 12 F / m Electron charge e = 1.60 x 10- 19 C

3

X

11 oo : oo : s4

[email protected]

@Sk J ahiruddin, 2020

2.1

Ans keys

2.1. d 2.2.

C

2.3.

C

2.4. b 2.5. b 2.6. a,b,c,d

2.7. 14to 15

20

physicsguide CSIR NET, GATE

Electrostatics: Part-2

11 oo :oo : s1 2. 7. 14 to 15

j [email protected]

@Sk J ahiruddin , 2020

2.2

21

physicsguide CSIR NET I GATE

Electrostatics: Part-2

solution

Sol 2.1. The +q charge will induce -q charge on the surfac e of the conductor and vice versa. So (d) is the right answer. Sol 2.2. This problem is discussed in details in Griffit hs Electrodynamics book. Sol 2.3. A conductor mus always be equipotential. For a charge configuration V rv ; . But for Ar > R (the radius of t he spherical conductor) so the charge accumulation at A must be more than the average charge density to balance it out. Similarly for B r < R so the charge accumulation at A must be less than the average charge density.

11 oo : 01 : oo t he spherical conductor) so t he charge accumulation at A must be more t han the average charge density to balance it out. Similarly for B r < R so t he charge accumulation at A must be less t han the average charge density. Sol 2 .4. Since the charges are rotating around t he axis in a circular path very rapidly. The magnetic field for t his

rotation of charge is along z direction . So the force on the charge is

= v B cp x z A

=pw B p

So for magnetic force on charges is along outward radius [email protected]

@Sk J ahiruddin ) 2020

22

physicsguide CSIR NET ) GATE

Electrostatics: Part-2

vector from t he axis and is proportional to radial distance. So (b) is t he right answer. Sol 2.5. This is obvious that more charge will be induced on the near side of the conductor from the charge than the far side. But it 's not t hat far side will have no induced

charges. Sol 2.6. All four statements are basic properties of a con-

ductor. The proofs are rigorous and you can find it in any book. Sol 2.7.

11 oo : 01

: 02 p r

1.. ,.,

ductor. The proofs are rigorous and you can find it in any book.

Sol 2.7.

[email protected]

@Sk J ahiruddin, 2020

3

23

physicsguide CSIR NET, GATE

Electrostatics: Part-2

Dipole

Pro b 3 .1. Four point charges are placed in a plane at the following positions:

+ Q at (1,0), -Q at (-1,0), + Q at (0,1), -Q at (0,- 1). At large distances the electrostatic potential due to t his charge distribution will be dominated by the [GATE 2007] (a) monopole moment (b) dipole moment (c) quadrupole moment (d) octopole moment Prob 3.2. Three point charges q, q and -2q are located

11 oo : 01 : os charge distribution will be dominated by the [GATE 2007] (a) monopole moment (b) dipole moment (c) quadrupole (d) octopole moment moment Prob 3.2. Three point charges q , q and -2q are located at (0,-a,a), (O ,a, a) and (0,0, -a) respectively. The net dipole moment of t his charge distribution is [GATE 2006

(b) 2qak

(a) 4qak

(d) -2qa]

(c) -4qai

Prob 3.3. A circular disc of radius a on the xy plane has a surface charge density a = aor ~osB. The electric dipole

[GATE 2008]

moment of t his charge distribution is 4

3

3

4

(a) o-o;a X (b) o-o;a X (c) - o-o;a X (d) o-o;a X Prob 3.4. An insulating sphere of radius a carries a charge

density p(r) = p0 ( a 2 - r 2 ) cos 0 : r < a. The leading order term for the electric field at a distance d, far away from the charge distribution, is proportional to [GATE 2010] j [email protected]

24

physicsguide CSIR NET, GATE

@Sk J ahiruddin, 2020

(b) d- 2

Electrostatics: Part-2

(c) d- 3

(d) d-4

Prob 3.5. A charge -q is distributed uniformly over a sphere,

with a positive charge q at its centre in (i). Also in (ii), a charge -q is distributed uniformly over an ellipsoid with a positive charge q at its center. With respect to the origin of the coordinate syst em , which one of the following state[GATE 2015] ments is correct?

11 oo : 01

: 01

charge -q is distributed uniformly over an ellipsoid with a posit ive charge q at its center. With respect to t he origin of t he coordinate system, which one of the following statements is correct? [GATE 2015] X

X

~t--

y

z

----- z y

(i)

(ii)

(a) The dipole moment is zero in both (i) and (ii). (b) The dipole moment is non-zero in (i) but zero in (ii). (c) The dipole moment is zero in (i) but non-zero in (ii). (d) The dipole moment is non-zero in both (i) and (ii). Prob 3.6. Charges Q, Q and - 2Q are placed on the vert ices of an equilateral t riangle ABC of sides of length a, as shown in the figure. The dipole moment of this configurat ion of charges, irrespective of t he choice of origin, is [NET June 2012]

25

[email protected]

physicsguide CSIR NET, GATE

@Sk J ahiruddin , 2020

Electrostatics: Part-2

C

-2Q

t:,

J

A.____.________. B Q a Q . __ _ _ _ _ 11,

I

(a) + 2aQz

(b)

+v13aQ]

(c) -v'3aQ]

(d) 0

11 oo : 01

: 1o

A------------- B Q a Q

(a) +2aQi

+v'3aQ]

(b)

(c) -v'3aQ]

(d) 0

Prob 3. 7. Consider an axially symmetric st atic charge dis-

[NET June 2013]

t ribution of t he form, p

=

p0

ro

2

-

r

2 e - ro cos cp

r T he radial component of t he dipole moment due to t his charge distribution is (a) 21r port (b) 1r port Pro b 3. 8. Three charges (2C , - 1C, - 1C) are placed at

t he vert ices of an equilateral t riangle of side lm as shown in t he figure.T he component of t he electric dipole moment about t he marked origin along they direction is [GATE 2017]

________________c m. 26

[email protected]

physicsguide CSIR NET, GATE

Electrostatics: Part-2

@Sk J ahiruddin , 2020

y

';:::::===~ ===-==--___, _ _. x 1.5 -1C

m

11 oo : 01

: 12

y

-1C L - - - - -......:....:...1---_____.____;__;:__.:..... x 1.5 m



Prob 3.9. A particle of charge e and mass m is located at

the midpoint of t he line joining two fixed collinear dipoles with unit charges as shown in t he figure. ( The particle is constrained to move only along the line joining the dipoles). Assuming that t he length of the dipoles is much shorter than t heir separation, the natural frequency of oscillation of the [NET June 2013] particle is

R

I( I I I I I I I

I I

>i
> d) is [JEST 2017] (a) zero (b) proportional to d 1 1 (c) proportional to r 3 ( d) proportional to r 4 Prob 3.13. The electrostatic potential (r) of a distribution of point charges has the form (r )ex r- 3 at a distance r from the origin (0, 0, 0) , where r > a. Which of the following distributions can give rise to this potential? [TIFR 2015]

Prob 3.14. Four charges (two +q and two -q) are kept fixed at the four vertices of a square of side a as shown [NET Dec 2012] j [email protected]

@Sk J ahiruddin, 2020

29

physicsguide CSIR NET, GATE

Electrost atics: P art-2

11 oo : 01 29

[email protected]

: 20

physicsguide CSIR NET, GATE

Electrostatics: Part-2

@Sk J ahiruddin, 2020

q

q

-q

q (0,0,0)

a

q

a from the centre of the square. Then V(2r)/ V(r) is

(a) 1

(b) ~

(c) ~

(d)

1

Prob 3.16. The electrostatic lines of force due to a system

of four point charges is sketched b elow. [NET Dec 2014]

At a large distance r , t he leading asymptotic behaviour of the electrostatic potential is proportional to

11 oo : 01 : 2s

At a large distance r , t he leading asymptotic behaviour of the electrostatic potential is proportional to 31

[email protected]

physicsguide CSIR NET, GATE

@Sk J ahiruddin, 2020

(a) r

Electrostatics: Part-2

(c) r- 2

(b) r- 1

(d) r- 3

Prob 3.17. T wo electric dipole P 1 and P 2 are placed at (0, 0, 0) and (1, 0, 0) respectively, with bot h of them pointing in the +z direction. Wit hout changing t he orientationsof t he dipoles P 2 is moved to (0, 2, 0). The ratio of t he electrostatic potential energy of t he dipoles after moving to that before moving is [JAM 2006]

(a)

1 16

(b) ~

(c)

!

(d) ~

Prob 3.18. MSQ: Multiple Select Question For an elect ric dipole with moment = p 0 ez placed at t he origin , ( Po is a const ant of appropriat e dimensions and ex, ey and ez are unit vectors in Cartesian coordiante system) [JAM 2016] (a) potent ial falls as r12 , where r is the distance from origin (b) a spherical surface cent ered at origin is an equipotential surface (c) electric flux through a spherical surface enclosing the • or1g1n 1s zero (d) radial component of £ is zero on the xy-plane.

P





11 oo :01 : 2a surface (c) electric flux t hrough a spherical surface enclosing t he or1g1n 1s zero •





(d) radial component of

15 is zero on t he xy-plane.

32

j [email protected]

physicsguide CSIR NET I GATE

@Sk J ahiruddin , 2020

Electrost atics: Part-2

Prob 3.19. MSQ: For a point dipole of dipole moment P = pz located at t he origin, which of the following is (are) correct? [JAM 2017] (a) The electric field at (0, 0, 0) is zero. (b) T he work done in moving a charge q from (O,b,O) to (0, 0, b) is qp b2 47rEo (c) The electrostatic potential at (b,0,0) is zero. (d) If a charge q is kept at (0,0,b) it will exert a force of magnit ude qp on t he dipole. 41rEob3 .....

,...

Prob 3.20. An electric dipole of dipole moment P = qbi is placed at t he origin in the vicinity of two charges +q and - q at (L , b) and (L , - b), respectively, as shown in the figure [NET Dec 2018] below. The electrostat ic potential at t he point (L/ 2, 0) is

(a) (c)

;! (p +

qb 1rcoL2

(d)

L2_;4b2) 3qb 1rc0 L 2

(b)

1rco[L;~~b2]3/2

11 oo : 01

: 31

The electrostatic potential at t he point ( L / 2, 0) is ( a)

(c)

qb ( 1 7rco V

qb 1re:0L2

+

2 ) L2+ 4b2

(d)

3qb 1re:0L2

(b)

4qbL 1re:o[L 2+ 4b2]3/ 2

Pro b 3. 21. Consider a system of t hree charges as shown in t he figure below:

[email protected]

physicsguide CSIR NET ) GATE

33

@Sk J a hiruddin ) 2020

Electrostatics: Part-2

y

+q(L.b)



• I

I

A

---0



I

•.

(Ll2.0)

--- -• X

I

' I I



-q (L,-b)

z

--t-- -2

!

I I I

q 0

d

q

.,

(r, 0)

q

--

2

,., I I

d

I

d

y

I I

)I I

For r = 10m; 0 = 60degrees; q = 10- 6 Coulomb, and d = 10- 3m, the electric dipole potential in volts (rounded off to t hree decimal places) at a point (r, 0) is _____ (Use 2 1 9 : = 9 X 10 Nm2 ) [GATE 2019] 47rEQ C

11 oo : 01

: 33

or r = m; egrees; q = ou om , an d = 10- 3m , the electric dipole potential in volts (rounded off to three decimal places) at a point (r, 0) is _____ (Use 2 1 9 : = 9 X 10 Nm [GATE 2019] ) 4~EQ C2

Prob 3.22. Charges are placed as follows: q at (a, a, 0) and ( -a, -a, 0), and - q at (a, - a, 0) and ( -a, a, 0) At large distances, how does the electrostatic potential behave as a function of the distance r from the centre (0,0,0)? [JEST

2020] [email protected]

34

physicsguide CSIR NET, GATE

@Sk J ahiruddin , 2020

(A) 1/ r 3

(B) 1/ r 2

Electrostatics: Part-2

(C) 1/r

(D) 1/r 4

11 oo : 01

j [email protected]

35

@Sk J ahiruddin , 2020

3.1

: 36

physicsguide CSIR NET, GATE

Electrostatics: Part-2

Ans keys

3.1. b

3.9. d

3.17. d

3.2. a

3.10. b

3.18. a,c,d

3.3. b

3.11.

3.19. b ,c

3.4.

C

3.12. d

3.20.

3.5. a

3.13. a

3.21. 0.0045

3.6.

3.14.

3.22. a

C

3.7. a

C

C

3.15. d

C

11 oo : 01 3.6.

3.14.

C

3.22. a

C

3.7. a

3.15. d

3.8. 1.735

3.16. d

[email protected]

: 38

36

physicsguide CSIR NET, GATE

@Sk J ahiruddin, 2020

3.2

Electrostatics: Part-2

Solutions

Sol 3.1. The monopole moment

.

'l

=0 The dipole moment

. 'l

= (Q + Q)x + (Q + Q) = 2Qx + 2QiJ

11

00:01 :41

= (Q + Q)x + (Q + Q) = 2Qx + 2QiJ Hence t he dominant term is dipole potential.

Sol 3.2. The dipole moment

= q(-a] + ak) + q(a] + ak) - 2q(-ak) A

= 4qak Sol 3.3. The dipole moment for a cont inuous charge distri-

[email protected]

37

physicsguide CSIR NET, GATE

@Sk J ahiruddin , 2020

Electrostatics: Part-2

bution

p=

o-(r')dr' R

a

a r=O 4 a-or a l a 4 2

271"

cos 0' (cos 0' x + sin 0' i)) r' dr' d0'

0'=0 21r

(1 - cos 20' )d0' x

0'=0

o-01ra3,...

--x 4

Sol 3.4. The monopole moment

11 oo : 01

(1 - cos 20' )d0' x

-

a 4 2 ao1ra3

: 44

0'=0

A

--x 4

Sol 3.4. The monopole moment Q

= 21r

p0 ( a

2

-

r

2

2

)

cos 0 cos 0 sin 0r d0dr

r

2

The dipole moment a

21r

p=

po (a r=O

2

-

)

2

cos 0 sin 0' r d0drdcp

0'=0 a

p = Po

21r

r'

3

(a

2

-

r

2

cos 0 sin 0'

)

r=O

0'=0

[sin 0' cos cp' x + sin 0' sin ' fJ

r

16 a

6

cos 0'

3

0

+ cos 0' z] d

-l

1

= non zero So the dipole moment exist, and the electric field at a disj [email protected]

38

@Sk J a hiruddin , 2020

physicsguide CSIR NET 1 GATE

Electrostatics: Part-2



tance d , far away from t he charge distribution, 1s proport ional to rv d- 3

Sol 3.5. In both of t he configuration for every charge q at rthere is a char·ge q at So t he dipole moment is zero in both (i) and (ii) .

-r.

Sol 3.6. Since the monopole moment Li Qi = 0. The dipole moment becomes indep endent of the origin. So,

11 oo : 01

: 46

rthere is a charge q at -r' . So t he dipole moment is zero in both (i) and (ii) .

Sol 3.6. Since the monopole m oment L i Qi = 0. T he dipole moment becom es independent of t he origin. So,

_v;;:;aQ 0_ . Q. J+ i 2 A

=2X

Q1,.

A

A

-

= v3aQ] Sol 3.7. 21r

00

ro Pr= 2 Po r r=O 0 . The region x < 0 is filled uniformly wit h a met al. The electric field at t he point (~, 0, 0) is [NET June 2014] (a) -

(c)

lOe 97rEo e

7rEO

d2 ( 1, 0, 0)

d2 (1, 0, 0)

(b) g

lOe

7rEo

e

(d) -

7rEO

d2 ( 1, 0, 0)

d2 (1,0,0)

+q is

placed at (0, 0, d) above a grounded infinite conducting plane defined by z = 0. There are no charges present anywhere else. What is t he magni[JEST 2012] t ude of electric field at (0, 0, -d)? Pro b 4. 7. A point charge

(a) ,, q ,,,

(b ) -oo

( c) 0

( d ) ~,, q ,,,

11 oo : 02 : 1o +q is placed at (0, 0, d) above a grounded infinite conducting plane defined by z = 0. There Prob 4. 7. A point charge

are no charges present anywhere else. What is t he magnit ude of electric field at (0, 0, - d)? [JEST 2012]

(a)

(b) -oo

B1r:od2

(c) 0

(d)

161r~od2

Prob 4.8. A point charge q of mass m is released from rest

47

[email protected]

@Sk J ahiruddin, 2020

physicsguide CSIR NET, GATE

Electrostatics: Part-2

at a distance d from an infinite grounded conducting plane (ignore gravity) . How long does it take for the charge to hit t he plane? 3 3 (a) J21r Eomd q

~-3 (b) J21r Eomd q

[JEST 2016] 3 3 (c) J1r Eomd (d) q

3 E-om-d ✓~1r~

q

Prob 4.9. A hollow metallic sphere of radius a, which is kept at a potential V0 has a charge Q at its centre. The potential at a point outside the sphere, at a distance r from

[NET Dec 2015]

t he cent re, is

(a) Vo (c)

Q 41rEor

(b)

+ Voa r

2

Q 41r > d t he electrostatic potent ial of t his charge configuration would approximately be [NET Dec 201 7]

j [email protected]

@Sk J ahiruddin , 2020

51

physicsguide CSIR NET I GATE

Electrostatics: Part-2

Prob 4.16. Two point charges + 2Q and -Q are kept at

points with Cartesian coordinates (1,0,0) and (2,0,0) respect ively, in front of an infinit e grounded conducting plate at x= O. The potent ial at (x,0,0) for x > > l depends on x as [NET June 2018] (a) x - 3 (b) x - 5 (c) x - 2 (d) x - 4

11 oo : 02 : 22 points with Cartesian coordinates 1,0,0 and 2,0,0 respect ively, in front of an infinite grounded conducting plate at x=O. The potent ial at (x,0,0) for x > > l depends on x as [NET June 2018] (a) x - 3 (b) x - 5 (c) x- 2 (d) x - 4 Pro b 4.1 7. Consider two concentric spherical metal shells of radii r 1 and r 2 ( r 2 > r 1 ) . The outer shell has a charge q and t he inner shell is grounded. What is the charge on t he

[JEST 2019]

inner shell?

(a) _ r1 q

(c) 0

r2

Prob 4.18. An infinitely long wire parallel to the x -axis

is kept at z = d and carries a current I in the positive x direction above a superconductor filling the region z < 0 (see figure). The magnetic field B inside t he superconductor is zero so t hat t he field just outside the superconductor is parallel to its surface. The magnetic field due to t his con[GATE 2019] figurat ion at a point (x, y, z > 0) is --+

52

[email protected]

physicsguide CSIR NET) GATE

@Sk J ahiruddin ) 2020

Electrostatics: Part-2

I d

superconductor

/

/

/

/

/

11 oo : 02 : 2s I d A

----r----,..---r----r---r---r...___,,.....----r--r---r-~

X

superconductor

//// A

-(z- d)j+yk [y2 + (z - d)2]

(a) (b)

- (z - d)j + yk + (z + d)] - yk y2 + (z - d)2 y2 + (z + d)2

(c)

-(z- d)j+yk y2 + (z - d)2 y J + (z - d) k

(z + d)] - yk y2 + (z + d)2

( z + d) k ----- + -----

(d)

y2

+ (z - d) 2

[email protected]

y3y2

+ (z + d) 2

physicsguide CSIR NET , GAT E

53

@Sk J a hiruddin , 2020

4.1

Electrostatics: Part-2

Ans keys

4.1. a

4.10. b

4.2. 11

4.11.

4.3. b

4.12. d

C

11 oo : 02 : 2a 4.1. a

4.10. b

4.2. 11

4.11.

4.3. b

4.12. d

4.4. a

4.13. 0.9142

4.5. b

4.14. 3.9

4.6. d

4.15. b

4.7. c

4.16. d

4.8. a

4.17. a

4.9. d

4.18. b

j [email protected]

@Sk J ahiruddin , 2020

4.2

54

C

physicsguide CSIR NET, GATE

Electrostatics: Part-2

Solutions

Sol 4.1. Place an image charge - q at x = -.5m and -2q at x = -l.5m. Hence the force on q for this charge config1,ri::1 +1()n

11 oo : 02 : 30 4.2

Solutions

Sol 4.1 . Place an image charge -q at x = -.5m and -2q at x = -l .5m. Hence the force on q for this charge configuration 2 1 F = - -q2 - 2q2 - _q

4

41rEo

7q2 47rEO

Sol 4.2. For an angle 0 which is integer divisor of 180°, the required number of image charges is given by 360°

n =-- -l

0

Here 0 is 30°, hence 360° n=---l 30°

= 11 Sol 4.3. Let x be the horizontal axis and y be t he vertical axis. Place an image charge -q at (d,-d), q at (-d,-d) -q at

[email protected]

@Sk J ahiruddin, 2020

55

physicsguide CSIR NET, GATE

Electrostatics: Part-2

(-d,d) . Hence the force on q for this charge configuration

11 oo : 02 : 33 @Sk J ahiruddin, 2020

Electrostatics: Part-2

(-d,d) . Hence the force on q for this charge configuration 1 q2 F = - - -2 41rEo d 1 q2 41rEo d 2 1 q2 41rEo d 2

t he minus sign indicates that the force is towards the corner

1

2

(2v'2 - 1) 2 (8y'2)

47rEo -

IF

2

41rE08

Sol 4.4. For equilibrium the the equation of motion is 1

q2

41rEo (2d) q2 1

2

=mg

- - - - = d2 47rEO 4mg

d=

q 4 ✓~m-g-1rE-o

Sol 4.5. Sol 4.6. P lace an image charge e at (-d, 0,0) . Hence t he [email protected]

physicsguide CSIR NET, GATE

56

@Sk J ahiruddin , 2020

,

,



,_a

11

I

1

Electrostatics: Part-2

1



I

,rl

I"'\

I"'\\



11 oo : 02 : 3s [email protected]

physicsguide CSIR NET, GATE

56

@Sk J ahiruddin, 2020

Elect rostatics: P art-2

electric field at the point ( g, 0, 0) is _, 1 E =-

4?TEo

1 41TEo d2 9

9

lOe

d2 (1, 0, 0)

1TEo

Sol 4. 7. Since the V = 0 at z = 0 plane and there are no charges in z < 0 , so t he magnitude of electric field at z < 0

is 0 Sol 4.8. Let z is t he distance of the point charge from t he infinite grounded conducting plane at any t ime . By New-

t on's law

dv q2 1 m v - = - - - -2 dz 41TEo z v2

dz v= -

dt

=

q2 1 ---41TEo 2m q

1

1

z

d

-- 1

1

2

d- z

~~

z

Let

j [email protected]

57

physicsguide CSIR NET , GATE

T""l

,

I

,



T""\



II"'\

11 oo : 02 : 38 [email protected]

57

physicsguide CSIR NET, GATE

@Sk J ahiruddin , 2020

Electrostatics: Part-2



z

d 0

0

II

0

2

dz = 2dsin 0cos 0d0

Hence 1

z d- z 2

dsin 0 d - d sin 2 0

2

dz =

q 1 ~~

dt

1 2

41rEo 1r

2

2d

sin2 0d0

0

d

1T

2

=

q

2md 1

~~ qt J81rEomd

dt t

J21r 3 Eomd3

t= - - - q

Sol 4.9. The potential at the surface of the sphere is given by

Vo=

!

q

41rEo a q

V0 a = - 41rEo

Hence he potential at a point outside the sphere, at a [email protected]

58

physicsguide CSIR NET, GATE

11

00:02:41 q

V0 a= - 41rEo

Hence he potential at a point outside the sphere, at a [email protected]

physicsguide CSIR NET, GATE

58

@Sk J ahiruddin, 2020

Electrostatics: Part-2

tance r from the centre, is

l Vout= 41rEo r q

Voa r

Sol 4 .10. Since the sphere is grounded , we must find an image dipole for which potential at any point on the surface A

of the sphere is zero. Let us take that point as ak, and p' be the image dipole. Then the potential for these dipoles at A

ak

Sol 4 .11 . Let an image line charge per unit lenght - ,,\ runs parallel to x-axis at y = -d. Then the potential for any

pointy> 0 is V (x, y, z)

=

1 '),rr C

,,\ In ~

3

+

11 oo : 02 : 43 Sol 4.11. Let an image line charge per unit lenght - A runs parallel to x-axis at y = -d. Then the potential for any pointy> 0 is

V (x,y,z) = [email protected]

1 - A ln 21rEo 59

S+

s_

physicsguide CSIR NET, GATE

@Sk J ahiruddin , 2020

Electrostatics: Part-2

where s+ = (x 2+z 2 +(y-d) 2)½ and s_ = (x 2+z2 +(y+d) 2)½. t hen surface charge density on t he conducting plane is

oV(x ,y,z) oy a - A y - d Eo 21rEo (x2 + z2 + (y _ d)2) ~ - Ad a =-----(x2 + d2 + z2) a

y+d y=O

Sol 4.12. Since the inner cylinder is insulated , no electric field penetrate the conductor ,and Since t he outer cylinder is grounded no electric field penetrat e the out er conductor either . Hence d is the right answer. Sol 4.13. We have found in Sol 4.3. t hat

1

q2 FA= -8d2 (2v'2 - 1) 41TEQ

And we know

q2 1

FB = - - - -2 41TEo 4d

11 oo : 02 : 46 And we know

j [email protected]

physicsguide CSIR NET 1 GATE

60

@Sk J ahiruddin, 2020

Electrostatics: Part-2

Hence

= .9142 Sol 4.14. Then the potential for any point at a distance sfrom the axis is

1 s V (x, y , z ) = --\ln R 21TEQ

On t he surface of inner conductor 10 10 ,\ ln ( ;)

=

1 27rEo

,\ ln

2

5

1 ,\ 21rEo

Then

V (3.5mm )

1

= - -,\ ln 2 7rEO

10 \lr1/2\

ln

3.5 5 3.5 c;

11 oo : 02 : 48 Then 1

3.5 V (3.5mm ) = - -,\ In 2 7rEO 5 10 ln 3.5 ,\ ln ( 5

i)

= 3.90 Sol 4.15. Place image charges - 3Q and + Q at (0, 0, -d) and (0, 0, - 2d), respectively. The monopole monement for 61

[email protected]

physicsguide CSIR NET, GATE

Electrostatics: Part-2

@Sk J ahiruddin, 2020

t his charge configuration is zero. The dipole moment A

p = (3Qd - 2Qd + 3Qd - 2Qd)k "' = 2Qdk Hence t he dominating term in t he potential will be t he dipole term. So, for z > > d the electrostatic potent ial of t his charge configuration would approximately be V( z)

= 1 2Qd 41rEo z 2

Sol 4.16. Place image charges - 2Q and +Q at (- 1, 0, 0) and (-2, 0, 0), respectively. The monopole monement for

t his charge configuration is zero. The dipole moment

p=

(2Q - 2Q

+ 2Q -

2Qd)i

=0 So the dipole moment is also zero. The quadrupole domin::i.t.in P- t.Prm . Sn nnt.Pnt,i::i.l ::i.t. ( r..n.n1 fnr r. '> '> 1 rl PnPnrl~ nn

11 oo : 02 : s1 p=

(2Q - 2Q

+ 2Q -

2Qd)i

=0

So the dipole moment is also zero. The quadrupole dominating term. So potential at (x,0,0) for x > > l depends on x as Sol 4.17. Let q' be the charge on the inner shell. So t he

potential of outer shell is

62

j [email protected]

physicsguide CSIR NET I GATE

@Sk J ahiruddin , 2020

Electrostatics: Part-2

Since the the inner shell is grounded potential on the inner •

IS

0=

~-~ +q+ q'~

q' 41rEo

0=

q'

r1

q'

q

q

l

2

r2

q'

+ + r1 r 41rEo r r 2

I

41rEo

r2

2

r1

= --q r2

Sol 4.18. The magnetic field inside and on t he plane of t he superconductor is zero. Using the method of image charges

we replace the superconductor with an infinitely long wire parallel to the x -axis is kept at z = -d and carries a current I in the negative x direction . The magnetic field for t his current configuration at a point (x, y, z > 0) is ".

"

i X S-

11 oo : 02 : s4 parallel to the x -axis is kept at z = -d and carries a current I in the negative x direction . The magnetic field for t his current configuration at a point (x, y, z > 0) is ".

"

S+

i X

".

"

i X S-

S_

S+

3+ (z -

l

µof

3 + y k (z + d) 3- y k -----+----

21r

x (y

d) k)

3+ (z + d) k)

µ 0I 21r

i

y 2 + (z - d) 2

x (y

y 2 + (z

+ d) 2

-(z - d) y2

+ (z -

d) 2

y2

+ (z + d) 2

where s+ = y] + (z - d)k and s_ = y] + (z + d)k) is position vector of any point from the upper and lower wire [email protected]

@Sk J ahiruddin ) 2020

respectively.

63

physicsguide CSIR NET ) GATE

Electrostatics: Part-2

11 oo : 02 : s6

64

[email protected]

physicsguide CSIR NET, GATE

@Sk J ahiruddin, 2020

5

Electrost atics: Part-2

Boundary Value Problems

Prob 5.1. Equipotential surface corresponding to a particular charge distribution are given by 2

2

2

4x + (y - 2) + z = ¼

E

, where the values of ½ are constants. The electric field at t he origin is [JAM 2011] (c) = 4i) (d) (a) = o (b) = 2x

E

E = -4iJ

E

E

11 oo : 02 : sg (a)

E=

E =-411

o

(b)

E

2x

(c)

E

4y

(d)

Prob 5.2. A spherical conductor of radius a is placed in a

E

uniform electric field = Eok. The potential at a point P(r, 0) for r > a, is given by ( r, 0) = constant - E 0 rcos0

+

E 0a 3

cos0

r2 where r is the distance of P from the centre O of the sphere and 0 is the angle OP makes with the z-axis. [GATE 2011] p

9

j [email protected]

-k

physicsguide CSIR NET, GATE

65

@Sk J ahiruddin, 2020

Electrostatics: Part-2

The charge density on the sphere at 0 = 30° is

(d)

EoEo 2

Prob 5.3. Consider a spherical shell with radius R such

t hat the potential on the surface of the shell in spherical coordinates is given by,

V(r

=

R , 0, )

=

2

Vocos 0

11 oo : 03 : 01 Prob 5.3. Consider a spherical shell with radius R such t hat the potential on the surface of the shell in spherical coordinates is given by, 2

V(r = R , 0, cp) = Vocos 0

where t he angle Bis shown in the figure.There are no charges except for those on the shell. The potential outside the shell at the point Pa distance 2R away from its centre C (see figure) is [TIFR 2017] I

I I

p

C

(a) V = ~0 (1 + cos 2 0) (b) V = 1t-(l + 2cos 2 0) (c) V = ~(l - cos 2 0) (d) V = ~(-2 cos 0 + cos 30) [email protected]

66

physicsguide CSIR NET, GATE

@Sk J ahiruddin, 2020

Electrostatics: Part-2

2

Prob 5.4. Solving Poisson's equation \/ cp = -Po/t=.0 for the electrostatic potent ial cp(x) in a region with a constant charge density p0 , two students find different answers, viz.

and The reason why these different solutions are both correct is because [TIFR 2014]

11 oo : 03 : 04 and The reason why these different solutions are both correct is because [TIFR 2014] (a) space is isotropic and hence x and y are physically equivalent . (b) we can add solutions of the Laplace's equation to both

c/J1(x) and c/J2(x) (c) the electrostatic energy is infinite for a constant charge density (d) t he boundary conditions are different in t he two cases.

Pro b 5. 5. For a scalar function cpsatisfying the Laplace equat ion, "\lcp has [GATE 2013] (a) zero curl and non-zero divergence (b) non-zero curl and zero divergence (c) zero curl and zero divergence (d) non-zero curl and non-zero divergence Prob 5.6. A charge distribution has the charge density given 67

[email protected]

physicsguide CSIR NET, GATE

@Sk J ahiruddin , 2020

Electrost atics: P art-2

by p = Q{o(x-x 0 )- o(x+x 0 ) }. For this charge distribution t he electric field at (2x 0 ,0, 0) [GATE 2013]

(a)

2Qx

91rcox5

(b)

Qx

41rcox5

(c)

Qx

41rcox5

(d)

Qx

l 61rcoX6

Prob 5. 7. A static, spherically symmetric charge distribut ion is given by p(r) = 1e-Kr where A and K are positive

11 oo : 03 : 06 c r1c

(a)

2Q A 9n d, where d is t he spacing between the plates, is charged to a potential V and then disconnected from t he

charging circuit. If now the plates are slowly pulled apart (keeping them parallel) so that t heir separation is increased [TIFR 2013] from d to d t he work done will be 1

,

1rcor ( )

a

2v 2

2d 2v2

1rEor ( C) 2d

(l _ .4.) d' , ( 4_ _ d

l

)

(b)

2v 2 , 1rcor 4_ 2d d

(d) 1rEor

2v2

2d

.4. d'

Prob 1.3. A parallel plate capacitor with square plates of

11 oo :oo :oa 2d ( C)

1rEor

2

2d

V

2

( d' _

d

)

1

d) (

1rEor

2v2

2d

.!i d'

Prob 1.3. A parallel plate capacitor wit h square plates of [email protected]

@Sk J ahiruddin ) 2020

3

physicsguide CSIR NET ) GATE

Capacitance and Dielectrics

side l m separated by l micro meter is filled wit h a medium of dielectric constant of 10. If t he charges on t he two plates are lC and - l C, the voltage across t he capacitor is __ _kV. (up to two deceimal places)(Eo = 8.854 x 10- 12F/m) [GATE 2017]

Prob 1.4. A rectangular piece of dielect ric material is in-

serted part ially into t he (air) gap between t he plates of a parallel plate capacitor. The dielectric piece will [NET Dec 2017] (a) remain stationary where it is placed (b) be pushed out from the gap between the plates (c) be drawn inside the gap between the plates and its velocity does not change sign (d) execute an oscillatory motion in the region between t he plates

Prob 1.5. A parallel plate capacitor is formed by two circular conducting plates of radius a separated by a distance d, where d < < a . It is being slowly charged by a current t hat is nearly constant . At an instant when the current is

11 oo : oo : 11 Prob 1.5. A parallel plate capacitor is formed by two circular conducting plates of radius a separated by a distance d, where d < < a. It is being slowly charged by a current t hat is nearly constant. At an instant when t he current is l , the magnetic induction between the plates at a distance [email protected]

physicsguide CSIR NET, GATE

4

@Sk J ahiruddin, 2020

C apacitance and Dielectrics

a/2 from t he centre of the plate, is

(a) µal

(b) µal

1ra

21ra

(c) µal a

[NET Dec 2016]

(d) µ0 1 41ra

Prob 1.6. Two conductors are embedded in a material of conductivity 10- 4 ohm-m and dielectric constant E = 80E 0 . The resistance between the two conductors is 106 ohm. What is the capacitance (in pF) between the two conductors? Ig[JEST 2018] nore the decimal part of the ans. (Next Two questions are linked) In a hydrogen atom, consider that t he electronic charge is uniformly distributed in a spherical volume of radius a(= 0, 5 x 10- 10m ) around the proton. The atom is placed in a 5 uniform electric field E = 30 x 10 V/m . Assume that the spherical distribution of the negative charge remains undistorted under the electric field. [GATE 2012]

Prob 1. 7. In t he equlibrium condition, the sepatration between t he positive and the negative charge centers is? (a) 8.66 x 10-16 m (b) 2.60 x 10- 15m

11 oo : oo : 13 Prob 1. 7. In t he equlibrium condition, the sepatration between t he posit ive and the negative charge centers is? 16

15

(a) 8.66 x 10- m (b) 2.60 x 10- m (c) 2.60 x 10-16m (d) 8.66 x 10- 15 m

Prob 1.8. The polarizability of t he hydrogen atom in unit j [email protected]

5

physicsguide CSIR NET, GATE

@Sk J ahiruddin , 2020

of (C

2

m/N)

C apacitance and Dielectrics

is

(a) 2.0 x 10- 40 m (b) 1.4 x 10-41 m (c) 1.4 x 10-40 m (d) 2.0 x 10-39 m

Prob 1.9. A conducting sphere of radius RA has a charge Q. It is surrounded by a dielectric spherical shell of inner radius RA and outer radius RB ( as shown in the figure below) having electrical p ermitivity c:(r ) = c: 0r [JAM 2006]

(a) Find t he surface bound charge density at r

= RA

(b) Find the total electrostatic energy stored in t he dielectric (region B)

11 oo : oo : 16 (a) Find t he surface bound charge density at r = RA (b) Find the total electrostatic energy stored in t he dielectric (region B) Prob 1.10. A dielectric sphere is placed in a uniform elec-

t ric field directed along the positive y axis. Which of the following represents t he correct equipotential surfaces . [GATE 2008]

[email protected]

6

physicsguide CSIR NET, GATE

Capacitance and Dielectrics

@Sk J ahiruddin, 2020

(b)

(a)

.---.... --..

.. ----.

--------- ....._____ _ -·-----·

(d)

(c)

.........

--------

-.... --. --.

Prob 1.11. The space between two plates of a capacitor carrying charges +Q and - Q is filled with two different

materials, a shown in t he figure. Across t he interface of t he two dielectric materials, which one of t he folllowing statements is correct? [GATE 2015]

11 oo :oo :1a materials, a shown in t he figure. Across the interface of the two dielectric materials, which one of t he folllowing statements is correct? [GATE 2015]



-



• •



-

(a) E and D are cont inuous jahir@physicsguide. in

7

@Sk J a hiruddin , 2020

physicsguide CSIR NET , GATE

Capacitance and Dielectrics

(b) E is continuous and D is discontinuous (c) E- is discontinuous and D- is continuous (d) E and D are discontinuous. Pro b 1.12. Two infinitely extended homogeneous isotropic dielectric media (medium-I and medium-2 wit h dielect ric constant E1 / E2 = 2 and E2 / Eo = 5, respectively) meet at t he z = 0 plane as shown in t he figure. A uniform electric field exists everywhere. For z > 0, t he electric field is given by E 1 = 2i - 3j = 5k. The interface separating the two media is charge free. The electric displacement vector in t he [GATE 2012] medium-2 is given by. A

A

A

11 oo : oo : 21 arge ree. medium-2 is given by.

[GATE 2012]

medium- I

medium - 2

(a)

D2 =

z- 0

Eo[l02 + 15] + 10k]

(b)

10k] (c) D2 = Eo[42 - 6] + 10k] j [email protected]

(d)

D2 =

D 2 =

Eo [l02 - 15] +

Eo[42 + 6] + 10k]

physicsguide CSIR NET 1 GATE

8

Capacitance and Dielectrics

@Sk J ahiruddin, 2020

Prob 1.13. Consider an electromagnetic wave at the int erface between two homogeneous dielectric media of dielectric constants E1 and E2 . Assuming E2 > E1 , and no charge on the surface, t he electric field vector E and the displacement vector D in the two media satisfy t he following inequalities -I

-I

[NET June 2014]

E2 < IE1I and (c) IE2I < IE1 and (d) E2 > IE1I and D2 < D1 (b)

-I

-,,

-I

-I

-J

-,,

Prob 1.14. Suppose t he yz - plane forms a chargeless bound, , ,. ,. •

I





I



11 oo : oo : 24 ~

~

~

-+

(c) IE2I < IEi and D2 > Di -+

-+

-+

-+

(d) E 2 > IEi l and D2 < Di Prob 1.14. Suppose t he yz- plane forms a chargeless bound-

ary between two media of permitivities Eze ft and Eriglit where Eze ft : Eriglit = 1:2. If t he uniform electric field on t he left -+ is E zeft = c( i + j + k) ( where c is a constant) , t hen t he _, electric field on t he right E right is [NET June 2015] (a) c(22 + 3+ k) (b) c(l + 23 + 2k) (c) c(l / 22+] + k) (d) c(2i + 1/ 2] + 1/ 2k) A

A

A

Prob 1.15. Suppose yz plane forms t he boundary between two linear dielectric media I and H with dielectric constant

3 and E2 = 4 respectively. If t he electric field in region 1 at t he int erface is given by E i = 4x + 3i) + 5z , t hen the Ei

=

[email protected]

9

physicsguide CSIR NET, GATE

Capacitance and Dielectrics

@Sk J ahiruddin , 2020

electric field E 2 at the interface in region 2 is [JEST 2016]

(a) 4x+3i)+5z (d) 3x

(b) 4x+3/ 4i)5/ 4z

(c) -3x +3i) +5z

+ 3i) + 5z

Prob 1.16. A ray of light inside Region 1 in the xy plane is incident at the semicircular boundary that carries no free

charges. The electric field at the point P (r 0 , 1r / 4) in plane _, polar coordinates is E i = 7e"-r - 3e4>, where er and e4> are the unit vectors. The emerging ray in Region 2 has the electric _, field E 2 parallel to x- axis. If Ei and E2 are the dielectric

11 oo : oo : 26 ary a carries no ree charges. The electric field at the point P (r 0 , 1r / 4) in plane -+ polar coordinates is E 1 = 7e"-r - 3e, where er and e are the unit vectors. The emerging ray in Region 2 has the electric -+ field E 2 parallel to x- axis. If E1 and E2 are the dielectric constants of Region 1 and Region 2 respectively then E1 / E2 is??? [GATE 2014]

r



0 1 - - - - + - - - - - - -... C

Region

c •. Region_

Pro b 1.1 7. The half space regions x > 0 and x < 0 are filled with dielectric media of dielectric constants E1 and E2 j [email protected]

physicsguide CSIR NET I GATE

10

Capacitance and Dielectrics

@Sk J ahiruddin, 2020

respectively. There is a uniform electric field in each part. In the right half, t he electric field makes an angle 01 to the interface. The corresponding angle 02 in t he left half satis[NET June 2016] fies

xO

11 oo : oo : 29 xO

-----------

sin 0 2 = E2 sin 01 (c) E1 tan 01 = E2 tan 0 2 (a)

E1

(b) E1 tan 02 = E2 tan 01 (d) E1 sin 01 = E2 sin 02

Prob 1.18. Two semi-infinite slabs A and B of dielectric constant EA and EB meet in a plane interface, as shown in t he figure below. [TIFR 2016]

[email protected]

11

physicsguide CSIR NET ) GATE

Capacitance and Dielectrics

@Sk J ahiruddin ) 2020

.-l B

-- - - _( - - - .

(a)

e.4 e,.

- -

. .. .

cos0A = EA /EBCOS 0B (b) sin0A = EA/EB Sin0B f~\ -i- ~-n ~ _ ; _ -i- ~-n f ;i \ ~:-n ~ _ ;_ ~:,~n

11 oo : oo : 31 .

(a) cos0A

- -

.

(b) sin0A

= EA/EECOS0E

(c) tan0A = EA/EE tan0E

..

= EA/EESin0E

(d) sin0A = EE/EA sin0E

Prob 1.19. A charge q is placed at the center of an other-

wise neutral dielectric sphere a and relative permitivity Er. 2 We denote the expression q/ 41rE0 r by E (r). Which of the following st at ements is false? [JEST 2013] (a) The electric field inside the sphere, r < a, is given by

E(r)/Er (b) The field outside the sphere r > a, is given by E(r) (c) The total charge inside a sphere of radius r > a is given by q (d) The total charge inside a sphere of radius r < a is given by q. Prob 1.20. A cylindrical rod of length L and radius r made of an inhomogeneous dielectric, is placed with its axis along

t he z direction with one end at the origin as shown below. [GATE 2009] 12

[email protected]

physicsguide CSIR NET, GATE

Capacitance and Dielectrics

@Sk J ahiruddin, 2020

X

I I

r

. > - - t - _ _ , _ _ - - t - - t - --+

=

11 oo : oo : 34 X

I I

r

. , > - - l f - - ' - - - - t - - - - t - -- .

-

=

L

If t he rod carries a polarization P = (5z volume bound charge inside the dielectric is -

(a) Zero

(b) 101rr2 L

2

+ 7) k, A

the

(c) -51rr 2 L

Prob 1.21. A dielectric sphere of radius R carries a polarization P = kr 2f, where k is a constant and r is t he distance from t he center. Take t he system spherical polar coordinate.

-

[GATE 2006]

(i) The bound volume charge density inside t he sphere at a distance r from the center is 2

(a)-4kR

3

(b)-4kr (c)-4kr (d)-4kr (ii) The electric field inside t he sphere at a distance d from t he center is

(a) -kd2 f / Eo

(c) - kd 20/ Eo j [email protected]

@Sk J ahiruddin, 2020

(b) -kR 2 f / Eo

(d) -kR 2 0/ Eo 13

physicsguide CSIR NET, GATE

Capacitance and Dielectrics

Prob 1.22. A sphere of radius R carries a polarizat ion P = kr, where k is a constant and r is measured from t he center of the sphere.

[GATE 2007]

00: 00: 37 apacitance and Dielectrics

-I

Prob 1.22. A sphere of radius R carries a polarization P = kr, where k is a constant and f is measured from t he center

[GATE 2007]

of the sphere.

(i) The bound surface and volume charge densit ies are given, respectively, by

(a) -k f1 and 3k

(b) -k f1 and - 3k (c) k f1 and - 41rkR (d) -k f1 and 41rkR (ii) The electric field E at a point r outside the sphere is given by (a) 0 (b) kR(R2 - r2) f -I

Eor3

(c) k R (R

2

2

-

(d) 3k(r - R ) f

r )f

Eor5

Eor4

Prob 1.23. A spherical shell of inner and outer radii a and b, respectively, is made of a dielectric material with frozen polarization P (r) = whe1~e k is a constant and r is the distance from the its center. The electric field in the region

;r,

a< r < bis. k (a) E = - f Eor

(c)

E= o

[JEST 2015]

(b)

E= -

(d) E =

[email protected]

@Sk J ahiruddin, 2020

k f Eor

k f

Eor2

14

physicsguide CSIR NET, GATE

Capacitance and Dielectrics

Prob 1.24. A long solid dielectric cylinder of radius a is

11 oo : oo : 39 @Sk J ahiruddin, 2020

Capacitance and Dielectrics

Prob 1.24. A long solid dielectric cylinder of radius a is permanently polarized ao that the polarization is every-

where radially outward, with a magnitude proportional to .... t he distance from the axis of the cylinder, i.e P = ½Porf. t he bound charge density in t he cylinder is given by [TIFR 2016]

(a) - Po

(b) Po

(c) - Po/2

(d) Po/2

Prob 1.25. An infinite conducting slab kept in a horizontal plane carries a uniform charge density er. Another infinite

slab of thickness t, made of a inner dielectric material of dielectric material of dielectric const ant k, is kept above the conducting slab. The bound charge density on t he upper [GATE 2016] surface of t he dielectric slab is (a) cr/2k (b) cr/k (c) cr(k - 2) / 2k (d) cr(k - l )/k Prob 1.26. Assume that z = 0 plane is t he interface between two linear and homogenous dielectrics (see figure).

The relative permitivities are Er = 5 for z > 0 and Er = 4 .... for z < 0. The electric field in the region z > 0 is E 1 = (3i - 5] + 4k)kV/m . If t here are no free charges on t he interface, the electric field in the region z < 0 is given by [JAM 2010]

[email protected]

@Sk J ahiruddin , 2020

15

physicsguide CSIR NET, GATE

Capacitance and Dielectrics

11 oo : oo : 42 15

[email protected]

physicsguide CSIR NET, GATE

Capacitance and Dielectrics

@Sk J ahiruddin, 2020

s, - s

..--o -

E2 = (3/4i - 5/ 4] + k)kV/m (b) E2 = (3i - 5] + k)kV/m (c) E2 = (3i - 5] - 5k)kV/ m (d) E2 = (3i - 5] + 5k)kV/m (a)

Prob 1.27. A conducting spherical shell of radius shell of radius R 1 carries a total charge Q. A spherical layer of a linear , homogenous and isotropic dielectric constant K and outer radius R 2 ( R 1 ) covers the shell as shown in t he figure. [JAM 2010]

(a) Find t he electric field and polarization vector P inside the dielctric. From this P calculate t he surface bound charge density, j [email protected]

a b.

on the outer surface of t he dielectric 16

physicsguide CSIR NET, GATE

11 oo : oo :44

side the dielctric. From this P calculate the surface bound charge density,

on the outer surface of t he dielectric

a b.

j [email protected]

16

physicsguide CSIR NET, GATE

C apacitance and Dielectrics

@Sk J ahiruddin , 2020

layer and the volume bound charge density Pb, inside the dielectric. (b) Calculate t he electrostatic energy stored in the region

R1 < r

~

R2

Prob 1.28. a Two concentric, conducting spherical shells of raddi R 1 and R 2 (R 1 < R 2 ) are maintained at potent ials

V1 and ½, respectively. Find t he potential and electric field [JAM 2012] in t he region R 1 < r < R 2 .

b A polarized dielctric cube of side 1 is kept on x - y plane as shown. If t he polarization in the cube is P = kxx, where k is posit ivwe constant, then find all t he bound surface charge densities and volume charge density.

-

G\1---------,. F

••



B X

Prob 1.29. A conducting solid sphere of radius a, carrying a charge q is kept in a dielectric of dielectric constant k,

such that half of the sphere is surrounded by t he dielectric

11 oo:oo:47 Prob 1.29. A conducting solid sphere of radius a , carrying a charge q is kept in a dielectric of dielectric constant k,

such t hat half of the sphere is surrounded by t he dielectric [email protected]

17

physicsguide CSIR NET, GATE

Capacitance and Dielectrics

@Sk J ahiruddin, 2020

as shown in the figure. Find t he surface charge densit ies in t he upper and lower hemispherical surfaces. [JAM 2013] q

Prob 1.30. In a parallel plate capacitor t he distance between the plates is 10 cm. T wo dielectric slabs of t hickness

5 m each and dielectric constant K 1 = 2 and K 2 = 4 respect ively, are inserted between t he plates. A potential of lOOV is applied across t he capacitor as shown in t he figure. The value of the net bound surface charge density at t he [JAM 2014] interface of t he two dielectric is

IOcrn

K2 = 4 K1 = 2

(a) -200to / 3

(b) - lOOto/ 3

-- I00 V

(c) - 250Eo

(d) -2000to/3

11 oo : oo : 49

(a) - 200Eo/3

(b) - 100Eo/ 3

(c) - 250Eo

(d) -2000Eo/3

Prob 1.31. MSQ: A unit cube made of a dielectric material has a polarization P = 3i + 4j units. The edges of -

[email protected]

@Sk J ahiruddin , 2020

A

18

A

physicsguide CSIR NET, GATE

Capacitance and Dielectrics

t he cube are parallel to t he cartesian axes. Which of the following statements are t rue? [JAM 2015] (a) The cube carries a volume bound charge of magnit ude 5 units. (b) There is a charge of magnitude 3 units on both t he surfaces parallel t o t he y - z plane. (c) There is a charge of magnit ude 4 units on both the surfaces parallel t o t he x - z plane. (d) There is a net non-zero induced charge on the cube. Prob 1.32. MSQ: Consider a spherical dielctric material of radius a centered at origin. If the polarization vector P =

-

P0e"'x, where Po is a constant of appropriate dimensions, t hen (e"'x, ey, ez ) are unit vectors in Cartesian coordinate sytstem. [JAM 2016] (a) The bound volume charge density is zero. (b) The bound surface charge density is zero at (0, 0, a) (c) The electric field is zero inside t he dielectric. (d) The sign of the surface charge density changes over the surface.

11 oo : oo : s2 (b) The bound surface charge density is zero at (0 , 0,a) (c) The electric field is zero inside t he dielectric. (d) The sign of the surface charge density changes over t he surface. Prob 1.33. T wo parallel plate capacitors, separated by dist ances x and 1. l x respectively, have a dielectric material

of dielectric constant 3. 0 inserted between t he plates and j [email protected]

19

physicsguide CSIR NET 1 GATE

Capacitance and Dielectrics

@Sk J ahiruddin, 2020

are connected to a battery of voltage V . T he differences in charge on t he second capacitor compared to t he first is [NET June 2016] (a) 66% (b) 20% (c) - 3.3% (d) - 10% Prob 1.34. An atom of atomic number Z can be modeled

as a point posit ive charge surrounded by a rigid uniformly negatively charged solid sphere of radius R. The electric polarisability a of t his system is defined as a = P.J where PE is t he dipole moment induced on application of electric field E which is small compared to the binding elect ric field inside the atom. It follows t hat a = [TIFR 2018]

Prob 1.35. MSQ: A dielectric sphere of radius R has con= P0z so t hat t he field inside the stant polarization

sphere is

>

E iri

P

= - ~~ z T hen which of t he following is( are)

11 oo : oo : 55 Prob 1.35. MSQ: A dielectric sphere of radius R has constant polarization = P0z so that the field inside the

sphere is correct?

> E in

P

- £:~z Then which of the following is(are) [JAM 2017]

(a) The bound surface charge density is P0 cos 0 (b) The electric field at a distance r on t he z - axis varies as r~ for r > > R [email protected]

20

physicsguide CSIR NET, GATE

@Sk J ahiruddin, 2020

Capacitance and Dielectrics

(c) The electric potent ial at a distance 2R on the z-axis is PoR 12Eo

(d) The electric field outside is equivalent to that of a dipole at t he origin.

Prob 1.36. Two dielectric spheres of radius R are separated by a distance a such that a >> R. One of the spheres

(sphere 1) has a charge q and t he other is neutral. If the linear dimensions of the systems are scaled up by a factor two, by what factor should the charge on the sphere 1 be changed so that the force between t he two spheres remain unchanged? [JEST 2018]

(a) 2

(b) 4\1'2

(c) 4

(d) 2\1'2

Prob 1.37. A dielectric interface is formed by two homo-

11 oo :oo : s1 [JEST 2018]

unchanged?

(a) 2

(b) 4v'2

(c) 4

(d) 2v'2

Prob 1.37. A dielectric interface is formed by two homo-

geneous and isot ropic dielectrics 1 and 2 with dielectric constants 4/ 3 and 1 respectively and it carries no residual free charge. A linearly polarized electromagnetic wave is incident on t he interface from dielectric 1 at a point where the 1 10 ,_ unit normal to t he surface is ii = ( v 3i + k ) pointing into 2 t he dielect ric 1. The incident wave, which is incident from 1 into 2, just before it reaches t he interface, has electric vector j [email protected]

physicsguide CSIR NET I GATE

21

Capacitance and Dielectrics

@Sk J ahiruddin , 2020

[TIFR 2019] E.. .1

. t + y'Ei z = "E i o exp iw C

where E 0 is a real constant . The electric vector just after it er ..... from E 1 by an angle (a)

1r

/6

(c) sin-

(b) tan1

1

v'l9

2 19

1

2

v'3

5 3

(d) csc- 1 3

4 -19

Prob 1.38. During the charging of a capacitor C in a series R C circuit, the typical variations in t he magnitude of t he

charge q( t) deposited on one of the capacitor plates, and t he current i (t) in the circuit, respectively are best represented ,

11 oo : 01 : oo Prob 1.38. During the cha1~ging of a capacitor C in a series

RC circuit, the typical variations in t he magnit ude of t he charge q(t) deposit ed on one of the capacit or plates, and t he current i (t) in the cir·cuit, respectively are best represented by [JAM 2019]

[email protected]

22

physicsguide CSIR NET ) GATE

@Sk J ahiruddin ) 2020

Capacitance and Dielectrics

Fig. I

q



1

t

0

Fig. III

q

0

t

(a) Fig. I and Fig. II (c) Fig. III and Fig. II

0

Fig. II

t

Fig. IV



l

0

t

(b) Fig. I and Fig. IV (d) Fig. III and Fig. IV

P rob 1.39. A oarallel olat e ca.o acitor. with 1cm seoara.t ion

11 oo : 01 (a) Fig. I and Fig. II (c) Fig. III and Fig. II

: 02

(b) Fig. I and Fig. IV (d) Fig. III and Fig. IV

Prob 1.39. A parallel plate capacitor, with 1cm separation between t he plates, has two layers of dielectric with dielec-

t ric constants K = 2 and K = 4, as shown in the figure below. If a potential difference of lOV is applied between the plates, t he magnitude of the bound surface charge density (in units 2 of C/m ) at the junction of the dielectrics is [NET D ec 2019]

[email protected]

@Sk J ahiruddin, 2020

23

physicsguide CSIR NET, GATE

Capacitance and Dielectrics

=2

1cm

(a) 250E0

• 4

(b) 2000c0 / 3

(c) 2000E 0

( d)

200c0 /3

Prob 1.40. Which one of the following relations determines

t he manner in which the electric field lines are refracted

11 oo : 01 : os (a) 250E0

(c) 2000E 0

(b) 2000E0 / 3

( d)

200E0 /3

Prob 1.40. Which one of the following relations determines

t he manner in which the electric field lines are refracted across t he interface between two dielectric media having dielectric const ants c 1 and e: 2 (see figure)? [GATE 2020]

(A) E1 sin 0 1 =

E 2 sin 02

(C) c 1 tan 0 1 = c 2 tan 0 2 j [email protected]

(B ) E1 cos 01

= E2 cos 02

(D ) e: 1 cot 0 1 = e: 2 cot 0 2

physicsguide CSIR NET, GATE

24

C apacitance and Dielectrics

@Sk J ahiruddin , 2020

Prob 1.41. What is t he charge stored on each capacitor C 1 and C 2 in t he circuit shown in the given figure? [JEST

2020] eo 2

l O

llf'

1 i,f

- - ,....------11.,_____. Cl

Cl

+ 11 V

lO

11 oo :01 : oa eo

)0

2 µF

--,-- - - - - 1 - Ct

Cl

+ 12 V

(A) 6µC , 6µC

(C) 3µC, 6µC

lO

(B) 6µC , 3µC (D) 3µC , 3µC

Prob 1.42. MSQ: A spherical dielectric shell with inner where radius a and outer radius b, has polarization P = -4r, r k is a constant and f is t he unit vector along the radial direction. Which of the fallowing statements is/ are correct? [JAM 2020] (A) The surface density of bound charges on the inner and outer surfaces are - k and + k respectively. The volume density of bound charges inside the dielectric is zero. (B) The surface density of bound charges is zero on both t he inner and outer surfaces. The volume density of bound charges inside t he dielectric is +k [email protected]

@Sk J ahiruddin , 2020

25

physicsguide CSIR NET, GATE

Capacitance and Dielectrics

(C) The surface density of bound charges on the inner and outer surfaces are :~ and -ffe respectively. The volume density of bound charges inside the dielectric is zero. (D) The surface density of bound charges is zero on both t he inner and outer surfaces. The volume density of bound charges inside t he dielectric is 41r(b~k_a3 )

11 oo : 01

: 1o

sity of bound ch arges inside the dielectric is zero. (D) The surface density of bound ch arges is zero on both t he inner and outer surfaces. The volume density of bound charges inside t he dielectric is

41r(b~k_a3 )

Prob 1.43. Using a battery, a lOpF capacitor is charged to 50V and then the battery is removed. After that, a second uncharged capacit or is connected to the first capacitor in parallel. If the final voltage across the second capacitor is 20V, its capacitance is - - - - - - - - - - pF? [JAM

2020]

Prob 1.44. A sinusoidal voltage of the form V (t) = V0 cos(wt) is applied across a p arallel plate capacitor placed in vacuum. Ignoring the edge effects, the induced emf within the region b etween the capacitor plates can b e expressed as a power series in w . The lowest non-vanishing exponent in w is - - - - - - - - - [GATE 2020]

jahir@physicsguide. in

26

@Sk J ahiruddin , 2020

1.1

physicsguide CSIR NET, GATE

Capacitance and Dielectrics

Ans keys

1.1. 48 nF

1.4. d

1. 7.

1.2. b

1.5. d

1.8. b

C

11 oo : 01 1.1

: 13

Ans keys

1.1. 48 nF

1.4. d

1. 7.

1.2. b

1.5. d

1.8. b

1.3. 11 .30

1.6. 007

1.9. (a)

Q2

RA - l Q

,,, , (b) 4

R~

/I

2

47rE0

l

1

1

3

R3B

R~

C

1.10. d

1.16. 2.35

1.22. (i) b, (ii) a

1.11.

C

1.17.

C

1.23. b

1.12. b

1.18.

C

1.24. a

1.13.

C

1.19. d

1.25. d

1.14.

C

1.20. d

1.26. d

1. 21. (i) b , (ii) a

1.15. d 1.27. (a)

E =

~ f 2 41r Ear Q

41rE0 r

j [email protected]

for R1

< r < R2

f for r > R2 2

27

physicsguide CSIR NET 1 GATE

Capacitance and Dielectrics

@Sk J a hiruddin , 2020

p =

(k-l ) Q f· k 41rr 2 '

O"l,

=

(k - 1) Q

.

.

-r-.'l ;

11 oo : 01

: 1s

@Sk J ahiruddin , 2020

p =

(k-l ) Q

41rr 2

k

Capacitance and Dielectrics

f· '

at t he outer surface

At the inner surfacce Pb =

_ _ 29 1 • • ab -

1 0 (b) _Q __ l - k2 41rE5R§

EOXeVo R '

Vo / R) , on northern hemisphere ( Eo Vo / R) (1 + Xe), on southern hemisphere ( Eo

1.30. a

1.35. a,c,d

1.40. d

1.31. b ,c

1.36. b

1.41. a

1.32. a,b ,d

1.37.

1.42. C

1.33. d

1.38. a

1.43. 15

1.34.

1.39. b

1.44. 2

C

[email protected]

@Sk J ahiruddin , 2020

1.2

solution

C

28

physicsguide CSIR NET, GATE

Capacitance and Dielectrics

11 oo :01

:1

a

@Sk J a hiruddin , 2020

1.2

Cap acitance and Dielectrics

solution

Sol 1.1. This is Jackson Chapter 1 problem no 7.

Two long, cylindrical conductors of radii a 1 and a 2 are parallel and separated by a distance d, which is large compared with either radius. Show that t he capacitance per unit length is given approximately by

C

d In a

~ 1fEo

-l

where a is the geometrical mean of the two radii. Use t his formula. Here d = 20 cm. a = 2 cm. Total capacitance approximately

C ~ 3.14 x 8.85 x 10-

12

x 2.3 x 750

We get approximately 48 x 10- 9 F

which does not match any ans. Sol 1.2. When t he capacitor is charged to a potential V and seperation between the plates is d, then , capacitance

C= EoA d j [email protected]

@Sk J a hiruddin , 2020

29

physicsguide CSIR NET , GATE

Capacitance a nd Dielectrics

11 oo : 01 [email protected]

: 21

29

physicsguide CSIR NET, GATE

@Sk J ahiruddin , 2020

Capacitance and Dielectrics

and, on the other hand, when t he seperation is d ', capacitance is C' = EoA

d'

. Now, when t he batte1·y is fully charged and t hen removed, no. of charges remain t he same on the capacitor, so,

Q connected = Q removed

Or, CV= C'V' Hence, V

V'

d

d'

or,

V' = ~d' d Now, energy stored in a capacitor, as we know,

Hence,work done to increase the seperation from d to d' is,

Or, W = !(CV)V' - ~(CV )V 2

[email protected]

2

30

physicsguide CSIR NET, GATE

11 oo : 01

: 23

Or,

W = !(CV)V' - ! (CV )V 2

2

j [email protected]

physicsguide CSIR NET 1 GATE

30

Capacitance and Dielectrics

@Sk J ahiruddin, 2020

Or,

w = 2 cv v - v ) 1

(

I

Hence

w = !cv 2 (d' 2

W

=

Eo1rr

2v

- 1)

d 2 d'

(- -

1)

d

2d

Hence option (c) is correct. Sol 1.3. The potential difference between the plates in the

presence of dielectrics

10

=

X

8.854

11.3

3

X

10

X

10-12

Sol 1.4. You have t o picturize t he problem. The field out-

side the capacitor go from positively charged plat e to negat ively charged plat e, and in t his processes t hese fields polarize the dielectric and induces negative bound surface charge on the upper surface of the dielectric. So the dielectrics

11 oo : 01

: 26

tively charged plate, and in t his processes t hese fields polarize the dielectric and induces negative bound surface charge on the upper surface of the dielectric. So the dielectrics

31

[email protected]

physicsguide CSIR NET, GATE

Capacitance and Dielectrics

@Sk J ahiruddin , 2020

always gets pulled inside the capacitor, and executes an oscillatory motion. Sol 1.5. From Maxwell's equation _,

_, \7

X

_, aE B = µoEo at

The electirc field

E = a(t) Eo

Q(t) 1ra2Eo

8E

dQ 1ra2Eo dt 1

at aE 8t

I 1ra2 Eo A

Magnetic field is along cp . We take a circular loop of radius a / 2 between the plates. Hence

_, _,

B · dl

= µoEo

I

1r 2 1ra Eo 2

a I B21r- = µ oEo2 4Eo B = µo f Jlnrrr

a

2

11 oo :01 : 2a ... ... B · dl

=

µ oEo

I

1r 1ra Eo 2 I 2

a B21r- = µoEo2 4Eo B = µo f 41ra

Sol 1.6. 32

j [email protected]

physicsguide CSIR NET I GATE

Capacitance and Dielectrics

@Sk J ahiruddin , 2020

Sol 1.7. In equilibrium

E =

1 qd 41rc0 a 3

E

41rco x - = d q

1

5 9 x 109 ( · x lO

30

-lO 3

10

X

5

) 1.6 x 10- 19 = d 2.60

X

10- 16

=d

Sol 1.8. p=a.E

qd = a.E qd a.= E 40 a.= 1.4 X 10Sol 1. 9. T he electric field in t he region B

E= _ l_

Q 41rc0 r 3

The polarization

11 oo : 01

: 31

Sol 1.9. The electric field in the region B

E=

_ l_ Q 41rEo r 3

The polarization

-p = EoXeEf = Eo(r - l )E

33

[email protected]

physicsguide CSIR NET ) GATE

Capacitance and Dielectrics

@Sk J ahiruddin ) 2020

At the surface r = RA ab

-

= p ·f = Eo (r - l ) x

l Q 41rEo r 3

Q (RA - l) R3A 47f The electrostatic energy in region B

- -

1 D. EdT 2 l Q 2 47f R B Q - -2- - - r dr 2 RA 41rr 41rEo r 3 RB l Q2 - dr 3 81rEo RA r Q2 1 1

R2B Sol 1.10. None of the picture is correct. Electric field inside the linear dielectric will be proportional to the external

11 oo : 01

: 33

1

Q2

l61rE0 R~

Sol 1.10. None of the picture is correct. Electric field inside t he linear dielectric will be proportional to t he external electric field. So it will be uniform as well . But (D) is close. _,

_,

Sol 1.11. D only dep ends on the free charge, while E de_,

pends on the free charge as well as the medium. So, E is _, discontinuous and D is continuous.

[email protected]

physicsguide CSIR NET, GATE

34

Capacitance and Dielectrics

@Sk J ahiruddin, 2020

Sol 1.12. In medium-I _,

D 1

= E1E1 = 2E0 (22 - 3] + 5k) -+

The z component of t he D is continuous, since there are no surface charge. Hence _,

D 2z

_,

= =

D 1z

10Eo _,

The parallel component of the E is continuous

Hence

11 oo : 01

: 36

Hence -+

(2) A

(2)A

+ E y J.) + D2z = 5c0 (22 - 3]) + 10c0 k = Eo[l02 - 15] + lOk]

D2 = E2(EX i

Sol 1.13. From previous problem we can write ---+

A

D 1 = E1(E xi

A

A

+ Eyj) + D zk an d

D1I = I Ei(E;, + E;) + D; I j [email protected]

35

physicsguide CSIR NET, GATE

Capacitance and Dielectrics

@Sk J ahiruddin , 2020

And ---+

A

D 2 = E2(E xi

D2 =

A

A

+ E yj) + Dzk and

E~(E; + E;) + D;II -+

-+

-+

-+

Since E2 > E1, E 2 < E1 and D 2 > D 1 Sol 1.14. E r'ight z

=

Eright

=

= Hence

l

D left z

Eleft E left z Eright

1 -c 2

11 oo : 01

: 39

=

Eleft E left

z

Eright

=

1 -c 2

Hence ~

A

E riglit

A

A

= c(l / 2i + j + k)

Sol 1.15. At t he boundary

nl

1

Dl2) =

)

=

ni

2

).

So

E1 Ei l )

= 12

[email protected]

physicsguide CSIR NET , GATE

36

Capacitance and Dielectrics

@Sk J a hirudd in , 2020

Now

12

4

=3 Hence

Sol 1.16. The radial component of electric field .....

E1 · e"-r = E1 cos 01

11

00:01 :41

Sol 1.16. The radial component of electric field .....

E1 · er 7

=

E1 cos 01

J588 = cos01 3

J58 58

cos01 = tan0 1 =

3

7

And tan0 2 = 1 tan 01 tan 02

7

3

tan01 =

[email protected]

7 3 physicsguide CSIR NET, GATE

37

@Sk J ahiruddin , 2020

Capacitance and Dielectrics

Sol 1.1 7. Since t here are no ch arge on the surface

D1 cos 01 = D2 cos 02 1 E 1 cos 01 1 E 2 cos 02

tan 02 E1 tan 01

E1

=

E2 t an 02

Sol 1.18. If the electric field in slab A makes an angle 0A with the normal to t he boundary and the electric field in slab B m akes an an~le 0 n with the same normal (see fi~ure )

11 oo : 01 : 4s

Sol 1.18. If the elect ric field in slab A makes an angle 0A with t he normal to t he boundary and t he electric field in

slab B makes an angle 0B with the same normal (see figure) t hen Sol 1.19. Just like the previous problem. tan0A = EA/EB t an0B Sol 1.20. (a), (b), (c) are basic. The t otal charge inside a sphere of radius r < a is also due to bound charges. Sol 1.21. Total bound charge inside the cylinder

-+

A

P · kdx dy 2

2

= -(5£ + 7)1rr + 7 2 2 = - 5L 1rr j [email protected]

physicsguide CSIR NET 1 GATE

38

Capacitance and Dielectrics

@Sk J ahiruddin, 2020

Sol 1.22. -+

-+

Pb= - V. p

= _ 1 8 ( 2k 2) r2 or r

r

= - 4kr Sol 1.23. Since t here are no free charge, D is zero inside

t he dielectric. Hence

11 oo : 01

: 47

= - 4kr Sol 1.23. Since there are no free charge, D is zero inside

t he dielectric. Hence _,

EoE

_,

+P = 0

_,

_,

p

E =-

Eo 2 = - kd f / Eo

Sol 1.24. (i) bound surface charge density _,

Clb

=f ·p = f · kRf =kR

bound volume charge density _,

_,

Pb= - \7 · p l a 8 (r 2 kr ) 2

=-r r = -3k [email protected]

39

physicsguide CSIR NET, GATE

Capacitance and Dielectrics

@Sk J ahiruddin, 2020 _,

_,

(ii) Outside the sphere , D is zero, and P is zero as well

Sol 1.25. Since there are no free charge, D is zero in the

region a < r < b. Hence _,

_,

r,

...

11 oo : 01 : so

Sol 1.25. Since there are no free charge, D is zero in the

region a < r < b. Hence -+

.....

EoE + P = 0 .....

...

p

E=-

Eo k -r A

Eor

Sol 1.26. bound volume charge density .....

.....

Pb = - V. p

1a 1 - - - r-P0 r r8r 2

= - Po Sol 1.27. We know for a dielectric .....

.....

D = EokE O"

=E

Eok

j [email protected]

40

physicsguide CSIR NET I GATE

Capacitance and Dielectrics

@Sk J ahiruddin, 2020

For linear dielectric .....

.....

P = Eo(k - l )E

(J"

= Eo(k - 1) k EQ

11 oo : 01 : s2 For linear dielectric

P = Eo(k -

l )E a = Eo(k - 1) Eok (k - 1) = - - -a k

(k -

1)

ab = - - -a k



Sol 1.28. Since t here are no free charges on t he interface, E zO E2

= 54 4

=5 Hence t he electric field in t he region z < 0 is

Sol 1 .29. If we consider a concentric spherical shell of radius shell of radius r, t he displacement vector at r 2

D 41rr = Q

D=

[email protected]

41

Q 41rr 2 physicsguide CSIR NET ) GAT E

Capacitance and Dielectrics

@Sk J ahiruddin ) 2020

The electric field at r _,

-+

D

11 oo : 01

: 55

@Sk J ahiruddin , 2020

Capacitance and Dielectrics

The electric field at r -+

D E=-

coK

=

Q - - - -r 4c0 K 1r r 2 1

A

-+

and polarization vector P -+

-+

P = co(K - l )E (K - 1) Q = - - - -r 4K 1r r 2

A

bound surface charge density at R 1 -+

O"b

=f ·p A

(K - 1) Q

A

=r· - - - -r

4K1r Rt (K - 1) Q 4K1r Ri

bound surface charge density at R 2 -+

O"b

=f. p A

(K - 1) Q

A

=r · - - - - r

4K1r R~ (K - 1) Q 4K1r R~

[email protected]

@Sk J ahiruddin , 2020

42

physicsguide CSIR NET, GATE

Capacitance and Dielectrics

11 oo : 01 : sa

@Sk J ahiruddin , 2020

Capacitance and Dielectrics

bound volume charge density .....

.....

Pb = - V. p

Q(K-1) --

r"

= - - - -V·

r2

4K1r

= 0 (for points away from origin) (b) electrostatic energy stored in t he region R 1 < r < R 2

1

1

Sol 1. 30. a The electric field in the region R 1 < r < R 2

E=-Q_l 41rEo r 2

Hence

Q

j [email protected]

(c)Sk J a,h ir11ddin. 2020

1

43

physicsguide CSIR NET, GATE

Ca,n aci ta.nee and Dielectrics

11 oo : 02 : oo 43

j ahir@physicsguide. in

physicsguide CSIR NET I GATE

Capacitance and Dielectrics

@Sk J ahiruddin , 2020

The potent ial at any point r

b bound surface charge density at ABCD -+

aABcn=x ·P

= x · kx =k bound surface charge density at OEFG aoEFG

=

-+

x·P

= x · kOx =0 bound volume charge density -+

-+

Pb= - v7 · p

= -k Sol 1.31. Since the sphere is made of conductor it's surface 1 is an equipotential surface, having potent ial V0 = 41rc 9.r . The 0

[email protected]

44

physicsguide CSIR NET, GATE

11 oo : 02 : 03 Sol 1.31. Since the sphere is made of conductor it's surface 1 g_ . The is an equipotential surface, having potent ial V0 = 41rc r 0

[email protected]

44

physicsguide CSIR NET ) GATE

@Sk J ahiruddin ) 2020

Cap acitance and Dielectrics

free surface charge density q O"J -

- 41ra 2 Vaca a

.....

The displacement vector D = 41rr q 2 • Hence t he electric field at a distance r from t he sphere .....

... D E =-

cok

Polarization vector

P = c0 (k - l )E bound surface charge density in t he dielectric .....

= - P .f

O"b

co(k - l)Vo a Hence t he total surface charge density

a=

O"j

+ O"b

Vaca

Eo(k - l )Vo

a

a

VoEok n

11 oo : 02 : 06 (J = (JJ + (Jb

VoEo

Eo(k - l )Vo

a

a

VoEok a [email protected]

physicsguide CSIR NET, GATE

45

Capacitance and Dielectrics

@Sk Jahiruddin , 2020

Sol 1.32. Between the parallel plate capacitors D = (J. The potential difference between t he plates

V = (Jd

+

(Jd

2E1

2 E2

(Jd

1

1

2Eo

+ K1 K2

(Jd

1

1

2Eo 2 + 4 3(Jd

8Eo 8VE0

(J = - -

3d

The electric field is the region 1

E = :!_ E1

8VE0 3Eok1d

4000 3 Similarly electric field is the region 2

E = :!_ E2

8VEo 0

1

1

11 oo :02 : oa Similarly electric field is the region 2

E = !!_ E2

8Vco 3cok2d 2000 3 46

j [email protected]

physicsguide CSIR NET, GATE

Capacitance and Dielectrics

@Sk J ahiruddin, 2020

P1 = Eo(k1 - l )E1 4000Eo

3 hence

Similarly for region 2

Hence the total surface charge

2000Eo 3 Sol 1.33. bound volume charge density -+

-+

Pb= -V · P

=0 bound surface charge density at surfaces parallel to t he y - z

11 oo : 02 : 11 -+

-+

Pb = -V. p

=0 bound surface charge density at surfaces parallel to t he y- z plane

47

[email protected]

physicsguide CSIR NET, GATE

Capacitance and Dielectrics

@Sk J ahiruddin, 2020

There is a charge of magnit ude 4 units on both t he surfaces parallel to the x - z plane -+

"'

a = P -j = 4

total charge Pb + a' s

= 0+ 3 - 3+4 - 4

Sol 1.34. bound volume charge density -+

-+

Pb= - V. p

=0 The bound surface charge density at (0, 0, a)

=0

11 oo : 02 : 13

=0 -I

The electric field is E

=

-

P .

Xe€o

t he surface charge density

-I

O'= P· f

= Pocos0

48

[email protected]

physicsguide CSIR NET, GATE

@Sk J ahiruddin , 2020

Capacitance and Dielectrics

Sol 1.35.

X

Similarly Q

2

_

Qi

=

3Eo V X

=

l _ 1 1.1

-9.1

Sol 1.36. We have done this problem before. The answer is 41rE0 R 3 . Sol 1.37. The bound surface charge density is

11 oo : 02 : 16 Sol 1.36. We have done this problem before. The answer is 41rc0R 3 . Sol 1.37. The bound surface charge density is

= P0 cos0 Since t he electric field is due to dipole, it should vary as

49

j [email protected]

@Sk J ahiruddin,

1 r3

physicsguide CSIR NET 1 GATE

2020

Capacitance and Dielectrics

The potent ial due to dipole is at a distance 2R

p·r V = --41rco(2R)2 -

A

-

41rR3 P · f 3 41rE0(2R) 2

PoR 12Eo Sol 1.38. Sol 1.39. Sol 1.40. Using Kirchhoff's voltage law we can write for a

series RC circuit Vo= Rf +

Q 11

11 oo : 02 : 19 Sol 1.40. Using Kirchhoff's voltage law we can write for a

series RC circuit Q

Vo = RI + V Vo dQ 1 Q R = dt + RCV

d

Vo

t

t

= -R e Rc

dt QeRc

Vo

t

t

QeRC = R

t

eRC dt

O t

QeRc = Q(t ) =

t

VoC eRc VoC 1 Vo _ t

dQ I =- =- e dt R [email protected]

1

t e - RC

Re

physicsguide CSIR NET, GATE

50

@Sk J ahiruddin , 2020

Capacitance and Dielectrics

So option (a) Sol 1.41. Between the parallel plate capacitors D The potential difference between t he plates V =ad + ad 2E1

2E2

ad

1

-K1 + -K2

2Eo ad 1 1 2Eo 2 + 4 3ad 8Eo 8V Eo

1

= a.

11 oo : 02 : 21 ad 1 1 -+2Eo 2 4 3ad 8Eo 8VEo a = 3d The electric field is t he region 1

E = :!_ El

8VEo 3Eok1d 4000 3

j [email protected]

physicsguide CSIR NET I GATE

51

Capacitance and Dielectrics

@Sk J ahiruddin , 2020

Similarly electric field is the region 2

E = :!_ E2

8V E0 3Eok2d 2000 3

P1 = Eo(k1 - l )E1 4000Eo 3 tlPnr-P

11 oo : 02 : 23 3

P1 = Eo(k1 - l )E1 4Q00Eo

3 hence

Similarly for region 2 O'b2

= -

6Q00Eo

3

Hence t he total surface charge

2Q00Eo

3

Sol 1.42. We have done t his before. Option (D) Sol 1.43. jahir@physicsguide. in

52

physicsguide CSIR NET ) GATE

@Sk J ahiruddin ) 2020

Capacitance and Dielectrics

Sol 1.44. bound surface charge density at inner radius a O"a

=

-+

-f · P

-k a2

bound surface charge density at outer radius b -+

0-a

=f . p k 1 ')

11 oo : 02 : 26 a2

bound surface charge density at outer radius b

bound volume charge density

Pb= - V. p =0 Sol 1.45. The initial total charge on t he first capacitor

Q = C1V

= 50 X 10 = 500 t he final voltage across both capacitor is 20V. Then

+ Q2 500 = C1 V + C2 V 25 = 10 + C2 Q = Q1

C2 = 15 [email protected]

53

physicsguide CSIR NET, GATE

C apacitance and Dielectrics

@Sk J ahiruddin , 2020

Sol 1.46. the induced emf

£= a

at

-B · dS-

Now if the distance between t he parallel plate capacitor is x then R

= _ Vo cos(wt)

11 oo : 02 : 29 j [email protected]

physicsguide CSIR NET, GATE

53

C apacitance and Dielectrics

@Sk J ahiruddin , 2020

Sol 1.46. the induced emf _,

£= a

_,

B · dS

at

Now if t he distance between t he parallel plate capacitor is x then E =_ Vo cos(wt) X

From Maxwell's equation

_,

aE

curl B = µoEo at

= µoEow

Vo sin(wt ) X

So, B ex w sin(wt), so is £. Hence lowest non-vanishing ex3 ponent of w is in the expansion of w sin (wt ) = w (w - tiw + ...) is 2.

j [email protected]

54

physicsguide CSIR NET, GATE

11 oo : 02 : 31 j [email protected]

physicsguide CSIR NET, GATE

53

C apacitance and Dielectrics

@Sk J ahiruddin , 2020

Sol 1.46. the induced emf _,

£= a

_,

B · dS

at

Now if t he distance between t he parallel plate capacitor is x then E =_ Vo cos(wt) X

From Maxwell's equation

_,

aE

curl B = µoEo at

= µoEow

Vo sin(wt ) X

So, B ex w sin(wt), so is £. Hence lowest non-vanishing ex3 ponent of w is in the expansion of w sin (wt ) = w (w - tiw + ...) is 2.

j [email protected]

54

physicsguide CSIR NET, GATE

11 oo : oo : oo

Problellls and Solutions in Magnetostatics: Part-1 Sk J ahiruddin * Shreya Mitra Sandip Biswas

*Assistant Professor Sister Nibedita Govt. College, Kolkata Author was t he topper of IIT Bombay NI.Sc Physics 2009-2011 batch He ranked 007 in IIT J AM 2009 and 008 ( JRF) in CSIR NET J une 2011 He has been teaching CSIR NET aspirants since 2012

1

@Sk Jahiruddin, 2020

:Niagnetostatics: Part-1

11 oo : oo : 03 1

@Sk J ahiruddin , 2020

Magnet ost atics: P art-1

Contents 1

Lorentz Force, Bio Savart law, Ampere's Law etc 1.1

Ans Keys

1.2 Solutions .

j [email protected]

3







































26







































28

2

Physicsguide

11 oo : oo : 06

j [email protected]

Physicsguide

2

@Sk J ahiruddin , 2020

Magnetost atics: P art-1

Problems from NET, GATE, JEST, TIFR & JAM papers

1

Lorentz Force, Bio Savart law, Ampere's Law etc

Prob 1.1. A charged particle is released at time t

= 0,

from the origin in the presence of uniform static electric and magnetic fields given by E = E 0 y and B = B 0 z respectively. Which of t he following statements is true fort> O? [JEST 2015] (a) The particle moves along the x-axis. (b) The particle moves in a circular orbit. (c) The particle moves in t he (x,y) plane. (d) particle moves in the (y,z) plane. Prob 1.2. The value of the magnetic field required to maintain non-relativistic protons of energy 1 MeV in a circular orbit of radius 100mm is __________Tesla.

(Given : mp= 1.67 2014]

X

10- 27 kg, e = 1.6

X

10- 19 C ) [GATE

11 oo :oo :oa (Given : mp= 1.67

X

10- 27 kg, e = 1.6

X

10- 19 C ) [GATE

2014]

[email protected]

3

@Sk J ahiruddin, 2020

Physicsguide

Nl agnetostatics: P art-1

Prob 1.3. An alpha particle in accelerated in a cyclotron. It leaves the cyclotron with a kinetic energy of 16 MeV . The potential difference between the D electrodes is 50 kilovolts. The number of revolutions the alpha particle makes in its spiral path before it leaves the cyclot ron is ------------· [GATE 2016]

Prob 1.4. A charged particle in a uniform magnetic field = Boezstarts moving from the origin with velocity ef' = (3e"x + 2ez) m / s. The trajectory of the particle and the t ime t at which it reaches 2 met ers above the x-y plane are [JAM 2016] (e"x , e"y, and ezare unit vectors in Cartesian-coordinate system) (a) Helical path; t = l s (b) Helical path; t = 2/ 3s (c) Circular path; t = l s (d) Circular path ; t = 2/ 3s

B

B

Prob 1.5. A magnetic field = B 0 ('i + 2] - 4k) exists at point. If a t est charge moving with a velocity, ef' = v 0 ( 32 - 3+ 2k) experiences no force at a certain point , the electric field at that point in SI units is [JEST 2012] (a) = -voBo(3'i - 2] - 4k)

E \

11 oo : oo : 11 Prob 1.5. magnetic eld = Bo i + 2j - 4k exists at point. If a test charge moving wit h a velocity, -i}' = v0 ( 3i - + 2k) experiences no force at a certain point, t he electric field at that point in SI units is [JEST 2012]

3

(a) (b)

E = -voBo(3i - 23 - 4k) E = -voBo(i + 3+ 7k )

[email protected]

@Sk J ahiruddin, 2020

4

Physicsguide

:Niagnet ost atics: Part-1

(c) E = voBo(l4J + 7k ) (d) E = -voBo(l43 + 7k ) Prob 1.6. A charged particle of mass m , charge q and constant velocity -;J enters a uniform magnetic field = B 0ex,(B 0 > 0) , at an angle 0to the direction of magnetic field . Find the angle 0,if in one revolution of t he helical motion, the particle advances along the direction of the magnetic field a distance equal to the radius of the helical path? [JAM 2013]

B

Prob 1.7. A charged particle moves in a helical path under t he influence of a constant magnet ic field. The initial velocity is such t hat t he component along the magnetic field is twice the component in the plane normal to t he magnetic field. The ratio 1 / R of the pitch 1 to t he radius R of t he [NET Dec 2014] helical path is

11 oo : oo : 14 [NET Dec 2014]

helical path is

j [email protected]

5

P hysicsguide

@Sk J ahiruddin , 2020

Magnetostatics: Part-1

J,..

To< (a) 1r/ 2

(b) 41r

(c) 21r

(d)

7r

Prob 1.8. A posit ively charged part icle, wit h a charge q,

E

enters a region in which t here is a uniform electric field and a unifrom magnetic field both directed parallel to t he positive y-axis. At t = 0, t he particle is at t he origin and has a speed v0 directed along the positive x axis. The orbit of the particle, projected on the x-z plane, is circle. Let T be t he time taken to complete one revolut ion of t his circle. The y-coordiante of the particle at t = T is given by [JAM 2015]

B,

11 oo : oo : 16 Let T be t he t ime taken t o complete one revolution of t his circle. The y-coordiante of the particle at t = T is given by [JAM 2015]

(d)

21rmvo

qB

Prob 1.9. A proton moves with a sp eed of 300m/s in a circular orbit in t he xy-plan in a magnetic field 1 tesla along [email protected]

P hysicsguide

6

@Sk J ahiruddin, 2020

Magnetostatics: Part-1

t he positive z-direction. When an electric field of 1 V / m is applied along the positive y-direction, t he centre of the [NET June 2015] circular orbit (a) remains stationary (b) moves at 1 m/s along the negative x-direction (c) moves at 1 m/ s along the positive z-direction (d) moves at 1 m/s along the positive x-direction

Prob 1.10. In t he laboratory frame two electrons are shot at each other with equal and opposite velocities ¼ and ½ respectively, but not along the same straight line, as shown [TIFR 2013] below z X

ii2 ◄ -- •

11 oo :oo :1a z X

ii2 • ◄ -- •

Each electron will be act ed on by the Coulomb repulsion due to the other, as well as t he Lorentz force due to its own motion in the magnetic field created by t he each ot her. Which of t he diagrams given below best describes the final velocities Vi and ½of these electrons? [You may assume t hat the electrons are distinguishable]

[email protected]

Physicsguide

7

@Sk J ahiruddin , 2020

Magnetostatics: P art-1

z

...

.•

V1

_..,. X

z

X

• --◄... Vz

.....

Vz -•► •

• -◄-

X

X

-

Prob 1.11. If the dimensions of mass, length, t ime and charge are M,L,T , and C respectively, t he dimensions of is [JAM 2013] t he magnetic induction field (a) ML2r - 1 c-1 (b) Mr- 1c- 1

B

11 oo : oo : 21 Prob 1.11. If the dimensions of mass, length, t ime and charge are M,L,T , and C respectively, t he dimensions of is [JAM 2013] t he magnetic induction field

B

(a) ML2 r - 1 c- 1 (b) Mr- 1c- 1 (c) L 2r- 1c (d) L- 1r- 1 c Prob 1.12. In terms of the basic units of mass (NI), length (L), t ime(T ), and charge (Q) , the dimensions of t he mag[JAM 2007] netic permeability of vacuum ( µ 0 ) are

(a) M LQ- 2

(c) LTQ- 1

(b) ML2r- 1 Q- 2 (d) LT- 1Q- 1

Prob 1.13. The magnetic field at a distance R from a long j [email protected]

8

@Sk J ahiruddin , 2020

Physicsguide

Nlagnetostatics: Part-1

straight wire carrying a steady current I is proportional to [NET June 2012] 2 2 2 (a) IR (b) I/ R (c) 1 / R (d) I/ R Prob 1.14. The force between two long and parallel wires carrying currents 11and 12 and separated by a distance D is

proportional to (a) 1112/ D (b) (11+12)/ D 2 1112/ D

[NET Dec 2013] 2 ( d) (c) (1112/ D)

Prob 1.15. Two parallel infinitely long wires separated by a distance D carry steady currents 11 and 12 ( 11 > 12 ) flow-

ing in t he same direction . A positive point charge moves

11 oo : oo : 24 Prob 1.15. Two parallel infinitely long wires separated by a distance D carry steady currents 11 and 12 ( 11 > 12 ) flow-

ing in the same direction. A positive point charge moves between the wires parallel to the currents with a speed v at at distance D / 2 from eithe1~ wire. T he magnitude of an electric field t hat must be turned on to maintain the trajectory of t he part icle is proportional to

(a) (I1- I2)v/D (b) (I1 + I2)v/D 2 2 (d) (I1 + I2)v / D

[JEST Sample] 2 2 (c) (I 1-I2)v / D

Prob 1.16. In an ionization experiment conducted in t he laboratory, different singly-charged positive ions are pro-

duced and accelerated simultaneously using a uniform elect ric field along x-axis. If we need to determine the masses of

[email protected]

@Sk J a hiruddin , 2020

9

Physicsguide

Magnetostatics: P art-1

various ions produced, which of t he following methods will NOT work [TIFR 2016] (a) Detect them at a fixed distance from the interaction point along x-axis and measure t heir time of arrival (b) Apply a uniform magnetic field along y-axis and measure the deviation (c) Apply a uniform electric field along y-axis and measure t he deviation (d) Apply a uniform electric field along y-axis and a (variale) uniform magnetic field along z-axis simulataniously and note t he zero deviation.

11 oo : oo : 26 (c) Apply a uniform electric field along y-axis and measure t he deviation (d) Apply a uniform electric field along y-axis and a (variale) uniform magnetic field along z-axis simulataniously and note t he zero deviation. Prob 1.1 7. Consider t hree ident ical infinite straight wires A,B and C arranged in par allel on a plane as shown in t he figure . [TIFR 2012]

X

I

I

I

-- d -

-

d-

'' • •

B

A

C

The wires carry equal currents I with directions as shown j [email protected]

10

@Sk J ahiruddin , 2020

Physicsguide

Magnetostatics: P art-1

in t he figure and have mass per unit length m. If the wires A and C are held fixed and t he wire B is displaced by a sm all distance x from its position, then it (B) will execute simple harmonic motion with a t ime period

(a) 21r

n;o (1)

(d) 21r

2:

0

(~)

(b) 21r

:;;i (1) (c) 21r

2

~

:; (

1)

(e) 21r : (~)

Prob 1.18. Consider a planar wire loop as an n -sided reg-

11 oo : oo : 29

Pro b 1.18. Consider a planar wire loop as an n -sided regular polygon , in which R is t he distance from the cent re to a vert ex. If a steady current I flows t hrough the wire, the magnit ude of t he magnetic field at t he centre of the loop is [NET June 2019] ( 21r) µof . ( a ) R sin -;:

2

(c)

i:~ tan( ~ )

Prob 1.19. A conducting wire is in t he shape of a regular hexagon , which is inscribed inside an imaginary circle of radius R , as shown. A current I flows through the wire. The magnit ude of t he magnetic field at the center of the circle [JAM 2015] is

[email protected]

@Sk J ahiruddin, 2020

11

Physicsguide

Nl agnetostatics: P art-1

11 oo : oo : 31 I I c.

Prob 1.20. The strength of t he magnetic field at the cent re of a regular hexagon with sides of length a carrying a steady

current I is:

(a) µof

[JEST 2016] (b)

vl31ra

v16µol

(c) 3µol

1ra

1ra

(d)

-./3µo l 1ra

Prob 1.21. A current I is flowing through t he sides of an equilateral triangle of side a. The magnit ude of t he magnetic field at t he cent roid of the triangle is [JAM 2018] (a) 9µol (b) µo l (c) 3µol (d) 3µo l 21ra

1ra

21ra

1ra

Prob 1.22. Consider two infinitely long wires parallel to t he z-axis carrying t he same current I . One wire passes

t hrough t he point L with coordinates (-1 ,1) and the ot her t hrough M with coordinates (-1 ,1) in the XY planes shown [email protected]

12

@Sk J ahiruddin, 2020

P hysicsguide

:Niagnetostatics: Part-1

in the figure. The direction of t he current in both the wires [JAM 2009] is in the posit ive z-direction. "-(,

I

11 oo : oo : 34 in t e e 1rection o t e current 1n e [JAM 2009] is in the positive z-direction. '{ I

I

, I

B1

(a) Find t he value of p .d along the semi circular closed path of radius 2 units shown in the figure. (b) A third long wire carrying current I and also perpendicular to t he XYplane is placed at a point N with coordinates (x, 0) so that the magnetic field at the origin is doubled. Find x and t he direction of t he current in t he t hird wire? Prob 1.23. [MSQ]Three infinitely-long conductors carrying currents I 1 , I 2 and I 3 lie perpendicular to the plane of paper as shown in t he figure. If t he value of the integral p .d for the loops C1, C2 and C3 are 2µo , 4µoand µo in [JAM 2016] t he units of ~ respectively, then

B1

j [email protected]

@Sk J ahiruddin , 2020

13

Physicsguide

Magnetostatics: Part-1

00: 00: 37 :Nlagnetostatics: P art-1

C2

I

12.



c, .11

(a) 11 = 3A into the paper (b) I2 = 5A out of the paper (c) l 3 = 0 (d) 13 = lA out of the paper

Prob 1.24. A circular loop of fine wire of radius R carrying a current I is placed in a uniform magnetic field B p erpen-

dicular to the plane of the loop. If the breaking t ension of t he wire is Tb, the wire will break when the magnetic field [TIFR 2016] exceeds (d) µoTb Tb ( ) Tb ( ) µoTb ( ) a IR b 21rIR c 21rIR 41rIR Prob 1.25. A parallel plate capacitor is formed by two cir-

cular conducting plates of radius a separat ed by a distance [email protected]

@Sk J ahiruddin, 2020

14

Physicsguide

Magnetostatics: Part-1

d , where d < < a. It is being slowly charged by a current

11 oo : oo : 39 @Sk J ahiruddin, 2020

Nl agnetostatics: Part-1

d, where d < < a. It is being slowly charged by a current t hat is nearly constant . At an instant when t he current is I, t he magnet ic induction between the plates at a distance a/2 from the center of t he plate is [NET D ec 2016]

(b) ~~~

0 ;

(a) ~

(c)

µ~I

( d)

f~~

Prob 1.26. A set of N concent ric circular loops of wire each carrying a st eady current I in the same direction , is arranged in a plane. The radius of t he first loop is r 1 = a and the radius of t he nt h loop is given by r n = nrn - l · The magnit ude B of t he magnet ic field at t he centre of t he circles in t he limit N ➔ oo, is [NET June 2017] 2 (a) µoI (e - 1) (b) µol (e - 1) 4na na

(c) µo I (e

2

-

1)

8a

(d) µo I (e - 1) 2a

Prob 1.27. A const ant current I is flowing in a piece of wire that is bent into a loop as shown in t he figure [NET June 2017]

[email protected]

@Sk J ahiruddin , 2020

15

P hysicsguide

Magnetostatics: P art-1

11 oo : oo : 42 15

[email protected]

Physicsguide

@Sk J ahiruddin , 2020

:Niagnetostatics: Part-1

t

.

0-..

I,

I

?J..o..' ,

~b

I ..

lo

..

.

. )(

Prob 1.28. A system of two circular co-axial coils carrying equal currents I along same direction having equal radius R and separated by a distance R (as shown i the figure below) . The magnit ude of magnetic field at t he midpoint P is given by [JEST 2014]

~

'l

1

I

I

R

[email protected]

16

Physicsguide

11 oo : oo :44

j [email protected]

Physicsguide

16

@Sk J ahiruddin , 2020

µal (a) 2v'2R

Magnetostatics: Part-1

(b) 4µo l

(d) 0

5v'5R

Prob 1.29. Consider a thin long insulator coated conducting wire carrying current I. It is now wound once around an insulating t hin disc of radius R to bring the wire back on t he same side,as shown in the figure.The magnetic field at t he cent er of the disc is equal to [JAM 201 7] I

Prob 1.30. A uniform surface current is flowing in t he positive y-direction over an infinite sheet lying in t he x-y plane. The direction of the m agnetic field is [GATE 2011] (a) along 1, for z > 0 and along -2for z < 0 (b) along k for z > 0 and along -k for z < 0 (c) along -2for z > 0 and along 1, for z < 0 (d) along -k for z > 0 and along k for z < 0 A

A

A

A

Prob 1.31. A conducting slab of copper PQRS is kept on t he xy plane in a uniform magnetic field along x-axis as in-

11 oo:oo:47 A

A

(d) along -k for z > 0 and along k for z < 0 Prob 1.31. A conducting slab of copper PQRS is kept on t he xy plane in a uniform magnetic field along x-axis as [email protected]

Physicsguide

17

@Sk J ahiruddin, 2020

Magnetostatics: Part-1

dicated in the figure. A steady current I flows t hrough the cross section of t he slab along the y axis. The direction of t he electric field inside t he slab, arising due to the applied [JAM 2014] magnetic field is along the

z

I

~ - + - - - ~- ----+ y B

X

(b) positive Y direction (a) negative Y direction (d) positive Z direction (c) negative Z direction Prob 1.32. A rectangular loop of dimension L and width

w moves wit h a constant velocity v away from an infinitely long straight wire carrying a current I in the plane of the loop as shown in the figure below. Let R be t he resistance of the loop. What is t he current in t he loop at the instant t he near side is at a dist ance r from t he wire? [JAM 2017]

11 oo : oo : so of the loop. What is t he current in t he loop at the instant t he near side is at a distance r from t he wire? [JAM 2017]

[email protected]

18

P hysicsguide

Magnetostatics: P art-1

@Sk J ahiruddin , 2020

fv

fw

R L

◄ ◄

µ 0 IL wv (a) 21rR r[r + 2w] µ 0 IL

wv

(c) 21rR r [r + w]

I

b µ 0 IL wv ( ) 21rR r[2r + w]

d µ 0 IL wv ( ) 21rR 2r [r + w]

Prob 1.33. The loop shown in t he figure below carries a

[NET June 2018]

steady current I .

0

11 oo : oo : s2 a

0

The magnitude of the magnetic field at the point O is (a) µof (b) µo f (a) µo f (d) µo f 2a 6a 4a 3a Prob 1.34. MSQ: In presence of a magnetic field B j and j [email protected]

19

Physicsguide

@Sk J ahiruddin , 2020

Nlagnetostatics: Part-1

"'

an electric field ( - E) k , a particle moves undeflected. Which of the following statements is (are) correct? [JAM 2018]

EA

(a) The particle has positive charge, velocity =

-Bj

(b) The particle has positive charge, velocity =

~3

E "' (c) The particle has negative charge, velocity= - Bj E "' (d) The particle has negative charge, velocity = B j

Prob 1.35. Two current-carrying circular loops, each of radius R, are placed perpendicular to each other, as shown in t he figure below.

The loop in the xy -plane carries a current I 0 while that in the x z -plane carries a current 210 . The resulting mag..... netic field B at t he origin is [NET Dec 2018]

z

y

11 oo : oo : 55 e or1g1n 1s

[2j + k] (d) µ2010 [-2j - k]

(a)

_µ2010

[email protected]

20

Physicsguide

@Sk J a hiruddin , 2020

Magnetostatics: P art-1

Prob 1.36. Consider three straight, coplanar, parallel wires of infinite length where t he distance between adjacent wires

is d. Each wire carries a current J in the same direction. The p erpendicular distance from the middle wire (on either side) where the magnetic field vanishes is

(a) d )3

(b) 2d

3

(c) ~

[TIFR 2019]

(d) 2d

)3

3

Prob 1.37. Consider a hydrogenic atom in its ground state as conceived in Bohr's theory, where an electron of charge

-e is rotating about a central nucleus of charge + Z e in a circular orbit of radius a= 41rc 0 n2 / Ze 2 m. In this model, the magnetic field at a distance r from t he nucleus, p erpendicular to t he orbit, will be [TIFR 2019]

r

11 oo :oo : s1 [TIFR 2019]

ular to the orbit, will be

r a r2

1+

a

- 1/ 2

2

j [email protected]

41T"EQ

fi 5

2

1+

r 2 a

- 1

Physicsguide

Magnetostatics: P art-1

r2

1+

3 m2

21

@Sk J ahiruddin , 2020

(c)

z e2

a

2

-3/ 2

z e2

3 m2

41T"EQ

fi 5

(d)

r2

1+

a

2

Prob 1.38. A cyclotron can accelerate deuteron to 16 MeV . If the cyclotron is used to accelerate a -particles, what will be their energy? Take t he mass of deuteron to be twice the

mass of proton and mass of alpha particles to be fou1· times [JEST 2019] t he mass of proton. (a) 8 MeV (b) 16 MeV (a) 32 MeV (a) 64 MeV Prob 1.39. A wire with uniform line charge density ,\ per

unit length carries a current I as shown in the figure. Take t he permittivity and permeability of t he medium to be co = µ 0 = l. A particle of charge q . is at a distance r and is traveling along a trajectory parallel to the wire. What is

-3/

11 oo : 01 : oo unit length carries a current l as shown in the figure. Take t he permittivity and permeability of t he medium to be co = µ 0 = l. A particle of charge q . is at a distance r and is traveling along a trajectory parallel to the wire. What is [JEST 2019] t he speed of the charge?

I charge density = A current=

[email protected]

r

22

Physicsguide

@Sk J ahiruddin, 2020

A (a) -

Nlagnetostatics: P art-1

A (c) 31

A (b) 21

l

(d) 4A l

Prob 1.40. Which one of the following is an impossible ...., magnetic field B ? [JAM 2019] ...., 2 2 (A) B = 3x z x - 2xz 3z

(B)

B=

(C)

B= ....,

- 2xyx + y z (xz

+ 4y) x

2

y+

- yx

3

2yz -

3

z3

z

y+

(D) B = -6xzx + 3yz y 2

Prob 1.41. Consider an annular region in free space containing a uniform magnetic field in the z -direction, schemat-

11 oo : 01

: 03

(D) B = -6xzx + 3yz y 2

Prob 1.41. Consider an annular region in free space containing a uniform magnetic field in the z -direction, schemat-

ically represented by the shaded region in the figure. A particle having charge Q and mass M starts off from point P (a, 0, 0) in the + x -direction with constant speed v . If t he radii of inner and outer circles are a and b, respectively, t he minimum magnetic field required so that the particle returns to the inner circle is [JAM 2020]

[email protected]

23

P hysicsguide

@Sk J ahiruddin, 2020

:Niagnet ost atics: Part-1

y

p

, . . Mv

(

b2 - a 2 \

- l

,_ . Mv

X

(

b2 - a2\ -

l

11 oo : 01 : os

(A) Mv

b2 - a2 b

Q (C) M v Q

b2 - a2

3b

- 1

(B) Mv Q

-1

(D) Mv Q

b2 - a2

-1

2b

b2 - a2

-1

4b

Prob 1.42. A positively charged particle is placed at the origin (with zero initial velocity) in the presence of a con-

stant electric and a constant magnetic field along the posit ive z and x directions, respectively. At large times, the overall motion of the particle is adrift along the [NET Dec 2019] (a) positive y -direction (b) negative z -direction (c) positive z-direction (d) negative y -direction

j [email protected]

@Sk J ahiruddin, 2020

24

Physicsguide

Magnetostatics: Part-1

Prob 1.43. Consider three infinitely long, straight, and coplanar wires which are placed parallel to each other. T he dis-

tance between the adjacent wires is d. Each wire carries a current I in the same direction. Consider points on either side of the middle wire where t he magnetic field vanishes. What is the distance of these points from t he middle wire? [JEST 2020] (A) 2d (B) 2d (D) d 3 v'3 v'3

11 oo :01 : oa side of the middle wire where t he magnetic field vanishes. What is the distance of these points from t he middle wire? [JEST 2020] (A) 2d (B) 2d (D) d 3 v'3 v'3

[email protected]

25

@ Sk J ahiruddin, 2020

1.1

Physicsguide

Magnetostatics: Part-1

Ans Keys 1.14. a

1.26. d

1.2. 1.44

1.15. a

1.27. b

1.3. 160

1.16.

1.28.

1.1.

C

C

C

11 oo : 01

: 11

1.2. 1.44

1.15. a

1.27. b

1.3. 160

1.16.

C

1.28.

1.4. a

1.17.

C

1.29. b

1.5. d

1.18. d

1.30. a

1.6. tan- 1 21r

1.19.

1.31. d

1.7. b

1.20. d

1.32.

1.8. b

1.21. a

1.33. b

1.9. d

1.22. (a) 2µ1r , (b) 1.34. b,d lm

1.10. a ,d

C

z '

1.35.

C

C

C

1.23. a,b 1.11. b

1.36. a 1.24. a

1.12. a

1.37. d 1.25. d

1.13. d

1.38.

[email protected]

C

26

@Sk J ahiruddin , 2020

Physicsguide

Magnetostatics: P art-1

1.39. a

1.41. b

1.40. d

1.42. a

1.43. D

11 oo : 01 1.39. a

1.41. b

1.40. d

1.42. a

j [email protected]

@Sk J ahiruddin, 2020

1.2

: 13 1.43. D

27

Physicsguide

Nlagnetostatics: Part-1

Solutions

Sol 1.1. The particle will start moving in x direction first

11 oo : 01

: 16

@Sk J ahiruddin , 2020

1.2

Magnetostatics: Part-1

Solutions

Sol 1.1. The particle will start moving in x direction first because of Electric field. The magnetic field will give it a ..... force to q(v x B ) direction. So the magnetic force is towards

x direction. The particle will continue to move in xy plane, not in z as t he forces are in xy plane. Circular orbit is not possible due to t he presence of constant acceleration towards y direction because of electric field. Sol 1.2. For a charged particle in Magnetic field in circular

orbit

qvB

mv 2

= -r

Hence

1 2 1 mv = qv B r 2 2 Energy is 1 MeV. So the velocity is

2E V= m 2 X 1 X 106 X 1.6 X l0- 19 1.6 X 10- 27 7 = \l'2 X 10 Put the values 6

(1 X 10 )(1.6

X

[email protected]

@Sk J ahiruddin, 2020

Hence

10-

19

)

19

7

= ~(1.6 X 10- )(,/2 X 10 )B (0.1 ) 28

Physicsguide

Magnetostatics: P art-1

11 oo :01

:1

a

@Sk J ahiruddin, 2020

Magnetost atics: Part-1

Hence

B = V2T Sol 1.3. In each revolut ions the a particle energy qV = 2eV 3

= 2 x 50 x 10 eV 5

= 10 eV , where e the charge of a proton. By the t ime it leaves the cyclot ron it has completed N

=

16MeV qV

16

X

106

105 = 160 number of revolutions.

B

Sol 1.4. The motion of the particle along i.e along e~ remains unchanged. And because of the velocity component perpendicular t o the magnetic field the particle will execute a circular mot ion with magnetic field providing the centripetal accceleration. Hence the trajectory of the part icle will be Helical. j [email protected]

@ Sk J ahiruddin, 2020

29

Physicsguide

Magnetostatics: P art-1

11 oo : 01 [email protected]

: 21

29

P hysicsguide

@Sk J ahiruddin , 2020

Magnetostatics: P art-1

Since t he z component of velocity remains unchanged, to t ravel 2 meters t he particle will take t ime

2

-2 = ls . Sol 1.5. The Lorentz force on the particle is zero. Hence ....

....

O=E +iJ x B

E = -vo(3i - 3+ 2k) x Bo(i + 23 = - vo B 0 (14] + 7k)

4k)

Sol 1.6. If T is the time for one revolut ion, t hen VzT Vz

=R

2

1rR = R

V > Ui

CUi

Magnetostatics: Part-2

(b)

Ug

=

CUi

(c)

Ug


O J-11 = 5 X 1Q-6H/m

_

K

= 100.x Alm

-B, =4.x - 6y+2z mT ~ ~ - - - - ~ ~ ---;-:::-::-;--::-:--.,.,..-..,.........,..""7

z =O

,.. X

-

,..

n

82 1-12 = IOx I0-6 Him

z< O 1. 20. Which of the following is not a correct boundary con-

dition at an interface between two homogeneous dielectric media? (In the following n is a unit vector normal to t he interface, a and Js are t he surface charge and current den[NET June 2019] sities, respectively.) (a) n x (D1 - D2) = 0 (b) n x (H1 - H2) = Js (c) n · (D 1 - D2) = a (d) n · (B1 - B2) = 0

j [email protected]

@Sk J ahiruddin, 2020

13

Physicsguide

Magnetostatics: Part-2

1. 21. The magnetic fields in tesla in the two regions separated by the z = 0 plane are given by Hi = 3x+ 5z and~ =

x+ 3y + 5z. The magnit ude of t he surface current density at

00: 00: 37 :Nlagnetostatics: Part-2

1.21. The magnetic fields in tesla in the two regions separated by the z = 0 plane are given by Hi = 3x+ 5z and~ =

x + 3y + 5z. The magnit ude of t he surface current density at t he interface between t he two regions is a x 106 A/m. Given 2 7 t he permeability of t he free space µ 0 = 41r x 10- N/A , t he value of a is - - - - - - - - - - (Round off to 2 decimal places) [JAM 2020 ]

1.22. Far from the Earth, the Earth's magnetic field can

be approximat ed as due to a bar magnet of magnetic pole strength 4 x 1014 Am. Assume this magnetic field is generated by a current carrying loop encircling t he magnetic equator. The current required to do so is about 4 x 10n A, where n is an integer. The value of n is - - - - - - - - [GATE 2020] (Earth's circumference: 4 x 107 m )

[email protected]

@Sk J ahiruddin, 2020

1.1

Ans Keys

14

P hysicsguide

Magnetostatics: Part-2

11 oo : oo : 39 @Sk J ahiruddin, 2020

1.1

Nl agnet ost atics: Part-2

Ans Keys

1.1. do yourself

1.9. a

1.17.

C

1.2.

C

1.10. b

1.18.

C

1.3. d

1.11. b

1.19. 2

1.4. b

1.12. a

1.20. a

1.5. 4

1.13. d

1.21. 2.86 to 2.88

1.6. d

1.14. d

1.22. 7

1. 7.

C

1.15. b

1.8.

C

1.16.

[email protected]

@Sk J ahiruddin , 2020

C

15

Physicsguide

Magnetost atics: Part-2

11 oo : oo : 42 15

[email protected]

Physicsguide

@Sk J ahiruddin , 2020

1.2

:Niagnetostatics: Part-2

Solutions

1.1. For r

< r 1 using ,

pii.di

I enc

J1rr2 Hence,

H.21rr = J1rr 2 Hence, we get, H = Jr 2 Similarly for , r1r < r2 , J1rrr Hence,

pfi.Jz

I enc

Hence, H.21rr

H = Jrr 2r and sarnne in case of r > r 2 1.2. given ,

1 = Jork Or,

-H .dl- = Jo

d

rdrd > r) and is at a distance x(x > > r) above it . If the lower loop is held fixed and upper loop moves upwards with a uniform velocity v = ~; , then t he induced emf and t he direction of t he induced current in t his loop will be .

?

2 R 2v

.1 k .

x 4 ; antic oc wise 2 2 R v 2 2 (b) 2i µ 01r r x 4 ; clockwise 2

(a ) 3iµo'lr"-r

2 2 2

(c) 3iµ 0 7r r R

2x

2 2R

(d) 2iµ 01r r

~;

anticlockwise

2

v

x 3 ; clockwise

3

1.25. A rectangular metallic loop with sides L 1 and L 2 is

11 oo:oo:47 2x 3 ' 2 2R

(d) 2iµ 01r r

2

v

x 3 ; clockwise

3

1.25. A rectangular metallic loop with sides L 1 and L 2 is [email protected]

17

Physicsguide

@Sk J ahirudd in, 2020

Electromagnetic Induction

placed in the vertical plane, making an angle cpwith respect to the x axis, as shown in the figure, and a spatially uni-

B

form magnetic field = By is applied. The loop is free to rotate about the z axis (shown in the figure with a double

[TIFR 2017]

line)

2

B y

••

X

••

••

••

The magnetic field changes with t he time at a constant

~f

rate = k If t he resitance of the loop is R , the torque r required to prevent the loop from rotating will be 2 2 2 2 (a) -kB (b) kB ) sin 2cpz ) sin cp cos cpz

(L~i

(c) kB (L~i ) 2

2

sin cpz

(L~i

(d) - kB (L

i

1

2

2 )

sin cpz

1.26. A long solenoid is embedded in a conducting medium and is insulated from the medium. If the current t hrough t hP. solP.noi1

= - B ldx

And, induced emf e

=

de/> -dt

=

Blv

Again, in second case, At x = dx, c/>1 = B l (w - dx )

And x

= 4w, c/>2 = Blw

11 oo : 01 x

dx, c/>1

=

: 42

=

B l(w - dx )

And x = 4w , c/>2 = B lw

Thus chane in flux ,

and related induced ernf e

= - Blv 0 opt ion (c) is

Inside, Magnet ic field is uniform , so e correct .

1.32. As the magnet ic field is uniform, T here is no change in flux. [email protected]

P hysicsguide

37

@Sk J ahiruddin , 2020

Electromagnetic Induction

1.33.

V = IR + L dI dt P utting the values, we get, dI -dt = I A /s 1.34. For a long solenoid,

B inside

i

Hence, B

=

µ 0n K

tz

= Kt

= µonl z

11 oo : 01 1.34. For a long solenoid,

= µonI z

B inside

i Hence, B

: 44

=

Kt

= µ 0 n K tz

1.35. Bo = O.OlTr = 0.02m

t

-t

Bo -e o

to

R = 2, to = l

E.&=

e =

IR

0.01. (0.02)2 .w

=

I = w(0.01)(0.02) 2 Q = 6.28 j ahir@physicsguide. in

2

dQ dt

Physicsguide

38

@Sk J ahiruddin , 2020

Electromagnetic Induction

1.36. The flux t hrough the loop

=

-B · dxdyz L 2

= (30

3y tdx dy 0

= f3otL3 The emf

11 oo : 01

: 47 2

= f3o

3y tdxdy 0

= f3otL3 The emf £

= - a

at

=f3o L3 1. 3 7. Since t he loop is closed to t he solenoid , we can safely assume that the magnetic field through t he loop is also µo n l s(t) .

the induced emf in the wire loop £

= - a

at

dl s t 2

i(t) = _µonr dI.(t) R

dt

So, the in t he wire loop will at first take narrow and big spike t he a small and short spike. [email protected]

Physicsguide

39

@Sk J ahiruddin , 2020

Electromagnetic Induction

1.38. the induced emf

£

= _a

at

= Bvl 3 90 X 10 4 1 = .2 x 10- x x 2Tms3600 -

1

,.-yo

T/

11 oo : 01 E= -

: 49

ot

= Bvl = .2 x 10

-4

_1 90 X 103 x x 2Tms 3600

= lmV

j [email protected]

@Sk J ahiruddin , 2020

2

40

P hysicsguide

E lectromag11etic Induction

Magnetic Vector potential

2 .1. The vector potential in a region is given as

1 (X, y , Z) = - yi + 2x)

11 oo : 01 : s2 2

Magnetic Vector potential

2.1. The vector potential in a region is given as

1 (X, y, Z) = -yi + 2x J The associated magnetic induction is

(a) I+ k

(b)

3k

(c)

-i + 2]

B is

[GATE 2006] (d)

-i + 3+ k

2.2. A current I = lOA flows in an infinitely long wire along the axis of a hemisphere (see figure) . The value of =t ➔➔ J(V x B ) .ds over the hemispherical surface as shown in t he [JAM 2017] figure is:

I = 10 A

(a) 10 µo

(b) 5 µo

(c) 0

(d) 7. 5µo

2.3. Given that t he magnetic flux t hrough the closed loop

[email protected]

@Sk J ahiruddin , 2020

PQRSP is cp. If

41

Physicsguide

Electromagnetic Induction

11 oo : 01 : s4 @Sk J ahiruddin, 2020

PQRSP is

Electromagnetic Induction

cp. If R

p

R

along PQR, the value of

J 1 .dl along PSR is

[GATE

p

2015]

Q

R

p

s

2.4. A rod of length L with uniform charge density .\per

unit length is in the xy plane and rotating about z-axis passing through one of its edge wit h an angular velocity cJ as shown in t he figure below. (f ,cp, z) refers to t he unit vect ors at Q, is the vector potent ial at a distance d from t he origin O along z-axis for d < < L and is current density due to the motion of the rod. Which one of t he following A

1

[email protected]

@Sk J ahiruddin, 2020

1

42

Physicsguide

Electromagnetic Induction

rr-.A'T'R ?flfl~l

11 oo :01 : s1

@Sk J a hiruddin , 2020

Electromagnetic Induction

[GATE 2008]

statements is correct?

I

tl I

I

1 along f; 1 along z; 11 ex¼ (b) 1 along ~; 1 along ~; 11 ex J (c) J along f; 1 along z; 11 cxJ2 (d) 1 along J; 1 along J; 1 1ex! (a)

2

2.5. Which one of the following current densities , 1,can

generate the magnetic vector potential

2 (a) -(xi+yj) µo 2 -(xi - yj) µo A

A

A

2 (b) --(i+ j) µo A

A

A=

(y 2i + x 2]) ? [GATE 2009]

2 (c) -(i-j) µo A

A

(d)

A

j a [email protected]

(c)Sk J a,h ir11ddin. 2020

43

Physicsguide

Electromae:netic Induction

11 oo : 02 : oo 43

j [email protected]

P hysicsguide

@Sk J a hiruddin , 2020

Electromag11etic Induct ion

2.6. Which of t he following magnetic vector potent ials give rise to a uniform magnetic field B 0 k? [GATE 2016] A

(a) B 0 zk

(b) -B0 x]

(c) ,(- yi+x])

(d) ,(yi+x])

2. 7. The magnetic filed corresponding to the vector poten-

t ial

1 = !1 2

X

7 + lQ7 3 r

1 is a constant vector, is [NET June 2012] 3 3 (a) 1 (b) -1 (c) 1 + ~7 (d) 1- ~7 r r

where

A due to a magnetic moment rri 7 at a point ?is given by A = ~~ . If rriis directed along 2.8. The vector potential

positive z-axis, the x-component of t he magnetic field, at t he point 7, is [NET June 2012]

(a) 3m;z r

(b) _ 3myz r5

(c) 3m:z r

(d) 3m(z

2 r5

xy)

2.9. A thin infinitely long solenoid placed along the z-axis

contains a magnetic flux . Which of the following vector potentials corresponds to the magnetic field at an arbitrary point (x, y, z)? [NET June 2014]

[email protected]

44

Physicsguide

11 oo : 02 : 02 _ _p_

y _p_ 21r x2 +y2 , 21r

X

x2+y2 ,

44

[email protected]

Physicsguide

@Sk J ahiruddin, 2020

(b) (Ax, Ay, Az) =

0

Electromagnetic Induction

_ _p_

_p_

y

X

21r x2+y2+z2, 21r x2+y2+z2, . 0 in free space ) t ravels t hrough an absorbing medium with di-

electric permit ivity given by E = ER + iEJ where Y... = v13. If ER t he skin depth is ~, t he ratio of the amplitude of the elect ric field E to t hat of t he magnetic field B, in the medium [NET June 2017] (in ohms) is (a) 1201r (b) 377 (c) 30v'21r (d) 301r Prob 1.10. A beam of light of frequency w is reflected from a dielectric metal interface at normal incidence. The refrac-

t ive index of the dielectric medium is n and t hat of t he metal is n 2 = n( l + ip). If the beam is polarised parallel to t he interface, t hen t he phase change experienced by the light ,...

..

.

r,

T...-,,......,

-Y

11 oo : oo : 16 a dielectric metal inter£ace at normal incidence. The refractive index of the dielectric medium is n and t hat of the metal is n 2 = n( l + ip). If the beam is polarised parallel to the interface, then the phase change experienced by the light [NET June 2014] upon reflection is (a) tan(2/ p) (b) tan- 1 (1/ p) (c) tan- 1 (2/ p)

(d) t an (2p)

Prob 1.11. An electromagnetically-shielded room is designed [email protected]

Physicsguide

6

@Sk J ahiruddin, 2020

ED Advanced: Part-1

7

so t hat at a frequency w = 10 rad/ s t he intensity of the external radiation that penetrates the room is 1% of t he in1 6 cident radiation. If a = 2~ x l0 (Dm)- is t he conductivity of t he shielding material, its minimum thickness should be (given t hat ln 10 = 2.3 ) [NET June 2014] (a) 4.60 mm (b) 2.30 mm (c) 0.23 mm (d) 0.46 mm

Prob 1.12. The magnetic field of t he T E 11 mode of a rectangular waveguide of dimensions a x b as shown in th figure is given by Hz = H 1cos(0.31fx)cos (0.41fy), where x and y are in cm. [NET June 2011]

11 oo : oo : 19

(i)The dimensions of the wave guide are (a) a = 3.33cm, b = 2.50cm (b) a = 0.40cm, b = 0.30cm [email protected]

7

@Sk J ahiruddin , 2020

Physicsguide

ED Advanced: Part-1

(c) a = 0.80cm, b = 0.60cm (d) a= 1.66cm, b = 1.25cm (ii) The entire range of frequencies f for which the T E 11 mode will propagate is (a) 6.0 GHz < f < 7.5GHz (b) 7. 5 GHz < f < 9.0GHz (c) 7.5 GHz < f < 12.0GHz (d) 7.5 GHz < f Prob 1.13. Consider a rectangular wave guide with transverse dimensions 2m x 1m driven with an angular frequency w

=

109rad/ s . Which transverse electric (TE) modes will

propagate in this wave guide? (a) TE10, TE01, and TE20 (c) T E o1 , T E10 , and T E11

[NET June 2015]

(b) TE10, TE11, and TE20 ( d) T Eo1, T E10, and T E 22

11 oo : oo : 21 propaga e 1n

1s wave gu1

(a) T E10, T Eo1, and T E 20 (c) TE01,TE10, and TE11

(b) T E10, T E11 , and T E 20 (d) T Eo1 , T E10, and T E 22

Prob 1.14. A waveguide has a square cross section of side

2a. For TM modes of wavevector k, the transverse electromagnetic modes are obtained in terms of a function 'l/J (x , y) which obeys the equation 32 32 w2 8 2 + 8 2 + ( 2 - k2) 'lf; (x,y) = 0 y

X

C

with t he boundary condition 'l/J (±a, y) = 'lf; (x, ± a) = 0. The frequency w of the lowest mode is given by [NET June j [email protected]

Physicsguide

8

@Sk J ahiruddin, 2020

ED Advanced: P art-1

2016]

(a) w2

= c2(k2 + ~2)

2 2 2 (c) w = c ( k +

;:2 )

2 2 2 (b) w = c (k + :: )

(d)

w

2

= c (k + ;:2) 2

2

Prob 1.15. The output intensity I of radiation from a sindJ wo gle mode of resonant cavity obeys dt = - Q I where Q is

t he quality factor of the cavity w 0 is the resonant frequency. The form of t he frequency spectrum of t he output is [JEST 2016] (a) Delta function (c) Lorent zian

(b) Gaussian (d) Exponential

11 oo : oo : 24 2016]

(a) Delta function (c) Lorent zian

(b) Gaussian (d) Exponential

Prob 1.16. The characteristic impedance of a co-axial ca[TIFR 2018] ble is independent of the

(a) dielectric medium between t he core and t he outer mesh (c) outer diameter (b) length of the cable (d) core diameter Prob 1.17. Consider a dipole antenna with length l, charge q and frequency w. The power emitted by the antenna at a large distance r is P. Now suppose the length l is increased

to v12l , the charge is increased to -/3q and the frequency is increased to ybw. By what factor is t he radiated power [email protected]

@Sk J ahiruddin, 2020

increased ?

Physicsguide

9

ED Advanced: Part-1

[TIFR 2018]

Prob 1.18. A hollow waveguide supports transverse elec1 t ric (TE) modes wit h t he dispersion relation k = - Jw 2 - w~in, C where Wmn is t he mode frequency. The speed of t he flow of

electromagnetic energy at t he mode frequency is June 2018] (a) C (b) W mn / k (C) 0 (d) oo

[NET

Prob 1.19. The permittivity tensor of a uniaxial anisotropic

11 oo : oo : 21 electromagnetic energy at the mode frequency is June 2018] (a) c (b) Wmn/k (c) 0 (d) oo

[NET

Prob 1.19. The permittivity t ensor of a uniaxial anisotropic 4£0 0 0 medium, in the standard Cartesian basis, is O 4£0 0 0 0 9£o where £o is a constant.The wave number of an electromagnetic plane wave polarized along t he x -direction, and propagating along the y -direction in t his medium (in terms of t he wave number k0 of t he wave in vacuum is [NET June 2019]

(a) 4co

(b) 2co

(c) 9co

i

(d) 3co

Prob 1.20. A metallic wave guide of square cross-section of side L is excited by an electromagnetic wave of wavenumber k . The group velocity of the T E 11 mode is [NET j [email protected]

10

Physicsguide

@Sk J ahiruddin , 2020

ED Advanced: Part-1

Dec 2019] (a) ckL / ✓~k2_L_2 _+_n_2

(C) C ✓k2 £ 2 kL

7r2

(b)

C

kL

✓k2 £2- 21r2

(d) ckL/ ✓k 2 L2 + 2 1r2

Prob 1.21. A medium (er > 1, µr = 1, a> 0) is semi-transparer t o an electromagnetic wave when [GATE 2020] (a) Conduction current >>. Displacement current

11 oo : oo : 29 Prob 1.21. A medium (Er > 1, µr

= 1, a> 0) is semi-transparer

to an electromagnetic wave when [GATE 2020] (a) Conduction current >>. Displacement current (b) Conduction current

> which Elives.

EW

00: 00: 37

Now, For metals,

a>>

EW

which gives, K,

Hence, skin depth, d ex

µaw

=

2

a~

Hence, option (b) is correct.

Sol 1. 7. The electric field in the region a < r < b

E=

1 qr 41rEo r 2

Then the potential difference between the inner and outer sphere 1

1

1

Vo= --q - - 41rEo a b 1 b -a

=

q

41rEo ab 41rE0 Voab q= b- a

Total current I flowing from inner to outer sphere

[email protected]

@Sk J ahirudd in, 2020

14

P hysicsguide

ED Advanced: Part-1

11 oo : oo : 39 @Sk J ahiruddin, 2020

ED Advanced: Part-1

I=

.....

J · da

a(r)E. da 1 A .!l_b2 f · drlf b2 41rEo b2

A 41rq b2 41rEo Aq 1 Eo b2 A 41rEo Voab b2 E0 b - a a

= 41rAb(b - a) Vo Vo=b(b - a) 1

41rAa

.

.

.

. b b- a

t Sol 1.8. We know, P1(t) = P1(0)e t ion, we get,

T

Here from t he ques-

t

which on calculation gives, t = 10s. Hence, option (d) is [email protected]

@Sk J ahiruddin , 2020

15

Physicsguide

ED Advanced: Part-1

11 oo : oo : 43 15

[email protected]

Physicsguide

@Sk J ahiruddin , 2020

ED Advanced: Part-1

correct.

Sol 1.9. Given,

E = ER+ iE1

a

and, f.R ~ =

v'3.

Hence,

= -/3

EW

Again, skin depth,

5 = Ao 41r (Given) Putting the expression for skin depth, we can get

And, Wave-vector K is given by, K

✓(k2

+ A: 2 )

which



gives,

k = wfaµ[ Hence,

E0 w 1 =V= - = - Bo k flqi, Ea c Ho= 4µ 0

c

= -

4

which on calculation gives the answer 301r Hence option (d ) is correct.

j [email protected]

16

Physicsguide

11 oo : oo : 4s IS

correc .

j [email protected]

Physicsguide

16

ED Advanced: Part-I

@Sk J ahiruddin , 2020

Sol 1.10. Parallel Incidence, so,

Where,

Hence, path difference,

Hence,

or, 2 0 = arctan (-) p

Hence, option (c) is correct. Sol 1.11. As we know, intensity decays by the formula,

2z I= I 0 e 6 Hence, I

- = lo

2z

-e 6

11 oo:oo:47 e

Hence,

2z

l

- =e lo

[email protected]

..

-

at

= -ax

which on calculation gives,

>.. = - atx Hence,

4

A

=

[email protected]



A

A -

ati P hysicsguide

37

@Sk J ahiruddin , 2020

ED Advanced: Part-1

Hence, opt ion (d) is correct. Sol 2.17. Here, From, Gauge Transformation ,

(- \7 >..) Hence,

1' = 1 +

J = q> + 9af Hence, option (a) is correct. 1

B

1,

1

x3,

Sol 2.18. If we try, = v7 x We will get , = B0 which satisfies, Coulomb-Gauge. Hence, opt ion (c) is cor-

rect. Sol 2.19. We know,

B=

v7 x

1.

which on calculation

11 oo : 01

: 44 A

- ati j [email protected]

Physicsguide

37

ED Advanced: Part-1

@Sk J ahiruddin, 2020

Hence, option (d) is correct. Sol 2.17. Here, From, Gauge Transformation,

(- v7 A) Hence,

J = q; + i 1

A' = A+

Hence, option (a) is correct.

A,

1J

A

x3,

Sol 2.18. If we try, = v7 x We will get , = B0 which satisfies, Coulomb-Gauge. Hence, option (c) is cor-

rect. Sol 2.19. We know,

t ion,

1J

= v7 x

A, which on calculation

tkµ 0 A 0c Hence, option (b) is correct -

Sol 2.20. Given, A

Gauge,

=

1 -

B x

2

x Now,

we know,Coulomb

v7.A = o Calculating, we can prove t his, i.e., valid in coulomb Gauge. If option (b) is correct.

j [email protected]

38

Physicsguide

11 oo : 01

: 47 A

- ati j [email protected]

Physicsguide

37

ED Advanced: Part-1

@Sk J ahiruddin, 2020

Hence, option (d) is correct. Sol 2.17. Here, From, Gauge Transformation,

(- v7 A) Hence,

J = q; + i 1

A' = A+

Hence, option (a) is correct.

A,

1J

A

x3,

Sol 2.18. If we try, = v7 x We will get , = B0 which satisfies, Coulomb-Gauge. Hence, option (c) is cor-

rect. Sol 2.19. We know,

t ion,

1J

= v7 x

A, which on calculation

tkµ 0 A 0c Hence, option (b) is correct -

Sol 2.20. Given, A

Gauge,

=

1 -

B x

2

x Now,

we know,Coulomb

v7.A = o Calculating, we can prove t his, i.e., valid in coulomb Gauge. If option (b) is correct.

j [email protected]

38

Physicsguide

11 oo : 01

: 49 A

- ati j [email protected]

Physicsguide

37

ED Advanced: Part-1

@Sk J ahiruddin, 2020

Hence, option (d) is correct. Sol 2.17. Here, From, Gauge Transformation,

(- v7 A) Hence,

J = q; + i 1

A' = A+

Hence, option (a) is correct.

A,

1J

A

x3,

Sol 2.18. If we try, = v7 x We will get , = B0 which satisfies, Coulomb-Gauge. Hence, option (c) is cor-

rect. Sol 2.19. We know,

t ion,

1J

= v7 x

A, which on calculation

tkµ 0 A 0c Hence, option (b) is correct -

Sol 2.20. Given, A

Gauge,

=

1 -

B x

2

x Now,

we know,Coulomb

v7.A = o Calculating, we can prove t his, i.e., valid in coulomb Gauge. If option (b) is correct.

j [email protected]

38

Physicsguide

11 oo : oo : 01

Problellls and Solutions EM Wave

• Ill

Sk J ahiruddin * Shreya Mitra Sandip Biswas

*Assistant Professor Sister Nibedita Govt. College, Kolkata Author was t he topper of IIT Bombay NI.Sc P hysics 2009-2011 batch He ranked 007 in IIT J AM 2009 and 008 ( JRF) in CSIR NET J une 2011 He has been teaching CSIR NET aspirants since 2012

1

@Sk J ahiruddin, 2020

E:NI Wave

11 oo : oo : 04 1

@Sk J ahiruddin, 2020

EM Wave

Contents 1

Electromagnetic Wave 1.1

Ans Keys

1.2 solution .

j [email protected]

3







































24







































27

2

Physicsguide

11 oo : oo : 06

j [email protected]

2

P hysicsguide

@Sk J ahiruddin , 2020

E:Nl Wave

Problems from NET, GATE, JEST, TIFR & JAM papers

1

Electromagnetic Wave

Pro b 1.1. The electric field of an electromagnetic wave prop-

agating t hrough vacuum is given by

E(7, t) = E 0 zcos(l00v'31rx -

[JAM 2009]

l001ry - wt )



(a) What is t he wave vector k? Hence find t he value of w.

(b) At t he t ime t = 0 t here is a point charge q wit h velocity ef' = v 0 x at t he origin. What is t he instantanious Lorentz force acting on t he part icle?

Prob 1.2. The magnetic field associat ed wit h t he electric field vect or£ = E 0 sin(k z - wt)] is given by [JAM 2010]

(a) (c)

B = -~ sin(kz-wt)l B = ~ sin(kz - wt)] 0

B B

(b) = ~0 sin(kz-wt)i (d) = ~ sin(kz - wt)k

11 oo :oo :og ro . . e magne IC e. . ~ associa e WI u...... e e ectric field vector £= E 0 sin(kz - wt)] is given by [JAM 2010]

B= B=

(a) (c)

(b) B = ~ 0 sin(kz-wt)i (d) B = ~ sin(kz - wt)k

sin(kz-wt)i ~ sin(kz - wt)] -~0

[email protected]

Physicsguide

3

@Sk J ahiruddin , 2020

EM Wave

Prob 1.3. A plane polarized electromagnetic wave in free space at time t=O is_iven by E (x,y) = IO]exp[i (6 x+8z)] .

(x, z, t) is given by [GATE 2012] (a) B(x , z , t) = ¼(6k - 8i ) exp [i(6x + 8z - lOct)]

The magnetic field

B(x, z , t) = ~(6k + 8i) exp [i(6x + 8z - l Oct)] (c) B(x , z, t ) = ¼(6k - 8i) exp [i(6x + 8z - et)] (d) B(x, z, t ) = ~(6k + 8i) exp [i(6x + 8z + et)] (b)

Prob 1.4. T he electric filed of an electromagnetic wave is [NET Dec 2013] given by

E=

E 0cos[1r(0.3x

+ 0.4y -

The associated magnetic field

l OOOt)]k

B is

+ 0.4y - l OOOt)]k (b) 10- 4 E 0 cos[1r(0.3x + 0.4y - 1000t)](4i - 3]) (c) E 0 cos[1r(0.3x + 0.4y - 1000t)](0.3i + 0.4]) 2 (d) 10 E 0 cos[1r(0.3x + 0.4y - 1000t)](3i + 4]) (a) 10- 3 E 0 cos[1r(0.3x

Pro b 1. 5. An electromagnetic wave of frequency w t ravels in t h x-direction through vacuum. It is oolarized in the v-

11 oo : oo : 11 2

(d)10 E 0 cos[1r(0.3x + 0.4y - 1000t)](3l + 43)

Pro b 1. 5. An electromagnetic wave of frequency w travels

in t h x-direction through vacuum. It is polarized in the ydirection and t he amplitude of the electric field is E 0 . With [email protected]

Physicsguide

4

ENI Wave

@Sk J ahiruddin , 2020

k = w / c where c is the speed of light in vacuum, the electric and magnetic fields are then conventionally given by [JEST 2013] (a) = E 0 cos(ky - wt) x andB = ~ cos(ky - wt) z (b) £ = E 0 cos(kx - wt)y and = ~ 0 cos(kx - wt) z (c)E = E 0 cos(kx -wt) z and = ~cos(ky-wt)y (d) £ = E 0 cos(kx - wt)x and = ~ cos(ky - wt)y

E

B B B

Prob 1.6. Consider two electromagnetic plane waves propagating in vacuum wit h their electric field vectors 1 = E 0 cos(kz - wt)i and

E2 = E 0 cos(kz + wt)i

£

[JAM 2005] (a) Evaluate the magnetic field vector corresponding to the

superposition of these two waves.

(b) Calculate the t ime averaged energy density as well as t he t ime averaged pointing vector for the resultant wave(The t ime averaged is carried over one period of oscillations)

Prob 1. 7. The electric field of uniform plane wave prop-

11 oo : oo : 14 t he t ime averaged pointing vector for the resultant wave(The t ime averaged is carried over one period of oscillations)

Prob 1. 7. The electric field of uniform plane wave propa ating in a dielectric non-conducting medium is given by

= x lOcos(61r x 107 t - 0.41rz)V /m The phase velocity of t he wave is ____ l0 8 m/s

[GATE 2014]

j [email protected]

5

Physicsguide

EM Wave

@Sk J ahiruddin , 2020

Prob 1.8. A monochromatic plane wave at oblique inciA

A

dence undergoes reflection at a dielectric interface. If ki, kr and n are the unit vectors in the directions of the incident wave, reflected wave and normal to t he surface respectively, which one of t he following expressions is correct? [GATE 2013] (a) (ki - kr) x n # 0 (b) ( ki - kr) .n = 0 A

A

A

A

A

A

(c) (ki x n) .kr = 0 Prob 1.9. A light source has a small filament at t he center

of a spherical glass bulb of radius 5cm and negligible thickness. If this source emits lOOWatts of power in the form of spherical electromagnetic waves, the r .m.s electric field E at t he surface of the bulb(in unit of Volt/ m) will be approximately [TIFR 2015] (a) 1094 (b) 109.4 (c) 15.47 (d) 1547 'II

-

...

_

........

,.

'

,

.

'

,

I"

, ,

,

.



11 oo : oo : 16 sp er1ca e ectrom agne 1c waves,

. .

at

t he surface of the bulb(in unit of Volt/m ) will be approxi[TIFR 2015] m ately

(a) 1094

(b) 109.4

(c) 15.47

(d ) 1547

Prob 1.10. Measurement of t he m agnitudes of the electric field ( E ) and t he m agnetic field (B ) in a plane-pola1·ized electromagnetic wave in vacuum leads t o t he following re[TIFR 2015] sults

[email protected]

P hysicsguide

6

EM Wave

@Sk J ahiruddin, 2020

aB at

aE

ay

8B

ay

1 aE --c2

at

at all points where t he measurement is made,.In this case the electric vector the magnetic vector and t he wave ➔

E,

B

vector k ( t he m agnitud e k) can be written in terms of unit vectors (x, y, z) along the Cartesian axes as

E = Ex, B = Bz, k = -ky (b) E = Ex, B = - Bz, k = ky (c) E = Ex, B = Bz, k = ky (d) E = - Ey, B = - Bz,k = - k x (a)

Prob 1.11. A beam of plane microwaves of wavelength 12 cm strikes t he surface of a dielectric at 45°. If t he refractive index of the dielectric is ; , what will be t he wavelength , in units of mm, of the microwaves inside the dielectric?

11 oo : oo : 19 Prob 1.11. A beam of plane microwaves of wavelength 12 cm strikes the surface of a dielectric at 45°. If t he refractive index of t he dielectric is ~, what will be t he wavelength , in units of mm, of the microwaves inside t he dielectric?

[TIFR 2017]

Prob 1.12. A plane electromagnetic wave traveling in a vacuum is characterized by the electric and magnetic fields

[TIFR 2013]

[email protected]

Physicsguide

7

EM Wave

@Sk J ahiruddin , 2020

£ = 1(301rVm-

fl = ](HoAm-

1

1

)expi(wt + kz)

)expi (wt + kz )

If w, k > 0, the value of H 0 must be

(a) 21r (b) 0.67

(c) 0.25

(d) 0.94

Prob 1.13. An electromagnetic wave wit h

£ (z, t) = Eocos(wt -

kz)l

is traveling in free space and crosses a disc of radius 2cm placed perpendicular to z-axis. If E 0

= 60Vm- 1, t he

aver-

age power,in Watt, crossing the disc along the z-direction is

[GATE 2007]

(a) 30

(b) 60

(c) 120

(d) 270

11 oo : oo : 21 1s trave 1ng 1n

ee space an

crosses a

m

placed p erpendicular to z-axis. If E 0 = 60Vm- 1, t he average power ,in Watt , crossing the disc along t he z-direction is

[GATE 2007]

(a) 30

(c) 120

(b) 60

(d ) 270

Prob 1.14. A beam of unnpolarized light in a medium with dielectric constant E1 is reflected from a plane interface formed with anot her medium of dielectric constant E2 = 3 E1. The two media have ident ical m agnetic permeability. If the an[NET Dec gle of incidence is 60°, t hen reflected light 2015] (a) is plane polarized p erpendicular to the plane of incidence j [email protected]

P hysicsguide

8

@Sk J ahiruddin, 2020

Eivl Wave

(b) is plane polarized parallel to the plane of incidence (c) is circularly polarized (d) has same polarization as t he incident light

Prob 1.15. The Fresnel relations between t he amplit udes of incident and reflected electromagnetic waves at an interface b etween air and a dielectric of refractive indexµ are E reflected

=

COS r

- µ COS 1, E incident cos r + µ cos i 11

11

E reflected

= µ COS r µ cos r

..1 •

.

I I

,



.

~

.

COS 1, E incident

+ cos i ...

.

.

..1

.

11 oo : oo : 24 E reflected

=

11

COST - µ

cos T

E reflected ..1_

The subscript



=

COS 'l E incident

+ µ cos i

11



µCOST -

µ cos T

COS 'l E incident

+ cos i

..1_

and ..l refer to polarization, parallel and nor-

m al to t he plane of incidence resp ectively. Here, i and r are t he angles o incidence and refraction respectively. [GATE

2007] (a) The condit ion for reflected ray t o be completely polarized is

(i) µcosi = COST (iii) µcosi

=

-COST

(ii) cosi = µcosT (iv) cosi

=

-µcosT

(b) For n ormal inciden ce at an air-glass interface wit h µ

=

1.5 the fraction of energy reflected is given by

[email protected]

Physicsguide

9

EM Wave

@Sk J ahiruddin, 2020

(i) 0.40

(ii) 0.20

(iii) 0.1 6

(iv) 0.04

Pro b 1 .16. A plane electromagnetic wave incident normally on t he surface of a m at erial is partially reflected . Measurements on the standing wave in t he region in front of t he interface.Show t hat t he ratio of the electric field amplit ude at the m axima and t he minima is 5. T he ratio of t he reflected intensity to t he incident intensity is

2014]

(a) 4/ 9

(b) 2/ 3

(c) 2/ 5

(d ) 1/ 5

[NET Dec

11 oo : oo : 26 interface.Show that the ratio of the electric field amplitude at the maxima and the minima is 5. The ratio of t he reflected intensity to t he incident intensity is [NET Dec 2014] (a) 4/ 9 (b) 2/ 3 (c) 2/ 5 (d) 1/5

Pro b 1.1 7. A plane electromagnetic wave propagating in air wit h E = (8i + 63 + 5k)ei(wt+ 3x- 4y) is incident on a perfectly conducting slab positioned at x=O. E field of the reflected wave is [JEST 2017]

(a) (-8i-6J-5k) ei(wt+3x+4y)

(b) (-8i+6J-5k)ei(wt+3x+4y)

(c) (-8i+ 6J - 5k )ei(wt-3x-4y)

(d) (-8i- 6J -5k)ei(wt-3x- 4y)

Prob 1.18. The electric field in an electromagnetic (EM) 4 6 wave is = vl61ri sin[21r(l0 z - 3 x 10 t)] . What is the intensity of the EM wave and the number of photons per sec-

E

j [email protected]

@Sk J ahiruddin , 2020

10

P hysicsguide

E:Nl Wave

ond falling on t he unit area of a perfectly reflecting screen kept perpendicular to t he direction of propagatio11? When a photon in this beam is reflected from t he screen, what is the impulse it imparts to the screen? Use t his to find t he pressure exerted by t he EM wave on t he screen. [JAM 2014]

Prob 1.19. A plane electromagnetic wave of frequency 5 x

11 oo : oo : 29 pressure exerted by the EM wave on t he screen. 2014]

JAM

Pro b 1.19. A plane electromagnetic wave of frequency 5 x 4 10 H z and amplit ude 103 V/ m t raveling in a homogeneous dielectric medium of dielectric constant 1.69, is incident normally at t he interface with a second dielectric medium of dielectric constant 2.25. The ratio of the amplitude of the transmitted wave to that incident wave is ____ . [JAM 2015]

Prob 1.20. MSQ: For an electromagnetic wave travelling in free space, the elctric field is given by

E = 100 cos( l 08t + kx )3 mV . Which of the following statements are t rue? [JAM 2015] (a) The wavelength of t he wave in meter is 61r (b) The corresponding magnetic field is directed along t he [email protected]

@Sk J ahiruddin , 2020

11

Physicsguide

EM Wave

posit ive z direction (c) The Poynting vector is directed along the positive zdirection (d) The wave is linearly polarized

Prob 1.21. The electric field of a light wave is given by

11 oo : oo : 32 direction (d) The wave is linearly polarized Prob 1.21. The elect ric field of a light wave is given by

E = E0 [isin(wt -

kz) +] sin(wt - kz -

1r ) ]

4

The polarization state of t he wave is [JAM 2015] (a) Left handed circular (b) Right handed circular (c) Left handed ellipt ical (d) Right handed ellipt ical

Prob 1.22. The electric field of an electromagnetic wave is

given by

E = (2k- 3]) x 10- sin[l 0 (x+ 2y+3z-,Bt )] 7

3

[JAM 2017]

The value of (3 is (c is the speed of light) (a) v'Mc (b) v'l2c (c) v'Ioc (d) if7c

Prob 1.23. In a non-conduct ing medium characterized by E = c0 ,µ = µ 0 and conductivity a = 0, t he elect ric filed (in

vm- 1 ) field

E

is given by = 20sin [l0 8t - kz]] . The magnetic in (Am- 1 ) is given by [GATE 2009]

H

[email protected]

P hysicsguide

12

ENI Wave

@Sk J ahiruddin, 2020

(a) 20k cos[108 t - kz]l 20 k sin[108t - kz]l (c) - 108µ0

6~~

8

(b) 1 0 sin[10 t - kz]] (d) - 20k cos[108t - kz]]

Prob 1.24 . Consider the propagation of electromagnetic waves in a linear , homogeneous and isotropic material medium , . , , .. . , ., .. . . . .

.

.

'

11 oo : oo : 34 a (c) -

cos 1 t - z i 20 8 t - kz]l k sin[10 108 µ0

sin 1 t - z j (d) - 20k cos[108t - kz]] 108µ a

/>

Prob 1.24. Consider the propagation of electromagnetic waves in a linear, homogeneous and isotropic material medium with electric permittivity E and magnetic permiability µ.

[GATE 2010] (a) For a plane wave of angular frequency w and propagation ➔ vector k propagating in the medium Maxwell's equations reduce to ➔➔ ➔➔ x = x = (i) k. E = O; k .H = O;

k E wEH;k H -wµ E

t.E = O; t.H = O; k x E = -wEH;k x H= wµE (iii) 7t. E = o; t. H = o; k x E = -wµH;k xH = wEE (iv)k.£ = O; k.H = O; kxE =wµH;kxfl = -wEE (ii)

(b) If E and µ assume negative values in a certain frequency range, t hen th directions of t he propagation vector and the Poynt ing vector in that frequency range are ➔ ➔ related as (a) k and S are parallel and are anti-parallel (b) ➔ ➔ (c) k and S are perpendicular to each other and makes an angle that depends on t he magni(d) t ude of E and lµI

f

S

k

S

k

S

j [email protected]

@Sk J ahiruddin, 2020

13

Physicsguide

EM Wave

Prob 1.25. A plane electromagnetic wave t raveling in free space is incident normally on a glass plat e of refractive index

3/2. If there is no absorption by the glass, its reflectivity is

00: 00: 37 EM Wave

Prob 1.25. A plane electromagnetic wave t raveling in free space is incident normally on a glass plate of refractive index

3/ 2. If there is no absorption by the glass, its reflectivity is [GATE 2012] (a) 4% (b) 16% (c) 20% (d) 50% Prob 1.26. The intensity of a laser in a free space is 150m W/m The corresponding amplit ude of t he electric field of t his laser 12 2 2 [GATE 2014] is _____ ~ (Eo = 8.854 X 10- c /N.m )

Prob 1.27. How much force does light from a 1.8W laser

exe when it is totally absorbed by an object? [JEST 2016] (a) 6.0 x 10- 9 N (b) 0.6 x 10- 9 N (c) 0.6 x 10- 8 N (d) 4.8 x 10- 9 N Prob 1.28. A plane electromagnetic wave has t he magnetic field given by (x, y, z, t) = B 0 sin[(x + y) ✓'i + wt]k where k is t he wave

B

A

A

A

number and i, jandk are the Cartesian unit vectors in x,y and z directions respectively. [GATE 2011] (a) The electric field [email protected]

@Sk J ahiruddin, 2020

above wave is given by

E(x, y, z, t ) corresp onding to the 14

Physicsguide

EM Wave

2

.

11 oo : oo : 40 @Sk J a hiruddin , 2020

EM Wave

above wave is given by A

A

(i) cBosin[(x + y) ~ + wt]i/2 A

A

+ y) ~+wt] i~ cBosin[(x + y) ~ + wt]i cBosin[(x + y) ~+wt]]

(ii) cB 0 sin[(x (iii) (iv)

(b) The average Poynting vector is given by

(i) cB

2 ~ -: 0 (i-J)

2µo v'2

(ii) _cB 2µo v'2

2 cB 0 (i+j)

2 cB 0 (i+j)

A

..111.) (

2 ~ -: 0 (i-J)

A

----

2µov'2

A

(.IV )

A

-----

2µov'2

Prob 1.29. The approximate force exerted on a perfectly reflecting mirror by an incident laser beam of power 1OmW at normal incidence is [JEST 2015] (a) 10- 13 N (b) 10- 11 N (c) 10- 9N (d) 10- 15N

Prob 1.30. A monochromatic plane wave in free space wit h electric field amplit ude of 1V / m is normally incident on a fully reflecting mirror . The pressure exerted on the mirror is ____ x 10- 12 P a (Upto two decimal places)(Eo = 8.854 x 12

2

10- C /N.m

2

[GATE 2017]

)

[email protected]

@Sk J a hiruddin , 2020

15

P hysicsguide

EM Wave

11 oo : oo : 43 15

jahir@physicsguide. in

Physicsguide

ENI Wave

@Sk J ahiruddin , 2020

Prob 1.31. A plane electromagnetic wave is propagating

in a lossless dielectric. The electric field is given by [NET June 2011] (x, y, z, t ) = E 0 (x + A z)exp[ik0 -ct + (x + J&z)] where c is the speed of light, E 0 , Aandk0 are constant and x and z are unit vectors along t he x and z axes. The relative dielctric constant of the medium Er and the constant A are

E

1

v'3 v'3

(a)

Er =

4 and A = -

(c)

Er =

4 and A =

(b) (d)

Er =

Er =

v'3 -v/3

4 and A =

4 and A =

1

Prob 1.32. An electromagnetic wave is incident on a waterair interface. The phase of the perpendicular component of t he electric field E _1_ of t he reflected wave into t he water is

found to remain the same for all angles of incidence. The phase of magnetic field H [NET June 2012] (a) does not change (b) changes by 31r / 2 (c) changes by

1r /

2

(d) changes by

1r

Prob 1.33. Consider the interference of two coherent elec-

t romagnetic waves whose electric field vector are given by £ 1 = iE0coswt and £ 2 = ] E0cos(wt + cp) where cp is the phase difference. The intensity of the resulting wave is given j [email protected]

16

Physicsguide

11 oo : oo : 4s E

1E 0 coswt and E2 = JE 0 cos (wt + ) where is the phase difference. The intensity of the resulting wave is given 1

=

j [email protected]

Physicsguide

16

@Sk J ahiruddin, 2020

EM Wave

by ~ (E ) , where(E intensity is 2

(a) 0 (b) E0 E5

2

)

2

is te time average of E . The total [NET Dec 2012]

(c) EoE5 sin

2

( d)

2

E0 E5cos cp

Prob 1.34. An electromagnetic wave is travelling in free space (of permittivity Eo) with electric field = kE 0 cosq(x-

E

ct). The average power (per unit area) crossing planes par[NET Dec 201 7] allel to 4x + 3y = 0 will be

(a) iEocE5 5

(b) EocE5

(c) ~EocE5 2

(d)

16 25

E0 cE5

Prob 1.35. A plane electromagnetic wave from within a dielectric medium (with E = 4E0 and µ = µ 0 ) is incident on its boundary with air, at z = 0. The magnetic field in the medium is 1f = ]Hocos(wt - kx - k./3z) , where w and k are posit ive constants.

The angles of reflection and refraction are, respectively, [NET Dec 2017] (a) 45° and 60° (b) 30° and 90° (c) 30° and 60°

(d) 60° and 90°

Prob 1.36. An electromagnetic plane wave is propagating with an intensity I = 1.0 x 105 W m - 2in a medium with

E

=

3Eo and µ

= µ0.

The amplitude of the electric field in-

11 oo : oo : 48 Prob 1.36. An electromagnetic plane wave is propagating with an intensity I = 1.0 x 10 5 W m - 2in a medium with

E

=

3Eo and µ

= µ0.

The amplitude of the electric field in-

[email protected]

17

P hysicsguide

EM Wave

@Sk J ahiruddin, 2020

side t he medium is ___ x 103 Vm- 1 (up to one decimal place) . [GATE 2018] 2 7 (Eo = 8.854 x 10-12F/m, µ 0 = 41r x 10- N A- , c 3 x l0 8ms- 1 ) Prob 1.37. The electric field of an electromagnetic wave is

E

= Eocos (k z + wt)l + 2Eosin(kz + wt)], where w and k

are positive constants. This represents [NET Dec 2016] (a) a linearly polarised wave t raveling in the positive zdirection (b) a circularly polarised wave traveling in the negative zdirection (c) an elliptically polarised wave t1·aveling in t he negative z-direction (d) an unpolarised wave traveling in t he positive z-direction Prob 1.38. A quarter wave plate introduces a path difference of A/ 4 between the two components of polarization parallel and perpendicular to t he optic axis. An electromagnetic wave wit h = (x + y)E0 ei(kz - wt) is incident normally on a quarter wave plate which has its optic axis

£

making an angle 135° wit h the x-axis as shown. 2018]

[GATE

11 oo : oo : so para e an perpen icu ar to t e optic axis. n e ectromagnetic wave wit h = (x + y)E0 ei(kz - wt) is incident normally on a quarter wave plate which has its optic axis making an angle 135° wit h the x-axis as shown. [GATE

£

2018]

[email protected]

18

P hysicsguide

@Sk J ahiruddin , 2020

EM Wave

''

'

The emergent electromagnetic wave would be (a) ellipt ically polarized (b) circularly polarized (c) linearly polarized with polarization as that of incident wave (d) linearly polarized but with polarization at 90 to that of t he incident wave

Prob 1.39. A light beam of intensity J0 is falling normally on a surface. The surface absorbs 20% of t he intensity and t he rest is reflected. The radiation pressure on t he surface is given by 0 , where X is _____ (up to one decimal place). Here c is the speed of light . [GATE 2018]

x:

11 oo : oo : s3 t he rest is reflected. The radiation pressure on t he surface is given by 0 , where X is _____ (up to one decimal place). Here c is the speed of light . [GATE 2018]

x:

Prob 1.40. The dispersion relation for electromagnetic wave where c t ravelling in a plasma is given by w 2 = c2 k 2 + and wp are constants. In this plasma, the group velocity is

w;,

j [email protected]

19

P hysicsguide

@Sk J ahiruddin, 2020

Eivl Wave

[JAM 2017] (a) proportional to but not equal to the phase velocity (b) inversely proportional to the phase velocity (c) equal to the phase velocity (d) a constant

• •

Prob 1.41. A plane electromagnetic wave, which has an electric field

E(";t, t)

= (P l + Q] ) exp iw(t - z) C

is passing through vacuum. Here P ,Q and w are all constants, while c is t he speed of light in vacuo. What is the average energy flux per unit time (in SI units) crossing a unit area placed normal to the direction of propagation of t his wave, in terms of t he above constants ? [TIFR 2018]

Prob 1.42. An electromagnetic wave of wavelength A is in-

11 oo : oo : 55 agation of t his wave, in terms of t he above const ants ? [TIFR 2018]

Prob 1.42. An electromagnetic wave of wavelength A is incident on a dielectric slab of t hickness t. If K is the dielectric constant of t he slab, t he change in phase of the emergent wave compared wit h the case of propagation in t he absence of the dielectric slab is [JEST 2018]

(a)

vK -

1

(b) 21r

[email protected]

(d)

(c) 21rt/ A

2It (vK -

1)

Physicsguide

20

EM Wave

@Sk J ahiruddin , 2020

Prob 1.43. In t he region far from a source, t he t ime dependent electric field at a point (r, 0,

o

o

Which on calculation gives,

IE0

2

o 2

1

Hence, intensity is,

1 2 coE0 2 Sol 1.34. Given ,

E

"'

Rn

kE0 cos q(x - et ) From this, we get A

11 oo : 01

: 47

Which on calculation gives, E 0

Hence, intensity is,

1 2 -2 EoE 0 Sol 1.34. Given,

magnetic field, vector,

E

B= -

= kE0 cos q(x - et ) From this, we get Eo cos(qx - wt )] From t his, poynting C

-

E5 S = - cos(qx - wt)i µo c A

and t ime averaged value of this quantity gives,

E5 ~ - -'l, 2µoc This value on t he given component gives ~ EocE5 Sol 1.35. We know,

sin ei sin 0t

n2

n1

Again kx l tan0i = kz J3 Which gives, 0i = 30 and 0t = 90 option (b) is correct . [email protected]

Physicsguide

39

EM Wave

@Sk J ahiruddin , 2020

1

- vcE5 I 2 culation, we get , Eo = 6.6 x l0 8 V / m Sol 1.36. As we know, I

=

=

1 1

- - -EE6 On cal2ffe

Sol 1.37. Since, amplit udes are different , Hence elliptically

polarised light moving in z- direction.

11 oo : 01 : so culation, we get, Eo = 6.6 x l0 V /m 8

Sol 1.37. Since, amplit udes are different, Hence elliptically

polarised light moving in z- direction. 7[

Sol 1.38. Incident electric field along - with x-axis. Now,

>.. / 4 path difference will introduces

7[

4

phase difference and 2 will make it Oo with optic axis. Hence, wave will be plane polarised. L

- 0.8Io

Sol 1.39. Radiation pressure, -2. -

Hence, value

C

C

of x= 1.8 Sol 1.40. In the plasma media, given, w 2 which gives

w k = -[l C

Then,

w Vp = -

c2k2

+ w2p

1 w 2

w

P] 2 2 C

= ----::====

k

l - (Wp)2 w

j [email protected]

40

Physicsguide

E:Nl Wave

@Sk J ahiruddin , 2020

And, group velocity, Vg

=

C

Hence, option (b) is correct.

11 oo : 01 : s2 And, group velocity, Vg

=

C

Hence, option (b) is correct. Sol 1.41. Given, A

z

A

(Pi+ Qj) exp iw(t - - ) C

Using t he formula,

-+

A

k X E B =-c -+

, we get, -+

l

z

C

C

A

A

B = - expiw(t - -)(P j - Qi)

Hence, Poynting Vector,

l 2 2 2 S = -cos (wt - kz)(P + Q )k µoc -+

A

Hence time-average of t hat gives

and, on t he direction of propagation its component will be,

[email protected]

Physicsguide

41

EM Wave

@Sk J ahiruddin , 2020

Sol 1.42. Assuming t he wave propgating along z-axis,

d>,

=

wd

kz - wt = -

-

wt

11 oo : 01

: 55

@Sk J ahiruddin, 2020

El\11 Wave

Sol 1.42. Assuming t he wave propgating along z-axis, wd

1 = k z - wt = -

- wt

C

Where d = thickness and wd wvfk 2 = k z - wt = - - wt = - -d - wt V

C

Changing in phase,

Hence, opt ion (d ) is correct.

Sol 1.43. Given , ➔

-6' (r, 0, )

A

= Eow

2

sin0 r

r cos w t - C

Now, calculationg the value of Poynting Vector, We get , 2

4

.

0

E 0w (sin ) 2 2 ( S = - - - - cos wt - k r ) µ 0c r Taking t he time average, we will get,

[email protected]

@Sk J ahiruddin, 2020

42

Physicsguide

El\11 Wave

11 oo :01 : s1

@Sk J a hiruddin , 2020

EM Wave

Hen ce, total power =

E5w4 2µ 0 cr 2 Which on calculation gives the answer ,

Hence,

option (b ) is correct.

Sol 1.44. Given ,

£ (z, t) = So, we get,

""=

1

3a

l Eoe-zf3a

and k

tancp

=

z

---wt

cos

1

v'3a

And we know,

10 v3a K,

= (-) = -

1

J3

k

Hence, B lags behind E by 30° Sol 1.45. As we know, E -

= - v (k A

x B) Calculating we get -

option (a) as the answer.

Sol 1.46. Given , n(w)

~

w

1-

2

wo

'

C

w2

vp

w5

-= l - j a [email protected]

(c)Sk J a,h ir11ddin. 2020

43

Physicsguide

EM Wa.ve

11 oo : 02 : oo 43

j [email protected]

P hysicsguide

@Sk J a hiruddin , 2020

E:Nl Wave

Which on calculation gives,

= ~ Again,

vp

Which gives, group velocity, v9 = 4c Hence, t he ratio gives t he answer 3. Hence, option (a) is correct. Sol 1.47. Given , .....

+ 4cos(kz -

E = 3 sin(kz - wt )x When, kz

wt )y

= 0, -,

A

E = -3 sin wti -,

A

+ 4 coswtj 1T

A

-I

A

,E When wt = 0, E = 4j , similarly, wt = -3i 2 Hence, elliptically polarized in counter-clockwise direction when seen travelling towards the observer -,

Sol 1.48. k

A

A

= 3j + 4k Hence, 3

-

4

Hence, 4

cos 0R = 5 [email protected]

44

Physicsguide

11 oo : 02 : 03 Hence,

cos 0R

4

=-

5

44

[email protected]

Physicsguide

EM Wave

@Sk J ahiruddin, 2020

Hen ce, power

21

P = - cos 0R C

Which gives on calculation the value 0.8 _,

Sol 1.49. Given , E = Ea exp [i (k 1 z - wt)]

x Hence,

Hen ce,

BR = - Ea exp [i (-k 1 z - wt)] y V1

Hence, option (c) is correct.

11 oo : 02 : os j [email protected]

44

Physicsguide

ENI Wave

@Sk J a hiruddin, 2020

Hence, power

21 P = - cos 0R C

Which gives on calculation the value 0.8 -;

Sol 1.49. Given , E = Ea exp [i (k 1 z - wt)]

x Hence,

Hence,

B R= - Ea exp [i (-k1z - wt )] y V1

Hence, option (c) is correct.

j [email protected]

45

Physicsguide

11 oo : oo : oo

Problellls and Solutions in Vectors Sk J ahiruddin Arnab Choudhury Assistant Professor Sister Nibedita Govt. College, Kolkata Author was the topper of IIT Bombay 1\11.Sc Pl1ysics 2009-2011 batch He ranked 007 in IIT J Al\11 2009 and 008 (JRF) in CSIR NET J une 2011 He has been teaching CSIR NET aspirants since 2012

1

@Sk J ahiruddin, 2020

Vect ors

11 oo : oo : 03 1

@Sk J ahiruddin , 2020

Vectors

Contents 1 Problems from NET , GATE, JEST, TIFR & JAM papers 1.1

Ans keys

1.2 Solutions .

j [email protected]

3







































24







































26

2

Physicsguide

11 oo : oo : 06

j [email protected]

2

P hysicsguide

@Sk J ahiruddin , 2020

1

Vectors

Problems from NET, GATE, JEST, TIFR & JAM papers A

Prob 1.1. (a) Consider a constant vector field iJ = v 0 k. _, Find any one of t he m any possible vectors il, for which V x _,

_,

U = V

(b) Using stokes theorem , evaluate t he flux associated with the field iJ through the curved hemispherical surface defined by x

2

+y +z = r 2

2

2

;

z > 0

[JAM 2005] Prob 1.2. For t he vector field

V = xz i-yz ]+z(x -y )k, 2

2

2

2

(a) Calculate t he volume integral of the divergence of out of t he region defined by -a

< x < a, -b
X

= 6.

4 6

6 9



Trace of the matrix is 13. So, sum of eigenvalues must be 13 and product must be 0. So, the eigenvalues must be, 0 and 13. 4 6 We can see t hat MT = = M. So, Mis a symmetric

6 9

matrix. 1 But as Det(M)=O, so, M- doesn 't exist. A matrix is invertible only when there exist the inverse matrix, So, M is not an invertible matrix. Finally, we know that eigenvectors of a symmetric matrix j [email protected]

68

@Sk J ahiruddin , 2020

Physicsguide

Linear Algebra

are always orthogonal. So, t hat must be true in this case also. Then the correct options are a, c, d. Sol 1.60. You need to know the following things (1) The

11 oo :03 : og are always orthogonal. So, t hat must be true in this case also. Then the correct options are a, c, d. Sol 1.60. You need to know the following things (1) The product of t he eigenvalues are the determinant of the matrix (2) Ort hogonal matrix means AAT = ~

So if Ax = AX (the eigenvalue equation)

So the eigenvalues will be of unit modulus, but t he eigenvalues could be real and imaginary. There are no problems on that. Now you can prove with the help of Mathematical Physics books t hat an orthogonal matrix with determinant 1 represents a rotation and has at least one eigenvalue = 1, whereas an orthogonal matrix with determinant = - 1 represents a reflection and has at least one eigenvalue = -1 . The other two eigenvalues in both cases are ei 0 and e-iB, . So wit h t hese informations in mind t he correct option is

(b) Sol 1.61. We know t he trace formula

[email protected]

@Sk J ahiruddin, 2020

Now, So,

Trace[A] = a. Det[eA] = ea.

69

Physicsguide

Linear Algebra

11 oo : 03 : 12 Linear Algebra

@Sk J ahiruddin, 2020

Now, Trace[A] = a. So, Det[eA] = ea. So, product of the eigenvalues of eA is ea, So, if one of t he eigenvalues is eA, t hen the product of the other two eigenvalues will be 1, so t hat t he product of the t hree eigenvalues can be eA . Sol 1.62. As At A

= 4~, the matrix A/ 2 is unitary. Hence

t he no of independent parameters of A will be same as of 4 x 4 unitary matrix. Now N x N unitary matrix can be written in the following form

2

2

2

So, there are N of a' sand N of b's, and 2N of parameters altogether. Unitary condition is given by

(ut u) ij

=

u itk ukj

= (aki

+ ibkj) + i (akibkj

- ibki) ( akj

= a ki akj

+ bki bkj

-

a kj bki) = 8ij

Hence a ki a kj

+ bki bkj =

akibkj -

akjbki

8ij

=0

l st eq is symmetric under i and j excahnge, so this equat ion actually shows N j [email protected]

@Sk J ahiruddin, 2020

+ (N

2

70

-

N) / 2 independent condiP hysicsguide

Linear Algebra

11 oo : 03 : 1 s [email protected]

70

@Sk J ahiruddin , 2020

P hysicsguide

Linear Algebra

t ions. 2nd eq is antisymmetric under i and j exchange, so this shows (N 2 - N) /2 independent condit ions. Hence, t here are N2 - N N2 - N N2 N +---+---= 2 2 independent conditions in t his unitary condition. Therefore, number of independent parameters in a N x N unitary matrix is 2 2 2 2N - N = N Hence there are 42

= 16 independent parameters.

Sol 1 .63. From Similarity Transformation, we know that

P matrix is called a similar matrix to A having same eigenvalue, determinant and trace. Here P matrix is the matrix is formed by placing the eigenvalues of A matrix in diagonal positions. And S matrix is formed by placing the corresponding eigenvecto1·s of A as columns. s-1 AS = P Now, from

s [s- 1 As] s-1 = sps- 1

A = S P s- 1

Here, t he eigenvalues are

[email protected]

71

Physicsguide

11 oo :03 : 1a

j [email protected]

71

Physicsguide

@Sk J ahiruddin , 2020

Linear Algebra

and t he eigenvectors are

So, 1

1

1

1 - 1

1 -2

2

1

0

s-1

3-1= 1

2 1

6

2

0

0 - 1 0

0

Now, we have to calculate

[email protected]

P=

&

0 0

1

2

1 -2 3 -3 0

72

Physicsguide

11 oo : 03 : 20

[email protected]

Physicsguide

72

@Sk J ahiruddin , 2020

A =SPS-

1

1 1 1 -1 1 - 2 0 1

1

-

6

1

1

1

-

1

1 1 - 1

1

6

-

Linear Algebra

1 -2 1

-

6

0

2 0 0 0 -1 0

2

2

2

1

1

-2

0

3 - 3

0 1

0

4 4 4 - 1 - 1 2

3 -3 0

6 0 6 0 6 6 6 6 0

1 0 1

0 1 1 1 1 0 S ol 1.64 . The Solution is easy. As we know and the question also says that vector along the axis of rotation remains unchanged during the rotation. So lets t ake an arbitrary

vector (x, y, z) and see the action of the matrix on this vector.

0 1 0

X

y

0 0 - 1 - 1 0 0

y

-z -x

z

Now as the vector must remain same to be a vector along

11 oo : 03 : 23 0

1 0

0 0 - 1 - 1 0 0

X

y

y

-z -x

z

Now as t he vector must remain same to be a vector along t he rotation axis, so x = y, y = -z, z = -x. So the general j [email protected]

73

Physicsguide

@Sk J ahiruddin, 2020

Linear Algebra

pattern of this vector would be (1,, 1- 1), when normalised, t his becomes }a(l , , 1 - 1). So the ans is (b ). Sol 1.65. You must know t hat (A+ B )- 1 # A- 1 + B-l. If

(A + B) is invertible then

A-1

+ B-l = B-l + A-l = B- 1 AA- 1 + B- 1 BA- 1 = B- 1 (A + B )A- 1

So A- + B- is the product of invertible matrices and thus is invertible, with inverse equal to 1

1

(A-l

+ B-1) - 1 = (B-l(A + B)A- 1) -l 1

= A(A+ B) - B Now t hink of the ans. Sol 1.66. You need to check individually. And you must know that (A B )t = B tAt and (A- 1 )t = (At)- 1

Now check option (a)

Again check option (b)

11 oo : 03 : 2s Now check option (a)

Again check option (b)

jahir@physicsguide. in

74

P hysicsguide

@Sk J ahiruddin, 2020

Linear Algebra

Now you can easily see that

(c- 1DC)(c- 1 n- 1 c) = ]_ Sol 1.67. T he matrix is not diagonalizable. So we can 't use t he similarity transformation method. Hence we use Caley Hamilton t heorem . The characteristric equation of t he matrix is

Using t his you can check that 8

A = 8A - 7 and so on. Hence

The m atrix also follow t he charact eristic equation.

A Hence

27

=

27A - 26li

11 oo :03 : 2a The matrix also follow t he charact eristic equation. A

27

=

27 A - 26 li

=

1 351

Hence A 21

0

1

Sol 1.68. If you can t hink just looking at the picture t hen

it is fine. Otherwise do as follows. [email protected]

Physicsguide

75

@Sk J ahiruddin , 2020

Linear Algebra

a b c Suppose the transformation matrix is d e f g h i This matrix transforms t he set of coordinates So a b c

1

0

f

1

g h i

0 0

a b c

0

f

1

0 0

g h i

0

1

a b c

0

2

d e f g h i

0

0 0

d e

d e

2

0

0 0 1 Hence you get the transformation matrix is S

=

1 0 0 .

0 1 0

11 oo : 03 : 30 it is fine. Otherwise do as follows. j [email protected]

Physicsguide

75

@Sk Jahiruddin, 2020

Linear Algebra

a b c Suppose the transformation matrix is d e f g h i This matrix transforms t he set of coordinates So a b c

1

0

f

1

g h i

0 0

a b c

0

f

1

0 0

g h i

0

1

a b c

0

2

f

0

g h i

2

0 0

d e

d e

d e

0

0 0 1 Hence you get the transformation matrix is S =

l O O .

0 1 0 Now we can further decompose the matrix but we need not to. We can see t hat t he determinant of S is - 1. And we also know t hat the determinant of rotaion mat rix is 1 and of reflection matrix is - 1. So S is composed of either two rotation or two reflection matrix but not of one rotation and one reflect ion. Only option (c) matches with the criteria. j [email protected]

76

Physicsguide

11 oo : 03 : 33 it is fine. Otherwise do as follows. j [email protected]

Physicsguide

75

@Sk Jahiruddin, 2020

Linear Algebra

a b c Suppose the transformation matrix is d e f g h i This matrix transforms t he set of coordinates So a b c

1

0

f

1

g h i

0 0

a b c

0

f

1

0 0

g h i

0

1

a b c

0

2

f

0

g h i

2

0 0

d e

d e

d e

0

0 0 1 Hence you get the transformation matrix is S =

l O O .

0 1 0 Now we can further decompose the matrix but we need not to. We can see t hat t he determinant of S is - 1. And we also know t hat the determinant of rotaion mat rix is 1 and of reflection matrix is - 1. So S is composed of either two rotation or two reflection matrix but not of one rotation and one reflect ion. Only option (c) matches with the criteria. j [email protected]

76

Physicsguide

11 oo : oo : oo

Problellls and Solutions in Differential Equations and Special Functions Sk J ahiruddin Arnab Choudhury Assistant Professor Sister Nibedita Govt. College, Kolkata Aut hor was t he topper of IIT Bombay 1\II.Sc Physics 2009-2011 batch He ranked 007 in IIT JAM 2009 and 008 (JRF) in CSIR NET J t1ne 2011 He has been teachi11g CSIR NET aspirants since 201 2

1

@Sk J ahiruddin, 2020

Diff. eq and Sp. ftmctions

11 oo : oo : 03 1

Diff. eq and Sp. functions

@Sk J ahiruddin, 2020

Contents 1 Problems from NET , GATE, JEST, TIFR & JAM papers

3

Ans keys .







































34

1.2 Solutions .







































37

1.1

j [email protected]

2

Physicsguide

11 oo : oo : 06

j [email protected]

2

P hysicsguide

@Sk J ahiruddin , 2020

1

Diff. eq and Sp. funct ions

Problems from NET, GATE, JEST, TIFR & JAM papers

Prob 1.1. A flat surface is covered with non-overlapping

disks of same size. W hat is the largest fract ion of t he area [JEST 2013] t hat can be covered?

(a)

3

(b)

5

(c)

1r

6

6 7 Prob 1.2. Given a function 1r

(d)

1T

2y3 f (x, t ) of both position x and



(where

j

(a) 02 f 8x 2

=

df (x, t ) x = dx) dt ' dt

(c) ~

(b) 8f

8x

X

[JAM 2012] (d) df dx

Prob 1.3. T he operator ( d - x)( d dx dx

d2 (a) d 2 X

d2 (c) dx~?

-

-

x2

d2 (b) d 2 X

-

is equivalent to

[JEST 2013]

x2 + 1 d2

d 2 x dX x + 1

+ x)

(d) d X 2

-

d 2 2x dX - x

Prob 1.4. Find t he solution of t he differential equation 2

d y dx 2

dy 5 + dx = O

11 oo :oo :og Prob 1.4. Find the solution of the differential equation

d2 y dx 2

dy 5 + dx = O

[email protected]

Physicsguide

3

Diff. eq and Sp. functions

@Sk J ahiruddin , 2020

with t he boundary condition

=2

y(O) = 2 and dy

dx

x=O

giving all steps clearly. Find the value of x where y = 0 [JAM 2014]

Prob 1.5. Consider the equation

!Yx =

2

y with the boundX

ary condition y( l ) = 1. Out of t he following t he range of x in which y is real and finite is [JAM 2015] (a) -oo < x < -3 (b) -3 < x < 0 (c) 0 < x < 3

(d) 3:::;

X


----------------------

X

t

t

(d)

(c) X

X

----------------------

t

t

Prob 1.10. Given the recurrence relation for the Legendre polynomials

Which of the following integrals has non-zero value? [GATE 2010] 1

(a) - 1 ,. 1

2 x Pn(x) Pn+1(x) dx

1

xPn(x )Pn+2(x )dx

(b) ,. 1

- 1

11 oo : oo : 16 Which of the following int egrals has non-zero value? [GATE 2010] 1

1 2

(b)

x Pn(x) Pn+1(x) dx

(a) - 1 1

- 1

1

xlPn(x )] 2 dx

(c)

xPn(x )Pn+2(x )dx 2

x Pn(x) Pn+2(x)dx

(d)

-1

-1

Prob 1.11. The solutions to the different ial equation dy

X

dx [email protected]

@Sk J ahiruddin,

y

+1

6

2020

P hysicsguide

Diff. eq and Sp. functions

are a family of [GATE 2011] (a) circles with different radii (b) circles with different centers (c) straight lines wit h different slopes (d) straight lines wit h different intercepts on t he y-axis

Prob 1.12. r (n + ½) is equal to [Given r (n + 1) = nf (n) and r (~) = y0r] [GATE 2013] 2n!~ c) (b) 2n!~ (d) n!~ ( 22n n !22n n !2n Prob 1.13. The solution of t he different ial equation d2 y dt2 - y = 0 subject to the boundary condit ions y(O) = 1 and y( oo) = 0 •

IS

[GATE 2014]

11 oo : oo : 19 dt2 - y = 0 subject to the boundary condit ions y(O) = 1 and y( oo) = 0 •

IS

[GATE 2014] (a) cos t + sin t (c) cost - sin t

(b) cos ht + sin ht (d) cos ht - sin t 2

2

Prob 1.14. If f( x) = e-x and g(x) = x e-x , then [GATE 2015] (a) f and g are differentiable everywhere (b) f is differentiable everywhere but g is not (c) g is differentiable everywhere but f is not [email protected]

7

@Sk J ahiruddin , 2020

Physicsguide

Diff. eq and Sp. functions

(d) g is discontinuous at x = 0

Prob 1.15. A function y(z) satisfies the ordinary different ial equation 2 1 ffi y + -y - -2y = 0 z z , where m = 0, 1, 2, 3, ... Consider the four stat ements P,Q,R,S as given below. II

I

• (P ) zm and z - m are linearly independent solut ions for all values of m

• (Q) zm and z - m are linearly independent solut ions for all values of m > 0 • (R) lnz and 1 are linearly independent solutions for m =O

11 oo : oo : 21 all values of m

• (Q) zm and z - m are linearly independent solut ions for all values of m > 0 • (R ) lnz and 1 are linearly independent solut ions for m = O

• (S) zm and lnz are linearly indep endent solutions for all values of m The correct opt ion for the combination of valid st at ement s •

lS

[GATE 2015] (a) P,R and S only (c) Q and R only

j [email protected]

@Sk J ahiruddin, 2020

(b) P and R only (d ) R and S only

8

P hysicsguide

Diff. eq and Sp. ftmctions

!~

Prob 1.16. Consider the linear differential equation xy. If y = 2 at x = 0 , then the values of y at x=2 is given by (GATE 2016] (a) e- 2 (b) 2e- 2 (c) e2 (d) 2e2 Prob 1.17. If [x] denotes t he greatest integer not exceed00 [JEST 2012] ing x, t hen f 0 [x]e- xdx (a) 1 (b) 1 ( c) e - 1 (d) 2 e e- l e e - 1 Prob 1.18. What are solutions to J" (x) - 2f' (x) + J(x) = O? [JEST 2014] (a) c1 ex/ x (b) c1 x + C2 / x (c) c1xex +c2 (d) c1ex + c2xex

11 oo : oo : 24 Prob 1.18. What are solutions to j" (x) - 2j' (x)

[JEST 2014]

O?

(a) (c)

+ f (x ) =

c1ex

/x

C1Xex

+ C2

(b) c1x + c2/x (d) c1ex + C2X ex 00

9

Prob 1.19. The value of the integral

dxx exp(-x 0

(a) 20160

(c) 18

(b) 12

2

)

is

[TIFR 2013]

(d) 24

Prob 1.20. The differential equation

d2y - 2dy dx 2 dx

- 0

+y -

[TIFR 2013]

(a) Aexp x + B xexpx (c) A ex p x + B exp(-x)

[email protected]

(b) Aexpx + B xexp(-x) (d) x A exp x + B xexp(-x)

Physicsguide

9

Diff. eq and Sp. functions

@Sk J ahiruddin , 2020

Prob 1.21. Consider t he differential equation

d2y = -4 dx 2 y

+

dy dx

with t he boundary condition t hat y(x) = 0 at x = ~- When 5 plotted as a function of x, for x > 0, we can say with certainty that the value of y [TIFR 2015] (a) oscillates from positive to negative wit h amplitude decreasing to zero (b) has an extremum in t he range O < x < 1 (c) first increases, then decreases to zero (d) first decreases, then increases to zero

11 oo : oo : 26 (a) oscillates from posit ive to negative wit h amplitude decreasing to zero (b) has an ext remum in t he range O < x < 1 (c) first increases, then decreases to zero (d) first decreases, then increases to zero Prob 1.22. The generating function for a set of polynomials in x is given by [TIFR 2015]

f (x, t) = (1 - 2xt + t 2 ) - 1 The t hird polynomial (order x 2 ) in this set is (a) 2x 2 + 1 (b) 4x 2 + 1 (c) 2x 2 - x (d) 4x 2 - 1 Prob 1.23. The generating function 00

F (x, t ) =

L Pn(x)tn n=O

for t he Legendre polynomials Pn(x ) is F (x, t ) = (1 - 2xt + 2 t )½ . The value of P 3 (- 1) is [NET Dec 2011] (a) 5/ 2 (b) 3/ 2 (c) +1 (d) -1 j [email protected]

10

Physicsguide

Diff. eq and Sp. functions

@Sk J a hiruddin , 2020

Prob 1.24. Let x 1 (t) and x 2 (t ) be two linearly independent solut ions of the differential equation 2

dx dx dt2 + 2 dt + f (t )x = 0 and let

dx 2 ( t ) W (t ) = X 1 (t ) dt -

If w(O) = 1 , then w(l ) is given by 2 (a) 1 (b) e ( c) 1/ e

( ) dx 1 ( t) X2

t

dt

[NET Dec 2011] 2 ( d) 1/ e

11 oo : oo : 29 and let

( ) dx 1 ( t) dx 2 ( t) W (t ) = X1 (t ) dt - X2 t dt If w(O)

= 1 , t hen w( l ) is given 2

(b) e

(a) 1

(

[NET Dec 2011] 2 ( d) 1/ e

by

c) 1/ e

Prob 1.25. Consider t h e different ial equation

2 d x dt2

dx + 2 dt

+X= 0

with the initial conditions x(O)

=

0 and !x

t

= 1.

The

t=O

solut ion (x( t )) attains it s maximum value when t is [NET

June 2014]

(a) l/2

(c) 2

(b) l

(d )oo

Prob 1.26. Given 00

L Pn(X )tn = (1 -

2xt + t

2

)-

1 2

l

,

for

t < l

n= O

t he value of P5 (-1) is

(a) 0.26

[NET June 2014]

(d) -1

(c) 0.5

(b ) 1

[email protected]

11

Physicsguide

Diff. eq and Sp. functions

@Sk J ahiruddin , 2020

Prob 1.27. The function X

-

2n+l

2

satisfies the differential equation 2

d f (a) x2 dx2 ,.()

n

df + x dx ,

+ (x2 + 1)f = 0 n

[NET Dec 2014]

11 oo : oo : 32

[NET Dec 2014]

satisfies the differential equation d2f df (a) x2 dx2 + x dx + (x2 + 1) f = 0

d2f 2 (b) x dx 2

df + 2x dx

+ (x

2

- 1) f = 0

d2f df (c) x2 dx2 + x dx + (x2 - 1) f = 0

Prob 1.28. Consider t he differential equation 2 dx dx dt2 - 3 dt

+ 2x =

0

. If x = 0 at t = 0 and x = 1 at t = 1, the values of x at

t = 2 is (a) e + 1 2

[NET June 2015]

(b) e + e 2

(c) e + 2

(d) 2e

Prob 1.29. Consider the differential equation!~ + ytan x = cos x . If y(O) 2017]

= 0, y(1r / 3) is - - -(two decimal) [GATE

[email protected]

@Sk J ahiruddin ,

2020

12

Physicsguide

Diff. eq and Sp. ftmctions

Prob 1.30. The function y(x) satisfies the differential equa. dy cos 1TX t1on x d + 2y = - -. If y (l) 1, the value of y(2) X X 1S [NET June 2017] (a)1r (b) l (d) l/4 (c) 1/2 •

11 oo : oo : 34 e unction y x sat1s es t e i erent1a equa-

1.30.

. dy t1on x dx is

+ 2y

cos1rx_ If y(l) x

(b) l

(a)1r

1, t h e va1ue of y (2) [NET June 2017]

(c)l/2

(d) l/4

Prob 1.31. Consider two part icles moving along t he x-axis. In t erms of their coordinates x 1 and x 2 , t heir velocit ies . dxi dx2 are given as dt = x 2 - xi and dt = xi - x 2 , respect ively. When they st art moving from their initial locations of x 1(0) = 1 and x 2 ( 0) = - 1, the t ime dependence of both x 1 and x 2 contains a t erm of t he form eat, where a is a con-

[JAM 2017]

stant. The value of a (an integer) is:

Prob 1.32. Consider the differential equation y'' + 2y' +y = 0. If y(O) = 0 and y' (0) = 1, t hen the value of y(2) is ?? (Specify ans up to two digits after decimal point) [JAM 2017]

Prob 1.33. The number of lineraly independent power series solut ions, around x = 0, of the second order linear . d2 y dy . equation x dx 2 + dx + xy = 0, is [NET Dec 2017] (a) 0 (this eq doesn 't have a power series solution)

(b) 1

(c) 2

(d) 3

j [email protected]

13

Physicsguide

@Sk J ahiruddin, 2020

Diff. eq and Sp. functions

Prob 1.34. Evaluate t he expression A

n!

Xn - 1

[TIFR 2017] Xn - 2

11 oo : oo : 36

Prob 1.34. Evaluate t he expression A

Xn - 1

[TIFR 2017] Xn - 2

n! 0

0

0

0

0

0

Prob 1.35. Write down x(t), where x(t) is the solution of the following differential equation [TIFR 201 7] d dt

+

d dt

2

+

1 x= l

with t he boundary condition

= O·

dx

dt

t=O

x (t) It=O

'

=-

1 2

Prob 1.36. Consider t he two equations [TIFR 2018] . x2 y2 (i) + = 1 (ii) x 3 - y = 1. 2 3 How many real simultaneous solutions does this pair of equations have? Prob 1.37. If y(x) satisfies t he differential equation y'' 4y' + 4y = 0, with boundary conditions y(O) = 1 and y'(O) = 0, t hen y(- 1/2) = [TIFR 2018] 2

(a) e

1 1 (b) 2 e + e

[email protected]

@Sk J ahiruddin, 2020

1

(c) e

14

e

(d) - 2 Physicsguide

Diff. eq and Sp. func tions

Prob 1.38. The solution of t he differential equation for y(t) : , ')

11 oo : oo : 39

Diff. eq and Sp. functions

@Sk J ahiruddin , 2020

Prob 1.38. The solution of the differential equation for y(t) :

0 and

!~

= 0 is:

[GATE 2010]

t=O

(a) ½cosh( t )+tsinh (t) (d) t sinh (t)

(b) - sinh( t)+t cosh( t)

(c) t cosh(t)

Prob 1.39. \iVhat is t he maximum number of extrema of x 4 x2) t he function j(x ) = Pk(x )e- ( 4 + 2 where x E (-oo, + oo)

and Pk(x) is an arbitrary polynomial of degree k? 2015] (a) k + 2

(b) k + 6

(c) k + 3

[JEST

(d) k

Prob 1.40. The Bernoulli polynomials Bn(s) are defined by xs

xe = ~ B (s) £ . Which one of t he following relations ex - l LI n n! is true? [JEST 2015] xex(l-s) xn (a) ex - l = L Bn(s) (n + l ) ! x ex(l-s ) x'n (b) - - = I:Bn(s)(- l)n( )! ex - l n+l . x(l-s) n (c) xe = I:Bn(-s) (-l) nx ex - l n! x(l-s) n (d) xe = L Bn(s)(- l )nx I ex - l n. [email protected]

@Sk J ahiruddin, 2020

15

Physicsguide

Diff. eq and Sp. functions

00: 00: 42

15

[email protected]

Physicsguide

@Sk J ahiruddin , 2020

Diff. eq and Sp. ftmctions

Prob 1.41. The solution of t he integral equation: X

f( x)=x -

dt

f (t)

0

has t he graphical form:

[TIFR 2014]

1

1

0.8

0.8

(a)

---._ 0.6

---._ 0.6

~

~

""-, 0 .4

~ 0.4

0.2

0.2

0 0

1

2

0

4

3

(b)

5

0

1

2

X

3

4

5

3

4

5

X

1.2

1

1 0.8

0.8

---._ 0 .6

---._ 0.6

~

(c)

~ 0 .4

~ 0 .4

0.2

0.2

0

0 0

1

2

(d)

~

3

4

0

5

1

2 X

X

Prob 1.42. Consider t he differential equation:

dy y+ dx

with t he boundary condition that y(x) = 0 at x = 1/5. When plotted as a function of x, for x > 0, we can say with j [email protected]

16

Physicsguide

11 oo : oo : 44 with t he boundary condit ion that y(x) = 0 at x = l / 5. When plotted as a function of x, for x > 0, we can say with j [email protected]

16

Physicsguide

Diff. eq and Sp. functions

@Sk J ahiruddin , 2020

certainty t hat the value of y

[TIFR 2015] (a) first increases, then decreases to zero (b) first decreases, then increases t o zero (c) has an ext remum in the range O < x < l (d) oscillat es from posit ive to negative with amplit ude decreasing to zero Prob 1.43. The function y(x) sat isfies the different ial equa-

t ion : x

!~= y(ln y -

ln x

+ 1)

wit h the init ial condit ion

y(l ) = 3. What will be the value of y(3) ? [TIFR 2016] Prob 1.44. Let Pn(x) (where n = 0, l , 2, ...) be a polynomial of degree n wit h real coefficient s, defined in the interval 4

2 < n < 4. If

Pn(X)Pm(x)dx = bmn, t hen 2

[NET June 2011]

(a) Po(x)

=

~ and P1(x) =

(b) Po(x)

=

~ and P1(x)

=

(c) Po(x) = ~ and P1( x) = (d) Po(x)

=

~ and P1(x) =

~( -3 - x) v'3(3 + x)

~(3 -

x)

11 oo:oo:47 (c) Po (x) = ½ and P1 ( x) = (d) Po (x) =

~ ( 3 - x)

}2 and Pl (x) =

[email protected]

~ ( 3 - x)

Physicsguide

17

Diff. eq and Sp. func tions

@Sk J ahiruddin, 2020

Prob 1.45. Let y(x) be a continuous real function in the range O and 21r satisfying the inhomogeneous differential equation:

. d2 y sin x dx 2

dy 1r + cos x dx = fJ x - 2

The value of :~ at the point x =

1r /

2 is

[NET June

2012] (a) is continuous (b) has a discontinuity of 3 (c) has a discontinuity of 1/ 3 (d) has a discontinuity of 1 Prob 1.46. Which of t he following is a self-adjoint operator in t he spherical polar coordinate system (r, 0, )? June 2012] in 8 in 8 (c) - sin 0 80 (a) - sin2 0 80

[NET

Prob 1.47. The function f (x) obeys the different ial equaand f (x) ➔ 0 as x ➔ oo. The value off (1r) is [NET Dec 20121

11 oo : oo : so Prob 1.47. The function f(x) obeys the differential equaand f (x) ➔ 0 as x ➔ oo. The value of f(1r) is [NET Dec 2012]

(a) e21r

(b) e- 21r

(

[email protected]

c) -e- 21r

(

d) -e 21ri Physicsguide

18

@Sk J ahiruddin , 2020

Diff. eq and Sp. functions

(We beleive there is a printing mistake. The given eq should d2f be dx 2 - ( 3 - 4i) f = 0) Prob 1.48. The graph of t he function f (x) shown below is best described by [NET Dec 2012]

1 r.------------, 0.75 0.5 0.25 ~

~

~

0.0 -0.25 -0.5 -0.75 -1 .0 .____ _ _ _ _ _ ___. o 1 2 3 4 5 6 7 8 9 10 X

(a) The Bessel function J0 ( x)

(b) cos x

(c) e- x cos x

(d) 1/x cosx Prob 1.49. The solut ion of the partial differential equation 32

.

-

32

-

11 oo : oo : s3 (a) The Bessel function J0 ( x)

(b) cos x

(c) e- x cos x

(d) 1/x cosx Prob 1.49. The solut ion of the partial differential equation

a2

a2

at 2 u(x, t) - ax 2 u(x, t) = 0 satisfying the boundary conditions u(O, t) = 0 = u(L , t) and

j [email protected]

19

Physicsguide

Diff. eq and Sp. ftmctions

@Sk J ahiruddin , 2020

initial conditions 21rx

t •

t=O

L

[NET June 2013]

1s:

L (a) sin(1rx/L ) cos(1rt/L ) + 1r sin(21rx/L) cos(21rt/L ) 2 (b ) 2 sin(1rx/L ) cos(1rt/L) - sin(1rx/L ) cos(21rt/ L ) L (c) sin(1rx/ L ) cos(21rt/ L ) + - sin(21rx/ L ) sin(1rt / L ) 7r

(d) sin(1rx / L ) cos( 1rt / L ) +

L 1r sin(21rx / L) sin(21rt / L ) 2

Prob 1.50. The solution of the differential equation : x 2 with t he initial condition x(O) to:

(a) 1

(b) 2

= 1 will blow up as t t ends [NET June 2013]

(c) 1/ 2 (d) oo

Prob 1.51. The function

0 is described by: 2 2 2 2 (a) e - (x - v t ) (b) e - (x- vt)

j [email protected]

[NET June 2015] (

C)

21

~

e-(x-vt)

2

Diff. eq and Sp. functions

+ e-(x+vt)

Prob 1.54. If y

(a) ln

(c) ln

y

+

1

y- l y- l y+ l

2

P hysicsguide

@Sk J ahiruddin , 2020

(d)

!e - (x - vt) + !e - (x+vt) 2

=

2

1

h

.

tan x

, t hen x 1s

[NET Dec 2015]

y- l (b) ln y+ l y+ l (d) ln y- l

Prob 1.55. The solution of t he different ial equation dx

dt

= 2J1 - x2

with initial condit ion x = 0 at t = 0 is (a) x

= sin 2t

O ::; t
~ and x = 1 t > ~

t ~ ~

11 oo : 01 : oo with initial condition x = 0 at t = 0 is [NET Dec 2015] (a) x = sin 2t O :::; t < ~ and x = sinh 2t t ?: ~ (b )x = sin 2t O :::; t < ~ and x = 1 t > ~ (c)x = sin2t O < t < j and x = 1 t > l (d)x =l -cos 2t, t >O

Prob 1.56. The Hermite polynomial Hn(x) satisfies t he differential equation: 2

d Hn dHn ( ) dx 2 - 2x dx + 2nHn x = 0 The corresponding generating function

[email protected]

@Sk J ahiruddin , 2020

satisfies the equation: 2

8G BG BG (a) 8x2 -2x ox +2t at = 0

22

Physicsguide

Diff. eq and Sp. functions

[NET Dec 2015]

(b)a2c - 2xac -2t2aG 8x 2 ox at

=0

2 2 8 G BG BG a G aG a G (C) f) x~? - 2x f) X + 2 f) t = 0 (d) 8x 2 - 2x OX + 2 axat = 0 Prob 1.57. The Gauss hypergeometric function F (a , b, c, z ) defined by the Taylor expansion around z = 0 as F (a, b, c, z) = 2

f n=O

a(a + 1) ... (a + n - l )b(b + l ) .. . (b + n - 1) zn c(c + 1) ... (c + n - l )n!

satisfies the recursion relation: [NET June 2016] d C (a) rl7. F (a, b, c, z) = n.h F( a - 1, b - 1, c - 1, z)

11 oo : 01

: 03

L a(a + I)... (a + n -

I )b(b + 1) ... (b + n - 1) zn

c(c + l ) ... (c + n - l )n!

n =O

satisfies the recursion relation: [NET June 2016] d C (a) dz F (a, b, c, z) = abF (a - 1, b - l , c - 1, z ) d C (b)dzF(a,b,c,z) = abF(a+ l , b+ l ,c+ l ,z) ab d (c) d F (a, b,c, z ) = -F(a - l , b - 1,c- l , z) Z C d ab (d)-F (a, b,c,z) = - F (a+ l , b+ l ,c+ l , z) c dz Prob 1.58. The integral equation:

cp(x, t) = A

dwdk e-ik(x-x')+iw(t-t') (21r )2 w 2 - k 2 - m 2+·iE qi(x' ,t')

dx' dt'

[NET June

is equivalent to the differential equat ion: 2016]

[email protected]

Diff. eq and Sp. ftmctions

@Sk J ahiruddin , 2020

a2

(a) at 2 a2

(b) at 2

P hysicsguide

23

a2

2

1

3 )

+ ax 2 - m + iE cp(x, t ) = - 6 Acp (x, t a2 2 . 2 ax 2 + m - iE cp(x, t) = +Acp (x, t )

Prob 1.59. A stable asymptotic solution of t he equation 3 X n +l = 1 + is x = 2. If we take X n = 2 + En and 1 +xn Xn+ l = 2 + En + l where En and En+l are both small, t he ratio E'Yl-1-1

.



r -

---

_

_

_

.

_,

11 oo : 01 : os 8t 2

-

8x 2

+m

-



1,E

X

'

t =-

Prob 1.59. A stable asymptotic solution of the equation 3 Xn+l = 1 + is x = 2. If we take Xn = 2 + En and 1 +xn Xn+l = 2 + En +l where En and En+l are both small, t he ratio En+ l is approximately: [NET Dec 2016] En

1 2 (c) -- (d)-3 3 Prob 1.60. Let f (x) be the solution of the heat equation 1 (a) -2

1 (b) -4

aatf

82

f . d. . Th . . . 1 d. . t 0 = ax2 1n one 1mens1on. e 1n1t1a con 1t1on at = is f (x, 0) = e- x for -oo < x < oo. T hen for all t > 0, D

2

00

1f]:

dxe- ax 2 =

f (x, t) is given by [Useful integral -oo

a [NET D ec 2016] 2

1 x (b) 1 + 2Dt exp{ - 1 + 2Dt}

1

x

2

x2

(d) exp{---} 1 + Dt

(c) 1 + 4Dt exp{- I + 4Dt} j [email protected]

@Sk J ahiruddin, 2020

Physicsguide

24

Diff. eq and Sp. functions

Prob 1.61. Consider the differential equation

dG(x) + kG(x) = b(x) dx where k is a constant. Which of the following statements is t rue? [JEST 2013] (a) Both G(x) and G' (x) are continuous at x = 0 (b) G(x) is continuous at x = 0 but G'(x) is not. (c) G (x) is discont inuous at x = 0 I

, \

f"'""T"'I ,

,

,.--.,/ 1

\

11 oo :01 : oa true? [JEST 2013] (a) Both G(x) and G' (x) are continuous at x = 0 (b) G(x) is continuous at x = 0 but G' (x) is not. (c) G (x) is discont inuous at x = 0 (d) The cont inuity propert ies of G (x) and G' (x) at x = 0 depend on the value of k

Prob 1.62. Given that

t he value of H4 (0) is (a) 12 (b) 6

[NET June 2013]

(c) 24

(d) -6

Prob 1.63. Consider t he differential equation

!; + ay =

with t he initial condition y(O) = 0. T hen the Laplace t ransform Y (s) of t he solut ion y(t ) is [NET Dec 2017] 1 1 e-bt

(c) a(s + b)

(a) (s + a)(s + b) [email protected]

@Sk J ahiruddin , 2020

25

Physicsguide

Diff. eq and Sp. functions

e-a, - e-b

(d)

b- a

Prob 1.64. T he generating function G(t , x) for t he Legendre polynomial is

11 oo : 01

: 11

Prob 1.64. The generating function G(t,x) for t he Legendre polynomial is

G(t,x) for xl < 1. If the function

=

f (x)

J

1

00

1 - 2xt + x 2

=

L xn Pn(t) 0

is defined by the integral equation

fox f(x')dx' = xG(l , x), it can be expressed as [NET Dec 2017] 00

(a)

L

00

(b)

xn+m Pn(l )Pm(l /2)

xn+m Pn( l )Pm(l )

n,m=O

n,m=O

00

00

(c)

L

L

Xn-mPn(l)Pm(l ) n,m=O

n,m=O

Prob 1.65. For which of the following conditions does the 1

Pm(x)Pn(x)dx vanish for m =/- n where Pm(x)

integral 0

and Pn(x) are t he Legendre polynomials of order m and order n respectively? [JEST 2018] (a) all m , m =/- n (b) m - n is an odd integer

26

jahir@physicsguide. in

Physicsguide

@Sk J ahiruddin , 2020

Diff. eq and Sp. functions

(c) m - n is a nonzero even integer

(d)n= m± l Prob 1.66. If y(x) satisfies

dy = ,,,fl

-4-

/ 1110- ,, ,\ 21

11 oo : 01

: 13

(c) m - n is a nonzero even integer

(d)n=m± l Prob 1.66. If y(x) satisfies

:~ = and y(O)

(a) 0

y [1 + (log y)

2

]

= 1 for x > 0, t hen y( 1r / 2) is (b) 1

[JEST 2018] (d) oo

(c) 1r/2

Prob 1.67. Which one of the following curves correctly represents (schematically) the solut ion for t he equation

df dx

+ 2f

=

3;

f (0) = 07 [JAM 2018]

j [email protected]

27

P hysicsguide

@Sk J ahiruddin, 2020

f(x)

Diff. eq and Sp. ftmctions

f(x) (a)

(b)

1/2 ----------------------

3/2 ----------------------

11 oo : 01

: 16 Diff. eq and Sp. functions

@Sk J ahiruddin , 2020

f(x)

f(x) (a)

(b)

1/2 ----------------------

3/2 ----------------------

X

X

f(x)

f(x) (c)

(d)

1/2 ----------------------

3/2 ---------------- ------

X

X

Prob 1.68. MSQ Let J(x) = 3x6 - 2x 2 - 8. Which of the following statements is (are) true? [JAM 2018] (a) The sum of all its roots is zero. (b) The products of its roots is -8/3 (c) The sum of all its roots is 2/3. (d) Complex roots are conjugates of each other. Prob 1.69. Consider the following ordinary differential equat ion dx

2

dt [email protected]

28

@Sk J ahiruddin, 2020

wit h the boundary condit ions

_ dx= O

dt Physicsguide

Diff. eq and Sp. functions

11 oo :01

: 1a

Diff. eq and Sp. functions

@Sk J ahiruddin , 2020

wit h the boundary condit ions x (t = 0) = 0 and x (t = 1) = 1

The value of x(t) at t

= 2 is

[NET June 2018]

(a) ✓e - 1

(c)

Je + 1 2

Prob 1. 70. In the function Pn(x) e- x of a real variable x, Pn(x) is a polynomial of degree n . The m aximum number of ext rema that t his function can have is [NET June 2018]

(a) n

+2

(b) n - I

(c) n

+I

(d) n

Prob 1.71. The polynomial f(x) = 1 +5x+3x2 is written as a linear combination of the Legendre polynomials

as f( x) 2018]

(a) 1/ 4

= I:n en Pn(x) . The value of co is (b) 1/ 2

(c) 2

[NET Dec

(d) 4

Prob 1. 72. In terms of arbitrary constants A and B , the general solution to the differential equation j [email protected]

@Sk J ahiruddin, 2020

29

Physicsguide

Diff. eq and Sp. functions

11 oo : 01

: 21

[email protected]

P hysicsguide

29

@Sk J ahiruddin, 2020

Diff. eq and Sp. functions

dy x dx2 + 5x dx 2d2y

+ 3y = 0 [NET Dec 2018]

is

A

(a) y = -

X

+ B x3

(c) y= A x+

(b) y =Ax+~ A B (d)y=-+ 3

3 Bx

X

X

Prob 1. 73. A set of polynomials of order n are given by t he formula x2

2 The polynomial P7 ( x) of order n

= 7 is

[TIFR 2019]

(a) x - 21x + 105x - 105x 4 2 6 5 3 (b) x - 21x + 105x - 105x + 21x + x 7 (c) x - 21x 5 + 105x 3 - 105x + 21 (d) x

7

5

7

5

-

21x

3

+ 105x + 35x 3 4

105x

Prob 1. 7 4. For the differential equation

d2y dx~9

y

-

n( n + 1) X 2 = 0

where n is a constant, t he product of its two indep endent solut ions is [GATE 2019]

(b) [email protected]

X 30

Physicsguide

11 oo : 01

(b)

: 23

X

j [email protected]

P hysicsguide

30

Diff. eq and Sp. ftmctions

@Sk J ahiruddin, 2020

Prob 1. 75. The differential equation

dy x dx - xy = exp (x) where y = e2 at x = l , has t he solution y = [TIFR 2019] (a) exp (1 + x) (1 + ln x) (b) exp (x) ln x + exp (1 + x) (c) (1 - x) exp (x) + exp (1 + x) (d) exp ( x 2 + x ) Prob 1. 76. The Euler polynomials are defined by

What is t he value of E 5 (2) + E s(3)?

[JEST 2019]

Prob 1. 77. The solution of t he different ial equation

dy x dx + (l + x)y = e-x wit h t he boundary condition y(x = 1) = 0, is [NET June 2019]

(a) (x - 1) e-x (b) (x ~ 1) e-x (c) (1 - x) e-x

(d)(x- l )2e-x

x2 Prob 1. 78. The equation of motion of a forced simple harX

X

monic oscillator is x+w 2 x = A cos Dt where A is a const ant. At resonance n = w the amplit ude of oscillations at large

11 oo : 01

: 26

(a) x - l e- x (b) x ~ I e- x (c) 1 ~ x e- x (d )(x- 1)2e- x X

X

X

Prob 1. 78. The equation of motion of a forced simple har2 monic oscillator is x+w x = A cos Dt where A is a constant. At resonance n = w t he amplit ude of oscillations at large [email protected]

31

Physicsguide

@Sk J a hiruddin , 2020

D iff. eq and Sp. functions

[NET June 2019]

t imes (a) sat urates to a finit e value (b) increases wit h time as v't (c) increases linearly wit h time (d) increases exporent ially wit h t ime

Prob 1. 79. Which one of t he following is a solut ion of

for k real?

(A) e- kx

(B) sin kx

(C ) cos kx

[GATE 2020] (D ) sinh x

Prob 1.80. T he solut ion of the different ial equat ion y'' 2y' - 3y = e2t is given as C 1 e-t + C 2 e2 t + C3e 3t T he values of the coefficieients C1, C2 and C3 are: [JEST 2020] (A) C1, C2 and C3 are arbitrary (B) C1 , C3 are arbit rary and C2 = - 1/ 3 (C) C 2 , C3 are arbit rary and C 1 = -1/ 3 (D) C1, C2 are arbitrary and C3 = -1/ 3 Pro b 1. 81. Some bact eria are added to a bucket at t ime 10 am . The number of bacteria doubles every minute and reaches a number 16 x 1015 at 10: 18 am. How many seconds

11 oo : 01

: 29

(D) C 1, C2 are arbitrary and C3

==

-1/3

Prob 1. 81. Som e bacteria are added to a bucket at t ime 10 am. The number of bacteria doubles every minute and 15 reaches a number 16 x 10 at 10: 18 am. How m any seconds after 10 am were there 25 x 10 13 bacteria? [JEST 2020]

32

j [email protected]

P hysicsguide

Diff. eq and Sp. funct ions

@Sk J ahiruddin , 2020

Prob 1.82. Which one of the following funct ions has a discontinuity in the second derivative at x == 0, where x is a real variable?

(A) f (x) == x 3 2 (D) f (x) == x

(B) f( x) == x x

Prob 1.83. lim x x is equal to

[JAM 2020]

x ➔O+

(A) 0

(B)oo

(C) e

[JAM 2020] (C) f (X) == cos (IX )

(D) 1

Prob 1.84. If a function y(x) is d escribed by t h e initial2

d y 1 bl va ue pro em , dx 2 y(O) == 2, and

dy

+

5 dx dy

+

6y == 0 , wit . h 1n1t1a . . . 1 con d·t· 1 ions

== 0, t h en the value of y at x == 1 is

dx x=O - - - - - - - - ? (R ound off to 2 decimal places) [JAM

2020]

11 oo : 01

[email protected]

: 32

Physicsguide

33

Diff. eq and Sp. functions

@Sk J ahiruddin , 2020

1.1

Ans keys Ans keys

1.1. d

1.17. a

1.2. b

1.18. d

1.3. b

1.19. b

1.4.

-½ ln 6

1.20. a

1.5. d

1.21. b

1.6. 27.0

1.22. d

1. 7.

C

1.23. d

1.8. d

1.24 . d

1.9. a

1.25. b

1.10. d

1.26. a

1.11. a

1.27.

C

00 : 01 : 34

1.9. a

1.25. b

1.10. d

1.26. a

1.11. a

1.27.

1.12.

1.28. b

C

C

1.13. d

1.29. 0.51 to 0.53

1.14. b

1.30. d

1.15.

1.31. -2 to + 2

C

1.32. 0.25 to 0. 29

1.16. d [email protected]

Physicsguide

34

@Sk J ahiruddin, 2020

Diff. eq and Sp. ftmctions

1.33. b

1.52.

1.34. A n

1.53. d

1.35. e- 2t - 2e-t + ~

1.54. d

1.36. 2

1.55.

1.37. a

1.56. a

1.38. d

1.57. d

1.39.

1.58. d

C

C

C

1.40. d

1.59.

C

1.41. b

1.60.

C

1.42.

1.61. b

C

1.4 3. 81

1.62. a

1.44. d

1.63. a

1.45. d

1.64. b

00 : 01 : 37 1.42.

1.61. b

C

1.43. 81

1.62. a

1.44. d

1.63. a

1.45. d

1.64. b

1.46.

C

1.65.

1.47.

C

1.66. d

1.48. a

1.67. b

1.49. d

1.68. a,b,d

1.50. a

1.69.

C

1.51. a

1. 70.

C

j [email protected]

C

35

P hysicsguide

@Sk J ahiruddin , 2020

1. 71.

C

Diff. eq and Sp. functions

1. 78.

C

1. 72. d

1. 79. a

1. 73. a

1.80. b

1. 7 4. b

1.81. 720

1. 75. b

1.82. b

1. 76. 64

1.83. d

1. 77. a

1.84. 0.60 to 0.62

11 oo : 01

[email protected]

: 39

Physicsguide

36

Diff. eq and Sp. func tions

@Sk J ahirudd in, 2020

1.2

Solutions

Sol 1.1. Let us consider a hexagon , t hat can be divided in 6 triangles of equal arm length. Now, we try to cover up t he area with circles of radius ; as shown in the figure . . . Now, t he area of a triangle 1s,

=

1 x ax 2

v'3a 2

2

3v'3a 2

11 oo : 01

: 42

a

l

Again we can see that , in each triangle t he shaded region is covered by the circle which is same as half of the area of a circle. So, the shaded region in any t riangle is I

-7r

2

a

2

1ra2

-

2

8

So, t he fraction of area covered , [email protected]

~1r_il 4

= v'33,ii 3

2

2\1'3 Physicsguide

37

@Sk J ahiruddin , 2020

Diff. eq and Sp. functions

Sol 1.2.

implies f = f (x, t ) aJ aJ df = ax dx + at dt df = j = af ax + af dt ax at at aJ. aJ = ax x + &t Hence

.

7r



aJ at

aJ ax

Sol 1.3. Let us consider a t rial function x .

11 oo : 01

: 44

Hence



of

of

ot

ox

Sol 1.3. Let us consider a t rial function x .

d - - x dx

d dx +x

f = =

2

. d y Sol 1.4. Given that, dx 2

d df - - x dx +xf dx d2 f d 2 dx2 - x dx - x f + f d2 dx2 - x2 + 1 f

dy + 5 dx

j [email protected]

=0

Physicsguide

38

Diff. eq and Sp. ftmctions

@Sk J ahiruddin, 2020

Now we consider that,

d + x dx

!~ = v dv dx

+ 5v = 0

dv = - 5dx V

In v= -5x

+c

11 oo : 01

: 47 dv = -5dx V

In v= -5x + c dy Now at x= O v = - = 2 dx V ln - = - 5x

c

v

2

dy dx

=

=

Now at x=O

y = 2

y

2e- 5x

5

12

2

+d

= - - -e

d = 12 5 - Sx

5 Now a t y= O e-5x = 6

[email protected]

=

2e- 5x

2 --e-5x

y

= ln 2

5

=}

x =

1 -- ln 6 5

39

P hysicsguide

Diff. eq and Sp. functions

@Sk J ahiruddin, 2020

Sol 1.5. dy

y2

dx dy

x dx

y2

X

1

- - = ln x+c y

11 oo : 01

: 49

dx dy

x dx

y2

X

1 - -= ln x+c y



Now, at x = l , y= l

c= - 1

1 - - = ln x- 1 y

1 y= 1 - ln x

So, t o have y value real and finit e, we need to have a range of x that doesn 't include 1 or 0. So, the only option will be

3< -

< 00 .

X -

. . df 8f 8f df Sol 1.6. Using Chain Rule, - = a + a x d dx x y x 2 3 3 Now, here f (x, y) = x + y and y = x + 1. J _ 2 . J _ 2 df _ So, ax - 3x , ay - 3y , dx - 2x .

a

a

• • •

• • •

j [email protected]

!~ = 3x df dx

2

=

2

2

+ 3(x + 1) x 2x 3 + 6 x (1 + 1)2 = 27

x=l

40

@Sk J ahiruddin, 2020

Physicsguide

Diff. eq and Sp. functions

Sol 1. 7. We can write t he equation as, 2x dy 3 3 1 d2 y - 2 - 2x - - -2 + - ( - + 1) - - y =O 2 dx (1 - x ) dx 2 2 (1 - x ) 2x

11 oo : 01 : s2 Sol 1. 7. We can write the equation as, 2x d2 y dy 3 3 1 - 2 - 2x - - -2 + - ( - + 1) - - y2= O dx (1 - x ) dx 2 2 (1 - x ) 2x P (x) = - 2x ( ) 2 1 -x 3 3 1 Q(x) = 2(2 + 1) (1 - x2).

So, and

Both P(x) and Q(x) are non-analytic at t he point x 2 That means at x

=

1.

= ±1

Sol 1.8. Given t hat,

dy x dx

+ y = x4

dy 1 - + -y= dx x



X

3

which is a linear first-order differential equation of t he form

dy dx + P y= Q who's solution is given by where Now, here in this problem , P

l =

Pdx

1

= -. X

I =

Pdx =

dx

eI = eln x = X

- = ln x X

[email protected]

41

Physicsguide

Diff. eq and Sp. functions

@Sk J ahiruddin , 2020

and

e

-J

1 =X

So, we get,

11 oo : 01 : s4 Diff. eq and Sp. ftmctions

@Sk J ahiruddin, 2020

e

and

l =-

- I

X

So, we get, •

yx

• •

dx

=

X

4

+C

x5

=-+ c 5

x

4

C

y=-+-



5

Now at x=l , y= l



x4



y

X

c = 1 - 1/5 = 4/5 4

= 5 + 5x

So l 1.9.

dx

k1 dt = -k2x + k3 dx dt

k3 - k2x k1 dx dt k3 - k2x k1 1 t - - ln(k3 - k2x) = k2 k1

[email protected]

@Sk J ahiruddin, 2020

42

+c

P hysicsguide

Diff. eq and Sp. ftmctions

11 oo :01 : s1

D iff. eq and Sp. functions

@Sk J ahiruddin , 2020

at t=O x=O

'

In

k3 - k2x k3 - k2 x - I2t - - - =e k 1 k3

So, as t -+ oo

k3 x = k , which means it approaching ,.2

t owards fixed value. So, option a satisfies t he funct ion perfect ly. Sol 1.10. You don 't need to do explicit calculations. Just use

Ot herwise use that the Legendre P olynomials are orthogonal. Then x Pn -+ Pn+l

j [email protected]

(c)Sk J a,hir11ddin. 2020

+ Pn-1 ·

So in t he first option

43

P hysicsguide

D iff. ea and So. ft1nctions

11 oo : 02 : oo

43

j ahir@physicsguide. in

@Sk J ahiruddin , 2020

➔ (Pn+2

P hysicsguide

Diff. eq and Sp. functions

+ Pn + Pn + Pn- 2) Pn+l

The integral will be zero according to t he ort hogonality. Again in option (b)

The integral is also zero. In opt ion (c)

This again goes to zero. In opt ion (d)

➔ (Pn+2

+ Pn + Pn + Pn- 2) Pn+2

In t his int egral t here will be a integral P n+2 P n+2 which will give non zero value.

[email protected]

44

P hysicsguide

11 oo : 02 : 03

44

[email protected]

Physicsguide

Diff. eq and Sp. functions

@Sk J ahiruddin , 2020

Sol 1.11. dy dx

y

(y

+ l)dy = -

y2

x2

X

+1 xdx

2 +y = - 2 +c

1 2 x2 1 - [y + 2y + 1] + - - - = 2 2 2 2 2 x + (y + 1) = (1 + c)

C

So, the equation represents circles having different radii depending on the value of the c. f [n + l ] = n f [n] . Now,

Sol 1.12. We know,

r

1 1 n+- = f n--+ 1 2

2

1

n- 2

1

n - -

2

r

1

n- 2 3 5 n - - n - - ··· 2 2

n - (2n -

1

1

1)2 r 2

(2n - 1)(2n - 3) (2n - 5) · · · l =---------~ 2n = 2n(2n - 1)(2n - 2) (2n - 2)(2n - 4) · · · 2.1 ~ 2n(2n - 2)(2n - 4) • • • 22n

11 oo : 02 : os -

-

2 2 (2n - 1) (2n - 3) (2n - 5) · · · 1 =---------~ 2n 2n(2n - 1)(2n - 2) (2n - 2)(2n - 4) · · · 2.1 = - - - - - - - - - - - - -~ 2n(2n - 2)(2n - 4) • • • 22n [email protected]

P hysicsguide

45

@Sk J ahiruddin, 2020

Diff. eq and Sp. ftmctions

(2n) !fo 2n_n!.2n

(2n) !fo 22nn!

Sol 1.13. Let us consider a trial solut ion, y = em>-. . P utting in the equation we get 2

m - 1= 0

m=±l

So, t he general solution will become, y

= A et + B e- t.

Now,

y(O) = 1 and

y(oo) = 0 ⇒

Sol 1.14. g(x) =

A+B = l



A=O



y = e- t = cosh t

xe-x

2

-xe-x

- sinh t

x >0 2

x O

=

-e-x

g'(x)

x=O-

m =

±(-2 + i)

Now t he solution can be written as e- 2x ( A cos x

+ B sin x)

or

e2 x ( A cos x - B sin x)

11 oo : 02 : s2 m

2

=

(3 - 4i)

~► m = ±(-2

+ i)

Now t he solution can be written as 2

e- x(A cos x

+ B sin x)

or

2

e x(A cos x - B sin x)

Now according to the boundary condition t he solut ion converges to zero as x solut ion. And from

-e



f

oo. So we will take only t he first (O) = 1 we get A= - 1 Hence f (1r) =

- 27r

Sol 1.48. This is a plot of Bessel funct ion J0 (x) . For other options, 62

j [email protected]

Physicsguide

@Sk J ahiruddin, 2020

Diff. eq and Sp. funct ions

(b) we know the plot of cos x . The amplitude doesn 't decrease t here. (c) This function doesn 't have a minima at x property of given plot.

= 3, which

is

(d) This function blows up at x = 0. Sol 1.49. Given the init ial condition is, u(x, 0)

= sin(1rx/ L )

We can check that , only option (c) and (d) satisfies t his boundary condition. Now, we will check t he second boundary condition, In between t his two options (c) doesn 't satisfy the different ial equation. and option (d) satisfy t he boundary condit ion. We can say t his from symmetry of the function. Sol 1.50. Given that,

dx

-=x dt

2



dx -x2 = dt



1

-- = t + C X

11 oo : 02 : 55 t ion . We can say t his from symmetry of t he function. Sol 1.50. Given that,

dx 2 -= x => dt

dx -=dt=> x2

1

- - = t +C X

Now given t hat

x(O) = 1 =>

1

c= - 1 =>

x=-1 -t

So, x will blow up if t tends to 1. Sol 1 .51. Just do the part ial derivatives. Sol 1 .52. (Same qs has been asked before .. Check t he previous problems) [email protected]

63

P hysicsguide

Diff. eq and Sp. functions

@Sk J ahiruddin , 2020

Sol 1 .53. The main work in t his problem is to do part ial derivatives of the solutions and mat ch with the given bound-

for option (d) and # 0 for all other options. Sol 1 .54. y

=

1 t anh x ex + e- x =y ex - e- x

ex e- X

y+ l y- 1 x = In

1 tanh x



y+l y- 1

=y

y+ l e2x = y- 1

11 oo :02 : s1 ⇒

ex e- x

y +l y - l

x

e

= In

Y + l =--

2x

y- l

y+l y- l

Sol 1.55.

= 2J1 -

dx

dt

dx

Jl -x2

x2

= 2t

sin - l x = 2t + c From boundary condition we get c So, x = sin 2t.

= 0.

For option (a) and (d) the differential equation is not satisfied. 64

[email protected]

P hysicsguide

Diff. eq and Sp. ftmctions

@Sk J ahiruddin, 2020

Again from the given equation, we get that :

= 0

x= l •

1.e.

x = sin 2t

= 1



7r

2t = -



2 So the correct option will be option (c) .

G(t ,x) =

Again,

4

1

1

n=O

So,

= -

L n. Hn(x)tn 00

Sol 1.56. Given that,

t

7r

11 oo : 03 : oo _ _ ~ -tn n 8x L-t n ! dx n=O

So,

a c ~ -tn 1 d Hn ___ 8x 2 L-t n ! dx 2 n=O 00 8G _ ~ 1 ndHn t t - L-t n n.1t dX 8 n=O 2

Again, And, Now,

a2 c ax9

-

00

2

ac ac 2x aX + 2t at

d oo d2 _ ~ 1 n Hn _ ~ l n Hn - L-t n.I t dx 2x L-t n.I t dx 2 oo

n=O

d ~ l n Hn + 2 L.-; n n.I t dx oo

n=O n= O 00 tn d2 H dH d n2xdri, + 2nHn(x) 2 I n. X X n= O

=L

=0

Sol 1.57. Again, don't be afraid just seeing the name of '' Gauss hypergeometric function'' . This problem is just a partial derivative calculation. j [email protected]

@Sk J ahiruddin, 2020

dF dz

Physicsguide

65

Diff. eq and Sp. functions

=f

a(a + 1) ... (a+ n - l )b(b + l ) ... (b n=O c(c+ l ) ... (c+n-l)n!

+ n - l ) nzn-l

Now write n ! as n(n - 1)! in t he denominator

=

f

a(a + 1) ... (a+ n - l)b(b + 1) .. . (b + n - l ) zn -l n=O c( c + 1) . .. (c + n - l ) (n - l ) ! ab , , (a + l ) . . . (a + n - l ) (b + l ) . . . (b + n - l )

zn- l

11 oo : 03 : 02 =

f

a(a + l ) ... (a +n -l )b(b + l ) .. . (b+n-1 ) zn-l n= O c( c + 1) ... (c + n - l ) (n - l )!

= ab "" (a+ 1) ... (a + n - l )(b + 1) . .. (b + n - 1) c ~

(c+ l ) . .. (c+n- 1)

zn- l

(n- 1)!

Now write the derivative in this way

= ab c

f (a + 1) . . . [a + (n - 1) - 1](b + 1) . .. [b + (n - 1) - 1] (c

n=O

+ 1) . . . [c + (n -

1) - 1]

z n-I

(n - 1) !

dF ab Hence we get d = -F (a + 1, b + 1, c + 1, z ) Z

C

Sol 1.58. Take Fourier transformation bot h side of the equat ion as done in a example problem in Fourier transformation chapter ?? Sol 1.59. Write the equation in terms of

2 + En+ 1

3 = 1 + l + (2 + En)

[email protected]

@Sk J ahiruddin, 2020

Hence

=>~>

66

En

and En+l

3 -En En+l = - - - 1 = - 3 + En 3 + En P hysicsguide

Diff. eq and Sp. functions

- 1 - 1 +E-+ as En-+ 0 3 3 En Sol 1.60. You can solve the partial differential equation directly. But t hat process is little lengthy. I recommend to directly check t he options by substit ut ing them in t he orig-

En+l

inal equation. Just a few partial derivative calculations are enough. Verify the ans.

11 oo : 03 : os rectly. But t hat process is little lengthy. I recommend to directly check t he opt ions by substit u t ing them in the original equation. Just a few partial derivative calculations are enough. Verify the ans .

Sol 1.61. Here, the equation suggests t hat, G(x) is Green 's Function. And we know t hat, at x

= 0 G(x)

is cont inuous,

but G' (x) will have a discontinuity because of t he presence of o(x).

Sol 1.62.

tn

L Hn(X) n . = oo

I

e-t2+2tx

n=O

~ Hn(O) tn = e - t2 ~

n!

oo

tn

n=O

-t2

( - t2)2

H4(0)

1 4! 2! 4! H4(0) = - = 12 2! 67

[email protected]

P hysicsguide

@Sk J ahiruddin , 2020

Diff. eq and Sp. functions

!~

Sol 1.63. Given that, + ay = e-bt . If we take Laplace Transformation of both side, we get

L [y'] + aL[y] = L[e-bt] sY - y 0 + aY

1

= --

s+b

1

[Where Y = L[y]]

11 oo : 03 : 01 If we take Laplace Transformation of both side, we get

L [y'] + aL[y] = L[e-bt] 1

sY - y 0 + aY = - -

[Where Y = L[y]]

s+ b



Y ( s + a) =



y =

1

s+

b

[as Yo = O]

1

(s + a)(s + b)

Sol 1.64. Let's calculate G(l, x) first

hence we see that 1

---;:==~ -

00

1

xnP 1

-- -

Now 00

xG(l , x)

x 1 -x

=

=

X

'"""" xn+l Pn( l ) L.t n=O

We can easily see that J(x) j [email protected]

68

@Sk J ahiruddin , 2020

00

=

d d

=

o

J(x')dx' 1

X

x l -x

(1 -

X) 2

Physicsguide

Diff. eq and Sp. ftmctions

11 oo : 03 : 1o

00

=

L

Xn+m pn(l )Pm(l )

n.m=O , Sol 1.65. Let us consider first, m = n = 1 T hen P 1 (x) = x, 1

1

2

[P1( x)] dx

=

0

1

2

x dx = -

3

0

So, it does not vanishes when m = n or m - n = 0 Now we consider m = 1, n = 2 1

x[(3x

2

) -

1] =

0

4

1

5

3

2

6

- - - = -

So, it doesn 't vanishes when m - n = ± 1 or an odd int eger. So, t he only option left is (c). Let us check. Now, we consider m = 0, n = 2. 1

1

1

Po(x) P1 (x)dx = -

[(3x

2

0

) -

l ]dx = 0

0

69

[email protected]

2

Physicsguide

Diff. eq and Sp. functions

@Sk J ahiruddin , 2020

Sol 1.66.

dy

r1

,

1,

11 oo : 03 : 12 Diff. eq and Sp. functions

@Sk J ahiruddin, 2020

Sol 1.66.

:~ = y[ l + (log

2 y) ]

dy y[l + (log y ) 2 ] Now, we t ake logy ⇒ ⇒





p

=

1 - dy y

=

dp

dp --=x+c 1 + p2 1 tan- = x + c

= tan(x + c) log y = t an (x + c)

p

From the boundary condit ion we get c = 0

logy So, at

x

7r

=

2

= tan x

y = oo

d2 f Sol 1.67. Here we can see dx 2 = - 2. So, the curvature of

function doesn 't change . Now, taking 1= 2 Jx we get the

j [email protected]

@Sk J ahiruddin , 2020

70

P hysicsguide

Diff. eq and Sp. functions

11 oo : 03 : 1 s [email protected]

70

P hysicsguide

@Sk J ahiruddin , 2020

Diff. eq and Sp. functions

solut ion , ye 2x

=3 =



3 - e 2x 2

e2xdx

+C

+c

3 y = - + ce-2x 2

Putting the boundary condit ion we get, c = ~. 3 So, y = (1 - e 2x). 2 3 So, at x ➔ oo y = . 2 Sol 1.68. For a general polynomial,

f (X) =

axn

b Sum of t he roots =-a

+ bx n - l + · · · + z z

and product of the roots =

a

Even degree polynomial

z Odd degree Polynomial a Here, the polynomial is a even degree polynomial, where

a = 3, b = 0, z = - 8.

8 So, sum= O and product=-= --. a 3 z

And , as t he product is real and sum is zero, t h at means complex roots are conj ugate of each other. [email protected]

71

Physicsguide

11 oo : 03 : 11 complex roots are conjugate of each other. j [email protected]

71

Physicsguide

Diff. eq and Sp. ftmctions

@Sk J ahiruddin, 2020

Sol 1.69.

dx dt put dx dt Then

2

_dx=O dt

=P d2x dt 2

=

dp dt

=

dp P dx

So t he equation becomes

dp 1 2 p-+-p -p = 0 dx x or

dp dx

+P-

x The integrating factor is exp

1=0

(J ~ ) =

x

The equation now

dp xdx +p-x=O d dx (xp) x2

Xp =

2

=x

+ C1

dx p= - = (x/2+c1/x) dt dx = d t X + C1 2

[email protected]

2

=

x + 2c1 -2x

X

72

P hysicsguide

11 oo : 03 : 20

[email protected]

Physicsguide

72

Diff. eq and Sp. functions

@Sk J ahiruddin, 2020

Now t he integration is easy

dt

= 2x dx x

put x 2 + 2c1 = z

~>

2xdx

=

2

+ 2c1

dz

So t he integration

dz = dt z or

x

2

+ 2c1 = X2

=

z = c2et t

C2e -

2C1

Now we will apply t he boundary conditions x(t

0, x(t

= 1) = 1

And

Hence at t - 2 2 - 1 e 2 x = - - ~~> x = ✓e + 1 e- 1

0)

11 oo : 03 : 23 e1 -

1

Hence at t - 2

j [email protected]

73

@Sk J ahiruddin , 2020

Physicsguide

Diff. eq and Sp. functions

Sol 1. 70. When a function is extremum (maximum or minimum) we know dy

dx

= 0 Hence

P~(x)e-x

2

2

+ Pn(x)(- 2x)e-x = 0

or

So we get

P~(x) - 2x Pn(x) = 0 Pn(x) is an algebric polynomial of order n. The x multiplied with it make is order n

+l

From the theory of differential equation an equation of order n can have maximum n distinct root. As our equation is of order n + l , so the n + l roots will be able to create maximum (n + l ) extrem as.

Sol 1. 71. Given that ,

Now,

11 oo : 03 : 26

Now, P2 ( x)

=

~ (3x

2 3x = 2P2(x)

jahir@physicsguide. in

@Sk J ahiruddin,

2

+1=

1)

-

2P2(x) + Po(x)

74

Physicsguide

2020

Diff. eq and Sp. functions

Now,

J(x) = l + 5x + 3x

2

+ 5P 1(x) + 2P2 (x) + Po(x) =2Po(x) + 5P1 (x) + 2P2(x)

= Po(x)

Now, cornparing with

we get

co = 2 Sol 1. 72. Let us first consider here , z

=

In x

So,

dy dy dz 1 dy dx dz dx x dz dy dy . . . [i] X-=dx dz d IT. dy l = d dy = d

I l

Idy l dz

11 oo : 03 : 2a



dy dydz 1 dy dx dz dx xdz dy dy .. . [i] X = dx dz dy - d dy d d dy dz dz dz dx dx x dx dx dz d2 y dy 1 d2y x dx 2 + dx xdz 2 d2y 2d2y dy · ·· [ii] x dx 2 + x dx dz 2

[email protected]

Physicsguide

75

Diff. eq and Sp. ftmctions

@Sk J ahiruddin , 2020

Now,

dy x dx 2 + 5x dx 2d2y 2

y dy dy x dx2 + x dx + 4x dx + 3y = 0 d2 y dy [iii] dz 2 + 4 dz + 3y = 0 2d



+ 3y = 0

Now, let u s con sider the trial solution is, y

[iii] we get that, 2

m +4m + 3 = 0

(m + l )(m + 3) = 0 m

=

-1 , m

= -3

So, t he general solution is ,

+ B e- 3z = Ae-ln x + B e- 3ln x

y = A e-z

= em z

So, from

11 oo : 03 : 31 So, the general solution is, y =Ae-z + B e- 3z

= A e-ln x + Be-3ln x

A B =-+-3 x x Sol 1. 73. Don 't do the explicit calculation. It will be huge. Think of t he properties of Legendre Polynomials 2

Sol 1.74. Write the equation as x 2 d solution is obvious y 1 j [email protected]

@Sk J ahiruddin,

;

dx

= n(n + l)y. One

= xn+l within a constant factor. As 76

Physicsguide

2020

Diff. eq and Sp. functions

we see from t he general form of 2nd order equation y'' + P (x )y' + Q(x )y = 0 t he P (x) = 0 in our case. So the second solution is X [Y1 (x2)]

2

(Refer to Arfken , 7th ed Section 7.6) So

dx x2n+2 1 2nl X ---2n- l x-n

2n+ 1 So the product of the two solutions is y 1y2

=

x

2n + 1 So t he ans would be x within a constant factor.

11 oo : 03 : 33 x-n 2n + 1 So t he product of t he two solutions is y 1y 2

=

x

2n + 1 So t he ans would be x within a constant factor.

Sol 1. 75. Write the equation as

dy exp(x) --y=--dx x P(x)

=

- 1. So the integrating factor is exp{f P(x )dx}

77

[email protected]

=

Physicsguide

@Sk J ahiruddin, 2020

Diff. eq and Sp. functions

e- x. Now t he solution is 1

e- xf racdydx - e- xy = -

X

d -ye-x dx

ye

-x

1 X

=

dx - = ln x+c X

Now use to the boundary condition y = e 2 at x = l to get c = e . Now t he solution is

ye- x = ln x Sol 1. 76.

+e

:=::-~>

y

= ex ln x + exp(x + 1)

11 oo : 03 : 36 ye- x

= ln x + e

Sol 1. 76.

For s = 2

Fors = 3

2e3x X

e

00

+1 =

xn

L E 1i ( 3) n . n=O I

Adding the two equations

78

jahir@physicsguide. in

@Sk J ahiruddin , 2020

Physicsguide

Diff. eq and Sp. functions

Hence

Now expand e2x 2 l

+

2x 1!

+

(2x ) 2!

2

+

(2 x ) 3

3!

+ .. .

Now the first term in the series is for n = 0, t he second t erm is for n = 1, the t hird term for n = 2 and so on. 25x5 • Equating So the sixth t erm for n = 5 in LHS is 2 5.1 5 with x term in RHS we get

11 oo : 03 : 38 n=O 78

j [email protected]

P hysicsguide

Diff. eq and Sp. ftmctions

@Sk J ahiruddin, 2020

Hence

Now expand e2x

2x 2 l + 1!

+

(2x ) 2!

2

+

(2x ) 3!

3

+ .. .

Now the first t erm in the series is for n = 0, the second t erm is for n = l , the t hird t erm for n = 2 and so on. 25 5 So t he sixth t erm for n = 5 in LHS is 2 ~ . Equating 5. wit h x 5 term in RHS we get

Hence

j [email protected]

79

P hysicsguide

11

00:03:41

n=O 78

j [email protected]

P hysicsguide

Diff. eq and Sp. ftmctions

@Sk J ahiruddin, 2020

Hence

Now expand e2x

2x 2 l + 1!

+

(2x ) 2!

2

+

(2x ) 3!

3

+ .. .

Now the first t erm in the series is for n = 0, the second t erm is for n = l , the t hird t erm for n = 2 and so on. 25 5 So t he sixth t erm for n = 5 in LHS is 2 ~ . Equating 5. wit h x 5 term in RHS we get

Hence

j [email protected]

79

P hysicsguide

11 oo : 03 : 44 n=O 78

j [email protected]

P hysicsguide

Diff. eq and Sp. ftmctions

@Sk J ahiruddin, 2020

Hence

Now expand e2x

2x 2 l + 1!

+

(2x ) 2!

2

+

(2x ) 3!

3

+ .. .

Now the first t erm in the series is for n = 0, the second t erm is for n = l , the t hird t erm for n = 2 and so on. 25 5 So t he sixth t erm for n = 5 in LHS is 2 ~ . Equating 5. wit h x 5 term in RHS we get

Hence

j [email protected]

79

P hysicsguide

11 oo : 03 : 4s n=O 78

j [email protected]

P hysicsguide

Diff. eq and Sp. ftmctions

@Sk J ahiruddin, 2020

Hence

Now expand e2x

2x 2 l + 1!

+

(2x ) 2!

2

+

(2x ) 3!

3

+ .. .

Now the first t erm in the series is for n = 0, the second t erm is for n = l , the t hird t erm for n = 2 and so on. 25 5 So t he sixth t erm for n = 5 in LHS is 2 ~ . Equating 5. wit h x 5 term in RHS we get

Hence

j [email protected]

79

P hysicsguide

11 oo : oo : 01

Problellls and Solutions in Integral Transforlllations Sk J ahiruddin Arnab Choudhury Assistant Professor Sister Nibedita Govt. College, Kolkata Author was the topper of IIT Bombay 1\11.Sc Pl1ysics 2009-2011 batch He ranked 007 in IIT J Al\11 2009 and 008 (JRF) in CSIR NET J une 2011 He has been teaching CSIR NET aspirants since 2012

1

@Sk J ahiruddin, 2020

Integral transformations

11 oo : oo : 03 1

@Sk J ahiruddin, 2020

Integral transformations

Contents 1 Problems from NET , GATE, JEST, TIFR & JAM papers

3

Ans keys .







































23

1.2 Solutions .







































24

1.1

j [email protected]

2

Physicsguide

11 oo : oo : 06

j ahir@physicsguide. in

2

P hysicsguide

@Sk J ahiruddin , 2020

1

Integral transformations

Problems from NET, GATE, JEST, TIFR & JAM papers

Prob 1.1:

f (x) is a

periodic function of x with a p eriod of

21r. In t he int erval - 1r


0. The ratio (:), where X

f (x) = x

x is the

exp(-f) where

most probable value

and (x) is t he mean value of the variable x, is 1 (a) 2 (b) + A (c) ~ (d) 1 [TIFR 2014]

l-A

A

Prob 1.21. If x is a continuous variable which is uniformly

distributed over t he real line from x = 0 to x -+ oo according to t he distribution f (x) = exp( - 4x), t hen the expectation vaalue of cos 4x is [TIFR 2016] (a) 0 (b) 1/ 2 (c) 1/ 4 (d) 1/ 16 Prob 1.22. A bag contains many balls, each with a number j [email protected]

@Sk J ahiruddin, 2020

8

Physicsguide

Probability

painted on it . There are exactly n balls which have the number n (namely one ball wit h 1, 2 balls with 2, and so on until N on them). An experiment consists of choosing a ball at random, noting t he number on it and returning it to t he bag. If the experiment is repeated a large number of [NET t imes, t he average value t he number will tend to June 12]

(a) 2N + 1

(d) N(N + 1)

2 3 Prob 1.23. Consider three particles A , B and C, each with

an attribute S t hat can take two values: ± 1. Let SA = l , SB = 1 and Sc = - 1 at a given instant. In the next instant, each S value can change to -S with probability 1/ 3. The

11 oo : oo : 24 Prob 1.23. Consider three particles A , B and C , each with an attribute S that can take two values: ± 1. Let SA = l ,

SB = land Sc= - 1 at a given instant. In the next instant, each S value can change to -S with probability 1/ 3. The [NET probability t hat SA+ SB+ Sc reains constant is; June 13]

(a) 2/ 3

(b) 1/ 3

(c) 2/ 9

(d) 4/ 9

Prob 1.24. Let u be a random variable uniformly distributed

in the interval [O , 1] and V = -c ln(u), where c is a real const ant . If V is t o be exponent ially distributed in the interval (0, oo) with unit standard deviation, then t he value of c should b e: (a) ln (2) (b) 1/ 2 (c) 1 (d) - 1 [NET June 13]

[email protected]

Physicsguide

9

@Sk J ahiruddin , 2020

P robability

Prob 1.25. Let y = i(x1 + x2 - µ) where x1 and x2 are indep endent and identically distributed Gaussian random

variables of mean µ and st andard deviation O". Then (y4 ) / 0" 4 is [NET June 14] (a) 1

(b) 3/ 4

(c) 1/2

(d ) 1/ 4

Prob 1.26. Three real variables a , b and c are each randomly chosen from a uniform probability distribution in the

interval [O, 1]. The probability that a + b > 2c is June 15] (a) 3/ 4

(b) 2/ 3

(c) 1/ 2

(d) 1/ 4

[NET

11 oo : oo : 21 domly chosen from a uniform probability distribution in the interval [O, 1]. The probability that a+ b > 2c is [NET June 15]

(a) 3/ 4

(b) 2/ 3

(c) 1/ 2

(d) 1/ 4

Prob 1.27. Let X and Y be two independent random variables, each of which follow a normal distribution with t he

same standard deviation a, but with means spect ively. Then t he sum X + Y follows a 16]



and - µ re[NET June

(a) dist ribution with two peaks at ±2 and mean O and standard deviation aJ2 (b) normal distribution with mean O and standard deviation 2a (c) dist ribut ion with two peaks at ±2 and mean O and standard deviation 2a (d) normal distribution with mean Oand standard deviation

aJ2 j [email protected]

@Sk J ahiruddin , 2020

10

Physicsguide

P robability

Prob 1.28. T wo independent variables m and n which can take t he integer values 0, 1, 2 ... oo , follow t he Poisson dis-

t ribution, with dist inct mean values µ and v respectively. Then [NET Dec 14] (a) The probability distribution of the random variable l = m + n is a binomial distribut ion (b) The probability distribution of t he random variable r = m - n is also a Poisson distribut ion (c) The variance of t he random variable l = m + n is equal

11 oo : oo : 29 (a) The probability distribut ion of t he random vai·iable l = m + n is a binomial distribution (b) The probability distribut ion of t he random variable r m - n is also a Poisson distribut ion (c) The variance of t he random variable l

=

= m + n is equal

toµ + V (d) The mean value of t he random variable r equal to zero

= m - n is

Prob 1.29. There are on average 20 buses per hour at a point, but at random t imes. The probability that t here are

no buses in five minut es is closest to

[JEST 2013]

(a) 0.07

(b) 0.60

(c) 0.36

(d) 0.19

Prob 1.30. You receive on average 5 emails p er day during a 365-days year . The number of days on average on which

you do not receive any em ails in t hat year are: 2016] (a) more th an 5 of the above [email protected]

(b) More t han 2

(c) 1

11

[JEST (d) None

Physicsguide

@Sk J ahiruddin , 2020

P robability

Prob 1.31. Consider two 1·adioactive atoms, each of which has a decay rate of 1 per year. The probability that at least

one of t hem decays in t he first two years is

(a)

!4

Dec 16]

(b)

3 4

(c) 1 - e- 4

(d) (1 - e- 2 ) 2 [NET

11 oo : oo : 31 one of t hem decays in the first two years is

1 3 (a) (b) 4 4 D ec 16]

(c) 1 - e- 4

(d) (1 - e- 2 ) 2 [NET

Prob 1.32. The random variable x(-oo < x < oo) is dist ributed according to the normal distribution

1 2 2 P (x) =-----;::::::= exp (-x /2a ) 21ra 2 The probability density of the random variable y = x 2

[NET June 1 7]

is (a) ~ exp (-y/2a

2

y

2 11"0"

(b) ~ exp (-y/2a 2 2

27ro- y

), 2

0


- 1, a > 0)

n+l 2

Hence

Hence 1 -00

00

1

4

-(x -µ / 2)2

x e

21r (O" I ~)2

2

dx

-oo

using t he result 4

(]"4

Solution 1.26. Let a+ b = x and 2c = y So x>y

e is ess than

a+ b

[email protected]

@Sk J ahiruddin, 2020

2

c
2c is 1/ 2 Solution 1.27. If X and Y are two independent random variables, each of which follow a normal distribution, such

t hat, X has standard deviation a x means µ x Y has standard deviation ay means µy

Z = X +Y

J

Then Z has standard deviation a z = al + a~ means µz = µx + µ y So, here X + Y is a normal distribution with ax +Y = v'2a and mean µ - µ = 0. Solution 1.28. Sum of two independent variable obeying Poission Distribution is also a poission distribution with

mean= µ+ v. 2 Variance of a poission distribution = a =mean. So, variance of sum of those random variables is = µ + v. Solution 1.29. Probability in Poission Distribution is given by,

j [email protected]

@Sk J ahiruddin, 2020

29

Physicsguide

P robability

11 oo : 01

: 21

[email protected]

29

Physicsguide

@Sk J ahiruddin , 2020

P robability

Where µ is the mean rate in the span asked, and n is the amount asked. Here, 20 buses per hour means ~ buses per five minutes. So, µ = 5/3, And n = 0. So, t he probability is P = e-~ = 0.188

Solution 1.30. Here µ = 5, n = 0. So, probability is P = e- 5 .

So, number of days are = 365 x e- 5 more t han 2.

2.49, that means

Solution 1.31. One atom per year means two atoms p er two year. So, µ = 2.

If an atom does not decays in two days, t hen the probability 2 of that occurrence will be given by, P1 = e- . 4 2 SO, for two atoms it will be P2 = P1 = eSo, Probability that at least one atom decays will be given by, 4 P = l - P2 = 1 - eSolution 1.32. As P(x) is even function then we can write 00

P (x) dx = -oo

00

1

21ra 2 1

= 2 --;::::=====:2 21ra

[email protected]

2

30

2

exp ( -x / 2a )dx

----;:=~

- oo 00

2

o

2

exp (-x / 2a )dx

Physicsguide

11 oo : 01

: 24

= 2 ----;::::=====:2 21ra

[email protected]

exp (-x 2 / 2a 2 )dx

o

30

P hysicsguide

@Sk J ahiruddin, 2020

P robability

Now if we put x 2 = y ⇒ 2xdx = dy , then we get, 00

1

00

2

2

exp ( -x / 2a )dx = 0

P(y)

=

2x

1

J exp (-y/2u 2 21ra y

2

2

1

) =

J 21ra y exp (-y/2u 2

2

)

Solution 1.33. Suppose we toss one fair coins hundred t imes, t hen probability of n number of head occurs at t he end of

100 toss is (using binomial distribut ion formula) n

lOOC n

1

2

l

100- n

2

So when you toss both coins t hen the probability of occurring same n number of head at t he end of 100 toss is just mult iply t he probabilit ies .

lOOCn

l -

2

n

~

100- n

l x lOO C n

2

n

-

2

l

100- n

-

2

Now just sum over n from Oto 100 to get t he probability to get same numbers. The ans is (d)

! n=l

100 100

L cooen) n =l

2

11 oo : 01

: 26

ow JUS sum over n rom to get same numbers. The ans is (d) 100

L

1 (l00Cn )2

n =l

[email protected]

@Sk J ahiruddin, 2020

200

!

-

2

100 100

L

eoocn)2

n =l

31

Physicsguide

P robability

Solution 1.34. The standard deviation of binomial distribution = y0Vpq_ step size to right or left is 2a so mean square displacement = 2ay0Vfig_ Solution 1.35.

11 oo : 01

: 29

n =l j [email protected]

n =l

31

@Sk J ahiruddin , 2020

Physicsguide

P robability

Solution 1.34. The standard deviation of binomial distribution = ,JNpg_ step size to right or left is 2a so mean square displacement = 2a,JF[pq_ Solution 1.35.

j [email protected]

32

Physicsguide

11 oo : 01

: 31

n =l j [email protected]

n =l

31

@Sk J ahiruddin , 2020

Physicsguide

P robability

Solution 1.34. The standard deviation of binomial distribution = ,JNpg_ step size to right or left is 2a so mean square displacement = 2a,JF[pq_ Solution 1.35.

j [email protected]

32

Physicsguide

11 oo : oo : oo

Problellls and Solutions in N ulllerical Methods, Group Theory, Tensors and Greens Functions Sk J ahiruddin Assistant Professor Sister Nibedita Govt. College, Kolkata Aut hor was the topper of IIT Bombay lVI.Sc Physics 2009-201 1 batch He ranked 007 in IIT JAlVI 2009 and 008 (JRF) in CSIR NET J une 2011 He has been teaching CSIR NET aspirants since 2012

1

@Sk J ahiruddin , 2020

Advanced topics in Mathematical Physics

11 oo : oo : 03 1

Advanced topics in Mathematical P hysics

@Sk J ahiruddin, 2020

Contents 1

Numerical Methods

Ans keys .







































9

1.2 Solut ions .







































9

1.1

2

3

4

4

Group Theory

16

2. 1 Ans keys and solutions

























20

2.2 Solutions . . . . . . . .

























20 26

Tensors

3. 1

Ans keys .







































30

3.2

Solutions .







































30

Green's functions

4.1

Ans keys . . ..

j [email protected]

38 •





2



























41

Physicsguide

00: 00: 06 •







































j [email protected]

2

@Sk J ahiruddin , 2020

Advanced topics in Mathematical P hysics

Physicsguide

4. 2 Solutions . . . . . . . . . . . . . . . . . . . .

41

11 oo :oo :oa

[email protected]

3

@Sk J ahiruddin , 2020

Advanced t opics in 1\ll athem atical P hysics

Physicsguide

Problems from NET, GATE, JEST, TIFR & JAM papers

1

Numerical Methods 1

#dx is to be evaluated up to

Prob 1.1. The int egral 0

3 decimal places using Simpson's 3-point rule. If the interval [O , 1] is divided into 4 equal parts, t he correct result is [NET June 14] (a) 0.683 (b ) 0.667 (c) 0.657 (d) 0.638 Prob 1.2. Given t he following xy data. X

1.02

2.0

3.0

4.0

5.0

f(x) 0.002 0.601 0.948 1. 21 1.42 Which of t he following would be t he best curve, wit h constant positive parameters a and b, to fit this data? [TIFR 2018]

(a) y = ax - b (c) y = a log 10 bx

(b) y = a + ebx

(d)y = a -

e(- bx)

Prob 1.3. Consider the differential equation ~~

= x2

-

y

11 oo : oo : 11 2018]

(a) y

=

ax - b

(b) y

(c) y = a log 10 bx

=

a + ebx

(d)y = a -

e(- bx)

Prob 1. 3. Consider the differential equation ~~ with the initial condition y = 2 at x = 0. Let

= x2

Y(i )

and

-

y

Y(~)

[email protected]

4

@Sk J ahiruddin, 2020

Advanced topics in Mathematical Physics

Physicsguide

be the solutions at x = 1 obtained using Euler 's forward algorithm with step size 1 and ½ resp ectively. The value of Y(1) -Y(½)

Y(½)

(a)





[NET June 15]

IS

(b) - 1

(c) ½

(d) 1 8

Prob 1.4. The value of t he integral

=

- - evaluated o x2 + 5 2 is

(c) 0.698

(d) 0.736 [NET

using Simpson's 1/ 3rd rule with h (a) 0.565

(b) 0.620

dx

Dec 15] Prob 1. 5. In finding the roots of the polynomial f (x) = 3x 3 - 4x - 5 using t he iterative Newton-Raphson m ethod, t he init ial guess is taken t o be x = 2. In the next iteration, [NET June 16] its value is nearest to: (a) l.671

(b) 1.656

(c) 1.959

(d) 1.551

Prob 1.6. Given the values sin45° 0.7071 , sin 50° 0. 7660, sin 55° = 0.8192 and sin 60° = 0.8660, the approximate value of sin 52°, computed by Newton's forward difference method, is

[NET Dec 16]

11 oo : oo : 14 Prob 1.6. Given the values sin45° = 0.7071 , sin50° = 0. 7660, sin 55° = 0.8192 and sin 60° = 0.8660, the approximate value of sin 52°, computed by Newton's forward dif-

[NET Dec 16]

ference method, is (a)0.804

(b )0. 776

(c)O. 788

(d )O. 798

Prob 1.7. The interval [0,1] is divided into 2n parts of equal j [email protected]

5

@Sk J ahiruddin , 2020

Advanced topics in Mathematical Physics

Physicsguide

1

2

ei 1rxdx using Simpson 's

length to calculate t he integral 0

1/ 3 rule. What is t he minimum value of n for t he result to

[NET June 1 7]

be exact ?

(a) oo

(d) 4

(c) 3

(b) 2

2.2

Prob 1.8. The value of

xexdx by using t he one-segment 0.2

t rap ezoidal rule is close to (a) 11.672 (b) 11.807

[JEST 2014] (c) 20 .099

(d ) 24.119

Prob 1.9. The approximation cos 0 l is valid up to 3 decimal places as long as I0 is less t han: (take 180° / 1r 57.29°) [NET June 13] f"'-.J

f"'-.J

(a) 1.28°

(b) 1.81°

(c) 3.28°

(d ) 4.01°

Prob 1.10. The differential equation dy( x ) = a X 2 with dx t he init ial condit ion y(O) = 0, is solved using Euler 's method. If YE(x) is the exact solution and y 1v (x) the numerical solut ion obtained using n steps of equal length, then the relative error

YN(x) - YE(x ) . l) , ....

r

ry, \

.

1s proportional to

[NET Dec 17]

11 oo : oo : 16 dx

t he initial condit ion y(O) = 0, is solved using Euler's method. If YE(x) is the exact solut ion and YN(x) the numerical solut ion obtained using n steps of equal length, then the relative YN(x - YE x .

(a) 1/n

2

YE

.

[NET Dec 17]

X

(b) 1/n

3

(c) 1/n

4

(d) 1/n

Prob 1.11. The interval [O, 1] is divided into n parts of 1

equal length to calculate the integral

exp (i21rx )dx uso

ing the trapezoidal rule. The minimum value of n for which [email protected]

6

@Sk J ahiruddin, 2020

Advanced t opics in :Nlathematical P hysics

t he result is exact is (a) 2 (b) 3 (c) 4

Physicsguide

[NET Dec 17] (d) oo

Prob 1.12. The fractional error in estimating the integral 1 f0 xdx using Simpson's ½rule, using a step size 0.1 , is nearest to [NET June 18]

(a) 10-4

(b) 0

(c) 10-2

3

X

10-4

Prob 1.13. The value of the integral

0

evaluated using t he trapezoidal rule with step size of 0.2, is [NET Dec 18] (a) 0.30 (b) 0.39 (c) 0.34 (d) 0.27

Prob 1.14. If the Newton-Raphson method is used to find t he positive root of the equation x = 2 sin x t he iteration

11 oo :oo :1a (a) 0.30

(b) 0.39

(c) 0.34

[NET D ec 18] (d) 0. 27

Prob 1.14. If the Newton-Raphson method is used to find t he positive root of the equation x = 2 sin x t he iteration equation is [NET June 2019] 2 x n - 2 (sin X n + Xri cos Xn) ( a) Xn+ l = - - - - - - - - - 1 - 2cosxn _ 2 (sin Xn - Xn COS Xn) (b) Xn+ l 1 - 2cosxn x~ - 1 + 2 (cosxn - Xn sin xn) ( C) Xn+ l = . Xn - 2 Sln Xn [email protected]

7

@Sk J a hiruddin , 2020

Advanced topics in l\llathem a tical Physics

P hysicsguide

_ x~ - 1 - 2 (cos Xn + sin Xn) (d) Xn+ l . Xn - 2 s1n xn Prob 1.15. The positive zero of the polynomial f (x) = x 2 - 4 is determined using N ewton-Raphson method, using an initial guess x = l . Let the estimate, after two iterations, x( 2) - 2 x 100% is [NET be x( 2) . The p ercentage error 2 Dec 2019] (a) 7.5% (b) 5.0% (c) 1.0% (d) 2.5%

11 oo : oo : 21

j [email protected]

8

@Sk J ahiruddin , 2020

Advanced topics in :Nl athematical Physics

1.1

Physicsguide

Ans keys

1.1. d

1.6.

C

1.11. a

1.2.

1.7. b

1.12. b

1.3. b

1.8.

1.13.

1.4. a

1.9. b

1.14. b

1.5. b

1.10. d

1.15. d

1.2

C

C

C

Solutions

Solution 1.1. The interval is divided into 4 equal parts.

11 oo : oo : 23 1.2

Solutions

Solution 1.1. The interval is divided into 4 equal parts. So, h = 0.25 X

f( x)

Xo

=0

X1

Yl

Yo= 0

= 0.25

S2

Now

=

= 0.5

v'xdx =

0.707 y3

Y2 h

=

= 0.75 =

X4 =

0.866 Y4

l

=1

0.707

[s1 + 4s2

3 0.25 [ = 1+4 3

0

X3

= Yo + Y4 = 1 + 0 = 0 = Y1 + Y3 = 0.5 + 0.866 = 1.366

S3 =

i

Y2

0.5

=

s1

X2

+ 2s3] X

1.366 + 2

X

] 0.707 = 0.657

[email protected]

9

@Sk J ahiruddin, 2020

Advanced topics in Mathematical Physics

Physicsguide

Solution 1.2. As first look f (x) is increasing with x, but t he rate of change is slowing down. So option (a) and (b) eliminated. Then ind the slopes of t he function. Also find t he double derivative. Then with t he values find the values of a and b. Then check what option matches. Remember in T IFR you are allowed to use calculator. Solution 1.3. After first step , . 2

Y(i )

=y +h x (x -y) =

A

1

2 + 1(0 - 2) n

,

1I A

r. \

=

0

11 oo : oo : 26 Solution 1.3. After first step, Y(l )

2

=y+ h x (x -y)

2 + 1(0 - 2)

=

1

Y(l /2) =

2 + (0 - 2) 2

Y(l ) - Y( ½) =

0- 1= - 1

And

=

0

=

1

So, 1

Y(~)

Solution 1.4. Const ruct t he table X

Xo = 0

f(x)

Yo = 0.2

2 Yi = 0. 111 X1

=

X2

= 4

Y2 = 0.048

j [email protected]

S2

= Y1

S3 =

0

X4 = 8 6 0.024 Y4 = 0.014

=

Physicsguide

Advanced t opics in Mathem atical P hysics

s1 = Yo + Y4 =

Now

y3 =

10

@Sk J ahiruddin , 2020

i

X3

.jxdx =

Y2 h

+ Y3 =

=

0.2 + 0.014 = 0.214 0.111

+ 0.024 = 0.135

0.048

[s1 + 4s2

+ 2s3]

3 2 = [0.214 + 4 3

X

0.135 + 2 X 0.048] = 0.565

11 oo : oo : 29 S3 = Y2 =

i

Now

.jxdx =

h

0.048

[s1 + 4s2

+ 2s3]

3 2 = [0.214 + 4 3

0

X

0.135

+2X

0.048] = 0.565

Solution 1.5. The algorithm of Newton Raphson method •

1S

Now here xo

=2

So, f (xo) = 3x - 4x - 5l xo = 11 2 And, f'( xo) = 9x - 4 xo = 32 So, x 1 = 2 - 1~ = 1.656 3

Solution 1. 6. construct t he table XQ

45° 50° 55° 60°

f (xo) =

l:iy1

Yo

0.7071 0.7660 0.8192 0.8660

!:iy2

!:iy2

0.0589 0.0532 -0.0057 0.0468 -0.0064 -0.0007

[email protected]

11

@Sk J ahiruddin , 2020

Physicsguide

Advanced topics in 1\ll athematical Physics

Now, we apply Newton's forward difference method ph

2 p=5

=2

p(p + 1) , l:iy2 sin 52 = Yo + pl:iy1 + 2 •

0

2

= 0. 7660 + ::-

X

0.0532 +

~(l + l) 5 5 ,,.. ,

( -0.0064)

11 oo : oo : 31 5 •

Sln 52

0

= Yo + pfl. y1 +

p(p + 1) I

2.

fl. y 2

~ ~

2

+1

= _

0.1064 7 X 0.0064 0 7660 + 5 25 0.7660 + 0.0213 - 0.0018

= = 0.788

Solution 1.7. T he exact value of t he int egral is zero. Let's divide the integral in two equal interval. T hen h = 0.5, x 0 = 0, x1 = 0.5, x2 = 1, and so f (xo ) = 1, f (x1) = - 1, f (x2) = 1 05 . Simpson 's l/3rd met hod give the ans 3 (1- 4 + 1) = - 1/ 3. hence t he ans is not exact. If we divide the int egral into four equal int ervals h ¼, XO= 0, X 1 = 0.25 , X2 = 0.5, X3 = 0.75, X4 = 1.

The value of t he int egral would be

1 (1+ 12

4(i- i) + 2(- 1)+

1) = 0 [email protected]

@Sk J ahiruddin , 2020

12

P hysicsguide

Advanced topics in Mathematical P hysics

Hence if we divide t he int erval in four equal length t hen t he integral will be exact. Hence according to t he question

n = 2.

Solution 1.8. Here f (x) = xex Now. f (2.2) = 19.855 and f(0 .2)

=

0.244

11 oo : oo : 34 ence 1 we

e 1nterva in our equa

engt

t en

t he integral will b e exact. Hence according to t he question

n = 2. Solution 1.8. Here f (x) = xex Now, f (2.2) = 19.855 and f (0.2) = 0.244 2.2 2 So, xexdx = -[19.855 + 0.244] = 20.099 0.2 2 Solution 1.9. Expand cos x in taylor series about taylor series about x = 0. The highest value provider is cos x 2 . Now if t he value has to be correct upto th1·ee decimal point t hen x 2 / 2 < 0.0005. Hence x ::; 57.29 x J0.001 in degrees. Solution 1.10. In Euler m ethod, Relative Error = Local error x no of step. Again, local error rv h 2 l where h = - where l is t he range and n is no of step. n 2 z2 z 1 So, Relative error = 2 xn=-~n n n

Solution 1.11. If we do t he integration analytically, we get 1

e(21rix)dx

e(21rix)

1

21ri

0

e21ri -

1

= --

o

21ri j [email protected]

@Sk J ahiruddin , 2020

13

=O Physicsguide

Advanced topics in Mathematical Physics

Now, let us consider t hat the interval is d ivided into two 1- 0 1 equal parts. So, h = = . 2 2

00: 00: 37 Advanced topics in Mathematical Physics

Now, let us consider that the interval is divided into two 1- 0 1 . equal parts. So, h = = 2 2 1

e(i

• • •

0

1rxldx = h [f (O) + f (l ) + 2f (1/2)] 2

2

= ~ [1 + e21ri + 2ei1r] 4

1 = -[1 + 1 - 2] = 0 4

So, for n

= 2 analytical and numerical solutions are same.

Solution 1.12. The error in Simpson's 1/ 3 rule is

where t is some point in b etween the interval. Here, h = 0.1, (b- a)= l , f (x) = x, so f''( x) = 0, so the term f 4 ( t ) term in t he error formula will give you ZERO. Hence the error is ZERO.

f(x) = x

Solution 1.13. Here,

2

5 0.2 0.4 0.6 0.8 1 0 Xn f( xn) 0.00 0.04 0. 16 0.36 0.64 1.00

n=

[email protected]

@Sk J ahiruddin, 2020

0

1

2

3

4

14

Physicsguide

Advanced topics in :Nlathematical Physics

Now, integration in trapezoidal rule is given by,

11 oo : oo : 39 @Sk J ahiruddin, 2020

Advanced t opics in Nl athematical P hysics

Now, integration in trapezoidal rule is given by,

So, here, 1

2

x dx

02

= ;

[{0.00

+ 1.00} + 2{0.04 + 0. 16 + 0.36 + 0.64}]

0

= 0.1 [1.00 + 2 = 0.1

X

[email protected]

@Sk J ahiruddin , 2020

3.40

X

1.20]

= 0.34

15

Physicsguide

Advanced topics in Nlathematical P hysics

11 oo : oo : 42 15

jahir@physicsguide. in

@Sk J ahiruddin , 2020

Physicsguide

Advanced topics in Mathem atical P hysics

Group Theory

2

Prob 2.1. Which of the following matrices is an element of t he group SU(2)? [NET June 11]

(a) (d)

I I 0 I v'3 -1 2

v'3 2

(b)

l +i

1

v'3

v'3

1

1- i

v'3

v'3

(c)



2 +i

'l

3

I+ i

2

-1 2

Prob 2.2. Let A and B b e two vectors in t hree-dimensional Eucledian space. Under rotation, t he tensor product: T i j =

[NET Dec 13]

A i BJ

(a) reduces t o a direct sum of three 3-dimensional representations (b) is an irreducible 9-dimensional representation (c) 1~educes to a direct sum of a I-dimensional, a 3-dimensional and a 5-dimensional irreducible representat ions (d) reduces to a direct sum of a I-dimensional and an 8dimensional irreducible representations Prob 2.3. Let a and f3 be complex numbers. Which of

the following set s of matrices forms a group under matrix multiplicat ion? [NET Dec 14]

j ahir@physicsguide. in

16

Physicsguide

11 oo : oo : 4s

j a [email protected]

16

@Sk J a hiruddin , 2020

(b)

0 0

a a* /3 /3*

(c) a

Advanced topics in Mathematical Physics

/3

(a)

2

+ 1/32 =

Physicsguide

where

1

/3

where

1

a/3* is real

(d)

a/3 -/=

1

/3 -/3* a*

where

1

Prob 2.4. A p art of t h e group multiplication table for a six elem ent group G = e, a, b, c, d, f is shown below (e is the identity element of G). e

a b

C

d

f

e

e

a b

C

d

f

a

a b

b b C

C

d

d

f

f

e

e d X

f

The entries x, y and z should b e: (a) x = a, y = d and z = c (c )x = c, y = d and z = a

y z

[NET June 16] (b) x = c, y = a and z = d

(d)x = a, y = c and z = d

11 oo:oo:47 (c) x = c, y = d and z = a

[email protected]

(d) x = a, y = c and z = d

P hysicsguide

17

@Sk J ahiruddin, 2020

Advanced topics in :Nlathematical Physics

Prob 2.5. The 2 x 2 ident ity matrix I and the P auli mat rices ax, aY, az do not form a group under matrix mult iplication . The minimum number of 2 x 2 matrices, which includes t hese four matrices, and form a group (under ma-

(NET Dec 16]

trix mult iplication) is:

(a)20

(b) 8

(c) 12

(d) 16

Prob 2.6. Which of t he following sets of 3 x 3 matrices (in which a and b are real numbers ) form a group under matrix multiplication? [a, b E ~ ]

[NET June 1 7]

(a)

1 0 a 0 1 0 b O1

(b)

1 a 0 0 1 b 0 0 1

(c)

1 0 a 0 1 b

:

0 0 1

1 a 0 (d)

b 1 0 0 0 1

Prob 2.7. Consider an element U(cp) of the group SU(2) , where is any one of the parameters of t he group. Under an infinitesimal change ➔

U( ) + 6U() =

+ 6,

it changes as U () ➔ (1+ X (6))U(cp) . To order 6, t he matrix rl\.TV ry, n .,.,. .,..

1 ,.,.l

11 oo : oo : so Prob 2.7. Consider an element U(cp) of the group SU(2), where

. which is



2000 2.426 x 10- 2

~A

=

82440

. he Sol 1.13. The Energy(E) of the photon 1s hv or A The incident wavelength is 0.2 nm. T he expression for Compton (0\ r1

' '





,\

\

r,. \



')

11 oo : 01 : oo ~A

=

2.426 x 10- 2

=

82440

Sol 1.13. The Energy(E) of the photon is hv or : c The incident wavelength is 0.2 nm. The expression for Compton

Sacttering is ~;\ = 2;\c sin

2

~2

Now for maximum wavelength shift we must have 0 = 180°. So the wavelength of the scattered photon

~;\ =

2;\c

;\' - ;\ =

2;\c

,:\' =

2;\c

+ ;\

,:\' = 4.86 pm + 0.2 nm

;\' = 0. 20486 nm [email protected]

@Sk J ahiruddin , 2020

22

Physicsguide

Old Quantum Theory

he Energy( E ) of the initial photon 0.2 nm he Energy( E' ) of t he scattered photon 0.20486 nm Sol 1.14. The expression for Compton Scattering is ~;\ = 2-Acsin

2

~

Here Ac = Compton wavelength of the electron = 0.00243 nm ;\ = Wavelength of incident t he X-rays= 0.24 nm 0 = Angle of the scattered beam is relative to t he incident beam = 60° ;\' = Wavelength of t he deflected X-rays Thus we get

11 oo : 01

: 03

ave eng e -rays = . 0 = Angle of the scattered beam is relative to t he incident beam = 60° A' = Wavelength of t he deflected X-rays Thus we get

9

A = 2 x 0.00243 x 10- x sin 1

2

60°

2

+ 0.24 X

10-

9

A = 0.241215 nm 1

23

[email protected]

Physicsguide

@Sk J ahiruddin, 2020

Old Quantum Theory

According to the law of conservation of energy K e = E - E' he he A A' 6.626 X 10- 34 X 3 0.24 x - 9 = 4.172 X 10- 18 J

= 26.04 eV Sol 1.15. That is

X

108

6.626

10- 34 X 3 X 108 0.241215 x - 9 X

11 oo : 01

: 06

= 4.172 X 10- 18 J = 26.04 eV Sol 1.15. That is h mpe

= 1836 t h of>..c 1 2.426 _ 12 A 1836 X l O

= 1.32 fm Sol 1.16.

~ ).. =

h (1 - cos 0) m ee

cos 0 = 1 - ~ Am ee h 3.25 X 10- 13 X 9.1 X 10- 31 X 3 X 108 = 1 - -----------6.626 X 10- 34 cos 0 = 0.866

0 = 30 j [email protected]

P hysicsguide

24

@Sk J ahiruddin , 2020

Old Quantum Theory

Sol 1.1 7. Equation for phot oelectric effect is hv = W

or ~c = W

+K

+K

Now we have t wo equat ions

he = W + K )..

he

).. = W + 5K

Solving these two equations we get ½ = 0.775V and in

p

second case we get ½ = 3.875V Now Current (I ) = E

11 oo :01 : oa he -A=

W +5K

Solving these two equat ions we get second case we get

½ = 0.775V and in p

½ = 3.875V Now Current (I ) = E I = p

E I = p he

or,

-A

= 50 mA

Putting all t he values we get I

Sol 1.18. Here we have two types of incident electric field. W1

W2

= =

6.28

10 15 s -

X

12.56

X

l

10 15 s-l

The corresponding wavelengths 21r x 3 x 1015 1- w 6.28 x 1015 = 300. 15 nm 21r X 3 X 1015 -A2 = 12.56 x 1015 = 150.07 nm

A _ 21re

[email protected]

25

@Sk J a hiruddin, 2020

Physicsguide

Old Q t1antum Theory

Incident Energy on the metal surface is E1

=

E2 =

10-34 X 6.28 X 10 15 hv = nw = - - - - - - - - - J 21r 6.622 x 10- 19 eV = 4.13 eV 1.602 X 10- 19 15 34 6.626 X 10- X 12.56 X 10 J 21r 6.626

X

11 oo : 01 E1 = hv =

liw

: 1o

= - ·- - - - - - - - - J

21r

19 6.622 X 10V 3 V ----e = 4.1 e 1.602 X 10- 19 34 15 E2 = 6.626 X 10- X 12.56 X 10 J

21r 83.22 X 10- 19 1.602 x 10- 19 eV ~ 52 eV T hus Kinetic Energy

Ek= E1 - Wo = 4.13-2 eV = 2.13 eV

Sol 1.19. T he E lectric F ield

+ sin wt cos w0 t) E = a cosw0t + ~ sin(w + w 0 )t + ~ sin(w E = a (cos w0t

Now t he m aximum angular frequency 6 X 10 15.

[email protected]

Wmax

w 0 )t

= w + wo

26

Physicsguide

@Sk J ahiruddin , 2020

Old Quantum Theory

Maximum energy incident on the metal surface

10- 34 X 5 X 10 15 = liwm ax = 1T J 2 5.272 x 10- 19 e V = 3.29 e V 6.626

E max

1

nn?.

x 1n- 19

X

11 oo : 01

: 13

Maximum energy incident on the metal surface

10- 34 X 5 X 1015 = rlwmax = 7r J 2 5.272 x 10- 19 eV = 3.29 eV 1.602 X 10- 19 6.626

E max

X

The maximum kinetic energy of the photons

(3.29 - 2) eV = 1.29 eV

E k= E max - Wniax =

Sol 1.20. The stopping potential is given by

½ = he

W

e;\

e

6.626 X 10-34 X 3 X 108 320 X 10- 9 X 9.109 X 10- 31

or,

2.1

1.602 X 10- 19 9.109 X 10- 31 X

½ = 1.77 V

Sol 1.21. For t ungsten

e½ = hv -W hv = e½

+W

l.8

=

+ 4.5 =

6.3 eV

For sodium



j [email protected]

=

hv - W = 6.3 - 2.3

= 4

eV

27

Physicsguide

@Sk J ahiruddin , 2020

Old Qt1antum Theory

Sol 1.22. Intensity(! ) ex T 4 (Temperature) Now

I2

Tf

T, -

T .4

11 oo : 01

: 1s

@Sk J ahiruddin, 2020

Old Quantum Theory

4

Sol 1.22. Intensity(! ) ex T (Temperature) Now

rt

I2 4 11 Tl 12 (2T) 4 11 T4

or,

12 = 16 11

or, Sol 1.23. Sol 1.24. Einstein relation

de Broglie wavelength

A= ~ p

From t his relation we will find the expression of moment um for free electron and photon. For photon We know that photon has no rest mass. Thus the expression for momentum becomes

E

Pph

= -

C

Thus the expression fo De-broglie wavelength is A ph

[email protected]

@Sk J ahiruddin, 2020

For free electron

= 28

he E Physicsguide

Old Quantum Theory

11 oo : 01

: 1a

@Sk Jahiruddin, 2020

Old Qt1antum Theory

For free electron Thus the expression for momentum becomes

Pe =

JE2 - (moe2)2 e

Thus the expression fo De-broglie wavelength is

Ae =

he (moc2)2

JE2 -

Sol 1.25. 1

A - A = 2AcSin

,\' - ,\ =

2

0 2

h

me ,\' ,\ l he he me2 1 1 1 -E' E me2 1 1 1 -=-+--2 E' E me 1 1 1 E' = 20 + Sll = 19.23 KeV According to the law of consevart ion of energy K e= E - E'

= (20 - 19.23) KeV = 0.77 KeV j [email protected]

@Sk J ahiruddin, 2020

29

Physicsguide

Old Quant11m Theory

11 oo : 01 [email protected]

: 21

29

P hysicsguide

@Sk J ahiruddin , 2020

Old Quantum Theory

Sol 1.26. This discussion is available in any book of Quant um mechanics. My suggestion is to read t he old quantum

t heory from Art hur Beiser's "Concept of modern physics" first. Sol 1.27. At first we have need t o have a simple knowledge of a charged part icle moving in a circle in a magnetic field.

At first we will equate Lorentz force with Centripetal force and derive the expression of radius of the circular field. 2 mv

qv B = - r

~ ~>

mv r=qB

Angular Momentum m2v2 mvr=--

qB

According to Bohr's model, Angular Momentum is given by nh - By equatinng we get 21r

qB

21r n hqB 2 mv = - 21rm 1 hqB 2 -mv = n 2 41rm [email protected]

30

P hysicsguide

11 oo : 01 1 2 -mv 2

: 23

=n

j [email protected]

hqB 41rm

Physicsguide

30

@Sk J ahiruddin, 2020

Old Qt1antum Theory

Sol 1.28. Bohr radius is propotional to n2

rn ex: Zµ

Here Z = Atomic number µ = Reduced mass of the atom n = Principle quantum number The reduced mass of positron is

(me- ) (me+) µ= (me- )+ (me+ ) (me- )(me+) 2(me+) (me- ) 2 In case of Hydrogen , electron and proton are moving around t he center of mass. As the mass of proton is very large compared to mass of electron, therfore centre of mass shifts nearer to proton. Infact proton is at t he position of centre of mass and may be considered stationary, while electron revolving around it. Now centre of mass of Hydrogen atom

(m e) (mp) µ= (me)+ (mp) (me) (mp) (mp) ,-...J ,-...J

~

_ _ __

me

11 oo : 01

: 26

31

[email protected]

Physicsguide

@Sk J ahiruddin, 2020

Old Quantum Theory

Here mp > > m e so of Positron Tn

~~> mp

+ m e ~ mp.

The Bohr radius

==

n2 P-

[P

IS T HE PROPORT IO NALITY CONSTANT]



2n 2 == P - Zme-

0

== 2

x 0.53A

0

==

1.06A

Sol 1.29. Refer to the previous problem. Energy is propotional to Zµ E n ex 2 n

Earlier we have shown t hat reduced mass of Positron is half of the reduced mass of Hydrogen atom. For this problem keeping all parameters constant in the expression for energy, t he ground state energy of the Positron will be half of the ground state energy of Hydrogen atom. Thus energy of ground state of Positron E ==

3 5 - l · 2

==

- 6.8 eV

11 oo : 01

: 29

will be half of the ground st ate energy of Hydrogen atom. Thus energy of ground state of Posit ron

3 6 E =- l2 · =-68 eV . Sol 1.30.

En= - 13.6

X

Z

2

n2 j [email protected]

32

@Sk J ahiruddin, 2020

Physicsguide

Old Quant11m Theory

For ground state n = l and for H e+ ion the atomic number is Z = 2 Thus we have -13.6 X 4 Eri = l - 54.4 eV Sol 1.31. Read the first chapter of Sakurai's "Modern Quantum Mechanics'' book or read the Spin section from notes

of angular momentum. Sol 1.32. Refer to previous problem.

This is an example of sequential experiments. As we know the, when the beam passes t hrough a Stern-Gerlach apparatus the beam is divided into two seperate beams depending on the orientation of spin along the magnetic field. Here the beam oriented along the magnetic field is allowed to pass through. So when t he unpolaried bean is incident on the first Stern-Gerlach apparatus, electrons whose spin is oriented along z-axis is allowed to pass. As discussed above, the intensity of this beam(I 1 ) is half of the incident beam(I 0 ). Now the incident beam (11 ) is passed through second SternGerlach apparatus, whose magnetic field is oriented along

11 oo : 01

: 31

along z-axis is allowed to pass. As discussed above, t he intensity of this beam(/ 1 ) is half of the incident beam(/ 0 ). Now t he incident beam (11 ) is passed through second SternGerlach apparatus, whose magnetic field is oriented along y-axis. As usual electrons whose spin is oriented along yaxis is allowed to pass. The intensity of t his final beam(I 1 ) is half of t he incident beam(/ 1 ). So we can easily say t hat intensity of I 1 is one-fourt h of 10 . [email protected]

33

@Sk J ahiruddin , 2020

P hysicsguide

Old Quantum Theory

Sol 1.33. This is Davisson-Germer experiment because t here's a maximum at ~ 50 deg in number of electrons detected

which supports electron interference

00 : 01 : 34 intensity of I 1 is one-fourt h of I 0 . j [email protected]

33

@Sk J ahiruddin, 2020

P hysicsguide

Old Quantum Theory

Sol 1.33. This is Davisson-Germer experiment because t here's a maximum at ~ 50 deg in number of electrons detect ed

which supports electron interference

j [email protected]

34

P hysicsguide

00 : 01 : 36 intensity of I 1 is one-fourt h of I 0 . j [email protected]

33

@Sk J ahiruddin, 2020

P hysicsguide

Old Quantum Theory

Sol 1.33. This is Davisson-Germer experiment because t here's a maximum at ~ 50 deg in number of electrons detect ed

which supports electron interference

j [email protected]

34

P hysicsguide

00 : 01 : 39 intensity of I 1 is one-fourt h of I 0 . j [email protected]

33

@Sk J ahiruddin, 2020

P hysicsguide

Old Quantum Theory

Sol 1.33. This is Davisson-Germer experiment because t here's a maximum at ~ 50 deg in number of electrons detect ed

which supports electron interference

j [email protected]

34

P hysicsguide

00:01 :41 intensity of I 1 is one-fourt h of I 0 . j [email protected]

33

@Sk J ahiruddin, 2020

P hysicsguide

Old Quantum Theory

Sol 1.33. This is Davisson-Germer experiment because t here's a maximum at ~ 50 deg in number of electrons detect ed

which supports electron interference

j [email protected]

34

P hysicsguide

00: 01 : 44 intensity of I 1 is one-fourt h of I 0 . j [email protected]

33

@Sk J ahiruddin, 2020

P hysicsguide

Old Quantum Theory

Sol 1.33. This is Davisson-Germer experiment because t here's a maximum at ~ 50 deg in number of electrons detect ed

which supports electron interference

j [email protected]

34

P hysicsguide

11 oo : oo : oo

Problems and Solutions in Basic formalism of Quantum Mechanics: Part-1 Sk J ahiruddin * Souradeep Mondal

*Assistant Professor Sister Nibedita Govt. College, Kolkata Author was t he topper of IIT Bombay M.Sc P hysics 2009-2011 batch He ranked 007 in IIT JAM 2009 and 008 (JRF) in CSIR NET J une 2011 He has been teaching CSIR NET aspirants since 2012

1

@Sk J ahiruddin , 2020

Basic Formalism: Part-1

11 oo : oo : 03 1

@Sk J ahiruddin, 2020

Basic Formalism: Part-1

Contents 1

Problems from NET, GATE, JEST, TIFR & JAM papers 1.1 Probability 1.2 Normalization of Wave Function . 1.3 Orthogonal Stat es . 1.4 Schrodinger Equation . • 1.5 Hysenberg Uncertainty Principle . 1.6 Commutators 1.7 Probability Current Density 1.8 Expectation Values 1.9 Ans Keys • • • 1.10 Solutions . • •













j [email protected]













































































2



















3 •





















































































































































3 4 5 6 9 13 16 16 20 21

Physicsguide

11 oo : oo : 06

j [email protected]

2

P hysicsguide

@Sk J ahiruddin , 2020

1

1.1

Basic Formalism: Part-1

Problems from NET, GATE, JEST, TIFR & JAM papers Probability

Prob 1.1. It is necessary to apply quantum statistics to a [GATE 2007] system of particles if

(a) t here is substantial overlap b etween the wave functions of the particles (b) t he mean free path of the particles is comparable to the inter-particle separation (c) the particles have identical mass and charge (d) t he particles are interacting Pro b 1. 2. Which of the following is an allowed wave funct ions for a particle in a bound state? N is constant and

a, /3 > 0.

[GATE 2010]

- ar

(a) 'lj; = Ne 3 (b) 'lj; = N(l r (c) 'ljJ = N e - a x e -f3(x2+y2 +z2 ) non-zero constant

0

e - ar)

if r < R if r > R

Prob 1.3. The state of a syst em is given by

11 oo :oo :oa if r > R

0

Prob 1.3. The state of a system is given by

[email protected]

3

@Sk J ahiruddin , 2020

Physicsguide

Basic Formalism: Part-1

Where c/> 1 ), c/>2 ) and c/>3 ) form an orthonormal set . The probability of finding the system in t he state lc/>2 ) is __ . (Give your answer upto two decimal places) [GATE 2016] Prob 1.4. A two-state quantum state has two observables A and B. It is known that t he observable A has eigenstates a1 ) and la2 ) wit h eigenvalues a 1 and a 2 respectively, while

B has eigenstates /3 1 ) and /32 ) with eigenvalues b1 and b2 respect ively, and that t hese eigenstates are related by, [TIFR 2015]

Suppose a measurement is made of t he observable A and a value a 1 is obtained. If the observable B is now measured, t he probability of obtaining t he value b1 will be (a) 0.80 (b) 0.64 (c) 0.60 (d) 0.36

1.2

Normalization of Wave Function

A normalised wave-function means t hat

Prob 1. 5. A particle in one dimension is in a potential

11 oo : oo : 11 A normalised wave-function means t hat

Prob 1. 5. A particle in one dimension is in a potential V (x) = Ab (x - a) . Its wavefunction \JI ( x) is continuous [email protected]

Physicsguide

4

@Sk J ahiruddin, 2020

Basic Formalism: Part-1

. t· ·t . d\J! . everywh ere. T h e d 1scon 1nu1 y 1n dx at x = a 1s

[NET

Dec 2016] 2m (a) /i2 A \JI (a)

Ii (b) A ('11 (a) - \JI (-a)) (c) m A (d) 0 2 Prob 1.6. The normalized wave function of a particle can be written as oc

1

where n(x) are t he normalized energy eigenfunctions of a

[TIFR 2017]

given Hamiltonian. The value of N is

(a)

1.3

1/ 7

(b)

6/7

(c)

3/7

(d)

(6 - 2v1) / 7

Orthogonal States

Prob 1.7. The quantum state sin xl t) + exp (itI

Here K is a constant, and a > d. The position uncertainty

[GATE 2019]

(~ x) of the particle is

12

[email protected]

Physicsguide

@Sk J ahiruddin , 2020

(a)

(c)

a

2

+ 3d

2

(b)

12

d2

Basic Formalism: Part-1

(d)

3a

2

+d

2

12

d2

24 6 Prob 1.29. Consider the m otion of a p article along t he x-

11 oo : oo : 34 (a)

(c)

(b)

12 d2

6

(d)

12

d2 24

Prob 1.29. Consider t he motion of a particle along t he xaxis in a potential V (x) = F x . Its ground state energy E 0 is estimated using the uncertainty principle. Then E 0 is proportional to [GATE 2019] (a) F 1 (b) F ~ (c) F ~ (d) F ~

1.6

Commutators

Now you know very well how commutators and uncertainty principles are relat ed. So we have discussed t his topic just after t he uncertainty principle. As you have been int roduced with commutators from your undergraduate days and hopefully you are well accustomed with commutators, here we will state the answer without explicitly solving. For your benefit we have listed different commutator relations at the beginning.

Prob 1.30. If [x, p] = i n, t he value of [x 3 ,p] is [GATE 2007] 2 2 ( c) 3ilix ( d) -3ilix (a) 2ilix 2 (b) - 2ilix 2

j [email protected]

@Sk J ahiruddin, 2020

13

Physicsguide

Basic Formalism: Part-1

Prob 1.31. If x and p are t he x components of t he posit ion and the momentum operators of a particle respectively, the 2 2 commutator [x ,p ] is [GATE 2016]

00: 00: 37 Basic Formalism: Part-1

Prob 1.31. If x and pare t he x components of t he posit ion and the moment um operators of a particle resp ectively, t he 2

commut ator [x , p

(a) i n(xp - px) (d) 2i!i(xp + px)

2

]

[GATE 2016] (c) i!i(xp + px)

is

(b) 2i!i(xp - px )

Prob 1.32. For operat ors P and Q, t he commutator [P, Q-

(b) - Q- 1 [P, Q]Q- 1

Prob 1.33. Consider t he operator a

= x+

!

acting on

smooth funct ions of x . The commutator [a, cos x] is [NET Dec 2016] (a) - sin x (b) cos x (c) cos x (d) 0 Prob 1.34. Given t he usual canonical commutation rela-

t ions, the commutator [A, B ] of A = i (xpy - YPx) and B =

(YPz + zpy) is (a) !i(xpz -PxZ ) (d) - !i(xpz + PxZ)

[NET Dec 2012] (b) -!i(xpz -PxZ) (c) !i(xpz+ PxZ )

Prob 1.35. If the operators A and B sat isfy t he commutation relation [A, B)= l , where I is t he identity op erat or ,

[NET June 2013]

t hen

(a) [eA, B] =

eA

[email protected]

@Sk J ahirudd in, 2020

(b) [eA, B] = [eB , A] 14

]

[JEST 2016] (c) Q- 1 [P, Q]Q

is

(a) Q- 1 [P, Q]Q- 1 (d) - Q[P, Q]Q- 1

1

(c) [eA, B] = P hysicsguide

Basic Formalism: Part-1

11 oo : oo : 39

@Sk J ahiruddin, 2020

Basic Formalism: Part-1

Prob 1.36. Let x and p denote, respectively, t he coordinate and momentum operators satisfying the canonical commu-

tation relation [x, p] = i in natural units (Ii = 1). Then the commutator [x, pe-P] is [NET Dec 2014] (a) i( l - p) e- P (b) i(l - p 2)e-P (c) i( l - e-P)

(d) ipe- P Prob 1.37. Three operators X , Y and Z satisfy t he com-

mutation relations

[X , Y ] = iliZ , [Y, Z]

and [Z, X ] = iliY The set of all possible eigenvalues of the operator Z, in units [GATE 2007] of Ii, is = iliX

1

3

5

(a) {0, ± 1,±2,±3, ..... } (b) { 2 ' 1, 2 ' 2' 2 ' ..... } 1 3 5 1 1 (c) {0,±-,±l ,±-,±2,±-, ..... } (d) {-2, + 2} 2 2 2 Prob 1.38. Let x denote t he position operator and p the canonically conjugate momentum operator of a particle. The commutator [2~p stants, is zero if

(a) , = f3

2

2

2

2

+ {3x , ¾iP + 1 x ] where f3 and, are con[NET Dec 2017]

(b) , = 2/3

(c) , =

v2/3

(d) 2, = {3

[email protected]

@Sk J ahiruddin , 2020

15

Physicsguide

Basic Formalism: Part-1

11 oo : oo : 42 15

[email protected]

Physicsguide

@Sk J ahiruddin , 2020

1. 7

Basic Formalism: Part-1

Probability Current Density

Prob 1.39. A particle of mass m is represented by the wavefunction w(x) = A eikx, where k is the wave-vector and A is a constant. The magnitude of the probability current density of the particle is [GATE 2006]

(a) IA 2 nk

(b) A 2 nk 2m

m

(c)

IA 2 2w 2w(nk )

2

( d)

A 2 (nk )

2

2m Prob 1.40. The wavefunction of particle moving in free space is given by W = eikx + 2e- ikx m

The probability current density for the real part of the wavefunction is [GATE 2012]

(a) 1

(b) nk

(c) nk

(d) 0

2m Prob 1.41. Consider t he wave function Aeikr (r 0 /r), where m

A is normalization constant . For r = 2r0 , t he magnitude of probability current density up to two decimal places, in units of (A 2 nk/m) is__ . [GATE 2013]

1.8

Expectation Values

Prob 1.42. A free particle of mass m moves along t he x direction. At t=O, t he normalized wave function of t he parj [email protected]

16

Physicsguide

11 oo : oo : 44 Prob 1.42. A free particle of mass m moves along t he x direction. At t=O, t he normalized wave function of t he parj [email protected]

16

@Sk J a hiruddin , 2020

Physicsguide

Basic Formalism: Part-1

1 t icle is given by w(x , 0) = ( 1ra)l/ 4 exp 2 a is a real constant

x2

--

4a 2

+ ix

, where

[GATE 2009]

i) The expectation value of the moment um, in this state is

(c) a ii) The expectation value of the particle energy is 2 2 2 n l n 2 n 4a + 1 ( c) 2m 4a 3/ 2 (a) 2m 2a3/ 2 (b) 2m a fi2

(d) 8ma3/2 Prob 1.43. If t he expectation value of t he momentum is (p) for the wave function W(x), then the expectation value of momentum for t he wave function eikx/nw(x) is [JEST 2013] (a) k

(b) (p )-k

(c) (p)

+k

(d) (p)

Prob 1.44. A particle moving in one dimension has the un2

normalised wave function w(x) = xexp - : 2

where A is

a real constant. The expectation value of its momentum is (p) = [TIFR 2014] n x2 n2 n n (a) A exp A2 (b) A2 - 2 A (c) A exp ( - 1) (d) zero Prob 1.45. Consider the two lowest normalized energy eigen-

11 oo:oo:47 ri

ri

(c) ,\ exp ( - 1)

(a) ,\ exp (d) zero

Prob 1.45. Consider t he two lowest normalized energy [email protected]

17

@Sk J ahirudd in, 2020

P hysicsguide

Basic Formalism: Part-1

functions \J! 0 (x) and \Jf 1 (x) of a one dimensional syst em. . d\J! o They satisfy \J! 0 (x) = \J!0(x) and \J! 1 (x) = a - , where a dx is a real constant. The expectation value of the moment um operator in t he state \JI 1 is ri ri

(a) -

a2

(b) 0

(c)

a2

Prob 1.46. The wave function of a quantum mechanical

particle is given by

where 1 (x) and 2 (x) are eigenfunctions with corresponding energy eigenvalues -1 eV and -2 eV, respectively. The energy of the particle in t he state \JI is [JAM 2011] -41 - 11 36 -7 (a) eV (b) eV (c) eV (d) eV 25 25 5 5 Prob 1.47. A particle is in the normalized state \JI) which is a superposit ion of t he energy eignestates E 0 = lOeV) and E 1 = 30eV). The average value of energy of t he particle in t he state \JI) is 20 eV. The state \JI) is given by [GATE 2009]

1 v'3 (a) Ea= l OeV) + IE 1 = 30eV) 4 2 1 2 (b) ~ E 0 = l OeV) + ~ E 1 = 30eV)

11 oo : oo : 49 is 20 eV. T he state '11 is given by t he state 2009] 1 v'3 (a) Ea= lOeV) + IE 1 = 30eV) 2 4 (b)

1

v'3 E 0 =

lOeV )

[email protected]

2

+ v'3 E 1 =

GATE

30eV)

18

@Sk J ahiruddin , 2020

Physicsguide

Basic Formalism: Part-1

v'3 1 (c) 2.IEo = lOeV ) - - 4 E1 = 30eV) 1 1 (d) y'2 E 0 = l OeV ) - y'2 E 1 = 30eV) Prob 1.48. The wave funct ion of a particle is given by 1 y'2 o + 1 , where o and 1 are the normalized eigen-

functions with energies E 0 and E 1 corresponding to the ground state and first excited state, respectively. The expectation value of the Hamiltonian in the state '11 is [NET June 2011] Ea (a) + E1 2

11 oo :oo : s2

j [email protected]

@Sk J ahiruddin , 2020

1.9

Physicsguide

19

Basic Formalism: Part-1

Ans Keys

1.1. a

1.17. b

1.33. a

1.2.

1.18.

1.34.

C

C

C

1.3. 0.28

1.19. b

1.35. a

1.4. d

1.20. nJ(a/ 2m)

1.36. a

1.5. a

1.21. b

1.37.

1.6. b

1.22. d

1.38. b

1.7. d

1.23.

C

1.39. a

1.8. a

1.24. b

1.40. d

1.9. d

1.25. b,d

1.41. 0.25

1.26. b

1.42. b,c

1.11. b

1.27. 0.3 to 3.97

1.43.

1.12. b

1.28. b

1.44. d

1.13.

1.29. d

1.45. b

1.10.

C

C

C

C

11 oo : oo : 55 1.11. b

1.27. 0.3 to 3.97

1.43.

1.12. b

1.28. b

1.44. d

1.13.

C

1.29. d

1.45. b

1.14.

C

1.30.

C

1.46. a

1.15. b

1.31. d

1.47. d

1.16. b

1.32. b

1.48. d

[email protected]

20

Physicsguide

@Sk J ahiruddin, 2020

1.10

C

Basic Formalism: Part-1

Solutions

Sol 1.1. This is standard t heory. Whenever there is a overlap of wave functions you need to apply quantum mechanics. To be more precise when the de Broglie wavelength becomes

comparable to mean t hermal wavelength , you must apply quantum mechanics and quantum stat mech to deal wit h t he problem. Sol 1.2. Think of the graphs of t he solut ions.

The first option diverges at r = 0, the second becomes constant as r increases, the fourth is a constant inside a definite region and discontinuous at r = R which is not allowed. Only option remains is option (c) . Sol 1.3. We need to find the probability that the system which is now in state \JI) will be in state 2 ) subsequent to A

a measurement of any observable Q. 'T1l, ~ h

rQ+ + ".:IQ

1- i Q

+"

n f"'\rTn ".:I

1i Q ~ I \TI \

'T1l, ~

n

T"\rf"'\

h ".:I hi 1i +"'( r " f h n rl _

11 oo :oo : s1 option remains is option (c). Sol 1.3. We need to find the probability that the system which is now in state '11) will be in state 2 ) subsequent to "' a measurement of any observable Q. The first task is to normalise '11). Then probability of finding the system in state 2 is given by

12 2 Probability = ( '11) (\JI W)

22 2 2 2 1 +2 +3 = 0.28 j [email protected]

2 7

21

@Sk J ahiruddin , 2020

Physicsguide

Basic Formalism: Part-I

Sol 1.4. The eigenstate is a 1 ) at the time of first measurement. Now write the state a 1) in terms of t he /31) and

/32)

3 4 la1) = /31) + /32) 5 5 Now if B is measured, the probability to get b1 will be 3

5

2

=

9 = 0.36

25

Sol 1.5. As we know to find the discontinuity we must approach the function from both sides. So the limit will be a - E to a + E. Now the Schrodinger Equation is d2 w - m dx 2 + Ab(x - a)W = E\JJ 2

n2

11 oo : 01 : oo a-

to a+ E. Now the Schrodinger Equation is E

d2 w - m dx 2 + A8(x - a) w = Ew 2

n2

Now integrating both sides within the discussed limit is n2

2m

a+E

d2w dx2 dx

a+E

a+E

b(x - a)w dx

+A

a-E

=

E

w dx

a-E

/i2

d\J!2 dx

2m

Thus the discontinuity is

a-E

d\J! 1 + A\ll(a) dx d'112 dw 1 dx dx

2m /i

2

=0 2

= ~;i w(a)

AW (a)

22

[email protected]

Physicsguide

@Sk J ahiruddin , 2020

Basic Formalism: Part-1

Sol 1 .6 . q>n(x) are t he normalized energy eigenfunctions of a given Hamiltonian. Now we will expand the w (x). CX)

n

1

vl7 = N

q>o +

1

1

1

vl7

vl7

vl7

Now 1 7 2

~> l =N (

1

l\

1- = )

+

1 7

1

2

+

3 • • •

7

11 oo : 01

: 03

1

7 1=

====;:~>

1

+ 7

2

1

3

+ 7

• • •

N2 _ _1__

1 1-7 1= N

====;:~>

2

X

1

6 -

7

6 7

Sol 1. 7. Orthogonal states mean that the inner product between the states will be zero

There is no short process to solve t his problem other than plucking the states from the options and putting in t he def23

[email protected]

@Sk J ahiruddin, 2020

Physicsguide

Basic Formalism: Part-1

inition and wait to see which option holds. We found t hat

option d satifies the definition of orthonormal states. Now W1)

= sin x

t)

+ exp(icp) cosx l -!-)

Thus

('112 W1) = ( - exp (icp) cos x(t

=-

+ sin x(-!- I) (sin x t) + exp(icp ) cos x -!-) )

exp( icp) cos x sin x(t I t) - exp (2icp) cos x(t I -!-) 2

+ sin

+ exp (icp) cosxsinx(-!- -!-) = - exp(icp) cos x sin x + exp (icp) cos x sin x =0

2

x(-!- t)

11 oo : 01 : os =-

exp (icp) cos x sin x(t

I t)

2

- exp (2icp) cos x(t .J,)

2

+ sin x(.J, t) + exp (icp) cosxsin x(.J, .J,) = - exp (icp) cos x sin x + exp (icp) cos x sin x =0 Sol 1.8. At first we will list out wh at is known t o us. "

0 1) = 111) (u v) = 0

"'

"'

0 12) = 2 2)

(v O v) = 7/4

Now

(v cos

2

6 v) = 7 / 4 2

cp + 2 sin cp = 7 / 4 2 sin cp = 7 / 4 - 1 cp =

j [email protected]

7r

3

P hysicsguide

24

@Sk J ahiruddin , 2020

Basic Formalism: Part-1

Simila rly

(u v) = cos 0 cos cp (1I 1) + sin 0sin cp (2 I2) = 0 =;►

cos(0 - cp) = 0 7r

0- cp = 2 0-

7r

7r -

- 2+ 3-

0=

51r Jr- -

6

51r

6

7r

= --

6

11 oo :01 : oa

Sol 1. 9. The Schrodinger Equation is

-n,2 d2\J! + V\J! = E\J! 2m dx 2 It is given that n = 2m = 1 and E = 0. Putting these values we have d2 \J! - V\J! = 0 dx 2 d2

dx 2 [A exp( -x

4

)] -

d2

dx 2 [exp(-x

4

[-12x exp(-x

4

)

+ 16x 4

6

exp(-x

exp (-x [16x )

6

-

4

12x

V[exp(-x

4

V[exp (-x

4

V[exp (-x

4

V[exp (-x

4

)] -

d 4 3 dx [-4x exp(-x )] 2

V [A exp (-x

4

-

)] 2

] -

V == l6x 25

[email protected]

6

-

)]

=0

)]

==

)]

== O

)] )]

12x

0

=0 =0

2

Physicsguide

Basic Formalism: Part-1

@Sk J a hiruddin, 2020

Sol 1.10. We will solve following the procedure illustrated in t he previous problem but we will add some insights.

Now the Schrodinger Equation is

n,2 d2\J! d ?. 2m x.,

+ V\J!

The wave function is \J! ( x) = ,...

==

e - iax+b

E\J!

11 oo : 01

: 1o

Now the Schrodinger Equation is

The wave function is \ll ( x)

=

e - iax+b

2

Now

d

dx 2

(e - iax+b ) =

(-ia)2 X e - iax+b =

-a2e- iax+b

Thus we have

n2a2 \ll 2m

+ V \ll = E\ll

Now we know that energy (E) is t he eigenvalue of t he Schrodinger Equation which is a constant number. In the solut ion of Schrodinger Equation we will equate the constant terms with energy (E) and all the t erms containing t he independent variable with potent ial (V). Thus from the solution we can easily say t hat the potential • 1s zero.

Sol 1.11. Needless to say we will follow t he steps depicted

26

[email protected]

@Sk J ahiruddin , 2020

. 1n

Physicsguide

Basic Formalism: Part-1

pro blem ?? ... \JI

2

= exp( -x /2) cosh (v'2x)

!! = -x exp(-x / 2) cosh(v'2x) + v'2 exp (-x / 2) sinh(v'2x) 2

..J2 ,T,

2

11 oo : 01

: 13

. pro 61em ?? 1n ...

w = exp( -x

2

/ 2) cosh ( h x)

!! = -x exp (-x / 2) cosh ( hx) + V2 exp(-x / 2) sinh( hx) 2

2

d w 2 2 2 dx 2 = (x + 1) exp( -x / 2) cosh( h x) - 2V2 exp(-x / 2) sinh( v 2

Now the Schrodinger Equation is

d2 w 2m dx2 + Vw = Ew

n2

ld2 w - 2 dx2 + Vw

-x 2 2

w + V2 exp (- x / 2) sinh( h x) 2

1

= Ew

w + Vw = 2

Ew

Now 2

V'V V

= ~ IV - V2 exp( -x / 2) sinh( h x) 2

=

x2 2

- v'2tanh(v'2x)

Sol 1.12. Same as previous problem . Sol 1.13. This is a t heory. Read Delta function well discussion from t he notes of one dimensional potentials. Sol 1.14. Bot h the ground and first excited states need to follow the Schrodinger equation. Using n = l and m = 1/ 2 j a hir@physicsguide. in

27

Physicsguide

@Sk J a hiruddin , 2020

Basic Formalism: Part-1

we get 2

8 7/Jo

- -ox2 + V 'l/Jo =

-4'l/Jo

11 oo : 01

: 16

@Sk Jahiruddin, 2020

Basic Formalism: Part-1

we get 2

8 7/Jo

- -ox2 + V 1/Jo = -41/Jo and

a - ox2 + V 'l/J1 = 2 1/J1

l 1P1

Now put '1µ 1 = 1/Jo sinh x the second equation. Keep in mind t hat

dsinh x h dcosh x - - - = cos x · dx dx ' Calculate t he derivatives and obtain . h 02'1f'o

Sln

=

X OX 2 ·

+

2

COS

h o'l/Jo X OX

=

. h ,,,,

+ Sln

X 'f'O

. h sin x

+ V 'ljJ0 sinh x

- 'ljJ0 sinh x

Now use the Schrodinger equation for ground state wave function 82 '1/Jo - ax2

_

+ V '1f'o -

-4'1po here. Doing some algebra you can easily get

0'1f'o

- 4'1µ 0 - 2 coth x -

OX

- '1f'o = -1Po

Now doing integration or solving first order linear equat ion you can get 2

'1f'o = sech x

[email protected]

28

@Sk J ahiruddin , 2020

Sol 1.15. The double derivative gives

Physicsguide

Basic Formalism: Part-1

11 oo :01

:1

a

@Sk J ahiruddin , 2020

Basic Formalism: Part-1

Sol 1.15. The double derivative gives

d2 dx2A exp

b2x2 -2

b2x2 = A exp - - 2

(-b + b x 2

4 2

)

Substitute the wave function in the Schrodinger equation.

n,2 2

m Aexp

= EAexp Hence

n,2

--

Now at x

=

2

4 2

(-b + b x

2m 0, b = 0. So

)

+V = E

n2b2

E =2m

n,2

4 2

V (x) = - b x 2m Sol 1.16. This is a theory. Read Delta function well discussion from the notes of one dimensional potentials. Sol 1.17. Think about t he boundary conditions. At t he boundary x = 0 t he potential diverges to infinity. So t he wave function must be ZERO there. And when x becomes

large, i.e x -+ oo the wave function again must converge to ZERO. The only option satisfies those is option (b). j [email protected]

@Sk J ahiruddin , 2020

29

Physicsguide

Basic Formalism: Part-1

11 oo : 01 [email protected]

: 21

29

P hysicsguide

@Sk J ahiruddin , 2020

Basic Formalism: Part-1

Sol 1.18. The original Hysenberg uncertainty principle is given by axay 2::

n

2

where ax and ay are the standard deviations in the measurement of any dynamical variables. Refer to Introduction of Quantum Mechanics(2 Ed.) by David J. Griffith (page 112 - 120) We have an "uncertainty principle'' for every pair of observables whose operators do not commute. The most important and commonly used pairs include 1) Position - Momentum 2) Energy- Time 3) Angular Position - Angular Momentum For the sake of completeness we must mention some important concept about Energy - Time uncertainty principle. Position, momentum, energy t hese are all dynamical variables but we cannot measure ''time'' of a particle. In particular flt in t he Energy - Time uncertainty principle is not the standard deviation of a collection of time measurements but rather the t ime the system takes to change substantially. Hopefully we have intrigued your interest in this context . Refer to Introduction of Quantum Mechanics(2 Ed.) by David J. Griffith (page 116) to know more. [email protected]

30

Physicsguide

11 oo : 01

: 23

Refer to Introduction of Quantum Mechanics(2 Ed.) by David J. Griffith (page 116) to know more. j [email protected]

Physicsguide

30

@Sk J a hiruddin, 2020

Basic Formalism: Part-1

The position-momentum uncertainty principle is written in t he form

n

i:l.x !.:l.p > - 2

where i:l.x is the standard deviation in the measurement of posit ion and i:l.p is t he standard deviation in t he measurement of momentum. Now !.:l.x ~ 21rr = 2 x 3.14 x 10-15 m Thus we have !.:l.p

=-

n

21.:l.x 6.63 X 10- 34 4 X 3.14 X 10- 15

= 5.279 ~ 100

x 10-

20

x

1

_ x _ 28 eV / c 5 36 10

MeV/c

Even t hough the actual uncertainty principle states that t he product of standard deviations must be greater t han or equal to ~' there are some problems where we assume the product to be greater than n only. Whether we take ~ or n t he order of the answers will be same. Here if we assume t he uncertainty principle as

11 oo : 01

: 26

t he order of the answers will be same. Here if we assume the uncertainty principle as

31

[email protected]

Physicsguide

@Sk J ahiruddin , 2020

Basic Formalism: Part-1

Then the expected value of momentum comes to be around ~P ~ 200 M eV /c

We notice that using both t he equations t he order of the •

answer 1s same.

Sol 1.19. Refer t o previous problem Prob 1.18 .. The posit ionmomentum uncertainty principle is written in t he form

Ii ~ x ~p > - 2 Now ~ x ~ L 3 = (10- 12) 3 m 3 = 10- 36 m 3 Thus we have

Ii ~ p=2~x 6.63 X 10- 34 2 X 10- 36 = 331.5 units Now expression for kinetic energy is given by E ~p2

E= 2m

(331.5)

2

=

~ 2

p

2m

11 oo :01 : 2a . . energy is . given . by E Now expression for k.inetic

6.p=-

2m

6_p2

E= 2m 2

=

(331.5) 2 X 1.67 X 10- 27 31 3.3 X 10 J 32

j [email protected]

Physicsguide

@Sk J ahiruddin , 2020

Basic Formalism: Part-i

Sol 1.20. In the last example we have seen how to estimate t he energy from the Position - Momentum uncertaint y

relation. This example is a perfect elaboration. The general expression of Energy is given by 2

E= P + V 2m p2

E = 2m

+ ax2

Now we replace x by 6. x and p by 6.p. Thus expression for Energy becomes

E =

6_p2

2 - - + a6. x 2m

Now according to the Uncertainty Principle (as given in the quest ion)

n

6.p= 26.x The relation for Energy becomes t;,2

E =

8m

6.

X

2

+ a6. x

2

11 oo : 01

: 31

---

2~x The relation for Energy becomes /i2

E =

8m

~

X

2

+ a~x

2

To get the minimum energy we need to take t he derivative

[email protected]

P hysicsguide

33

@Sk J ahiruddin, 2020

Basic Formalism: Part-1

of energy dE

d(~ x) = O 2'/i2 8m~ x3

+ 2a~x =

0 1

/i2 ~ X=

4

8am

Now t he minimum energy is 1

/i2

2

E = - - - - -1 + a /i2

8m

2

8am

8am

liva, =-----+-2

li xv'Sam,

v]m

8mli

liva, = --+-riy'a

v]m 2

X

v]m

riVo,

2 x v'2ffi .

,--

11 oo : 01

: 34

8mli nva + _li_ va_a ~

~

/iyia 2 x v2ffi 2

X

nva y2m Sol 1.21. The uncertainty in Energy or more accurately the standard deviation in the measurement of energy is given by

[email protected]

P hysicsguide

34

@Sk J ahiruddin , 2020

Basic Formalism: Part-1

Now t he considered state is \JI =

W1

1

\JI

+ W2

NOT NORMALISED

1

= y'2 \JI 1 + y'2 \Jf 2

The expectation value of Energy of t he oscillator in state \JI •

IS

1

v'2 Similarly the expectation value of square of Energy of the oscillator in state \JI is

11 oo : 01

: 36

Similarly the expectation value of square of Energy of the oscillator in state \JJ is

2

Ef+

1

v'2

35

j [email protected]

Physicsguide

Basic Formalism: Part-1

@Sk J ahiruddin , 2020

Now t he standard deviation of the energy of t he state \JI bt.E =

J (E

2

) -

(E) 2

1 2 1 2 1 ( - E 1 + -E2 - - E1 2 2 4 2Ef

+ 2E? -

+ E2

)2

Ef - E? - 2E1E2 4

Ef

+ E? -

2E1E2

4 E1 -E2 2

Sol 1.22. This is little bit lengthy. You need to calculate the average value of the momentum operator

11 oo : 01

: 39

Sol 1.22. This is little bit lengthy. You need to calculate

t he average value of t he momentum operator

(p) =

a

v'f5 (a2 -

-a

4a5/ 2

x2) (-iii)

v'f5 a (a2- x2) dx 4a 5/ 2

ax

This integration will be ZERO as you will be integrating over an odd function with symmetric limit both side of t he or1g1n. T he average value of p 2 over the wave function is •



\P2) = -n2

15

a

x 16a5

( a2 -

x2)

82 8x2 (a2 -

x2) dx

- a

I leave the integration on you. The result will be

sn

2

2a 2 [email protected]

Physicsguide

36

@Sk J ahiruddin, 2020

Basic Formalism: Part-1

Hence

sn

2

2a2

_

= 0

v'5n v"ia

Sol 1.23.

Get t he posit ion space wave function 1

1/J(x ) = y2n -

CX)

2

Then calculate t he average values of x and x over 'lj; (x). Sol 1.24 . C;nm n::1.t .ih lP( rnmm11 t.inP-l nnP.r ::1.t nr ~ h::1., rP. r.n m nl P.t .P.

11 oo : 01 'lj; (x) =

: 42 +oo

1

J21r

cp(k )eikx dk

- oo

Then calculate the average values of x and x 2 over 'ljJ (x) . Sol 1.24. Compatible( commuting) operators have complet e sets of simultaneous eigenfunctions. Here operat ors A and B share all t he eigenstates i.e. t hey are commuting. So the product of uncertaint ies ~A~B is 0. Refer to Introduction of Quantum Mechanics(2 Ed.) by David J. Griffith (page 112). Sol 1.25. Standard Theory In problem Prob 1.24. we have ment ioned that compatible operators have complet e sets of simultaneous eigenfunctions. Here we will elaborate this point a bit. If two operators are commutating then every non-degenerate eigenfunction of one operator must be the [email protected]

Physicsguide

37

@Sk J ahiruddin , 2020

Basic Formalism: P art-1

eigenfunction of the other operator. Let us explain this with an example. Let the two observables 2

be 01 = J and 0 always commute.

2

2

= l z. We know that [J , Jz] = 0 They

Now let the states be denoted by J, l z) For J = 2 we have Jz = - 2, - 1, 0, 1, 2 Now we consider a state '11)

1

1

= v'2 2, 1) + v'2 12, - 1)

Now I'll) is an eigenfunction of J 2 but not of l z. We see t hat for a degenerate eigenst at e of J 2 we do not have a simultaneous eigenstate of l z.

11 oo : 01

: 44 J

Now we consider a state '11)

1

1

= v'2 2, 1) + v'212, - 1) 2

Now '11) is an eigenfunction of J but not of l z. We see t hat for a degenerate eigenstate of J 2 we do not have a simultaneous eigenstate of Jz. Note : To know how J 2 and Jz act on a state refer to t he chapter on Angular Momentum. Sol 1.26. This is easiest. The width of the potential well a is t he posit ion uncertainty.

Momentum uncertainty is fj_p= -

Ii

2/j_x

2a

The actual moment um will at least of the order of moment um uncertainty. We take

j [email protected]

Physicsguide

38

@Sk J ahiruddin , 2020

Basic Formalism: Part-1

Hence t he energy p2 E = 2m

2

2a

1

fi2

2m

8ma 2

Now substitute t he values. Sol 1.27. Proceeding similarly as previous problem fj_p

=

Ii 2a;

11 oo : 01

: 47

Now st1bstitute the values. Sol 1.27. Proceeding similarly as previous problem

n

l:ip

= 2a;

Sol 1.28. Calculation is lengthy. Better to skip Sol 1.29. From uncertainty principle. (Ignored t he factor 1/ 2 as t hat won't change the result) . !:ip

=-

n

!:ix

The potential is a modulus potential. Let t he part icle is confined between t he length !:ix . As in t he discussion in t he previous problems, t he moment um is of the order of uncertainty of t he momentum. So t he energy 2

E = (l!.p) 2m

or

+ F (l!.x)

fi2 E

[email protected]

=

2m(!:ix)2

+ F!:ix Physicsguide

39

@Sk J ahiruddin, 2020

Basic Formalism: Part-1

Now to find t he best estimate of the energy, we need to minimize it with respect t o variable !:ix (we did it when we derived t he energy of hydrogen atom or harmonic oscillator using uncertainty principle) dE d!:i x

fi2 m(!:ix)3

+F =

0

11 oo : 01

: 49

derived t he energy of hydrogen atom or harmonic oscillator using uncertainty principle)

dE d~x

n,2 m (~ x)3 + F = 0

You get n2

1; 3

mF

Substitute the value of ~ x in the expression of energy. Hence you get the energy proportional to F 213 Sol 1.30. Simple

Sol 1.31. The commutator relations we will be using are list ed at t he b eginning.

[x2, p2]

=

[x2, pp]

=

p[x2,p]

+ [x2, p]p

= 2i n px + 2i n xp

=

2i n (xp + px)

Sol 1.32. Commutator [P, Q- 1] = PQ- 1

j [email protected]

Q- 1p

40

@Sk J ahiruddin , 2020

Now

-

P hysicsguide

Basic Formalism: Part-1

_ Q-l [P, Q]Q-1 = -Q- l [PQ _ QP]Q-l

= -Q-l [PQQ-l - QPQ-l ] = - Q- l [P - QPQ- l]

11 oo : 01 : s2 Now

- Q-l [P, Q]Q-l = - Q-l [PQ - QP]Q-l

= = = =

- Q-l [PQQ- l - QP Q-l] - Q- l [P - QPQ- l] - Q- lP

+ Q- lQPQ - l

PQ- l - Q-lp d

Sol 1.33. The operator is a= x + dx . Now we will calculate t he commutator

[a, cos x]

=

x

d

+ dx , cos x

= [x, cos x]

d

+ dx ' cosx

Let the commutator act on a state W

d

[x, cos x] W + dx , cos x W = [x cos(x) - cos(x )x]w + =0+

d dw dx cos(x )w - cos(x) dx

dw dw cos(x) dx - sin(x )w - cos(x) dx

= - sin(x) w Thus we have [a, cos x]

[email protected]

@Sk J ahiruddin , 2020

Sol 1.34.

= - sin(x)

41

Physicsguide

Basic Formalism: Part-1

11 oo : 01 : s4 @Sk J ahiruddin, 2020

Basic Formalism: Part-1

Sol 1.34.

[A, B] = [i(xpy - YPx), (YPz + zpy)]

= i [xpy, YPz] + i [xpy, zpy] - i[YPx, YPz] - i[YPx, zpy] = i [xpy, YPz] + 0 - 0 - i[YPx, zpy] = ix[py, YPz] + i[x, YPz]Py - iy [px, zpy] - i[y, zpy]Px

= ix[py, YPz] + 0 - 0 - i [y, zpy]Px = ix X ( -i fi )pz - iz X ( i fi )Px = fi[ XPz + ZPx] Sol 1.35. To solve this problem we will use the list of formulae mentioned above. Now 2 A

A A3 eA= l+ -+-+-+ · ·· 1! 2! 3! Now [

A

e '

B] = [1 B] '

2

[A, B] +

_ O

-

1!

3

[A , B] +

2!

[A , B] ... +

3!

A[A, B] + [A, B]A

2

+ l + 2! + 2 2A 3A =1 + - + + ··· 2! 3! A A2 A3 = l +-+-+-+ ··· =eA 1! 2! 3!

2

A [A , B] + [A , B]A . . . 3!

Thus we have [eA, B] = eA [email protected]

@Sk J ahiruddin, 2020

42

P hysicsguide

Basic For malism: Part-1

11 oo :01 : s1

@Sk J ahiruddin , 2020

Basic Formalism: Part-1

Sol 1.36. You can solve the problem just as we have solved the previous problem. Here we have used the relation from

t he previous problem.

[x,pe-P] = [x, p]e-P + p[x, e-P] = ie-P - ipe-P

= i( l

- p)e- P

Sol 1.37. These operators follow the Angular momentum formalism. So their eigenvalues will be integers or half inte-

gers. Both positive and negative, including zero. Sol 1.38. The commutator b ecomes

/3 m

ry 2m

We have calculated b efore

which is non zero. Hence the commutator will be zero only if ry /3 = 0 ⇒ ry = 2/3 m 2m j [email protected]

(c)Sk J a,hir11ddin. 2020

43

Physicsguide

Basic Forma.lism: Part-1

11 oo : 01 : sg

43

j [email protected]

P hysicsguide

@Sk J ahiruddin, 2020

Basic Formalism: Part-1

Sol 1.39. The expression for probability current density in 1-d is given by

.

iii J = 2m Now we know that

w(x) = A eikx w*(x) = Ae-ikx Putting these values in t he expression we get

j = i ii [A eikx

X ( -ik )A e- ikx

2m iii . = - x( - 2ik)x 2m = A 2lik

- Ae- ikx

X ( ik )Aeikx ]

Al

2

m

Sol 1.40. This problem is somewhat similiar to problem Prob 1.39. but here we have to consider t he real part of the wavefunction The expression for probability current density in 1-d is given by

.

i ii J= 2m [email protected]

44

Physicsguide

11 oo : 02 : 02 .

J=

in 2m 44

[email protected]

Physicsguide

@Sk J ahiruddin, 2020

Basic Formalism: Part-1

Now we know that \ll (x)

=

eikx

+ 2 e - i kx

Now the real part of t he wave-function is \ll (x)

real =

cos(kx)

+ 2 cos(kx) = 3 cos(kx)

Since W = \II* the probability current density is zero.

j =

in [-9cos(kx) sin(kx) + 9cos(kx) sin(kx)] 2m

=0 Sol 1.41. Refer to problem Prob 1.39. The expression for probability current density in 1-d is given

by

.

J=

in 2m

Now we know that

w(x) = w(x)* =

Aeikr

ro r

A e - ikr

ro r

11 oo : 02 : os w(x) = w(x)* =

[email protected]

A e ikr

-

r

ro

A e - i kr

r

45

Physicsguide

@Sk J a hiruddin , 2020

Basic Formalism: Part-1

Putting these values in the expression we get

. in

J=

To -

ikr

Ae

2m

in

Ae

2m

r

X

m

nk

-

X

m

(ik) A e

r

2

-

r

ro -

-ikr

in - x Al X 2m in 2 - x Al X 2m nk

( - i·k)A e - i kr -ro

(-ik)

ro ro

r

ro -

r

·k

- Aei r

r 2

r5 r3

r

(-2ik) r

ro A l2

2·k

- A e- i kr ro2

ro r2

ro (ik) -:;:

-

2

+

r2

0 r3

2

2

2r0

l

A l2 -

4

= o. 25 nk x IA 2 m

Sol 1.42. (i) The definition of expectation value of any her-

mitian operator is given by A

(A) =

A

A

w*A w dx = (w A w)

The first definition involving the integral over the whole space follows from the wave-function representation of Quant um Mechanics. The second definition follows from the m a-

11 oo : 02 : 01 (A)

=

= (\JI A \JI)

\JI* A \JI dx

The first definit ion involving t he integral over the whole space follows from the wave-function representation of Quant um Mechanics. The second definit ion follows from t he mat rix representation of Quantum Mechanics. The expect ation value of momentum is calculated as fol46

j [email protected]

Physicsguide

@Sk J ahiruddin , 2020

Basic Formalism: Part-1

lows A

A

(P) =

'l!* P 'l! dx

1

00

-oo 00

-oo

-

x2

.

x2

1

A

-ix p (2wa)l/4 exp - 4a2 + ix dx (21ra)l/4 exp 4a 1 x2 . d l x2 - ix (- in dx) (2na)l/4 exp - 4a2 + ix 1/4 exp 2 (2?ra ) 4a x2 x2 exp - - 2 -ix- - 2 +ix [- : 4a 4a 2 oo

00

1

= (-in) (21ra)l/2

-

-

2

dx

+ i] dx

2

00

(-in) (21ra)1/2

2

x exp - 2a2 oo

[

X

+i

- 2a2

00

(in) x exp 1 2 2 (2na) / x (2a ) _00

]

dx

n

x2 2a

dx

2

+ (21ra)1/2

x2

00

-oo

exp

2a

2

The first part of the integral is an odd int egral. We know t he integ1nation of odd integral over a complete interval is zero. So we neglect the first part.

,. ,

n

(P ) X

Let

v122a = t

= (2na )1 / 2

=~► dx

,. ,

(P ) =

x2

00

- oo

exp

2a2

= v12a dt

nv12a I~

roe exp r-t l dt 2

dx

dx

11 oo : 02 : 1o exp

(P) = (21ra)l/2 X

Let

=~► dx

v1220'. = t

"

2a2

- oo

= y12a

nv12a

OO

(P) = v'2iro,_ 21ra

ny12a

dt

exp [-t

2

]

dt

- oo X

ft

v'2ira = nv/a 47

[email protected]

@Sk J ahiruddin,

Physicsguide

2020

Basic Formalism: Part-1

(ii) The expect ation value of Energy is given by "

(E ) =

\J!*E \J! dx 1

00

=

-oo

x2

n2 d2



-i x (21I"O'. )1/4 exp -4a 2

2mdx 2

1 (21ra)l/4 exp -:

Now d\J! 1 x2 -=---e xp -+ix - 4a 2 dx (21ra) 1/ 4 d 2 \J! dx 2

=

1 x2 (21ra)l/4 exp - 4a2

+ ix

x2 - (21ra)l/4 2a2 exp - 4a2 1

1

X

.

X

--+i 2a 2

X

2

- 20'.2

+i

+ ix

Thus we have

" - n,2 (E ) = 2m(21ra) 112

..

r

x2 . exp - -2 -ix 4a 00

CX)

')

77

x2

exp --+ix 4a 2

2

11 oo : 02 : 12 Thus we have - fi2

A

x2

00

(E ) = 2m(21ra)l/2 1

-oo

x

exp - -2 + ix 4a

- - -ix

exp

4a 2

2

- 2a2 exp - 4a2 - n2

x2

.

+ ix

dx

x2 exp ix + ix 2m(21ra) 1/ 2 _00 4a2 4a2 x2 ix 1 - 4- -2- 1 - -2 dx 4a a 2a x2

oo

48

[email protected]

Physicsguide

@Sk J ahiruddin , 2020

Basic Formalism: Part- I

x2

oo

2m(21ra) 1/ 2 •

00

4a4

00

'l

a

_

2

x2

exp

2a 2

dx-

x2

xexp

2a

-oo

t;,2

+2m(21ra)1/2

dx

2

1+

1 2a2

00

x2

exp - oo

2a

2

dx

We will neglect t he integration with imaginary part which is actually an odd integral and per£orm the rest of the integration. Let

X

M

v2a

=t

-ri2 (E) = 2m(21ra) 1/ 2 A

1+

1

2a

2

x2 x2 4 4 exp 2 2a -oo a 00 x2 exp dx 2a 2 oo

dx

2

11 oo : 02 : 1 s "'

oo

-/i2

(E)

=

2m(21ra) 112

-

1 1 +2a2

- oo oo

4

a

4

x2 exp

-oo

dx

t2

v'2a exp [-t2] 00

1 2 2a

dx -oo

49

j [email protected]

Physicsguide

@Sk J a hiruddin, 2020

Basic Formalism: Part-1

yW 2m(21ra) 112 -fi2

2y'2a

2m(21ra) 1/ 2

2v'2a

1+ l 2a 2

Vr.;--2 ~7rQ

yW - ~a - yW

v'2a

-/i2

yW - 4a 2 yW - 2yW

2m(21ra) 112

2y'2a

2 2 /i (4a

2

dx

2a2

- oo

2m(21ra) 1/ 2

2a

x2

exp 00

1+

x2

+ 1)

8ma3/ 2

Sol 1.43. We know how to find t he expectation value of any operator. The expectation value of moment um for t he wave funct ion w(x) is 00

(p) = - 00

w*

-iii d

dx

'11 dx

11 oo :02 : 1a The expectation value of moment um for t he wave funct ion w (x) is 00

(p) =

. d w* -in dx -oo

'11 dx

00

= (-in)

w*w' dx - oo

Now the new wave function is e·ikx/nw (x). The expectation

[email protected]

Physicsguide

50

@Sk J ahiruddin, 2020

Basic Formalism: Part-1

value of momentum for this wave function is 00

-ikx * exp n w

. d - in dx

ikx exp n w dx

-00 00

= (-in)

exp

-ikx ,T,*

n

-oo 00

+ (-in)

exp

'±'

n

-ikx ,T,*

n

ikx exp n wdx

ik

'±'

exp

ikx ,T,, d

n

'±'

X

- oo 00

= (k)

00

w*w dx - oo

+ (-in)

w*w' dx - oo

= (p) + k Sol 1.44. In this problem before computing t he integral to

11 oo : 02 : 20 = (k)

\JJ*\JJ dx

+ (-iii)

\JJ*\JJ' dx

- oo

- oo

= {p) + k Sol 1.44. In this problem before computing t he int egral to find t he expectation value of momentum we must normalise t he wave function . As we have solved the previous problems following t he footsteps we will get opt ion d as t he answer.

d\Jlo Sol 1.45. The st at e is '111(x) = a dx The expectation value of moment um for t he wave funct ion \JJ (x) is oo -

(X)

\JI* -iii d 1 dx

j [email protected]

\JI 1 dx

51

P hysicsguide

@Sk J ahiruddin , 2020

Basic Formalism: Part-1

Integrat ing by parts

d\JJ o d2\JJ o d I = (- i fia*a) 2 X _ 00 dx dx 00 d'1lo dwo dx dx _00 oo

_ 00

I = -

\JI* x a la*

- oo

00

. d -iii dx

00

I = - {

X

'117 ( -iii~

00

- oo

2

d \JI o dw o dx dx 2 dx

X

d\Jf 0 a dx dx

l \JI , dx

11 oo : 02 : 23 1 = 0 - ( -in a

2

) _

1= -

oo

Wf

X O'.

- oo 00

1= -

dx2 dx dx 00 . d d\llo d -indx a dx x

w*1 -in d

'111 dx

dx

- oo

0

0

1= - 1

21 = 0

1= 0 Thus we have \Px) = 0 Sol 1.46. We had already shown how to find the expecta-

tion (average) value of any operator in the wave-function representaion of Quant um Mechanics. Here in this sum we will learn how to find t he expectation (average) value of any operator in t he matrix representaion of Quantum Meehan•

lCS .

We know t hat in Hilbert space any wave function can be written as a linear combination of ort honormalized eigen52

[email protected]

Physicsguide

@Sk J ahiruddin, 2020

Basic Formalism: Part-1

functions of an observable operator provided t he eigenfunct ions are complete.

w (x, t) =

L Cn(t)fn(x) n

Here f n, ( x) = Normalized eigenfunction of t he operator which is complete en(t ) = It is t he co-effecient offn which basically t ells ll l

1



1



J

1



,, T , , ,

11 oo : 02 : 2s n

Here f n(x) = Normalized eigenfunction of the operator which is complete en ( t ) = It is the co-effecient of f n which basically tells us "how much of f n is contained within \JI" The expect ation value of any operator will be given by

Here Qn = Eigenvalue of the eigenfunction f n when operator Q acts Cn = Probability of the eigenfunction f n to occur To understand more about this and to study the complete derivation please refer to Introduction of Quantum Mechanics(2 Ed.) by David J. Griffith (page 108 - 110) The expression for average Energy is A

Here En = Energy of t he n -t h state Cn = Probability of the n-th state to occur [email protected]

P hysicsguide

53

@Sk J ahiruddin, 2020

Basic Formalism: Part-1

The average Energy is

(E ) =

L 3 5 - 9

2

Cn En 2

= 25 +

4 E1 + 5 - 16 x2 25

11

oo : 02 : 2a

(E) =

L 3

2

Cn En 2

-

E1

5 - 9

4

+ -

5 - 16x2

= 25 +

25

41

= -25 eV Sol 1.47. In problem number Prob 1.46. we learnt how to find the expectation value of an operator if the operator and t he probability is given. In t his problem we have to find the state which will give us the average energy 20 eV. Now if we look at t he given solutions we will see that t he wavefunction in option d will give us average energy of 20

eV . The average Energy is

2

1

J2 = 5 + 15

1

Eo+

J2

= 20 eV Sol 1.48. The wave function is w) j [email protected]

=

54

Physicsguide

@Sk J ahiruddin , 2020

Basic Formalism: Part-1

expectation value of t he Hamiltonian

H

The

"

= (w H w)

(w w) 1,T, 1

Eo

1 ,/..

,

I

Dl ,J.. \

11 oo : 02 : 30

j [email protected]

Physicsguide

54

@Sk J ahiruddin, 2020

Basic Formalism: Part-1

expectation value of t he Hamiltonian

-

2

j [email protected]

55

Physicsguide

11 oo : 02 : 33

j [email protected]

Physicsguide

54

@Sk J ahiruddin, 2020

Basic Formalism: Part-1

expectation value of t he Hamiltonian

-

2

j [email protected]

55

Physicsguide

11 oo : 02 : 36

j [email protected]

Physicsguide

54

@Sk J ahiruddin, 2020

Basic Formalism: Part-1

expectation value of t he Hamiltonian

-

2

j [email protected]

55

Physicsguide

11 oo : 02 : 39

j [email protected]

Physicsguide

54

@Sk J ahiruddin, 2020

Basic Formalism: Part-1

expectation value of t he Hamiltonian

-

2

j [email protected]

55

Physicsguide

11

j [email protected]

00:02:41

Physicsguide

54

@Sk J ahiruddin, 2020

Basic Formalism: Part-1

expectation value of t he Hamiltonian

-

2

j [email protected]

55

Physicsguide

11 oo : 02 : 44

j [email protected]

Physicsguide

54

@Sk J ahiruddin, 2020

Basic Formalism: Part-1

expectation value of t he Hamiltonian

-

2

j [email protected]

55

Physicsguide

11 oo : oo : 01

Problems and Solutions in Basic formalism of Quantum Mechanics: Part-2 Sk J ahiruddin * Souradeep Mondal

*Assistant Professor Sister Nibedita Govt. College, Kolkata Author was t he topper of IIT Bombay M.Sc P hysics 2009-2011 batch He ranked 007 in IIT JAM 2009 and 008 (JRF) in CSIR NET J une 2011 He has been teaching CSIR NET aspirants since 2012

1

@Sk J ahiruddin , 2020

Basic Formalism: Part-2

11 oo : oo : 04 1

@Sk J ahiruddin, 2020

Basic Formalism: Part-2

Contents 1

Problems from NET, GATE, JEST , TIFR & JAM pap ers 1.1 Hermitian Operators 1.2 Operators • • • 1.3 Time Evolution 1.4 Mixed Problems . 1.5 Ans Keys • 1.6 Solutions . •



j [email protected]















































3 5























8

























































































































2

3



14 21 22

Physicsguide

11 oo : oo : 06

j [email protected]

@Sk J ahiruddin , 2020

1

P hysicsguide

2

Basic Formalism: Part-2

Problems from NET, GATE, JEST, TIFR & JAM papers

1.1

Hermitian Operators

Prob 1.1. Which of t he following operators is Hermitian? [GATE 2016] d2 d3 d

(a) dx

(b) dx2

(d) dx3

Prob 1.2. Which one of the following operator is Hermit ian?

[GATE 2017] (c) exp (i Pxa)

Prob 1.3. The hermitian conjugat e of the operator

(a)

a

ax

a

--

8x

[JEST 2013]

(b) -

a

ax

(d) -i

a

ax

Prob 1.4. The adj oint of a differential operator d acting

dx

on a wavefunction W (x) for a quantum m echanical system is: [JEST 2016]

11 oo : oo : 09 Prob 1.4. The adjoint of a differential operator - acting dx on a wavefunction \JI (x) for a quantum mechanical system is: [JEST 2016]

[email protected]

Physicsguide

3

@Sk J ahiruddin , 2020

d (a) dx

Basic Formalism: Part-2

(b) -in d

dx

,...

d (c) - -d x

,...

Prob 1.5. A and B represent two physical characteristics of ,... a quantum sysyt em. If A is Hermitian, t hen for the product ,... ,... AB to be Hermit ian , it is sufficient t hat [GATE 2009] (a) B is Hermitian (b)

B is anti-Hermitian

(c)

B is Hermitian and A and B commute

(d) Bis Hermit ian and

A and B anti-commute

Prob 1.6. If A, B and C are non-zero Hermitian operators, which of the following relations must be false? [NET Dec 2013] (a)[A, B]=C (b)AB + BA= C (c)ABA = C

(d)A

+B=

C

Prob 1. 7. Consider a quantum mechanical syst em with three linear operators A , B and C , which are related by AB - C = d I where I is the unit operator. If A = - and B = x, then dx ,... C must be [TIFR 2013] d d (b) d (a) zero (c) -x dx (d) x dx dx A

A

A

A

/"',

A

A

A

A

A

11 oo : oo : 11 1near opera ors

, w 1c are re a e d I where I is the unit operator. If A= - and B = x, then dx C must be [TIFR 2013] A

,

an

A

(a) zero

A

A

d

(b) d dx

d

(d) x dx

(c) -x dx

[email protected]

Physicsguide

4

@Sk J ahiruddin, 2020

1.2

Basic Formalism: Part-2

Operators

Prob 1. 8. In a quantum mechanical system, an observable A is represented by an operator A. If 1/;) is a state of the system, but not an eigenstate of A , then t he quantity A

satisfies the relation (a) r < 0 (b) r = 0

[TIFR 2013] (d) r > 0

(c) r > 0

Prob 1. 9. The quantum mechanical operator for t he moment um of a particle moving in one dimension is given by

[GATE 2011] n2 d2 (d) 2mdx 2

d (b) -in dx

Prob 1.10. If a Hamiltonian H is given as

H =

IO)(0

- 1) (1I + i 0) (1 - i i1) (0

, where IO ) and I1) are orthonormal states, t he eigenvalues [JEST 2015] of H are (a)

±1

(b)

±i

(c)

±v'2

(d)

±iv'2

11 oo : oo : 14 H = IO) (0 - 1) (1I + i 0) (1 - i i1) (0 , where IO) and 11) are orthonormal states, t he eigenvalues of H are [JEST 2015]

(a) ±1

(b) ±i

(c) ±v'2

(d) ±iv'2

Prob 1.11. The Hamiltonian operator for a two-state system is given by j [email protected]

Physicsguide

5

@Sk J ahiruddin , 2020

Basic Formalism: Part-2

H = a( 1) (1 - 2) (2 + 1) (2 + 2) (1 ) where a is a positive number with t he dimension of energy. The energy eigenstates corresponding to t he larger and smaller eigenvalues respectively are: [JEST 2014] (a) 11) - (v'2 + 1) 2), 1) + (v'2- 1)12)

(b) 1) + (v'2 - 1) 2) , 1) - (v'2 + 1) 2) (c) 11) + (v'2-1) 2) , (J2 + 1) 1) -12) (d) 1)-(v'2 + 1) 2),(v'2-1) 1)+ 2) Prob 1.12. The state 'lj;) of a quantum mechanical syst em, in a certain basis, is represented by the column vector

'lj;) =

1/v'2 0 1/ v'2

A

The operator A corresponding to a dynamical variable A , is given, in the same basis, by t he matrix

11 oo : oo : 11

"

The operator A corresponding to a dynamical variable A , is given, in t he same basis, by t he matrix

A

A=

1 1 1 1 2 1

1 1 2 If, now, a measurement of t he variable A is made on the [email protected]

Physicsguide

6

@Sk J ahiruddin, 2020

Basic Formalism: Part-2

system in t he state 'l/;), the probability that t he result will [TIFR 2013] be + 1 is (a) 1/ v'2 (b) 1 (c) 1/ 2 (d) 1/4 Prob 1.13. Two different sets of orthogonal basis vectors

1 0 0 ' 1

and

1

1

1

1

y'2

1

' y'2

- 1

are given for a two dimensional real vector space. The mat rix representation of linear operator A in these bases are related by a unitary t ransformation. The unitary matrix may be chosen to be [NET June 2015] 0 -1 0 1 1 1 1 (a) 1 0 (b) 1 0 (c) v'2 1 - 1 A

(d) 1

v'2

1 0 1 1

11 oo : oo : 20 (a) 1

0

(b)

(c)

1 0

v'2

1 -1

1 0

(d) 1

v'2

1 1

Prob 1.14. The wavefunction of a particle in one-dimension is denoted by 'lj; (x) in the coordinate representation and ipx •

by cp(p) = J 'lj) (x)e n dx in t he momentum representat ion. If t he action of an operator T on 'ljJ (x) is given by T'lj)(x) = 'lj)(x + a) , where a is a constant t hen T 'lj) (p) is [NET June 2015] given by A

A

[email protected]

Physicsguide

7

@Sk J ahiruddin , 2020

Basic Formalism: Part-2





iap



(a) -

'l

napcp(p)

(b) e-

n (P)

i ap

(C) e +

n (p)

. 'l

(d) (1 + nap )cp(p) Prob 1.15. Let 'lj;1 )

1

0

represent

1 0 two possible states of a two-level quantum system . The state obtained by t he incoherent superposition of 'lj;1 ) and 'lj;2 ) is given by a density matrix t hat is defined as p c1 'l/J1) {'l/J1 I+c2 'l/J2) {'l/J2 . If c1 = 0.4 and c2 = 0. 6, the matrix element p 22 (rounded off to one decimal place) is [GATE 2019]

11 oo : oo : 23 element p22 ( rounded off to one decimal place) is 2019]

1.3

[GATE

Time Evolution

Prob 1.16. The wave function W of a quantum mechanical system described by a Hamiltonian H can be written as a linear combination of 'lj;1 and 'lj;2 which are the eigenfunctions of H with eigenvalues E 1 and E 2 respectively. At t =0, A

A

!

t he system is prepared in the state \¥ 0 = : 'lj;1 + 'lj;2 and t hen allowed to evolve wit h time. The wavefunction at t ime h T = (Ei _ E ) will be (accurate to within a phase) [TIFR 2 2 j [email protected]

Physicsguide

8

@Sk J ahiruddin, 2020

Basic Formalism: Part-2

(d) 'l/J2

Prob 1.17. Consider a spin-1/2 particle in the presence of homogeneous magnetic field of magnitude B along z-axis 1 which is prepared initially in a state 'lj;) = (I t) + --1-)) at 2 time t = 0. At what time t will t he particles be in the state - 'lj;) (µB is Bohr magnetron)? [JEST 2012]

(a) t

= -

1rn

µB E (d) Never

(b) t

21rn

= -

µBE

( c)

t

1rn

= --

2µBE

11 oo : oo : 24 - ?jJ) (µB is Bohr magnetron)? 2 (a) t = 1r n (b) t = 1rn µB E

µB E

[JEST 2012]

1rn

(c) t = 2µBE

(d) Never Prob 1.18. A spin-½ particle in a uniform external magnetic field has energy eigenstat es 1) and 2). The system

minimum energy difference between two levels is 2016] h

(b) h 4T

(a) 6T

[JEST

h (c) 2T

(d) h T [email protected]

@Sk J ahiruddin , 2020

9

Physicsguide

Basic Formalism: Part-2

Prob 1.19. The wave function of a part icle at time t = 0 1 is given by 1'11 (0) ) = v'2 (lu1) + u2)), where lu1) and u2) are the normalized eigenstates with eigenvalues E 1 and E 2 respectively., (E2 > El). The shortest t ime after which ?jJ (t)) will become orthogonal to '11 (0)) is [NET Dec 2011]

- n1r (a) 2(E2 - E1) (d)

2n1r E2 - E1

11 oo :oo :2a -li1r

(a) 2(E2 - E1 ) (d)

2/i1r E2 -E1

Prob 1.20. Let 7f;1 and 7f;2 denote the normalized eigenstates of a particle with energy eigenvalues E 1 and E 2 re-

spectively, with E2 > E 1. At time t prepared in a state

The shortest time T at which \JI (t to w(t = 0) is 2/i1r (a) (E2 - E1) 2/i1r

=

0 the particle is

= T ) will be orthogonal [NET Dec 2014] li1r

(c) 2(E2 - E 1)

(d)

Prob 1.21. A two-state quantum system has energy eigenj [email protected]

10

Physicsguide

@Sk J ahiruddin , 2020

Basic Formalism: Part-2

values ±c corresponding to t he normalized st ates 7/J±). At 1 t ime t = 0, t he system is in quantum state v'2 [ 7/J+) + 7/J- )]. The probability that the system will be in the same st ate at

t=

:E is ...... (up to two decimal places) .

[GATE 2018]

Prob 1.22. The Hamiltonian operator for a two-level quan-

t um system is H =

E1 0 0 E2

. If the state of the syst em 1

(

1 \

11 oo : oo : 30 Prob 1.22. T he Hamiltonian operator for a two-level quan-

E1 0 0 E2

t um system is H = at t

=

0 is given by 'lj;(O))

. If t he state of t he syst em

f2

=

1

1

t hen

at a later time t is 1 (a) ( + e - (E1 - E2)t/ li) 2

(c)

[GATE 2019]

1

(b) ½( 1 _ 1

I('lj;(O) l'l/J(t))

2

e-(E1- E2)t/li)

(1 + cos [(E 1

-

E 2 ) t/n])

2 1 (d) (1 - cos [(E1 - E 2 ) t / n]) 2 Prob 1.23. Consider a three-state system wit h energies E , E and E - 3g (where g is a constant) and respective eigenstates 1

1

1P2)

=

J6

1 -2

1

'lj;3)

=

J3

1 1

11

[email protected]

P hysicsguide

@Sk J ahiruddin , 2020

Basic Formalism: Part-2

If the system is initially(at t

=

0 ) , in st ate 1Pi)

=

1 0

0 what is t he probability t hat at a later time t , system will 0

be in st ate 'l/J1) =

[JEST 2014]

0 1 A

11 oo : oo : 33 what is the probability that at a later t ime t , system will 0 be in state 'l/J1) = 0 [JEST 2014] 1 4 3gt 3gt 2 (a) 0 (b) sin 2n 2n 9

4 .

(d) 9 Sln

2

E - 3gt

2n

Prob 1.24. The Hamiltonian of a two-level quantum sys1 1 1 . A possible initial state in tem is H = 2 1 - 1 which t he probability of the system being in t hat quant um stat e does not change wit h t ime, is [NET Dec 2017] cos 4 cos 8 (a) (b) sin 1r4 sin 1r

nw

1[

1[

8

(c)

cos !!:2 sin rr2

(d)

cos!!:6 sin 1r 6

Prob 1.25. The wavefunction of a particle in a one-dimensional

potential at time t = O is

[email protected]

12

@Sk J ahiruddin, 2020

P hysicsguide

Basic Formalism: Part-2

where w0 (x) and '11 1(x) are the ground and t he first excited states of t he particle wit h corresponding energies E 0 and E 1 . The wavefunction of t he particle at a time t is [GATE 2006]

(a) lr.::-e-i(Eo+E1)t/2h [2'llo(x)- '111 (x))

(b) ~ e-iEot/li,[2 '11o(x)-

11 oo : oo : 3s x an 1 x are t e groun an t e rst excite states of the particle with corresponding energies E 0 and E 1 . The wavefunction of t he particle at a time t is [GATE 2006] 0

W1 (x)] W1( X )e- iE1 t/2l'i] Prob 1.26. At t = 0, t he wavefunction of an otherwise free particle confined between two infinite walls at x = 0 and

x = Lis

w(x, t

=

0)

2 L

=

. 7r X . 31rx Sln L - sin L

Its wavefunction at a lat er time t

m L2

= - - is a1rn

2018]

(a) (b)

(c)

2

L 2

L

2 L

. 7rX . 31rx sin L - sin L

. 7rX sin L

.

+ sin

31rx

L



i1r

exp

-1,Jr

exp

L

j [email protected]

6 •

. 7rX . 31rx Sln - - Sln - - exp L

6 •

-1,Jr

8

13

Physicsguide

@Sk J ahiruddin, 2020

(d)

2

L

. 7rX sin L

[NET June

Basic Formalism: Part-2

.

+ sin

31rx

L



-1,Jr

exp

8

00: 00: 38 Basic Formalism: Part-2

2

L

(d)

1.4

. 7rX sin L

.

+ sin

31rx

L



-i1r

exp

8

Mixed Problems

Before you start attempting these problems I request you to read the Identical Particles notes first Prob 1.27. The wavefunctions of two identical particles in

states n and s are given by n(r1) and cp8 (r2), respectively. The particles obey Maxwell-Boltzmann statistics. The state of the combined two-particle system is expressed as [GATE 2006]

(a) n(r1)

+ s (r2)

[1i(r1)s(r2)

+ n(r2) s(r1)]

(c) J2[n(r1)s(r2)

n (r2) s(r1)]

(b)

12

(d) n(r1) s(r2) Prob 1.28. Two electrons are confined in a one dimensional box of length L. The one-electron state are given by

Wn ( x) =

2 L sin

n1rx

L

. What would be the ground state

wave function w(x 1 , x 2 ) if both electrons are arranged to have the same spin state? [JEST 2013]

[email protected]

@Sk J ahiruddin, 2020

14

Physicsguide

Basic Formalism: Part-2

11 oo : oo : 40

@Sk J ahiruddin , 2020

Basic Formalism: Part-2

1

2 . y'2 L sin

2 .

+ L sin



Sln



Sln

1rx2 L

(b) w(x1, X2) = 2 . 7rX1 y'2 L Sln L 1

(c) w(x1, X2) (d)



= L Sln

w (x1 , X2) =

-

Sln

2 .

2 .

7rX1

L

2 . 21rx1 L sin L

-

L

Sln



Sln

21rx2



Sln

L •

Sln

7rX2

L

Prob 1.29. Consider the wave function 1/; = 1/;(7 1 , 7 2)Xs for a fermionic system consisting of two spin-half particles. The spatial part of the wave function is given by

'l/;(71, 72) = }i[1(71)2(72) + 2(71) 1(72)] where 1 and 2 are single particle states. The spin part Xs of the wave function with spin states a(+ 1/2) and /3(- 1/ 2) should be [GATE 2012] (a) ~(a/3 + f3a) (b) }i(a/3- f3a) (c) aa (d) /3/3 Prob 1.30. The ground state and first excited state wave function of a one dimensional infinite potential well are 1/;1 [email protected]

@Sk J ahiruddin, 2020

15

Physicsguide

Basic Formalism: P art-2

11 oo : oo : 43 15

[email protected]

Physicsguide

@Sk J ahiruddin , 2020

Basic Formalism: Part-2

and 'lj;2 respectively. When two spin-up electrons are placed in t his potential which one of t he following wit h x 1 and x 2 denoting t he posit ion of t he two electrons correctly represents the space part of the ground state wave function of t he system? [GATE 2014]

(a) (b) (c) (d)

~['l/J1(x1)'l/J2(x1) - 'l/J1(x2)'l/J2(x2)] ~ ['l/J1(X1 )'l/J2(X2) + 'lp1(X2 )'l/J2(X1)] ~['lf;1(x1)'l/J2(x1) + 'l/J1(x2)'l/J2(x2)] ~ ['l/J1(x1)'l/J2(x2) - 'l/J1(x2)'l/J2(x1)]

Prob 1.31. Consider a system of two non-interacting ident ical fermions, each of mass m in an infinite square well potential of width a . (Take the potential inside the well to be zero and ignore spin) The composite wavefunction for t he system with total energy E =

2014]

2

(a) -

a

(b)

2 a

2

.

Sln

1I"X1

-

a

.

Sln

21rX2

.

(c) a sin

1rx1

a

sin

2a

2 (d) sin 1rx1 cos

1rx 2

a

a

a

j [email protected]



Sln

+ sin

31r x2

.

- sin -

sin

a

a



Sln

a

31rx1 2a

1rx 2

a 16

[NET June

is

- Sln

a

.

2ma

2



2 2 1 sin 1rx sin 1rx a

51r2n2

.

a 1rx2

Sln

a

cos 1rx1 a Physicsguide

11 oo : oo : 4s (d)

~ a



cos

Sln

j [email protected]



-

cos

Sln

Physicsguide

16

@Sk J ahiruddin, 2020

Basic Formalism: Part-2

Prob 1.32. The wave function of a free part icle in one dimension is given by w(x) = A sin x + B sin 3x. Then w(x) is an eigenstate of [JEST 2012] (a) t he position operator (b) the Hamiltonian (c) t he momentum operator (d) the parity operator Prob 1.33. For the parity operator P, which of t he following statements is NOT t rue? [GATE 2016] (a) p t = P (b) P 2 = - P (c) p t = I (d) p t= p-1

i

X

wavefunction in two dimensions? [JEST 2017] (a) kg m- 1 s- 2 (b) kg s- 2 (c) kg m 2 s- 2 (d) kg s- 1

Prob 1.35. It is required to construct t he quantum t heory of a particle of mass m moving in one-dimension x under the influence of a constant force F . The ch aract eristic lengthscale in this problem is [TIFR 2015] 1

(a)

mF h,2

(b)

n

mF

(c)

n m 2F

3

1 2

(d)

ri mF

3

Prob 1.36. Which one of the functions given below repred2 sents t he bound state eigenfunction of t he operator - dx 2

11 oo : oo : 49 m 2F

Prob 1.36. Which one of the functions given below repre-

sents the bound state eigenfunction of the operator [email protected]

2

P hysicsguide

17

@Sk J ahiruddin, 2020

!

2

Basic Formalism: Part-2

in t he region, 0 < x < oo , with t he eigenvalue -4? [GATE 2009] 2 2 (a) A0 e x (b)Aoe- x (c) Ao cosh 2x (d)Aosinh 2x Prob 1. 3 7. The wavefunction of particle moving in free space is given by W = e i kx + 2 e-ikx

The energy of the part icle is

5n2 k2 (a) 2m

3n2 k2 (b) 4m

n2 k2 (c) 2m

[GATE 2012] (d) ri,2k2 m

Prob 1.38. Suppose Hamiltonian of a conservative system in classical mechanics is H = wxp , where w is a constant

and x and p are the posit ion and moment um respectively. The corresponding Hamiltonian in quantum mechanics, in t he coordinate representation, is [NET D ec 2014] b 1 b 1 b (a)-inw(x x - ) (b)-inw(x x + ) (c)-i nwx x 2 2 6 6 6

(d)

_i':x8~

Prob 1.39. Consider a hypothetical world in which t he elec-

t r·on has spin ~ instead of ~. What will be the electronic configuration for an element wit h atomic number Z = 5? [JEST 2019] 4 1 2 2 1 1 1 5 3 (a) 1s , 2s (b) 1s , 2s , 2p (c) 1s (d) 1s , 2s ,2p

11 oo : oo : so tron has spin ~ instead of ~. What will be the electronic configuration for an element with atomic number Z = 5? [JEST 2019] (a) 1s 4 ,2s 1 (b) 1s 2 , 2s 2 , 2p1 (c) 1s 5 (d) 1s 3 , 2s 1 , 2p1 Prob 1.40. The wave function of which orbital is spherijahir@physicsguide. in

18

Physicsguide

@Sk J ahiruddin , 2020

Basic Formalism: P art-2

cally symmetric: (a) Px (b) Py

[GATE 2017]

(c) s

(d) Pxy

-

Prob 1.41. Consider the operator if= jJ- qA , where jJ is

t he momentum operator, A = (Ax, Ay, Az is the vector potential and q denotes the electric charge. If B = (Bx, By, B z)

denotes the magnetic field , the z-component of t he vector operator if x if is [NET Dec 2016] (a) iqnBz+ q(AxPy-AyPx) (b) -iqnBz- q(AxPy-AyPx) (c) -iqnBz (d) iqnBz Prob 1.42. 1000 neutral spinless particles are confined in

a one-dimensional box of length 100nm. At a given instant of time, if 100 of these particle have energy 4E and the remaining 900 have energy 225E t hen the number of part icles in t he left half of t he box will be approximately [TIFR 2015] (a) 441 (b) 100 (c) 500 (d) 625 Prob 1.43. If the root-mean-squared momentum of a part icle in the ground state of a one-dimensional simple har-

11 oo : oo : s4 (a) 441

(b) 100

(c) 500

(d) 625

Prob 1.43. If the root-mean-squared momentum of a part icle in the ground state of a one-dimensional simple har-

monic potential is p 0 , then its excited state is June 2017]

(a) Pov'2

(b) Pov'3

j [email protected]

(c) Po 19

@Sk J ahiruddin, 2020

2/3

(d) Po

[NET 3/2

Physicsguide

Basic Formalism: Part-2

Prob 1.44. Suppose the spin degree of freedom of two part icles (nonzero rest mass and nonzero spin) is described com-

pletely by a Hilbert Space of dimension twenty one. Which of t he following could be the spin of one of t he particles? [JEST 2018]

(a) 2

(b) 3 2

(c) 1

11 oo : oo : s6

[email protected]

20

@Sk J ahiruddin , 2020

1.5

Physicsguide

Basic Formalism: Part-2

Ans Keys

1.1. b

1.16.

C

1.31. a

1.2. a

1.17. a

1.32. d

1.3. a

1.18.

1.33. b

1.4.

C

1.19. b

1.5.

C

1.20. b

C

1.6. a

1.21. 0.25

1. 7. d

1.22.

1.34. d 1.35. d 1.36. b

C

1.37.

C

1.23. b

1.8. a

1.38. b 1.9. b 1.10.

1.24. d C

1.11. b 1.12. d

1.25. d 1.26. b 1.27. d

1 .39. a 1.40.

C

1.41. d

11 oo : oo : sg 1.9. b 1.10.

1.24. d 1.39. a

1.25. d

C

1.11. b

1.40.

1.26. b

C

1.12. d

1.27. d

1.41. d

1.13.

C

1.28. b

1.42.

1.14.

C

1.29. b

1.43. b

1.30. d

1.44. a

1.15. 0.6

21

j [email protected]

P hysicsguide

@Sk J ahiruddin, 2020

1.6

C

Basic Formalism: Part-2

Solutions

Sol 1.1. The definit ion of Hermit ian operator is 00

00

w;Aw1 dT =

(Aw2)*w1 dT

-oo

-oo

Now let us check for opt ion a 00

oc

- oo

w*d'111 d 2

- oo

dx

= [w*w ]00 2 1 -

(X)

x

00 dw; w d d l X

- oo

X

00

- oo

As Quantum mechanical wave function must be well behaved so the first part of integration is equal to zero.

11 oo : 01

: 01

00

- oo

As Quantum mechanical wave function must be well behaved so the first part of integration is equal to zero. 00

- 00

[email protected]

22

Physicsguide

@Sk J ahiruddin, 2020

So t he operator option b.

Basic Formalism: Part-2

d! is not Hermitian. Now we will check for

00

w*d2'111 dx 2 dx2 -oo oo

-oo

w*d'111 2 dx

oo - (X)

00

_ 00

d'11 2d'll 1 dx dx dx

00

w*dw 1 2 dx

- (X)

00

w*d'111 2 dx

+

- oo

00

- oo

d2 . H .. Thus t he operator dx 2 1s erm1t1an .

11 oo : 01

: 04

X

X

- oo

- oo

CX)

- (X)

.. Thus t h e operator dx 2 1s erm1t1an. There is a second and fast method to check if an operator is hermitian. An operator is hermitian if d2

H

.

We must be careful while using this definition as always t his definit ion will not give us a definit ive answer. For t his particular sum this definit ion will not be beneficial to us. Sol 1.2. Refer to problem Prob 1.1. ,.. ,.

PxX2 - X2Px

,.

23

[email protected]

Physicsguide

Basic Formalism: Part-2

@Sk J ahiruddin, 2020

A_t = - i((PxX2)t - (x2px)t) 2 2 2 . (X Px - PxX ) = -i- - - - 2 2 2 . (PxX - X Px) =i----2 A

=A A

Sol 1.3. The operator A is

a

ax

11 oo : 01

: 06 2

"

=A

a

"

Sol 1.3. The operator A is

ax

00 00

-00

w* - aw1 dx 2

ax

- 00

00

=-

,T,* ,Tr ] 00

[ 'J:' 2 'J:' 1 -oo

Thus the hermit ian conjugate is

+

aw5 d 8x '±' 1 X ,Tr

-oo

a

ax

Sol 1.4. This sum will be solved exactly as t he previous

sum is solved.

j [email protected]

@Sk J ahiruddin, 2020

"

The operator A is 00

Physicsguide

24

Basic Formalism: Part-2

d

dx 00

dx -oo

-oo

00

- oo

11 oo : 01

: 09

CX)

-

CX)

d dx

Thus the hermitian conjugate is Sol 1.5. Operator

(A.B)t

=

(A.B)T =

A is Hermitian i.e. (.A)t = A _BTAT = _BT AT = _Bt At = BA = AB A

A

Thus we see that for the product of AB to be Hermitian B must be Hermitian and A and .B must commute (i.e.

AB = BA) Sol 1.6. We have shown in the previous problem that AB

operator is hermitian then the two operators must commute. That is [A, BJ = 0. But C i- 0. So option a is incorrect.

25

[email protected]

Physicsguide

@Sk J ahiruddin , 2020

Basic Formalism: Part-2

A

A

A

Sol 1. 7. Let the operator AB - C act on any state f.

d -dxx - C

(AB - C)f =

A

f

d dx (x f) - C(f) A

A

A

A

A

d = (f + x dx (f) - C (f)) A

A

For this AB - C = I equation to hold true C must be equal ,J

11 oo : 01 AB-C

f =

-dxx - C

: 11 f

d dx (xf) - C(f) A

A

A

A

A

A

For this AB - C = I equation to hold true C must be equal d to x dx. Sol 1.8. Basic theory Sol 1.9. The form of the Hamiltonian is given but if we need to find t he eigenvalues of t he Hamiltonian then we need to form the matrix form of the operator . Here we would learn how to form the matrix form of any operator. We will use the eigenstate of the operator as a base ket and will represent the operator as a square matrix. (a(l )

A a (1))

(a(l )

• • •

A a(2))

·••

• • •







Please refer to Modern Quantum Mechanics by J .J. Sakurai (page 20) Now we return to the problem at hand. 26

[email protected]

Physicsguide

@Sk J ahiruddin, 2020

H O)

=

Basic Formalism: Part-2

0) - i 1) and H I)

=

i IO) - 1)

Now the matrix form of t he Hamiltonian

(0 H 0) (0 H 1)

H=

( 11HIO\

(11H l1\

1



1,

-i - 1

11 oo : 01

: 1s

0) - i I1) and H l) = i IO) - 1) Now t he matrix form of t he Hamiltonian H O)

=

(0 H 0) (0 H 1)

H=



1

(1 H 0) (1 H 1)

1,

-i - 1

The characteristic equation •

1,

-i

=0

- 1- A

[-(1 - A)(l + A) - (ix -i)] = 0 [- 1 + A

2

2

A

1] = 0

-

-

2= 0

A=

±v12

Sol 1.10. Here in this problem we know t he form of t he

hamiltonian but we have no information about its eigenstates. Now at first we must know how the hamiltonian acts on a state H 1) = a( 1)(1 - 12)(21+ 1)(21+ 2)(1 ) 1) = a( 1) + 2)) H 2) = a(l l )( l j [email protected]

@Sk J ahiruddin, 2020

2)(2 + 1)(2 + 2)(1 )12) = a( 1) - 2)) 27

Physicsguide

Basic Formalism: Part-2

For this sum we have to individually check which of t he option is an eigenstate of t he Hamiltonian. We have found t hat option b is t he answer. Here we will work out t he

11 oo : 01

: 16

@Sk J ahiruddin , 2020

Basic Formalism: Part-2

For this sum we have to individually check which of t he opt ion is an eigenstate of t he Hamiltonian. We have found t hat option b is the answer. Here we will work out t he sum with option b and show you how to solve t his type of problems. H [ 1 )+( ✓2 - 1) 2)]

=a( ll ) + 2)) + a( ✓2 - 1)(11) - 12)) =a( l + ✓2 - 1) 1) + a( l - ✓2 + 1)12) =a ✓2 [ 1) + ( ✓2 - 1) 2)]

Again

H [ 1 )-(✓2+ 1) 2)] =a(l)+ 2))-a( ✓2 + 1) ( 1 )- 2))

V2 - 1) 1) + a(l + V2 + 1)12) ah [11) - (h + 1) 2)]

= a( l -

=Sol 1.11. Sol 1.12.

Sol 1.13. One set of basis is the unit vectors. Hence t he problem becomes quite easy. It operator will be t he second

set of basis written each vector column-wise.

[email protected]

@Sk J ahiruddin, 2020

28

Physicsguide

Basic Formalism: Part-2

Please see the Linear Algebra notes under Mathematical

11 oo : 01

: 20

@Sk J ahiruddin , 2020

Basic Formalism: Part-2

Please see the Linear Algebra notes under Mathematical Physics. The change of basis and similarity transformation are discussed well. Sol 1.14. This is translation operator. Discussed in Generators of Transformation in Basic postulates of quantum mechanics notes. Sol 1.15. From the definition of the matrix element of Op-

erator Aij

= ('l/Ji A I'lp_j )

P22

=

('l/J2 P lrf2)

Now substituite

And calculate

Sol 1.16. 'lj;1 and 'l/J2 are the eigenfunctions of Hamiltonian H. This means rf 1 and 'lj;2 are stationary states. Now t he A

time developement of a stationary state is given by

n

j [email protected]

@Sk J ahiruddin , 2020

29

Physicsguide

Basic Formalism: Part-2

11 oo : 01

: 22

29

[email protected]

@Sk J ahiruddin , 2020

P hysicsguide

Basic Formalism: Part-2

Now

= exp

-iE 1t ri

4 1P1 5

+

3 1P2 exp 5

-iE1 h = exp - - x - - - ri 2(E1 - E2) 4n /, 3n/, -i(E2-E1) h - > ml 2 ) is approximately 2 Dec 2015]

n, 2Vo ( a ) vo+-TT

L m

(b)

Vo+~ Vo Lm

(c)

Vo+

[NET

n Vo

4Lm

n Vo

(d)Vo+ 2L m

Prob 1.4. A part icle of mass m moves in one dimension

under the influence of t he potential V (x) = -ab (x), where a is a posit ive constant. The uncertainty in the product (~ x)(~p) in its ground state is [NET June 2016] (a) 2n (b) n/2 (c) n/-/2 (d) -/2n Prob 1.5. In a one-dimensional system, the boundary con-

dition that t he derivative of the wavefunction 'l/J' (x) should be continuous at every point is applicable whenever [TIFR 2019] (a) t he wavefunction 'ljJ (x) is itself continuous everywhere. (b) t here is a bound state and t he potential is piecewise continuous. (c) t here is a bound state and t he potential has no singularity anywhere. (d) there are bound or scattering states with definite momentum.

11 oo : oo : 11 t inuous. (c) t here is a bound st ate and the potent ial has no singularity anywhere. (d) there are bound or scattering states with definit e momentum.

[email protected]

Physicsguide

4

@Sk J ahiruddin, 2020

1.1

lD Potentials: Part-1

Free Particle in 1D Box

Prob 1.6. A free particle is moving in +x direction with a linear momentum p. The wavefunction of the particle normalized in a length Lis [GATE 2006] •

(b)

l

p

vfL cos nx

(c)

1 -

ipx

v!Le n



i px

Prob 1. 7. The wave function \JI n ( x) of a particle confined t o a one-dimensional box of length L with rigid walls is given by Wn(x)=

j; sin

n~x , n= l , 2, 3, ...

[JAM 2008]

i) Determine t he energy eigenvalues. Also, determine t he eigenvalues and the eigenfunctions of t he momentum operator. ii) Show that the energy eigenfunctions are not t he eigenfunctions of the momentum op erator. [This Qs contains many important concepts. Read infi-

11 oo : oo : 13 ator. ii) Show that t he energy eigenfunctions are not t he eigenfunctions of the momentum operator. [This Qs contains many important concept s. Read infinite p otent ial well from Arthur Beiser first , then from David Griffiths, then think of the problem again]

j [email protected]

Physicsguide

5

@Sk J ahiruddin , 2020

1D Potent ials: Part-1

Prob 1. 8. A particle of mass m is confined in a one-dimensional box of unit length. At time t = 0 t he wavefunction of the

particle is Wx,o = A sin(21rx) cos(1rx), where A is t he nor[JAM malization constant . 2009]

i) Write the wavefunction W(x ,t ) at a later t ime t. ii) Find t he expectation values of momentum and energy at t = 0. Prob 1. 9. A particle of mass m is confined in the ground

state of a one-dimensional box, extending from x = -21 to x = +21. The wavefunction of t he part icle in t his state is 7r X \ll (x) = Wo cos L where \ll 0 is a constant. [GATE 4 2007] i) The normalization factor WO of t his wavefunction is

(a)

(b)

1



(c)

1 2£

(d)

11 oo : oo : 16 2007]

i) The normalization factor WO of t his wavefunction is

(a)

(b)

1 4L

1 2L

(c)

(d)

ii) The energy eigenvalue corresponding to this state is n,27r2

(a) 2mL2

n,2 7r2

n,27r2

(b) 4mL 2

(c) 16mL 2

n,27r2

(d) 32mL 2

iii) The expect ation value of p 2 (p is the momentum opera-

[email protected]

6

@Sk J ahiruddin, 2020

P hysicsguide

lD Potentials: Part-1

tor) in this state is n,27r2

(a) 0

(b) 32£2

n,27r2

(c) 16£2

Prob 1.10. A part icle is confined inside a one-dimensional

box of length l and left undisturbed for a long time. In the most general case, its wave-function MUST be, [TIFR 2018] (a) t he ground st at e of energy. (b) p eriodic, where l equals an integer number of periods. (c) a linear superposition of the energy eigenfunctions. (d) any one of the energy eigenfunctions. Prob 1.11. Consider a syst em of eight non-interacting, iden-

t ical quantum particles of spin - ; in a one dimensional box of length L. The minimum excitation energy of the system , 7r2n,2

in units of m L 2 is __ [GATE 2015] 2 Here before solving sums on this topic we must bring to

11 oo :oo :1a •



v .1..1. u l

Y

uv.1..1..1.

lh.._....

-1

u~i u.v

1--.., l 1

t ical quantum particles of spin -~ in a one dimensional box of length L. The minimum excitation energy of the system, 7r2fi2

in units of m L 2 is __ [GATE 2015] 2 Here before solving sums on this topic we must bring to your attention one simple mistake which we have observed many students make. What is t he difference between probability density and probability? To find probability we have to integrat e t he square of a wave function within a range.

Prob 1.12. A particle is placed in a one dimensional box [email protected]

@Sk J ahiruddin, 2020

7

P hysicsguide

ID Potent ials: Part-1

of size L along t he x-axis (0 < x < L). Which one of the following is t rue? [GATE 2008] (a) In t he ground state, the probability of finding the L 3L particle in the interval is half 4' 4 (b) In t he first excited state, the probability of finding the L 3L particle in the interval is half. This also holds for 4' 4 states with n=4, 6, 8, ..... (c) For any arbitrary st at e Iw), the probability of finding t he particle in the left of the well is half (d) In the ground state, the particle has a definite moment um

Prob 1.13. Given t he wave function of a particle 1/J(x) 1 : sin ( :

x) for O < x < L and Oelsewhere t he probability

11 oo : oo : 21 (d) In the ground state, the particle has a definite moment um Prob 1.13. Given t he wave function of a part icle 'l/J(x) = 2 7r L sin L x for O < x < L and O elsewhere t he probability

of finding the particle b etween x = 0 and x = L / 2 is __ (Round off t o 1 decimal places) [JAM 2019] Prob 1.14. A part icle is in t he ground st ate of an infinite square well potent ial given by, 0 for -a< x < a V (x ) = oo otherwise The probability t o find the particle in the interval between

[NET Dec 2013]

- ; and ; is j [email protected]

P hysicsguide

8

@Sk J ahiruddin, 2020

1 (a) 2

1

(b) 2

1D Potent ials: Part-1

1

+-

1

1

2

7r

(c) - - -

7r

Prob 1.15. A part icle moving in one dimension is confined

inside a rigid box located between x

~ a and x

=

= ;.

If

2 1rX t he particle is in its ground st at e wO( x) = - cos - the a a quant um mechanical probability of its having a moment um

p is given by

,

-- ....

~

[TIFR 2016]

rri,

1•

1



("

I



,

.

,

11 oo : oo : 24

Prob 1.16. The normalised eigenfunctions and eigenvalues of t he Hamiltonian of a p article confined to move between 0

< x < a is one dimension are 2

n1rx

a

a

'l/Jri(x) = - sin - and n21r2 n 2

En = - - 2ma2 respectively. Here 1, 2, 3, · · · . Suppose the state of the p art icle is 7rX 7rX 'lf;(x) = A sin 1 + cos a a [email protected]

Physicsguide

9

@Sk J ahiruddin , 2020

lD Potentials: Part-1

where A is the normalisation constant. If the energy of the particle is measured , the probability to get as 7r2fi2 X • - - lS 2ma2 100

What is t he value of x?

[JEST 2018]

Prob 1.1 7. A particle of mass m is in a one dimension al potential 0 for O < x < L V (x ) = 00 otherwise At some inst ant its wave function is given by

11 oo : oo : 21 Prob 1.1 7. A particle of m ass m is in a one dimensional potential 0 for O < x < L V (x) = 00 ot herwise At some inst ant its wave function is given by

'lp (X)

1

= J3 'lp1(X) + i

where 'lj;1 ( x) and 'lj;2 are t he ground and t he first excited states, respectively. Identify the correct statement. [JAM 2018] L n2 31r2 (a) < x >= 2; < E >= 2m £2

(b) < x >=

2L

3

; < E >=

L

n,2 w2

2m L 2 n 2 81r2

(c) < x >= -· < E >= - 2

2' 2m L 2 2 2L n 4 1r (d) < x >= 3 ; < E >= 2m3L 2

j [email protected]

10

P hysicsguide

@Sk J ahiruddin , 2020

1D Potent ials: Part-1

Prob 1.18. A particle in t he infinite square well V (x) 0 O 0

as shown in the figure

j [email protected]

16

Physicsguide

11 oo : oo : 44

j [email protected]

Physicsguide

16

@Sk J ahiruddin , 2020

1D Potentials: Part-1

V(x)

Vo i - - - - - -

E 0

X

For E < Vo , the space part of the wavefunction for x > 0 is of the form. Where a is a real positive quantity. [GATE 2011]

(b)

e - ax

(d)

e - iax

Prob 1.31. A particle of mass m and energy E moving in t he positive x direction, encounters a one dimensional po-

tent ial barrier at x = 0. The barrier is defined by 2005]

[JAM

= 0 for x < 0 V = Vo for x > 0 (Vo is positive and E > V0 ) V

If t he wave function of t he particle in t he region x < 0 is given as Aeikx + B e - ikx (a) Find the ratio B/ A (b) If B / A = 0.4, find E /Vo , and t he transmission coefficients.

11 oo:oo:47 given as

A eikx

+ B e - ikx

(a) Find the ratio B/ A (b) If B / A = 0 .4, find E / Vo , and the transmission coefficients. [email protected]

P hysicsguide

17

@Sk J ahiruddin, 2020

lD Potentials: Part-1

Prob 1.32. A particle of mass m and energy E, moving in t he posit ion x direction, is incident on a step potential at

x= O, as indicated in the figure. The height of the potential is V0 , where V0 > E . At x = x 0 , where x 0 > 0, the probability of finding the electron is ¼ times t he probability of finding 2m V0 -E it at x= O. If a = 2016] V(x) '

E -~

X=0

2

(a) -

O'.

1

(c) 2a

X = Xo

(d) 1 40'.

Prob 1.33. A two-dimensional square rigid box of side L

contains six non-interacting electrons at T = OJ( . The mass of the electron is m. What is the ground state energy of the n,27r2

system of electrons, in units of m L 2 ? 2

[GATE 2016]

11 oo : oo : so contains six non-interacting electrons at T = OK. The mass of t he electron is m. What is the ground state energy of the fi27r2

system of electrons, in units of m L 2 ? 2

[GATE 2016]

Prob 1.34. A particle of energy E moves in one dimension under t he influence of a potential V(x). If E > V(x) for [email protected]

18

Physicsguide

lD Potentials: P art-1

@Sk J ahiruddin, 2020

some range of x, which of the following graphs can represent a bound state wave function of the particle? [TIFR 2013] 'I'(x) 'lfx)

X

'¥(x)

X

X

Prob 1.35. A beam of identical particles of mass m and energy E is incident from left on a potential barrier of width L (between O < x < L ) and height Vo as shown in the figure (E < Vo) For x > L , t here is tunneling with a transmission coefficient T > 0. Let Ao , AR and AT denote the amplit udes for the incident, reflected and t he t ransmitted were, respectively. [GATE 2008]

11 oo : oo : s3 0

L , t here is t unneling wit h a transmission

coefficient T > 0. Let Ao, A R and A r denote t he amplit udes for the incident, reflected and t he t r ansmitted were, respectively. [GATE 2008] i) Throughout O < x < L , the wave-function (a) Can be chosen to be real (b) is exponent ially de-

j [email protected]

19

P hysicsguide

@Sk J ahiruddin, 2020

1D Potentials: Part-1

"

V(x)

·-----------

------·

E .

(c) is generally complex

(d) is zero

ii) Let the probability current associat ed with the incident wave be S0 . Let R be the reflection coefficient. Then (a) t he probability current vanished in the classically forbidden region (b) t he probability current is T S 0 for x > L (c) for , x < 0, t he probability current is S 0 (1 + R) (d) for ,x > L , the probability current is complex iii) The ratio of the reflected to t he incident amplitude A R/ A 0 ,• c

11 oo : oo : 55 0

(c) for , x < 0, the probability current is S 0 (1 + R) (d) for ,x > L , the probability current is complex iii) The ratio of the reflected to the incident amplitude AR/ A 0 •

18

(b) ✓( 1 - T ) in mag-

(a) 1 - Ar/Ao nitude (c) a real negative number

(d)

IAT )2 E Ao Vo- E

(1

Prob 1.36. Consider a potential barrier V (x) of t he form: [email protected]

Physicsguide

20

@Sk J ahiruddin , 2020

lD Potentials: Part-1

V(x)

V(x)

= oo

t-------.Vo

x=O

x=a

where Vo is a constant. For particles of energy E < Vo incident on this barrier from the left , which of the following schematic diagrams best represents t he probability density 'l/J (x) 2 as a function of x. [GATE 2019]

(B)

(A)

x= O

x=a

x= b

X

x= O

I tJJ(x)l 2

x=a

x= b

X

11 oo : oo : sa (B)

(A)

x=O

1I/JCx)l

" '

"

x=a

2

x=O

X

lt/J(x)l 2

(C)

x=a

X

(D)

A

1""x=O

x=a

x= b

x=O

X

x= a

x= b

X

Prob 1.37. A free particle of energy E collides with a one-

dimensional square potential barrier of height V and width W. Which one of the following statement (s) is / are correct? j [email protected]

@Sk J ahiruddin , 2020

21

P hysicsguide

1D Potent ials: Part-I

[JAM 2016] (a) For E > V , the t ransmission coefficient for the part icle across the barrier will always be unity (b) For E < V , the transmission coefficient changes more rapidly with W t han with V (c) For E < V , if V is doubled, t he transmission coefficient will also be doubled (d) Sum of t he reflection and the t ransmission coefficients is always one Prob 1. 38. Consider a potential barrier A of height

Vo

and width b, and another potential barrier B of height 2V0 and t he same width b. The ratio TA/T B of tunneling probabilit ies TA and T B, t hrough barriers A and B respectively, for a particle of energy Vo/ l 00, is best approximated by [NET

11 oo : 01 : oo Prob 1.38. Consider a potential barrier A of height

Vo

and width b, and another potential barrier B of height 2V0 and t he same width b. The ratio TA / T B of tunneling probabilities T A and TB, t hrough barriers A and B respectively, for a particle of energy Vo/100, is best approximated by [NET June 2017] (a) exp [(vT.99 - Jo.§9) ✓8mV0 b 2 /n2 ]

(b) exp [(J"f98 -vif§8) ✓8mVob 2 /n2 ] (c) exp[(~ - Jo.§9) ✓8mV0 b2 /n2 ] (d) exp [(v'2.§8 - vif§B) ✓8mV0 b 2 / n2 ] Prob 1.39. A free electron of energy l eV is incident upon a

one-dimensional finite potential step of height 0.75eV. The [email protected]

Physicsguide

22

lD Potent ials: Part-1

@Sk J ahiruddin, 2020

probaility of its reflection from t he barrier is __ (up to two [GATE 2017] decimal places)

1.4

Finite Potential Well

Prob 1.40. There are only t hree bound states for a particle of mass m in a one- dimensional potential well of t he

form shown in the figure. The dept h Vo of the potential satisfies [GATE 2007]

-a / 2

V

+a / 2 y

11 oo : 01

: 03 [GATE 2007]

satisfies



-a / 2

V

+a / 2

-----+- ~ - -; - - - + .-

--.....---·····-·........... _

X

~

0

Prob 1.41. A particle moving in one dimension, is placed in an asymmetric square well pote11t ial V (x) as sketched below.

[email protected]

23

Physicsguide

@Sk J ahiruddin, 2020

lD Potentials: Part-1

-a

V(x) +a

X

The probability density p( x) in t he ground state will most closely resemble [TIFR 2019]

(a) p

(c)

p

11 oo : 01 : os

p

(a)

(c)

0'====:...___,!..._ _ ____..::...X

Ob=c------=~--"------=x -a +a

+a

-a

(d)

(b) p

0 1::::::::::...._ _L___

p

p

----=::::::~X

+a

-a

+a

-a

Prob 1.42. Consider a square well of depth

a with Voa fixed. Let

Vo -+

well has (a) No bound states (c) 2 bound states

and width

oo and O -+ 0. This potential [JEST 2014]

(b) 1 bound state (d) In finitely rnany bound states

j [email protected]

24

Physicsguide

@Sk J ahiruddin , 2020

1.5

-Vo

1D Potentials: Part-1

Ans Keys

1.1. b

1.5. c

1.2. a 1.6. d 1.3. d 1.4.

1. 7. do yourself

C

1.8. sin(1rx) exp (-iE 1t/n)+sin(31rx) exp (-i E 3t/n) wh ere

n / 2m,

2 2 2

E,,., = n 1r

0 and 51r

n / 2m

2 2

11 oo :01 : oa 1.3. d 1.4.

1. 7. do yourself

C

1.8. sin(1rx) exp (-i E1t/ ti)+sin(31rx) exp ( -iE3t/ti) where 2 2 2

2 2

En = n 1r ti / 2m, 0 and 51r ti / 2m gn,27r2

1.9. c,d ,c

1 • 20 •

1.10.

C

1.21. a

1.11. 5

1.22. d

1.12. a

1.23. b

1.13. 0.50

1.24. a

1.14. b

1.25. b

1.15. a

1.26. a or b

1.16. 80

1.27. b

1.17. a

1.28. b,c

1.18.

C

1.29. d

1.19. d

1.30. b

[email protected]

2mL 2

25

Physicsguide

@Sk J ahiruddin, 2020

1.32.

C

1.33. 24

lD Potentials: Part-1

1.38. a

1. 3 9. 0 .10 to O.12

1.34. a 1.40. a 1.35. b,b,b

00 : 01 : 10 1.32.

1.38. a

C

1.33. 24

1.39. 0.10 to 0.12

1.34. a 1.40. a 1.35. b ,b ,b 1.36. a

1.41. a

1.37. b ,d

1.42. b

[email protected]

26

lD Potentials: P art-1

@Sk J ahirudd in , 2020

1.6

P hysicsguide

Solutions

1.1. Here the potential is symmetrical. We know that for

a symmetric (even) potential the eigenfunctions of t he one dimensional Schrodinger equation can always be chosen to ,

,

rl

• '

'

,

.

,

,

11 oo : 01 1.6

: 13

Solutions

1.1. Here the potential is symmetrical. We know that for

a symmetric (even) potential the eigenfunctions of t he one dimensional Schrodinger equation can always be chosen to have definite parity i.e. t he eigenfunctions can be chosen to be odd or even. We also know t hat int egration of odd function over a complete interval is equal to zero. Thus we have

('1/Jo x 'l/Jo) = 0

and

The expectation value (x) of t he position operator x in the stat e 'ljJ) is

(x)

=

+ a1 ('lj; )x(ao 'l/Jo) + a 1 'lj;)) = aoa1[('1/Jo x '1/J1) + ('1/J1 x 'l/Jo)]

(\Jll x \JI ) = (ao('l/Jo

11

111

-VO - - -

1.2. To solve this problem we have to closely follow t he

solutions of a particle in a one-dimensional square well. j [email protected]

@Sk J ahiruddin, 2020

27

P hysicsguide

1D Potentials: Part-1

For bound stat e the range of energy must be - Vo < E < 0 The wavefunctions of the different regions are as follows.

11 oo : 01

: 16

@Sk J ahiruddin , 2020

lD Potentials: Part-1

For bound stat e the range of energy must be - Vo < E < 0 The wavefunctions of the different regions are as follows. and and

'11 2 = A sin kx + B cos kx

W3=Cexp(-ax)

Here a=

2ml E I n,2

and

2m(Vo + E )

k=

n,2

Please go through example 4.13 in page 270 in t he book Quant um Mechanics written by Nouredine Zettili(Second Edition) . The lowest value Vo giving n bound states is given by

Vo =

7r2ri2

8ma 2

2

(2n - 1)

where n is t eh number of bound st at es. Now for n = 1, we get

Vo=

7r2ri2

8ma2

[email protected]

28

@Sk J ahiruddin , 2020

1.3. We have to expand the potential

Physicsguide

lD Potentials: Part-1

11 oo : 01

: 1a

@Sk J ahiruddin , 2020

lD Potentials: Part-1

1.3. We have to expand t he potential X

Vo cosh L = Vo 2

Vo 2

Vo 2

x

[exp(x/ L ) + exp(-x/ L)]

+

Vo 2 1

= Vo + 2

+ Vo

Vo x 2 L

£2

Vo

2

x l + L + 2! L l

+ ... + 2

x

l -

l

L + 2!

X

L

2

+ .. .

2

x2

This potential looks like t hat of Simple harmonic motion. Now we can say k

= Vo

£2

and

w=

Vo m£ 2

So,the ground state energy is

Vo - Vr Ii m £ 2 - o + 2£

Vo m

1.4. At first you need to study Delta Function Well from t he 1D Potential notes or any books like Griffit hs or Zettili The X < ( foexp(ax) wave funt ion is written as W(x) = X > ( foexp(-ax) As the wavefunction is even about x = 0 so we can easily depict t hat j [email protected]

@Sk J ahiruddin , 2020

29

Physicsguide

lD Potentials: Part-1

11 oo : 01 [email protected]

: 21

29

P hysicsguide

lD Potent ials: Part-1

@Sk J ahiruddin , 2020

X

(x) = 0

(p) = 0

and

Now 00

2

x exp(-2ax) dx =

0

1 2a 2

Thus we have

~x =

✓ (x2) - (x)2 =

_ 1_

v12a

Before calculating the uncertainty of momentum please go t hrough the remark section in page 510 in t he book Quant um Mechanics written by Nouredine Zettili(Second Edit ion). In t he problems where the first derivative of the wave funct ion is discontinuous at a given value of x, one has to be careful while calculating momentum or one would get neg-

[email protected]

30

Physicsguide

11 oo : 01

: 24

careful while calculating momentum or one would get neg-

j [email protected]

Physicsguide

30

@Sk J ahiruddin , 2020

1D Potentials: Part-1

ative momentum. We have to use t he following expression

- 00

Thus the uncertainty in momentun

Thus we have !J.x/J.p

l

= -- X

v'2a

ria

= -

Ii

v'2

1.5. There are many discussions on when the derivative of wave function becomes discont inuous. Physically, when the wave function has a boundary condition involving an infinite potential, there is a discontinuity in t he derivative because t he t he wave function must immediately vanish at

t he boundary (because you will never find t he particle in t he potent ial), and it may not do so smoothly, usually creating a corner. For a finite potent ial, the particle may be found inside it, even if it's not very likely - so t he wave funct ion is not required to immediately drop to zero and instead smoothly transit ions to an exponential. For mathematical argument, please follow t he derivation of Dirac Delta potential well. At that problem the derivative of 1/J(x) is discont inuous.

11 oo : 01

: 26

smoothly t ransit ions t o an exponential. For mat hematical argument, please follow the derivat ion of Dirac Delta potent ial well. At that problem the derivative of 'ljJ (x) is discontinuous. 31

[email protected]

P hysicsguide

@Sk J ahiruddin , 2020

lD Potentials: Part-1

discussion in Arthur Beiser Concepts of Modern P hysics, Chapter 4. 1. 7. This is basic t heory. P lease read t he Discussions from

Beiser and Griffiths and One dimension pot ent ial not es form t he Quantum mechanics not es t hen come in problem solv1ng. •

1.8. Now the wave function is Wx,o = A sin 21rx cos 1rx A 'Y x,o = (sin 31rx + sin 1rx)

2

Now we have t o find t he Normalising Const ant A.

(w w) = 1 A 2

2

A

+ -2

2

=~► A =

=1

v'2

Now t he actual wave function at t ime t = 0 is 1 'Y x,o = -J2 (sin 31rx + sin 1rx) The Wave Funct ion w(x, t) at a later time (i.e. t

= t) is

11 oo : 01 : 2a Now the actual wave function at time t = 0 is 1 Wx,o = v'2 (sin 31Tx + sin 1TX) The Wave Function w(x, t) at a later time (i.e. t

1

= t) is

1

J2 sin (7rx) exp (-iE 1t/n) + J2 sin (37rx) exp (-iE3t/ n) 32

j [email protected]

Physicsguide

@Sk J ahiruddin , 2020

1D Potentials: Part-1

where E1 =

?T

2 2

n / 2m 2 2

E3 = 9?T n / 2m Now t he expectation value of Momentum is

1 (PA) -_ 2

1 ( sin 31TX 0

. 31TX -1 i(Sln 2 O ( -in) 2 (- i n) 2

+ sin 1TX)P" (sin 31TX + sin 1TX) dx

. 1TX ) + Sin

·t:;

-'l, 11,

a (Sln . 31TX + Sin . 1TX ) d X

8X

1

(sin 31Tx

+ sin 1TX) (3 cos 31Tx + cos ?TX)

0 1

( 3 sin 31Tx cos 31TX

+ sin 31Tx cos 1TX

0

+ 3 sin 1TX cos 31Tx + sin 1TX cos 1TX)

=0

dx

1

-in 4

dx

(3 sin 61Tx + sin 21Tx 0

+ 4 sin 41Tx -

2 sin 21Tx) dx

11 oo : 01 4

(3 sin 61rx

: 31

+ sin 21rx + 4 sin 41rx -

2 sin 21rx) dx

0

=0

[email protected]

Physicsguide

33

lD Potent ials: Part-1

@Sk J ahiruddin, 2020

1.9. (i) 2L

- 2L

w5 w 5 [ 2 X

X

w02

2L

2

- 2L

w 5 - 2L + 2 X

]2L

2L

cos

2

-2L

1fX

-

4L

dx

=l

dx

=

1 + cos

1fX



1fX

2L



- 2L

1f



Sln



w2

l

= 1

° X 4£ = 1 2 1

2£ (ii) Here the length of the one-dimensional box is 4£.

11 oo : 01

: 34

Here the length of the one-dimensional box is 4£.

n21r2 n,2 En=--2ma2

[email protected]

P hysicsguide

34

@Sk J ahiruddin, 2020

lD Potentials: Part-1

(iii)

2L

1rX

cos

n,2

-

2£ n,2



2L

cos,.

-2L

1rX

1 + cos

x - -2 16£

- 2L

7r2

X

16£2

[x]2~L

7r2

X

16£2

1rX

+ 4L

dx

2L

n,2

X

4L

dx

dx

4L

2L

n,2



?

x

n,2



cos

4L

- 2L

1rX

7r2

X

16£2

7rX

X

sin 2£

2L - 2L

ri27r2

X



= 16£2

1.10. To answer t his question we must be well versed with

t he postulates of Quantum Mechanics. Please read properly t he Mathematical Background and Basic postulates notes. When a system is left undisturbed or no measurement is done on the syst em t hen the wave function is a linear

11 oo : 01

: 37

1.10. To answer this question we must be well versed with t he postulates of Quantum Mechanics. Please read properly t he Mathematical Background and Basic postulat es notes. When a system is left undisturbed or no measurement

is done on the system t hen the wave function is a linear superposition of t he energy eigenfunctions. If the system is disturbed or a measurement is made then after a long time the system tends to attain a wave function which is a linear superposition of t he energy eigenfunctions 1.11. The spin is - ~. So the degeneracy will is (2s + 1)

= 4.

The sysytem conists of 8 particles. The ground state of the syst em will be formed of 4 particles in the ground state and 35

j [email protected]

Physicsguide

@Sk J ahiruddin , 2020

1D Potent ials: Part-1

4 particles will be in the first excit ed st ate. 2 2 4n n n n Ea = 4 x 2m L2 + 4 x 2mL2 2 2

2 2 20n n 2mL2

The first excited state of t he system will be formed of 4 particles in the ground state and 3 particles will be in t he first excited state and 1 particle in the second excited state. n 2 n2 E1 = 4 x 2mL2

4n 2n2 9n 2n2 + 3 x 2mL2 + 1 x 2mL2

25n 2n2 2mL2

Now t he minimum excitation energy is 25n 2n2 E1 - Ea = 2mL2

20n 2 n2 2mL2

5n2 n2 2mL2

1.12. The wave function in posit ion space is given by

11 oo : 01

: 39

251r2fi 2 EI - E o = 2m£2

201r 2!i2 2m £ 2

57r2fi2 2m L 2

1.12. The wave function in posit ion space is given by

2 .

-

Slil

L

[email protected]

n1rx

L

Physicsguide

36

@Sk J ahiruddin, 2020

-3L4

ID Potentials: Part- I

w*w dx

L 4

-3L 4

2 L

Sln 4

SIIl

3L

1 - cos

L

4

3L

L

---

4

L

4

X

1

1

--

L 21r

=---= 0.1817 2 7r

L

dx

L

L

1

dx

21rx

4

-

1TX



L

L

1

1

1TX



L



SIIl

3L L x 4

21r

-



SIIl

21r L -XL 4

11 oo : 01

: 42

L

X

21r

1 1 = - - - = 0.181 7

2

7r

-3L 4

w*w dx

L 4

3L

2 L

21rx

4 •

Slll

Slll

L

L 4

3L

1 L

L

1

3L

4

1 - cos

4

-

L

L

-

-

4 -

1 L

-

4

41r L



Slll

[email protected]

21rx



L

41rx

dx

L

41r



Slll

L

X -

3L

L x 4 L

X

4

dx

1 2

41r

P hysicsguide

37

@Sk J ahiruddin , 2020

lD Potentials: Part-1

1.13. -L

2

\¥ 2=

w*w dx

0

2

-

-L2 •

Slll

1I"X

L 0 L L 2 1 1 - cos L 0 1 L 1 - - 0 -L 2 L 1

-

1I"X



Slll

L

21rx

dx

L •

Slll

dx

21r

L

xL 2

-



Slll

21r L

xO

L X

21r

11 oo : 01 1 - cos

L - - 0 L 2

1 -L

1

dx

L

0

: 44

21r L xL 2



Sln

-



Sl n

211" L

X

Q

X

L 21r

1

-

2

1 .14. Be careful and see that the infinite square well ranges from -a ::; x ::; a and not our usual r ange that extends from

0 < x < a. Basically when we choose t he range O < x < a t he origin always contains ant i-nodes of the wave funtion. When t he range extends b etween -a < x < a t he origin contains bot h t he nodes and ant i-nodes of the wave function. The wave function of a particle within an infinite square

j [email protected]

Physicsguide

38

@Sk J ahiruddin , 2020

ID Potentials: Part-1

potential well extending from -a < x < a

2 n1rx -cos L L 2 n1rx - Sln L L

W=

for odd value of n



for even value of n

For the ground state n=odd= l . Now to the probability to ("'

1

I

1

I



1

• , 1



11



1

11 oo : 01

: 47

2 . n1rx - Sln L L

for even value of n

For the ground state n= odd= l . Now to the probability to find the particle wit hin the said range a 2 - a 2 a 2 -a 2

P=

w*w dx 2 cos 2a

a 1 2 cos2 -a

a

2 cos 2a

1fX

2a

1fX

2a

dx =

1fX

2a

dx

a 2 2 cos 2 -a

1

2a

2

1fX

2a

dx

2

[email protected]

Physicsguide

39

@Sk J ahiruddin , 2020

lD Potentials: Part-1

a 1 2a 1 x 2a 1 ra

2 - a 2

1 + cos

a + -sin 7f

a

21rx

2a

a 1fX

a

a i ..

2

-a

2 .. \ l

dx

11 oo : 01

: 49

2 l 2a

a .

X

+ -1r Sl n

a 2 -a 2

1rx a

1.15. Find t h e momentum space wave function

1 ~

cp(p) =

CX)

'l/Jo(x) e- ipx/fidx - (X)

1

~

2 a

a/2

cos - a/2

7rX fi

e-ipx/!idx

a

Do t he integration and t ake m odulus.

40

j [email protected]

P hysicsguide

@Sk J ahiruddin , 2020

1D Potentials: Part-1

1.16. At first we have to normalise t he wave-function . 'lj; ( x)

= A sin aA 2

7rX

a

1 + cos 2 • - Sl n a

7rX

a

7rX

a

+

2 • - Sl n a

7rX

a

cos

7rX

a

11 oo : 01 : s2 1.16. At first we have to normalise t he wave-function. 1rX

'lj; (x) = A sin

1 + cos

a

2 - Sl n a

~A



2

1rX

a

1rX

a

a



+

2 . 1rX -sin -

~A 2

2 1rX 1rX - Sln cos a a a

2 1rX - x 2 sin cos a a

l +2

a

1rX

a

aA 2 Now to normalise

a 2 -A (1 1) 2

==>~►

1 + 4 (2 2)

==!l~>

A

2

5a

X -

8

=1 =1 8 5a

Finally the wave function is

'l/J (x) =

a 2

Now the prob ability to get t he ground state energy

·1·t

P ro b a b 1 1 y [email protected]

=

I(1

w) 2 ('11 '11)

41

4 5 Physicsguide

@Sk J ahiruddin , 2020

ID Potentials: Part-1

Finally we n eed to find x 4

X

==>~► X

= 80

11 oo : 01 : s4 @Sk J ahiruddin, 2020

1D Potent ials: Part-1

Finally we need to find x 4

X

5

100

-

=~► X

== 80

1.17. In this problem just by finding the expectation value of energy we will be able to find t he correct answer. Nev-

ertheless we will provide you wit h t he idea how to find the expectation value of position. The given wave function is normalised. So the expectation value of Energy can be written as

Thus we get answer a as t he correct option. Now if we wanted to compute t he expectation value of post ion we had to follow t he next few steps \ X)

1 == 3 (Wl IX Wl)

2

+ 3('l/;2 X 'l/;2)

i +3

3 • • •

1.18. To solve t his problem we need the following t [email protected]

@Sk J ahiruddin, 2020

42

P hysicsguide

1D Potent ials: Part-1

11 oo : 01 : s1

@Sk J ahiruddin , 2020

lD Potentials: Part-1

metric ident ity. sin(3A)

~~►

3

= 3 sin A - 4 sin A 3

3

sin A =

sin A -

4

~ sin(3A) 4

Now we will try to rearrange t he wave function to a well known form.

\J! (x) = A sin

7rX

3

a~ a 2 2 a

3 . = A- Sln

1 . - A - sin

7rX

4

a

4

a

~

. 3 Sln

X

2 . - Sln a

a

7rX

2

a

31rx

a

a - cp3 2

Normalise

(\JI \JI) =

A2

A=

9a2 32

2

a =1 + 32

32 l Oa

F inally, t he form of the wave function stands as a 2

32 lOa T A oc exp [email protected]

Vo

and E

=

~~ . Thus 1

Vo Vo - 100

- J2m (0.99½ ) 52

Physicsguide

lD Potentials: Part-1

@Sk J ahiruddin , 2020

Similarly for barrier B T B oc exp

-

~~> T B oc exp

2m

2Vr0 -

Vo 100

- J2m (1.99V0 )

Only opt ion a has an expression consist ing of the calcut at ed t ransmission probabilit ies.

11 oo : 02 : 2s ===>~► TB ex exp - ✓2m (1.99V0 ) Only opt ion a has an expression consisting of the calcutated t ransmission probabilities. 1.39. Just use the expression of reflection coefficient v'E - ✓E - Vo

R=

2

v'E + ✓E - Vo

E = 1, o = 0.75 here. 1.40. You can prove t hat if t he number of energy levels are

(N + 1), then N1r

1r

- 2 - 0 if X < 0

where w is a const ant. Which one of the following represents t he possible ground state wave function of the p art icle?

[JEST 2019]

11 oo : oo : 09

[email protected]

Physicsguide

3

lD Potent ials: Part-2

@Sk J ahiruddin , 2020

q,

+I

(8)

(A)

+oo

+oo :t

X •

-1

II/

+l

- 1

.

(D)

(C)

+oo

+oo -1

-1

Prob 1.3. Consider t he 1-D asymmetric double-well potent ial V(x) as sketched below. The probability distribution

p(x) of a part icle in t he ground state of this potential is best represented by [TIFR 201 7]

V(x)

11 oo : oo : 11

[email protected]

P hysicsguide

4

@Sk J ahiruddin, 2020

lD Potentials: P art-2

p(x)

p(x)

X

X

p(x)

p(x)

X

----=:::::___

_,____-=----:x

Prob 1.4. A part icle is confined inside a one-dimensional

potential well V (x), as shown below.

v(.r)

One of t he possible probability distributions 'ljJ (x)

2

for

11 oo : oo : 14

One of t he possible probability distributions 'ljJ (x) 2 for [TIFR 2016] an energy eigenstate for this particle

j [email protected]

P hysicsguide

5

@Sk J ahiruddin , 2020

1D Potent ials: Part-2

IV' (x)l2

IV'( X )!2 (a)

IV'(x)lz (d)

(b) X

0

(c)

X

0

X

0

X

Prob 1. 5. A part icle is constrained to move in a truncated

harmonic potential well (x > 0) as shown in the figure. Which one of the following st atements is CORRECT? [GATE 2012]

V(.Y)

X

(a) The parity of t he first excited state is even (b) The parity of the grounded state is even (c) the ground state energy is ½liw

11 oo : oo : 16 X

(a) The parity of the first excited state is even (b) The parity of the grounded state is even (c) the ground state energy is ½nw (d) The first excit ed state energy is ~ nw Prob 1.6. The energy levels of a particle of mass m in a 00

potential of the form V (x) =

X

O [email protected]

Physicsguide

6

lD Potentials: Part-2

@Sk J ahiruddin, 2020

are given in terms of quant um number n=0,1,2,3 .... by [GATE 2007] (a) (n + ~)nw (b) (2n + ~)nw (c) (2n + ~)nw (d) (n + ~)nw Prob 1. 7. T wo harmonic oscillator A and B are in excited eigenstates wit h t he same excitation energy E , as measured

from their respective ground state energies. The natural frequency of A is twice that of B.

- 10

-5

0

5

10

If the wavefunction of B is as sketched in t he above pict ure, which of the following would best rep1~esent t he wave-

11 oo : oo : 19 -10

0

-5

5

10

If t he wavefunction of B is as sketched in t he above pict ure, which of the following would best represent t he wave[TIFR 2016] fucntion of A?

[email protected]

Physicsguide

7

lD Potentials: Part-2

@Sk J ahiruddin , 2020

(a)

- 10

(b)

-5

5

10

-10 -5

1.2

10

5

10

(d)

(c)

-10

5

-5

5

10 -10

Probability

Prob 1. 8. A one-dimensional harmonic oscillator is in t he

state

11 oo : oo : 22 Prob 1.8. A one-dimensional harmonic oscillator is in the

state

where w0(x) \lf 1(x) and w2(x) are the ground, first excited and second excited states, respectively. The probability of finding the oscillator in the ground state is [GATE 2006] 3 9 (a) 0 (b) JM (c) 14 (d) 1 Prob 1.9. Consider a harmonic oscillator in t he state

l"P) =

a2

e

2

e a at

0) , where 0) is the ground state, at is the raising

j [email protected]

8

@Sk J ahiruddin, 2020

Physicsguide

1D Potentials: Part-2

operator and a is a complex number. What is t he probability t hat t he harmonic oscillator is in the n-th eigenstate n) ? [JEST 2015] a2 2n

la2J _a_ (a ) e n 1.

(b)

2

e

la n Vnf

a2 a12n

(d)e

1.3

2

n!

Uncertainty Principle

Prob 1.10. Let 0) and 1) denote the normalized eigen-

states corresponding to t he ground and t he first excited

11 oo : oo : 24 1.3

Uncertainty Principle

Prob 1.10. Let 0) and 1) denote the normalized eigenstates corresponding to t he ground and the first excited states of a one-dimensional harmonic oscillator. The uncertainty ~ x in t he state 0) + II)) is [NET D ec 2012] (a) ~ x = J!i/2mw (b) ~ x = J!i/mw

f2(

(c) ~ x = J2!i/mw

(d) ~ x = J4!i/mw

Prob 1.11. The product ~ x ~ p of uncertainties in t he position and momentum of a simple harmonic oscillator of mass m and angular frequency w in the ground state IO) , is !i/ 2. The value of the product ~ x ~p in the state e- ipf/h 0) (where R, is a constant and p is t he moment um operator) is [NET [email protected]

P hysicsguide

9

@Sk J ahiruddin, 2020

lD Potentials: Part-2

Dec 2018]

(a) fi 2

mwf

2

(b) Ii

h

(c) !i/ 2

h2 (d) mw£2

Prob 1.12. Let 0) and 1) denote t he normalized eigenstates corresponding to t he ground and first excited states of a one dimensional harmonic oscillator. The uncertainty ~ Pin the state 0) + 1) ), is [NET Dec 2011]

f2 (

(a) ~p = ~ / 2

(c) ~ p =

!imw

(b) ~p = J!imw / 2 (d) ~ p = ✓2!imw

11 oo : oo : 21 ~Pin t he state

f2 (0) + 1) ), is

(a) ~P = ~

/2

(c)

(b) ~P (d ) ~P

~ p=~

1.4

=

[NET Dec 2011]

Jnmw/2

= ✓2nmw

Expectation Values

Prob 1.13. A particle is in a state which is a superposition of t he ground state 0 and t he first excited state 1 of a one-dimensuonal quantum harmonic oscillator. The state is

.

given by

1 = ✓5 0 5

+

2 . ✓5 1 . The expectation value of the 5

energy of t h e particle in t his state (in units of nw, w being [JAM 2015] t he frequency of the oscillator) is _ _

Prob 1.14. In a one-dimensional harmonic oscillator , 0 , 1 and 2 are respectively the ground, first and the second excited states. These tree states are normalized and are ort hogonal to one another. W1 and W2 are two states defined j [email protected]

Physicsguide

10

@Sk J ahiruddin, 2020

1D Potent ials: Part-2

by

where a is a constant

[GATE 2011]

i) The value of a which '11 2 is orthogonal to '11 1 is

(a) 2

(b ) 1

(c) -1

(d ) -2

ii) For the value of a det ermined in t h e above question, the PYn Prt.:::itinn , ,.:::i l11 P nf Pn Pr o-, , nf t.h P n~rill.:::it.nr in t h P ~t.:::.tP \lJ "

11 oo : oo : 29 i) The value of a which '11 2 is orthogonal to '11 1 is (a) 2 (b) 1 (c) -1 (d) -2 ii) For the value of a determined in the above question, the expectation value of energy of the oscillator in the state w2 •

IS

(a)

nw

(b) 3nw/ 2

(c)

3nw

(d) 9nw/ 2

Prob 1.15. A one dimensional harmonic oscillator is in t he superposition of number state n) given by

The average energy of t he oscillator in the given state is _ nw [GATE 2014] Prob 1.16. The ground state energy of 5 identical spin 1/ 2 particles which are subject to a one dimensional simple [JEST harmonic oscillator potential of frequency w is 2012]

(b) (13/2)nw

(a) (15/2)nw [email protected]

(c) (1/2)nw

11

(d)

5nw

Physicsguide

@Sk J ahiruddin , 2020

ID Potent ials: Part-2

Prob 1.17. A harmonic oscillator has the wave function,

w(x, t) =

1

5

[3~0 (x, t) - 2J2~ 1 (x, t) + 2J2~ 2 (x, t)]

where ~n(x, t) is the eigenfunction belonging to t he n-th 1 energy eigenvalue (n + )nw. The expectation value (E) of

2

energy for the state w(x, t) is (a)

1.58nw

(b)

0.46nw

[TIFR 2013] (c)

nw

(d )

1.46nw

11 oo : oo : 32 where o - 21>1 + 31>2 1>o - 1>1 + a 1>2) = 0 (1>o 1>o)

+ 2(1>1 1>1) + 3a(1>2l 1>2) = 0 3a = -3 a = - 1

Here vve have used t he property

[email protected]

25

@Sk J ahiruddin, 2020

P hysicsguide

ID Potentials: Part-2

Thus t he state \JI 2 is given by

The state \JI 2 is orthogonal to \JI 1 but is not normalised. As t he state is simple it can be normalised easily by inspection. The normalised state is

11 oo : 01

: 11

The state \JI 2 is orthogonal to \JI 1 but is not normalised. As t he state is simple it can be normalised easily by inspection. The normalised stat e is

(ii) The expectation value of Energy of the oscillator in state \JI 2 is

1.15. This type solved previously 1 1.16. The spin of t he particle is . The degeneracy due to 2 1 spin is (2s + 1) = (2 x + 1) = 2. This means that in

2

a particular state two particles can accommodate wit hout 26

[email protected]

P hysicsguide

lD Potentials: Part-2

@Sk J ahiruddin , 2020

breaking t he P auli's Exclusion Principle. Now the ground state energy of t he system E

= 2 x o+

1 -2

'fu»+2x

1 2+ 2

11 oo : 01

: 13

breaking t he P auli 's Exclusion Principle. Now t he ground state energy of t he system

E =2x

1 0+ 2

1 2+2

liw+2x

= 13 liw 2 1.17.

9

8

(E) = 25Eo + 25E1

8

+ 25E2

where

1 n+- liw 2 1.18. We solved previously that for half harmonic oscillator

3 7 Eo = nw, E1 = nw 2 2 So, t he expectation value of energy

(E) =

1 3

4 7

-5 · -+· 2 5 2

1.19. The expression for the position operator of a linear harmonic oscillator is X"' = •

1,1[

The state is w)

j [email protected]

@Sk J ahiruddin, 2020

Now

= 1

J2 27

Physicsguide

1D Potent ials: Part-2

11 oo : 01

: 16

@Sk J ahiruddin , 2020

lD Potentials: Part-2

Now !i (a+at) w)

2mw

_n_{w (a+ at) w)

-

2mw

2 1 2

1

2

2mw •



1,Jr

'l7r



1,7r

Ii

1 2

2mw

e 2 {'llo 'llo) •

-

2

e2

2mw 1

2v'2

mw

1

Ii

2v'2 =0

mw

cos

+-

2



1,Jr

1

l

-1,Jr

+e 7r

2

2

+ isin

7r

2

+ cos

7r

2

-



'l



Sln

7r

2

xO

[email protected]

@Sk J ahiruddin, 2020

28

Physicsguide

lD Potentials: Part-2

1.20. The expression for the position operator of a linear

11 oo :01

:1

a

@Sk J ahiruddin , 2020

lD Potentials: Part-2

1.20. The expression for the position operator of a linear harmonic oscillator is i;

=

2 li (a+ al) mw

i

1

i

The state is w) = Now

v'3IO) + v'3 1) + v'3 2)

(w x w) = (w

fi

2mw

(a + at) w)

!3

2mw

~

;w (W l(a 0) + a1IO)) + (ai jl ) + a1i 1)) + (ai 2) + ali j2) 2

1 3

!3

fi (w (a+at) (0) +il l ) +i 2))

n

2mw fi

2mw

(w lv'l 1) + v'li o) + v'2il 2) + v'2i 1) + v'3i 3)

(0 - i( l l - i(2 i v'l 0) + (v'l

+ i v'2) 1) + iv'2 2) + iv'3 3) l 3 l 3 2 3

ri ivl(O 0) - (i 2mw fi [i - i +

2mw

v'2 +

V1 + i

X

X

i v'2)(1 l ) - i

X

iv'2(2 2)

v'2]

fi mw

j [email protected]

@Sk J ahiruddin , 2020

29

Physicsguide

lD Potentials: Part-2

11 oo : 01 3

: 21

mw 29

[email protected]

P hysicsguide

lD Potentials: Part-2

@Sk J ahiruddin , 2020

1.21. We can writ e t he position operator in terms of ladder operator

or n ' X" n

2mw

\fn8n',n- l

+ Jn + l 6n' ,n+l

or

n

"

X IO) =

2mw

10)

It will be b etter to use Heisenb erg picture here. Still we need to do lit t le bit calculation. The operator will evolve in t ime as

or

(0 X (t)X (O)IO) = 0 eiHt/nx (O)e-iHt/nx (o) O

2mw

O eiHt/nx (O)e-iHt/n l

Now e- iHt/n is t he t ime transnat ional operator which make t he state evolve in t ime

e- iHt/n 0) = 0(t)) [email protected]

30

Physicsguide

11 oo : 01

: 23

make the state evolve in time e- iHt/n j [email protected]

0) = O(t)) Physicsguide

30

@Sk J ahiruddin, 2020

1D Potentials: Part-2

and e- iHt/ n 1)

= l (t))

You can do explicit integration from here. But better to A

say that t he action of X on 1 (t)) will give and

n - - 1 (t)) . 2mw

. There 1s

another

?

n 2mw

IO(t))

n at t he outside.

-rru,,;

Hence t he factor outside will be h . 2mw Please complete t he explicit integration also. 1.22. You can solve t hese types easily if you have solved t he coherent states of Harmonic oscillator. I'm t rying to give a

solution here. See t hat

where f is a complex number. We define a state f ) as quantum state

If) = e-lfl2/2efat 0) t he action of a a f) =ff)

and at

11 oo : 01

: 26

a J) =

f J)

and at

31

[email protected]

Physicsguide

@Sk J ahiruddin, 2020

lD Potentials: Part-2

Aft er t his my calculation becoming little complicat ed . If you can do in simpler way, please inform m e.

1.23. You need to perform the integral. 00

1 X

-

7r[2

CX)

1/4

1+

v'2x l

2

x2

exp

2z 2

you need to use

oo

xn exp [-axm ] dx =

1 f 2x - x

(n+l)

xnexp [- axm] dx =

1 f 2x - x

(n+l)

oo

m

m

n +l a

1.24. Use

oo 00

m

m ·n +l a

1. 2 5. The normalized wave function is w(x ) = Co cf>o (x) + C 1 c/> 1 ( x) . As t he wave function is normalised it means t hat

C5 +Cl== 1. Now we need to find the values of C0 and C 1 for which t he expaectation value of displacement for the state be maximized.

w will

For a harmonic oscillator we know that (c/>i x Ic/>j ) = Now

Oij

11 oo : 01

: 29

Now we need to find the values of C0 and C 1 for which the expaectation value of displacement for the state '11 will be maximized. For a harmonic oscillator we know that (i x lj) = bij Now

32

j [email protected]

Physicsguide

@Sk J ahiruddin, 2020

1D Potent ials: Part-2

(w(x)lx l'll (x))

+ C1(1 (x)) x (Co o (x)) + C1 1 (x))) = CJ(o x o) + cf (1 x 1) + CoC1 (ox 1) + CoC1 (1 x o) = (Co (o (x)

= 2CoC1 (o x 1) We want to m aximize (x) so 2C0 C1(o x 1) must be maximum. •

(x)max = [( C5

2

+ cf) - (Co - C1 ) ] (o X 1) 2

= [1 - (Co - C1) ](o x 1) For the expectation value to be maximized we need to have

(Co-C1) 2 =0

~► Co=C1

1.26. We will solve this problem in t he footsteps of t he previous problem number Prob 1.25. We want to minimize

(x) so 2b1b2(0lx ll) must be minimum. 2 (X) min = [(bf + b~) - (b1 - b2) ](0 X I1)

= r1 - (b1 - b2) l (Olx 1) 2

11 oo : 01

: 31

vious problem number Prob 1.25. We want to minimize (x) so 2b1b2(0lx ll ) must be minimum.

(X) min

2 = [(bi + b~) - (b1 - b2) ] (0 XI1) 2 = [1 - (b1 - b2) ](Olx 1)

For the expectation value to be minimised we need to maximise (b 0 - b1 ) 2

[email protected]

P hysicsguide

33

lD Potentials: Part-2

@Sk J ahiruddin , 2020

1.27. From problem Prob 1.25. we will get t he following results.

Applying t he normalised condit ion we will get t he value of 1 Co= C1 =

v'2

Now t he expression of x in t erms of raising and lowering operator is

x=

n (at + a) 2mw 1

Now 1

n

(x) max = 2coc1 (0 X 1) = 2 v'2 v'2

2mw

(o (at + a) 1)

n 2mw 1.28. T his is coherent st at e for

00

1

f =

l

00

1

11 oo : 01

: 34

2mw 1. 28. This is coherent st at e for

In t his coherent stat e for

[email protected]

f =

f =

1

1, the expectation values

P hysicsguide

34

@Sk J ahiruddin, 2020

lD Potentials: Part-2

you can prove

(f x

2

f)

n, =2mw

=

(f p

2

f (a + a )

2

f

n (! + !* + (f*f + 1) + f *!) 2mw 2

mnw f) = -2

=-

t

2

t

2

(a - a )

f

mnw (! + ! * 2 2

2

-

f

(f *f

+ 1) -

f*f)

Now put f = 1 and use t he expression of Hamiltonian The det ailed discussion of coherent states are found in problem in Griffiths 3.35 (2nd edition), 3.42 (3rd edit ion) and in Sakurai page no 96 and problem 2.19 (second edit ion). I'll include t he coherent states in our notes as soon as possible.

11 oo : 01

: 36

problem in Griffiths 3.35 (2nd edition), 3.42 (3rd edition) and in Sakurai page no 96 and problem 2.19 (second edit ion). I'll include t he coherent states in our notes as soon as p ossible. 1.29.

1.30. In the Heisenberg picture we know that t he observables of the syst em evolve wit h t ime.

da = dt

Now

a does

[a iI] aa in ' + at 1

not depend on time explicitly.

j [email protected]

35

Physicsguide

@Sk J ahiruddin, 2020

1D Potent ials: Part-2

da

1

in [a, iIJ

dt

2

1

"a -+-mw-x Px l ? 2 ' 2m 2

in 1 in

+

+

0,

+o

2

" Px a , 2m

!mw 2 x 2 '2

1

mw

in

2n

mw 2n



A

ip

x+ mw . "'

ip

x+ mw

p; '2m

1 2 2 -mw x '2

11 oo : 01 1

mw

in

2n

mw

in

2n

mw

+

2n mw

+

2n

A

ip

p; ' 2m

x+ mw A

ip

1 2 2 -mw x '2

x+ mw

2n

1





mw

+

: 39

2

, ., Px x, 2m

+

mw

2n

. ,...

ip

p~ mw' 2m

x' ~2 mw 2 x 2 ip •

A

1

- mw 2 x 2 mw' 2

[email protected]

P hysicsguide

36

@Sk J ahiruddin, 2020

lD Potentials: Part-2

2

1

mw

,. . Px

in

2n

x, 2m

1

mw

in

2n

1

mw

in 2n

+

2n

2

,. . Px x, 2m

ip , -l mw 2 x 2

+

mw 2

in X 2p iw ( - - - + - - 2X

2n

mw

mw

2m

p

2

X

.t;_ )

'l 1i



- - iwx m

= -iwa •

A

1.31. This type of problem has been solved in the chapter

''Basic Formulation'' . Still here we will give an outline of how to solve the problem. The stat e at time t = 0 is

11

00:01 :41

= -iwa •

A

1.31. This type of problem has been solved in the chapter

''Basic Formulation''. Still here we will give an outline of how to solve the problem. The stat e at time t = 0 is

The evolved state at time t

l'l/J (t)) =

1

v'2

exp

-

= t is

i Eot

2)

0) + exp

Ii

Now we need to find t he time at which t hese two states will

[email protected]

Physicsguide

37

lD Potentials: P art-2

@Sk J ahiruddin, 2020

be orthogonal

~►

1

2

iEot

exp

+ exp

Ii

~~►

exp

{'lf;(O) 'l/; (t)) = 0 iE2t

exp

D

\ -1- \

=0

Ii

iEot Ii

i(E2 - Eo )t Ii

Equating the real part we get If D

-

= -exp

= -1

11 oo : 01 : 4s =-1

exp Equating t he real part we get

=-1

cos

~► t =:::::,~►

==:,~►

= (E

t=

2

Ii _

Ea) arccos( -1)

Ii --X7r 2/iw 7r

t=-

2w

1.32. This problem has been solved in t he chapter ''Basic

Formulation'' . Here if you closely follow the sum you will not ice t hat t he ground state and the first excit ed has flipp ed . We alredy know t he minimum time required for flipping of states which is

T =

h

2(E zower j [email protected]

-

E higher ) Physicsguide

38

@Sk J ahiruddin , 2020

1D Potentials: Part-2

Putting all t he necessary values we will get option d as t he answer. 1.33. Energy Level of two Dimensional harmonic potential

well is

E (nx, ny) = (nx + ny

+ 1)/iw

Now the total energy of t he state is 4/iw. Thus we have n x + ny

=3

11 oo : 01

: 47

E (nx, ny) = (nx + ny

+ l )nw

Now the total energy of t he state is 4nw. Thus we have

nx + ny = 3

1

2

2

1

II Total 3 3

So the st ate is 2 fold degenerate. 1.34. The trick to solve this type of problem is to express

t he 'x' or 'y' part of the equat ion as whole square terms. We will explain this by solving the following examples. But t his problem just requires some re-arranging. 1 1 1 2 2 2 2 2 2 2 V (x , y) = mw (x + 4y ) = mw x + m4w y

2

2

2

Thus the Energy expression will be En =

nx +

1

2

nw +

1 ny + 2nw 2 For the first excited state we have n x = l ; ny = 0

[email protected]

P hysicsguide

39

@Sk J ahiruddin , 2020

lD Potential s: Part-2

Thus 1 1 +2 1.35. This is not a perturbation problem. By rotating t he

axis t he problem becomes two simple harmonic oscillator problem. What are the frequencies? The PE matrix is /

'\

11 oo : 01

j [email protected]

: 49

Physicsguide

39

@Sk J ahiruddin, 2020

lD Potentials: Part-2

T hus 1 1 +2

1 o+-2

1uu+

2ru.u

=

5 luu 2

1.35. This is not a perturbation problem. By rotating the

axis the problem becomes two simple harmonic oscillator problem. What are the frequencies? T he P E matrix is mw

2

5

3

2

2

3 2

5 2

The KE matrix m

1 0 0 1

Let the eigenfrequencies are w' 5 2 3 2

-32 5 2

- mw'

2

1 0 0 1

=0

Solving we get eigen frequencies 2w,w

Hence t he energy for ground state is 3 = - luu 2 2 2 1.36. The x state is in first excited state and the y state is in ground state. 1

1 - n2w + - luu

j [email protected]

40

Physicsguide

11 oo : 01 : s2

j [email protected]

Physicsguide

39

@Sk J ahiruddin, 2020

lD Potentials: Part-2

T hus 1 1 +2

1 o+-2

1uu+

2ru.u

=

5 luu 2

1.35. This is not a perturbation problem. By rotating the

axis the problem becomes two simple harmonic oscillator problem. What are the frequencies? T he P E matrix is mw

2

5

3

2

2

3 2

5 2

The KE matrix m

1 0 0 1

Let the eigenfrequencies are w' 5 2 3 2

-32 5 2

- mw'

2

1 0 0 1

=0

Solving we get eigen frequencies 2w,w

Hence t he energy for ground state is 3 = - luu 2 2 2 1.36. The x state is in first excited state and the y state is in ground state. 1

1 - n2w + - luu

j [email protected]

40

Physicsguide

11 oo : 01 : s4

j [email protected]

Physicsguide

39

@Sk J ahiruddin, 2020

lD Potentials: Part-2

T hus 1 1 +2

1 o+-2

1uu+

2ru.u

=

5 luu 2

1.35. This is not a perturbation problem. By rotating the

axis the problem becomes two simple harmonic oscillator problem. What are the frequencies? T he P E matrix is mw

2

5

3

2

2

3 2

5 2

The KE matrix m

1 0 0 1

Let the eigenfrequencies are w' 5 2 3 2

-32 5 2

- mw'

2

1 0 0 1

=0

Solving we get eigen frequencies 2w,w

Hence t he energy for ground state is 3 = - luu 2 2 2 1.36. The x state is in first excited state and the y state is in ground state. 1

1 - n2w + - luu

j [email protected]

40

Physicsguide

11 oo : 01 : sa

j [email protected]

Physicsguide

39

@Sk J ahiruddin, 2020

lD Potentials: Part-2

T hus 1 1 +2

1 o+-2

1uu+

2ru.u

=

5 luu 2

1.35. This is not a perturbation problem. By rotating the

axis the problem becomes two simple harmonic oscillator problem. What are the frequencies? T he P E matrix is mw

2

5

3

2

2

3 2

5 2

The KE matrix m

1 0 0 1

Let the eigenfrequencies are w' 5 2 3 2

-32 5 2

- mw'

2

1 0 0 1

=0

Solving we get eigen frequencies 2w,w

Hence t he energy for ground state is 3 = - luu 2 2 2 1.36. The x state is in first excited state and the y state is in ground state. 1

1 - n2w + - luu

j [email protected]

40

Physicsguide

11 oo : 02 : oo

j [email protected]

Physicsguide

39

@Sk J ahiruddin, 2020

lD Potentials: Part-2

T hus 1 1 +2

1 o+-2

1uu+

2ru.u

=

5 luu 2

1.35. This is not a perturbation problem. By rotating the

axis the problem becomes two simple harmonic oscillator problem. What are the frequencies? T he P E matrix is mw

2

5

3

2

2

3 2

5 2

The KE matrix m

1 0 0 1

Let the eigenfrequencies are w' 5 2 3 2

-32 5 2

- mw'

2

1 0 0 1

=0

Solving we get eigen frequencies 2w,w

Hence t he energy for ground state is 3 = - luu 2 2 2 1.36. The x state is in first excited state and the y state is in ground state. 1

1 - n2w + - luu

j [email protected]

40

Physicsguide

11 oo : 02 : 02

j [email protected]

Physicsguide

39

@Sk J ahiruddin, 2020

lD Potentials: Part-2

T hus 1 1 +2

1 o+-2

1uu+

2ru.u

=

5 luu 2

1.35. This is not a perturbation problem. By rotating the

axis the problem becomes two simple harmonic oscillator problem. What are the frequencies? T he P E matrix is mw

2

5

3

2

2

3 2

5 2

The KE matrix m

1 0 0 1

Let the eigenfrequencies are w' 5 2 3 2

-32 5 2

- mw'

2

1 0 0 1

=0

Solving we get eigen frequencies 2w,w

Hence t he energy for ground state is 3 = - luu 2 2 2 1.36. The x state is in first excited state and the y state is in ground state. 1

1 - n2w + - luu

j [email protected]

40

Physicsguide

11 oo : 02 : os

j [email protected]

Physicsguide

39

@Sk J ahiruddin, 2020

lD Potentials: Part-2

T hus 1 1 +2

1 o+-2

1uu+

2ru.u

=

5 luu 2

1.35. This is not a perturbation problem. By rotating the

axis the problem becomes two simple harmonic oscillator problem. What are the frequencies? T he P E matrix is mw

2

5

3

2

2

3 2

5 2

The KE matrix m

1 0 0 1

Let the eigenfrequencies are w' 5 2 3 2

-32 5 2

- mw'

2

1 0 0 1

=0

Solving we get eigen frequencies 2w,w

Hence t he energy for ground state is 3 = - luu 2 2 2 1.36. The x state is in first excited state and the y state is in ground state. 1

1 - n2w + - luu

j [email protected]

40

Physicsguide

11 oo :02 : oa

j [email protected]

Physicsguide

39

@Sk J ahiruddin, 2020

lD Potentials: Part-2

T hus 1 1 +2

1 o+-2

1uu+

2ru.u

=

5 luu 2

1.35. This is not a perturbation problem. By rotating the

axis the problem becomes two simple harmonic oscillator problem. What are the frequencies? T he P E matrix is mw

2

5

3

2

2

3 2

5 2

The KE matrix m

1 0 0 1

Let the eigenfrequencies are w' 5 2 3 2

-32 5 2

- mw'

2

1 0 0 1

=0

Solving we get eigen frequencies 2w,w

Hence t he energy for ground state is 3 = - luu 2 2 2 1.36. The x state is in first excited state and the y state is in ground state. 1

1 - n2w + - luu

j [email protected]

40

Physicsguide

11 oo : oo : 01

Problems and Solutions in 3D Systems in Quantum Mechanics Sk J ahiruddin * Souradeep Mondal

*Assistant Professor Sister Nibedita Govt. College, Kolkata Author was t he topper of IIT Bombay M.Sc P hysics 2009-2011 batch He ranked 007 in IIT J AM 2009 and 008 (J RF) in CSIR NET J une 2011 He has been teaching CSIR NET aspirants since 2012

1

@Sk J ahiruddin , 2020

3D Systems

11 oo : oo : 04 1

@Sk J ahiruddin, 2020

3D Systems

Contents 1 Problems from NET, GATE, JEST, TIFR & JAM papers 1.1 Free Particle in a 3D Box 1.2 Free Particle in 3D Harmonic Well Potential 1.3 Hydrogen Atom Problems 1.4 Ans Keys 1.5 Solutions . • •

j [email protected]





























3 6 7





















































































2

3

17 18

Physicsguide

11 oo : oo : 06

j [email protected]

2

P hysicsguide

@Sk J ahiruddin , 2020

1

1.1

3D Systems

Problems from NET, GATE, JEST, TIFR & JAM papers Free Particle in a 3D Box

Prob 1.1. A particle of mass m is confined in a potentialbox of sides Lx, Ly, Lz, as shown in the figure. By solving schrodinger equation of the part icle, find its eigenfunctions and energy eigenvalues. [JAM 2012]

z

•• ••• •

j(0,0,0)

••···-····--··········· ..........

(O,l ,0)

y

••••••••••••

••

.x

Pro b 1. 2. The ground st ate energy of a particle of mass m in a t hree-dimensional cubical box of side l is not zero 2 3h but Bml . This is because [TIFR 2015]

11 oo : oo : 09 Pro b 1. 2. The ground state energy of a particle of mass m in a three-dimensional cubical box of side l is not zero 2 3h but Bml . This is because [TIFR 2015] [email protected]

Physicsguide

3

@Sk J ahiruddin , 2020

3D Systems

(a) the potential at the boundaries is not really infinite, but just very large. (b) this is the most convenient choice of the zero level of potential energy. (c) potential and momentum cannot be exactly determined simultaneously. (d) t he ground state has no nodes in t he interior of t he box. Prob 1.3. A particle of mass m is in a cubic box of size a. The potent ial inside the box (0 < x < a, 0 < y < a, 0
H ~rni ltoni ~n

h11t not o f T.2 or T. _

11 oo : oo : 19 ro

e

.

nlm

eno e VAAe e1gens a es o a y rogen

atom in the usual notation. The state IV = ~[2 W200-3W211 + is an eigenstate of [NET Dec 2015] but not of the Hamiltonian or Lz

v'7'11 210 -

(a) L 2 ,

y'5'11 21 _ 1]

(b) the Hamiltonian , but not of L 2 or Lz 2

(c) the Hamiltonian L and Lz (d) L 2 and Lz, but not of the Hamiltonian Prob 1.13. The ground state wavefunction for the hydro-

gen atom is given by

'11 100

1

1

J4rr

ao

= --(-) 312 e- r/ao , where ao is

t he Bohr radius. The plot of t he radial probability density, [email protected]

Physicsguide

7

@Sk J ahiruddin , 2020

3D Systems

P (x) for the hydrogen atom in the ground state is [GATE 2012]

(a)

(b)

P(r)

P(r)

(d)

(c) P(r)

P(r)

11 oo : oo : 21 P(rj

P(rj

Prob 1.14. A part icle in the 2s state of hydrogen has t he wave function 1

1P2s(r) = y'21r 4

1 a0

3 2

_ r 2 a0

exp

-

r

2a0

where r is t he radial coordinate w.r.t. t he nucleus as origin and a0 is t he Bohr radius. The probability P of finding the electron somewhere inside a sphere of radius Aao centered at the nucleus, is best described by the graph [TIFR 2014]

j [email protected]

Physicsguide

8

@Sk J ahiruddin, 2020

3D Syst ems

p

p

(a) (b )

).

).

p

p (c)

(d)

/ J..

11 oo : oo : 24 (c)

(d)

Pro b 1.15. An electron is in the 2s level of the hydrogen atom, with the radial wave-function

qJ(r) =

1

v'2 3/ 2 2 2a

2- r

ao

0

exp

r 2ao



The probability P (r) of finding this electron between distances r to r + dr from t he centre is best represented by the [TIFR 2018] sketch

[email protected]

Physicsguide

9

@Sk J ahiruddin, 2020

(a)

3D Systems

"(b)

P(r )

~

r)

. _ _---1 r (c)

(r )

_

(d)

P(r )

~

r

11 oo : oo : 26

---==-._

Prob 1.16. Let drogen atom. given below

\lf 0)

___.J r

denoted the ground state of the hy-

Choose the correct statement from those

(a) [Lx, Ly] Wo) = 0 (d) [Sx, Sy] Wo) = 0

(b) J

2

IwO) =

0

[GATE 2008] (C) L.§ -/- 0

Prob 1 .17. The radial wave function of t he electrons in t he state of n 2 r exp 312

a0

= 1 and l = 0 in a hydrogen atom is R 10 =

ao

, ao is the Bohr radius. The most probable

[GATE 2008]

value of r for and electron is

(a) ao

(b) 2ao

(d) 8ao

(c) 4ao

Prob 1.18. In t he ground state of hydrogen atom , t he j [email protected]

Physicsguide

10

@Sk J ahiruddin , 2020

3D Systems

most probable distance of the electron from t he nucleus, in unit s of Bohr radius a0 is: [JEST 2016] 1 (d) 3 (b) 1 (c) 2 (a) 2 2 Prob 1.19. An electron in the ground state of t he hyr 1 drogen atom has t he wave function \lJ ( r) = -----;::===e ao na~

where a0 is constant. The expectation value of t he operator

Q=

z

2

/

r

2

,

where z = rcos0 is I

[GATE 2014]

11 oo : oo : 29 r 1 drogen atom has t he wave function w(r) = -----;:===e

ao

na~

where a0 is constant . The expectation value of t he operator

Q=

z2

Hint:

r 2 , where z = rcos0 is

-

--

---

a n+l

0

[GATE 2014] -

an+l

(b)

-a5

Prob 1.20. The normalized ground state wavefunction of 41ra

is the Bohr radius and r is the distance of t he electron from t he nucleus, located at t he origin. The expectation value 1 r 2 is [GATE 2011] 4 (d) 2 (a) ? ( c) 2 a2 a2 a= a Prob 1.21. The electronic ground state energy of the Hy1T"

(b) 41r

drogen atom is -13.6 eV. The highest possible electronic en-

[GATE 2017] ergy eigenstate has an energy equal to (a) 0 (b) 1 eV (c) +13.6 eV (d) oo [email protected]

@Sk J ahiruddin, 2020

11

Physicsguide

3D Systems

Prob 1.22. The wavefunction of a hydrogen atom is given by the following superposit ion of energy eigen functions Wnzm(r)(n , l, mare the usual quantum numbers) :

The ratio of expectation value of t he energy to the ground

11 oo : oo : 32 Wnlm r n , l, m are the usual quantum numbers :

The ratio of expectation value of t he energy to the ground state energy and t he expectation value of £ 2 are, respect ively: [JEST 2016] 2 2 101 229 d 12n 101 nd 121i (b) 2 c and n (a ) 504 an ( ) 7 504 a 7 504 (d)

~~: and Ji

2

Prob 1.23. The normalized wave functions of a Hydrogen atom are denoted by wn,l,rri (x)' where n, l and m are, respectively, the principal, azimuthal and magnetic quant um numbers respectively. Now consider an electron in the mixed st ate

W(X) = ~ W1,0,0 ( X) + ~ '¥2,1,0( X) + ~ W3,2,-2 ( X) The expectation value (E) of the energy of this electron, in [TIFR 2012] electron-Volts (eV) will be approximately (a) -1.5 (b) -3.7 (c) -13.6 (d) -80.1

Prob 1.24. Let the wavefunction of the electron in a [email protected]

@Sk J ahiruddin , 2020

12

Physicsguide

3D Systems

drogen atom be

where nzm(r) are the eigenstates of the Hamiltonian in t he

11 oo : oo : 34 rogen atom

e

where ef>nlm(r) are the eigenstates of the Hamiltonian in t he standard notation. The expectation value. of the energy in t his stat e is [NET Dec 2018] (a) - 10.8 eV

(b) - 6.2 eV

(c) - 9.5 eV

(d) -5.1 eV

Prob 1.25. An energy eigenstate of the Hydrogen atom has t he wave function

312

r Wnzm(r, 0,ef>) = ~ sin0 cos 0 exp[-(+ ief>)] 81 1r ao 3ao where a0 is t he Bohr radius. The principal (n), azimuthal (l ) and magnetic (m) quantum numbers corresponding to t his wave function are [TIFR 2013] (a) n = 3, l = 2, m = 1 (b) n = 2, l = l , m = 1 (c) n = 3, l = 2, m = -1 (d) n = 2, l = l , m = ±1 1

1

Prob 1.26. A rigid rotator is in a quantum state described 3 by t he wave function 'l! (0, q>) = sin 0 sin q> where 0 and 41r q> are t he usual polar angles. If two successive measurements of L 2 are made on this , the probability that the second measurement will yield t he value + Ii is [TIFR 2014] j [email protected]

13

Physicsguide

@Sk J ahiruddin , 2020

(a) 0.25

(b) 0.33

3D Systems

(c) 0.5

(d ) negligible

Prob 1.27. An electron is in the ground state of a hydro£len atom. The nroba,bilitv that it is within the Bohr radius

00: 00: 37

(a) 0.25

(b) 0.33

(c) 0.5

(d) negligible

Prob 1.27. An electron is in the ground state of a hydro-

gen atom. The probability that it is within the Bohr radius [NET June 2014] is approximately equal to

(a) 0.60

(b) 0.90

(c) 0.16

(d) 0.32

Prob 1.28. If an electron is in t he ground state of the hydrogen atom , t he probability that its distance from the

proton is more t han one Bohr radius is approximately [NET June 2011]

(a) 0.68

(b) 0.48

(c) 0.28

(d) 0.91

Prob 1.29. If t he position of t he electron in the ground state of a Hydrogen atom is measu1~ed, the probability that it will be found at a distance r > a0 ( a0 being Bohr radius) is nearest to [NET Dec 2018]

(a) 0.91

(b) 0.66

(c) 0.32

(d) 0.13

Pro b 1. 30. The normalized wavefunction of a particle in 1 t hree dimensions is given by 1jJ (r, 0, cp) = e-r / 2a, where 81ra3

a> 0 is a constant . The ratio of the most probable distance from the origin to the mean distance from t he origin, is, [NET Dec 2017]

(a) 1/3

(b) 1/ 2

(c) 3/ 2

(d) 2/3

Prob 1.31. A simple model of a helium like atom with [email protected]

@Sk J ahiruddin , 2020

14

Physicsguide

3D Systems

electron-electron interaction is replaced by Hooke's law force

11 oo : oo : 39

@Sk J ahiruddin, 2020

3D Systems

electron-electron interaction is replaced by Hooke's law force is described by Hamiltonian

-n 1 - (\72 + \72) + -mw2(r2 + r2) 2

m

2

1

2

2

1

2

A -mw2 4

What is the exact ground state energy?

(a) E (b) E (c) E (d) E

71- 722 [JEST 2013]

= ~nw( l + ✓1 + A) = ~nw( l + ~ ) = ~ nw ( ✓l - A) = ~nw(l + ✓,--1-A)

Prob 1.32. The energy levels of the non-relativistic elect ron in a hydrogen atom (i.e. in a Coulomb potent ial V (r) ex 2 -1 /r ) are given by Enzm ex -1/n , whe1~e n is the principal quantum number, and the corresponding wave functions are given by Wnzm , where l is t he orbital angular moment um quant um number and m is the magnetic quantum number. The spin of t he electron is not considered . Which of the [NET June 2011] following is a correct stat ement? (a) There are exactly (2l + 1) different wave functions Wnzm, for each Enlm· (b) There are l ( l + l ) different wave functions wnlm, for each Enlm · (c) Enzm does not depend on l and m for t he Coulomb potent ial. [email protected]

@Sk J ahiruddin , 2020

15

Physicsguide

3D Systems

11 oo : oo : 43 15

[email protected]

Physicsguide

@Sk J ahiruddin, 2020

3D Systems

(d) There is a unique wave funct ion

for each

W nzm

E nlm ·

Prob 1.33. Suppose t hat the Coulomb potential of t he

hydrogen atom is changed by adding an inverse-square term 2

Ze

g

such t hat t he total potential is V (7) = - - + 2 , where g r r is a constant. The energy eigenvalues Er,,lm in the modified potential [NET June 2016] (a) depend on n,l but not on m (b) dep end on n but not on l and m (c) dep end on n and m, but not on l (d) depend explicitly on all t hree quantum numbers n,l and m

Prob 1.34. Given that

tainty

t)..p1•

c5 - in c5r

p

1•

+ - , the r

uncer-

in the ground state

[NET June 2014]

of the hydrogen atom is

(a) rt ao

1

(b) ,/'in a0

[email protected]

2/i

()-

c 2a0

16

(d) ao

Physicsguide

11 oo : oo : 4s

j [email protected]

Physicsguide

16

@Sk J ahiruddin , 2020

1.4

3D Systems

Ans Keys

1.1. Standard

1.12. b

1.24. d

Theory

1.13. d

1.25.

C

1.2.

1.14. d

1.26.

C

C

1.3. d 1.4.

1.15. b

C

1.16. a

1.5. a

1.27. d 1.28. a

1.17. a 1.29. b

1.6. a

1.18. b

1.7. b

1.19. d

1.8. a

1.20. d

1.30. d 1.31. b

1.9. a

1.21. d

1.32.

1.10. 2.5

1.22. a

1.33. b

1.11.

1.23. b

1.34. a

C

C

11 oo : oo : 48

[email protected]

Physicsguide

17

@Sk J ahiruddin, 2020

1.5

3D Systems

Solutions

Sol 1.1. This is theory. Can be found in any Quantum

book like Griffiths and Zettili Sol 1.2. This is also the basic concepts . No quantum system has ZERO ground stat e energ)'· Kinetic energy, and

t hus velocity of a quantum particle can't be zero as the zero velocity will precisely determine the momentum which will violate uncertainty principle. Thus every quantum particle will have some velocity at every quantum st ate. Sol 1.3. For the cubic box potential, energy expression is 2 2

E nxnynz

n 1r 2 2) ( 2 = 2m £ 2 nx + ny + n z

Wave function 8 • Sln a3

n x 7r --X

a

. Sln

n y1r

a y



Sln

a

To make 14 in numerator of the energy, the n' s should be 3, 2, 1 not necessarily in the same order. Only option (d) matches t his requirement . Sol 1.4. Following the same argument from previous problem , then' swill be 2, 2, 1, not necessarily in the same order.

11 oo : oo : so 3, 2, 1 not necessarily in the same order. Only option (d) matches this requirement . Sol 1.4. Following the same argument from previous problem, then' swill be 2, 2, 1, not necessarily in the same order.

There could be three combination of the eigenvalues 2, 2, 1, 2, 1, 2 and 1, 2, 2. Thus t here would be three eigenvalues. [email protected]

@Sk J ahiruddin ,

Physicsguide

18

2020

3D Systems

Sol 1.5. Ground state eigenvalue is for 1, 1, 1 31r2 n2

E111 = - -2 = Eo 2m L

First excited state

61r2n2

E 211 = - -2 = 2Eo 2mL Sol 1.6. Solving this from scratch is difficult in t he exami-

nation hall. I have tried to solve by using uncertainty principle but could to get the exact answer. If you can solve then please inform me. Sol 1. 7. When the potential is

1

w

2 2

x

,

the energy is

1

nw.

2 2 2 2 So when potent ial 2w y , you can think t hat w is trans1 formed into 2w. So t he energy is !i2w . And when potential 2 2

8w z

2

you can think t hat w is t ransformed into 4w. So the 1 energy is n4w. So the total energy will be ,

2

Sol 1.8. The EnerQ'v exoression for 3-dimensional isotrooic

11 oo :oo : s2 energy is

2

fi4w. So the total energy will be 1

n(w

2

1

+ 2w + 4w) = 2n7w

Sol 1. 8. The Energy expression for 3-dimensional isotropic quantum harmonic oscillator is fiw= j [email protected]

3 n +2

19

Physicsguide

@Sk J ahiruddin, 2020

3D Systems

The expression to find t he degeneracy for 3-d isotropic quantum harmonic oscillator is

+ l)(n + 2) Degeneracy = - - - - - (n

2

For the third energy level we have n = 2. (2 + 1)(2 + 2) Thus Degeneracy = - - - - - = 6 2

1

1

0

2

1

0

1

0 2

1

1

2 2

0 2

0 0 2

0 0

0

2 2 2

Sol 1.9. The degeneracy of n th level of 3D isotropic har-

monic oscillator is

11 oo : oo : 55 2

0

0

2

Sol 1.9. The degeneracy of n th level of 3D isotropic har-

monic oscillator is

1 9n = (n 2

+ l ) (n + 2)

better to remeber t his result. Proved in Zettili. Another result which can b e useful for furt her problems is t he degeneracy of Hydrogen atom which is n 2

[email protected]

20

P hysicsguide

@Sk J ahiruddin, 2020

3D Systems

Sol 1.10. There are many ways to solve this problem. One is direct integration. Another is breaking the r 2 into

r2

= x2 + y2 + z 2

and then use op erator relations a and at . Finally t he easiest way is to use virial theorem If there is a cent ral force rn so V(r) ex rn+l t hen

n+l T = - -V 2 Here n

=

l. So -

-

- E V =-

-

T = V· ' Energy of 3D isotropic oscillator is E=

3 n+2

Energy in first excited state is

2

f (r)

ex

11 oo : oo : sa Energy of 3D isotropic oscillator is

Energy in first excited state is

E= V

=

~nw 2

1 ') 2 -mw--(r ) 2

=

5 -nw 2

(r2) = 5 Ii 2mw Ii Hence in the unit of - , the ans is ~ mw j [email protected]

21

Physicsguide

@Sk J a hiruddin , 2020

3D Systems

Sol 1.11. We need to do a scaling in the integration to t he expectation value of kinetic energy 00

(T) =

'lj;* (r,t)

2m When we change t he wave function, t he integration becomes 0

n,2

00

(T ) =

'lj;* (ar, t )

2m

0

Now put ar

= r'

The Laplacian changes to

Hence t he Kinetic energy integration

11 oo : 01 : oo The Laplacian changes to

Hence t he Kinetic energy integration 2

00

(T) = a a3 1 O'.

n2

'lj)* (r' ,t )

2

0

2m

00

fi2

'lj)* (r' ,t)

"\1 'ljJ (r', t ) r' dr' 2

2

V 'lj; (r' , t) r' dr'

2m

0

2

(T )

Sol 1.12. In the chapter of Angular Momentum we have solved t hese type of problems. But for the sake of complet eness we will solve it here also. We will use the process 22

[email protected]

Physicsguide

@Sk J ahiruddin , 2020

3D Systems

of elimination to solve the problem. We know that

LW = ...

Jl (l + l)nw

Lz l, m) =

mn

l, m)

We will use these relations to check which operator.

w is

an eigenstate of

l

11 oo : 01

: 03

-

5

1

2

= n, [(0 5

X

1)2'lf 200 - (1

X

2)3'lf 211

+(1 X 2)V7'lf210 - (1 X 2)V5\Jl21-1

= ~ n [-3'11211 + v'7'11210 # alw) 2

Lz'Y =

=

1 Lz[2'11200 - 3'11211 + \/7'11 210 - V5'1121- 1] 5 1 - n, 0 X 2\Jf 200 - 1 X 3'11211 + 0 X V7\Jl210 5

-(- 1)

X

V5\Jl21-1

2 2 = n, [-3\JI 211 5 # /3 \JI)

~

+ V 5 \JI 21- 1]

[email protected]

@Sk J ahiruddin,

V5'1121-1]

23

2020

Thus we see that

Physicsguide

3D Systems

w is neither the eigenstat e of L 2 nor of Lz.

This eliminates all t he options except option b.

Sol 1.13. Follows from t he wave function. Given in every book. Sol 1.14. This is not easy. You are asked to find the probability within a radius. Not t he probability density. So the probability P vvhich was plotted is

1 r.:::

1

3/ 2 (

r 2 - - exp ( - Ar

2

dr

11 oo : 01

: 06

a i ity wit in a ra ius. Not t e pro probability P which was plotted is

1

2

3/ 2

2- r

ao

0

ns1 y.

ao

exp

r -2a0

dr

This will increase as A increases. Between the option (b) and (d) t he option (d) is easily chosen seeing t he Gaussian nature of t he function. Sol 1.15. This problem is different than t he previous one.

Here probability yo find within a range is asked. Here P is

r+dr

1

r 2- -

ao

r

exp

r 2a0

2

dr

Now you need to look carefully at the graphs. There is no t rick, I guess! j [email protected]

24

@Sk J ahiruddin , 2020

P hysicsguide

3D Systems

Sol 1.16.

. The value Lz is zero on t he ground state where L = 0. Spin is not zero as S = 1/ 2. All the other t hree operators involves spin. Sol 1.17. The probability density 1

11 oo :01 : oa involves spin .

Sol 1.17. T he prob ability density

1P =

e- r/a

1

~ 2 2 P = 'ljJ 41rr dr =

4

- e- 2rfar 2 dr

p(r)dr

=

a3

4

p(r) = -r2e-2r/a a3

Take t he derivative and m ake it to ZERO

dp = 4 2re-2r/a dr a3 =

+ r2

a

8r e-2r I a 1 - r a3

Hence t he ans is r

- 2 e-2r/a =

0

a

=a

Sol 1.18. Sam e problem as previous.

25

[email protected]

Physicsguide

@Sk J ahiruddin, 2020

3D Systems

Sol 1.19. CX)

Q

-0

21r

7f

2 'lj}* Q'lj}r dr

A

sin 0 d0 0

0

CX)

21r

7f

'lj}* (z 2 0

dcp

-

r 2 )'lj; r 2 dr

sin 0 d0 0

CX)

2 2 2 2 'lj}*(r cos 0 - r )'lj} r dr A

d, < O"y > and < O"z > [NET June 2018] (a) only < O" x > changes with time. (b) only < a y > changes with time. (c) only < a z > changes with t ime. (d) all three changes with time.

B

1.5

Expectation Values

Prob 1.35. A hydrogen atom is in t he state 8 -

21

[email protected]

@Sk J ahiruddin , 2020

'l'200 -

12

Physicsguide

Angular Momentum

where n , l , m in 'Ynzm denote the principal, orbit and magnetic quantum numbers, respectively. If L is the angular momentum operator, the average value of £ 2 is __ /i2 [GATE 2014] -+

11 oo : oo : 37 w ere n, , m 1n nlm enote t e pr1nc1pa , or it an mag-+ netic quantum numbers, respectively. If L is the angular momentum operator , the average value of £ 2 is __ /i2 [GATE 2014]

Prob 1.36. If L+ and L_ are the angular momentum ladder operators then the expectation value of (L+L- + L_L+) in t he state ll = 1, m = 1) of an atom is __li [GATE 2014] Prob 1.37. Let ll, m) be the simultaneous eigestates of £ 2 and Lz. Here L is the angular momentum operator with Cartesian components (Lx, Ly, Lz), l is the angular momentum quantum number and m is t he azimuthal quant um number .The value of (1, OI(Lx + iLy) 1, - 1) is [GATE 2016] (a) 0 (b) n (c) v2li (d) ~n -+

Prob 1.38. A spin-I particle is in a stat e w) described 2

J2

in the Bz basis. What is

2i t he probability t hat a measurement of operator S z will yield t he result n for the state Bx w)? [JEST 2016] 1

(a) -

2

1

(b) 3

j [email protected]

@Sk J ahiruddin , 2020

1

(c) -

4

1

(d) 6

13

Physicsguide

Angular Niomentum

where t) and t) are the eigenstates of Sz operator. The P.xn P.) =

1

v3 I1' -

1) + I1' 0) ei"!

3

+ I1'

2 3 1) e i" /

where, S, M 8 ) denote t he spin eigenst ates wit h eigenvalues n2 S(S + l ) and nMs respectively. Find (Sx), i.e. t he expectation value of x component of t he spin. [TIFR 2018] Prob 1.41. If s1 and s2 are the spin operators of the two elect rons of a He atom , t he value of (s1 .s2) for the ground state is [GATE 2016]

(a) - 3 !i,2 2

(b) - 3 !i,2 4

(

c) 0

(d )

! !i,2 4

Prob 1.42. The wa,re function of a st ate of t he hydrogen atom is given by \ll = W200 + 2 W 211 + 3 W210 + v'2W21- 1 where Wnlm is t he normalized eigen funct ion of the st ate wit h quant um number n , l and m in t he usual notation. T he expectation value of L z in t he state W is [NET Dec 2012] (a)15n/ 16 (b)lln/ 16 (c)3n/8 (d)n/8 [email protected]

14

@Sk J ahiruddin, 2020

Prob 1.43. Let

Physicsguide

Angular Momentum

W nzm

denote t he eigenfunct ions of a Hamil-

11 oo : oo : 42

@Sk Jahiruddin , 2020

Angular JVIomentum

Prob 1.43. Let W nzm denote the eigenfunctions of a Hamiltonian for a spherically symmetric potential V (r). The expectation value of Lz in the state [NET Dec 2013] 1 w = 6 [w200

+ -Jsw210 + v'law21- 1 + v'20w211]



IS

(a) - 5!i 18

(b)

5n 6

(c)

(d) 5!i 18

n

Prob 1.44. Electron in a given system of hydrogen atoms are described by t he wave function •

'l7r

\J!(r, 0, cp) = 0.8\J!100 + 0.6e 3 W311 where t he \JI nlm denoted normalized energy eigenstates. angular moIf (Lx, Ly, Lz) are the components of t he orbital ,... mentum operator, t he expectation value of L~in t his system is [TIFR 2017] (a) l.5!i2 (b)0.36!i2 (c) 0.18!i2 (d)zero Prob 1.45. If \JI nlm denotes t he eigenfunction of t he Hamiltonian with a potential V = V(r) t hen the expectation value of the operator L ; + L~ in t he state 1 \JI = [3\Jf 211 + W210 - v115w21-1] 5

[NET June 2013] 2 2 (c) 2!i ( d) 26!i / 25

is 2 (a) 39!i / 25 [email protected]

@Sk J ahiruddin , 2020

2

(b) 13!i / 25 15

Physicsguide

Angular Momentum

11 oo : oo : 4s 15

[email protected]

Physicsguide

@Sk J a hiruddin , 2020

A ngular Moment um

Prob 1.46. An electron in a hydrogen atom is in a state described by t he wavefunct ion:

where 'lfJnPm (x) denotes a normalized wavefunction of the hydrogen atom with the principal quantum number n, angular quantum number £ and magnetic quantum number m. Neglect ing the spin-orbit interaction, t he expectation values of Lz and £2 for this state are [TIFR 2019]

(a) 10 ' 5 2 3n 9n (c) 5' 10

-

25

8n 3n (d) 10 '

2

5

Prob 1.47. (Data fo1· next two questions)

i) In a system consisting of two spin 1/ 2 particles labeled 1 and 2 let §( 1) = ~5(1 ) and § (2) = ~5(2) denote t he corre' 2 2 sponding spin operators. Here 5 (ax, ay, az) and ax, ay, az are t he three Pauli matrices. In the standard basis the ma1 2 1 2 ) ) trices for the operators ) and ) are respectively, [NET June 2011] 2 t;,2 1 0 n -1 0 (a) 4 0 1 0 - 1 ' 4

si si

(b) n,2 4

i

O

0 -i

j [email protected]

2

ri ' 4

si si

-i 0

0



1,

16

Physicsguide

11 oo:oo:47 2

(b) li

2

O

i

0 -i

4

-i 0 0 i

li ' 4

j [email protected]

16

P hysicsguide

@Sk J ahiruddin , 2020

Angular Niomentum



0 0 0 -1, 0 0 1, 0 . 0 -1, 0 0 . 1, 0 0 0

fi2

(c) 4

0 1 0 0 1 0 0 0 0 0 0 - 1, 0 0 'l 0

(d) n,2



4

0 0 0 0 0 - 1, . 0 1, 0 . 1, 0 0

n,2



0

' 4



-1,



' 4

n,2



0 0 0



0 0 'l 0 0 0 0 0 0 1 0 0 1 0 -1,



ii) These two op erat ors satisfy t he relation

(a) {s1) st 1

2

),

st ) si 1

2

)} =

si ) si ) 1

2

(b) { s1) st

2

1

),

s~)s1) } = 1

2

0

(c)

[si )si ), st )sl 1

2

1

2

)J =

Prob 1.48. If Y xy

iSi )si ) 1

2

(d)

[sl )st ),st )si 1

2

1

2

)J =

o

1

= v'2 ( Y 2,2- Y 2,-2) where Yt ,m are spher-

ical harmonics then which of t he following is t ru e? [JEST

2016] (a) Y x y is an eigenfunction of both L 2 and Lz (b ) Y x y is an eigen function of L 2 b ut not Lz (c) Yxy is an eigenfunction of both L z but not L 2 (d) Y x y is not an eigenfunction of eit her L 2 and Lz

P rob 1.49 . Let W nlm denote t he eigenfunct ion of a Hamilt onian for a sph erically symmet ric p otent ial V (r). The wave1 function W = - ['11 210 + v'5'11 21 _ 1 + ,Jiow 211 ] is an eigenfunc4

t ion on ly of

[N ET June 2012]

11 oo : oo : so Prob 1.49. Let Wnzrri denote t he eigenfunction of a Hamiltonian for a spherically symmetric p otential V (r). The wave1 function W = -['11210 + \ibW21- 1 + vtiOw211] is an eigenfunc4 t ion only of [NET June 2012] [email protected]

Physicsguide

17

@Sk J ahiruddin, 2020

Angular Momentum

2

(a) H , L and L z (c) H and L 2

(b) H and L z (d) L 2 and L z

Pro b 1. 50. Consider the normalized wavefunction (3k - w

2

m)

2

-

2

T =0 k

2

=0

Solving this equation we will have Wx

=

4k

2k

m

m

- and Wy

Now the expression for energy will be

1 n Y +2 The ground state energy will be

n

Eo = 2

n 2k -m + -2 -m + VO 4k

TT

11 oo : 02 : 16 The ground state energy will be

n

Eo = 2

n 2k -m + -2 -+Vo m 4k

The first excited state energy will be 4k

3n

m+2

2k

l

-+ vo m ,T

Note :Be careful to choose the first excited state. Thus the difference between the two energies will be

2k m j ahir@physicsguide. in

49

@Sk J ahiruddin , 2020

Physicsguide

Approximation Methods: part-1

Sol 1.36. This is a problem of Stark Effect. This problem has been explicitly solved and explained in Quantum Mechanics Concepts and Applications(2 Ed.) by Nouredine Zettili (page 494, Example 9.2) Sol 1.37. Sol 1.38. The wave function is given in spherical coordinate system. So we have to convert the perturbing potential into spherical co-ordinates. Put x = r sin 0 cos cp 2 2 2 We have H' = br sin 0 cos cp The first order correction to the ground state energy is 1

E6) = ('l/Jo IH' 'l/Jo) =

r1/,!

T-l 11/,,.,r 2 Q1n {}rl{}rlrf,

11 oo : 02 : 19 We have H'

2

2

2

= br sin 0 cos cp

The first order correction to the ground state energy is

Ea = (wo lH' wo) 1

)

2

'i/;0H''i/Jor sin 0d0dcp 00

b

4

r exp

7f

- 2r

cos 2 cpd

sin3 0d0

dr

1ra~ 0 ao b 3a 5 4 -X OX - X 7r 1ra5 4 3

27f

0

0

= ba5 We have not solved t he integration explicitly hoping that you will be able to solve t his simple integration on your own. [email protected]

Physicsguide

50

@Sk J ahiruddin, 2020

Approximation Methods: part-1

Sol 1.39. Int egration 00

o

1 / 2r 1 ; - - e- r ao E cos - - - - e - r a 0 dr

v,ia,3

ao v,ia,3

Better to write the cos as exp and tal a

1/ 3

3

(c) -

4

n

2g-?

1/ 3

m

1/3

I I,

m

Prob 1.5. T he ground state energy of a particle of m ass m

ized trial wavefunction 'ljJ (x)

=

a

1/ 4

2

e-ax /2 IS

'

7r

June 2016] Use and

(a) 3

2m

n,2131 / 3

a

00

1r

- oo

a

00

7r

-oo

2 2 x e - ax

dx

1

=-

2a

2 4 x e-ax

(b) 8 n2131/3 3m



dx

=-

3

4a2

[NET

11 oo : oo : 13 and 3 (a) n,2(31/3 2m

2 (c) n,2(31/3 3m

Q

00

7r

-oo

4

2

x e-ax dx

=-

3

4a2

8 (b) n2 (31/3 3m 3 (d) n2 (31/3 8m

Prob 1.6. Using t he t rial function j [email protected]

Physicsguide

5

Approximation Methods: part-2

@Sk J ahiruddin, 2020

-a< x < a

\J!(x) =

otherwise the ground state energy of a one-dimensional harmonic oscillator is ~ [NET June 2017]

(a)

nw

(b)

5

nw 14

(c)

!tuu 2

5

nw 7

(d)

Prob 1. 7. Given a part icle confined in one-dimensional box between x = -a and x = +a, a student attempts to find

t he ground state by assuming a wave-function

(x) =

A(a 0

2

-

2 3 2 x ) 1

for Ix < a for Ix > a

The ground state energy Erri is estimated by calculating t he exp ectation value of energy wit h this trial wave-function. If E 0 is the true ground state energy, what is the ratio

Em/ Ea?

[TIFR 2018]

11 oo : oo : 16 The ground state energy Em is estimated by calculating t he expectation value of energy with this trial wave-function. If E 0 is the true ground state energy, what is t he ratio

Em/ Eo?

1.2

[TIFR 2018]

WKB Approximation

Prob 1. 8. Consider a particle confined by a potential V (x) = klx , where k is a positive constant . The spectrum E n of the [email protected]

Physicsguide

6

@Sk J ahiruddin, 2020

Approximat ion Methods: part-2

system , within the WKB approximation is proportional to [JEST 2017] (a) (n + ;)3/2 (b) (n + ;)2/3

(c) (n + ½)1/2

(d) (n

+ ½) 4/3

Prob 1.9. Consider a part icle of mass m in t he potent ial V (x ) = alx, a> 0. The energy eigenvalues E n(n = 0, 1, 2, .. ) in t he WKB approximation, are, [NET Dec 2014] [NET June 2016] ~ 1/3 ~ 2/3 3a,"1r ( 3a,"1r ( 1) 1) (b) 4-J2rn n + 2 (a) 4-J2m n + 2 4/ 3

3an1r ( 1.) (C) ~ n+ 2 4v 2m

Prob 1.10. A one-dimensional system is described by t he 2

;m x

Hamiltonian H = + >. (where >. > O) The ground stat e energy varies as a funct ion of ). as [NET Dec 2018]

11 oo : oo : 19

Prob 1.10. A one-dimensional system is described by t he 2

Hamiltonian H =

;m + >. x (where >. >

0) The ground

stat e energy varies as a function of A as [NET Dec 2018]

(a)

,\5/ 3

(c)

,\4/ 3

(b)

,\2/3

Prob 1.11. A particle in one dimension moves under t he influence of a potential V ( x) = ax 6 , where a is a real constant. For large n the quantized energy level En depends on

n as:

[NET June 2011] [NET Dec 201 7]

[email protected]

Physicsguide

7

@Sk J ahiruddin , 2020

Approximation Methods: part-2

(b) En rv n 413

(a) En, rv n 3

(c) En rv n 615

(d) En rv n 312

Prob 1.12. The n-th energy eigenvalue En of a one-dimensional 2

Hamiltonian H = p + ;\x 4 ( where ,\ > 0 is a constant) in 2m t he WKB approximation, is proport ional to [NET June

2018]

(a)

(c)

1.3

1

4/3 ,\ 1/ 3

n+-

2

1

n+ -

2

(b)

1 n+2

4/ 3

1 2

4/3

5/ 3 ,\ 1/3

(d)

n+

_\2/ 3

,\ 1/ 3

Time D ependent Perturbation Theory

Prob 1.13. A particle of charge q in one dimension is in a

11 oo : oo : 21 C

1.3

n+ -

n+ -

2

2

Time Dependent Perturbation Theory

Prob 1.13. A particle of charge q in one dimension is in a

simple harmonic potential with angular frequency w . It is 2 7 subjected to a time-dependent electric field E (t) = A e- (t/ ) where A and T are positive constants and WT > > l. If in the distant past t -+ -oo the particle was in its ground state, t he probability t hat it will be in the first excited state as t -+ +oo is proportional to [NET Dec 2016]

2 (a) exp - ~(wT)

2 (b) exp ~(wT)

j [email protected]

(c) 0

(d)

Physicsguide

8

@Sk J ahiruddin , 2020

Approximation Methods: part-2

Prob 1.14. Consider t he Hamiltonian

1 0 0

H (t) = a

O 2 0

0 0 3

+ {3t

0 0 1 O O 0 1 0 -2

The time dependent function f3 (t) = a for t < 0 and zero fort > 0. Find ( (w(t < 0) w(t > 0)) )2 , where w(t < 0)) is the normalised ground state of t he syst em at a t ime t < 0 and l'll (t < 0)) is t he state of the syst em at t > 0. [JEST 2017] 1 (~1 -( l +rn~(?.rvf11

11 oo : oo : 24 fort> 0. Find ( (w(t < 0) w(t > 0))1) 2, where w(t < 0)) is t he normalised ground state of t he syst em at a t ime t < 0 and l'11 (t < 0)) is t he stat e of the syst em at t > 0. 2017] 1 1 (a) (1 +cos(2at)) (b) (1+cos( at)) 2 2 1 . (d) (1 + sin (at))

[JEST

2

Prob 1.15. In t he usual notation n l m) for the states of a hydrogen like atom, consider the spontaneous t ransitions 2 1 O) ➔ 1 0 0) and 3 1 0) ➔ 1 0 0). If t 1 and t2 are t he lifetimes of the first and the second decaying st at es

!~

[NET

respectively, then t he ratio is proport ional to June 2017] 3 3 2 3 3 3 32 27 (d) (a) 27 (b) 32 (c) 2 3 [email protected]

@Sk J ahiruddin,

1.4

Physicsguide

9

2020

Approximation Methods: part-2

Sudden Expansion

Prob 1.16. A particle of mass m is contained in a onedimensional infinite well extending from x == -L/ 2 to x = L/ 2. The particle is in its ground state given by o(x) == 2/ L cos(1rx / L ). The walls of the box are moved suddenly t o from a box extending from x==-L to x=L. What is the probability that t he particle will be in the ground state after t his sudden expansion? [JEST 2013]

(a) (8/ 31r)

2

(b) 0

(c) (16 / 31r)

2

( d)

(4/ 31r )

2

11 oo : oo : 21 probability that t he particle will be in the ground state after t his sudden expansion? [JEST 2013] 2

(a) (8/31r)

(b) 0

(c) (16/31r)

2

(d) (4/ 31r)

2

Prob 1.1 7. A particle is confined to a one-dimensional box of length L . If a vanishingly thin but strongly repulsive partition is introduced in the exact center of the box, and the particle is allowed to come to its ground state, then the probability density for finding the particle will appear as [TIFR 2014]

j [email protected]

10

Physicsguide

@Sk J ahiruddin , 2020

Ap proximation Methods: part-2

N

8

(b)

;l>-

X

(c)

X

11 oo : oo : 29 X

X

X

X

(c)

Prob 1.18. A particle is in t he ground state of a cubical box of side l . Suddenly one side of t he box changes from l

to 4l. If p is t he probability of finding t he particle in t he [TIFR ground state of the new box, what is l OOOp ? 2018] Prob 1.19. Consider a quant um part icle in a one-dimensional

box of length L. The coordinates of the leftmost wall of t he box is at x = 0 and that of the rightmost wall is at x = L. The particle is in t he ground state at t = 0. At t = 0, we suddenly change the lengt h of t he box to 3L by moving the [email protected]

@Sk J ahiruddin , 2020

11

P hysicsguide

Approximation 1\/fethods: part-2

right wall. What is the probability that the part icle is in t he ground state of the new system immediately after the change? [JEST 2019] 81 9 05 (a) 0.36 (b) (c) (d) · 8n 64n 2 n

1.5

Scattering

11 oo : oo : 32 JEST 2019

change?

(a) 0.36

1.5

(b) 9

81r

81 (c) 641r2

(d) 0.5 7r

Scattering

Prob 1.20. Consider an elastic scatt ering of particles in l = 0 states. If t he corresponding phase shift 80 is 90° and the magnitude of the incident wave vector is equal to v12n" fm- 1 t hen the total scattering cross section in units of fm 2 is __ .

[GATE 2016] Prob 1.21. A phase shift of 30° is observed when a beam of

particles of energy O. l MeV is scattered by a target. When t he beam energy is changed , t he observed phase shift is 60° Assuming that only s-wave scatt ering is relevant and that t he cross-section does not change with energy, t he beam en[NET Dec 2017] ergy is (a) 0.4MeV (b) 0.3MeV (c) 0.2MeV (d) 0.15MeV Prob 1.22. The scattering of particles by a potential can be analyzed by Born approximation. In particular , if the scatt ered wave is replaced by an appropriate plane wave, [email protected]

@Sk J ahiruddin , 2020

12

Physicsguide

Approximation Methods: part-2

t he corresponding Born approximation is known as the first Born approximation. Such an approximation is valid for [GATE 2016] (a) large incident energies and weak scattering potentials. (b) large incident energies and strong scattering p otentials.

11 oo : oo : 34 t e correspon 1ng orn approx1mat1on 1s nown as t e rst Born approximation. Such an approximation is valid for [GATE 2016] (a) large incident energies and weak scattering potentials.

(b) large incident energies and strong scattering potentials. (c) small incident energies and weak scattering potentials. (d) small incident energies and strong scattering potentials.

Prob 1.23. A free particle is described by a plane wave and moving in t he positive z-direction undergoes scattering by a potential

V(r ) =

Vo ,

if r < R

0,

if r > R

(1.1)

If Vo is changed to 2Vo, keeping R fixed, then the different ial scatt ering cross-section , in t he born approximation [NET June 2012] (a) increases to four t imes t he original value (b) increases to twice the original value (c) decreases to half t he original value (d) decreases to one fourth t he original value

Prob 1.24. In the Born approximation, the scattering amplitude f (0) for the Yukawa potential

j [email protected]

13

Physicsguide

@Sk J ahiruddin, 2020

Approximation Methods: part-2

V (r) = f3e - µr r

00: 00: 36 pproximation Methods: part-2

V (r) = f3e - µr r

is given by :(in the following b

= 2k sin ~, E = n2 k 2 /2m) [NET June 2013]

2m/3 (b) -

n2(µ2 + b2)

2m/3 (d) -

n2(µ2 + b2)3

Prob 1.25. The scattering amplit ude f (0) for the potential V(r) = f3e - µr, where /3 and µ are positive constants, is given , in the Born approximation , by [NET June 2014] 0 n2 k 2 (in the following b = 2k sin and E = m) 2 2 4mf3 µ 4mf3 µ (a) - n 2(b2 + µ2)2 (b) -n2b2(b2+µ2) µ 4m/3 () c - n2J(b2 + µ2)

(d)

4m/3 µ - n 2( b2 + µ2)3

Prob 1.26. A particle of energy E scatters off a repulsive spherical potent ial

V (r) =

Vo for r < a 0

where

Vo

for r > a

and a are positive constants. In the low energy

[email protected]

@Sk J ahiruddin, 2020

Physicsguide

14

Approximation Methods: part-2

limit , the total scattering cross-section is O" = 41ra

2

1 (k a

tanh ka -

11 oo : oo : 42 15

[email protected]

Physicsguide

@Sk J ahiruddin , 2020

(a) 16a

2

64a

2

(c)

mV0 a

Approximation Methods: part-2

2

2

mVoa

(b) 16a

fi2

fi2

7r2

mVoa

2

2

(d) 64a

fi2

2

2

2

m V0 a

2

fi2

7r2

Prob 1.29. The differential scattering cross-section : ; for

t he cent ral potential V (r) = (3 e - µr, where (3 and µ are posir tive constants, is calculated in t he first Born approximation. Its dependence on the scattering angle 0 is proportional to (A is a constant below. ) [NET June 2018] A

(a)

(c)

A

2

2

+ sin

+ sin

2

0 -

2

j [email protected]

2

0 -

2

(b)

A

- 2

A

(d)

16

2

2

+ sin

2

0

- l

-

2

+ sin ~

2

2

2

Physicsguide

11 oo : oo : 44

j [email protected]

@Sk J ahiruddin , 2020

1.6

Physicsguide

16

Approximation Methods: part-2

Ans Keys

1.1. d

1.11. d

1.21. b

1.2.

C

1.12. a

1.22. a

1.3.

C

1.13. a

1.23. a

1.4. a

1.14. a

1.5. d

1.15. a

1.6. b 1. 7. Do yourself

1.16. a 1.17. a

1.24. b 1.25. d 1.26. a

1.8. b

1.18. 58

1.27. a

1.9. b

1.19.

C

1.28.

C

1.10. b

1.20. 2

1.29.

C

11 oo:oo:47

[email protected]

@Sk J ahiruddin, 2020

1. 7

Physicsguide

17

Approximation Methods: par t-2

Solutions

Sol 1.1. Almost all the types of Variational approximation problems are solved in the notes. Please go through t hem first .

The Hamiltonian is given by

-n2

d2 H = 2m dx 2

+V

Now we know t hat the potential of the well is V = 0 ranging from x = -a to x = a. As the wave function is normalised then the ground state energy can be calculated as follows

=

-n,2

a

15 (a2 - x2) X 16a5 2m - a a 2 15n 2 2 x ) dx (a 16ma5 -a 4a 3 15n2 5n2 X l6ma5 4ma2 3

d2 dx 2

(a2 - x2)

dx

Sol 1.2. In problem number Prob 1.1. , we did not find the minimum energy t he ground state can attain. We just learnt

11 oo : oo : so 2 2 3 15!i 4a 5!i =---X-=-l6m a5 3 4ma 2

Sol 1.2. In problem number Prob 1.1. , we did not find the minimum energy t he ground state can attain. We just learnt how to find t he epression of energy of t he ground state. In t his problem we will minimise t he expression of energy for a [email protected]

18

P hysicsguide

@Sk J ahiruddin , 2020

Approximation Methods: part-2

given parameter and will try to derive the minimum energy of the ground state. At first we have to normalise t he wave function 00

exp(-2bx

2

)

dx = 1

-oo

2b Now t he expression for ground state energy E

= ('11 H '11) =

A

- !i2

2

X

2m

d2

CX)

- 2 exp(-bx dx

- oo

2

)

dx

00

2

exp(-2bx )6(x) dx -oo 2 2

-4b !i =A x 2m

00

2

2

_00

x exp(-2bx

2

)

2

dx - aA x 1

-4b2!i2 (2b)- 3/ 2 ~ -x x---- - a 2m 2 7r b!i2 2b = -- a 2m 7r 2b

., ,

. . .

.,

.

2b

.

.

,

11 oo : oo : s2 2b

-

X

1r

2

bn

= --a

-4b2 n2 --2m 2b

X

(2b)- 3/ 2 ~ ----- - a 2

2b

2m

Now we will minimise the expression against the parameter

j [email protected]

Physicsguide

19

@Sk J ahiruddin , 2020

Approximation Methods: part-2

"b,, dE db

n2

a 2 - - --X-= 0 2m 2vb 1r

Then, the minimum energy the ground stat e can att ain can be calculated by putting t he value of parameter ''b'' n,2 2a2m2 E =-x -a 2m nn4

2 7r X

am fi2

2 X

7r

a 2m

E =-1rn2

Sol 1.3. Same as previous problem Sol 1.4. We will solve t he problem exactly as in problem number Prob 1.2 ..

The normalising const ant c2 n5

11 oo : oo : 55 Sol 1.4 . We will solve t he problem exactly as in problem number Prob 1.2 .. The normalising constant c2 a5

-a

c=

[email protected]

15 16

20

@Sk J ahiruddin , 2020

Physicsguide

Approximation Methods: part-2

Now t he expression for ground state energy E

= ('11

H '11) c2 -fi2

dx

=-X--

a5

+

2gc a5

2m

-a

2 0

= _15 X _2_n2_

a (a2

- x2) dx

16 2ma 5 - a 2g 15 a 4 • 2 3 5 2 + -5 x (a x - 2a x + x ) dx a 16 0 2 15 n 4a3 5ga -x-x + 16 ma5 3 16 2 5n 5ga 4ma2 + 16

Now we will minimise the expression against the parameter ''a''

11 oo : oo : sa 2

5n 4ma2

+

5ga 16

Now we will minimise the expression against t he parameter ''a'' 2

+ 5g = O

- 5n 2ma3

dE da

16

n,2

a= 2

===::,~>

j [email protected]

1/3

mg

21

Physicsguide

@Sk J ahiruddin , 2020

Approximation Methods: part-2

Then , t he minimum energy t he ground state can attain 5 n,2 5 n,2 E = --+ g x 2 4ma 2 16 mg

15

n2g2

16

m

1/ 3

1; 3

Sol 1. 5. The expression for ground stat e energy

E = ('1! H '1!)

-n2 2m

X

7r

n2/3

+ 6m

a a

00

exp - oo

d2

dx 2 exp

2

00

4

x exp( -ax

X 7r

-ax 2

- 00

2

)

dx

-ax2 2

dx

00 : 01 : 00 - oo

7r

00

4

x exp( -ax 7r

~2

- ,i a

2

)

dx

- 00

2

00

2

x exp( -ax

2m

2

)

- oo

7r

a

00

1r

- oo

exp(-ax

2

)

dx

2 n /3

3 dx + x 2 6m 4a

n a n a n /3 = --+-+-2

4m n2a

2

2

2m

8ma2

n,2/3

=4m - +8ma - -2 Now we will minimise the expression against the parameter 22

[email protected]

Physicsguide

@Sk J ahiruddin, 2020

Approximation 1\/fethods: part-2

''a'' dE

n -

2

da 4m ~~► a = 131/3

n /3 ---= 0 2

-

4ma 3

Then, t he minimum energy the ground state can attain n,2131/3

n,2/3

4m

8mf3 1

E = -- +-2 3 = 3 n,2131/3 8m

Sol 1.6. The normalising constant

11 oo : 01

: 03

= 3 ri2131/3 8m

Sol 1. 6. The normalising constant a

A

2

(a

2

2 2 x )

-

dx

=l

-a

15

A=

l6a5

23

jahir@physicsguide. in

Physicsguide

@Sk J a hiruddin , 2020

Approximation Methods: part-2

Now t he expression for ground state energy

E = ('11 H IW)

= A2

X -n,2

2m 2mw 2A2 +--2

dx -a a

x2 ( a 2

-

x 2 ) 2 dx

0

2

5n

15

2

- - +mw x - -5 4ma2 l6a 5n2 mw 2 a 2 =--+-4ma2 14 l\ T

• 11







a

I

1

4 2

(a x

-

2 4

2a x

+

0





I

I

1

6 2 x )

dx

11 oo : 01 : os 5n

2

4ma 2

+ mw

2

x

a

15

4 2

(a x

16a 5

+

2 4

2a x

-

6 2 x )

dx

0

5n2

mw 2a 2 =--+--4ma2 14

Now we will minimise the expression against t he parameter ''a" 2

dE = 5(-2)n

+ mw

4ma3

da ~~> a2

=

a = O

7

112

35

2

n

2

mw

Then, t he minimum energy tl1e ground state can attain

E=

5n2 4m

mw

X

nw

n 2 35

5 2

2

2 35

mw 2

+

1

+7

14

35 2

X

mw

35 2

5 14 j [email protected]

Physicsguide

24

@Sk J ahiruddin , 2020

Approximation Methods: part-2

Sol 1. 7. You can do it yourself. We will just provide you with t he result. The normalising constant a

A

2

(a

2

-

2 3 x )

dx = 1

-a

A2 = 35 32a7

The expression for ground state energy

11 oo :01 : oa A2 = 35 32a7 The expression for ground st at e energy

- fi2

=_

x

35

2m 32a7 2 21fi 16ma2

a

(a2 _ x2)3/2

dx

-a

Now t he ratio 21fi2 16ma 2

21

fi27r

2 1r2

8ma2

Sol 1.8. Please go t hrougth t he WKB notes first. All problems can be done by the the following formula. If t he potent ial varies as

[email protected]

25

@Sk J ahiruddin, 2020

P hysicsguide

Approximation Methods: part-2

Then the energy varies as

En= Ak (n - 1/ 2)fi where n = 1, 2, 3 .. . In t his problem use v = l

2mk

11 oo : 01

: 1o

En = Ak (n - 1/ 2)fi

2mk

where n = 1, 2, 3 ... In t his problem use v = l

Sol 1.9. Use v = l in the general formula as given in the solution of the previous problem Sol 1.10. Use v

= 1 in the general formula as given in the

solution of the previous problem

Sol 1.11. Use v = 6 in the general formula as given in t he solution of the previous problem. Also solved in the notes. Sol 1.12. Use v = 4 in the general formula as given in the solution of the previous problem. Also solved in the notes.

Sol 1.13. Here we have t o calculate t he probability of transition of the particle from the ground state to first excited state. The probability of transition(first order) is given by

l P1; = n,2

t

2

('¥1 V(t') l'¥;)exp(iw1;t

1 )

dt'

0

In the problem we have been provided with the time depen26

[email protected]

Physicsguide

@Sk J ahiruddin , 2020

Approximation Methods: part-2

dent potent ial along with the time for which the particle was subj ected to the potential. Also we are fairly acquainted with the expression of states of a h armonic oscillat or. Now we will calculate the expression of w f i . 1

1

11 oo : 01

: 13

dent potential along with the t ime for which the p article was subjected to the potential. Also we are fairly acquainted with the expression of stat es of a h armonic oscillator. Now we will calculate the expression of w Ji .

3

1

2 =

2

- ru.u - - ru.u

Wji

n

= W

Now t he transition probability Pfi 2

CX)

exp (iwt') dt'

exp - CX)

Now if you look at the options given, you will know t h at we m ust fo cus only on the integration. We will try to arrange the integration as p er our convenience. CX)

I =

- t'2

exp

72

- CX) CX)

exp - 00 CX)

- 1 72

2

t' -

CX)

4

dt'

iwt' 7 + 2

-W2 7 2

exp -

+ iwt'

exp

- 1 , 2

iw7

2

2

iw7

2

t' -

2

2

dt'

2 iW7

2

2

dt'

2

Now t he transition probability CX)

2 j [email protected]

exp - oo

- 1 T

2

27

@Sk J ahiruddin, 2020

.

'lW7

2

t' - - 2

Physicsguide

Approximation Metho ds: part-2

Thus we can say t h at the transit ion probability -W272

Pf i ex exp

2

dt

11 oo : 01

: 16

@Sk J ahiruddin, 2020

Approximation Methods: part-2

Thus we can say t hat t he transition probability -W2T2

Pfi ex exp

2

Sol 1.14. Sol 1.15. Sol 1.16. We have thoroughly discussed the sudden approximation problems in t he notes with examples. Please go t hrough these first. A brief review is as follows. 1) In Sudden Expansion the initial wavefunction would not resp ond to the sudden change and the system will evolve in such a way that t he expression of t he wavefunction will remain same (only the normalising constant will change). Eg : A potential well of length l is changed to length 2l suddenly. Init ially if the syst em was in ground state then aft er expansion t he system will b e in first excited state. Thus to keep the expression of t he wavefunction same (except the normalising constant) t he system changes t he state. 2) In Adiabatic Expansion, t he wavefunction will change. Eg : A potential well of length l is changed to length 2l very slowly. Initially if t he system was in ground state then after expansion t he syst em will be in ground state also. Thus to remain in ground state the expression of the wavefunction [email protected]

@Sk J ahiruddin , 2020

will seem to have changed.

28

Physicsguide

Approximation Methods: part-2

11 oo :01

:1

a

@Sk J ahiruddin, 2020

Approximation Methods: part-2

will seem to have changed. The wavefunction of t he ground state i:I.

:--

,., ~ ! (c)

l

tl,

0 V'X

(d)

0 Vx

Prob 1. 3. Electrons of mass m in a thin long wire at a temperature T follow a one-dimensional Maxwellian velocity didstribution. The most probable speed of these electrons [JEST 2015] is,

(c)O Prob 1.4. A spherical closed container wit h smooth inner wall contains a monatomic ideal gas. If the collisions between the wall and the atos are elastic, then the Maxwell speed-distribution funct ion ( d:Vv) for t he atoms is best rep-

resented by

[JAM 2016]

11 oo : oo : 11 tween the wall and the atos are elastic, then the Maxwell speed-distribution funct ion ( d~v) for the atoms is best represented by [JAM 2016] Prob 1.5. In low density oxygen gas at low temperature, [email protected]

4

physicsguide CSIR NET, GATE

@Sk J ahiruddin, 2020

I(inetic Theory

only the translational and rotational modes of t he molecules are excited. The specific heat per molecule of the gas is [NET Dec 2014] 3 5 (c) -kB (d)-kB (b)kB 2 2 Prob 1.6. The specific heat per molecule of a gas of diatomic molecules at high temperatures is [NET June 2016] (a) 8kB (b) 3.5kB (c) 4.5kB (d) 3kB ,

.

,

.,

• r"'

,

,

I

~

I

~

\

11 oo : oo : 14 Prob 1.6. T he specific heat per molecule of a gas of di-

atomic molecules at high temperatures is 2016] (a) 8kB (b) 3.5kB (c) 4.5kB (d) 3kB

[NET June

Prob 1. 7. For which gas t he ratio of specific heats (Gp/ Gv)

will be t he largest? (a) mono-atomic (b)di- atomic j [email protected]

5

[JEST 2014] (c)tri-atomic (d)hexaphysicsguide CSIR NET, GATE

@Sk J ahiruddin , 2020

Kinetic Theory

atomic Prob 1. 8. Consider t he following linear model of a molecule of hydrogen cyanide (H CN) depicted b elow.

0 --H

--C

N

It follows that t he molar specific heat of hydrogen cyanide gas at constant pressure must be [TIFR 2019]

(a) 6R

(b) 4.5R

(c) 5R

(d) 5.5R

Prob 1.9. The molar specific heat of a gas as given from

t he Kinetic t heory is ~R . If it is not sp ecified whet her it is Gp or Gp , one could conclude that t he molecules of t he gas [JAM 2005] (b) are definitely rigid (a) are definit ely mono atomic diatomic

11 oo : oo : 16 ""'"'ry 1 2 . pec1 Gp or Gp , one could conclude that t he molecules of t he gas [JAM 2005] (b) are definitely rigid

(a) are definitely mono atomic diatomic (c) are definitely non-r·igid diatomic

(d) can be monoatomic

or rigid diatomic

Prob 1.10. A rigid triangular molecule consists of t hree non-colinear atoms joined by rigid rods. The constant pressure molar sp ecific heat (Gp) of an ideal gas consisting of [email protected]

physicsguide CSIR NET, GATE

6

@Sk J ahiruddin, 2020

Kinetic Theory

such molecules is

(a) 6R

(b)5R

[JAM 2015]

(c)4R

(d)3R

Pro b 1.11. The speed v of the molecules of mass mof an ideal gas obeys Mazwell's velocity distribution law at an equilibrium temperature T. Let (Vx, Vy, Vz ) denote the components of t he velocity and kB t he Boltzmann constant. The 2 average value of (avx - /3vy ) , where a and j3 are constants, is [NET Dec 2013]

(a)(a

2

-

2

/3 )kBT /m

(c)(a + /3) kBT /m 2

(b) (a

2

+ /3

2

)kBT/m]

2

(d)(a - j3 ) kBT /m

Prob 1.12. At a given t emperature T , t he average energy per particle of a non-interacting gas of two-dimensional classical harmonic oscillator is ______kBT [GATE 2014] (kB is t he Boltzmann constant) Prob 1.13. A classical gas of molecules each having mass m, is in therm:1.l eo11ili hri11m : -1.t the ahsol11te temn er::i.t 11re T.

11 oo : oo : 19 per part icle of a non-interact ing gas of two-dimensional clas[GATE 2014] sical harmonic oscillator is ______kBT (kB is t he Boltzmann constant)

Prob 1.13. A classical gas of molecules each having mass m is in thermal equilibrium at t he absolute temperature T . The velocity components of the molecules along the Carte2 sian axes are Vx, Vy and V z · The mean value of (V x + Vy) is [GATE 2012] (a) kBT (b) 3k BT (c) kBT (d) 2kBT m 2m 2m m Prob 1.14. A syst em of N non-interacting classical point particles is constrained to move on the two-dimensional [email protected]

7

physicsguide CSIR NET , GATE

@Sk J a hiruddin , 2020

Kinetic Theory

face of a sphere. T he internal energy of t he system is [GATE 2010]

(a) ~NkBT

(b) ~NkBT

(c) NkBT

(d) ~NkBT

Prob 1.1 5 . The mean internal energy of a one-dimensional classical harmonic oscillator in equilibrium wit h a heat bat h [GATE 2006] of t emperature T is (a) ~kBT (b) kBT (c) ~kBT (d) 3k BT Prob 1.16. Two boxes A and B contain an equal number of molecules of the same gas. If t he volumes are VA and VB and AA and AB denote respective mean free paths, t hen [JAM 2018]

11 oo : oo : 21 [JAM 2018]

AA

AB

A

B

(c) vl/2 = vl/2

(d) AAVA = ABVB

Prob 1.17. one mole of an ideal gas with average molecular speed vo is kept in a container of fixed volume. If the temperature of the gas is increased such t hat the average speed gets doubled , then [JAM 2016] (a) the mean free path of t he gas molecule will increase (b )the mean free path of the gas molecule will not change (c)the mean free path of the gas molecule will decrease (d) the collision frequency of the gas molecule with wall of j [email protected]

8

physicsguide CSIR NET 1 GATE

@Sk J ahiruddin, 2020

Kinetic Theory

t he container remains unchanged.

Prob 1.18. Consider a particle diffusing in a liquid contained in a large box. The diffusion constant of the particle 2 2 in t he liquid is 1.0 x 10- cm /s. The minimum t ime after which t he root-mean-squared displacement becomes more [NET June 2018] t han 6 cm is (a) 10 min

(b) 6 min

(c) 30 min

(d)

v'6 min

Prob 1.19. Consider a uniform distribution of particles with volume density n in a box. The particles have an isotropic velocity distribution with constant magnitude v . The rate at which t he particles will be emitted from a hole of area A ·1

r ,,.

1



r Ty-,nrr,

n. n -. nl

11 oo : oo : 24 Prob 1.19. Consider a uniform dist ribut ion of part icles with

volume density n in a box. The part icles have an isotropic velocity dist ribut ion with constant magnit ude v . T he rate at which t he part icles will be emitted from a hole of area A on one side of this box is [JEST 2013] (a)nv A (b)nvA / 2 (c)nv A /4 (d)none of the above Prob 1.20. For a diatomic ideal gas near room t empera-

t ure, what fract ion of t he heat supplied is available for external work if t he gas is expanded at const ant pressure? [JEST 2013] (a) l / 7 (b)5/7 (c) 3/ 4 (d)2/ 7 Prob 1.21. T wo containers are maint ained at t he same t emperature and are filled wit h ideal gases whose molecules

have mass m 1 and m 2 respectively. The mean sp eed of molecules of t he second gas is 10 t imes t he r .m.s. speed [email protected]

9

@Sk J ahiruddin , 2020

physicsguide CSIR NET, GATE

Kinetic Theory

the of t he molecules of t he first gas. Find t he ratio of mi m2 [TIFR 2016] nearest int eger Prob 1.22. A gas of molecules each having mass m is in

t hermal equilibrium at a t emperature T . Let Vx, Vy, V z be t he Cart esian component s of velocity, iJ , of a molucule. T he mean value of (vx - a vy + f3vz) 2 is [JAM 2010] ( ) 2 kBT 2 2 kBT 2 ( a) 1 + a + /3 b 1 - a + /3 m

m

11 oo : oo : 21

m

11

Prob 1.23. A t iny dust p article of m ass 1.4 x 10- kg is floating in air at 300K. Ign oring gravity, its rms sp eed (in µm/ s ) due to random collisions wit h air molecules will b e closest t o [JAM 2012] (a) 0.3 (b) 3 (c) 30 (d) 300

j [email protected]

10

physicsguide CSIR NET I GATE

@Sk J ahiruddin , 2020

1.1

Kinetic Theory

Ans Keys

1. 1.

C

1. 9. d

1. 17. b

1. 2.

C

1. 10.

1. 3.

C

1. 11. b

1. 4.

C

1. 12. 2

C

1. 18. a 1. 19.

C

1. 20. d 1. 5. d 1. 6. b

1. 13. d 1. 14.

C

1. 21. 118

11 oo : oo : 29 1. 3.

C

1. 11. b

1. 4.

C

1. 12. 2

1. 19.

C

1. 20. d 1. 5. d

1. 13. d 1. 21. 118

1. 6. b

1. 14.

1. 7. a

1. 15. a

1. 22. a

1. 8. d

1. 16. b

1. 23.

[email protected]

C

physicsguide CSIR NET ) GATE

11

@Sk J ahiruddin ) 2020

1.2

C

Kinetic Theory

Solutions

Sol 1.1. From t he maxwell boltzman dist ribut ion we came to know that t h e number of particles wit h in t h e velocit y v to v+dv is given by m

3

dn = 41rN(- -) 2 e 21rkT

(-mv 2 ) 2 2 kT

v dv

11 oo : oo : 32 t o know that t he number of particles wit h in t he velocity v to v+dv is given by ffi

3

(-mv 2 )

?

dn = 41rN(- - ) 2 e 2 kT v~dv 21rkT now the rootmeansquared velocity is defined as the square

root of the average of t he square of the velocity.mathematically speaking 00 2 f0 v dn 3RT Vrms = fooo dn = m in similar way we can calculate most probable velocity which is given by dfv = O dv for t he most probable velocity where f v is given by

= 41rN( m )~e(fv 21rkT

2

2~ ;

)v 2

. from t his we will get t hat Vmp

2kT

=

m

hence the ratio of most probable velocity to rms velocity is given by ( Vmp ) = Vrms

[email protected]

@Sk J ahiruddin, 2020

12

physicsguide CSIR NET, GAT E

I(inetic Theory

Sol 1.2. From the Maxwell Boltzmann distribut ion we can calculate that most probable velocity is proportional to v'T

so if temperature increases then the peak of the curve will also be held in higher temperature. The velocity distribut ion curve calculates t he number of part icles wit h in t he velocity

11 oo : oo : 34 rom t e axwe o tzmann istr1 ut1on we can calculate that most probable velocity is proportional to vT so if temperature increases then the peak of the curve will also be held in higher temperature. The velocity distribution curve calculates t he number of particles wit h in t he velocity range v to v+dv so integration of t he velocity distribut ion curve over the the full range will give you the total number of particles which is not changing, so integration over the full range in any temperature will get you same value and is given by o 1.2.

00

0

so we have to consider t hat in higher temperature the area under the curve must constant that is why t he curve will be more stipper where as in the lower temperature t he curve will be tending to flatten.so at T1 curve will be flatter than t hat of T 2 . so option c is correct.

Sol 1.3. The Maxwell one dimensional velocity distribution is given by

again i can calculate t he most probable velocity from the definition of t he most probable velcoty but here we have to j [email protected]

13

physicsguide CSIR NET, GATE

@Sk J ahiruddin, 2020

Kinetic Theory

take dgvx dvx

=Q

fnr thP mnc:::t. nrnh~hlP , rPlnri+., , ~n rl it. a-i,rPC::: th~+. (1, .. 1....

=n

00: 00: 37 Kinetic Theory

take dgvx = 0 dvx for the most probable velocity and it gives that (vx)rrip = 0 so option c is correct . Sol 1.4. It is the basic Maxwell Boltzmann distribut ion curve

which t ells that t he option c is correct. Sol 1.5. The

of the gas is defined as heat needed to increase t he temperature of the gas by 1 unit at costant volume and t he cp is defined as t he heat needed to the gas to increase t he temperature by i unit at constant pressure and we know that for an ideal gas t hey are related by the following equat ion Cp - Cv = R where R is the universal gas constant.To calculate the Cv of the gas it is most important to know the internal energy of t he gas because we know that Cv

Cv

=

au f)T

we know from the equipartition t heorem that each of the digrees of freedom will get an equal amount of energy and at t emperature T it is given by k~T SO if t he gas has f numbers of digrees of freedom t hen that gas will have total internal energy is given by U

=

[email protected]

14

@Sk J ahiruddin , 2020

so the gas wll have

fKBT 2 physicsguide CSIR NET, GATE

Kinetic Theory

Cv

=

f~B

now we know t hat f is related

11 oo : oo : 40

@Sk J ahiruddin, 2020

Kinetic Theory

so the gas wll have Cv = 1~n now we know that f is related by the total numbers of particles in the following way f = 3N - k where N is the total numbers of the particle and k is t he obstacles. for this problem we have oxygen gas which is diatomic so we have number of digrees of freedom is 5 so we have t he specific heat per molecule is ~kB

Sol 1. 6. Whenever you are asking to calculate the specific heat at higher temperature then you have to take all the degrees of freedom in to account. Total degrees of freedom =transnational and rotational digresses of freedom( contribut all the values of T)+ vibrational degrees of freedom(gets effective in only higher temperature) it is found that for linear molecules the vibrational degrees of freedom is f v = 3N - 5 and for the nor linear molecules this degrees of freedom is f v = 3N - 6

the diatomic molecule is always linear so the vibrational degrees of freedom is 1 and each vibrational digress of freedom takes an amount of energy kB T so total specific heat is given

[email protected]

@Sk J ahiruddin , 2020

15

physicsguide CSIR NET, GATE

Kinet ic Theory

11 oo : oo : 42 15

[email protected]

physicsguide CSIR NET, GATE

@Sk J ahiruddin, 2020

I(inetic Theory

by 5 7 Cv = -kB + kB = -kB = 3.5kB 2 2 Sol 1. 7. We define

2 1=-= l +Cp

f

cv

in order to do so we have to calculate t he Cp and Cv of different gas and take ratio for mono atomic its value is 1.67 for di-atomic its value is 1.40 for linear triatomic value is 1.28 and non linear 1.33 for hexa atomic 1.14 (molecules 6 and obstacles 6) so t he highest values is for mono atomic molecule.

Sol 1.8. Here the all degrees of freedom is accessible as the dotted curve represents so the possible degrees of freedom is given by rotational degrees of freedom 9 and t he vibrational degrees of freedom is 1 so total Cv

=

9 R+R 2

=

5.5R

Sol 1. 9. It is surely be monoatomic or rigid diatomic as we know t hat t he Cv for diatomic=cp for monoatomic as it is not mentioned if the solution is cporcv so both possibility •

remains.

Sol 1.10. As molecule is non linear so obstacles 3 so we will have f (t he degrees of freedom) is 6 so cp = Cv + R = 4R j [email protected]

16

physicsguide CSIR NET, GATE

11 oo : oo : 4s Sol 1.10. As molecule is non linear so obstacles 3 so we will have f (the degrees of freedom) is 6 so Cp = Cv + R = 4R j [email protected]

16

physicsguide CSIR NET, GATE

Kinetic Theory

@Sk J ahiruddin , 2020

Sol 1.11. We have

now a and /3 are constanst so they come of the integration. now to evaluate the

and this is equatl to

and in similar way if you calculate
, < Vy > v.re will get

zero because Vx, Vy these are the components so logically if we take the velocity distribution curve then t he components are equally distributed from -oo to oo so it's average is obviously zero. then we are getting putting all the values 2

a2

+ 132

< (avx - /3vy) >= ---kBT m

Sol 1.12. Here the system is in two dimension so contribut ion comes from 2 co ordinates and each co ordinate will have potential energy contribution and the kinetic energy

contribution and from the generalized equipartition t heorem will give us the potential energy contribution from the

11 oo : oo : 47 t ion comes from 2 co ordinates and each co ordinate will have potential energy cont ribution and the kinetic energy cont ribution and from the generalized equipart ition t heorem will give us t he potential energy contribution from the [email protected]

17

physicsguide CSIR NET, GATE

@Sk J ahiruddin, 2020

Kinetic Theory

each co ordineat e is k~T and similarly t he contribution for kinetic energy from each coordinat e also k~T so total energy

=4(k~T)=2kBT hence the answer is 2. Sol 1.13. We have

and we know from t he previous discussions t hat
=
= 0 and the rest of the two t erms will give you equal contribution which is kBT ( remind here it is only < (vx)2 > 2 m

Vy

but not


(v

2

so we get only k1:nT) . so total value k::,_T.

Sol 1.14. Here t he particle is constr ained to move on t he

surface of t he sphere so it is two dimensional surface hence contribution comes from only kinetic energy in two dimension and hense we get t he energy for N numbers of particle is NkBT. Sol 1.15. For one dimensional classical harmonic oscillator

we have total cont ribution to internal energy is coming from 2 2 1) kinetic energy = (~~ and 2) potential energy k; and each having average value k~T so total energy contribution kBT.so option b is correct . Sol 1.16. We know that mean free path of a gas is given by

11 oo : oo : so 2

1) kinetic energy = (~~ and 2) potential energy each having average value k~T kBT.so option b is correct .

2

and so total energy contribution k~

Sol 1.16. We know that mean free path of a gas is given by

A=

1

--/21rnd2 [email protected]

18

physicsguide CSIR NET, GATE

@Sk J ahiruddin , 2020

Kinet ic Theory

where we haven= ~ puut ing t his back in t his equation we get t hat

A=

V

~1rNd2 so from this i can conclude that .A is proportional to V so we will get

so option B is correct . Sol 1.17. We know t hat mean free path is defined as the average distance t raveled by a gas molecule or other particle

between collisions with t he other particles.from the correct ion made by Maxwell to t he fo1~mulae made by the Clausius we get t he mean free path of the gas molecule is given by v7 =

1

--/21rnd2 where n is the number of molecules per unit volume and d 2 is t he diameter so 1rd effective cross sectional area and we see that this is independent of the temperature so mean free path of t he molecules will not change. Sol 1.18. The root mean square displacement is propor-

11 oo : oo : s3 w ere n 1s e num er o mo ecu es per uni vo ume an is t he diamet er so 1rd2 effect ive cross sectional area and we see that t his is independent of t he t emperature so mean free path of the molecules will not change. Sol 1.18. The root mean square displacement is proport ional to t ime so with t he increment of t ime t he displace-

ment also increases so we get


= 6Dt

19

physicsguide CSIR NET 1 GATE

@Sk J ahiruddin, 2020

Kinetic Theory

where D is the diffusion constant . Here we get D = 10- 2 cm 2 / s and r is given by which is 6 cm so < r rms 2 >= 36cm 2 so putting all values in t he equation we get t = 10 min Sol 1.19. We know from t he first law of t hermodynamics dQ = dU + dw where dU is called the change in t he internal

energy of t he system , and dw is the workdone on t he syst em so first law of thermodynamics just tells you t he fact t hat if you give some amount of energy dQ to t he syst em then one of it's part will be given to internal energy and t he other part will be expended to get in the work of t he system.now in isobaric process the work done is given by P dV = R dT and we know t hat dU = cvdT .now we know t hat Cp - Cv = R ep = 1 so putting all the values in t he equation and we define Cu we get dU = ~~1; now i can say t hat dw

=, - l dQ r

and for diatomic molecule we have , = ; and hence we get dw = ; dQ so here we get only ; of total energy will be

00: 00: 55

dw

ry

and for diatomic molecule we have ry = ; and hence we get dw = ~dQ so here we get only ~ of total energy will be converted to workdone so opt ion d is correct. Sol 1.20. The ans is (c) . I'm writing the proof in two ways below.

Consider t he process by which molecules escape t hrough a hole in a vessel and into a vacuum [email protected]

20

physicsguide CSIR NET, GATE

@Sk J ahiruddin , 2020

Kinetic Theory

Hole of Area "A"

We assume t hat: (1) A is so small t hat t he pressure in the vessel is unchanged ; (2) The effusion does not perturb the velocity of t he gas in t he vessel; (3) There are no collisions when the molecules pass through t he slit . Molecules t hat would have been incident on t he portion of t he wall where the hole is, now pass through the hole. n

r

,

r-

1

1

r

11 oo :oo : s1 (3) There are no collisions when the molecules pass through t he slit . Molecules t hat would have been incident on t he portion of t he wall where the hole is, now pass through the hole. This creates a flux of particles defined as t he number of particles per unit area per unit t ime that leave the vessel. Consider a square hole of area dA . A particle that is a distance vdt from the hole moves with speed v and at angle 0 from the surface normal toward the hole.

j [email protected]

21

@Sk J ahiruddin, 2020

physicsguide CSIR NET I GATE

Kinet ic Theory

0

vdt

Draw a parallelepiped around the hole wit h lengt h equal to v dt , at angle 0 from the normal. All molecules within t his volume moving toward t he hole (i.e. , with the correct

11 oo : 01 : oo

Draw a parallelepiped around the hole with length equal to vdt, at angle 0 from the normal. All molecules within t his volume moving toward the hole (i.e., with the correct

0, cp angle) with speed v will pass t hrough the hole in t ime interval dt.

n

=

molecular density of gas.

Volume of parallelepiped = v cos 0 dA dt (Note that , at grazing angles, 0 r-v 1r /2, t he volume is small) No of molecules crossing t hrough da in dt is

nvcos0dadt

[email protected]

22

physicsguide CSIR NET ) GATE

@Sk J ahiruddin ) 2020

Kinetic Theory

(number density t imes volume) No of molecules - - - - - - - = nv cos 0 = FLUX. dAdt We must integrate t his expression over the distribution of velocities of the gas to obtain the average flux J. The Maxwell-Boltzm ann distribution is F(V)dv

=

F(v, 0, cp)v dvsin0d0dcp

m 21rkT Thus for average flux T

2

3 2

e-

mv 2 kT

2

v dv sin 0d0dcp

11 oo : 01

: 03

m 21rkT Thus for average flux

J=n

m 21rkT

3 2

3 2

e-

mv 2

kT

v 2 dv sin 0d0d(j)

7r

00

e-

2 mv

kT

21r

2

v 3dv

cos 0 sin 0d0 0

0

def> 0

The integration limit in 0 is taken to

1r /

2 instead of

1r

to take t he forward direction only. In the integration of v, 2 contribution of v comes from volume element and v comes from flux. The result is 3/2 1 m J =n 21rkT 2

n

8kT

4

1rm

11 2

2kT m

2

1 - [21r] 2

nv 4 23

[email protected]

physicsguide CSIR NET, GATE

@Sk J a hiruddin , 2020

I(inetic Theory

The alternative way to obtain t he int egral is Consider a volume of gas behind the hole t hat contains n dV molecules. Then J simply is:

0

m 21rkT

3 2

e

mv

2

2kT

Only t he molecules with v z > 0 can exit t he hole. Integration give you same result.

Sol 1.21. The mean speed of gas molecules can be obtained from the maxwell boltzman velocity distribution and we get

11 oo : 01

: 06

Only the molecules with Vz > 0 can exit t he hole. Int egration give you same result . Sol 1.21. The mean speed of gas molecules can be obtained from the maxwell boltzman velocity distribution and we get t hat

and doing this we get
=

BkBT m1r

and we know 3 k::,_T

from t he previous discussion we know Vrms = so now putting condition 10(Vrms)1 = (Va,ug)2 will give as ratio of m 1 and m 2 is 118. Sol 1.22. Using t he generalized equipartition t heorem as

discussion before avg of the square of the velocity only cont ributes but any other t erms or only average of t he velocity terms contributes zero so taking that into account

j [email protected]

24

physicsguide CSIR NET, GATE

@Sk J a hiruddin , 2020

which gives us



Sol 1.23. Use the formulae of rms velocity.

Kinetic Theory

11 oo :01 : oa j [email protected]

24

physicsguide CSIR NET, GATE

@Sk J ahiruddin , 2020

I(inetic Theory

which gives us



Sol 1.23. Use the formul ae of rms velocity.

j [email protected]

25

physicsguide CSIR NET, GATE

11 oo : 01 j [email protected]

: 1o

24

physicsguide CSIR NET, GATE

@Sk J ahiruddin , 2020

I(inetic Theory

which gives us



Sol 1.23. Use the formul ae of rms velocity.

j [email protected]

25

physicsguide CSIR NET, GATE

11 oo : 01 j [email protected]

: 13

24

physicsguide CSIR NET, GATE

@Sk J ahiruddin , 2020

I(inetic Theory

which gives us



Sol 1.23. Use the formul ae of rms velocity.

j [email protected]

25

physicsguide CSIR NET, GATE

11 oo : 01 j [email protected]

: 16

24

physicsguide CSIR NET, GATE

@Sk J ahiruddin , 2020

I(inetic Theory

which gives us



Sol 1.23. Use the formul ae of rms velocity.

j [email protected]

25

physicsguide CSIR NET, GATE

11 oo :01 j [email protected]

: 1a

24

physicsguide CSIR NET, GATE

@Sk J ahiruddin , 2020

I(inetic Theory

which gives us



Sol 1.23. Use the formul ae of rms velocity.

j [email protected]

25

physicsguide CSIR NET, GATE

11 oo : 01 j [email protected]

: 21

24

physicsguide CSIR NET, GATE

@Sk J ahiruddin , 2020

I(inetic Theory

which gives us



Sol 1.23. Use the formul ae of rms velocity.

j [email protected]

25

physicsguide CSIR NET, GATE

11 oo : 01 j [email protected]

: 24

24

physicsguide CSIR NET, GATE

@Sk J ahiruddin , 2020

I(inetic Theory

which gives us



Sol 1.23. Use the formul ae of rms velocity.

j [email protected]

25

physicsguide CSIR NET, GATE

11 oo : 01 j [email protected]

: 26

24

physicsguide CSIR NET, GATE

@Sk J ahiruddin , 2020

I(inetic Theory

which gives us



Sol 1.23. Use the formul ae of rms velocity.

j [email protected]

25

physicsguide CSIR NET, GATE

11 oo : oo : 01

Problems and Solutions



Ill

Second Law- of Thermodynamics Sk J ahiruddin * Suchismito Chattopadhyay

*Assistant Professor Sister Nibedita Govt. College, Kolkata Aut hor was t he topper of IIT Bombay M.Sc Physics 2009-2011 batch He ranked 007 in IIT J AM 2009 and 008 (J RF) in CSIR NET J une 2011 He has been teaching CSIR NET aspirants since 2012

1

@Sk J ahiruddin , 2020

Second Law of Thermodynaics

11 oo : oo : 03 1

Second Law of Thermodynaics

@Sk J ahiruddin, 2020

Contents 1

Problems from NET, GATE, JEST , TIFR & JAM pap ers 1.1 Ans keys . 1.2 Solut ions . • • •

j [email protected]































2





3 •



































17 18

physicsguide CSIR NET, GATE

11 oo : oo : 06

j ahir@physicsguide. in

2

Second Law of Thermodynaics

@Sk J ahiruddin , 2020

1

physicsguide CSIR NET I GATE

Problems from NET, GATE, JEST, TIFR & JAM papers

Prob 1.1. A Carnot cycle operates as a heat engine be-

tween two bodies of equal heat capacity unt il their t emperat ures become equal. If t he init ial temperatures of the bodies are T1 and T 2 respectively and T1 > T 2 t hen t heir common fin al temperature is [NET Dec 2013]

2 (a)T1 /T2

2 (b)T2 /T1

(c)~

(d)½(T1 +T2 )

Prob 1.2. Efficiency of a perfectly reversible (Carnot) heat

engine operating between absolute t emperature T and zero [JEST 2012] is equal to (a) 0 (b) 0. 5 (c) 0.75 (d)l Prob 1.3. The ent ropy-temperature diagram of two Carnot

engines, A and B, are shown in t he figure. The efficiencies of the engines are T/A and T/B respectively. Which one of the following equalities is correct? [JEST 2015]

(a) T/A == 11,f T/A

(b) T/A == T/B

(c) T/A == 3TJB

(d)

== 2TJB

Prob 1.4. The pV diagram for a Carnot cycle executed by

an ideal gas wit h Cp / Cv = , > l is shown below. Note t hat 1,2,3 and 4 label t he change-over points in t he cycle.

11 oo :oo :oa Prob 1.4. The p V diagram for a Carnot cycle executed by

an ideal gas with Cp / Cv = , > 1 is shown below. Note that 1,2,3 and 4 label t he change-over points in the cycle.

[email protected]

physicsguide CSIR NET ) GATE

3

@Sk J ahiruddin ) 2020

Second Law of Thermodynaics

,II

/

"

,t

s \

v A ,,

'

'

/.13 ✓• ,...

' I

'r



-



-

If, for t his cycle, P2 p3

X

[TIFR 2013]

t hen X =

p

1 2

4 3

~------~ V

ft... \ (\

(;J\

1 /_.

11 oo : oo : 11 4 3

.____ _ _ _ _ ___, V

(a)

1 - 1/,

(b) O

(c) l

[email protected]

physicsguide CSIR NET, GATE

4

Second Law of Thermodynaics

@Sk J ahiruddin , 2020

Prob 1.5. Consider a Carnot engine operating between t emperature of 600K and 400K . T he engine performs 1000 J

of work per cycle. T he heat (in Joules) extracted per cycle from the high temperature reservoir is. . . . . . . . . . . . .. (Specify your answer to two digits after t he decimal point) [JAM 2017] Prob 1.6. Which one of t he figures correctly represents t he

T - S diagram of a Carnot engine?

T

[JAM 2018]

T

(a)

(b)

s T (c)

s T

(d)

11 oo : oo : 14 T

T

s j [email protected]

s 5

physicsguide CSIR NET, GATE

Second Law of Therrnodynaics

@Sk J ahiruddin, 2020

Prob 1. 7. In a heat engine based on the Carnot cycle, heat is added to the working substance at const ant [JAM 2019]

(A) Entropy (B) Pressure (C) Temperature (D) Volume Prob 1.8. A heat pump working on t he carnot cycle maintains t he inside temperature of a house at 22°C by supplying 450k J s- 1 . If t he outside temperature is 0°C , the heat taken, [GATE in kJ s- 1 , from the outside air is approximately

2007] (a) 487

(b) 470

(c) 467

(d) 417

Prob 1.9. A Carnot cycle operates on a working substance

between two reservoir at temperatures T 1 and T2 with T1 > T2 . During each cycle an amount of heat Q 1 is extracted from t he reservoir at T1 and an amount Q 2 is delivered in t he reservoir T2 which of t he following statements is INCORRECT? [GATE 2011] ( a) work done in one cycle is Q1 - Q2

11 oo : oo : 16 1

\.Al .I. .I.

2

l,,A/Ol .I. .I.

t he reservoir T 2 which of t he following statements is IN-

CORRECT?

[GATE 2011]

(a) work done in one cycle is Qi - Q2

(b)

Q1

T1

=

Q2 T2

(c) entropy of the hotter reservoir decreases (d) ent ropy of t he universe (consisting of t he working substance and t he two reservoirs) increases [email protected]

@Sk J ahiruddin,

6

2020

physicsguide CSIR NET, GATE

Second Law of Thermodynaics

Prob 1.10. A reversible Carnot engine is operated between temperatures T1 and T 2 (T 2 > T 1) wit h a photon gas as the working substance. The efficiency of the engine is [GATE 2017] (a) 1 - 3T1 / 4T2 (b) 1 - T1 / T2 (c) 1 - (T1/T2)¾

4

(d) 1 - (T1/ T2) 3

Prob 1.11. When a gas expands adiabatically from V1 t o V2 by a quasi-static reversible process, it cools from t emperature T 1 to T 2 . If now t h e same process is carried out adiabatically and irreversibly, and T~ is the temperature of the gas when it has equilibrated, then (a)T~ = T2

(b)T~ > T 2

(c)T~

[NET Dec 2014]

= T2( ½ -

½

Vi)

(d)T~ = T2 V1

½ Prob 1.12. A h eat engine is operated b etween two bodies t h at are kept at constant pressure. The constant - pressure

11 oo : oo : 19 2 -

2

Prob 1.12. A heat engine is operated between two bodies t hat are kept at constant pressure. The const ant - pressure heat capacity Gp of the reservoirs is independent of temperature. Initially the reservoirs are at temperatures 300 Kand 402 K . If, after some t ime, they come to a common final

temperature T1, the process remaining adiabatic, what is [TIFR 2018] t he value of T1 (in Kelvin ?)

[email protected]

7

physicsguide CSIR NET, GATE

Second Law of Thermodynaics

@Sk J ahiruddin , 2020

Prob 1.13. Consider a monoatomic ideal gas operating in a closed cycle as shown in t he P - V diagram given below. The ratio ; ~ is ____ [JAM 2018]

(Specific your answer upto two digits after the decimal point)

P-2

adiabatic

V

11 oo : oo : 21 Pi l---~~--4-----=:::==--,. V

Prob 1 .14. The heat capacity of t he interior of a refrigerator is 4.2 kJ / K The minimum work t hat must be done to lower t he internal temperature from 18°C to 17°C when the outside temperature is 27°C will be [NET Dec 2015] (a)2.20kJ (b)0.80kJ (c)0.30kJ (d)0 .14kJ Prob 1 .15. When an ideal monatomic gas is expanded adiabatically from an initial volume Vo to 3V0 , its temperature

[NET

changes from To to T . Then t he ratio T / To is June 2016] j [email protected]

physicsguide CSIR NET 1 GATE

8

Second Law of Thermodynaics

@Sk J ahiruddin , 2020

(a) 1/ 3

(b)

(½)

2

3

(c)

(½)

1

3

(d) 3

Prob 1 .16. An ideal gas engine is run according to t he cycle shown in the s -T diagram below, where the process from D to A is known to be isochoric (i.e. maintaining V = constant s

D

.

A



B

-

C

T

The corresponding cycle in t he p - V diagram will most l

l

l

l

rm

TT'."\T""-

""r,.

-t r,. 1

00: 00: 24 B..___

___._ _ ____,(

T

The corresponding cycle in the p - V diagram will most

[TIFR 2019]

closely resemble

[email protected]

physicsguide CSIR NET, GATE

9

Second Law of Thermodynaics

@Sk J ahiruddin , 2020

A

p

C

p

...___o.

(a)

A

p

(b) V

V

D p

B

(d)

(c)

V

V

11 oo : oo : 21 p

B

(d)

(c) A

V

D

V

Prob 1.17. An ideal gas reversible engine operates in a [JAM closed cycle. The P-V diagram is shown below. 2011]

p

V j [email protected]

@Sk J ahiruddin , 2020

10

physicsguide CSIR NET I GATE

Second Law of Thermodynaics

(a)Find t he efficiency of the reversible engine assuming both specific heat , Cp and Cv as constants. (b) Identify the t hermodynamic processes and draw the corresponding T-S diagram schematically. Prob 1.18. One mole of monoatomic ideal gas is init ially at pressure P0 and volume Vo. The gas t hen undergoes a

t hree-stage cycle consisting of the following processes: (i) An isothermal expansion till it reaches volume 2V0 , and hP.::i.t O -Rows int.n t.hP. !T::J.s.

11 oo : oo : 29 Prob 1.18. One mole of monoatomic ideal gas is initially at pressure P0 and volume Vo. The gas t hen undergoes a

t hree-stage cycle consisting of the following processes: (i) An isothermal expansion till it reaches volume 2Vo , and heat Q flows into the gas. (ii) An isobaric compression back to the original volume V0 (iii)An isochoric increase in pressure till t he original pressure P0 is regained [TIFR 2017] The efficiency of this cycle can be expressed as p To -----------

Po

I I I I I I I I I I I

I I I I I I I

' I I I

-----------?-----, I I

I I I

1 2

: To Po -------, ---~: - - ~:------2 To : : I

I I I

I I I

I

I

I I

I I

.___. . ._,______, - -- v Vo

(a)

E=

4Q + 2RTo (b) 4Q + RTo

[email protected]

E

4Q + 2RT0 (c) 4Q - 3RTo 11

@Sk J ahiruddin ) 2020

(d)

E=

2Vo E=

4Q-2RT0 4Q + RTo

physicsguide CSIR NET ) GATE

Second Law of Thermodynaics

= 4Q - 2RT0 4Q + 3RTo

Prob 1.19. 1 m 3 of an ideal gas with ry = Cp/Cv = 1.5 is at a pressure of 100 kPa and atemperature of 300 K. initially the state of the gas is at the point a of the PV diagram shown. The gas is taken through a reversible cycle

a



b



c



a. The pressure at a point b is 200 kPa

11 oo : oo : 31 p

V

= .

is at a pressure of 100 kPa and atemperature of 300 K. initially the state of t he gas is at t he point a of t he PV diagram shown. The gas is taken t hrough a reversible cycle a ➔ b ➔ c ➔ a. The pressure at a point b is 200 kPa and the line ba, when extended, passes through the origin. [JAM 2007]

b

/

/

V (a)Calculate the work done by the gas in each of t he steps a ➔ b ➔ c and c ➔ a (b) Calculate the change in entropy of t he gas in each of the t hree steps above [email protected]

@Sk J ahiruddin, 2020

12

physicsguide CSIR NET, GAT E

Second Law of Thermodynaics

Prob 1.20. One mole of an ideal monatomic gas in an initial state a wit h pressure, Pi and volume ¼ is to be t aken 2 to a fin al st ate d wit h PJ = B pi and VJ = ¼/ B t hrough t he path a ➔ b ➔ c ➔ d as shown in t he figure below for a particular value of B(> 1). Here a ➔ b and c ➔ d are

11 oo : oo : 34 1.20. ne mo e o an 1 ea monatom1c gas in an init ial state a wit h pressure, Pi and volume ¼ is to be taken

to a fin al st ate d with PJ = B 2pi and VJ = ¼/ B t hrough t he path a ➔ b ➔ c -+ d as shown in t he figure below for a particular value of B(> 1). Here a ➔ b and c -+ d are adiabatic paths while b ➔ c is an isot herm with temperat ure T 0 . States b and c correspond to (p 1, ½) and (p 2 , V2 ) , respectively. [JAM 2010]

1)

b(Pi, V.)

V Find the ratio ~ and t he total work done by t he gas in terms of Pi , ¼ , T 0 and B. Prob 1.21. The P-V diagram below represents an ideal j [email protected]

@ Sk J ahiruddin, 2020

13

physicsguide CSIR NET, GATE

Second Law of Therrnodynaics

monatomic gas cycle for 1 mole of a gas. In terms of the gas constant R , calculate the temperatures at the points J ,K,L and M. Also calculate the heat rejected and heat absorbed

00: 00: 37 econd Law of Thermodynaics

monatomic gas cycle for 1 mole of a gas. In terms of the gas constant R , calculate the temperatures at the points J ,K,L and M. Also calculate the heat rejected and heat absorbed during t he cycle, and t he efficiency of t he cycle. [JAM 2012]

80 - - ~-------' 'I

I

I

I ,

~,

0•10 I

I

Pro b 1. 2 2. A hollow cylinder (closed at both ends) with adiabatic walls is divided into n equal cells (C 1 , C2 , . .. C,n)

using discs D1, D2, ... D n -1 (see figure) . The discs can slide freely wit hout f1·iction. The first (D 1 ) is adiabatic and the remaining discs are diathermal (t hermally conducting) . Each cell contains one mole of ideal monoatomic gas. Let t he init ial pressure , volume and temperat ure of each cell be P0 ,V0 [email protected]

@Sk J ahiruddin , 2020

14

physicsguide CSIR NET, GATE

Second Law of Thermodynaics

and T0 , respect ively. The gas in cell C 1 (first cell) is heated

11 oo : oo : 39

@Sk J ahiruddin, 2020

Second Law of Thermodynaics

and To, respectively. The gas in cell C1 ( first cell) is heated slowly until t he temperature of t he gas in cell Cn (last cell) reaches final equilibrium temperature 4T0 . Find the volume of t he first cell interms of the number of cells (n) and the initial volume (Vo) . [JAM 2013]



c,

cl-

s

• •



.... . - ..



en

Prob 1.23. In the t hermodynamic cycle shown in t he figure, one mole of a monatomic ideal gas is taken t hrough a cycle.AB is a reversible isothermal expansion at a temperat ure of 800K in which the volume of the gas is doubled. BC is an isobaric contraction to t he original volume in which t he temperature is reduced to 300K. CA is a constant volume process in which the pressure and temperature return to t heir initial values. The net amount of heat (in joules) absorbed by the gas in one complete cycle is [JAM 2015]

[email protected]

@Sk J ahiruddin , 2020

15

physicsguide CSIR NET, GATE

Second Law of Thermodynaics

11 oo : oo : 42 15

[email protected]

physicsguide CSIR NET, GATE

Second Law of Thermodynaics

@Sk J ahiruddin , 2020

I

.

'

'\

PJ

'

• V

Prob 1.24. For an ideal gas , which one of t he following

T-S diagram is valid?

j [email protected]

[JAM 2016]

16

physicsguide CSIR NET, GATE

11 oo : oo : 44

j [email protected]

16

physicsguide CSIR NET, GATE

Second Law of Therrnodynaics

@Sk J ahiruddin , 2020

s

s

1.1

5

Ans keys Ans keys

1.1.

C

1. 7.

C

1.13. 0.16

1.2. d

1.8 . d

1.14 . d

1.3. d

1.9. a

1.15. b

1.4. a

1.10. b

1.16. a

1.5. 3000

1.11. b

1.17. Do yourself

1.6. b

1.12. 347

1.18. d

11 oo : oo : 48 1.4. a

1.10. b

1.16. a

1.5. 3000

1.11. b

1.17. Do yourself

1.6. b

1.12. 347

1.18. d

[email protected]

physicsguide CSIR NET, GATE

17

Second Law of Thermodynaics

@Sk J ahiruddin, 2020

1.19. 100 - ln2 KJ/ K

KJ , ~PiVi ( l

1.20. w

1.2

-

B)

2RT0 ln B 1.21. rJ

= 0.736

+

1.22.

V

=

Vo

3

8(n- l ) o

1.23. 453 1.24. a

Solutions

Sol 1.1. Heat engines are special kind of engine that works between two different t emperature source one is higher t emperature and t he other is lower t emperature.an ideal heat engine takes heat energy from the hot source and convert all the heat in to t he work that is why the efficiency of an ideal heat engine is I .let heat engine takes Q 1 amount of heat from the hot source and leave Q 2 ( < Q 1 ) amount of heat in t he cold source converting Q 1 - Q2 amount of heat in to the work so the efficiency of t he heat engine is defined as follows . e ffi c1ency

=

work output ----heat input

hence for ideal gas engine no heat is left at t he cold source so the total efficiency is 1, but total efficiency of value 1 is impossible practically. Carnot cycle is a reversible cy-

11 oo : oo : so heat input hence for ideal gas engine no heat is left at t he cold source so t he total efficiency is 1, but total efficiency of value 1 is impossible practically. Carnot cycle is a reversible cycle containing an ideal gas.t he gas is first taken through an isothermal expansion at temperature T1 from t he volume v1 [email protected]

@Sk J ahiruddin , 2020

18

physicsguide CSIR NET, GATE

Second Law of Thermodynaics

to volume v2 (½ < V2 ) t hen t he gas is been t hrough an adiabatic expansion to volume v 3 and the temperature of t he gas becomes T2 ( < T1 ) and t hen t he gas is taken t hrough an isothermal compression to volume ¼ and then the gas is been taken through an adiabatic compression to temperature T1 and volume V1 so t he gas is been taken back to t he init ial state. if we perform t his process still then we get efficiency of the system is given by

but you see t hat this is less than one and only equal to 1 if t he lower temperatu1·e is zero kelvin but zero kelvin is un achievable ( we will discuss it in the later problem that why it is un achievable) . now er are comparing t his result to the efficiency of the heat engine and we get t he following

that is we can say t hat t he factor ;J_; remains unchanged during t he reversible process. this quantity is defined as

11 oo : oo : s2

t hat is we can say t hat the factor f remains unchanged during t he reversible process. t his quantity is defined as t he entropy of the system t hat is entropy of the system is defined as Q S(entropy) = T j [email protected]

@Sk J ahiruddin , 2020

19

physicsguide CSIR NET 1 GATE

Second Law of Thermodynaics

its unit is joule/kelvin. It 's change in the reversible process is zero as we see from t he previous example. (reversible process is t hat kind of process where a system is brought back to it's initial stage in with out accompanying any kind of change in the surroundings) t he entropy of t he system is also a state dependent function that is it only depend on t he final st ate of t he system and t he initial state of t he system and it does not depend on the factor t hat how the state has been achieved and we only can measure the change of t he entropy but the exact value of entropy in a particular state can not be determined. we are going to use t hese fact to solve t his problem.as the system is working between two different temperature so t he syst em will keep working unt il the two different temperature are coming in t he equilibrium. say t hat the equilibrium temperature is T so t he hot source will loss it's entropy and t he cold source will gain t he entropy until t he change in t he entropy of t he whole system comes to zero so we get the change in the entropy of the hot source i c rri "l ron

h"',

11 oo : oo : 55 t hat t he equilibrium temperature is T so the hot source will loss it 's entropy and t he cold source will gain t he entropy until t he change in t he ent ropy of t he whole system comes to zero so we get the change in the entropy of the hot source is given by dQ

dS=

T

( the change in the ent ropy is counted as t he ratio of the heat taken or given and at the particular temperature it is

[email protected]

20

physicsguide CSIR NET, GATE

@Sk J ahiruddin , 2020

Second Law of Thermodynaics

given or taken)so the entropy change of t he hot body is

entropy change of the hot body

=

T

epdT

T

T1

and t he change of the entropy of the colder one is

entropy change of the cold body

=

T T2

CpdT

T

so the total change of the system is given by

change of the entropy of the hot body + change of the cold body entropy ==total change of the entropy of the system==O using t his principle we get that

T1 T Cp loge T = Cp loge T

2

which gives us that final temperature is Im

rn

11 oo : oo : sa T1

Cp

loge T

=

T

Cp

loge T2

which gives us that final t emper ature is

Sol 1. 2. It is an ideal Carnot cycle so t he efficiency is 1 as no heat ha been left to t he cold source and the total heat taken from the hot source by t he system is converted to the work output. Sol 1.3. The most important fact about t he systems are t hat t hey are operating in the different t emperature. The j [email protected]

21

physicsguide CSIR NET I GATE

Second Law of Thermodynaics

@Sk J ahiruddin, 2020

Carnot cycle in the t-s diagram looks like a rectangle(why?) . Carnot cycle is consisting of t he two isothermal process and two adiabatic process. adiabatic process in t he T-S diagram is 1·epresented by the straight line parallel to t he T (temperature) because the fact t hat ds = 0 in t he adiabatic process as the dq=O in t he adiabatic process so change in t he ent ropy is zero so t he entropy is const ant where as the isothermal process is represented as t he straight line parallel to s curve. so as a whole the Carnot cycle is represented by a rectangle. so this two is representing nothing but two different Carnot cycle op erate in b etween 4 different t emperature.let each of the vertical line represent 1 kelvin and each of the pa1·allel line represent 1 joule/kelvin so the a cycle is operating between 1 kelvin and 2 kelvin temperature ~nn thP R r,,r lP i ~ nnPr~tinO"

h P t .,~TPP TI

thP

~

k Pl,,in ~nn thP 4

11 oo : 01 : oo different Carnot cycle operate in b etween 4 different t emperature.let each of the vertical line represent 1 kelvin and each of the parallel line represent 1 joule/kelvin so the a cycle is operating between 1 kelvin and 2 kelvin temperature and t he B cycle is operating between the 3 kelvin and t he 4 kelvin.so the efficiency of A Cycle and B cycle is given by rJ A

T2

= (1 - Ti ) = 1 - ( 1/ 2) = 1/ 2

T2 represents t he lower t emp and the T1 is representing the higher t emp. now in similar way 'r/B

= 1/ 4

'r/A

=

so we get that

[email protected]

2r;B

22

physicsguide CSIR NET ) GATE

Second Law of Thermodynaics

@Sk J ahiruddin ) 2020

Sol 1.4. It is actually representing a Carnot cycle but now in the p-v diagram. I ➔ 2 represent isothermal process so

temperature T2 is constant t hrough out the process and we have V2 Pl P2 V1 but t he second one is representing (2 ➔ 3) an adiabatic process so we have

p3 P2

where we have use the concept t hat as t he gas is going t hrough t he adiabatic expansion t he temperature also changes

11 oo : 01

: 03 p3 P2

where we have use the concept t hat as t he gas is going t hrough the adiabatic expansion t he temperature also changes and goes t o T 3 . again t he 3 ➔ 4 represents isothermal compression hence we get p3

V4

p4

V3

and t he final process we get again

so from the second condit ion we get t hat option 1 is correct. Sol 1.5. Efficiency of t he Carnot's cycle = [email protected]

@Sk J ahiruddin, 2020

23

4

1- -

6

=

1/ 3

physicsguide CSIR NET, GAT E

Second Law of Thermodynaics

we know that

work output rJ = heat input so work out put is lOOOjoule so we get heat input per cycle is 3000 Joule. Sol 1. 6. The Carnot cycle consisting of t he two isothermal and two adiabatic process and isotherm process are repre-

sents in t h T-S diagram through a straight line parallel t o s and adiabatic process are represents by the parallel to T line. the first process in the Carnot cycle is represented by t he isot hermal expansion so t he volume of the system will

11 oo : 01 : os and two adiabatic process and isotherm process are represents in t h T-S diagram through a straight line parallel to s and adiabatic process are represents by the parallel to T line. the first process in the Carnot cycle is represented by t he isothermal expansion so the volume of the syst em will increase and hence the entropy of the system must increase and heat will be taken inside the system hence the option b is correct because t he first process represents t he entropy increase. •

Sol 1. 7. A refrigerator is working on the basic principle of t he fact that a heat engine is now taking heat from the cold source and rejecting heat in the hot source by amount of some work doing on it externally. so let a heat engine is taking is t aking Q 2 amount of heat from the cold source and by doing Q1 - Q2 amount of work on it externally rejecting Q1 ( > Q2 ) amount of the heat to the hot source.so hot source is getting hotter and cold sink is getting colder. in this case we define a paramet er called coefficient of p erformance and j [email protected]

24

physicsguide CSIR NET, GATE

Second Law of Therrnodynaics

@Sk J ahiruddin , 2020

is denoted by/3 and defined as

/3 = heat t aken from the cold source v.rrokdone perfromed so we can say th at where w denotes the workdone on the syst em and w = Q 1 Q 2 so the heat reject ed in t he hot source can be defined as

11 oo :01 : oa so we can say where w denotes the workdone on the syst em and w = Q 1 Q 2 so the heat rejected in the hot source can be defined as Q1 = w( l

+ /3)

, on the basis of t hese we can say that here

where we use the fact that Q1

T1

Q2

T2

and hence we have t he relation between

/3

and 'T/ is given by

1 rJ=

l +/3

so here i can calculate the eta and that is

/3 =

22 295

and hence

~;~ and we know that heat t aken from t he cold source

is Q 2 = W /3 = 4l 7kj / s [email protected]

@Sk J ahiruddin, 2020

25

physicsguide CSIR NET, GATE

Second Law of Thermodynaics

Sol 1.8 . Entropy of the universe increases only in t he case of irreversible process and for the reversible process it is

equal to zero so here for the cycle being consisting of the configuration of the Carnot cycle so the entropy change must be equal to zero. Sol 1 .9. In a Carnot cycle t here is only one process where

11 oo : 01

: 11

equal to zero so here for the cycle being consisting of the configuration of the Carnot cycle so t he ent ropy change must be equal to zero. Sol 1.9. In a Carnot cycle there is only one process where heat is entering the system and that is isothermal expansion

of the working substance and in the isothermal process the temperature of the system is constant so only temperature is constant when heat is added to the system hence the option c is correct. Sol 1.10. From the previous discussion we came to know t hat the carnoot cycle's efficiency is not depending upon t he substance you are using in t he system it is depending

on the temperature of the hot and sink temperature hence T1 77 = l - T2 as T1< T2 Sol 1.11. Irreversible process is always accompanied with positive change in th entropy so we must have dS > 0 for the irreversible process and for the reve1~sible one we must

have dS = 0 . This summerise the second law of the thermodynamics t hat entropy must be greater or equal to zero. [email protected]

@Sk J ahiruddin ,

2020

26

physicsguide CSIR NET, GATE

Second Law of Thermodynaics

The first case t ell you t he process is reversible as well adiabatic where as the case of second one the process in irreversible and adiabatic, the first one is isoentropic process but the second is definitely not an isoent ropic process. so t he t emperature in t he second one would be greater than

11 oo : 01

: 13

The first case tell you t he process is reversible as well adiabatic where as the case of second one the process in irreversible and adiabatic, t he first one is isoent ropic process but the second is definitely not an isoentropic process. so t he temperature in t he second one would be greater t han t han previous one. Sol 1.12. As the system is working between two different temperature so t he system will keep working until the two

different temperature are coming in the equilibrium. say t hat the equilibrium temperature is T so the hot source will loss it 's entropy and t he cold source will gain t he entropy until t he change in t he entropy of t he whole syst em comes to zero so we get the change in t he entropy of the hot source is given by dQ dS= T ( the change in the ent ropy is counted as t he ratio of the heat taken or given and at the particular temperature it is given or taken)so the entropy change of the hot body is entropy change of the hot body

j [email protected]

@Sk J ahiruddin, 2020

27

=

physicsguide CSIR NET 1 GATE

Second Law of Thermodynaics

and t he change of the entropy of the colder one is entropy change of the cold body

=

Ti

CpdT

r

11 oo : 01

: 16

@Sk J ahiruddin, 2020

Second Law of Therrnodynaics

and t he change of the entropy of the colder one is

entropy change of the cold body

=

so the total change of the system is given by Change of t he ent ropy of t he hot body + change of t he cold body entropy = total change of the ent ropy of the system= O. Using t his principle we get t hat

T1 Cp loge T1 =

T1 Cp loge T2

which gives us that final t emperature is

T1 =

T2T2

so now putting all the values of T1 and T2 i get Ti = 347.2K

Sol 1.13. From the figure we can say t hat in the adiabatic process P2V1'

= Pl 3v1 'Y

but for the monoatomic gas the ry = Pl P2

=

i now we will have

3-5/3 = 0.16.

Sol 1.14. We know that the relation between the coefficient of performance and efficiency is given by 1 rJ= [email protected]

l + ,8 28

@Sk J ahiruddin, 2020

and we can easily calculate the

physicsguide CSIR NET, GATE

Second Law of Thermodynaics

11 oo :01

: 1a

@Sk J ahiruddin, 2020

Second Law of Thermodynaics

and we can easily calculate the

+ 18 = 273 + 27

= l _ 273

rJ

0.03

so the coefficient of performance

/3

1 = 0.03 - 1 = 32.33

now the heat must be taken out to decrease t he temperature is given by CpdT = 4.2 x (1) = 4.2kJ and from the principles of refrigerator we know t hat heat has been taken out from the cold source is q2

= w/3

so t he workdone must be 0.14KJ. Sol 1.15. From t he adiabetic process we come to know t hat

Tova, - l = T3v0, - 1 so we get that 2 3

1 T 3 To Sol 1.16. First the engine goes from say A -+ B it is an isothermal process but as t he ent ropy decreases so we can j [email protected]

@Sk J ahiruddin, 2020

29

physicsguide CSIR NET, GATE

Second Law of Thermodynaics

11 oo : 01 [email protected]

: 21

29

physicsguide CSIR NET, GATE

Second Law of Thermodynaics

@Sk J ahiruddin , 2020

say that this co responds to release of heat t hat means it is an isothermal compression so in pv diagram volume must decreases.then B -+ c is an adiabatic process but again it corresponds to the adiabatic compression so volume again decreases. now c -+ D corresponds to an isothermal expansion as t he ent ropy increasing so in pv diagram volume must increase now finally D -+ A is an isochoric process so in pv diagram is a straight line parallel to P as the volume is unchanged so the best represented curve is option a. Sol 1 .17. (a) according to the given graph

W AB

= 0 (IS-

COCHORIC PROCESS) so we have that

we know that Pv

= n RT T = pv

' nR so t he workdone in t he AB process can be written down

and during BC process heat input is zero as it is a adiabatic process and we have dT

[email protected]

=

30

-cv(P2V1 - p1v2) R physicsguide CSIR NET, GATE

11 oo : 01

: 24

dT = -cv(P2V1 - p1v2) R j [email protected]

physicsguide CSIR NET 1 GATE

30

Second Law of Thermodynaics

@Sk J ahiruddin , 2020

and lastly

~QcA=ncp

Cp

dT = R (p1v1- p1 v2

)

and t he work done

we get the efficiency total wor kdone rJ = heat absorped WAB + WBc + WcA ~QAB + ~QBc + ~QcA hence we have all the elements calculated before putting all we get t he exact result. (b)

- 0

A ..........____,. : - ~ C

Cp

11 oo : 01

: 26

Cp

31

[email protected]

physicsguide CSIR NET, GATE

Second Law of Thermodynaics

@Sk J ahiruddin , 2020

change in the entropy of system is given as !:lS

= !:lQ T

now for A to B process i:lSAB =

fraccvdTT

so we have from this

similarly sor t he C to A process we will have

T =

kl e(s/ cp)

but we know that Cp > Cv so we have the slope of one is different from the other and curt of AB is more steeper t han t hat of t he AC. So t he figure is given above in the sT diagram is justified. Sol 1.18. The gas has been expanded to two times of it 's initial volume at const ant temperature T0 so heat inserted Q

=

dQin

=

RToln 2

t hen t he gas has been compressed by t he isobaric process so t he heat rejected

11 oo : 01

: 29

initial volume at const ant temperature T0 so heat inserted Q = dQin = RToln 2

t hen the gas has been compressed by the isobaric process so the heat rejected dqout

=-

3 To Po R + (-Vo ) 2 2 2 32

j [email protected]

physicsguide CSIR NET I GATE

Second Law of Thermodynaics

@Sk Jahiruddin, 2020

here the negative sign denotes that heat is going out . Again for isochoric process t he gas is gaini11g heat

as t he temperature of the gas is increasing.so efficiency 1] = 1 -

which can be resolved 3RTo+2Povo

'l]

= 1-

4

4Q+3RTo 4

so we further simplify and get 4Q- 2RTo 'lJ = 4Q + 3RTo

where we use the ideal gas equation PoVo = RT0 Sol 1.19. (a) As for t he path Pa

a ➔

Pb

b p ex: v so we have

11 oo : 01

: 31

where we use the ideal gas equation PoVo = RT0 Sol 1.19. (a) As for t he path a~ b p ex: v so we have

so we have

l OOkP a l m3

[email protected]

200kPa vb 33

@Sk J ahiruddin ) 2020

physicsguide CSIR NET ) GATE

Second Law of Thermodynaics

and hence we get

now let us take the process AB and analyses it. for this we take p = a v and we need to know that t he a is to evaluate t he a we take init ial condition that when pressure is 100 Kpa volume 1 cubic meter and t his give us a = lOOK P a/m3 and t he work done in the process is 2

2

avdv = 50K J

pdv = 1

1

but during t he BC the work done is zero as the process is isochoric so no volume change so work done is zero. during t he CA process it is the isobaric process so work done WcA =

Pa(V a

-

vb) = 100(1 - 2)kJ = -100 K J

(b) For the process AB t he change in the entropy is given by

11 oo : 01 WcA =

Pa(Va - vb)

=

: 34 100(1 - 2)k J

=

- 100 K J

(b) For t he process AB t he change in the entropy is given by ~QAB

T

= ncv

now from the ideal gas equation we get

[email protected]

34

physicsguide CSIR NET, GATE

Second Law of Thermodynaics

@Sk J ahiruddin, 2020

sow ecan use t his formulae to solve this chnageinentropy

=

so applying all the values we get change is entropy during AB process is given by

100 x 103 x ln(4 ) x 3

l

+

1 1.5 - 1

= 2ln2kj /k

in similar way for t he second means BC process change in t he ent ropy is ncv

T as we know that in the isochoric process no work is done so total heat is been used up to enhance the internal energy of t he system. and hence we get T2

rn R

?1 A

')

11 oo : 01

: 36

as we know t hat in t he isochoric process no work is done so total heat is been used up to enhance t he internal energy of t he system. and hence we get

T3 nR Pc nevln - = - -ln T2 ry - 1 Pb

= -

2 - x ln2kj / k 3

now we come to t he last process that is CA process ao the entropy change in t he CA process is given by

llQcA

=

ncp

dT T3T1 T = n R

1 1 + -1' - 1

In Va Vb

= - ln2 KJ/ k 35

j [email protected]

physicsguide CSIR NET, GATE

Second Law of Therrnodynaics

@Sk J ahiruddin , 2020

Sol 1.20. According to the PV curve we have a to b is an adiabatic process so we will have 5/ 3

V1

Pi Pl

Vi

for t he b to c process as it is a iso thermal process we have V2 V1

Pl P2

and for t he cd process being adiabatic again we have Vi

5/3

P2 1 B 3pi

now multiplying t he relation of a to b process by the c to d process we get

11 oo : 01

: 39 1

B 3pi

now multiplying the relation of a to b process by the c to d process we get 1 V2 B2 now we are coming to t he second part of calculating t he t otal workdone

W=w(ab)+w(bc)+w(cd) Now we have

w(ab) = p;v;2/:1v1 t he work done in the be process

[email protected]

36

physicsguide CSIR NET , GATE

@Sk J a hiruddin, 2020

Second Law of Thermodynaics

and t he work done is the cd process is given by

so total work done can be written as

here we use the fact that p 1v1 = p 2 v2 as we get in t he isothermal process.but we know t hat PJVJ = Bpivi putting this in t he equation we get

11 oo : 01

: 42

mal process.but we know that PJVJ = Bpivi putting this in t he equation we get

Sol 1. 21. Using t he ideal gas equation ,pv one mole gas .so

= n RT for the

and t he Tk

=

PkVk

= 580K

R and the other two temperatures are using same technique is given by TL = 193.33K, TM = 97 K now we come to know t he fact that area under the curve is going to given you the total wor kdone in the cycle as t hey both have same

[email protected]

37

physicsguide CSIR NET, GATE

Second Law of Thermodynaics

@Sk J ahiruddin , 2020

dimension but only in pv curve the area under curve is equal to the work dome so the total work done areaaunderJKLM = 0.07 x (80 x 103 ) = 5600Joule

change in the internal energy during t he cycle ~Q

=

0 + w = 5.65

as t he system returns to the init ial state so t he total internal change in the energy is zero.heat associated with each path can be calculated as follows

11 oo : 01 ~Q

: 44

= 0

+w =

5.65

as t he syst em returns to the init ial state so the total internal change in the energy is zero .heat associated with each pat h can be calculated as follows Cv OTJK

3R = 2

X

(580 - 290)

again for KL process

and we again have

and for ~ Q MJCpTJ - T M=

4.03kJ

so heat absorbed in the cycle

~Qa = (3.6 + 4.03)K J j [email protected]

=

7.63K J

physicsguide CSIR NET 1 GATE

38

Second Law of Thermodynaics

@Sk J ahiruddin, 2020

and heat reject ed

~Qr = 1.2 + 8.02

=

9.22kJ

=

0.736

so efficiency of the cycle rJ =

W ~Qa

=

5.6 _ 76



,

.

...

..

...

..

..,,

...

11 oo : 01

: 47

W

5.6 _ 76

rJ = !:::,.Qa =

0.736

=



Sol 1.22. Let us consider a hollow cylinder which is devided in to n equal compartments.final equilibrium temperature of t he gas in the compartment 2 to n is 4T0 , for the adiabetic expansion of the first compartment is given by

PoVo 1 = pv 1 for the adiabet ic expansion of t he remaining (n-l)cells we can write 1 1 (n - l )Po - T01 = p 1- -r 4T 01 and here we know t hat 5 ry = 3

and we get that p

= 32Po(n - 1)

[email protected]

39

physicsguide CSIR NET, GATE

@Sk J ahiruddin , 2020

Second Law of Thermodynaics

now we are using this pressure in the first equation we get

Pavo1 = 32P0 (n - l )v1 and hence we get the volume V

Vo = 3 8(n - l ) s

11 oo : 01 ovo

: 49

= 32Pa

n - l v

and hence we get the volume

v=

Vo 3

8(n - 1)5

Sol 1.23. AB is the isothermal expansion. work done from A to B process W AB =

V2

n RTln -

=

l x 8.31 x 800ln2

=

4608j

V1

so t his is t he heat absorbed in the AB process , now let us calculate t he heat absorbed in the BC processes QAB

= CpdT = - 10387.5]

as the process makes t he compression so t he heal is being rejected so there is a negative sign. Now during t he AB process t he total work done in t he process is zero because it is an isochoric process so heat absorbed is equal to increase in t he internal energy. so

dU

= qisochoric =

Cv(6T ) =

3

R (800 - 300)

2

=

6232.5]

so net heat absorbed

Qr= 4608 + (- 10387.5) j [email protected]

@Sk J ahiruddin , 2020

40

+ 6232.5 = 453J physicsguide CSIR NET I GATE

Second Law of Thermodynaics

Sol 1.24. Change in t he ent ropy of system is given as

now for A to B process

11 oo : 01 : s2 Qr

=

4608 + (- 10387.5)

j ahir@physicsguide. in

40

+ 6232.5 =

453J

physicsguide CSIR NET, GATE

Second Law of Thermodynaics

@Sk J ahiruddin , 2020

Sol 1.24. Change in t he ent ropy of system is given as

now for A to B process CvdT

T so we have from this

similarly for the C to A process we will have

but we know t hat Cp > Cv so we have the slope of one is different from t he other and curt of AB is more steeper t han t hat of t he AC. So the figure a is given above in t he TS diagram is justified.

j ahir@physicsguide. in

41

physicsguide CSIR NET, GATE

11 oo : 01 Qr

=

: 55

4608 + (- 10387.5)

j ahir@physicsguide. in

40

+ 6232.5 =

453J

physicsguide CSIR NET, GATE

Second Law of Thermodynaics

@Sk J ahiruddin , 2020

Sol 1.24. Change in t he ent ropy of system is given as

now for A to B process CvdT

T so we have from this

similarly for the C to A process we will have

but we know t hat Cp > Cv so we have the slope of one is different from t he other and curt of AB is more steeper t han t hat of t he AC. So the figure a is given above in t he TS diagram is justified.

j ahir@physicsguide. in

41

physicsguide CSIR NET, GATE

11 oo :01 : s1 Qr

=

4608 + (- 10387.5)

j ahir@physicsguide. in

40

+ 6232.5 =

453J

physicsguide CSIR NET, GATE

Second Law of Thermodynaics

@Sk J ahiruddin , 2020

Sol 1.24. Change in t he ent ropy of system is given as

now for A to B process CvdT

T so we have from this

similarly for the C to A process we will have

but we know t hat Cp > Cv so we have the slope of one is different from t he other and curt of AB is more steeper t han t hat of t he AC. So the figure a is given above in t he TS diagram is justified.

j ahir@physicsguide. in

41

physicsguide CSIR NET, GATE

11 oo : oo : 01

Problems and Solutions in Max-well's relation and Thermodynamic Potentials Sk J ahiruddin * Suchismito Chattopadhyay

*Assistant Professor Sister Nibedita Govt. College, Kolkata Aut hor was t he topper of IIT Bombay M.Sc Physics 2009-2011 batch He ranked 007 in IIT J AM 2009 and 008 (J RF) in CSIR NET J une 2011 He has been teaching CSIR NET aspirants since 2012

1

@Sk J ahiruddin , 2020

Nlaxwell's relation and Thermodynamic Potent ials

11 oo : oo : 03 1

@Sk J ahiruddin, 2020

Maxwell's relation and Thermodynamic Potentials

Contents 1

Problems from NET, GATE, JEST , TIFR & JAM pap ers 1.1 Free expansion of Gas 1.2 Ans keys . • • • 1.3 Solut ions . •

j [email protected]































2









3 •





















































9

12 13

physicsguide CSIR NET, GATE

11 oo : oo : 06

j ahir@physicsguide. in

@Sk J ahiruddin , 2020

1

physicsguide CSIR NET I GATE

2

Maxwell's relation and Thermodynamic Potent ials

Problems from NET, GATE, JEST, TIFR & JAM papers

Prob 1.1. A thermodynamic system is maintained at a con-

stant t emperature and pressure. In thermodynamic equilib(JAM 2008] rium, its (a) Gibbs free energy is minimum (b) ent halpy is maximum (c) Helmholtz free energy is minimum (d) Int ernal energy is zero Prob 1.2. If U,F ,H and G represent internal energy, Helmholtz free energy, ent halpy and Gibbs free energy respectively,then which of t he following is a correct thermodynamic relation? [JAM 2016]

(a) dU

=

P dV - T dS

(c)dF = - P dV

(b )dH

+ SdT

=

V dP

(d)dG = V dP

+ T dS + sdT

Prob 1. 3. W hich among t he following sets of Maxwell relat ions is correct? (U-internal energy,H -ent halpy,A-Helmholt z

free energy and G-Gibbs free energy) (a) T

=

(Zi)8 and P =

(i~ )v

(b) V

=

(i ~) 8

(~~)P

and T

=

[GATE 2010]

11 oo :oo :oa

(a) T

=

(b) V =

(ii)8 and P =

(i~)v

(i ~) 8

(~~)P

and T =

[email protected]

3

@Sk J ahiruddin, 2020

(c) P = (d) P

=

physicsguide CSIR NET, GATE

l\/[axwell's relation and Thermodynamic Potentials

-(ii )T and V = (ii)s - (~1)T and S = (it )v

Prob 1.4. The internal energy of n moles of a gas s given by E = ~nRT where V is te volume of t he gas at temperature T and a is a positive constant. One mole of t he gas in state (T1 , Vi ) is allowed to expand adiabatically into vaccum to a final state (T2 , ½). The t emperature T2 is [GATE 2006]

t,

(a) T1 + Ra( J2 +

(b) T1 - ~ Ra (

J 1

)

J J 2

-

1

)

..l..) (c) T1 + ~3 Ra(..l.. V2 V1

Prob 1.5. The equat ion of state of one mole of a van der Walls gas is P

+;

2

(V - b)

=

RT and its internal energy U (T , V )

;2 ,

is given by U(T, V) = Uo + CvT where Uo and Cv can be t aken as constants. [JAM 2009] (a) prove that in a reversible adiabatic process t he t emperature and volume satisfy the equation T (V - b)Rf Cv =

11 oo : oo : 11 is given by U (T , V ) = Uo + CvT - {:2 , where Uo and Cv can be taken as constants. [JAM 2009] (a) prove t hat in a reversible adiabatic process t he t emperature and volume satisfy t he equation T (V - b)Rf Cv = constant. [email protected]

physicsguide CSIR NET, GATE

4

@Sk J ahiruddin , 2020

Nlaxwell's relation and Thermodynamic Potentials

(b) Calculat e t he change in entropy of the gas when it undergoes a reversible isot hermal expansion from volume V0 to 2Vo. Prob 1.6. The relation bet ween the internal energy U, ent ropy S, temperature T , pressure p , volume V, chemical potential µ and number of particles N of a t hermodynamic system is dU = T dS - pdV + µdN. T hat Uis an exact different ial implies t hat [NET Dec 2017] ap (a) - as

V,N

au

(c) Par

s,N

=

aT av l s ,N

=

l au - r av

s ,µ

au

(b) par

au

l s,J\T

ap (d) - as

= s av

V,N

=

aT av

S,µ

S,N

Prob 1. 7. A t hermodynamic function G(T , P, N) = U T S + PV is given in t erms of t he internal energy U, Temperature T , ent ropy S, pressure P, volume V, and t he number of particles N. Which of t he following relations is t rue? (µ is chemical potential) [NET June 2017] aT

(c) V = -

N,P

ac

N,P

(d) µ = -

aP

N .T

ac aN

P.r

11 oo : oo : 14 is chemical potential) fJG (a) S = fJT N,P (c )V= -

[NET June 2017] fJG fJT

(b) S =

fJG fJP

(d) µ = N,T

N ,P

fJG f)N

PT

'

Prob 1.8. Starting with the equation T DS

= dU + pdV

and using t he appropriate Maxwell's relation along with the expression for heat capacity Gp (see useful information), the j [email protected]

@Sk J ahiruddin , 2020

derivat ive

5

physicsguide CSIR NET, GATE

Maxwell's relation and Thermodynamic Potentials

op

for a substance can be expressed in terms fJT s of its specific heat cp, density p , coefficient of volume expansion /3 and temperatu1~e T. 3 3 For ice cp = 2010 J/ Kg - K , p = 10 km/m and /3 = 1.6 x 10-4 /° K. If t he value of

at 270 K is N x 107 Pa/ K ,

: s

t hen t he value of N is ..... . Specify your answer to two digits after the decimal point [JAM 2017] Prob 1.9. For temperat ure T1 > T 2

the qualitative temperature dependence of t he probability dist ribution F( v) of t he speed v of a molecule in t hree dimensions is correctly represented by t he following figures [NET June 2013] ,

00 : 00 : 16

[email protected]

@Sk J ahiruddin, 2020

6

physicsguide CSIR NET, GATE

Nlaxwell's relation and Thermodynamic Potentials

Prob 1.10. For a system at constant temperature and volume, which of the following statements is correct at equilib[GATE 2016] rium? The Helmholtz free energy attains a local minimum The Helmholtz free energy attains a local maximum

11 oo : oo : 19 ume, which of the following statements is correct at equilib[GATE 2016] rium? The Helmholtz free energy attains a local minimum T he Helmholtz free energy attains a local maximum The Gibbs free energy attains a local minimum The Gibbs free energy attains a local maximum

Prob 1.11. The free energy of a photon gas enclosed in a volume V is given by F = -½aT4 where a is a constant and T is the temperature of the gas. The chemical potent ial of t he photon gas is [GATE 2006] (a) 0 (b) ;avT 3 (c) ! aT 4 (d) aVT3 [email protected]

physicsguide CSIR NET, GATE

7

Maxwell's relation and Thermodynamic Potentials

@Sk J ahiruddin , 2020

Prob 1.12. MSQ: A thermodynamic system is described by t he P, V, T coordinates. Choose t he valid expression(s) for the system. [JAM 2019]

(A)

(gC)r (i~ )p = - (g; )v

(gC)r (ii) P = (i; )v

(B)

Prob 1.13. MSQ: Which of the following relations is (are) t rue for t hermodynamic variables? [JAM 2018]

(a) T dS = CvdT

8P

+ T BT

dV V

(b) T dS = CpdT -T

(c) dF = -SdT + PdV (d) dG = - SdT + VdP .. - . ., .....

~

av 8T

dP p

,.,

"

_,

,

11 oo : oo : 21 av oT

(b) TdS = CpdT - T

dP p

(c) dF = -SdT + PdV (d) dG = -SdT + VdP Prob 1.14. Consider an ensemble of t hermodynamic sys-

tems each of which is characterized by the same number of particles, pressure and temperature. The t hermodynamic [JAM 2018] function describing the ensemble is (a) Enthalpy (b) Helmholtz free energy (d) Entropy (c) Gibbs free energy Prob 1.15. A real gas has specific volume v at temperat ure T. Its coefficient of volume expansion and isothermal

compressibility are a and kr, respectively. Its molar spej [email protected]

@Sk J ahiruddin, 2020

8

physicsguide CSIR NET, GATE

:Niaxwell's relation and Thermodynamic Potentials

cific heat at constant pressure C p and molar specific heat at constant volume Cv are related as [JAM 2014] T va (a)Cp = Cv + R (b )Cp = Cv + kr

Tva 2 (c)Cp = Cv + kr

(d)Cp = Cv

Prob 1.16. A thermally insulated ideal gas of volume ½

and temperature T expands to another enclosure of volume ½through a prous plug. What is t he change in the temperature of the gas? [JEST 2012] (a) 0 (b) Tln(½ /½ ) (c) Tln(V2 /½)

(d) T In [(½ - V1) / V2]

11 oo : oo : 24 ature of t he gas? (a) 0 (b) Tln(½ / V2 )

[JEST 2012]

(c) Tln(½/½)

(d) T ln [(½ - V1) / V2]

1.1

Free expansion of Gas

Prob 1.17. During free expansion of an ideal gas under adiabatic condition, the internal energy of t he gas [JAM 2019] (A) Decreases (B) Initially decreases and then increases (C) Increases (D) Remains constant

Prob 1.18. MSQAn isolated box is divided into two equal compartments by a partition (see figure) . One compartment [email protected]

@Sk J ahiruddin, 2020

9

physicsguide CSIR NET, GATE

Maxwell's relation and Thermodynamic Potentials

contains a van der Waals gas while the other compartment is empty. The partition between the two compartments is now removed. After the gas has filled the entire box and equilibrium has been achieved, which of the following statement (s) is (are) correct? [JAM 2017] (a) Internal energy of t he gas has not changed (b) Internal energy of the gas has decreased (c) Temperature of t he gas has increased (d) Temperature of the gas has decreased

Prob 1.19. A frictionless, heat conducting piston of neglio-ihlP m::-i.~~ ::in n hP::-it. r::-in::-irit,r ni,rin P~ ::-i '\TPrtir::-il in ~,, 1:::-it.Pn

11 oo : oo : 21 (b) Internal energy of the gas has decreased (c) Temperature of t he gas has increased (d) Temperature of the gas has decreased Prob 1.19. A frictionless, heat conducting piston of neg-

ligible mass and heat capacity divides a vertical, insulat ed cylinder of height 2H and cross sectional area A into two halves. Each half contains one mole of an ideal gas at temperature T0 and presure P0 corresponding to STP. The heat capacity ratio , = Gp/ Cv is given. A load of weight W is tied to t he piston and suddenly released. After t he system comes to equilibrium, t he piston is at rest and the temperat ures of the gases in t he two compartments are equal. What is t he final displacement y of t he piston from its initial position, assuming yW > ToCv? [JEST 2018] (A) 2H (D) 2H (C) H (B) H , ~ ~ Prob 1.20. A thermally-insulated container of volumeV0 is

'

j [email protected]

10

@Sk J ahiruddin , 2020

physicsguide CSIR NET I GATE

Maxwell's relation and Thermodynamic Potentials

divided into two equal halves by a non-permeable partition. A real gas with equation of st ate 3

b

a2 p+ V 3

= n RT

where a and b are constants,is confined to one of these halves at a t emperature T0 . The partition is now removed suddenly and t he gas is allowed to expand to fill the entire container. The final t emperature of the gas, in t erms of its specific

11 oo : oo : 29 where a and bare constants,is confined to one of these halves at a temperature T0 . T he part it ion is now removed suddenly and t he gas is allowed to expand to fill t he ent ire container. T he final t emperature of the gas, in terms of its sp ecific heatCv , will be [TIFR 2014] 3a2 2a2

(a)To - 2Cv¼2 3a

(b)To - 3Cv¼2

2

(c) To + 2CvV02

2a

(d)To + 3Cv½2

[email protected]

@Sk J ahiruddin, 2020

1.2

2

11

physicsguide CSIR NET, GATE

l\/[axwell's relation and Thermodynamic Potent ials

Ans keys Ans keys

1.1. a

1.8. 4.65

1.15.

C

1.2. b

1.9. a

1.16.

C

1.3. b

1.10. a

1.17. d

11 oo : oo : 32 1.1. a

1.8. 4.65

1.15.

C

1.2. b

1.9. a

1.16.

C

1.3. b

1.10. a

1.4.

1.11. a

C

1.17. d 1.18. a,d

1. 5. Do yourself

1.12. a .c

1.6. a

1.13. b ,d

1.19.

1.7. a

1.14.

1.20. a

[email protected]

@Sk J ahiruddin, 2020

1.3

J

C

12

C

physicsguide CSIR NET, GATE

Nlaxwell's relation and Thermodynamic Potentials

Solutions

Sol 1.1. The Gibbs free energy is defined as

G == U +PV-TS

11 oo : oo : 34 Sol 1.1. The Gibbs free energy is defined as

G == U + PV - TS so we can say

dG == dU

+ PdV + VdP -

TdS - SdT

but we actually know that from t he first law of thermodynamics that dU + PdV == dQ == TdS so putt ing t his in the equation we get

dG == VdP - SdT but at constant pressure and temperature dP and dT is zero so Gibbs free energy should have t he minimum. because at t he constant t emperature and pressure the change in the Gibbs energy is zero that is it remains unchanged. Sol 1.2. The enthalpy function is defined as

H == U + PV so

dH = dU + PdV + VdP j [email protected]

@Sk J ahiruddin , 2020

13

physicsguide CSIR NET, GATE

Maxwell's relation and Thermodynamic Potentials

but first law of t hermodynamics is telling us that

dQ = dU

+ PdV == TdS

00: 00: 37 Maxwell's relation and Thermodynamic Potentials

but first law of t hermodynamics is telling us that dQ = dU

+ P dV =

T dS

and putting this in the equation we get dH = V dP

+ TdS

Sol 1.3. We get to know t hat dH

= VdP+TdS

So from this we can say that is p is constant then

and if we say t hat s is constant t hen ds=O so we are getting

so option b is correct . Sol 1.4. For the adiabatic system we always have that the heat received or heat rejected is zero that is no heat can enter t he system and no heat can go away from the system

so we have dU

= -dw

where dU is t he change in t he internal energy and U is the internal energy here U is equal to E as E is defined as t he [email protected]

@Sk J ahiruddin, 2020

14

physicsguide CSIR NET, GATE

Nlaxwell's relation and Thermodynamic Potentials

internal energy and hence dE is t he change i n the internal

11 oo : oo : 40

@Sk J ahiruddin, 2020

l\/[axwell's relation and Thermodynamic Potentials

internal energy and hence dE is t he change i n the internal energy.so we have 3

dw == -dE ==

a nRdT + v 2 dv

2

now to get the total work done in the free expansion is zero so we have T2 3 -nRdT + T1 2 and t his gives us that 2

1

1

T2 == Ti + - Ra(- - -) 3 ½ V1 . (in t he free expansion there is no work done because t here is no external pressure. a free expansion is an irreversible process and as t he vacuum has no pressure so a gas is allowed to expand in vacuum is essentially a free expansion as t he gas has to do no work against t he ext ernal pressure as p ==

0). Sol 1.5. (a) during an isoentropic process t here are no dissipative effects and the system neither absorbs nor give off t he heat. for t his reason the isoentropic process is called t he reversible adiabatic process. now t he given equation of state is a P + V 2 (V - b) == RT [email protected]

@Sk J ahiruddin , 2020

15

physicsguide CSIR NET, GATE

Maxwell's relation and Thermodynamic Potentials

11 oo : oo : 42 15

[email protected]

@Sk J ahiruddin, 2020

physicsguide CSIR NET, GATE

Nlaxwell's relation and T hermodynamic Potent ials

from t his we can say that

P=

RT V-b

a

v2

now we come to know t hat from the first law of thermodynamics dQ = dU + dW and it is given by that a U(T, V) = Uo + CvT - V 2

so the change in t he internal energy is given by

and t hat is why we can say t hat

dQ = evdT +

RT V _ bdV

but due to isoentropic process dQ= O and hence we get

dT Cv T

+

R dV Cv

= O

V- b

so we get

so we get

T (V - b)Rf cv = constant j [email protected]

16

physicsguide CSIR NET, GATE

11 oo : oo : 4s so we ge

T(V - b)Rf cv = constant j [email protected]

16

@Sk J ahiruddin , 2020

physicsguide CSIR NET, GATE

Maxwell's relation and Thermody namic Potentials

(b) from the first law of thermodynamics

dQ

=

dU + dW

=

RT cvdT + V _ bdV

and we know that

dQ = TdS so using this we get

dS = CvdT T

RdV + V-b

but now we are considering the isothermal process so we must get dT=O and this condition will give us change in the entropy 2 0 V RdV 2½ - b Vo V - b = Rloge Vo - b Sol 1.6. From t he relation i can conclude t hat

au = T OS

'

au

av

=- P

'

now we take the first one and we take

BT

av but it also can be written as

au f)N



11 oo : oo : 47 avas

av

but it also can be written as 2

a u 8s8V [email protected]

aP 8S physicsguide CSIR NET, GATE

17

@Sk J ahiruddin, 2020

Nlaxwell's relation and Thermodynamic Potentials

and we know that

so the option a is correct. Sol 1. 7. Given that

G(T, P, N)

= U - TS + PV

we can say that dG

aG aG 8G aTdT + aPdP + aNdN

= = dU - TdS - SdT + PdV + VdP

and we know from t he first law of the t hermodynamics dQ Tds = dU + PdV and we are using this and get dG = -SdT

=

+ VdP

now we compare the co efficient of dT on both side and get

S =-

8G

BT

N, P

Sol 1.8. So we have if entropy as a function of pressure and

temperature then

11 oo : oo : so ac aT

N ,P

Sol 1.8. So we have if ent ropy as a function of pressure and temperature t hen

[email protected]

physicsguide CSIR NET, GATE

18

Maxwell's relation and Thermodynamic Potentials

@Sk J ahiruddin , 2020

now multiply both side by T we get T dS = T

as aP

dP + T T

as aT

dT p

so we get the fact dS = TV f3dP

+ epdT

so here we put t he fact

/3 =

! aV v aT now we are to calculate in const ant entropy t he term value

ap aT

Cp

TV/3

s

= 4.65

X

107

so N = 4.65 putting all the values of the given quantities. Sol 1. 9. T he area under t he curve is always denoting t he total number of the part icles and t he most probable velocity of the particle is proport ional to JT so more temperature gets the curve more steeper. So option a is correct. Sol 1.10. T he Helmholtz free function is defined as V

-

TT

rrc

11 oo : oo : s2 of the particle is proportional to JT so more temperature gets the curve more steeper. So option a is correct. Sol 1.10. T he Helmholtz free function is defined as

F = U-TS So the change in the Helmholtz free function is given by

dF = dU - TdS - SdT j [email protected]

19

@Sk J ahiruddin , 2020

physicsguide CSIR NET, GATE

:Niaxwell's relation and Thermodynamic Potentials

but we know that from t he first law of t hermodynamics

dQ

=

dU + P dV

=

T dS

and putting this in the equation we

dF = - P dV - SdT so keeping volume and temperature is constant will give us no change in the Helmholtz free energy and hence Helmholtz free function attains a local minimum. Sol 1.11. Chemical potential is t he change in t he internal

energy due to add or subtract of particle but in a system t he total number of particles is not conserved so chemical potential is zero. Sol 1.12. We have t hat for an ideal gas we get

av

8P

8T

8P r

8T v

av

p

=-1

so from this relation we get to know that A and C is correct

11 oo : oo : 55 Sol 1.12. We have t hat for an ideal gas we get

av

8P

8T

8P r

8T v

av

p

=

- 1

so from this relation we get to know that A and C is correct option. equation of state f(x,y,z)=O [equation of the state f(p,V,T )=O)

df =

8f

dx+

ax

y,z

8f 8y

[email protected]

dy+

az

x,z 20

@Sk J ahiruddin , 2020

8f

dz

=

0

y,x

physicsguide CSIR NET, GATE

Maxwell's relation and Thermodynamic Potentials

setting dx,dy,dz equal to zero, successively we get a set of equation like

8y

az

X

and again we have

az ax

y

fx fz

hence we get

av

8P

8T

8P r

8T v

av

=

- 1

p

Sol 1.13. From t he previous discussion we have seen t hat dG = -SdT + V dP and most importantly we have also see t hat one of the T dS relation is given by

TdS = CpdT - T

av

BT

r,

dP

11 oo : oo : sa e previous 1scuss1on we ave seen dG = -SdT + V dP and most importantly we have also see t hat one of the T dS relation is given by ~

av ar

TdS=CpdT- T

ii

dP p

one of the other TdS relation can be obtained by using the fact that s can be a function of volume and the temperat ure.so option b and d is correct . Sol 1.14 . From the previous discussion we get t his is Gibbs free energy.

j [email protected]

21

@Sk J ahiruddin , 2020

physicsguide CSIR NET I GATE

Maxwell's relation and Thermodynamic Potentials

Sol 1.15. We have

aQ

ar

aQ

ar

p

v

and we know that dQ= TdS and we are going to use this

as ar

- T p

as ar

v

and we use the

dS=

as av

dv + T

as ar

dT V

now we have so we

as ar

p

as av

T

av as + ar ar p

v

11 oo : 01 : oo T

V

now we have so we

as aT t hen

av as as + av raT P aT v

P

as as aT P aT v

as av avT ar

p

and we know from the maxwell relation

as av

aP ar

V

V

AND WE USE THIS RELATION IN THE ABOVE EQUATION Cp-C-u = T

[email protected]

av aP aTpaT v

22

@Sk J ahiruddin , 2020

physicsguide CSIR NET, GATE

l\/[axwell's relation and Thermodynamic Potent ials

for ideal gas we know t hat

av aP

r

aP ar

v

ar av

we use t his relation to replace the t ion and we get

p

=-1

(if)v in t he above equa-

T va 2 Gp= Cv+ kr where a and kr carries as usual meaning.

Sol 1.16. The porous plug experiment is essentially giving you a more important condition which is given by the change inn the enthalpy is zero and t he dQ=O so we have the ent halpy function is defined as

11 oo : 01

: 03

Sol 1.16. The porous plug experiment is essentially giving you a more important condition which is given by the change

inn the enthalpy is zero and the dQ=O so we have the ent halpy function is defined as

H = U + PV hence the change in t he enthalpy is given by

dH

=

dU + PdV + VdP

=

T dS + VdP

and we have dH= O and T dS = 0 so we have fin ally V dP = O now we know PV = nRT So, we can say

VdP + PdV = n RdT bu we know that vdP = 0 so we are getting

PdV = RdT [email protected]

@Sk J ahiruddin, 2020

23

physicsguide CSIR NET, GATE

Nlaxwell's relation and Thermodynamic Potentials

and from the ideal gas equation p = n RT/ V so putting this we get V2 dv = Tln V2 dT = T V1 V V1 Sol 1.1 7. In free expansion t he gas is allowed to expand in to a vacuum.t his happens quickly , so there is no heat t ransferred. as the gas expands in vacuum so there is no work done against the external pressure.so according to the first law t his means that internal energy of t he system is not changing so t he temperature is constant . The temperature remains the same. (some times it is also called irreversible

11 oo : 01

: 06

t ransferred. as the gas expands in vacuum so there is no work done against the external pressure.so according to the first law this means that internal energy of t he system is not changing so the t emperature is constant . The temperature remains the same. (some times it is also called irreversible isothermal expansion) Sol 1.18. As we know t hat t he internal energy is a funct ion of t mperature and volume as we are considering t he

vanderwalls gas and in the vanderwalls gas t he internal energy depends on t he volume so we have dU =

au

ar

dT + V

au

dV

av

r

and the change in the t otal internal energy is zero as the gas's internal energy is not changing so according to t his condiation we can say

au

ar

au

dT +

av

V

j [email protected]

24

@Sk J ahiruddin , 2020

r

dV =

o

physicsguide CSIR NET, GATE

Maxwell's relation and Thermodynamic Potentials

and we have TdS = dU +pdV

So, we have

T

as av

au

av

+p

and now we are going to use the maxwell's second equation

as av

TP '

aP ar

VS

'

11 oo :01 : oa and now we are going to use t he maxwell,s second equation

as av

8P 8T

TP )

VS

'

using t his in t he above equation will gives us

T aP -p= au

aT

av

from t he 2nd equation we can say t hat

au

0=

dV

av

T

from t his we can say that l

TJ

=~- -

Cv

VJ Vi

au

av

dV T

so temperature is decreasing. so option a and d is correct. Sol 1.19. At equilibrium say finally t he to sector comes to some equilibrium temperature say Tp 1 so for the first compart ment gas we can write

PoVo To [email protected]

@Sk J ahiruddin, 2020

25

physicsguide CSIR NET, GATE

Nlaxwell's relation and Thermodynamic Potentials

now Vo= H A and Vp = (H - y)A where we have consider t he fact t hat aft er t he weight has been released t he piston comes down y dist ance so t he first or say one of the compart ment will have increased lengt h by y amount where as ot her corn part ment will have decreased length by t he same amount of length and t he first corn partment has final pressure P P, and final temperature T P,. now we can sav t hat

11 oo : 01

: 11

comes down y dist ance so the first or say one of the compartment will have increased length by y amount where as other compartment will have decreased length by t he same amount of length and the first compartment has final pressure Pp1 and final temperature Tp1. now we can say t hat

PF l

_ Tp1 x P0 H To(H - y)

in similar way the final pressure in the second compartment is given by p _ Tp1 x P0H 2 F - To( H + y) in this case we say t hat the equilibrium temperature would be t he same for the both gas.now according to the given condition is pressure difference must be equal to ~ so mathematically

w

Pp1 - Pp2 = A

now again we are considering the fact t hat the process is adiabatic process so we will have dU = ev(TF1 - To) = Wy = dw

but we have t he condition(yW > ToCv so we get Tp1 = ~: now we will apply t his in t he previous equation of Pp1 and [email protected]

@Sk J ahiruddin, 2020

26

physicsguide CSIR NET, GATE

Maxwell's relation and Thermodynamic Potentials

get the following

which is giving us

1

1

H- y

H +y

1 A

11 oo : 01

: 13

get the following 1

1

H- y

H +y

1 A

which is giving us H

y=

Sol 1.20. From t he given equation of state 2

p = nRT _ a b3 v3

This sudden change of wall leads to free expansion of gas. For free expansion Change of internal energy is ZERO. From internal energy equation

au av

T

aP - P =T aT V TNR nRT b3 = a 2/ V 3

j [email protected]

@Sk J a hiruddin, 2020

27

b3

a2

+ V3

physicsguide CSIR NET , GATE

:Niaxwell's relation and Thermodynamic Potentials

Now write the internal energy in the differential form

dU = nCvdT+

au

dV

11 oo : 01

: 16

j [email protected]

27

@Sk J ahiruddin , 2020

physicsguide CSIR NET, GATE

Nlaxwell's relation and Thermodynamic Potentials

Now writ e the internal energy in the differential form dU

au

= nCvdT+ -

av

r

dV

a2

= n,CvdT + V 3 dV T1

=

nCv

dT

Vo

+

Vo/2

To

a2

V 3 dV = 0

as dU = 0

Hence The answer a2

Vo

=0

nCv(T1 - ~ ) - 2v2 Vo/ 2

=

n Cv(T1 - ~)

a2

1

2

(Vo/2) 2

2

hence

T.t = ~ -

j [email protected]

2n

3a C v, 2 V

0

28

physicsguide CSIR NET, GATE

11 oo : 01

: 1a

j [email protected]

27

@Sk J ahiruddin , 2020

physicsguide CSIR NET, GATE

Nlaxwell's relation and Thermodynamic Potentials

Now writ e the internal energy in the differential form dU

au

= nCvdT+ -

av

r

dV

a2

= n,CvdT + V 3 dV T1

=

nCv

dT

Vo

+

Vo/2

To

a2

V 3 dV = 0

as dU = 0

Hence The answer a2

Vo

=0

nCv(T1 - ~ ) - 2v2 Vo/ 2

=

n Cv(T1 - ~)

a2

1

2

(Vo/2) 2

2

hence

T.t = ~ -

j [email protected]

2n

3a C v, 2 V

0

28

physicsguide CSIR NET, GATE

11 oo : 01

: 21

j [email protected]

27

@Sk J ahiruddin , 2020

physicsguide CSIR NET, GATE

Nlaxwell's relation and Thermodynamic Potentials

Now writ e the internal energy in the differential form dU

au

= nCvdT+ -

av

r

dV

a2

= n,CvdT + V 3 dV T1

=

nCv

dT

Vo

+

Vo/2

To

a2

V 3 dV = 0

as dU = 0

Hence The answer a2

Vo

=0

nCv(T1 - ~ ) - 2v2 Vo/ 2

=

n Cv(T1 - ~)

a2

1

2

(Vo/2) 2

2

hence

T.t = ~ -

j [email protected]

2n

3a C v, 2 V

0

28

physicsguide CSIR NET, GATE

11 oo : 01

: 23

j [email protected]

27

@Sk J ahiruddin , 2020

physicsguide CSIR NET, GATE

Nlaxwell's relation and Thermodynamic Potentials

Now writ e the internal energy in the differential form dU

au

= nCvdT+ -

av

r

dV

a2

= n,CvdT + V 3 dV T1

=

nCv

dT

Vo

+

Vo/2

To

a2

V 3 dV = 0

as dU = 0

Hence The answer a2

Vo

=0

nCv(T1 - ~ ) - 2v2 Vo/ 2

=

n Cv(T1 - ~)

a2

1

2

(Vo/2) 2

2

hence

T.t = ~ -

j [email protected]

2n

3a C v, 2 V

0

28

physicsguide CSIR NET, GATE

11 oo : oo : 01

Problems and Solutions in Thermodynamic equation of State Sk J ahiruddin * Suchismito Chattopadhyay

*Assistant Professor Sister Nibedita Govt. College, Kolkata Aut hor was t he topper of IIT Bombay M.Sc Physics 2009-2011 batch He ranked 007 in IIT J AM 2009 and 008 (J RF) in CSIR NET J une 2011 He has been teaching CSIR NET aspirants since 2012

1

@Sk J ahiruddin, 2020

Thermodynamic equation of State

11 oo : oo : 03 1

Thermodynamic equation of State

@Sk J ahiruddin, 2020

Contents 1

Problems from NET, GATE, JEST , TIFR & JAM pap ers 1.1 Ans keys . 1.2 Solut ions . • • •

j [email protected]































2





3 •



































11 12

physicsguide CSIR NET, GATE

11 oo : oo : 06

j [email protected]

physicsguide CSIR NET I GATE

2

@Sk J ahiruddin , 2020

1

Thermodynamic equation of State

Problems from NET, GATE, JEST, TIFR & JAM papers

Prob 1.1. The internal energy of a system is given by E

=

tt,

where b is a constant and other symbols have t heir usual meaning. The temperature of this system is equal to [NET Dec 2011] 2 3 bS (a) bS2 (b) 3bS (d)( S )2 (c) v2N N VN VN Prob 1.2. For a part icular t hermodynamic system t he ent ropy S is related to t he internal energy U and volume V by S = cU3/4vl/4 where c is a constant. The Gibbs potential G = U - TS+pV for this system is [N ET June 2014]

(a) 3pU 4T

(b) cU

( )

3

c zero

(d) US 4V

Prob 1.3. A theoretical model for a real (non-ideal) gas gives the following expressions for the internal energy ( U)

and t he pressure ( P) U (T, V)

= av-

2 3 /

+ bV

2 3

1T

2

and

P (T , V)

= ~av- ! + ~bv- 1 T 5 3

1 3

2

11 oo :oo :og U (T , V )

= a v-

2 3 /

+ bV

2 3

1T

2

and

[email protected]

physicsguide CSIR NET ) GAT E

3

Thermodynamic equation of State

@Sk J ahiruddin ) 2020

where a and b are constants. Let Vo and To be t he init ial volume and initial temperature respectively. If t he gas expands adiabatically, t he volume of the gas is proport ional to [JEST 2018]

(a)T

(b)T 3/2

(d)T- 2

(c)T -3/2

Prob 1.4. Suppose that the number of microstat es available to a syst em of N part icle depends on N and t he com2

bined variable UV , where U is the internal energy and V is t he volume of the syst em. The syst em init ially has volume 2m 3 and energy 200J. It undergoes an isent1~opic expansion t o volume 4m 3 . What is the final pressure of t he system in

[JEST 2017]

SI units?

Prob 1.5. The equation of state of a gas is given by

V == RT _ !2_

p

T

where R is the gas constant and b is another constant paramet er. The specific heat at constant pressure C p and the sp ecific heat at constant volume Cv for t his gas is related by Gp - Cv = [TIFR 2015]

11 oo : oo : 11 where R is t he gas constant and b is anot her constant parameter. The sp ecific heat at constant pressure C p and the specific heat at constant volume Cv for this gas is related by Gp - Cv = [TIFR 2015] RT2 2 (a) R (b) R 1 + bP [email protected]

Thermodynamic equation of St ate

@Sk J ahiruddin, 2020

(c) R 1 +

physicsguide CSIR NET, GATE

4

2

bP RT2

Prob 1.6. An ideal gas consists of t hree dimensional polyatomic molecules. The temperature is such that only one vibrational mode is excited. If R denotes t he gas constant, t hen the specific heat at constant volume of one mole of t he [JAM 2018] gas at this t emperature is

(b) ;R

(a) 3R

(c) 4R

(d) ~R

Prob 1. 7. The free energy for a photon gas is given by F = -(~)VT4 , where a is a constant. The entropy Sand t he pressure P of t he photon gas are [GATE 2007]

4 c) S = a VT 4 P ( 3 '

(d) S

=

=

_aT 3 3

1aVT3 p = _ 4aT 4 3

'

3

Prob 1.8. The energy density and pressure of a photon gas are given by u = aT4 and P = u/3, where T is t he t emperature and a is the radiation const ant. The entropy per unit

11 oo : oo : 13 (d) S

=

laVT3 p = _ 4aT 4 3

'

3

Prob 1. 8. The energy density and pressure of a phot on gas 4 are given by u = aT and P = u/ 3, where T is t he t emper-

ature and a is t he radiation const ant. The ent ropy per unit volume is given by aaT 3. The value of a is ______ (up t o two

[GATE 2017]

decimal places)

j [email protected]

5

physicsguide CSIR NET, GATE

Thermodynamic equation of State

@Sk J ahiruddin , 2020

Prob 1 .9 . The ent ropy of a gas containing N particles en-

closed in a volume V is given by S = NkB ln

3 2 ;

a~f1

,

where E is t he total energy, a is a constant and kB is the Bolt zmann const ant. T he chemical potent ial µ of t he system at a t emperature T is given by [GATE 2015]

(a) µ = -kBT ln

(b) µ = - kBT In (c) µ = - kBT In

aV E 312 N 5/ 2

5 2

aVE 312

3 2

aVE 312

5 2

N 5/ 2

N 3/ 2

aV E 312 N 3/2

3 2

Prob 1 .10. The ent ropy function of a syst em is given by

S (E ) = aE (E o - E ) where a and E 0 are posit ive constant s. [GATE 2013] T he temp erature of t he system is

11 oo : oo : 16 Prob 1.10. The entropy function of a syst em is given by

S(E) = aE(Eo - E ) where a and E 0 are posit ive constants. The t emperature of the system is [GATE 2013] (a) negative for some energies (b) increases monotonically with energy (c) decreases monotonically with energy (d) zero [email protected]

physicsguide CSIR NET, GATE

6

Thermodynamic equation of State

@Sk J ahiruddin, 2020

Prob 1.11. The total energy, E of an ideal non-relativistic where Fermi gas in three dimensions is given by E ex

f:;:

N is the number of part icles and V is t he volume of the gas

Identify the CORRECT equation of state (P being the pressure) . [GATE 2012] (a) PV = ½E (b) PV = j E (c) PV = E (d) PV = i E Prob 1.12. A red star having radius rR at a temperature TR and a white star having radius rw at a temperature Tw ,

radiate the same total power. If these stars radiate as per[JAM 2019] fect black bodies, then (A) rR > rw and TR > Tw (C) rR > rw and TR < T1,v

(B) rR < rw and TR > Tw (D) rR < rw and TR < Tw

Prob 1.13. The equation of st ate for one mole of a non-

i ),

where t he coefficient ideal gas is given by PV = A (1 + A and B are temperature dependent. If the volume changes fr()rn 1/4 f() 1/n 1n ~ n l~flt.h Prrn~ l

nr()('P~ ~

t.h P 1xr()rk rl ()n P h1r

11 oo : oo : 19 Prob 1.13. The equation of state for one mole of a non-

i) ,

ideal gas is given by PV = A (1 + where t he coefficient A and B are temperature dependent . If the volume changes from V1 to ½ in an isothermal process, the work done by the gas is [JAM 2018] 1 1 ½ (a) AB -½ ½ (b) ABln Vi

(c) Aln

V2

V1

1 -1

+ AB

½

V2

[email protected]

7

(d) Aln

½-½ Vi

+B

physicsguide CSIR NET, GATE

T hermodynamic equation of Stat e

@Sk J ahiruddin , 2020

Prob 1.14. The equation of state for a gas is given by aN

p+

2

V

where P,V,T,N and kB represent pressure, volume,temperature, number of atoms and t he Boltzmann constant respectively, while a and /3 are constants specific to the gas. If the critical point C corresponding to a point of inflexion of t he p-V curve, then t he critical volume Ve and critical pressure Pc for t his gas are given by [TIFR 2016] 2 2 (a) Vc = 3/3 N , p0 = a /3/3

a/ 27/3 2 2 (c)Vc = 3(3 N , pc = 8a / 27/3 (d)Vc = 3(3 N , p0 = a 2 / 27/3 2 (b)Vc

=

3/3N, p0

=

11 oo : oo : 22 (a) Vc (b)Vc

= =

3f3 N ,Pc 3f3N,pc

= a =

/3/3

a / 27/3

2

2

(c)Vc = 3/3 N ,pc = 8a / 27(3 (d )Vc

=

3f3 N ,pc

=

a 2 /27/3 2

Prob 1 .15 . The molar equation of state of a gas at tem-

perature T , pressure P and volume V is given by

RT a p = V - b - TV 2 where, a and b are two constants and R is the gas constant. The critical temperature and pressure for this gas will be [TIFR 20 19]

j [email protected]

8

physicsguide CSIR NET 1 GATE

T hermodynamic equation of Stat e

@ Sk J ahiruddin, 2020

a RTc (a) Tc = Pc = 27Rb' b 8a RTc (b) Tc= Pc= 27Rb' 8b 4a RTc (c) Tc = Pc = 4b 27Rb ' 8a RTc (d) Tc = Pc= 3R ' 8b Prob 1 .16. The van der Wall's equation of stat e for a gas is given by (P

a

+ V 2 )(V -

b) = RT

where P,V and T represent the pressure, volume and t emperature respectively, and a and b are constant parameters.

11 oo : oo : 24 is given by

(P

a

+ V 2 )(V -

b) = RT

where P,V and T represent the pressure, volume and t emperature respectively, and a and b are constant parameters. At the critical point, where all the roots of the above cubic [NET equation are degenerate , the volume is given by June 2014] a a 8a (d) 3b (a) 9b (b) 27b2 (c) 27bR Prob 1.17. The van der Waals equation for one mole of a gas is (p + ; ) (V - b) = RT . The corresponding equation of state for n moles of t his gas at pressure p, volume V and temperature T , is [NET June 2018]

p +~

(a) (b)

2

2

(P

+~

2)

(V-nb)=n RT

(V - nb)

[email protected]

= n RT 9

@Sk J ahiruddin , 2020

(c) (d)

p +~: (V -nb)= RT (p+ ~2 ) (V-nb )= RT

physicsguide CSIR NET, GATE

Thermodynamic equation of State

11 oo : oo : 21

j [email protected]

10

@Sk J ahiruddin , 2020

1.1

physicsguide CSIR NET I GATE

Thermodynamic equation of State

Ans keys

1.1. b

1.6. c

1.11. b

1.16. d

1.2.

C

1.7. a

1.12.

C

1.17. a

1.3.

C

1.8. 1.33

1.13.

C

1.4. 25

1.9. a

1.14. d

1.5.

1.10. a

1.15. b

C

11 oo : oo : 29 1.8. 1.33

1.13.

1.4. 25

1.9. a

1.14. d

1.5. c

1.10. a

1.15. b

1.3.

C

[email protected]

11

physicsguide CSIR NET ) GATE

@Sk J ahiruddin ) 2020

1.2

C

Thermodynamic equation of State

Solutions

Sol 1.1. We know that dq

= TdS =

dE

+ pdV

but here we have fixed volume and hence dv = 0 so we easily can say that

11 oo : oo : 32 dq = TdS = dE

+ pdV

but here we have fixed volume and hence dv = 0 so we easily can say that aE =T

as

v

now we are going to use t he upper formulae and we are getting 2 3bs T = VN . so option b is correct.

Sol 1.2. We know from t he Maxwell's equations of t hermodynamics that

as

1

au

T

using this we get

T = 4 u1J4 v - 1J4 3c

and we have TdS=dU+ PdV

12

[email protected]

physicsguide CSIR NET, GATE

Thermodynamic equation of State

@Sk J ahiruddin , 2020

so we can easily say

as av and from this we get

P

r

r

11 oo : oo : 34 so we can eas1 y say

as av

P

T

T

and from t his we get

G = u - 4 u l J4v-l /4

cu 3J4vl /4+

X

3c 4 u1J4v - 114 x c u 3J4v - 3J4 x v = o 3c 4 Sol 1.3. We know for t he ad iabat ic process dq= O so t he first law of thermodynamics says that dU = -dw and we know t h at dw = pdV so we get

au dT 8T

au dV = [~ v-

+ 8V

3a

5/ 3

2bv- 113T 2 ]dV

+3

hence we get

from t h is we can easily integrate to get t he dependence of v on T and is given by

dT

T

2dV

---

3V

and we integrate both side and get Vex

2 3 T- /

SO opt ion c

is correct. j [email protected]

@Sk J ahiruddin , 2020

13

physicsguide CSIR NET, GATE

Thermodynamic equation of State

Sol 1.4. solution t he t h ermodynamic prob ability is given by

00: 00: 37 Thermodynamic equation of tate

Sol 1.4. solution the thermodynamic probability is given by

where b is merely a proportionality constant. now we know that

and as the process is isoentropic we get dS = 0 = bkB

+ 2 dV

dU U

V

which essentially gives me dU U

= _ 2 dV V

and we know t hat from the first law of thermodynamics we get dU

=

-pdV

so we get

p=

2U

V so init ially U =200J and V =2 cubic meter and hence we get

p=initially =200 pascal.now we know that TdS = 0 = dU [email protected]

@Sk J ahiruddin, 2020

which essentially gives us

14

+ pdV physicsguide CSIR NET, GATE

Thermodynamic equation of State

11 oo : oo : 40

@Sk J ahiruddin, 2020

Thermodynamic equation of State

which essentially gives us

au

av

s

=

- p

and we get pV= 2U

so we get

so we get pV 3

=K

and we we use this to get the final pressure 25 atm.

Sol 1.5. We have known before that

8P ov 8T v 8Tp Now we are going to derive the each term first let us calculate v . in order to do that we use

(if)

RT P=--

v +;

so we get

8P 8T v [email protected]

@Sk J ahiruddin , 2020

15

physicsguide CSIR NET, GATE

Thermodynamic equation of State

11 oo : oo : 42 V

15

[email protected]

physicsguide CSIR NET, GATE

Thermodynamic equation of St ate

@Sk J ahiruddin , 2020

and we get the

av 8T

R

b

p + r2

P

putt ing all this values we get bP

2

1 + RT2

Sol 1.6. We know that for the poly-atomic molecules the specific heat at constant volume is given by in 3 dimension c,;

= (3 + f)R

where f gives vibrational mode here Cv

f = l so we have

= 4R

Sol 1. 7. we know that F is defined as follows F = U-TS

so dF

= dU - T dS - S dT

and we have from the first law of the thermodynamics dQ = TdS = dU j [email protected]

16

+ P dV physicsguide CSIR NET, GATE

11 oo : oo : 4s dQ

=

TdS

j [email protected]

=

dU + P dV

16

physicsguide CSIR NET, GATE

Thermodynamic equation of State

@Sk J ahiruddin , 2020

so we replace this in the equation and we obtain

= -S

8F

8T

v

8F

8V

= -P T

using this we get from the first relation S

= 4aVT3 P = aT4 3

3

Sol 1.8. From t he previous several discussion we come to know that

dS= dQ T and we have from the first law of the thermodynamics dQ = dU +PdV now from the given relation we get 4

3

dU = 4aT dT

aT P =3

u 3

now using this relation in t he above equation

Now we know for t he photon gas

PV = ~U 3

11 oo : oo : 48 + -T Now we know for t he photon gas

PV= ~U 3 [email protected]

physicsguide CSIR NET, GATE

17

Thermodynamic equation of State

@Sk J ahiruddin, 2020

from t his we can say that

P dV

= ~dU - VdP

to simplify this we use V = 3~ and putting instead of V in the above equation and we get

P dV =

1 -dU -dP 3

and we know that P = U/ 3 so workdone is actually zero. and we are left with

dS = 4a

2

T dT = iaT 3 3

so a= 1.33 Sol 1.9. We know t hat

µ =-

8G

8N

S =-

8G 8T

from t he second relation we get to know t hat G that is Gibbs free energy can be written as follows /

... ?",........") / ' ) ,

11 oo : oo : so µ=-

S=-

8N

ar

from t he second relation we get to know t hat G that is Gibbs free energy can be written as follows

G [email protected]

dT

N 5/2

physicsguide CSIR NET, GATE

18

Thermodynamic equation of Stat e

@Sk J ahiruddin, 2020

so we can write aVE 312 Ns/ 2

T

+A

where A ia just a mere integration constant . now from the first relation we get

aNk B ln µ=

aVE3/ 2 N s/2

8N

and hence we get

[ actually µ calculates the change in the Gibbs energy due to adding or subtracting particle from the system t hat is we are considering here t hat number of particle also can change so in general t he change in the Gibbs energy is defined as dG = V dP - SdT - µdN

]

11 oo : oo : s2 so in general t he change in the Gibbs energy is defined as dG = V dP - SdT - µdN

] Sol 1.10. We use t he relation

as

1 T

8E v j [email protected]

19

physicsguide CSIR NET 1 GATE

Thermodynamic equation of State

@Sk J ahiruddin , 2020

t his gives us 1 T = a(Eo - E)

+ (-a E)

a(Eo - 2E)

=

now when E < Ea Then temperature would be negative so for some case temperature is negative. Sol 1.11. We know from t he first law of thermodynamics

dQ = dE+pdV

t he total energy is given for t he system so we have dQ= O and t his giving us p= -

8E

av

N

2E -3V

hence t he equation of t he state is given by 2 PV = E 3

11 oo : oo : 55 hence t he equation of t he state is given by

PV =

2

3

E

Sol 1.12. Black body radiation is a remarkable discovery of t he modern science. when heated a solid glows and emits

t hermal radiation but it is different from the radiation emitted by a gas. when a solid radiates t hen it's sharp contrast [email protected]

@Sk J ahiruddin , 2020

20

physicsguide CSIR NET, GATE

Thermodynamic equation of State

of continuous radiat ion pattern with that of t he discrete pattern structure of a gas really bound us to investigate the problem. a perfect black body is that body which absorbs all t he radiation failing in it so appears to be black when subjected to some external radiation. when t hese perfect black bodies are heated t hen it radiates energy and the radiation process follows a particular continuous pattern. A perfect black body is a good absorber as well as a good radiator. (which is actually the core idea or core t hought of the Kirchhoff 's law of black body radiation) Max Plank was t he first person to give the satisfactory theoretical justification to t he black body radiation considering t he fact t he bodies radiates in small packets of energy (which is known as quanta) . he first showed t hat energy density per unit wave length is can be written as 1

11 oo : oo : s1 body radiation considering the fact t he bodies radiates in small packets of energy (which is known as quanta) . he first showed t hat energy density per unit wave length is can be written as 81rhc 1 u(A, T ) = A5 ehc/ >.kT _ 1 and hence the energy radiated by the blackbody at temperature T per unit area per unit time can be obtained 00

00

uv, Tdv = 0

0

81r hv3 c3(ehv/ kT _

1)

dv = 4 a-T4 c

and this is the Stephan Boltzman law and the maximum of j [email protected]

@Sk J ahiruddin , 2020

physicsguide CSIR NET I GATE

21

Thermodynamic equation of St ate

t he energy density curve satisfies the following relation

this is known as the Wien's displacement law. This summarizes the whole black body radiation. now coming to this particular problem. the red star has maximum wave length say Ar and the white star has Aw . according to the Wien's displacement law

and this enables us to conclude that

11 oo : 01 : oo and t his enables us to conclude t hat

so we get

now t hey are emitting same power so we have 4

4

E = - aTw x 41rrw

2

C

t his is t he energy radiated by the white star. the energy radiated by the red star

[email protected]

physicsguide CSIR NET ) GATE

22

@Sk J ahiruddin ) 2020

Thermodynamic equation of State

but according to t he given condition t hey are both emitting same power so we get

TR

4

TR

2

=

'77 4 2 .L W rw

which enables us to conclude t hat 2

> 1

so we must have rR > rw Sol 1.13. The work done in t he isothermal process is given by V2

pdv

=

A+

AB

_ _v_dV 1/

11 oo : 01

: 03

so we must have rR > rw Sol 1.13. The work done in t he isothermal process is given by V2

pdv

=

A+

AB

_ _v_dV V

integrating we get

Sol 1.14. There exists a critical temperature for any gas and p-V curve at t his temperature is called the critical temperature Tc. We know that in van der Walls t heoretical

curve , minima and maxima appear and they approach each other as we consider higher and higher isothermal process [email protected]

physicsguide CSIR NET, GATE

23

Thermodynamic equation of State

@Sk J ahiruddin, 2020

.At a particular temperature t he maxima and minima coalesce to a single point called t he critical point.the pressure and volume corresponding to t hat critical point are called critical pressure and critical volume. Here we have the van der Walls equation of state of the real gas and at crit ical point we have

8P

av a2 p av2

=0 =0

11 oo : 01

: 06

8P

av a2p

av

2

=0 = 0

from t he given equation of state we can say that aN V

2

so according to the first relation we have

in similar way the second relation giving us

from third and fourth equation we can easily conclude that t he critical volume Ve = 3f3 Nnow we can put t his value of j [email protected]

physicsguide CSIR NET, GATE

24

Thermodynamic equation of State

@Sk J ahiruddin, 2020

Ve

in the 3rd equation and we get 2

8a Tc= 27f3 K B and putting t his both value in expression of Pressure obtained from the equation of state will give us a2

p

option d is correct.

= 27/3 2

11 oo :01 : oa tained from t he equation of state will give us 0'.2

p= 27/3 2

opt ion d is correct. Sol 1.15.

8P

=0

av f)2 p

= 0

av 2 t he 1st equation gives us 2a TV 3

RT (V - b)2

and t he 2nd equation giving us

RT (V - b)3

3a TV 4

now we get by doing 7/ 8 Ve = 3b and putt ing t his in t he 3rd equation will give us Tc =

2~~b

and putting all in this in

t he equation of state will give us Pc = correct. [email protected]

@Sk J ahiruddin, 2020

25

~fc. so option b is

physicsguide CSIR NET, GATE

Thermodynamic equation of State

Sol 1.16. From t he discussion of the problem 115 we get t o see that the crit ical volume is Ve = 3b. so option d is

correct . Sol 1.1 7. For n mole real gas t he opt ion A is t he equation of state as t he correction terms to be evaluated for n mole gas.

11 oo : 01

: 11

correct. j [email protected]

25

physicsguide CSIR NET, GATE

Thermodynamic equation of St ate

@Sk J ahiruddin, 2020

Sol 1.16. From t he discussion of the problem 115 we get to see that t he crit ical volume is Ve = 3b. so option d is

correct. Sol 1.17. For n mole real gas the option A is t he equation of state as t he correction terms to be evaluated for n mole gas.

j [email protected]

26

physicsguide CSIR NET, GATE

11 oo : 01

: 13

correct. j [email protected]

25

physicsguide CSIR NET, GATE

Thermodynamic equation of St ate

@Sk J ahiruddin, 2020

Sol 1.16. From t he discussion of the problem 115 we get to see that t he crit ical volume is Ve = 3b. so option d is

correct. Sol 1.17. For n mole real gas the option A is t he equation of state as t he correction terms to be evaluated for n mole gas.

j [email protected]

26

physicsguide CSIR NET, GATE

11 oo : 01

: 1s

correct. j [email protected]

25

physicsguide CSIR NET, GATE

Thermodynamic equation of St ate

@Sk J ahiruddin, 2020

Sol 1.16. From t he discussion of the problem 115 we get to see that t he crit ical volume is Ve = 3b. so option d is

correct. Sol 1.17. For n mole real gas the option A is t he equation of state as t he correction terms to be evaluated for n mole gas.

j [email protected]

26

physicsguide CSIR NET, GATE

11 oo : oo : oo

Problems and Solutions



Ill

Entropy Calculation Sk J ahiruddin * Suchismito Chattopadhyay

*Assistant P rofessor Sister Nibedita Govt. College, Kolkata Author was the topper of IIT Bombay 1\11.Sc Physics 2009-2011 batch He ranked 007 in IIT J Al\lI 2009 and 008 (JRF) in CSIR NET J une 2011 He has been teaching CSIR NET aspirants since 2012

1

@Sk J ahiruddin, 2020

Ent ropy Calculation

11 oo : oo : 03 1

Entropy Calculation

@Sk J ahiruddin, 2020

Contents 1

Problems from NET, GATE, JEST , TIFR & JAM pap ers 1.1 Ans keys . 1.2 Solut ions . • • •

j [email protected]































2





2 •



































12 13

physicsguide CSIR NET, GATE

11 oo : oo : os

j ahir@physicsguide. in

@Sk J ahiruddin , 2020

1

2

physicsguide CSIR NET I GATE

Entropy Calculation

Problems from NET, GATE, JEST, TIFR & JAM papers

Prob 1.1. The internal energy E )T ) of a system at a fixed volume is found to depend on the temperature T as E (T ) = aT 2 + bT 4 . T hen t he ent ropy S (T ), as a function of t emperature is [NET June 2016] 2 4 2 4 3 (a) ~aT +!bT (b) 2aT +4bT (c) 2aT +ibT (d) 2aT + 2bT 3 Prob 1.2. The heat capacity Cv at constant volume of a met al, as a function of t emperature, is aT + f3T 3 , where a and f3 are const ants. The temperature dependence of t he entropy at constant volume is [NET D e c 2018]

(a) aT + 1f3T 3 3 (c) ~aT + 1f3T

(b) a T + f3T 3 3 (d) ~aT + ! f3T

Prob 1.3. In a syst em comprising of approximately 1023 distinguishable particles, each particle may occupy any of 20 distinct states. The maximum value of the entropy per particle is nearest to [NET June 2019] (a) 20kB (b) 3kB (c) 10(ln2)kB (d) 20(ln2)kB Prob 1.4. Consider an ideal gas of mass m at temperature

11 oo : oo : os [NET June 2019] (c) lO (ln 2)kB (d) 20(ln 2)kB

particle is nearest to (a) 20kB (b) 3kB

Prob 1.4. Consider an ideal gas of mass mat temperature [email protected]

physicsguide CSIR NET ) GATE

3

@Sk J ahiruddin ) 2020

EntrOP)' Calculation

T1 which is mixed isobarically (i.e. at const ant pressure) wit h an equal mass of same gas at t emperature T2 in a thermally insulat ed container. What is the change of entropy of t he universe? [JEST 2012] (a) 2mCp ln( Ti+ T2 ) (b ) 2mCp ln( Ti - T2) 2~ 2~ 1 1 2 (c) 2m Cp ln(T + T ) (d) 2m Cp ln(T + T2 ) 2T1T2 2T1T2 Prob 1.5. A 100 ohms resistor carrying current of 1 amp is maintained at a constant t emperature of 30°C by a heat bath. What is the rat e of entropy increase of the resistor? [JEST 2014] (a) 3.3 Joules/K/sec

(b)6.6 Joules/K/sec

(c)0.33 Joules/K/sec

(d) None of t he above

Prob 1.6. A certain amount of fluid with heat capacity Gp joules/°C is initially at a t emperature 0°C. It is then brought into contact with a heat bath at a t emperature of 100°C , and the system is allowed t o come into equilibrium. In t his process, t he ent ropy (in J oules/°C) of t he Universe changes by [TIFR 2013] (a) lOOCp (b) 0 (c) 0.055Cp (d) 0.044Cp Prob 1. 7. Consider a sealed but t hermally conducting con('I





,

,

TT

,



,





., ., .

. , '

11 oo : oo : 11 In this process, the entropy (in J oules/°C) of the Universe [TIFR 2013] changes by (a) l OOCp (b) 0 (c) 0.055Cp (d) 0.044Cp Prob 1. 7. Consider a sealed but t hermally conducting container of total volume V, which is in equilibrium with a [email protected]

4

physicsguide CSIR NET, GATE

Entropy Calculation

@Sk J ahiruddin , 2020

t hermal bath at t emperature T. The container is divided into two equal chambers by a thin but impermeable partit ion. One of these chambers contains an ideal gas , while [TIFR 2012] t he other half is vaccum (see figure)

gas

vacuu

If the partition is removed and ideal gas is allowed to expand and fill the entire container , t hen the entropy per molecule of the system will increase by an amount

Prob 1.8. Experimental measurements of heat capacity per mole of Aluminum at low temperatures show t hat the data can be fitted to t he formula Cv = aT + bT3 , where a =

11 oo : oo : 13 Prob 1.8. Experimental measurements of heat capacity p er

mole of Aluminum at low temperatures show t hat t he data can be fitted to t he formula Cv = aT + bT 3 , where a = 0.00135J K- 2moze- 1 , b = 2.48 x 10-5 J K- 4 moze- 1 and T is t he temperature in Kelvin. THe ent ropy of a mole of Aluminum at such temperat ures is given by t he formula [JAM j [email protected]

5

physicsguide CSIR NET, GATE

@Sk J ahiruddin , 2020

Ent ropy Calculation

2007]

(a)aT + i T 3 + c , where c > 0 is a const ant (b) + 3 + c , where c > 0 is a constant 3 (c)aT + t T + C

ai :r

(d)ai +

1T3 +

C

Prob 1.9. 2 kg of a liquid ( specific heat= 2000JK - 1 kg- 1 ,indepc

of temperature ) is heated from 200K to 400K by eit her of t he following two processes P 1 and P 2 : P 1 :bringing it first in contact wit h a reservoir at 400K. P 2 :bringing it first in contact wit h a reservoir at 300K t ill equilibrium is reached , and t hen bringing it in contact with another reservoir at 400K. Calculate the change in the entropy of the liquid and t hat of the universe in processes p 1 and P2 . Neglect any change in volume of the liquid. [JAM 2012] Prob 1.10. Consider the free expansion of one mole of an

11 oo : oo : 16 of t he universe in processes p 1 and P2 . Neglect any change in volume of the liquid. [JAM 2012] Prob 1.10. Consider t he free expansion of one mole of an ideal gas in an adiabatic container from volue V1 to ½ . The ent ropy change of the gas, calculated by considering a reversible process between t he original state (Vi, T ) to the final state (½, T ) where T is t he temperature of the system , is denoted by ~ S 1. The corresponding change in the [email protected]

@Sk J ahiruddin, 2020

6

physicsguide CSIR NET, GATE

Ent ropy Calculation

entropy of t he surrounding is ~ S 2 . Which of t he following [JAM 2011] combinations is correct ? (a) ~ S1 = R ln (V1/½), ~ S2 = -R ln (Vi/V2) (b) ~ S1 = -Rln(V1/V2), ~ S2 = R ln(Vi/V2) (c) ~ S1 = Rln(V2/V1), ~ S2 = 0 (d)~ S1 = - Rln(V2/V1), ~ S2 = 0 Prob 1.11. A solid met allic cube of heat capacity S is at temperature 300K. It is bought in contact wit h a reservoir at 600K. If t he heat t ransfer takes place only between the reservoir and the cube, the entropy change of t he universe [JAM 2014] after reaching t he t hermal equilibrium is (a)0.69S (b)0.54S (c)0.27S (d)0. 19S Prob 1.12. One gram of ice at 0°G is melted and heated to water at 39°G. Assume t hat t he specific heat remains constant over t he ent ire process. The latent heat of fusion

11 oo : oo : 1s (a)0.69S

(b )0.54S

(c)0.27S

(d)0. 19S

Prob 1.12. One gram of ice at 0°G is melted and heated

to water at 39°G. Assume t hat the specific heat remains constant over the ent ire process. The latent heat of fusion of ice is 80 Calories/ gm . The entropy change in the process [JAM 2015] (in Calories per degree) is Prob 1.13. A rigid and thermally isolated tank is divided into two compartments of equal volume V , separated by a t hin membrane. One compartment contains one mole of an

ideal gas A and the other compartment contains one mole [email protected]

@Sk J ahiruddin , 2020

7

physicsguide CSIR NET, GATE

Entrop)' Calculation

of a different ideal gas B. The two gases are in t hermal equilibrium at a temperature T. If t he membrane raptures, t he two gases mix. Assume that the gases are chemically inert. The change in t he total entropy of t he gases on mixing is [JAM 2015] (a) 0 (b) R In 2 (c) ~ R In 2 (d) 2R ln 2 Prob 1.14. Consider two identical, finite, isolated systems of constant heat capacity C at temperature T1 and T 2 (T1 >

T 2 ). An engine works between them until t heir temperatures become equal. Taking into account that the work performed by the engine will be maximum (= W max). if t he process is reversible (equivalently, the entropy change of the entire system is zero), the value of Wmax is: [JAM 2017]

11 oo : oo : 21 by the engine will be maximum (= Wmax). if t he process is reversible (equivalently, the ent ropy change of the ent ire system is zero) , t he value of Wm a,x is: [JAM 201 7]

(d) C(PJ - P2) 2 j [email protected]

@Sk J ahiruddin, 2020

8

physicsguide CSIR NET 1 GATE

Ent ropy Calculation

Prob 1.15. Each of t he two isolated vessels, A and B of fixed volumes, contains N molecules of a perfect monatomic gas at a pressure P. The temperatures of A and B are T1 and T2 respectively. The two vessels are brought into t hermal contact . At equilibrium, t he change in entropy is [GATE 2006]

Prob 1.16. In a thermally insulated container , 0.01kg of ice at 273K is mixed with 0.1kg of v\rater at 300K. Neglecting t he specific heat of the container , the change in t he

11 oo : oo : 24 (c) ~NkB ln

(~~i) 2

(d ) 2NkB

Prob 1.16. In a thermally insulated container , 0.01kg of ice at 273K is mixed with 0.1kg of V\rater at 300K. Neglect-

ing t he specific heat of the container, the change in the entropy of t he system in J / K on attaining thermal equilibrium (rounded off to two decimal places) is _____ [GATE 2019] (Specific heat of water is 4.2kJ /kg - K and t he latent heat of ice is 335kJ / kg ) . Prob 1.17. The ent ropy S of a system of N spins, which may align either in t he upward or in the downward direc-

t ion, is given by S = - kB N[p lnp + (1 - p) ln(l - p)]. Here kB is t he Boltzmann constant. The probability of alignment

[email protected]

@Sk J ahiruddin, 2020

9

physicsguide CSIR NET, GATE

Entropy Calculation

in the upward direction is p . The value of p , at which the entropy is maximum, is _____ . (Give your answer upto one decimal place)

[GATE 2016]

Prob 1.18. N atoms of an ideal gas are enclosed in a container of volume V . The volume of t he container 4V , while

keeping the total energy const ant. The change in t he ent ropy of the gas, in units of NkB In 2, is ---, where kB is the Boltzmann constant. [GATE 2016]

11 oo : oo : 26 keeping t he total energy constant. The change in t he ent ropy of t he gas, in units of NkB In 2, is ---, where kB is the Boltzmann constant . [GATE 2016]

Prob 1.19. A system of N non-interacting distinguishible particle of spin 1 is in t hermodynamic equilibrium. The en[GATE 2011] t ropy of t he system is (a) 2kBlnN (b) 3kB lnN (c) NkB ln2 (d) NkB ln3 Prob 1.20. Consider a syst em of N atoms of an ideal gas of type A at t emperature T and volume V. It is kept in diffusive contact with another syst em of N atoms of another ideal gas of type B at the same temperature T and volume V. Once t he combined syst em reaches equilibrium , [GATE 2008]

(a) t he total entropy of the final system is t he same as the sum of the ent ropy of the individual system always j [email protected]

10

physicsguide CSIR NET I GATE

@Sk J ahiruddin , 2020

Entropy Calculation

(b) t he entropy of mixing is 2N kB In 2 (c) the entropy of the final system is less t han t hat of the sum of the init ial entropies of the two gases. (d) the entropy of mixing is non-zero when the atoms A and B are of t he same type.

Prob 1.21. An ideal gas with adiabatic exponent , undergoes a process in which its pressure P is related to its volume V by t he relation P = P0 - a V , where P0 and a are posi-

11 oo : oo : 29 B are of t he same type. Prob 1.21. An ideal gas wit h adiabatic exponent , under-

goes a process in which its pressure P is related to its volume V by the relation P = P0 - a V , where P0 and a are posit ive constants. The volume st arts from being very close to zero and increase monotonically t o ~. At what value of t he volume during t he process does t he gas have maximum entropy? [JEST 2016] Po (b) , Po , Po d Po

(a)a(l+,1)

a( l -,)

(c)a(l+,)

( )a(l -,)

Prob 1.22. The value of entropy at absolute zero of t em-

perature would be [JAM 2005] (a) zero for all t he materials (b) finite for all the materials (c)zero for some materials and non-zero for others (d)unpredictable for any material

[email protected]

physicsguide CSIR NET ) GATE

11

@Sk J ahiruddin ) 2020

1.1

EntrOP)' Calculation

Ans keys Ans keys

1.1.

C

1. 7.

C

1.12. 0.39

1.18. 2

1.13. d

1.19. d

1.2. a

1.8. a

1.3. b

1.9. Do your- 1.14. d

1 _20_ h

11 oo : oo : 31 1.1.

1. 7.

C

C

1.12. 0.39

1.18. 2

1.2. a

1.8. a

1.13. d

1.19. d

1.3. b

1.20. b

1.4. a

1.9. Do your- 1.14. d self 1.15. C

1.5.

1.10.

1.16. 1.03

1.21.

1.17. 0.5

1.22. a

C

1.6. d

C

1.11. d

12

[email protected]

physicsguide CSIR NET, GAT E

Entropy Calculation

@Sk J ahiruddin, 2020

1.2

C

Solutions

Sol 1.1. We know t hat dq

== TdS == dE + pdV

11 oo : oo : 34 ons Sol 1.1. We know t hat

dq = TdS = dE

+ pdV

but here we have fixed volume and hence dv=O so we easily can say that 1 8E v T now we are going to use the upper formulae as Eis given in t he problem. dE dS= T and we have 3 dE = 2aTdT + 4bT dT

as

we will put this in t he integration and we get S(T )

=

. So option c is correct. Sol 1. 2. Assuming t hat t he volume is not changing we will

get

dU dS = T which gives us the right option is option a where c is an integration const ant. j [email protected]

@Sk J ahiruddin , 2020

13

physicsguide CSIR NET, GATE

Entropy Calculation

Sol 1.3. We know that if there is N number distinct of sates in a system and n umber of particles in that syst em t hen

t here is accessible st ates in the syst em is given by

11 oo : oo : 36 , 2020

Entropy

alculation

Sol 1.3. We know that if there is N number distinct of sates in a system and n umber of particles in t hat system t hen

t here is accessible st ates in the syst em is given by

we use t his formulae to evaluate t he entropy of the system as we ha,,e known that ent ropy is related to the microst ates of t he system and as t he microstates increases the entropy of the syst em also increases in t hat sense we can say that t he entropy measures t he disorderness in t he syst em and the relation is given by

s = kB lnn so the ent ropy is given by

S

= kB

ln(20

1023

=

)

23

23

10 kBln20 = 10 kB3

so t he ent ropy per molecule is given by

Sol 1.4. First of all t he fact that we have to calculate the

final temperature TJ

= T1 + T2

2 t he ent ropy gain by t he gas at temperature T1 is given by dSgain =

mCp

T1

T1 [email protected]

@Sk J ahiruddin, 2020

and t he entropy loose

14

dT

T

T1

=

mcplnT1

physicsguide CSIR NET, GATE

Ent ropy Calculation

11 oo : oo : 39 @Sk J ahiruddin, 2020

EntrOP)' Calculation

and t he ent ropy loose

dSzose = mep non total change is

+ dszoss =

dsgain

2mcp ln

Sol 1. 5. To calculate t he increase of ent ropy you have to understand first why the heat is increasing? as t he heat is

increasing the disorderness in t he system also will increase and hence t he entropy must increase. now t he resistor is maintained at constant t emperature (273+30)Kelvin t emperature. and 1 amp current is flowing t hrough it so it must consists of joule heat energy and it is given by

H

=

2

i R

=

2

1 x 100 = lOOJ

so t he increase rate of entropy is given by 100 ds = T = = 0.33J oules / k / sec 303 dQ

Sol 1.6. The heat bath will be at constant temperature as we are assuming that it is an endless source of heat energy

comparing to t he body so at equilibrium the body temperature become equal to the heat bath temperature. So the [email protected]

@Sk J ahiruddin , 2020

15

physicsguide CSIR NET, GATE

Entrop)' Calculation

11 oo : oo :42 15

[email protected]

physicsguide CSIR NET, GATE

Ent ropy Calculation

@Sk J ahiruddin, 2020

loss of the entropy of the heat bath is given by 373 - 273 100 dszoss = Gp = Gp 373 373

and t he gain of the entropy is given by 373

ds 9 ain =

dT

Gp T = Gp ln

273

373 273

so change in t he ent ropy of the universe= 100 dSroTAL = -Gp 373

373 + Gp In 273 = 0.044Gp

Sol 1. 7. The gas has been allowed to expand in the vacuum so it is t he free expansion and the temperature of t he gas is remaining unchanged so the change in entropy is given by V

dS

=

pdvT

= Rln2

V/ 2

it is t he total change of the gas system so t he change in entropy per molecule is

j [email protected]

16

physicsguide CSIR NET, GATE

11 oo : oo : 4s

j [email protected]

16

physicsguide CSIR NET, GATE

@Sk J ahiruddin , 2020

Entropy Calculation

Sol 1.8. Assuming that the volume is not changing we will get dU dS =

T

which gives us the right option is option a where c is an integration constant.

Sol 1.9. For an ideal liquid change in the volume , dV using t he entropy ds = dU

T

and specific heat at constant volume. we have 2

ds

= 1

evdT

T

so from this we know

for an isother·mal process change in entropy S

1

Qr = 300

2

Qr = 400

and S

=0

11 oo:oo:47 and S

2

[email protected]

Qr = 400 physicsguide CSIR NET, GATE

17

Entropy Calculation

@Sk J ahirudd in, 2020



using ~U + ~KE + ~ pE

=

Q- w

so we get 1 1 bS = - 2000 = 5 400 200 Sol 1.10. Free expansion of one mole of an ideal gas in adiabatic container from v1 ,T to v 2 ,T and T is not changed. during isothermal process ds1

v2

=

V1

pdV

=

Rln v2

T

V1

and t he change of entropy for the surroundings we get ds2

=

dQ

=0

T

AS the adiabatic container no heat was coming out side t he container and no heat goes inside t he container so the entropy of the surrounding remains same. Sol 1.11. Entropy loss for t he reservoir 600k where we t ake t hat the reservoir becomes at constant temperature .

~s _ s(600 loss -

300)

600

and t he entropy gain from _ann

__

11 oo : oo : so t hat the reservoir becomes at constant temperature . l::l.S

_ loss -

s(600 - 300) 600

and t he entropy gain from

dT

600 l::l.Sgain

=C

T

300 [email protected]

physicsguide CSIR NET, GATE

18

@Sk J ahiruddin , 2020

Entrop)' Calculation

so total change 6S = s(-0.5

+ ln 2) =

0.19s

[here we take reservoir temperature is constant as it is endless source of heat energy so just the body temperature increase and in equilibrium it 's temperature would be equal to reservoir temperature] Sol 1.12. T he gain of entropy

dS . - (0.001

X

335)

273

gain -

39 273

+

+

0.001

X

4.2dT

T

273

So, We get 0.0012 + 0.0005 = 0.0017 J/ K. SO the change in the entropy = gain in entropy + loss in entropy = 0.39 cal per degree Celsius Sol 1.13. Change of entropy of the gas 1

dS=

CvdT

T +

2v

R

=

p

-dV V

T

as the gas has initial volume v and after mixing the total vol-

11 oo :oo : s2 dS

=

CvdT T

+

2v

R V

p

-dV T

as the gas has init ial volume v and after mixing t he total volume is available to the gas but as the temperature remains unchanged so the the first term is zero but the second term dS1 j [email protected]

=

Rln2

19

physicsguide CSIR NET 1 GATE

Ent ropy Calculation

@Sk J ahiruddin, 2020

and same for t he second gas also so we are getting again dS2

=

Rln2

so t he mixture has entropy more t hat previous configuration 2Rln2.

Sol 1.14. From the previous discussions on the Carnot cycle gave us the in sight that during this reversible process t he total change in the entropy is ds = 0 and that leads us to t he the fact that t he final t emperature of t he hot source or cold source becomes ,JT;T;, so the heat that has been taken from the hot source is

and t he heat rejected at t he cold source is

SO total workdone

11 oo : oo : 55 and t he heat rejected at t he cold source is

SO total workdone

[email protected]

@Sk J ahiruddin, 2020

20

physicsguide CSIR NET, GATE

Entropy Calculation

Sol 1.15. First of all the fact that we have to calculate the final t emperature Tf = T1 + T2 2 t he entropy gain by t he gas at temperature T1 is given by

dSgain = NCv and t he entropy loose dSzose = NCv non total change is

11 oo :oo : s1

Sol 1.16. Let us first calculate the final temperature after t he mixing. it is just the use of the calorimetry.

(0.01 x 335)

+ 0.01

x 4.2(T1 - 273) = 0.1 x (300 - T1)

from this we get that t he final temperature is 290.29kelvin. now let us calculate the loss of the entropy for the water at j [email protected]

physicsguide CSIR NET I GATE

21

Entropy Calculation

@Sk J ahiruddin , 2020

300k and we get 290

dSloss

= 300

0.1 X 4.2dT T

and the gain of entropy dS . _ (0.01 x 335) gain 273

290

+

273

0.01 x 4.2dT T

SO the change in the ent ropy =gain in entropy entropy= 1.03 Joule/K.

+

loss in

Sol 1.17. For a particular value of the entropy is maximum

so we have ds = O dp

and t his condition will be giving us

00 : 01 : 00 so we have ds = O dp and t his condit ion will be giving us lnp + l - ln( l - p) - 1 = 0 and t his gives us that p = 1/ 2. Sol 1.18. As t he total energy of the system remains constant so t he temperature of t he gas remains constant and prom the previous discussion we can say t hat t he change in

t he entropy is just due t he enhancement of the volume of t he gas and is given by 4v

4v

pdV = NKB V

V

[email protected]

@Sk J ahiruddin ) 2020

22

dv R - = 2N K Bln 2. V

physicsguide CSIR NET ) GAT E

EntrOP)' Calculation

so t he correct ans is 2. Sol 1.19. We know that the ent ropy of the system is depending on the thermodynamic probability of a system . What is t hermodynamic probability? it is t he number of microstates accessible t o t he system. t he more is t he number of microst at es the more will be t he entropy so in a way

you can say that this entropy is a measure of disorderedness in a system as the more t hermodynamic probability t he more is t he disorderedness. one can show that t hat the t he entropy is related to t he thermodynamic process probability through t he given relation

11 oo : 01

: 02

t he more is t he disorderedness. one can show that t hat the t he entropy is related to t he thermodynamic process pro bability t hrough t he given relation

where D is t he thermodynamic probability of t he system. for a spin one system the number of accessible state of the syst em is given by

n = (2J + 1) = 3 as J=l . Now we can say that

as the system consists of N particles. [email protected]

23

physicsguide CSIR NET, GATE

@Sk J ahiruddin, 2020

Ent ropy Calculation

Sol 1.20. Change of entropy of the gas A is

dS=

CvdT

T +

2v

NK

p

-dV

B V

T

as the gas has initial volume v and after mixing the total volume is available to t he gas but as the temperature remains unchanged so t he the first term is zero but t he second term

and same for t he second gas also so we are getting again

11 oo : 01 : os unchanged so t he the first term is zero but t he second term •



and same for t he second gas also so we are getting again

so t he mixture has entropy more t hat previous configuration 2NKBln2 . Sol 1. 21. The process is related as follows

P = Po - aV the entropy of the system can be obtained ds

= dU + P dV T

T

now we know t hat P V= RT j [email protected]

24

physicsguide CSIR NET, GATE

Entropy Calculation

@Sk J ahiruddin , 2020

so we get P dV

+ Vd P =

R dT

and we al so know t hat R Cv = - '°Y - 1 putting all t his in t he equation we get T dS

= P dV + Vd P + P dV '°Y - 1

11 oo : 01 : os Cv = - ' - 1 putting all t his in t he equation we get TdS = PdV + VdP

+ P dV

,- 1

SO solving this we get

T dS=

'

,- 1

P dV+ VdP

,- 1

now we know that

dP = -adV from the process it is given. Putting it back in the equation we get , VdV TdS = - - (Po - aV)dV - a - 1- l , -1 but for maximum of S dS= O so we get

0=

' ( Po -aV )dV-aVdV

,- 1

,- 1

from t his we can see

, Po = V a (l + , ) [email protected]

@Sk J ahiruddin, 2020

25

physicsguide CSIR NET, GATE

Entropy Calculation

Sol 1.22. Zero for all the materials as t he t hird law of t hermodynamics says that as the temperature goes to zero en-

t ropy also tends to zero.

00 : 01 : 10 a(l j [email protected]

+ ,) 25

physicsguide CSIR NET, GATE

Ent ropy Calculation

@Sk J ahiruddin, 2020

Sol 1.22. Zero for all the materials as t he third law of t hermodynamics says that as the temperature goes to zero en-

tropy also tends to zero.

j [email protected]

26

physicsguide CSIR NET, GATE

11 oo : oo : oo

Problems and Solutions in Phase Transition Sk J ahiruddin * Suchismito Chattopadhyay

*Assistant P rofessor Sister Nibedita Govt. College, Kolkata Author was the topper of IIT Bombay 1\11.Sc Physics 2009-2011 batch He ranked 007 in IIT J Al\lI 2009 and 008 (JRF) in CSIR NET J une 2011 He has been teaching CSIR NET aspirants since 2012

1

@Sk J ahiruddin, 2020

Phase Transition

11 oo : oo : 03 1

@Sk J ahiruddin, 2020

Phase Transition

Contents 1 Problems from NET, GATE, JEST , TIFR & JAM pap ers 1.1 Ans keys . 1.2 Solut ions . • • •

1







































































2 12 13

Problems from NET, GATE, JEST, TIFR & JAM papers

Prob 1.1. For a liquid to vapour phase t ransit ion at T tr which of the following plots between specific Gibbs free energy g and t emperature T is correct? [JAM 2012]

j [email protected]

2

physicsguide CSIR NET, GATE

11 oo : oo : 06

j ahir@physicsguide. in

2

physicsguide CSIR NET I GATE

@Sk J ahiruddin , 2020

Phase 'Iransition

I

r

.

.

d I

Prob 1.2. In the given phase diagram for a pure substance, regions I, II, III, IV, respectively represent [JAM 2019]

p Critical •••••••••••••••• Point

III



• ••

II~• I T

11 oo : oo : 09 •

II;• I T

[email protected]

3

@Sk J ahiruddin ) 2020

physicsguide CSIR NET ) GATE

Phase Transition

(A) Vapor, Gas, Solid, Liquid (B) Gas, Vapor, Liquid, Solid (C) Gas, Liquid, Vapor, Solid (D) Vapor, Gas, Liquid, Solid Prob 1.3. Consider the transition of liquid water to st eam as water boils at a temperature of 100°C under a pressure of 1 atmosphere. Which of the following quant ities does not [NET June change discontinuously at the t ransition? 2011] (a)The Gibbs free energy (b) The Internal Energy (c) The Entropy (d) The Specific Volume Prob 1.4. Consider the melting transition of ice into water at a constant pressure. Which of the following thermodynamic quant ities does not exhibit a discont inuous change across the phase transition? [NET Dec 2013] (a) Internal energy (b) Helmholtz free energy (c) Gibbs free energy (d) entropy Prob 1. 5. The entropy of a system , S , is related to the accessible phase space volume r by S = kB In r (E , N, V ) where E ,N and V are t he energy, number of particles and volume respectively. From this one can conclude that r [NET Dec 2012]

11 oo : oo : 12 Pro b 1. 5. The entropy of a system , S , is related to the accessible phase space volume r by S = kB In r (E, N, V )

where E,N and V are t he energy, number of particles and volume respectively. From t his one can conclude that r [NET Dec 2012] (a)does not change during evolut ion to equilibrium [email protected]

4

physicsguide CSIR NET, GATE

@Sk J ahiruddin, 2020

Phase Transition

(b )oscillates during evolution to equilibrium (d) is a minimum at (c)is a maximum at equilibrium equilibrium Prob 1. 6. The entropy S of a thermodynamic system as a

function of energy E is given by the following graph

t

s

The temperature of the phases A,B and C , denoted by TA, TB and Tc , respectively, satisfy the following inequalit ies: [NET Dec 2013] (a)Tc > TB > TA (b)TA > Tc > TB

(c) TB > Tc> TA

(d)TB > TA > Tc

Prob 1. 7. The condition for the liquid and vapour phass

11 oo : oo : 14 [NET Dec 2013]

t ies:

(a)Tc > TB > TA

(b )TA > Tc > TB

(c) TB > Tc> TA

(d )TB > TA > Tc

Prob 1. 7. The condit ion for the liquid and vapour phass of a fluid to be in equilibrium is given by t he approximate

~f

equat ion ~ T Vvap Qt (Clausius- Clayperon equation), where Vvap is t he volume per part icle in t he vapour phase, and Qz j [email protected]

5

physicsguide CSIR NET, GATE

@Sk J ahiruddin , 2020

Phase Transition

is t he latent heat , which may be taken to be a const ant. If t he vapour obeys ideal gas law, which of the following plots is correct? [NET June 2015]

0

T

Qnf 0

t--+-----

-r

Qt-+---- - -

-r

Prob 1.8. The phase diagram of a pure substance is given in t he figure below. Where T denotes the triple point and

11 oo : oo : 11

Prob 1.8. The phase diagram of a pure substance is given in the figure below. Where T denotes the t riple point and C denotes the critical point.

The phase transitions occurring along the lines marked a, /3 and , are [TIFR 2011]

[email protected]

physicsguide CSIR NET, GATE

6

Phase 'I'ransition

@Sk Jahiruddin, 2020

C

~

--

::,

~

Q)

a. E

~

T

Pressure - -

(a)a=melting: /3=condensation: ,=sublimation (b )a=sublimation: /3=vapourisation: ,=melting (c) a=melting: /3 = vapourisation: ,=condensation (d)a=sublimation: /3= melting: ,=vapourisation Prob 1.9. The vapour pressure p (in mm of Hg) of a solid, at a temperature T , is expressed by lnp = 23 - 3863/T and

t hat of its liquid phase by ln p = 19 - 3063/ T. The triple

11 oo : oo : 20 (d)a=sublimation: ,B= melting: ,1=vapourisation Prob 1.9. The vapour pressure p (in mm of Hg) of a solid, at a temperature T , is expressed by In p = 23 - 3863 /T and

t hat of its liquid phase by ln p = 19 - 3063/ T. The triple [GATE 2007] point (in Kelvin) of the material is (a) 185 (b) 190 (c) 195 (d) 200 Prob 1.10. The pressure versus temperature diagram of a given system at certain low temperature range found to be parallel to the temperature axis in t he liquid-to-solid tran-

sition region. The change in t he specific volume remains constant in this region. The conclusion one can get from [email protected]

7

physicsguide CSIR NET, GATE

Phase 'Iransition

@Sk J ahiruddin, 2020

[GATE 2008]

t he above is

(a) t he entropy solid is zero in this temperature region (b) t he entropy increases when the system goes from liquid to solid phase in t his temperature region (c) t he ent ropy decreases when the system transforms from liquid to solid phase in t his temperature region (d) t he change in entropy is zero in the liquid-to-solid transition region Prob 1.11. Identify which one is a first order phase tran-

sition? [GATE 2009] (a) A liquid to gas transition at its critical temperature (b) A liquid to gas transition close to its triple point (c) A paramagnetic to ferromagnetic transition in the abQt:)nf"P

f'lf

Q rn~O'nPtif"'

h Plrl

11 oo : oo : 22 sition? [GATE 2009] (a) A liquid to gas transition at its critical temperature (b) A liquid to gas transition close to its triple point (c) A paramagnetic to ferromagnetic transition in the absence of s magnetic field (d) A metal to superconductor t ransition in the absence of magnetic field Prob 1.12. In a first order phase t ransition , at the transition temperature, specific heat of the system [GATE 2011]

(a) diverges and its entropy remains the same (b) diverges and its entropy has finite discontinuity (c) remains unchanged and its entropy has finite discontij [email protected]

physicsguide CSIR NET 1 GATE

8

@Sk J ahiruddin, 2020

Phase Ttansition

nuity (d) has finite discontinuity and its entropy diverges Prob 1.13. Across a first order phase transition , the free energy is [GATE 2013] (a) proportional to the temperature (b) a discontinuous function of the temperature (c) a continuous function of t he t emperature but its first derivative is discontinuous (d) such that the first derivative with respect to temperature is continuous Prob 1.14. Ice of density p1 melts at pressure P and absolute t emperature T to form water of density r ho 2 . The 1

J

J

,

r

1

I



('





T

,. T Tl

,



, 1

11 oo : oo : 24 (d) such t hat the first derivative wit h respect to temperature is cont inuous Prob 1.14. Ice of density p1 melts at pressure P and absolut e t emperature T to form water of density r ho2 . The latent heat of melting one gram of ice is L. What is t he change in the internal energy ~ U resulting from t he melt-

ing of 1 gram of ice? (a) L + P ( l - l) (b) L - P ( l - l ) P2

Pl

P2

(c) L- P (l l) P1 P2

[JEST 2014]

Pl

(d)L + P (l - l) P1

P2

Prob 1.15. The free energy F of a system depends on a t hermodynamic variable ~ as F

2

= - a~ + b~

6

with a,b > 0. T he value of ~ , when t he syst em is in t hermodynamic equilibrium, is [NET June 2014] [email protected]

9

physicsguide CSIR NET, GATE

@Sk J ahiruddin , 2020

(a) 0

Phase Transition

1 4

(b) ±(a/ 6b) !

1 4

(c) ±(a/ 3b) 1

1 4

(d) ± (a / b) 1

Prob 1.16. Water freezes at o° C at atmospheric pressure (1.01 x 105 P a) . The densities of water and ice at t his

temperature and pressure are 1000 kg /m 3 and 934 kg/m3 respectively. The lat ent heat of fusion is 3.34 x 105 J / Kg . T he pressure required for depressing t he melting temperat ure of ice by 10°c is _____GPa (up to two decimal places) [GATE 2017] Prob 1.17. At atmospheric pressure (=105 Pa), aluminium r

11 oo : oo : 21 t ure of ice by 10°c is _____GPa (up to two decimal places) [GATE 2017]

Prob 1.17. At atmospheric pressure (=105 Pa), aluminium melts at 550K. As it melts, its density decreases from 3 x l0 3 kg/m 3 to 2.9 x l0 3 kg/ m 3 . Latent heat od fusion of aluminum is 24 x l03 J /kg . The melting point of aluminum at a pressure of 107 Pa is closest to [JAM 2014] (a)551.3K (b)552.6K (c)558.7K (d)547.4 Prob 1.18. A many-body system undergoes a phase transition between two phases A and B at a temperature Tc. The temperature-dependent specific heat at constant volume Cv of t he two phases are given by c~) = aT 3 + bT and ciB) = cT 3. Assuming negligible volume change of the system, and no latent heat generated in the phase transition, Tc is [TIFR 2018]

j [email protected]

10

physicsguide CSIR NET I GATE

@Sk J ahiruddin , 2020

(a)

4b

c- a

Phase 'Iransition

(b)

3b

c- a

(c)

-2b C

(d)

b c- a

11 oo : oo : 30

[email protected]

11

physicsguide CSIR NET ) GAT E

Phase Transition

@Sk J ahiruddin ) 2020

1.1

Ans keys Ans keys

1.1.

C

1. 7.

C

1.13.

C

1.2. b

1.8. b

1.14. d

1.3. a

1.9. d

1.15.

C

11 oo : oo : 32 1.1.

1. 7.

C

1.13.

C

C

1.2. b

1.8. b

1.14. d

1.3. a

1.9. d

1.15.

1.4.

C

1.10.

1.5.

C

1.11. b

1.17. a

1.6.

C

1.12. b

1.18. b

[email protected]

1.16. 0.17

C

12

physicsguide CSIR NET, GATE

@Sk J ahiruddin, 2020

1.2

C

Phase Transition

Solutions

Sol 1.1. Standard theory. The phase t ransition is not dis-

cussed in details in the notes. It will be updated quickly. P lease refer to Garg, Banshal and Ghosh. .

.

11 oo : oo : 3s Sol 1.1. Standard theory. The phase transition is not dis-

cussed in details in the notes. It will be updated quickly. Please refer to Garg, Banshal and Ghosh. Sol 1.2. The triple point refers to that temperature and pressure where three states of substance namely solid , liquid

and vapour co exist. 1) the equilibrium pressures and temperatures of a solid in contact with it's vapour up to triple point when plotted in a p-T diagram gives rise to t he sublimat ion curve . (the curve between II and III region denotes to the vapourization and t he curve between IV and II is sublimation curve and the curve between III and IV is called fusion curve and the point where t hey meet is called triple point .) 2)the equilibrium pressures and temperatures of a liquid in contact with it 's vapour up to the crit ical point when plotted gives rise to t he vapourisation curve is a continuation of the sublimation curve. 3) t he fusion curve is t he locus of equilibrium pressures and temperatures when a solid in contact with it 's liquid starting point being the t riple point. if

8P 8T

8P

8P 8T

= U1 and aT I

j [email protected]

@Sk J ahiruddin, 2020

Then we must have

I

13

II = U3 1

physicsguide CSIR NET, GATE

Phase Transition

00: 00: 37 Phase TI.·ansition

Then we must have

must not equal to zero. The III denotes liquid state IV denotes solid state I state denotes gas and the II denotes vapour. Sol 1.3. From t he previous discussion the Gibbs energy is

t hat quantity which changes cont inuously across the phase t ransition temperature as at phase transition point pressure and the temperature remains constant so change in the Gibbs energy is zero. Sol 1.4. The entropy is the quantity which cahnges abruptlty or discontinuously at phase transition temperature as the melting of ice into water is a first order phase t ransition. Sol 1.5. Entropy of the syst em always tend to maximize which is known to us so if we are associated to some ir-

reversible process then we must accompany the increase of t he entropy. and as we know s is related to t he accessible microstates of t he system so we must have that microstates will be maximum at equilibrium. Sol 1.6. We know t hat

[email protected]

@Sk J ahirudd in, 2020

as

1

8E

T 14

physicsguide CSIR NET, GATE

Phase TI.· ansition

(g~) this is called t he slope of the curve. so for t he b case the

11 oo : oo : 40 @Sk J ahiruddin, 2020

Phase Transition

(g~) t his is called t he slope of the curve. so for t he b case t he slope is zero so t emperature at b process is much larger t han c and b process but for c process t he internal energy E is greater than t hat of A process so definitely t he temperature at A process is less t han t hat of the c process so we have



~f

Sol 1. 7. ~ T ~Lap and we knovv that PVvap=RT and we will put t hat in the equation we will get t hat

dT

r2

so from this we get

C lnP = - - +a T so t his satisfies t he plot c. Sol 1.8. From t he previous discussion we can tell t hat t he

opt ion b is correct . Sol 1.9. The P - T diagram is continuous so at crit ical temperature

23 _ 3863 Tc [email protected]

@Sk J ahiruddin , 2020

= 19 _ 3063 Tc 15

physicsguide CSIR NET, GATE

Phase Transition

11 oo : oo : 42 [email protected]

15

@Sk J ahiruddin, 2020

physicsguide CSIR NET, GATE

Phase Transition

Hence or

Tc= 200 Sol 1.10. When a solid transforms to liquid t hen it requires energy and the solid has less disorderness that liquid so when

liquid converts to solid t hen it rejects heat and it's disorderness reduces so ent ropy decreases. Sol 1.11. The option 2 is correct because the last two de-

notes 1) lambda phase transition where the second order derivative at the t ransit ion t emperature has an infinite discont inuity and t he 2) the last one is 2nd order phase transit ion as the 2nd order derivative at t he transit ion temperature has t he finite discontinuity. so option 3 and 4 can never be the answer. only 2 denotes t he 1st order phase transition •

Sol 1.12. In a first order phase transition the first order

derivative of t he Gibbs free energy has a finite discont inuity but it's second order derivative diverges at t he crit ical temperature. we know that entropy of as system is the first order derivative of the gibbs free energy with respect totemperature so it has the finite discontinuity at the transit ion j [email protected]

16

physicsguide CSIR NET, GATE

11 oo : oo : 4s order derivative of the gibbs free energy with respect to temperature so it has the finite discontinuity at the transition j [email protected]

16

physicsguide CSIR NET, GATE

@Sk J ahiruddin , 2020

Phase Transition

temperature but t he second order derivative is t he specific heat so it diverges. dG = - SdT + V dP so we can say 8G 8T

ac 2

p

=

- S ,T

ar2

8Q

p

=

=

-T

as

8T

p

-Cp

8T p hence we see 2nd order derivative is actually related to the specific heat. Sol 1.13. The first order phase t ransition is a special kind of phase transition where the first order derivatives of gibbs free energy becomes discontinuous accross the critial or phase changing temperature. so only the option 3 is correct. Sol 1.14. From t he previous discussion we have seen t hat t he internal energy changes wit h volume in the following

manner

au

=T 8P - P 8V 8T and t he Clausius Clapeyron equation gives us 8P 8T

L

11 oo : oo : 48 av

ar

and t he Clausius Clapeyron equation gives us

8P

L

ar [email protected]

17

physicsguide CSIR NET, GATE

@Sk J ahirudd in, 2020

Phase 'I'ransition

so putting t his in t he equation we get

L

dU= - -

dV -

P dV

and we know that v = 1/ p so punting all t his in t he equation we get ~U

=

L

+P( l PI

-

l ) P2

[as at t he phase changing temperature t he pressure remains constant so it comes simply out of the integration .]

Sol 1.15. So at equilibrium for a particular value of 1/; F would be maximized so we have 8F 82 F = 0 81/; = 0' 8'lj) 2 2

so we have from t he first condit ion -2a'l/; + b'l/J 6 = 0 from t his 1 4 we get 'l/;=±(a/3b) 1

Sol 1.16. From t he Clausius Clapeyron equation we came to know t hat 8P L 8T . the meting point of ice is 273 kelvin , and t he depression of temperature 10°c here v 2 = (1/ 1000) = 1/ p1 and the v 1 = (1/ 934) = 1/ p2 and hence

11 oo : oo : so 8T . the meting point of ice is 273 kelvin , and the depression of temperature 10°c here v2 = (1/1000) = 1/ p 1 and the v 1 = (1/ 934) = 1/ p2 and hence L 263 - - - dT 273 T (V2 - v1) 18

[email protected]

physicsguide CSIR NET, GATE

@Sk J ahiruddin , 2020

Phase 'Iransition

SO integrating we get P2 = 1.01 x 10

5

+(

3.34

1 - 934

X

105

+

1 ) 1000

263 5 ln = 0 .1 7 x 10 pa 273

Sol 1.17. From the Clausius Clapeyron equation we came to know that

8P 8T

L

now we have T =550K L= latent heat of phase change = 24 x 10 3 J / kg densities are given so volume can be easily deter3 3 mined P1 = 3 x 10 = lV1 similarly p2 = 2.9 x 10 = lV2 so we have 7 10 - 105 24 X 103 1 ( - 3x 10 3

55 0 ~T + 2_9;103 ) from this we get the change in the melting point in creases by 2.607 kelvin. so the final melting point at t he given pressure would be 552.6K. Sol 1.18. The Entropy is continuous in the second order phase transition where there is no lat ent heat. Entropy in state A is

CvdT = aT3

r

q

+

bT

11 oo : oo : s3 Sol 1.18. The Entropy is continuous in t he second order phase t ransition where t here is no latent heat. Entropy in state A is CvdT = aT3 bT T 3 +

Entropy in state B is CvdT = cT T 3 j [email protected]

@Sk J ahiruddin , 2020

19

3

physicsguide CSIR NET 1 GATE

Phase Ttansition

At crit ical temperature t hese two entropy become same

Hence the ans

00: 00: 55 T j [email protected]

19

@Sk J ahiruddin, 2020

3 physicsguide CSIR NET, GAT E

Phase Transition

At critical temperature these two entropy become same SA= SB

Hence t he ans

j [email protected]

20

physicsguide CSIR NET, GAT E

11 oo : oo : oo

Problems and Solutions in Microcanonical Ensembles Sk J ahiruddin * Suchismito Chatterjee

*Assistant P rofessor Sister Nibedita Govt. College, Kolkata Author was the topper of IIT Bombay 1\11.Sc Physics 2009-2011 batch He ranked 007 in IIT JAl\lI 2009 and 008 (JRF) in CSIR NET June 2011 He has been teaching CSIR NET aspirants since 2012

1

@Sk J ahiruddin, 2020

Microcanonical Ensembles

11 oo : oo : 03 1

@Sk J ahiruddin, 2020

Microcanonical Ensembles

Contents 1

Problems from NET, GATE, JEST , TIFR & JAM pap ers 1.1 Ans Keys 1.2 Solutions . • • •

j [email protected]































2





3 •



































11 12

physicsguide CSIR NET, GATE

11 oo : oo : os

j [email protected]

2

@Sk J ahiruddin , 2020

1

physicsguide CSIR NET I GATE

Microcanonica l Ensembles

Problems from NET, GATE, JEST, TIFR & JAM papers

Prob 1.1. Thermodynamic variables of a syst em can be

volume V , pressure P , t emperature T , number of part icles N , internal energy E, chemical potent ial µ etc. For a syst em to be microcanonical (MC) , canonical (CE) and grand canonical ( GC) ensembles, the parameters required [GATE 2008] are (a) JVIC:(N, V, T ); CE:(E, N, V ); GC:(V, T , µ) (b) MC:(E, N, V); CE:(N, V, T ); GC: (V, T , µ) (c) MC:(V, T , µ); CE:(N, V, T) ; GC:(E, N, V ) (d) MC:(E , N , V ); CE:(V, T, µ); GC: (N , V, T ) Prob 1.2. A paramagnetic syst em consist ing of N spin- ~

particles is kept in an external magnetic field. It is found t hat N / 2 spins are aligned parallel and t he remaining N / 2 spins are aligned anti-parallel to the magnetic field. The stat istical entropy of the syst em is [GATE 2012]

(a) 2NkBln2

(b) (N/2)kBln2

(c) (3N/ 2)kBln2

(d) NkBln2

Prob 1.3. Consider a linear collection of N independent

spin-~ particles, each at a fixed location. The ent ropy of the

11 oo : oo : os

Pro b 1. 3. Consider a linear collection of JV independent spin- ~ particles, each at a fixed location. The ent ropy of the [email protected]

physicsguide CSIR NET ) GATE

3

Microcanonical Ensembles

@Sk J ahiruddin ) 2020

syst em is

(a) 0

(b) NkB

(c) (l / 2)NkB

[GATE 2013] (d) NkBln2

Prob 1.4. A system of N distinguishable particles, each of which can be in one of t he two energy levels O and c, has a total energy nc, where n is an integer. T he entropy of t he [NET June 2015] system is proportional to NI

(a) Nlnn

(b) nlnN

(c) ln

n !.

(d) ln

NI n !(N ~ n)!

Pro b 1. 5. Consider a gas of N classical part icles in a twodimensional square box of side L. If t he total energy of the gas is E, t he entropy (apart from an additive constant) is [NET Dec 2016] 2 (a) NkB ln(L E / N ) (b) NkB ln (LE/ N)

Prob 1.6. In a t hermodynamics system in equilibrium each molecule can exist in t hree possible states with probabilities 1/ 2, 1/3 and 1/ 6 resp ectively. The entropy per molecule is [NET June 2017]

1

(d) kB ln 2 + ~kB ln 3

Prob 1. 7. The number of microstates of a gas of N par-

11 oo : oo : 11 [NET June 2017]

molecule is (a) kB ln 3

(b) ~kB ln 2 + ~kB ln 3 (d) ~kB ln 2 + ~kB ln 3

Prob 1. 7. The number of microstates of a gas of N par-

[email protected]

4

@Sk J ahiruddin, 2020

physicsguide CSIR NET, GATE

Microcanonical Ensembles

t icles in a volume V and of internal energy U, is given by

n (u, V, N) = (V - Nb)N

a;:

3N/ 2

(where a and b are positive constants) . Its pressure P , volume V and temperature T , are related by [NET Dec 2017]

(a) (P + at)(V - nb) = NkBT (b) (P - at)(V - nb) = NkBT (c) PV = NkBT (d) P (V - nb) = NkBT Prob 1.8. In a system comprising of approximately 10 23 distinguishable particles, each particle may occupy any of 20 distinct states. The maximum value of the entropy p er [NET June 2019] particle is nearest to (a) 20kB (b) 3kB (c) 10(ln 2)kB (d) 20(ln2)kB Prob 1.9. Consider a system maintained at temperature T with available energy states E 1 and E 2 each with degen-

11 oo : oo : 14 particle is nearest to (a) 20kB (b) 3kB (c) lO (ln 2)kB

[NET June 2019] (d) 20(ln 2)kB

Prob 1.9. Consider a system maintained at temperature T with available energy states E 1 and E 2 each with degeneracies g1 and g2. If P1 and p2 are the probabilities of occupancy of these ststes, what is t he entropy of the system? [JEST 2012] j [email protected]

physicsguide CSIR NET, GATE

5

@Sk J ahiruddin , 2020

Microcanonical Ensembles

(a) S = - kB[P1ln(p1/ g1 ) + p2ln(p2 / g2) ]

+ p2ln(p2g2) ] (c) S = - kB [p1 l n (Pi + P2 l n (p~ (d) S = - kB [(I / p1)ln(p1 / 91) + (l / p2)ln(p2/ 92)] (b) S = -kB[P1ln(p1g1) 1

2

)

)]

Prob 1.10. Consider a particle with 3 possible spin states: s = 0, ± 1. There is a magnetic field h present and t he energy for a spin state s is -hs. The system is at temperature T . Which of t he following is true about t he entropy S(T )? [JEST 2013] (a) ln3 at T = 0, and 3 at high T (b) ln 3 at T = 0, and O at high T (c) 0 at T = 0, and 3 at high T (d) 0 at T = 0, and ln3 at high T

Prob 1.11. Consider a system of 2N non-interacting spin~ particles fixed in position and carrying magnetic moment µ . The system is immersed in a uniform magnetic field B.

11 oo : oo : 16 (d) 0 at T = 0, and ln3 at high T Prob 1.11. Consider a system of 2N non-interacting spin~ particles fixed in position and carrying magnetic moment

µ. The system is immersed in a uniform magnetic field B.

The number of spin-up particles for which the entropy of [JEST 2014] t he system will be maximum is (a) 0 (b) N (c) 2N (d) N/2 Prob 1.12. A system having N non-degenerate energy eigenstates is populated by N identical spin-zero particles [email protected]

physicsguide CSIR NET, GATE

6

@Sk J ahiruddin, 2020

Microcanonical Ensembles

and 2N identical spin-half particles. There are no interactions between any of these particles. If N = 1000, the entropy of the system is equal to [TIFR 2011] (a) 13.82kB (b) 0 (c) 693.lkB (d) lOOOkB (e)5909.693kB

(f)6909kB

Prob 1.13. The entropy S of a black hole is known to be of the form S = akBA, where A is the surface area of

the black hole and a is a const ant , which can be written in terms of c (velocity of light in vacuum) , Ii (reduced Planck's constant) and GN (Newton's constant of gravitation). Tak2 ing t he radius of the black hole as R = GJM, it follows that t he entropy is

__

___

A(nc) 4 AGNM (Hint: Use dimensional analysis)

[TIFR 2013]

Alie

Prob 1.14. A system at temperature T has 3 energy states: ,. . , . , , .

11 oo : oo : 1s 2

kB G7vM ( ) a A(lic) 4

2

(b) lickB AGNM

( ) G'JvM kB C

Alic4

(Hint: Use dimensional analysis) Prob 1.14. A system at temperature T has 3 energy states:

0,±c. The entropy of the system in t he low t emperature (T ➔ 0) a nd high temperature (T ➔ oo) are respectively [TIFR 2013]

(a)

sT➔O =

0, sT➔co

(b) sT➔O

= sT➔oo =

(c) sT➔O

=

=

kB exp (-3)

kB ln 3

0, sT➔oo = kB In 3

[email protected]

7

physicsguide CSIR NET, GATE

@Sk J ahiruddin , 2020

Microcanonical Ensembles

Prob 1.15. Consider the CO molecule as a syst em of 2

point particles which has both t ranslational and rotat ional degrees of freedom. Using classical statistical mechanics, t he molar specific heat Cv of CO gas is given in t erms of [TIFR 2014] Boltzmann constant kB by

(a) ~kB

(b) 2kB

(c) ~kB

(d) ~kB

Prob 1.16. Cosmic r ay muons, which decay spontaneously with proper lifetime 2.2µs are produced in t he atmosphere,

at a height of 5 km above sea level. These move straight downwards at 98% of t he sp eed of light. Find the percent rat io 100 x (NA/NB) of the number of muons measured at t he top of two mountains A and B , which are at heights 4, 848m and 2, 682m respectively above mean sea level. [TIFR

11 oo : oo : 21 downwards at 98% of t he speed of light. Find the percent ratio 100 x (N A/NB) of the number of muons measured at the top of two mountains A and B, which are at heights 4, 848m and 2, 682m respectively above mean sea level. [TIFR 2017]

Prob 1.17. Consider N non-interacting distinguishable particles in equilibrium at an absolute temperature T. Each particle can only occupy one of two possible states of energy 0 and E respectively (E > 0). The entropy of the system, in terms of /3 = c/kBT is [TIFR 2019] (a) NkB In (1 + e- 13 ) - 1:~~13 j [email protected]

8

physicsguide CSIR NET 1 GATE

@Sk J ahiruddin, 2020

Microcano11ical E nsembles

(b)

(c) (d)

NkB In

(1 + e-f3) -

13 el -e-/3

Prob 1.18. A monatomic crystalline solid comprises of N atoms, out of which n are in interstitial positions. If t he available interstitial sites are N' , then t he number of possible microstates is [GATE 2006] (N' + n) ! b N! N'! (a) n!N! ( ) n!(N + n)! n!(N' + n )! I N ( ) . c n !(N' - n) !

(d)

NI

.

N'I

.

n !(N - n)! n !(N' - n) !

00: 00: 24 (a) (N' + n) ! n !N !

NI

(b) n !(N ~ n) ! n !(N'

NI

(c) n!(N'

~ n)!

N' I

NI

(d) n !(N

~

~ n) !

N' 1 n) ! n !(N' ~ n) !

Prob 1.19 . Consider a system of N non-interacting spin~ particles, each having a magnetic moment mu in a magnetic field B = B z. If E be t he total energy of t he system, t he number of accessible microstates D is given by [GATE 2015] E N N! µB (a) n = (b) D = 1 E I~ E E I• NN+ N +

-

2

µB



µB

2

[email protected]

µB

' •

physicsguide CSIR NET, GATE

9

@Sk J ahiruddin , 2020

1

I



Microcanonical Ensembles

E

,~ N - - ·2 (c) D = 2 µB

E

N + µB !

N! (d) D= - - N+ µB. E '

Prob 1.20. Consider N non-int eracting, distinguishable particles in a two-level system at temperature T . The energies of the levels are O and c. In t he high temperature limit (kBT >> c), what is t he populat ion of part icles in t he level [GATE 2017] wit h energy c? N N 3N ( b) N (d) (a) (c) 2 4 4 Prob 1.21. A microcanonical ensemble consists of 12 atoms

11 oo : oo : 26 par IC es In

[GATE 2017]

with energy c?

N N (d) 3N (a) (b) N (c) 2 4 4 Prob 1.21. A microcanonical ensemble consists of 12 atoms with each taking either energy O state, or en ergy

E

state.

Both states are non-degenerate. If t h e total energy of t his ensemble is 4E, its entropy will be ____ kB (up to one decimal place), where kB is the Boltzmann constant. [GATE 2018]

j [email protected]

10

@Sk J ahiruddin , 2020

1.1

physicsguide CSIR NET I GATE

Microcanonical Ensembles

Ans Keys

1. 1. b

1. 8. b

1. 15. a

1. 2. d

1. 9. a

1. 16. 052

1. 3. d

1. 10. d

1. 17. b

1. 4. d

1. 11. b

1. 18. d

1. 5.

C

1. 12. f

1. 19. a

1. 6.

C

1. 13. d

1. 20. a

11 oo : oo : 29 1. 3 . d

1. 10. d

1. 17. b

1. 4 . d

1. 11. b

1. 18. d

1. 5.

C

1. 12. f

1. 19. a

1. 6.

C

1. 13. d

1. 20. a

1. 14 .

1. 21. 6.2

1. 7. d

[email protected]

@Sk J ahiruddin ) 2020

1.2

C

11

physicsguide CSIR NET ) GATE

Microcanonical Ensembles

Solutions

Sol 1.1. t he concept of ensemble in the statistical mechanics is very important concept . each microstate of a N part icle gas is represented by a point in the 6N dimensional phase space .sO if one considers all possible microstat es which t he gas can have , one will have a huge collection of points in t he

11 oo : oo : 31 ics is very important concept. each microstate of a N particle gas is represented by a point in the 6N dimensional phase space .sO if one considers all possible microstates which t he gas can have , one will have a huge collection of points in t he phase space of all possible microstates of t he syst em, this is called ensemble.one just can think of t hat each point in t he phase space is just a copy of t he possible configuration of t he syst em and such a haze collection of the points is called t he ensemble. there are three kind of possible ensemble in t he statistical mechanics . these are 1) microcanonical ensemble 2) canonical ensemble 3) grand canonical ensemble. in the micro canonical ensemble we take the volume of t he ensemble and energy of the ensemble and the number of t he particle in t he ensemble remains constant. in the canonical ensemble we take the energy number of particle and t he temperature remains constant. and in t he grand canonical ensemble we take the volume temperature and the chemical potential of the syst em is constant. so option b is correct.

Sol 1.2. t he important concept of the statistical entropy is t hat t he entropy is the measure of the dis order of the sytem and the n1ore microstates available to a system the more will [email protected]

@Sk J ahiruddin, 2020

12

physicsguide CSIR NET, GATE

Microcanonical Ensembles

be the entropy of the system. the relation of the entropy and t he available microstates of t he system is given by

whP-r P- ko is thP- Rolt,7,m::1,n n r.onst.::1,nt ::t.nrl t,hP- w is thP- m i-

11 oo : oo : 34 e entropy o t e system. t e re ation o t e entropy and the available microstates of t he system is given by

where kB is the Bolt zmann constant and the w is t he microstates available to the syst em. here only two microstates are available to the system of N particles , so we have

hence t he option d is correct. Sol 1. 3. if the spin of a syst em is ½ then the number of

available microst at es of the syst em is given by

hence the entropy of the system in the case of the N number of the particle is given by

hence the option d is correct. Sol 1.4. this one again is required to calculate the number

of ways the arrangements can be done such that the total energy is remaining constant . this can be dome if n number j [email protected]

@Sk J ahiruddin , 2020

13

physicsguide CSIR NET, GATE

Microcanonical Ensembles

of particle to be in t he higher energy levels and all the other particles remain in O energy level. that is now the question is how much way we can choose n number of t he particle

00: 00: 36 Microcanonical Ensembles

of particle to be in t he higher energy levels and all the other particles remain in O energy level. that is now the question is how much way we can choose n number of t he particle from the N number of the particle so this can be done in the following number of t he ways and is given by

D=

N! (N - n) !(n) !

hence we have the entropy of the system

N! S = kn ln (N _ n) !(n) ! . so option d is correct.

Sol 1. 5. we know t hat the number of shells in the phase space 1s _ dxdpxdydpy dnn,2 •

( here we consider that if a particle has momentum change say dp and position change say dx t hen if there implication is less than Ii then we will be unable to detect that if t he particle makes t he change of its position ) so we can say that is the phase space is n dimensional then number of shells we have in the phase space is

_ dx1 ...... dxNdPxl·····dPxN dn n,N [email protected]

@Sk J ahiruddin, 2020

14

physicsguide CSIR NET, GATE

Microcanonical Ensembles

from this concept we write down the upper equation. so in

11 oo : oo : 39 @Sk J ahiruddin , 2020

Microcanonical Ensembles

from t his concept we writ e down t he upper equation. so in t his case t he available space is

and hence we have

ftEfN

L2

n = r,,2 o

L2

21rpdp =

E r,,2 21r N

so t he microstates available

D= n N

L221r E r,,2 N2

=

now entropy would be S=NkB lnD = 2NkBln L,/E/N

Sol 1.6. the entropy of the system is given by

hence we have

1

1

S = -kB( ln(l/2) + ln(l /3) 2 3 2 1 = - kB ln 2 + - kB ln 3 3 2

1

+ 6ln(l/6))

Sol 1. 7. we know that

[email protected]

@Sk J ahiruddin, 2020

15

physicsguide CSIR NET, GATE

Microcanonical Ensembles

11 oo : oo : 42 15

[email protected]

physicsguide CSIR NET, GATE

@Sk J ahiruddin, 2020

Microcanonical Ensembles

so we can write down that

aU

3N/ 2

N

which can be simplified to write down as

but we know that from the first law of thermodynamics dQ= TdS= dU+ PdV so from this we can say that

as

1

au v

-

r

now from the above relation we can write that 1 T

3N k N a 2 Bau N

now we want to evaluate t he pressure and from the first law of thermodynamics we again see

as av

P

u

r

this gives us t hat

p

1 T =NkBV -Nb j [email protected]

16

physicsguide CSIR NET, GATE

11 oo : oo : 4s p

1 T =NkBV -Nb j [email protected]

16

physicsguide CSIR NET, GATE

@Sk J ahiruddin , 2020

Microcanonical Ensembles

so t he above relation can be writ ten as

ao t he last option is correct. Sol 1.8. we know t hat if there is N number distinct of sates in a system and n umber of particles in t hat syst em t hen t here is accessible st ates in the system is given by

we use t his formulae to evaluate the entropy of the system as we ha,,e known that ent ropy is related to the microstates of t he system and as t he microstates increases the entropy of the system also increases in t hat sense we can say that t he ent ropy measures t he disorderness in t he syst em and the relation is given by so t he ent ropy is given by

so the ent ropy per molecule is given by S / N = 3kB

11 oo:oo:47 so t he ent ropy per molecule is given by

S / N = 3kB [email protected]

physicsguide CSIR NET, GATE

17

@Sk J ahiruddin, 2020

Microcanonical Ensembles

Sol 1. 9. t he entropy of t he system is defined also as S

= - kB

L Piln pi .

'l

where p represents t he probability of each st ate. so we have

as t he first state has probability p 1 and t he second state probability p 2 Sol 1.10. to solve t his kind of problem we need to know what is part ition funct ion . by mathematical expression the

partit ion function is given by for discrete syst em

and in terms of t he partit ion function we can define t he Helmholtz free energy function

and we know t hat

8F

S =entr opy = - BT

for this part icular system we can define

11 oo : oo : 49 and we know that

8F

s =entropy= - ar

for t his particular system we can define

Z =l +

h

ekBT

[email protected]

+ e-

18

h

kBr

physicsguide CSIR NET, GATE

@Sk J ahiruddin , 2020

Microcanonical Ensembles

now we can define In Z

h

= ln(l + e kBT + e -

h

kBr)

so we can say that h

F = - kB T In ( 1 + e kBT

h

+ e- kBr )

so t he entropy can be calculated from the previous relation S

=

kB ln (l

+ kBT

h

+ ekBr + e -

h

kBT )

h

l h ekB r

+

hence we see t hat as T



(1 +

h e - kBT )

h

2 (-ekBT

+ e-

h

kBr )

kBT

oo the S = kB In 3. so option d is

correct.

Sol 1.11. let there be n numbers of spin up and the spin down are then (2N-n) now we have to calculate in how many way t hat spins can be arranged because we need to calculate t he thermodynamic probability and we have n _ ~

t. -

n n _ ~ l,1 ~ l,2 -

2N! 2N! (2N - n) !n ! n !(2N - n) !

as the thermodynamic probability is multiplicative quantity.

11 oo :oo : s2 t he thermodynamic probability and we have n _ ~

G -

2N! 2N! (2N - n)!n! n !(2N - n)!

n

n _ ~ Gl~ l2 -

as the thermodynamic probability is multiplicative quantity. so t he entropy of the system we have S = kBlnO t hen the entropy of the system 2N ! S = 2kB ln (2 N _ n)!n! j [email protected]

19

physicsguide CSIR NET 1 GATE

@Sk J ahiruddin, 2020

Microcano11ical Ensembles

now we will use t he star ling approximation and that tells lnN! = NlnN - N hence we will have S = 2kB [2Nln2N - 2N - (2N - n) ln(2N - n)

+(2N - n) - n ln n + n] for maximum we must have

as= 0 an which gives n= N hence option B is correct . Sol 1.12. for classical one t he number of orientation is given by w = NN hence the entropy S=6908KB but for t he spin half ones we have 2N number of particle will be accommodate in N states so only one arrangement is possible so we

have total entropy S=6909KB Sol 1.13. from the concept of t he dimension analysis we takes= cxnYGNzkBA So we have r

1

rr

1

rrn - l 1 r .ry

1

r

11

,r - l

r

2rn-21

r-1--1

r

11

,r2rn- l 7

11 oo : oo : 55 have total entropy S=6909KB Sol 1.13. from t he concept of the dimension analysis we take S = cxfiYGNzkBA So we have

so applying this in the above equation we have

[S] = [M L 2r - 2 0- 1 ] = [[L][T-l]] x[M-1 L2T-2] z[M2T-l]Y [M LT-20- 1] [email protected]

20

physicsguide CSIR NET, GATE

@Sk J ahiruddin, 2020

Microcanonical Ensembles

now comparing both side we get S

2

= !iGNM kB Arie

Sol 1.14. see the problem 15. Sol 1.15. let us calculate the degrees of freedom t hat is

given by f=3N-k= 3x 2-1=5 and from t he equipartition t heorem we have ea h degrees freedom will have 0. 5kBT hence we internal energy U = ~kBT hence Cv =

j~

= ~kB

Sol 1.16. Sol 1.17. let us first calculate t he part ition function and that is given by z

= (1 + e-c/kB T)N

hence t he Helmholtz free function

11 oo :oo : s1 z

= (1 + e-c/ kBT)N

hence t he Helmholtz free function

now we have to calculate t he entropy

S= _ 8F 8T so doing this we get

S

=

E

NkB ln(l

which gives S

=

e - c/ kBT

+ e - c/kBT) + - - - - kBT 1 + e - c/ kBT

NkB In (1

+ e- /3c ) + l+ef3e - f1 .B

21

j [email protected]

@Sk J ahiruddin , 2020

physicsguide CSIR NET I GATE

Microcanonical Ensembles

Sol 1.18. first let us see t hat in how much way t he n is arranged in t he N number of t he atoms and it is given by D1

=

n!ff~n!

now we are given N' interstitial sites so in how

much way n is arranged in that sites we have D2 so t he total probability will be n n _ ~ll~ l2 -

=

(N'~~)!n!

N' ·I N I· n !N - n ! (N' - n) !n !

hence opt ion d is correct.

Sol 1.19. we have let n 1 part icle has spin 1/ 2 and n 2 has spin down (-1/ 2) so we have n1 + n2 = N and also-n1µ B + n 2 µ B = E hence we can say n 1 = ~ N - µ~ and we have n 2½ N

+ µ~

so t he number of states are given by

N! R \

1 /

R \

so option a is correct .

11 oo : 01 : oo spin down (-1/ 2) so we have n1 + n2

=N

and also-n1µ B

n 2 µB = E hence we can say n 1 = ~ N h ave n 2~ N

+ µ~

2

N- -

E

µB

and we

so t he number of states are given by

N!

O= 1

µ~

+

so option a is correct.

,!·2

N+ E I µB.

Sol 1.20. t he part ition function Z

= 1 + e c:/kBT so the frac-

t ion of population in t he higher energy state is given by e-c:/ kBT N2

= N 1 + e c:/kBT

so as t is going to higher t emperature N 2 =

1

Sol 1.21. we have to see t hat how many ways we can arrange 12 atoms in t he 2 energy state such that t he energy [email protected]

22

physicsguide CSIR NET ) GATE

Microcanonical Ensembles

@Sk J ahiruddin ) 2020

is 4E t he number of the ways is given by 12!

0 = =495 4!8! hence t he entropy S = kBlnO = 6.204kB so upto one decim al we have 6.2 .

11 oo : 01

: 03

range 12 atoms in t he 2 energy state such that t he energy j [email protected]

22

physicsguide CSIR NET, GATE

@Sk J ahiruddin , 2020

Microcanonical Ensembles

is 4E the number of the ways is given by 12! D= =495 4!8! hence t he entropy S = kBlnO = 6.204kB so upto one decimal we have 6.2 .

j [email protected]

23

physicsguide CSIR NET, GATE

11 oo : oo : oo

Problems and Solutions in Canonical Ensembles: Part-I Sk J ahiruddin * Suchismito Chatterjee

*Assistant P rofessor Sister Nibedita Govt. College, Kolkata Author was the topper of IIT Bombay 1\11.Sc Physics 2009-2011 batch He ranked 007 in IIT JAl\lI 2009 and 008 (JRF) in CSIR NET June 2011 He has been teaching CSIR NET aspirants since 2012

1

@Sk J ahiruddin , 2020

Car1onical Ensembles: Part-1

11 oo : oo : 03 1

@Sk J ahiruddin, 2020

Canonical Ensembles: Part-1

Contents 1

Problems from NET, GATE, JEST , TIFR & JAM pap ers 1.1 Ans Keys 1.2 Solutions . • • •

j [email protected]































2





3 •



































14 15

physicsguide CSIR NET, GATE

11 oo : oo : 06

j [email protected]

@Sk J ahiruddin , 2020

1

2

physicsguide CSIR NET I GATE

Canonical Ensembles: Part-1

Problems from NET, GATE, JEST, TIFR & JAM papers

Prob 1.1. The partition function of a single gas molecule

is Za . T he partition funct ion of N such non-interacting gas [GATE 2007] molecules is t hen given by (a) (Za)N / N ! (b) (Za)N (c) N(Za) (d) (Za)N / N Prob 1.2. C0 2 molecule has t he first few energy levels separated by approximately 2.5 meV. At a temperature of

300K, the ratio of number of molecules in t he 4 th' excited state to t hat in t he 2n d excited state is [GATE 2010] (a) 0. 5 (b) 0. 6 (c) 0.8 (d) 0. 9 Prob 1.3. In I-dimension , an ensemble of N classical parp2 1 2 t icles has energy of the form E = x + -kx . T he average 2m 2 internal energy of the system at temperature T is [JAM 2014]

(a) ~NkBT

(b) ½NkBT

(c) 3NkBT

(d) NkBT

(Hint: you can directly use equipartit ion theorem) Prob 1.4. An ensemble of quantum harmonic oscillators is kept at finite temperature T = l /kB /3 - [GATE 2007]

(i) T he partition function of a single oscillator wit h energy levels (n + ½) nw is given by

11 oo : oo : os is kept at finite temperature T = 1/ kB/3 - [GATE 2007] (i) T he partition function of a single oscillator wit h energy levels (n + ½) nw is given by [email protected]

3

physicsguide CSIR NET ) GATE

@Sk J ahiruddin ) 2020

(a) Z =

Canonical Ensembles: Part-1

e-f3nw/ 2 1 - e- /3nw

1

-(c) Z -- 1 - e- /3nw

e-f3nw/ 2

(b) Z -- 1 + e- /3 nw 1 (d) Z -- 1 + e-f3nw

(ii) The average number of energy quanta of t he oscillators is given by

(a) (n) = (c) (n) =

1 ef3nw - 1

1 ef3nw

(b) (n) =

+ 1 (d) (n) =

e- f3nw ef3nw - 1 e- f3nw e- f3nw + 1

Prob 1.5. Consider a system whose 3 energy levels are 0, E and 2c. T he energy level c is 2-fold degenerate and t he other energy levels are non-degenerate. T he partition

function of the system with /3 = 1/kBT is given by [GATE 2012] (a) 1 + 2e-/3c (b) 2e- /3c + e- 2/3c

(c) (1 + e- 13€)2

( d)

1 + e- /3c + e- 2/3c

Prob 1.6. A gas of N non-interacting particles is in thermal equilibrium at temperature T . Each part icle can be in any one of the possible non-degenerate stat es of energy 0, 2c and 4c. T he average energy per part icle of t he gas, when

/3E > L kept vertically in t he Earth's gravitational field. The average energy of t he gas at low temperatures such that mgL >> kBT is given by [NET Dec 2011] (a) NkBT/2 (b) 3NkBT/ 2 (c) 2NkBT (d) 5NkBT/2 Pro b 1. 9. The free energy of a gas of N part icles in a volume V and at a temperature T is

11 oo : oo : 13 Prob 1.9. The free energy of a gas of N particles in a volume V and at a temperature T is

where a0 is a constant. The internal energy of the gas is

[NET June 2012] (b) (5/ 2)NkBT

(a) (3/ 2)NkBT j [email protected]

5

physicsguide CSIR NET, GATE

Canonical Ensembles: Part-1

@Sk J ahiruddin, 2020

5 2

(c) NkBTln[aoV(kBT) 1 /N] - (3/ 2)NkBT 5 2

(d) NkBTln[a0 V /(kBT ) 1 ] Prob 1.10. A system has two normal modes of vibration, with frequencies w1 and w2 = 2w1 . What is the probability t hat at temperature T , t he system has energy less than 4nw1 ? [In t he following x = e-f3nwi and Z is the partition function.] [NET June 2012]

(a) x 312 (x + 2x 2 )/ Z (c) x 312 ( l

+ 2x 2 )/Z

+ x + x 2 )/Z (d) x 312 (1 + x + 2x 2 )/Z (b) x 312 (l

Prob 1.11. A system can have 3 energy levels: E = 0, ±E. The energy level E = 0 is doubly degenerate, while others are non-degenerate. The average energy at inverse temperature {3 is [NET June 2014]

(c) 0

Prob 1.12. A system of N non-interacting classical parti-

11 oo : oo : 16

Prob 1.12. A system of N non-interacting classical particles each of mass m is in a 2-dimensional harmonic oscillator

potential of the form V(r) = a(x 2 +y 2 ), where a is a posit ive constant. The canonical part ition function of the syst em at temperature T is [,B = l / kBT] [NET June 2015]

[email protected]

physicsguide CSIR NET, GATE

6

Canonical Ensembles: Part-1

@Sk J ahiruddin, 2020

(a)

(c)

a 2m

an

2 7r

N

(b)

,B N

(d)

2m,B

2mn

2N

a,B

2mn

2

N

a,82

Prob 1.13. For a system of independent, non-interacting

I-dimensional oscillators, the value of free energy per oscil[NET Dec 2015] lator in the limit T ➔ 0 is

(a) (U;J/2

(b)

(c) 3nw/2

(U;J

(d) 0

Prob 1.14. A gas of non-relativistic classical particles in

one dimension is subj ected to a potential V (x) = a x (where a is a constant). The partition function is (,B = 1/ K = kBT) [NET June 2016]

(a)

(b)

11 oo : oo : 19 [NET June 2016]

(a)

(c)

41rm f32a,2 n,2

81rm 132a2 n,2

(b) (d)

21rm f32a,2n,2

31rm f32a,2 n,2

Prob 1.15. The partition function of a two-level system governed by the Hamiltonian ,' - > ~ )varies with temperature [NET Dec 2018] as j [email protected]

10

Canonical Ensembles: Part-1

@Sk J ahiruddin, 2020

3 2

(a) 1/ T 1

(b) 1/ T

physicsguide CSIR NET I GATE

3

(

c) 1/T

(d) 1/ T

2

Prob 1.24. The Hamiltonian of a classical nonlinear one

dimensional oscillator is H = 2~p2 + Ax 4 where A > 0 is a constant. The specific heat of a collection of N independent such oscillators is [NET June 2019] (a) 3NkB/2 (b) 3NkB / 4 (c) NkB (d) NkB/2 (Hint: Generalized equlipartition theorem) Prob 1.25. For a system in t hermal equilibrium with a

11 oo : oo : 29 [NET June 2019] (a) 3NkB/2 (b) 3NkB / 4 (c) NkB (d) NkB / 2 (Hint: Generalized equlipartition theorem )

such oscillators is

Prob 1.25. For a syst em in t hermal equilibrium with a heat bath at t emperature T , which of the following equali-

==

t ies is correct?

[JEST 2015]

Prob 1.26. A particle in t hermal equilibrium has 3 possible states with energies -E, 0, c . If the system is maintained at a temp erature T > > c / kB, the average energy of the part icle can be approximated to [JEST 2015] 2 2 2 2c 2c c

(a) 3kBT

(b) - 3kBT

(d) O

(c) - kBT

Prob 1.27. A gas of N molecules of mass m is confined 3 in a cube of volume V = L at t emperature T. The box is in a uni£orm gravitational field - g z. Assume that the potential energy of a molecule is U = mgz where z E [O, L ] jahir@physicsguide. in

11

physicsguide CSIR NET ) GATE

@Sk J ahiruddin ) 2020

Canonical Ensembles: Part-1

is the vertical co-ordinate inside the box. The pressure P (z)

[JEST 2016]

at height h is

mg

exp N

(z -

½)

kBT

L

(a) P (z) = V m2g - - - - - mgL sinh (

2kBT ma ( z - ~) \

11 oo : oo : 31 mg z - 2 exp kBT N L (a) P(z ) = V m2g - - - - - mgL sinh 2kBT exp

(b) P (z) =

mg

(z - ~)

kBT N L Vm2g - - - - - m gL cosh 2kBT

(c) P (z ) = kBT N V

N (d) P( z ) = Vmg z Prob 1.28. For a quantum mechanical harmonic oscilla-

tor wit h energies E n = ( n

+ t) rii.,J,

where n = 0, l , 2, · · · , [JEST 2016]

Prob 1.29. If t he men square fluctuations in energy of a [email protected]

@Sk J ahiruddin, 2020

12

physicsguide CSIR NET, GATE

Car1onical Ensembles: Part-1

system in equilibrium at temperature T is proportional to ra, then t he energy is proportional to [JEST 2017] (a) y a-2 (b) y a/2 (c) y a-l (d) y a

11 oo : oo : 34 system in equi 1 r1um at temperature T a, then the energy is proportional to (a) r a-2 (b) r a/2 (c) r a -l

j [email protected]

@Sk J ahiruddin ,

1.1

13

2020

is proport1ona to [JEST 2017]

(d)

Ta

physicsguide CSIR NET, GATE

Canonical Ensembles: Part-1

Ans Keys

1. 1. a

1. 11. d

1. 21. d

00: 00: 36 anonical Ensembles: Part-1

1.1

Ans Keys

1. 1. a

1. 11. d

1. 21. d

1. 2.

C

1. 12. d

1. 22. a

1. 3. d

1. 13. a

1. 23. d

1. 4. (i) a, (ii) a

1. 14.

1. 5.

1. 15. b

C

1. 6. a 1. 7.

1. 16. 1. 17.

C

C

1. 24. b 1. 25. a

C C

1. 26. b 1. 27.

C

1. 8. d

1. 18.

1. 9. b

1. 19. a

1. 28. d

1. 10. d

1. 20.

1. 29.

[email protected]

@Sk J ahiruddin, 2020

1.2

Solutions

C

C

14

C

physicsguide CSIR NET, GATE

Canonical Ensembles: Part-1

11 oo : oo : 39 @Sk J ahiruddin, 2020

1.2

Canonical Ensembles: Part-1

Solutions

Sol 1.1. we have t he single particle partition function Za so for the N particle we will have Za N but as we are t aking non interacting particle so we will have counted N! number of

states extra so we must have to devide them by N! so actual partit ion function [2

=

Z aN

N! Sol 1.2. t he ground state energy is E 0 then t he first excited energy (Eo + 2.5)meV t hen the 2nd excited energy (Eo + 5)meV and t he 3rd excited state (E 0 + 7.5 )meV and the fourth excited state has the energy (E0 + lO )meV. so the ratio is N4 Ni0 e-(Eo+l0)/300ks = 0 8 N Na e-(Eo+5)/300kB · 2

Sol 1.3. each degrees of t he freedom has t he energy 0.5knT

at temperature Thence t he total energy 1s < E >=< fm > + < ~kx 2 >= 2 x ~knT = knT so for the N number of particle we have •

E avg

2

= NknT

( here we have directly use t he equipartition theorem for details of equipartition see thermodynamics part first )

[email protected]

@Sk J ahiruddin , 2020

15

physicsguide CSIR NET, GATE

Canonical Ensembles: Part-1

11 oo : oo : 42 15

[email protected]

physicsguide CSIR NET, GATE

Car1onical Ensembles: Part-1

@Sk J ahiruddin, 2020

Sol 1.4. t he partition function

so we have t here is a constant terms in the partition and it will not have any contribution in any macroscopic contribution.so to calculate first we subside the constant terms and calculate the other term. so we will have the fact that

z=

L e- nnw(3 =

l

+ e- nw(3 + e- 2nw(3 + .....

so this can be written as like as the following z

=

e- f3nw/2 _ _1__ 1 - e- nw(3

so option a is correct. Sol 1.5. t he partition function is defined as

Now here we have that t he first level is non degenerate and t he 2nd level is doubly degenerate and t he t he third level is non degenerate so we have the partition function for t his system Z = l + 2e- /3c + e- 2/3c = (1 + e- (3€ ) 2 so the c option is correct. j [email protected]

16

physicsguide CSIR NET, GATE

11 oo : oo : 4s Z = 1 + 2e- /3c + e- 2/3c = (1 + e - /3c) 2

so the c opt ion is correct. j [email protected]

@Sk J ahiruddin ,

16

physicsguide CSIR NET, GATE

2020

Canonical Ensembles: Part-1

Sol 1.6. t he average energy is defined as

< E >= -

a 813

in z

where z is the partition function . now we have single part icle partition funct ion for this case is z

= 1 + e- 2/3c + e- 4/3c

so form this we can define N particle partition function as follows Z

= (1 + e- 2/3c: + e - 4f3c: ) N

now we can define the average energy of t he syst em

8 N (2ce- 2/3c: + 4ce- 4f3c:) < E >= - - In z = - - - - - - - 8(3 1 + e-2/3c + e -4/3c now as

/3E = 2c N so energy par particle

= - - - = 2c N

so option a is correct.

11 oo:oo:47 so energy par particle

= - - - = 2c N

so option a is correct. [email protected]

physicsguide CSIR NET, GATE

17

Canonical Ensembles: Part-1

@Sk J ahiruddin, 2020

Sol 1. 7. here rod can be arranged parallel or perpendicular to the length of the chain. let n 1 be the numbers of the rods which are parallel and n 2 be the number of the rods that are perpendicular to t he length. hence t he partition function of t he system where we take parallel states has the energy E and the perpendicular states has the energy -E now average length we have

= -e --/3c-+-e-f3c- n1 ae- c/3

it should be the average length . but we have that if we make T is very large energy then probability of the rods having parallel situation gets lowered and tensed to zero and the probability of having length perpendicular gets maximized or one can say all the rod take perpendicular configuration so n 2 -+ N hence applying t his in the equation we get N aec/3

< L >=---e-/3c + e/3c which can be simplified to write down as

< L >= Na/ (1 + e-2c/kBT)

11 oo : oo : so < L >= - - - e-f3c:

+ ef3c:

which can be simplified to write down as

< L >= Na/ (1 + e- 2c:/kBT) so option c is correct. [email protected]

physicsguide CSIR NET, GATE

18

Canonical Ensembles: Part-1

@Sk J ahiruddin , 2020

Sol 1.8. t he energy of the particle has two contribution one for the kinetic energy and t he other is due to t he potential

energy . so the energy 2

2

Px+Py +Pz + E = - - - - - mgz 2m where we take the single part icle is at a distance z from t he bottom of the cylinder and we have defined before t he partition function for t he cont inuous system hence we will have

z=

CX)

1

CX)

e- Px2 /3 / 2m dpx

fi3

e- Py 2 /3 / 2m dpy

- oo

-

CX)

L

CX)

e- Pz2f3/2mdpz

e-mgzf3 dz

dxdy

0

-00

so calculating we get Z

= 1rR

2

mkBT 21rn2

312

1 _ e-mgL/kBT

mg/ kBT

so for the n particle system we will have

Zn

=ZN

11 oo :oo : s2 so for the n particle system we will have Zn =ZN

and we have


- B 8T 2

as we take the L is very very large compare to R . j [email protected]

19

physicsguide CSIR NET 1 GATE

Car1onical Ensembles: Part-1

@Sk J ahiruddin, 2020

Sol 1.9. we know that the Helmholtz free energy is related to t he partit ion function t hrough t he relation is given by

so comparing t his to the given quant ity of t he F and we get t hat

which is t he N particle partit ion function . we know that the average energy is related to t he part it ion function is given by _8lnz_ U

8/3 -


so we will now use the expression of ln z in this case we get

so option a is correct. Sol 1.10. t he total energy associated to the system is given

11 oo : oo : 55

so option a is correct. Sol 1.10. t he total energy associated to t he system is given by

E = (n1

1

1

+ 2 )nw1 + (n2 + 2 )n2w1

so t he ground state energy of the syst em is when n 1 = n 2 = 3 1 hence E 1 = ~w and the first excited state has the energy 5 E 2 = ~w and t he 2nd excited state is doubly degenerate and [email protected]

20

physicsguide CSIR NET, GATE

@Sk J ahiruddin, 2020

Canonical Ensembles: Part-1

7

has the energy E 3 = ~w and the other states have energy ..., greater than the given energy so we have probability p

e-3(3nw/2 + e-5(3nw/2 + 2e-7f3nw/ 2

= _ _ _ _ _ _ _ _ _ _ = x 312 (l + x + 2x2 )/ Z z

Sol 1.11. t he average energy of the system is given by

I: E ie - Ei/3

= - - - -

z

where Z is the partition function of the syst em hence we have £e- c/3 - sec/3

= - - - - 2 + ec/3 + e- c/3

so we will have

< E >=

e/3c/2 _ e - /3c/ 2 -s e/3c/2+ef3e12

/3£ =

- £

tanh 2

Sol 1.12. t he energy of the harmonic oscillator in two di-

mension is given by

11 oo :oo : s1 e/3c/2 _ e-/3c/2

< E >=

/3£ = -£ tanh

e/3c/2+ef3e12



2

Sol 1.12. t he energy of t he harmonic oscillator in two di-

mension is given by

E

PX 2+pY 2 +

=----

2m

ax2+ y 2

t hen the single part icle part ition function is given by 1 Z = h2 CX)

00

00

e- Py 2 /3 /2m dpy

e- Px2f3/2mdPx - oo

- oo

2 e- o:f3x dx

- (X)

00

2

e- o:f3y dy - (X)

j [email protected]

21

physicsguide CSIR NET I GATE

Canonical Ensembles: Part-I

@Sk J ahiruddin , 2020

so we have

41rmkBT1rkBT z=------Ji2 O'.

so we will have Zn

=

2m7r2 a f32

N

Sol 1.13. t he part ition function

z=

e - nw/32

now we know t hat t he Helmholtz free function we have

so we put t he value of t he partition function in t he part !iw

F • 11

1

= kBT 2kBT

11 oo : 01 : oo so we put t he value of t he part it ion function in t he part F

so as T



=

nw kBT 2kBT

0 we will have F = nw/ 2

Sol 1.14. t he part ition function is defined as 00

Z = 2_ h

00

p2 e2mkBT

-

dp

alxl

e ksT dx -oo

-00

1/2

21rm kBT

0

aJxJ

eksT dx

h2

- oo

00

+

_ aJxJ

e ksT dx 0

1/ 2

so opt ion c is correct. [email protected]

@Sk J ahiruddin ) 2020

22

physicsguide CSIR NET ) GATE

Canonical Ensembles: Part-1

Sol 1.15. let us first calculate the eigen value of t he system solving eigen value equation we get

now the eigen value has two possible values A = ✓rr2 + 62 now we will have the second value A = - ✓ry 2 + 62 so the partition function

Sol 1.16. Sol 1.17. t he part ition function of the system

11 oo : 01

: 02

Sol 1.16. Sol 1.17. t he part ition function of t he syst em

Z =l

+ 2e- 136

so definitely the energy would b e

8 ln z 2ce- /3c = - - - = - - 8/3 l + 2e-/3c and we know that

- au -

2 - /3€ 1 + 2e-/3c cv - 8T - 2kB(/3c) e (1 + 2e- /3c)2 so we t ake the limit ing condition t hat is /3c > > 1 so we haveCv = 2kB(/3c ) 2 e- /3c 23

[email protected]

physicsguide CSIR NET, GATE

@Sk J ahiruddin, 2020

Car1onical Ensembles: Part-1

Sol 1.18. here the energy is proportional to the cube of t he moment um but we first try to think that if energy is proportional to some power of the momentum then we get

E ex: PS , but we know that pressure is related to the energy is given by

-

sE P =-3V where P is the pressure. hence we have

p

-

s

()(

-

11 oo : 01 : os by

-

sE P =-3V where P is the pressure. hence we have

p

-

E

s

ex -

3

here s=3 given so we have pressure P = E V

at fixed T. so option is c. Sol 1.19. t he partition function of the syst em is given by

z = 1 + e-f./kBT + ef./kBT SO t he probability wit h energy E=O and we have

eo

eo

P =-=---z 1 + 2 cosh2

E

= 2kBT hence

1 1 + 2 cosh2

so option a is correct. j [email protected]

physicsguide CSIR NET, GATE

24

Canonical Ensembles: Part-1

@Sk J ahiruddin , 2020

Sol 1.20. from t he kinetic theory of the gases we have that force is given by 2

m < v > ML - 2 < F >=---= t L we have n mole ideal gas so we have nIVI=m where M is t he molecular mass. hence we have

,

In

11 oo : 01 : os we have n mole ideal gas so we have nJVI=m where M is t he molecular mass. hence we have

which gives that t=LT- 112 Sol 1.21. t he Helmholtz free energy is related to t he par-

t ition function is given by F = -kBT In Z now we are to evaluate t he partition function and is given by

z

=

L (2l + 1)e- Et/kBT = L (2l + l

2 1 )e- h l(l+l )/2I0 k 8 T

l

now t his can be simplified F

=

-ksTinZ

=

-ks Tln

1 + L (2l + l )e-h2l(l+l) / 2Ioks T l

but we know that ln( l F = - K ET L (2l

+ x)

2

= x - x / 2... so

+ l )e- h2l (l+l )/2IokBT =

- 3kBTeh2 / IokBT

l

where we neglect t he higher order terms [email protected]

@Sk J ahiruddin, 2020

25

physicsguide CSIR NET, GATE

Canonical Ensembles: Part- I

Sol 1.22. t he energy of diatomic molecule considering t he molecule as the simple harmonic oscillator we get En =

(n + ~) nw now we nee to calculate t he part ition function of t he system and to do this we would like to refer to the problem to the previous one problem 30 from which we get t he partition function which is given by

11 oo : 01 (n

: 11

+ ~) fiw

now we nee to calculate t he part ition function of t he system and to do this we would like to refer to the problem to the previous one problem 30 from which we get t he partit ion funct ion which is given by e- f3nw/2

Z =--1 - e- f3nw

so the probability we have for t he part icle being in t he lowest possible energy state

so the option a is correct. Sol 1.23. t he part ition function of t he system

z = 1 + e- 6./ kBT + e- 26. /kBT hence we can say that the average energy is given by

8 ln 1 + e-6./3 < E >= 8{3

+ e- 26./3

2~ e- 26./3 + ~ e- 6./3 1 + e-6./3 + e- 26./3

now t he specific heat

au

cv = 8T jahir@physicsguide. in

@Sk J ahiruddin , 2020

which essentially gives us

26

physicsguide CSIR NET, GATE

Canonical Ensembles: Part-1

11 oo : 01

: 13

which essentially gives us

now we take

/3~ =
= _kB_T_

ax

4

and we have

j [email protected]

27

physicsguide CSIR NET 1 GATE

Car1onical Ensembles: Part-1

@Sk J ahiruddin , 2020

hence we have

< E >= _3N_kB_T_ 4

00 : 01 : 16 Canonical Ensembles: Part-1

@Sk J ahiruddin , 2020

hence we have

< E >= _3N_kn_T_ 4

for N particle and as we have {JU 3Nkn Cv ={JT= 4

Sol 1.25. we know that the


-

8/3

now we are to evaluate

8 8(3

8

8 In Z

8(3

8/3

l8Z 8/3 z 8/3 {)

now t his can be written as

8 8(3

1

az

z2

8(3

2

1 a2 z

-

-

z

8/3 2

now let us calculate what t his quant ity gives you if we define

t hen t he 2nd t erm of the above equation is giving you 1 82 Z -

z

~ E 2e- f3E

z

a13 2 28

[email protected]

Canonical Ensembles: Part-1

@Sk J ahiruddin, 2020

which is basically < E

physicsguide CSIR NET, GATE

2

> and the first term is basically

11 oo : 01

: 1s

@Sk J ahiruddin, 2020

Canonical Ensembles: Part-1

2

which is basically < E > and the first term is basically 2 giving you < E > so t he option a is correct. Sol 1.26. t he average energy is given by ce-c/3 - c ec/3

=----l

+ ec/3 + e-c/3

which can be written as function to get t he following

[here we take Taylor expansion and t ake only first order contribution] Sol 1.27. to solve t his problem we have to first find out t he partition function . this partit ion function has been

calculated in t he problem 34 please see that we will take t he result here. the partition function 21rmkBT

3N/ 2

h2 so Helmholtz free energy function is given by

so t he pressure is given by

j [email protected]

@Sk J ahiruddin, 2020

29

physicsguide CSIR NET, GATE

Canonical Ensembles: Part-1

11 oo : 01

: 21

P = -

= --

8V

j [email protected]

V

29

physicsguide CSIR NET, GATE

Car1onical Ensembles: Part-1

@Sk J ahiruddin , 2020

Sol 1.28. See solution of problem Prob 1.4. Sol 1.29. t he mean square fluctuation is defined as

which gives us

au 8T

r a- 2

ex

so by integrating we get U ex

r a -l

option c is correct.

j [email protected]

30

physicsguide CSIR NET, GATE

11 oo : oo : oo

Problems and Solutions in Canonical Ensembles: Part-2 Sk J ahiruddin * Suchismito Chatterjee

*Assistant P rofessor Sister Nibedita Govt. College, Kolkata Author was the topper of IIT Bombay 1\11.Sc Physics 2009-2011 batch He ranked 007 in IIT JAl\lI 2009 and 008 (JRF) in CSIR NET June 2011 He has been teaching CSIR NET aspirants since 2012

1

@Sk J ahiruddin , 2020

Car1onical Ensembles: Part-2

11 oo : oo : 03 1

Canonical Ensembles: Part-2

@Sk J ahiruddin, 2020

Contents 1

Problems from NET, GATE, JEST , TIFR & JAM pap ers 1.1 Ans Keys 1.2 Solutions . • • •

j [email protected]































2





3 •



































13 14

physicsguide CSIR NET, GATE

11 oo : oo : 06

j a hir@physicsguide. in

2

physicsguide CSIR NET I GATE

Canonical Ensembles: Part-2

@Sk J a hiruddin , 2020

1

Problems from NET, GATE, JEST, TIFR & JAM papers

Prob 1.1. If t he Hamiltonian of a particle is 2

H =

t hen (x + xy + y 2017] 2

(a)

kBT

(b)

2

)

2

Px + Py+ - - - xy

2m

at temperature T is equal to 1

kBT

2

(c)

2kBT

[JEST

(d)

(Hint: Generalized equlipartition t heorem) Prob 1.2. Let a particle of mass 1 x 10- 9 Kg, constrained

to have one dimensional motion, be initially at t he origin (x = Om) . T he particle is in equilibrium wit h a t hermal bath (KBT = 10- 8 J . What is (x 2 ) of the particle atfer time t = 5s? [JEST 2017] Prob 1.3. For a classical system of non-interacting par-

ticles in t he presence of a spherically symmetric potential V (r) = , r 3 , what is t he mean energy per particle? , is a [JEST 2018] constant.

11 oo : oo : os

,r

V (r) = constant.

3,

what is the mean energy per particle? , is a [JEST 2018]

[email protected]

3

physicsguide CSIR NET ) GATE

Canonical Ensembles: Part-2

@Sk J ahiruddin ) 2020

(Hint: Generalized equlipartition theorem) Prob 1.4. A large cylinder of radius R filled with particles

of mass m . The cylinder spins about its axis at an angular speed w radians per second , providing an acceleration g for t he particles at the rim. If t he temperature T is constant inside the cylinder , what is t he ratio of air pressure P0 at [JEST 2018] t he axis to the pressure Pc at the rim? (a) exp

mgR

(b) exp

2kbT

(d) 2kbT mgR

Prob 1.5. A quantum system has 3 energy levels -0.12 eV, -0.2 eV and -0.44 eV respectively. 3 electrons are dis-

t ributed among these levels. At a temperature 1727°C, the system has total energy -0.68 eV. The free energy of the [TIFR 2010] system is approximately (a) +1.5 eV (b) + 0.3 eV (c) -0.1 eV (d) -0.3 eV (e) - 1.0 eV (f)- 1.5 eV Hint : Calculate Helmholtz Gibbs both Prob 1.6. N particles are distributed among 3 states having energies E = 0, kBT and 2kBT respectively. If t he total

11 oo : oo : 11 (e)-1 .0 eV (f) - 1.5 eV Hint : Calculate Helmholtz Gibbs both

Prob 1.6. N particles are distributed among 3 states having energies E = 0, kBT and 2kBT respectively. If t he total equilibrium energy of t he system is 138.06 kBT, find the [email protected]

@Sk J ahiruddin, 2020

number N of particles.

4

physicsguide CSIR NET, GATE

Car1onical Ensembles: Part-2

[TIFR 2016]

Prob 1. 7. In a monatomic gas, t he first excited state is only l .5eV above the ground state, while t he other excited states are much higher up. The ground state is doublydegenerate, while the first excited state has a 4-fold degeneracy. If now t he gas is heat ed to a temperature of 7000 K, t he fraction of atoms in t he excit ed state will be approximately [TIFR 2015] (a) 0.42 (b) 0.3 (c) 0.14 (d) 0.07 Prob 1.8. Consider a system of non-interacting part icles with integer angular momentum J at a t emperature T. This system is placed in a magnetic field B in t he z direction. The energy of a state wit h Jz = m ri is Em = mµB B with µB > 0. The fr actional magnetization of the particles as a function of µBB / kBT can be represented as [TIFR 2017]

00 : 00 : 14

j [email protected]

physicsguide CSIR NET, GATE

5

@Sk J ahiruddin , 2020

Canonical Ensembles: Part-2

--0.3

0.0

--0.4

-0.2

--0.5

-

-, -0.4 E -0.6

~

E

--0.6 --0.7 --0.8 --0.9

-0.8

· 1.0

0.0

0.5

1 .0

1.5

2.0

1.=::::...__ _ __ 0.0

0.5

1.0

1.5

2.0

µ8 B/K 8 T

IJeBIKeT

0.8

1.0

--....._

0.9

0.6

0.8 ~

E

0.4

-,

0.7

e o.6

0.3

0.5

0.4

0.0

0.3

0.0

0.5

1.0

1.5

2.0

0.0

1Je8/K8 T

0.5

1.0 µ9 B/K 9 T

1.5

2.0

Prob 1.9. A one-dimensional quantum harmonic oscillator of natural frequency w is in t hermal equilibrium with a

heat bath at temperature T . The mean value (E ) of the energy of t he oscillator can be written as [TIFR 2017]

nw

nw

nw

(a) 2 sech 2kBT

(b)

,, nw

, _, nw

_fnw\

2

csch

nw

11 oo : oo : 16 heat bath at t emperature T . The mean value (E ) of t he [TIFR 2017] energy of the oscillator can be writ t en as

nw

nw

nw

(a) 2 sech 2kBT

nw

(c) 2 coth

(b ) 2 csch

nw

nw

(d) 2 tanh

2kBT

nw

2kBT

nw 2kBT

Prob 1.10. A classical ideal gas of atoms wit h masses m is confined in a three-dimensional pot ential

[email protected]

6

@ Sk J ahiruddin, 2020

physicsguide CSIR NET, GATE

Canonical Ensembles: Part-2

at a temperature T . If kB is the Boltzmann const ant, the root mean square (r.m.s.) distance of the at oms from the or1g1n 1s [TIFR 2018 •





(b)

(a)

(c)

(d)

Prob 1.11. N particles are distributed among three en-

ergy levels having energies: 0, kBT and 2 kBT resp ectively. If the total equilibrium energy of t he syst em is approximately 42 .5 kBT then find the value of N (to t he closest [TIFR 2018 integer).

Prob 1.12. A statist ical system , kept at a t emperature T , has n discret e energy levels wit h equal level- spacing E,

11 oo : oo : 19 integer).

[TIFR 2018

Prob 1.12. A statistical system , kept at a temperature T , has n discrete energy levels wit h equal level- spacing c, starting from energy O. If, now, a single part icle is placed in the system what will be the mean energy of t he system in the limit as n ➔ oo? [The answer should not be left as a [TIFR 2018 summation ] Prob 1.13. Consider a thermal ensemble at temperature T which is composed of identical quant um harmonic [email protected]

7

physicsguide CSIR NET, GATE

@Sk J ahiruddin , 2020

Canonical Ensembles: Part-2

lators of frequency w 0 wit h non-overlapping wavefunctions. The probability that there will be an even number of energy quanta in t he system is [TIFR 2019] (a) 1 exp (- nwo/ kBT) + 1

(C)

i

(d )

tanh ( nw0 I 2kBT )

exp (-nwo/kBT) - 1 Prob 1.14. Consider a system of 2 non-interacting classical particles which can occupy any of t he 3 energy levels: E = 0, E and 2c with degeneracies g(E) = 1, 2, 4 respect ively. T hee mean energy of the system is [GATE 2008] 4 exp(-c/kBT)

+ 8 exp(-2c/kBT )

(a)c

1 + 2 exp(-c/kBT ) + 4 exp(-2c/kBT )

(b)c

2 exp(-c/kBT) + 4 exp(-2c/kBT) 1 + 2exp( -c/kBT) +4exp(-2c/kBT) I

'"' ~--- I

_

I,_

rr \

,

o ~--- I

() _ I 1_

rr \

\

11 oo : oo : 21 (a)c:

4 exp(-c:/kBT ) + 8 exp(-2c:/kBT ) 1 + 2 exp ( -E / kB T ) + 4 exp ( - 2c /kBT )

(b )c:

2 exp(-c:/kBT) + 4 exp(-2c:/ kBT) 1 + 2 exp(-c:/kBT) + 4 exp(- 2c:/kBT)

(c)c:

2 exp(-c:/kBT) + 8 exp(-2c/ kBT) 1 + 2 exp(-c:/kBT ) + 4exp(-2c:/kBT )

(d)c:

ex p(- c:/kBT ) + 2 exp(- 2E/ kBT ) 1 + exp(-c:/kBT) + exp(-2c:/kBT)

Prob 1.15. Consider a gas of atoms obeying MaxwellBoltzmann statist ics. T he average value of eia-f over all the moments

p of each of t he particles (where a is a constant

vector, a is the magnitude, m is t he mass of each atom, T is t he t emperature and kB is Boltzmann constant) is [GATE j [email protected]

8

Car1onical Ensembles: Part-2

@Sk J ahiruddin , 2020

2013] (a) 1

physicsguide CSIR NET 1 GATE

(b) 0

Prob 1.16. At a given t emperature T , the average per particle of a non-interacting gas of two-dimensional classical harmonic oscillators is _____ kBT. [GATE 2014]

Prob 1.17. The average energy U of a I-d imension al qu ant um oscillator of frequency w and in contact wit h a h eat bath at temperature T is given by [GATE 2015] 1 1 1 1 (a) U = 1uv coth (3nw (b) U = 1uv sinh (3!uv 2 2 2 2 1 1 1 1 (c) U = !uv tanh f3 nw (d) U = !uv cosh (3 !uv

2

2

2

2

11 oo : oo : 24 at temperature T is given by (a) U (c) U

1

1

2

2

1

1

2

2

= nw coth = nw tanh

[GATE 2015]

{3nw

(b) U

{3nw

(d) U

1

1

2 1

2

= nw sinh = nw cosh

2

{3nw

1

{3nw

2

Prob 1.18. Consider a triatomic molecule of the shape shown in the figure below in t hree dimensions. T he heat capacity of this molecule at high t emperature (t emp much higher than t he vibrational and rotational energy scales of the molecule but lower than it s bond dissociation energies) is [GATE 2017] (a) 3/2kB (b) 3kB (c) 9/2kB (d) 6kB Prob 1.19. T he partition function of an ensemble at a N

k;T ,

temperature T is Z 2cosh where kB is the Boltzmann constant. The heat capacity of t his ensemble at [email protected]

physicsguide CSIR NET, GATE

9

Canonical Ensembles: Part-2

@Sk J ahiruddin, 2020

•••••• •• ♦ ♦ ♦ ♦





T =



♦ ♦ ♦

is X NkB, where t he value of X is ____ (up to two decimal places) . [GATE 2018] k~

Prob 1.20. Consider a one-dimensional gas of N noninteracting part icles of mass m with t he Hamiltonian for a single particle given by,

11 oo : oo : 26 Prob 1.20. Consider a one-dimensional gas of N noninteracting particles of mass m wit h t he Hamiltonian for a single part icle given by,

1

p2

H

+ -mw (x + 2x) 2m 2

=-

2

2

The high temperature sp ecific heat in units of R = NkB (K B is t he Boltzmann constant ) is [GATE 2019]

(A) 1

(C) 2

(B) 1.5

(D) 2.5

(Hint: use generalized equipartition theorem)

Pro b 1. 21. A 2-level syst em has energies O and E . The level with zero energy is non-degenerate, while t he level wit h energy E is t riply-degenerate. T he mean energy of a classical part icle in this system at temperature T is [GATE 2016] E e - E/kBT

(a) 1 + 3e- E/ kBT

E e - E/kBT

(b) 1 + e - E/kBT

j [email protected]

10

(c)

3 E e - E/ kBT 1 e - E / kBT

+

(d)

physicsguide CSIR NET I GATE

Canonical Ensembles: Part-2

@Sk J ahiruddin , 2020

3 E e-E/ kBT

1 + 3e- E/ kBT

Prob 1.22. A syst em has 2 energy levels wit h energies c and 2c . The lower level is 4-fold degenerate while the upper level is doubly degenerate. If there are N non-interacting classical part icles in t he system, which is in t hermodynamic equilibrium at temperature T , t he fraction of part icles in t he upper level is [GATE 2011]

(a)

1

1

:FI Qr k

( b)

1

,

r,

~FI Qr k

11 oo : oo : 29 egenera e. non-in erac 1ng classical part icles in the system, which is in thermodynamic equilibrium at t emperature T , t he fraction of particles in t he upper level is [GATE 2011] 1

1

(a) 1 + eE-/kBT

(c)

2e•fkaT

(b) 1 + 2eE-/kBT

~ 4e2 0). One system is populated by spin-~ fermions and t he other by bosons. What is the value of E p - EB where E p and EB are the ground state energies of the fermionic and bosonic systems respectively? [email protected]

@Sk J a hiruddin , 2020

4

physicsguide CSIR NET , GATE

Quantum Stat Mech

[NET June 2013]

(a) 6co

(b) 2co

(c) 4cO

( d)

co

Prob 1. 7. The minimum ener·gy of a collection of 6 noninteracting electrons of spin-½ and mass m placed in a onedimensional infinite square well potential of widt h L is [NET Dec 2012] 2 2 2 2 2 2 2 2 2 (a) 141r n /mL (b) 911r n /mL (c) 71r n /mL (d) 31r2 n2 / mL 2 Prob 1.8. The number of ways in which N identical bosons can be distributed in two energy levels is (a) N + 1 (b) N(N - 1)/ 2 (c) N (N + 1)/2 (d) N [NET June 2012] Prob 1.9. 3 identical spin-~ fermions are to be distributed in 2 distinct, non-degenerate energy levels. The number of ways t his can be done is [NET Dec 2013] (a) 8 (b) 4 (c) 3 (d) 2 Prob 1.10. Consider a gas of Cs atoms at a number den-

11 oo : oo : 14 in 2 distinct, non-degenerate energy levels. The number of ways this can be done is

(a) 8

(c) 3

(b) 4

[NET Dec 2013]

(d) 2

Prob 1.10. Consider a gas of Cs atoms at a number den12

sity of 10 atoms / cc. When the typical inter-particle distance is equal to the thermal de Broglie wavelength of the particles, the temperature of t he gas is nearest to (Take the j [email protected]

5

physicsguide CSIR NET, GATE

Quantum Stat Mech

@Sk J ahiruddin, 2020

26

mass of a Cs atom to be 22.7 x 10- kg) 2016] (a) lx10- 9 K (b)7x10- 5K

[NET June

(c) 1 x 10- 3 K

Prob 1.11. The number of ways of distributing 11 indist inguishable bosons in 3 different energy levels is. June 2018] (a )

311

(b) 11 3

(13) ! (c) 2! (11 )!

[NET

(11)!

( ) d 3!8!

Prob 1.12. Consider a syst em of two particles A and B. Each can occupy one of three possible quantum states 1), 2) and 3). The ratio of the probability t hat the two particles are in the same state to t he probability that two particles are in different states is calculated for bosons and classical (Maxwell- Boltzmann) particles. They are respectively [JEST 2013] (a) l , O

(b)l/2,1

(c) l , 1/ 2

(d) 0, 1/ 2

11 oo : oo : 16 cles are in different states is calculated for bosons and classical (Maxwell- Boltzmann) particles. They are respectively [JEST 2013] (a) 1, 0 (b) 1/ 2, 1 (c) 1, 1/ 2 (d) 0, 1/ 2

Prob 1.13. Consider 3 situations of 4 particles in a 1dimensional box of width L with hard walls. In case(i) t he particles are fermions, in case(ii) they are bosons and in case(iii) t hey are classical. If the total ground state energy of t he 4 particles in these 3 cases are E p ,EB , Eel respectively, [email protected]

6

physicsguide CSIR NET, GATE

Quantum Stat Mech

@Sk J ahirudd in, 2020

which of the following is true? (a) E p=EB=Ecz (b) Ep > EB=Ecl

(c) E p < EB < Eel

[JEST 2013]

(d) Ep > EB > Eel

Prob 1.14. The lowest quantum mechanical energy of a particle confined in a I-dimensional box of size L is 2 eV . The energy of the quantum mechanical ground state of a [JEST system of 3 non-interacting spin- ~ particles is 2014] (a) 6eV (b) lOeV (c) 12eV (d) 16eV Prob 1.15. A 2-dimensional box in a uniform magnet ic field B contains N/ 2 localized spin-t particles with magnetic moment µ and N / 2 spinless particles which do not interact with each other. The average energy of the system at temperature T is [JEST 2016] 1 µB (a) 3NkBT- NµBsinh kBT 2

11 oo : oo : 19 netic moment µ and N / 2 spinless particles which do not interact with each other . T he average energy of the syst em at temperature T is 1 . (a) 3NkBT - NµBs1nh 2

[JEST 2016] µB kBT

1 (b) NkBT- - NµBtanh

µB 2 kBT 1 1 µB (c) -NkBT- -NµBtanh k T 2 2 ~B

3 1 (d) NkBT+ Nµ B cosh 2 2

µB kBT

Pro b 1.16. The energy spectrum of a particle consists of jahir@physicsguide. in

@Sk J a hiruddin , 2020

7

physicsguide CSIR NET , GATE

Quantum Stat lVIech

four states wit h energies 0, E, 2E, 3E. Let ZB(T), Z p(T ) and

Zc(T ) denote the canonical partit ion functions for four noninteracting particles at temperature T. The subscripts B , F and C corresponds to bosons, fermions and distinguishable classical particles, respectively. Let y == exp - k~T . Which one of t he following statem ent s is true about ZB(T), Zp(T ) and Zc(T )? [JEST 2019] (A) They are polynomials in y of degree 12, 6 and 12, resp ectively. (B) They are polynomials in y of degree 16, 10 and 16, resp ectively. (C) They are polynomials in y of degree 9, 6 and 12, respect ively. (D) They are polynomials in y of degree 12, 10 and 16, re-

11 oo : oo : 21 sp ectively. (C) They are polynomials in y of degree 9, 6 and 12, respect ively. (D) They are polynomials in y of degree 12, 10 and 16, respectively. Pro b 1.1 7. A gas of non-interacting particles, each of rest

mass l Me V , is at a temperature T = 2.0 x 107 K and has an average particle density n = 2.7 x 1034 cm- 3 . We can obtain a reasonably correct treatment of t his system [TIFR 2016] (a) only by using special relativity as well as quantum mechanics. (b) by neglecting quant um mechanics but not special t heory j [email protected]

@Sk J ahiruddin, 2020

8

physicsguide CSIR NET 1 GATE

Quant um Stat Mech

of relativity. (c) by neglecting sp ecial relativity but not quantum mechanics. (d) by neglecting bot h special relativity and quantum mechanics. Prob 1.18. A quant um mechanical system consist s of a

one-dimensional infinite box, as indicated in the figures below.

• • •

• ••• • • •

11 oo : oo : 24

•••• • • •

• • •

--------3 (three) identical non-interacting spin 1/ 2 particles, are first placed in t he box, and t he ground state energy of the system is found to be E 0 = l 8eV. If 7 (seven) such ident ical particles are placed in the box, what will be t he ground state energy, in units of eV? [TIFR 2017]

Prob 1.19. MSQ Consider a one dimensional harmonic oscillator of angular frequency w . If 5(five) identical particles occupy the energy levels of t his oscillator at zero t [email protected]

9

physicsguide CSIR NET, GATE

@Sk J ahiruddin , 2020

Quantum Stat Mech

perature, which of the following statement (s) about t heir [JAM 201 7] ground state energy E 0 is (are) correct? . 13 (a) If the particles are electrons, E 0 = liw. 2 . 25 (b) If t he particles are protons, Ea = liw. 2 2 (c) If t he part icles are spin-less fermions, E 0 =

;nw.

(d) If the particles are bosons, E 0 =

~nw.

Prob 1.20. For a system of 2 bosons, each of which can occupy any of the 2 energy levels: 0 and c, t he mean energy of the system at t emperature T is [GATE 2014] n

no n

n

11 oo : oo : 26 (d) If t he part icles are bosons, E 0 =

~nw.

Prob 1.20. For a system of 2 bosons, each of which can occupy any of t he 2 energy levels: 0 and c, t he mean energy

of the system at t emperature T is c:e- f3c: + 2c:e- 2f3c: l + c:e- f3c: (a) 1 + 2e- f3c: + e- 2f3c: (b) 2e- f3c: + e- 2f3c:

+ c:e- 2f3c: (c) 2 + e-f3c: + e-2f3c: 2c:e-f3c:

[GATE 2014]

+ 2c:e- 2f3c: 2 + e-f3c: + e-2/3c: c:e-f3c:

(d) Prob 1.21. Consider a system of 3 fermions which can occupy any of t he 4 available energy states wit h equal prob[GATE 2014] ability. The ent ropy of t he system is (a) kBln2 (b) 2kBln2 (c) 2kB ln4 (d) 3kBln4 Prob 1.22. Consider a system having 3 energy levels wit h

energies: 0, 2c: and 4c:, wit h respect ive degenerates 2, 2 and 3. Four bosons of spin-0 have to be accommodated in t hese levels such that t he total energy of t he system is 1Oc. T he number of ways in which it can be done is ? [GATE 2016] j [email protected]

@Sk J ahiruddin , 2020

10

physicsguide CSIR NET I GATE

Quant um Stat Mech

Prob 1.23. Consider two particles and two non-degenerate

quant um levels 1 and 2. Level 1 always contains a particle. Hence, what is t he probability t hat level 2 also contain a particle for each of t he two cases. (i) When t he two particles are dist inguishable (ii) when t he two particles are Bosons. [GATE 2017] (a) (i) 1/ 2 and (ii) 1/ 3 (b) (i) 1/ 2 and (ii) 1/ 3

11 oo : oo : 29 particle for each of the two cases. (i) When t he two particles are distinguishable (ii) when t he two particles are Bosons. [GATE 2017] (a) (i) 1/ 2 and (ii) 1/3 (b) (i) 1/ 2 and (ii) 1/ 3 (c) (i) 2/ 3

(d) (i) 1

and (ii) 1/ 2

and (ii) 0

Prob 1.24. Three particles are to be distributed in four non-degenerate energy levels. The possible number of ways of distribution: (i) for distinguishable particles, and (ii) for [GATE 2018] identical Bosons, respectively, is

(a) (i) 24, (ii)4 (c) (i)64, (ii)20

(b) (i) 24, (ii )20 (d) (i)64, (ii)l6

Prob 1.25. The quantum effects in an ideal gas become important below a certain temperature TQ when de Broglie wavelength corresponding to the root mean square thermal speed becomes equal to the inter-atomic separation. For such a gas of atoms of mass 2 x 10726 kg and number density 6.4 x 1025 /m3 , TQ = ___ x 103 K (up to one decimal place) [email protected]

@Sk J a hiruddin ) 2020

11

physicsguide CSIR NET ) GATE

Quantum Stat Mech

[GATE 2018] (kB

= 1.38 x 10- 23 J / K, h = 6.6 x 10-34 J-s)

Prob 1.26. The energy-wave vector (E-k) dispersion relation for a particle in two dimensions is E = Ck, where C is a constant. If its density of states D (E) is proportional

11 oo : oo : 32 (kB = 1.38 x 10- J / K , h = 6.6 x 10- J-s) Prob 1.26. The energy-wave vector (E-k) dispersion relation for a particle in two dimensions is E = Ck, where C

is a const ant . If its density of states D (E) is proportional to E P t hen the value of p is - [GATE 2019]

Prob 1.27. Consider two syst ems A and B each having two dist inguishable particles. In bot h the syst ems, each

part icle can exist in states wit h energies 0, 1, 2 and 3 units wit h equal probability. The total energy of the combined system is 5 units. Assuming t hat t he system A has energy 3 units and t he system B has energy 2 units, the ent ropy of t he combined system is kB ln ..\. The value of ,\ is - - - [GATE 2019] Prob 1.28 . There are 4 energy levels: E, 2E, 3E and 4E (where E > 0). The canonical part it ion function of 2 parti-

[GATE 2013]

cles, if these are (i) 2 ident ical fermions

(a)

e - 2/3E

+ e - 4/3E + e - 6/3E + e - 8/3E

[email protected]

12

physicsguide CSIR NET, GATE

@Sk J ahiruddin, 2020

+ e-4/3E + e-5/3E + e-6/3E + e-7/3E (C) (e - f3E + e - 2/3E + e - 3/3E + e - 4f3E)2 (d) e-2/3E _ e-4/3E + e-6/3E _ e-8/3E (b) e-3/3E

Quantum Stat Mech

11 oo : oo : 34 (C) (e - f3E + (d)

e - 2(3E

e-2f3E _ e-4f3E

+ e - 3(3E + e - 4f3E) 2

+ e-6f3E _

e-8f3E

(ii) 2 distinguishable particles

a)

e - 2f3E

(b)

+ e - 4f3E + e - 6/3E + e - 8f3E

e - 3(3E

+ e - 4(3E + e - 5(3E + e - 6(3E + e - 7(3E

(C) (e - f3E + (d)

e - 2(3E

e - 2f3E _ e - 4f3E

+ e - 3(3E + e - 4f3E) 2

+ e - 6f3E

_ e - 8f3E

Prob 1.29. Consider a system of non-interacting particles in d dimensional obeying the dispersion relation E = Ak , E is the energy, k is the wave vector, s is an integer and A is a constant . T he density of states N ( E) is proportional to 8

[NET June 2012]

(a)E: -l

(b )E~- l

(c)E~+l

(d)E ~+l

Prob 1.30. Consider a gas of Cs atoms at a number density of 10 12 atoms / cc. When the typical inter-particle is equal to t he t hermal de Broglie wavelength of t he particles, t he temperature of the gas is equal to (Mass of Cs atom j [email protected]

13

Quantum Stat Mech

@Sk J ahiruddin , 2020

= 22.7

X

26

10- kg

(a) l xl0- 9 K 10-8 K

physicsguide CSIR NET, GATE

[NET Dec 2016]

(b)7xl0- 5K

(c) 1 x 10-3 K

(d) 2 x

00: 00: 37

= 22.7

X

10- 26 kg

(a) l x 10- 9 K

[NET Dec 2016] (b )7x 10- 5K

(c) 1 x 10-3 K

(d) 2 x

10-8 K

[email protected]

@Sk J ahiruddin, 2020

1.1

Ans Keys

14

physicsguide CSIR NET, GATE

Quantum Stat Mech

11 oo : oo : 40 @Sk J ahiruddin, 2020

1.1

Quantum Stat Mech

Ans Keys

1.1. a

1.11.

C

1.21. b

1.2.

1.12.

C

1.22. 18

C

1.3. a

1.13. b

1.23.

C

1.4. d

1.14.

C

1.24.

C

1.5. d

1.15.

C

1.25. 84

1.6. b

1.16. a

1.26. 1

1. 7. a

1.17.

1.27. 12

1.8. a

1.18. 132

1.28. (i) b (ii) c

1.9. b

1.19. a,c,d

1.29. b

1.10. d

1.20. a

1.30. d

1.2

C

solution

Sol 1.1. t he dist inguishable particle t hat is the classical particles obey t he Maxwell Boltzmann distribution function where as t he in dist inguishable part icle obey t he quant um statistics. act ually due to t he indist inguishablity we can not say t hat which particle belongs to which stat e and the probability remains unchanged from this condit ion arise two different quant um st at ist ics hence we can conclude t hat [email protected]

@Sk J ahiruddin , 2020

15

physicsguide CSIR NET, GATE

Quantum Stat lVIech

11 oo : oo : 42 [email protected]

15

physicsguide CSIR NET, GATE

Quantum Stat Mech

@Sk J ahiruddin, 2020

t ion a should be the answer b ecause indistinguishable part icle does not obey t he classical statistics. Sol 1. 2. as we are considering t he spin half particle so each

sate can contain two particles so t he lower energy state will have 2 particle where as t he higher energy state will have 1 particle as we have total 3 part icle in hand and hence t he energy is given by n 2 n2 4n 2 n2 3n2 n2 E = 2x - - + - = 2ma2 2ma2 ma 2

hence option c is correct. Sol 1.3. t he number of ways t he we can arrange according to t he MB statistics is given by N = aj nJ = 22 = 4 now we

are going to evaluate the the number of ways we can arrange in t he quantum fermi statistics N = nj 1.aj-nj ai' = 1 and we 1. 1 1 ((ni_-=_a )· -1 _)1' a1 1 .n 1 .

have for t he Bose Einst ein statistics N = hence if we take the ratio we will have NMB 4 : 3 : 1 hence option a is correct.

:

=

3

N BE : N F n

=

Sol 1.4. let t here are 3 states and 3 particle hence we can say t hat t he possible configuration we have 1) t he lower state

have 2 particle and the middle state has only one part icle 2) t he middle takes 2 and t he lowe1~st at e has one 3)the middle has 2 and the upper most has one particle 4) t he upper most j [email protected]

16

physicsguide CSIR NET, GATE

11 oo : oo : 44 t he middle takes 2 and the lower state has one 3)the middle has 2 and the upper most has one particle 4) t he upper most j [email protected]

16

physicsguide CSIR NET, GATE

Quantum Stat Mech

@Sk J ahiruddin , 2020

take 2 and t he middle takes only one 5) t he upper most t ake 2 particle and t he lower most take one 6) t he upper most take only one and lower most take 2 particle 7) each of state takes 1 particle each. so possible configuration is 7. option d is correct. Sol 1.5. two identical particle to be arrange in the 3 states.

now let us see the possible configuration in the fermeonic case and in the fermeonic sate there can never be two part icle in the single particle state hence would be zero. now take t he bosonic state t here could be t hree double particle configuration and there could be 3 single particle configuration so ratio is one . where in the classical state we will have to in corporate the distiguishability and hence we have to divide t he ratio of b statistics by 2! and hence we have t he ratio is given by rB : rp: re = l : 0: 1/ 2 Sol 1.6. let us first calculate the fermeonic case we have t he

for t he lowest energy to have t he most justified configurat ion of the system would be 2 particle would be t he lowest possible state and 1 particle would be in t he state of 3c state hence t he energy of t he configuration would be E p = 2c

+ 3c =

5c

and for t he bosonic case all particle will be at t he lowest

11 oo:oo:47 hence the energy of t he configuration would be E p = 2c + 3c = 5c

and for the bosonic case all part icle will be at t he lowest [email protected]

physicsguide CSIR NET, GATE

17

Quantum Stat Mech

@Sk J ahiruddin, 2020

energy sate hence we have EB = 3c hence we have the difference in the energy is given by

Sol 1. 7. each state can occupy two electrons hence t he energy of t he system is given by

n n E = 2x + 2 x 2ma 2 2 2

4n n 2ma 2

2 2

2 2

+ 2x

9n !i 2ma2

ma2

hence the opt ion a is correct . Sol 1.8. we have seen from the previous discussion that the number of ways we can arrange in t he bosonic syst em is given by ( N + 2 - 1) ! n=-----=----=N+ l (aj - l )!nj ! N !l! ( nj

+ aj

- l) !

so opt ion a is correct. Sol 1.9. we have the possible configuration given below

[A( 1/ 2, - 1/ 2 > )B( - 1/2 > )] [A( ll /2, - 1/ 2 > ), B ( 1/ 2 > )] [A(ll / 2 > ), B ( 1/ 2, - 1/ 2 > )]

fA(I - 1/ 2 > )B (l l / 2, - 1/ 2 > )l

11 oo : oo : so [A( 1/ 2, - 1/ 2 > )B( - 1/ 2 > )] [A( ll /2, - 1/ 2 > ), B( 1/2 > )] [A( ll /2 > ), B( 1/ 2, - 1/2 > )]

[A ( - 1/ 2 > )B ( 1/ 2, -1/2 > )] so possible configuration is 4. 18

[email protected]

physicsguide CSIR NET, GATE

@Sk J ahiruddin , 2020

Quantum Stat lVIech

Sol 1.10. t he thermal debroglie wave lengt h satisfies t he given relation N n3 - - - - - -2 = 1 V (21rmkBT) 31

at any temperature T now we have the atomic density hence we have

12 10

h2(n)2/3 T =-----

(2.61)213 21rmkB now just we are putting t he requi1~ed values and we will get

T =

6.28

X

6 62 10- 7 . x = 1.16 x 10-8 22.7 X 1.38 X (2.61 ) 2/3

hence option d is correct. Sol 1.11 . we have seen from the previous discussion that t he number of ways we can arrange in the bosonic system

is given by

( nj + a_i - l ) ! 13! n=-----=-(aj - l)!n_i ! 2!1 1!

hence we have the correct option is option c. So l 1.12. two particle will be in the same sate for t he bosonic

system will be 3 and the for classical system there will also

11 oo :oo : s2 l)!nj ! hence we have the correct option is option c. (aj -

Sol 1.12. two particle will be in the same sate for t he bosonic syst em will be 3 and t he for classical system there will also be 3 sates where there will be two particle in the same state hence the ratio of the occupancy is given by

p(B) P= j [email protected]

p(M) = 1 19

physicsguide CSIR NET 1 GATE

Quantum Stat Mech

@Sk J ahiruddin, 2020

and the single particle st ates in t he bosonic configuration will be 3 but for t he classical case the configuration will be 6 hence the ration will be 1/ 2 hence the we have t he answer 1, 1/ 2 and the option c is correct. Sol 1.13. in t he classical and t he bosonic system there could be we sate where all the part icles come to the same particle stage. so there is the lowest possible state energy for the boson and the classical case would be equal when the all particle comes to the lowest possible energy state and the energy of the syst em is given by

EB = Ec= 4

7r2t;,2 X --

2ma2

but in the fermeonic state each single part icle sate can occupy only one particle or none because if we want to accommodate t he 2 particle in the same single part icle state t hen t he wave funct ion vanishes t hat is the system prohibits to put two particle in the same single particle state hence the energy in the fermeonic case is given by

11 oo : oo : 55 cupy on y one part1c e or none ecause 1 we want o accommodate t he 2 particle in the same single particle stat e t hen t he wave function vanishes that is the system prohibits to put two particle in the same single particle state hence the energy in the fermeonic case is given by w2n2 4w2n2 9w2n2 16w2n2 Ep = - - + - + + 2ma2 2ma 2 2ma 2 2ma 2 hence we see that E p > EB = E e so option bis correct. Sol 1.14. t he lowest possible energy state has the energy 2eV hence we have to state that the lowest possible energy [email protected]

20

physicsguide CSIR NET, GATE

Quantum Stat Mech

@Sk J ahiruddin, 2020

of the one D box is given by

w2n2 --= 2eV 2 2ma

from the given condition we have and the system has 3 spin 1/ 2 particle hence t he lowest state comprises t he 2 particle and the higher state has t he only one particle. hence the energy of the system

w2n 2 4w 2 n2 E = 2 x - - + - -2 2ma2

2ma

6w 2 n 2

--=

2ma 2

12eV

hence the option c is correct. Sol 1.15. here is two contribution to t he internal energy 1) the first contribution is coming from the degree of freedom here we are considering only two dimensional system and hence two degrees of the freedom and the each degree of

t he freedom has l / 2kBT amount of t he energy and hence f"'{)ntrih11ti{)n ic:: k- r..T tl--iic:: f"'{)ntrih11ti{)n f()r th P N / ½ ? [NET Dec 2014] (a) ~ N ln 2 (b) 2 ln 2 (c) ½ln 2 (d) N ln 2 Prob 3.6. Consider 3 Ising spins at the vertices of a tri-

11 oo : 01

: 23

1

µB / kBT for which the mean value (Si) > ? [NET Dec 2014] (a) ½N ln 2 (b) 2 ln 2 (c) ½ln 2 (d) N ln 2 Prob 3.6. Consider 3 Ising spins at the vertices of a triangle which interact with each other with a ferromagnetic j [email protected]

physicsguide CSIR NET, GATE

30

@Sk J ahiruddin, 2020

Two state arid other systems

Ising interaction of strength J. The partition function of t he system at temperature T is [,B = 1/ kBT ] [NET June 2015] (a) 2e3f3 1 + 6e-f3 1 (b) 2e- 3f3J + 6ef3J (c) 2e3f3 1 + 6e- 3f3 J + 3e131 + 3e-f3J ( d) (2 cosh ,BJ) 3 Prob 3. 7. The partition function of N Ising spins is Z = Af + Af where A1 , A2 are functions of temperature but in-

dependent of N. If A1 > A2 , the free energy per spin in the limit N ➔ oo is [NET Dec 2015] (a) -kBTln(A1/ A2 ) (b)-kBTlnA2

(c) -kBTln(A1A2) (d) -kBTlnA1 Prob 3.8. The Hamiltonian of a syst em of N non-

31

spin- ½particles is H = - µoB LJiS[ , where S f= ± 1 a:_ _ _ ___. ponents of itli spin along an external magnetic field B . At µoB a temperature T , such that ekBT = 2, the specific heat per particle is [NET Dec 2015]

(a) * kB

(c) kB(ln2)

(b) ~kBln2 2

(d) * kB(ln2)

2

Prob 3.9. The Hamiltonian for three Ising spins S0 , S 1 , S2

,

11 oo : 01

: 26

particle is

[NET Dec 2015]

(b) ~kBln2

(a) ~kB

(c) kB(ln2)

(d) ~~ k B (ln2)

2

2

Prob 3.9. The Hamiltonian for t hree Ising spins S0 , S 1 , S 2 taking values ± 1, is H = - J S 0 (S1 + S 2 ). If t he system is in equilibrium at temperature T , t he average energy of t he [NET June 2017] system, in terms of f3 = (kBT )- 1 is 31

j [email protected]

physicsguide CSIR NET, GATE

@Sk J ahiruddin , 2020

Two state and other systems

a) _ 1 + cosh (2/3 J ) ( sinh(2/3 J ) 213 (c) _ 21/3

(b) - 2J(l

+ cosh (2/3 J )]

(d) _ 2J

sinh(2f3J ) 1 + cosh(2/3 J) Prob 3.10. The Hamiltonian of a one-dimensional Ising model of N spins (N large) is N

=-J

H

L O"ZO"i+l i= l

where t he spin az = ± l and J is a posit ive constant. At inverse temperature /3 = l / kBT t he correlation 32 between t he nearest neighbour spins (azaz+i) is Dec 2018] (a) e-f3J / ( ef3J + e-f3J) (b) e- 2f3J (d) coth(f3 J )

(c) tanh(/3 J )

Prob 3.11. The Hamiltonian of t hree Ising spins S 1 , S 2 and S3 , each taking values ± 1, is TT

T

/

rv

r'"'f

.

rv

rv \

'l

rv

,

11 oo : 01 (c) tanh(,BJ )

: 29

(d) coth(,BJ )

Prob 3.11. The Hamiltonian of t hree Ising spins S 1 , S 2

and S3 , each taking values ± 1, is

where J and h are posit ive constants. The mean value of S3 in equilibrium at a temperature T = 1/ (kB,B), is [NET June 2019] 2 3 (a) tanh (,BJ ) (b) tanh(,Bh) tanh (,BJ ) 32

j [email protected]

@Sk J ahiruddin , 2020

physicsguide CSIR NET 1 GATE

Two state and other systems

2

(c) sinh (,8 h) sinh (,8 J )

(d) 0

33

11 oo : 01

[email protected]

33

@Sk J ahiruddin ) 2020

3.1 3. 1.

: 31

physicsguide CSIR NET ) GATE

Tv.ro state arid other systems

Ans Keys C

3. 2. b

3. 5.

C

3. 9. d

3. 6. a

3. 10. 3. 3. d

3. 7. d

3. 4. b

3. 8. d

C

3. 11. b

34

11 oo : 01

[email protected]

@Sk J ahiruddin, 2020

3.2

34

: 34

physicsguide CSIR NET, GATE

Two state and other systems

solution

Sol 3.1. let us first take S 1 =S2=83 =1 t hen the energy of

t he system would be E = -J(l x l + l x 1+ 1 x 1) = -3Jnow again we take S1 =S 2=8 3=-l so we have E = - J (- 1 x - 1 + - 1 x - 1 + - 1 x - 1) = -3 J so there would be degeneracy 2 hence we have option 2 is correct.

Sol 3.2. let t he magnetic moment of the system . . . .. 34 angle 0 in general wit h the magnetic field hence we H =

e - µB /3 - eµB /3 B/3 B /3 e- µ eµ

+

= - tanh µB /3

so for N number of the particle of t he system we have < Si >= - N tanh µB /3 now according to the given condition we have

/N =l/3 at least. so we have tanh µB /3

= 1/3

t his gives us 1 µB /3 = ln2 2 Sol 3 .6 . we have the Hamiltonian

[email protected]

@Sk J ahiruddin , 2020

37

physicsguide CSIR NET I GATE

Two state arid other systems

and each spin can take either 1 or -1 value . so we have to see t he possible number of configuration and t here corresponding energy . we have taken a simple designation s 37 which gives you t he energy and t he spin configura t his is (S1 , s2, s3 , E ). so possible cases are (1 ,1,1,3J), (1,1,1,-J), (1,-1 ,1,-J), (-1 ,1,1,-J), (-1 ,-1 ,1,-J), (-1 ,1,-1 ,-J), (1,-1 ,-

11 oo : 01

: 44

ing energy . we have taken a simple designation skim here which gives you t he energy and t he spin configuration and t his is (S1, s2, s3, E ). so possible cases are (1,1 ,1,3J), (1,1,1,-J) , (1,-1 ,1,-J), (-1 ,1,1,-J), (-1 ,-1 ,1,-J) , (-1 ,1,-1 ,-J), (1,-1 ,1,-J) , (-1 ,-1 ,-1 ,3J) hence we see t hat 3J energy is doubly degenerate and the -J energy state has degeneracy 6 hence we have the partit ion function

Z = 2e- bf3J

+ 6e131

hence the option b is correct . Sol 3. 7. we know that

and as the z is given by we have that

F = - kBTln(A1 N

+ A2N)

and we have t he A1 > A2 hence we modify the case and get

F = -kBTln

N

A1 (1

+

A 2N

A1

N)

is the N is very large then A2 N / A1 N ➔ 0 hence we only have F = - NkBTlnA 1 so we have F / N = - kBTlnA 1 so option d is correct. j [email protected]

@Sk J ahiruddin, 2020

38

physicsguide CSIR NET, GATE

Two state arid other systems

Sol 3.8. from the previous discussion we have sever ~ ~.Q.Q..---. prove that the energy of the given kind of t he syst 38 -µ 0 B t anh µ 0 B /3 now we have t o calculate t he specr_ _ _ _ of t he system is given by

......a

11 oo : 01

: 47

Sol 3.8. from the previous discussion we have several t imes prove t hat t he energy of t he given kind of t he system E =

-µ 0 B t anh µ 0 B /3 now we have to calculate t he specific heat of the system is given by

dU 2 2 Cv = dT = (µo B /3 ) NkB sec hµ 0 B f3 but we know sec h(µo B /3) = e - µoB/3~ eµoB/3 and we are given t hat eµoB /3 = 2 so putting this in the equation we get Cv / N = ~~ kBln 2 hence option d is correct. 2

Sol 3. 9. here we can say t he Hamiltonian of the system is given by H = - J (s 0 s 1 + s0 s 2 ) and each spin can take either + 1 or -1 hence we are now wanting to calculate the possible configuration and t here corresponding energy and this will be denoted in t he manner given by (s 0 , s 1 , s 2 , E) .T he possi-

ble states are (1,1,1, -2J ),(-1 ,1,1,2J),(1,-1 ,1,0),(-1 ,-1,1 ,0),(1,1,1,0),(-1 ,1,-1 ,0) ,(1,-1 ,-1 ,2J) ,(-1 ,-1 ,-1 ,-2J) hence we see that t he energy of zero state has degeneracy 4 and 2J has degeneracy 2 and dengen of -2j is also 2 .hence t he average energy of the system is given by 0 + 2J e- 2f3J - 2J e2f3 1 E =-----4 +2e- 2f31 + 2e2f3 1

2J sinh 2f3 J 1 + cosh 2f3 J

hence option d is correct. j a [email protected]

@Sk J a hiruddin , 2020

39

physicsguide CSIR NET, GATE

Two state and other systems

39 Sol 3.10. here O"i = 1 or - 1 and hence we have the combination and t he are designat ed as (O"i O"i+ l , E) an?n:l'IESF_ ___.

11 oo : 01 j [email protected]

39

@Sk J ahiruddin, 2020

: 49

physicsguide CSIR NET, GATE

Two state arid other systems

Sol 3.10. here a i = 1 or - 1 and hence we have the different combination and the are designat ed as (a i a i+l, E ) and t hese

are given by (1,1,-J),(1,-1 ,J ),(-1 ,1,J ),(-1 ,-1 ,-J) hence t he energy J has degeneracy 2 and similarly -J has . so we have Z = 2e131 + 2e- 131 hence t he average correlation function we have


= - - - - - - - - - = tanh f3 J z

hence opt ion c is correct . Sol 3 .11. Lengthy calculation

j [email protected]

40

physicsguide CSIR N

40

11 oo : oo : oo

Problems and Solutions Ideal Fermi Systems



Ill

Sk J ahiruddin * Suchismito Chatterjee

*Assistant P rofessor Sister Nibedita Govt. College, Kolkata Author was the topper of IIT Bombay 1\11.Sc Physics 2009-2011 batch He ranked 007 in IIT JAl\lI 2009 and 008 (JRF) in CSIR NET June 2011 He has been teaching CSIR NET aspirants since 2012

1

@Sk J ahiruddin, 2020

Ideal Fermi Systems

11 oo : oo : 04 1

@Sk J ahiruddin, 2020

Ideal Fermi Systems

Contents 1

Problems from NET, GATE, JEST , TIFR & JAM pap ers 1.1 Ans Keys 1.2 Solut ions . • • •

j [email protected]































2





3 •

















8



















9

physicsguide CSIR NET, GATE

11 oo : oo : 06

j ahir@physicsguide. in

2

physicsguide CSIR NET I GATE

Ideal Fermi Systems

@Sk J ahiruddin , 2020

1

Problems from NET, GATE, JEST, TIFR & JAM papers

Prob 1.1. T he Fermi-Dirac distribut ion function [n(c:)] is (kB is t he Boltzmann const ant , T is t he temperature and c p is t he Fermi energy) (JAM 2019] 1 (a) n (c) = € - cp e k BT - l

Prob 1.2. The probability t hat an energy level c at t emperature T is unoccupied by a fermion of chemical potent ial µ is [GATE 2009]

(a)

1 exp

c-µ ka T

(b)

+1

1 exp

c- µ k BT

(c) -

1

1 exp

µ -c

kB T

+1

1 (d) - - exp - 1

Prob 1.3. Which one of t he following is a fermion? [GATE 2014] (a) a p art icle (b) Be~ nucleus (c) hydrogen atom (d) deut eron Prob 1.4. The pressure of a free, non-relativist ic Fermi gas in 3-dimensions dep ends, at T = 0, on the density of

11 oo : oo : os deuteron

Prob 1.4. The pressure of a free, non-relativistic Fermi gas in 3-dimensions depends, at T = 0, on the density of [email protected]

physicsguide CSIR NET ) GATE

3

@Sk J ahiruddin ) 2020

fermions n as (a) n5/3 (b) n l/3

Ideal Fermi Systems

(c) n 2/3

[NET June 2014] (d) n 4/3

Prob 1.5. The electrons in graphene can be t hought of as a two-dimensional gas wit h a linear energy-moment um relation E = fi1v, where p = (Px, Py) and v is a const ant . If p is t he number of electrons per unit area, the energy per unit area is proport ional t o [NET Dec 2016] (a) p3/2 (b) p (c) pl/3 (d) p2 Pro b 1. 6. The dispersion relation of a gas of spin-½ fermions ➔ ➔ in two dimensions is E = nv k , where Eis t he energy, k is t he wave vector and v is a constant with the dimension of velocity. If the Fermi energy at zero temperature is Ep , t he number of particles per unit area is [NET Dec 2017] €3

(a) (4;~n)

3/2

(b) (G1r2{3n2 )

(c) (;::n3)

€2

(d) (21rJn2)

Prob 1. 7. Consider an ideal Fermi gas in a grand canonical ensemble at a const ant chemical potent ial. The variance of t he occupation number of the single part icle energy level with mean occupation number n is [NET Dec 2018]

(a ) n( l - n)

(b)

vfn

(c) n

(d)

1

vn

11 oo : oo : 11 of the occupation number of the single particle energy level [NET Dec 2018] with mean occupation number n is

(a) n(l - n)

(b)

\ffi

(c) n

(d) 1

vn

Prob 1.8. At low t emperatures, in the Debye [email protected]

4

physicsguide CSIR NET, GATE

Ideal Fermi Systems

@Sk J ahiruddin, 2020

t ion, the contribut ion of the phonons to t he heat capacity of a two- dimensional solid is proportional to [NET Dec 2018] (a) T 2 (b) T 3 (c) T 112 (d) T 312

Prob 1.9. An ideal gas of non-relativistic fermions in 3dimensions is at OK. When both t he number density and t he mass of the particles are doubled, then t he energy per particle is multiplied by a factor [JEST 2014] (a) 21 / 2 (b) 1 (c) 21/ 3 (d) 2- 1/ 3 Prob 1.10. A cylinder at temperature T = 0 is separated into two compartments A and B by a free sliding piston. Compartments A and B are filled by Fermi gases made of spin 1/2 and 3/2 particles respectively. If particles in both t he compartments have same mass, the ratio of equilibrium density of the gas in compartment A to that of gas in compartment Bis [JEST 2017]

(a) 1

1

( b) 32/5

1

(c) 22/s

1

(d) 22/3

Prob 1.11. Consider a non-relativistic two-dimensional gas of N electrons with t he Fermi energy E p . What is the

11 oo : oo : 14 IS

1

(a) 1

(b)

32/ 5

1

1 (d) 22/3

( C) 22/ 5

Prob 1.11. Consider a non-relativistic two-dimensional gas of N electrons with t he Fermi energy E p . What is the

average energy p er particle at temperature T 2019] (A) ~E p (B) i E p (C) ~Ep (D) E p

j [email protected]

= O? [JEST

physicsguide CSIR NET, GATE

5

Ideal Fermi Systems

@Sk J ahiruddin , 2020

Prob 1.12. The energy per oscillator of an isolated system of a large number of non-interacting, identical fermions

in a I-dimensional harmonic oscillator potential is 5fU.;J / 4 where w is the angular frequency of t he harmonic oscillator. The ent ropy of t he syst em p er oscillator is given by [TIFR 2016] (a) 0.25 (b) 0.56 (c) 0.63 (d) 0.75 Prob 1.13. In two dimensions, two metals A and B , have t he number density of free electrons in t he ratio nA : nB = 1 : 2. The ratio of t heir Fermi energies is [TIFR 2017]

(a)

2:3

(b)

1:2

(c)

1:4

(d)

1:8

Prob 1.14. A white dwarf star has volume V and contains N electrons so t hat the density of electrons is n =

N/V

.

- - Taking t he temperature of t he star to be OK, the av•

1 m where m is t he mass of t he elect ron. The electronic pressure

in t he st ar is

[JAM 2017] 1

2

11 oo : oo : 16 - - Taking t he temperature of t he star t o b e OK, the av•

1 m where m is t he mass of the electron. The electronic pressure

[JAM 2017] 1 2 (a) n Eo (b) 2nEo (c) -nEo (d) -nEo 3 3 Pro b 1.15. Consider a 2-dimensional electron gas wit h a density of 1019 m - 2 . The Fermi Energy of the syst em is ______eV (up t o two decimal places) (me= 9.31 x 10-31 kg , h = 6.626 x 10- 34 J s, e = 1.602 x 10- 19 C) [GATE 2017] in t he st ar is

[email protected]

6

physicsguide CSIR NET , GATE

Ideal Fermi Systems

@Sk J a hiruddin, 2020

Prob 1.16. At t emperature T Kelvin (K ), the value of the Fermi function at an energy O.5eV above the Fermi energy is 0.01 . Then T , t o the nearest integer , is - - - [GATE 2019] 5 (kB = 8.62 x 10- eV / K) Prob 1.17. For an ideal Fermi gas in 3 dimensions, t he electron velocity Vp at t he Fermi surface is related to electron concent ration n as [GATE 2012] (a)Vp ex n 213 (b)Vp ex n (c)Vp ex n 1/ 2 (d)Vp ex n l/3

Prob 1.18. The pressure of a non relativistic free Fermi gas in t hree-dimensions depends, at T = 0, on the density [NET June 2014] of fermions n as (a) n 5/3

(b) nl/3

( c) n 2/3

( d) n4/3

11 oo : oo : 19 Prob 1.18. The pressure of a non relativistic free Fermi gas in t hree-dimensions depends, at T = 0, on t he density of fermions n as [NET June 2014]

(a) n 5/3

(b) n l/3

(c) n2/ 3

[email protected]

7

physicsguide CSIR NET, GATE

Ideal Fermi Systems

@Sk J ahiruddin , 2020

1.1

(d) n4/ 3

Ans Keys

1.1.

C

1.7. a

1.13. b

1.2.

C

1.8. a

1.14. d

1.3. b

1.9. d

1.15. 2.36

1.4. a

1.10.

C

1.16. 1263

1.5. a

1.11.

C

1.17. d

1.6. d

1.12. b

1.18. a

11 oo : oo : 21

j [email protected]

8

physicsguide CSIR NET 1 GATE

@Sk J ahiruddin, 2020

1.2

Ideal Fermi Systems

Solutions

Sol 1.1. t he fermi statistics applied for only t he fermeonic

system where a single particle state can accommodate only one or none particle. and the fermi energy is defined as t he energy level upto which all states a1·e completely occupied and at temperature higher than O kelvin t he occupancy is half and the for t he fermi systems µ = Ep where E p is the fermi energy level hence we have

so opt ion c is correct. ~

..

-

...,.

.,

,

, .,. ,

,.



1

11 oo : oo : 24 n(c) =

1 -E -EF--

e kBT

+l

so option c is correct.

Sol 1.2. the occupancy means the probability of a single particle state gets occupied and for the fermionic system it is given by 1 exp

+1

Sol 1. 3. if we consider the nuclear energy level particle dist ribution then we have even number of protons and neutron hence B e~ can never be fermeonic syst em. only deuterium has t he PROTON+ ELECTRON+ NEUTRON=3 and t hat is odd hence the system is fermeonic and hence option d is correct. [email protected]

9

physicsguide CSIR NET, GATE

@Sk J ahiruddin, 2020

Ideal Fermi Systems

Sol 1.4. t he pressure of a fermeonic system can be defined as Pp = jnEp now we can show t hat Ep ex n 213 hence we can surely say t hat Pp ex n 513 hence the option a is correct. Sol 1.5. now we are considering the momentum space, the number of state in K to K + dK is given by

2 n(k )dk = (L / 21r ) 21rKdK now we h ave E = nKV so we take dE = ndKV now we are calculating the number of the particle Ep

N = "

L2 L2 'lrn.21,2 EdE = ?.7rn.21,2 EP2

11 oo : oo : 26 now we have E = nKV so we take dE = ndKV now we are calculating the number of the particle Ep L2 L2 N = o 7r!i,2v2 EdE = 21rri,2v2 E p2 2

so we can say Ep = 21rn2v 2 p now we are calculating the average energy at zero kelvin foEF

E .g(E)dE

2L 2 J2it,,nv p312

< E >= _.;;..__ _ _ _ = - - - - - N

so we can say E / L

2

3N

2 3 = j E p 2 3 but we know that Ep ex n 1 the pressure of the system is defined 2N

P=3V < E >

now N/V= n hence we have P ex n 513

j [email protected]

13

physicsguide CSIR NET, GATE

11 oo : oo : oo

Problems and Solutions



Ill

Ideal Bose Systems Sk J ahiruddin * Suchismito Chatterjee

*Assistant P rofessor Sister Nibedita Govt. College, Kolkata Author was the topper of IIT Bombay 1\11.Sc Physics 2009-2011 batch He ranked 007 in IIT JAl\lI 2009 and 008 (JRF) in CSIR NET June 2011 He has been teaching CSIR NET aspirants since 2012

1

@Sk Jahiruddin, 2020

Ideal Bose Systems

11 oo : oo : 03 1

@Sk J ahiruddin , 2020

Ideal Bose Systems

Contents 1 Problems from NET, GATE, JEST, TIFR & JAM papers 1.1 Basics, Bose Einstein condensation superconductivity . . . . . . . . . . . . . . . . . . . . 1.2 Photon gas: Blackbody radiation . . . . . . 1. 3 Ans Keys . . . . . . . . . . . . . . . . . . . 1.4 Solutions . . . . . . . . . . . . . . . . . . . .

j [email protected]

2

3 3 7 16 17

physicsguide CSIR NET, GATE

11 oo : oo : 06

j ahir@physicsguide. in

2

physicsguide CSIR NET I GATE

Ideal Bose Systems

@Sk J ahiruddin , 2020

1

Problems from NET, GATE, JEST, TIFR & JAM papers

1.1

Basics, Bose Einstein condensation and

superconductivity Prob 1.1. Which of the following atoms cannot exhibit Bose-Einstein condensation even in principle? [GATE 2010]

(a) H t

(b) H e~

(c)

Narr

(d)

Kt8

Prob 1.2. A system has energy levels: E 0 , 2E0 , 3E0 , · · · where t he excited states are triply degenerate. 4 non-interacting bosons are placed in this syst em. If t he total energy of t hese bosons is 5E0 , the number of microstates is [GATE 2007] (a) 2 (b) 3 (c) 4 (d) 5

Prob 1.3. In Bose-Einstein condensates, the particles [GATE 2015]

(a) have strong inter-particle attraction. (b) condense in real space. (c) have over lapping wave functions. (d) have large and positive chemical potential. Prob 1.4. The chemical potential of an ideal Bose gas at

11 oo : oo : os (c) have over lapping wave functions. (d) have large and positive chemical potential.

Prob 1.4. The chemical potential of an ideal Bose gas at [email protected]

physicsguide CSIR NET ) GATE

3

@Sk J ahiruddin ) 2020

Ideal Bose Systems

any temperature is (a) necessarily negative

[JAM 2008] (b) eit her zero or negative (d) either zero or positive

(c) necessarily positive

Prob 1.5. An ideal Bose gas in cl-dimensions obeys t he -+

dispersion relation: c( k = Ak8 ) where A and s are constants. For Bose-Einst ein condensation to occur, the occupancy of excited states 00

0

e

d- s s

e /3 (c-µ)- l

where c is a constant, should remain finite even for µ = 0. This can happen if [NET June 2015] d l 1 d l d l d (a) s < 4 (b) 4 < s < 2 (c) s > l (d ) 2 < s < l

Prob 1.6. Consider an ideal Bose gas in 3-dimensions with energy-momentum relation c ex p 8 , with s > 0. The range of s for which this syst em may undergo Bose-Einstein condensation at a non-zero temperature is [NET June 2011]

(a) 1 < s < 3

(b) 0 < s < 2

(c) 0 < s < 3

(d) 0 < s < oo 4

Prob 1. 7. Bose condensation occurs in liquid He kept at

11 oo : oo : 11 ~.rrr,v t.L ti'J I lb OJ O OGl.fi i":llOij

densation at a non-zero temperature is [NET June 2011]

(a) 1 < s < 3

(b ) 0 < s < 2

(c) 0 < s < 3

(d) 0 < s < oo 4

Pro b 1. 7. Bose condensation occurs in liquid He kept at an ambient pressure at 2. 17K. At what t emperature will [email protected]

physicsguide CSIR NET, GATE

4

@Sk J ahiruddin, 2020

Ideal Bose Systems

Bose condensation will occur in He4 in t he gaseous state, t he density of which is 1000 times sm aller than that of He4 ? [Assume that it is a p erfect Bose gas.] [NET June 2012]

(a) 2.17mK

(b) 21 .7mK

(c) 21 .7µK

(d ) 2.17µK

Prob 1.8. Non-int eracting bosons undergo Bose-Einstein condensation (BEC) when trapped in a 3-dimensional isotropic simple harmonic p otential. For BEC to occur, the chemical

[NET Dec 2012]

potential must be equal to

(a) nJ.;J/ 2

(b)

nJ.;J

(c) 3nJ.;J/ 2

(d) 0

Pro b 1. 9. T wo identical bosons each of m ass m are placed 2 2 in a I-dimensional potential V (x) = ½mw x . The bosons interact via a weak potent ial ½ 2 = Vo exp[-mD(x 1 -x 2 ) /4n] where x 1 and x 2 denote t he co-ordinates of t he part icles. 2

Given that t he ground-state wave function of t he h armonic oscillator is 'lj; 0 (x)

= (7;~) ! exp[-mwx 2 / 2n].

The ground

stat e energy of the two boson system , to the first order in V0 is [NET June 2013]

(a)

nJ.;J

+ 2Vo

(b)

nJ.;J

+ Vo£

(c)

nJ.;J

+ Vo( l + £)-~

11 oo : oo : 13 2

oscillator is 'lj;0 (x) = (7;~ )! exp[-mwx / 2n]. The ground state energy of t he two boson system , to the first order in V0 is [NET June 2013]

nw + 2Vo (d) nw + Vo~ (a)

(b)

nw + Vo£

(c)

nw + Vo( l + £)-~

Prob 1.10. An ideal Bose gas is confined inside a container that is connect ed to a particle reservoir. Each parj [email protected]

@Sk J ahiruddin , 2020

5

physicsguide CSIR NET, GATE

Ideal Bose Systems

t icle can occupy a discret e set of single-particle quantum states. If the probability that a part icular quantum st ate is unoccupied is 0.1, t hen the average number of bosons in t hat state is [NET Dec 2014] (a) 8 (b) 9 (c) 10 (d) 11

Prob 1.11. Consider a quant um syst em of non-interacting bosons in contact wit h a particle bath. The probability of finding no particle in a given single particle quantum state is 1o- 6 . The average number of part icles in that state is of [NET Dec 2017] t he order of (a) 10 3 (b) 106 (c) 109 (d) 1012 Prob 1.12. T wo ident ical bosons may occupy any of two energy levels 0, E where E > 0 lowest energy state is doublydegenerate and t he excited state is non-degenerate. Assume t hat the two-particle system is in thermal equilibrium at a temperature T. Calculat e the average energy (E). What - - - - at low temperature? exp - E fTTFR. 20171

11 oo : oo : 11 egenera e an v~~e exc1 e1 egenera e. t hat the two-particle system is in t hermal equilibrium at a temperature T. Calculate t he average energy (E). What

- - - - at low temperature? exp - E [TIFR 2017]

Prob 1.13. A large number N of ideal bosons, each of

[email protected]

physicsguide CSIR NET, GATE

6

Ideal Bose Systems

@Sk J ahiruddin, 2020

mass m, are trapped in a three-dimensional potential 2 2

V(r)

= mw r 2

The bosonic system is kept at temperature T which is much lower than t he Bose-Einstein condensation temperature Tc. The chemical potential (µ) satisfies [GATE 2019]

(A) µ < ~nw (c)

1.2

(B) 2nw > µ > ~nw

3nw > µ > 2nw

(D) µ = 3nw

Photon gas: Blackbody radiation

Prob 1.14. The blackbody spectrum of an object 0 1 is such t hat its radiant intensity (i.e. intensity per unit wavelength interval) is maximum at a wavelength of 200 nm. Another object 0 2 has maximum radiant intensity at 600 nm. The ratio of power emitted per unit area by 0 1 to that

11 oo : oo : 1s ro e o ac o y sp ec rum o an o Jec 1 1s such t hat it s radiant intensity (i.e. intensity per unit wavelength interval) is maximum at a wavelengt h of 200 nm.

Another object 0 2 has maximum radiant int ensity at 600 nm . T he ratio of power emitted per unit area by O1 to that by 0 2 is [JAM 2007] (a) 1/81 (b) 1/ 9 (c) 9 (d) 81 Prob 1.15. When the temperature of a blackbody is doubled , t he maximum value of it s sp ectral energy density, wit h respect to t hat at init ial t emperature would b ecome [JAM 2012] (a) 1/ 16 t imes

(b) 8 t imes

[email protected]

(c) 16 t imes 7

(d) 32 times

physicsguide CSIR NET, GATE

Ideal Bose Systems

@Sk J ahiruddin , 2020

Prob 1.16. A blackbody at temp erat ure T emit s radiat ion at a p eak wavelength ,\. If it s t emperature becomes

4T , t he new p eak wavelength is

(a)

2;6 ,\

(b)

614 ,\

( c)

[JAM 2013]

f6 ,\ (d) ¼,\

Prob 1.17. In t he radiation emitted by a black body, t he rat io of t he spectral densit ies at frequencies 2v and v will [JAM 2017] vary wit h v as

(a) [ehv/kBT (c)

]-l 1]

_ l

[ehv/ ksT _

+ l ]-l [ehv/ ksT _ 1]

(b) [ehv/ ksT (d)

Prob 1.18. A red star having radius rR at a temperature

TR and a white st ar having radius rw at a temperature T w, radiat e the same t otal power. If these st ars radiate as perfect black bodies, then [JAM 2019] (A) rR > rw and TR > Tw (B) rR < rw and TR > T w

11 oo : oo : 22 Prob 1.18. A red star having radius rR at a temperature TR and a white star having radius rw at a temperature Tw, radiate the same total power. If these stars radiate as perfect black bodies, then [JAM 2019] (A) rR > rw and TR > Tl,v (B) rR < rtt1 and T R > Tw (C) rR > rw and TR < Tw (D) rR < rw and TR < Tw Prob 1.19. The partition function of a gas of photons is given as: •

[GATE 2010] (i) The specific heat of the photon gas varies with temperature as

j [email protected]

8

physicsguide CSIR NET 1 GATE

@Sk J ahiruddin, 2020

Ideal Bose Systems

T

T

(ii) The pressure of the photon gas is

T

T

11 oo : oo : 24 T

T

(ii) The pressure of the photon gas is

(a) n2 (ksT )3 15n3 c3

(c) n2(ksT)4

(b) n2(ksT)4 8n3c3 (d) n2(ksT)3/2

45n3 c3 45n3 c3 Prob 1.20. For a blackbody radiation in a cavity, photons are created and annihilated freely as a result of emission and absorption by the walls of t he cavity. T his is because [GATE 2015] (a) t he chemical potential of the photons is zero. (b) photons obey P auli exclusion principle. (c) photons are spin-1 particles. [email protected]

9

physicsguide CSIR NET, GATE

@Sk J ahiruddin , 2020

Ideal Bose Systems

(d) t he entropy of the photons is very large. Prob 1.21. The total power emitted by a spherical black-

body of radius R at t emperature T is P1 . Let P2 be the power emitted by another spherical blackbody of radius R / 2 at temperature 2T. The ratio (P1 / P2 ) is [GATE 2016] (Give your answer upto two decimal places.) Prob 1.22. A cavity contains blackbody radiation in equi-

librium at temperature T. The specific heat per unit volume of the photon gas in t he cavity is of t he form Cv = ,1T 3 , where 1' is a constant. The cavity is exp anded to twice its

11 oo : oo : 21 Prob 1.22. A cavity contains blackbody radiat ion in equilibriu m at t emperature T . The specific heat per unit volum e of t he photon gas in t he cavit y is of t he form Cv = ryT 3 , where ry is a const ant. The cavity is expanded to twice it s origin al volume and t hen allowed to equilibrium at t emp erature T . T he new internal energy per u nit volume is [NET

June 2011] 4 4 (a) 4ryT (b )2ryT

( c )ryT

4

4

( d )ryT /

4

Prob 1.23. Consider black-body radiation contained in a cavity whose walls are at t emperature T . The radiat ion is in equilibrium wit h the walls of the cavity. If the t emperature of t he walls is increased to 2T and t he radiat ion is allowed t o come to equilibrium at the new t emperature , the ent ropy

[NET June

of the radiation increases by a factor of

2012] (a) 2

(b) 4

j [email protected]

(c) 8

(d) 16

10

physicsguide CSIR NET I GATE

Ideal Bose Systems

@Sk J ahiruddin , 2020

Prob 1.24. Which of t he graphs below gives t he correct qualitative behaviour of t he energy densit y Er(>..) of blackbody radiation >.. at two t emperatures T1 and T2,(T1 < T2)?

[NET June 2014]

,,--

c
T2 so the peak of the curve in t he case one will be higher t han t hat of t he peak of the 2nd one hence

t he option d is correct . Sol 1.33. t he volume would be t ripled in t he fut ure times now the energy radiated per unite area is E = aT4 in future ,xrP ,xr-ill h ~ , rP Ti:, = rrT, 4 h,,t tri+~ l PnPr> a from t he loop. T he current loop and the line are all in the same plane.

[email protected]

2

physicsguide

11 oo : oo : 06

[email protected]

2

physicsguide

EMT: Test-1

@Sk Jahiruddin: 2020

r

I

Figure 1: 2

2

(a) µoa 2I (b) µoa I r

Ans= c Solution:

r

Consider t he line to be part of an infinite loop , with the return part at infinity. By t he reciprocal law (M 12 = M 21 for mutual inductance), flux through t he infinite loop due to . current I in t he small loop is t he same as the flux through the small loop due to the current I in the infinite loop. Flux =

1ra2 µo f

21rr

= µoa2 I

2r

Qs.3: Consider a thin conducting spherical shell with radius

11 oo : oo : os Flux = 1ra2 µo f = µoa2 I 21rr 2r Qs.3: Consider a thin conducting spherical shell wit h radius [email protected]

physicsguide

3

@Sk J ahiruddin: 2020

EMT: Test-1

R carrying a total charge Q . W hat is t he force per unit area

due to t he electric field acting on t he shell's surface? 1

1

Q2

( a) 41rE 0 81r R 3

(C)

Q2

(b) 41rEo 41r R 4

Q2 (d) 1 Q2 41rEo 41r R 3 41rEo 81r R 4 Ans = d Solution: Capacity of a charged sphere is C = 41rE 0 R. Stored energy is W = ~ = ~ 4,;co Increasing t he radius by dR yields 1

°d

a;

dW l 1 Q2 41r R A dR = dR dR = - 2 41rEo R 2dR 2F

Thus, ~ erywhere.

= 4,;co 8~~4 . The force points radially outwards ev-

Qs.4: In a double-slit int erference experiment, d is t he dis-

tance between t he centers of the slits and w is t he width of each slit , as shown in t he figure above. For incident plane waves, an interference maximum on a distant screen will be '' missing'' when

11 oo : oo : 11 '' missing'' when

[email protected]

physicsguide

4

E:NIT : Test-I

@Sk Jahiruddin: 2020

w{ • •

(a) d = v!2w

d

(b) d = \!'3w

(c) 2d = w

(d ) 2d = 3w

Ans= d Based on the given diagram, we know it can't be true that d < w so (C) can be eliminated. Next, recall that the equation for constructive interference in double slit diffraction is dsin(0)

= m 1A

addit ionally, we know t hat we will get a '' missing'' interference maximum when the constructive double slit equation coincides with the single slit diffraction, so we use w sin(0)

= m2 A

get both equations equal to sin (0) and set them equal to one another and solve for d,

11 oo : oo : 14 •



co1nc1

S Wl

sin

ac 10n, so we use w sin(0) = m2A

get both equations equal to sin (0) and set them equal to one another and solve for d, w

d

m 2

m 1

d= m 1w m2 [email protected]

5

physicsguide

EMT: Test-1

@Sk Jahiruddin: 2020

comparing t his to (A) and (B) , we aren 't going to get an irrat ional number (i.e. v'2 or v'3) with some fraction of integers so we can confidently choose (D).

Qs.5: An electro-magnetic plane wave propagates in free space with angular frequency wand wave number k. The wave is incident normally on an absorbing sheet. The resistance of t he sheet is adjust ed until there is no reflection. Find an expression for t he resistance of a square of the sheet of side l , and estimate t he numerical value in Ohm.

(a) 81

(c) 280

(b) 190

(d) 370

Ans= d

Solution: This is sometimes taught as theory. The Electric potential V = El Amperes' law gives

E l= µof

I =

El mu0

11 oo : oo : 16 Amperes' law gives

I = El mu0 Resistance

V

R

El E Bl/µo = µ 0 B

=I=

For EM wave in free space

E 1 -=c=--B ~ [email protected]

physicsguide

6

EMT: Test-1

@Sk J a hiruddin: 2020

So,

µo = 41r R

=

X

10-7 ,

co = 8.85

8~~:~i-=-:2

rv

✓1 .3

X

X

10- 12

10 5 rv 370D

Qs.6: Two parallel infinite line charges are fixed a distance h above a grounded conducting plane. They are a distance 2h apart and have linear charge density - .A. A positive charge

Q is placed a distance h above the plane midway between the two line charges and feels no net force. Find Q in t erms of ,\ and h . (a) 32:h (b) 21:h (c) 13:h (d) 7~h

Ans= a Solution: Charged conductors over a grounded conducting plane Use the method of images: Remove the conducting plane and replace with line charges

11 oo : oo : 19 Ans= a

Solution: Charged conductors over a grounded conducting plane Use the method of images: Remove the conducting plane and replace with line charges of +,\ a distance h b elow the line charges and a charge of -Q below t he point charge. Obviously, the parallel components of t he electrostatic force cancels. The upward force from both the line charges is 2,\Q

Fz = 2 v'5 cos 0 h 5

[email protected]

physicsguide

7

@Sk Jahiruddin: 2020

EJVIT: Test-1

and cos 0 = 2/ v'5. The force from the point image charge is Q2

FQ

= - 4h2

Then the total force must sum to zero: F, - T -

Q2

4h 2

+

8,\Q -

5h - 0

-+ Q = 32-\h 5

Qs. 7: A point charge +q is fixed at the point (0, 0, d) above the grounded conducting plane z = 0 What would be the radius of the circle centered at the origin in the xy plane which encloses half of the total charge induced on the plane? (a) d (b) 2d (c) dv0, (d) dv13 Ans= d

Solution: From image problem the potential r

7

11 oo : oo : 23 origin in t he xy plane which encloses half of t he total charge induced on the plane? (a) d (b) 2d (c) dv'2 (d) dv13 Ans= d Solution: From image problem the potential 1 q 47rEO J x 2 + y2 + (z - d)2

V (x, y ,z) =

q

Induced surface charge a = - Eo

av

av

l

az

47rEo

8z

z =O

+ d) ) ------+ - - - - - - -2 2 3 [x2 + y2 + (z _ d)2] / [x2 + y2 + (z + d)2)3/ J - q (z - d)

q(z

- qd a (x' Y) = 2 ( 2 2 d2) 3/2 1f X +y + [email protected]

@Sk Jahiruddin:

physicsguide

8

2020

ENIT: Test-1

We use polar coordinate

- qd a( r ) - - - - - 21r (r2

+ d2)3/2

Charge bounded wit hin a circle of radius x is 21r

0

- qd qd - - - - - rdr dcp = -----;:::::= = x o 21r (r2 + d2)3/2 J r2 + d2 o X

=

qd

-----;:::::= = - q

J x 2 + d2

T his should be half of t he total charged induced in the plane. The total charge induced in t he plane is - q, as it should be. Then (]

11 oo : oo : 26 This should be half of t he total charged induced in the plane. The total charge induced in the plane is - q, as it should be. Then qd ✓x2

+d2 -

q

q

=-2

Now do the algebra to get X

=

v13d

Qs.8: A point charge q is located at the center of a toroidal coil of rectangular cross section, inner radius a , outer radius a+ w, and height h, which carries a total of N tightly-wound turns and current I.

What will be t he electromagnetic momentum p of this configuration, assuming that w and h are both much less t han [email protected]

9

physicsguide

EMT: Test-1

@Sk Jahiruddin: 2020

a (so you can ignore the variation of the fields over t he cross

section). a) µo qNiwh (b) µo qNiw2 h (c) µo qNiwh (d) µo qNiw 2 h ( 41r 2a2 41r 2a3 41r a2 41r a3 Ans= c Solution: For points inside t he toroid the fields are

The electromagnetic moment um density q µoN I A µoqNI A g = Eo(E x B ) = Eo z= z 2 2 47rEoa 21ra 81r a 3

Inside the toroid r

~

as and s

~

a So, the electromagnetic

11 oo : oo : 30 The electromagnetic momentum density q

µoNI A µo qNI A g = Eo (E x B) = Eo z = ? z 2 3 41fEoa 21ra 81r,., a

Inside the toroid r ~ as and s ~ a So, the electromagnetic momentum is t he momentum density multi- plied by the volume µo q N I h) µo q N I wh 1raw = -z 2 p = - 2- -z 81r a3 41r a2 A

(

A

Qs. 9: Consider two statements (a) T wo conducting rings, A and B , have identical dimensions. The self-inductance of ring A will be the same as that of ring B even when t he rings are made of very different metals. (b) In comparing two capacitors which each are l µF but which differ significantly in size, t he largest of the two can probably be charged up to a higher potential difference t han the smaller one. It will thus be able to hold more energy than the smaller one. [email protected]

@Sk Jahiruddin: 2020

10

physicsguide

EMT: Test-1

Which of the statements are true? (a) a only (b) b only (c) Both (d) None Ans= c Solution: Both are true (a) TRUE: Identical currents in the two rings will generate identical magnetic fields, which will in turn result in identical magnetic fluxes. Self-inductance is simply the ratio of this flux to the current , therefore it will be the same for the two rings regardless of their differing composition.

11 oo : oo : 33 identical magnetic fields, which will in turn result in ident ical magnetic fluxes. Self-inductance is simply the ratio of this flux to t he current , therefore it will be the same for the two rings regardless of their differing composition. (b) TRUE: Consider two parallel-plate capacitors in air: a smaller one with plate area A and plate separation d and a larger one with with plate area 2A and plate separation 2d. Both have t he same capacitance C = EoA/ d. However, if both are charged to the same potential difference V, the electric field between the plates of t he smaller capacitor will be E 1 = V / d while t he electric field between the plates of the larger one will be E 2 = V / 2d. Thus the larger capacitor can be charged to twice as high a potential difference before reaching 2 the breakdown E for air, and can hold more energy ( CV /2) . If the dielectric constant, K,, is t he same for both, t he same reasoning holds. It is t herefore very probable that the capacitor which is significantly larger in size t han t he other, can be charged to a higher potential.

[email protected]

11

@Sk Jahiruddin: 2020

physicsguide

EMT: Test-1

Qs.10: Consider two statements

(a) Charge particles in a magnetic field can not experience ..... an acceleration due to t he iJ x B force. (b) When the magnetic flux t hrough an open surface attached to a conduct ing loop is zero, there cannot be an induced EMF in t hat loop. Which of the statements are t rue? (a) a only (b) b only ( ("l \

TI "+ h ( rl \ l\Tf"\Yl Cl

11 oo : oo : 3s [email protected]

11

physicsguide

E:NIT: Test-I

@Sk Jahiruddin: 2020

Qs.10: Consider two statements

(a) Charge particles in a magnetic field can not experience _, an acceleration due to the v x B force. (b) When the magnetic flux through an open surface attached to a conducting loop is zero, there cannot be an induced EMF in t hat loop. Which of the statements are true? (a) a only (b) b only (c) Both (d) None Ans= d Solution: None are t rue (a) FALSE: Any unbalanced force on a particle causes an acceleration (F = ma) If t he force is perpendicular to the particle's velocity (as is the case for magnetic forces) , then the acceleration causes a change in the direction of velocity. (b) FALSE: It is not the value of the flux itself that gives an induced EMF, but rather the time derivative of the flux. 'PB can be zero at an instant, and d'PB / dt can still be non-zero.

[email protected]

12

physicsguide